You are on page 1of 398

F R O M T H E E X P E R T S I N E N D O C R I N O L O G Y

2021 ENDOCRINE CASE MANAGEMENT:


MEET THE PROFESSOR

ENDOCRINE ._,
SOCIETY _ , ...
chased by Dr. Khalid H. Seedahmed, Jr., MRCP PGDip, CertEndocrinology SA - ID 283
FROM THE EXPERTS IN ENDOCRINOLOGY

ENDO 2021
MEET THE PROFESSOR

ENDOCRINE
CASE MANAGEMENT

ENDOCRINE II
END92021 SOCIETY.-.1

chased by Dr. Khalid H. Seedahmed, Jr., MRCP PGDip, CertEndocrinology SA - ID 283


ENDOCRINE
SOCIETY_,.1
•1

2055 L Street, NW, Suite 600


Washington, DC 20036
www.endocrine.org

Other Publications:
endocrine.org/publications

The Endocrine Society is the world’s largest, oldest, and most active organization working to advance the clinical
practice of endocrinology and hormone research. Founded in 1916, the Society now has more than 18,000 global
members across a range of disciplines.

The Society has earned an international reputation for excellence in the quality of its peer-reviewed journals,
educational resources, meetings, and programs that improve public health through the practice and science of
endocrinology.

Clinical Practice Chair, Endo 2021:


Bulent O. Yildiz, MD

The statements and opinions expressed in this publication are those of the individual authors and do not
necessarily reflect the views of the Endocrine Society. The Endocrine Society is not responsible or liable in any
way for the currency of the information, for any errors, omissions or inaccuracies, or for any consequences
arising therefrom. With respect to any drugs mentioned, the reader is advised to refer to the appropriate medical
literature and the product information currently provided by the manufacturer to verify appropriate dosage,
method and duration of administration, and other relevant information. In all instances, it is the responsibility of
the treating physician or other health care professional, relying on independent experience and expertise, as well
as knowledge of the patient, to determine the best treatment for the patient.

Copyright © 2021 by the Endocrine Society, 2055 L Street, NW, Suite 600, Washington, DC 20036. All rights
reserved. No part of this publication may be reproduced, stored in a retrieval system, posted on the internet, or
transmitted in any form, by any means, electronic, mechanical, photocopying, recording, or otherwise, without
the written permission of the publisher.

Requests for permissions for reproduction should be directed to Copyright Clearance Center (CCC) at
www.copyright.com. For more information or to purchase copies, please contact Society Services by
telephone at 202-971-3646, fax at 202-736-9704, or email at info@endocrine.org or visit the online store:
www.endocrine.org/store.

TRANSLATIONS AND LICENSING: Rights to translate and reproduce Endocrine Society publications
internationally are extended through a licensing agreement on full or partial editions. To request rights for a local
edition, please visit endocrine.org/publications or contact licensing@endocrine.org.

ISBN: 978-1-879225-91-6
eISBN: 978-1-879225-92-3
Library of Congress Control Number: 2020947883

chased by Dr. Khalid H. Seedahmed, Jr., MRCP PGDip, CertEndocrinology SA - ID 283


ENDO 2021
CONTENTS

ADIPOSE TISSUE, APPETITE, OBESITY, AND LIPID


METABOLISM
When Should You Order Advanced Lipoprotein Testing?. . . . . . . . . . . . . . . . . . . . . . . . . . . . . . . 2
Ronald M. Krauss, MD

Challenging Cases in Nutritional and Medical Treatment of Obesity. . . . . . . . . . . . . . . . . . . . . 8


Michael D. Jensen, MD

Managing Weight Effects of Medications to Optimize Health in Patients


With Overweight/Obesity.. . . . . . . . . . . . . . . . . . . . . . . . . . . . . . . . . . . . . . . . . . . . . . . . . . . . . . . .13
Andrea D. Coviello, MD, MSc, MMCi, FACE

What an Endocrinologist Should Know About the Surgical Treatment of Obesity. . . . . . . .21
Daniel H. Bessesen, MD

ADRENAL
Selection, Dosing, and Monitoring of Steroid Replacement in the Patient
With Adrenal Failure. . . . . . . . . . . . . . . . . . . . . . . . . . . . . . . . . . . . . . . . . . . . . . . . . . . . . . . . . . . . .36
Irina Bancos, MD, and Richard Auchus, MD, PhD

Complexities in the Diagnosis and Treatment of Cushing Syndrome. . . . . . . . . . . . . . . . . . . .44


Ashley B. Grossman, BA BSc, MD, PhD (Hon.causa), FRCP, FMedSci

Diagnosis and Management of Endocrine Hypertension: Deciding Who to Screen


for Genetic Causes. . . . . . . . . . . . . . . . . . . . . . . . . . . . . . . . . . . . . . . . . . . . . . . . . . . . . . . . . . . . . . .52
William F. Young, Jr., MD, MSc

Difficulties in Diagnosis and Management of Adrenal Insufficiency in the Critically


Ill Patient. . . . . . . . . . . . . . . . . . . . . . . . . . . . . . . . . . . . . . . . . . . . . . . . . . . . . . . . . . . . . . . . . . . . . . .63
David J. Torpy, MBBS, PhD, FRACP

BONE AND MINERAL METABOLISM


Evaluation of the Patient With Hypophosphatemia . . . . . . . . . . . . . . . . . . . . . . . . . . . . . . . . . .74
Peter J. Tebben, MD

Challenging Cases in Bone and Mineral Disease . . . . . . . . . . . . . . . . . . . . . . . . . . . . . . . . . . . . .87


Neil J. L. Gittoes, BSc, PhD, FRCP

ENDO 2021 • Contents  iii

chased by Dr. Khalid H. Seedahmed, Jr., MRCP PGDip, CertEndocrinology SA - ID 283


Bone Turnover Markers in the Management of Osteoporosis . . . . . . . . . . . . . . . . . . . . . . . . . .96
Richard Eastell, MD

Using the Best Available Evidence to Personalize Osteoporosis Treatment. . . . . . . . . . . . .101


E. Michael Lewiecki, MD, and Micol S. Rothman, MD

Is There an Ideal Vitamin D Level and Dosage for All Circumstances? . . . . . . . . . . . . . . . . .106
Ghada El-Hajj Fuleihan, MD, MPH

Stopping Bone-Active Medications. . . . . . . . . . . . . . . . . . . . . . . . . . . . . . . . . . . . . . . . . . . . . . . .119


Ian R. Reid, MD

Modern Management of the Patient With Paget Disease . . . . . . . . . . . . . . . . . . . . . . . . . . . .125


Thomas J. Weber, MD

DIABETES MELLITUS AND GLUCOSE METABOLISM


Closing the Loop With Pump and Sensor Communication in Diabetes Mellitus:
How to Digest Data and Develop Treatment Recommendations During a Clinic Visit. . . .136
Steven D. Wittlin, MD

Practical Approaches to the Genetic Diagnosis and Management of Patients


With Diabetes Mellitus . . . . . . . . . . . . . . . . . . . . . . . . . . . . . . . . . . . . . . . . . . . . . . . . . . . . . . . . . .144
Rochelle N. Naylor, MD

Using Clinical Trial Results to Optimize Patient Care: Cardiovascular Outcomes


in Patients With Type 2 Diabetes Mellitus Treated With GLP-1 Receptor Agonists
and SGLT-2 Inhibitors. . . . . . . . . . . . . . . . . . . . . . . . . . . . . . . . . . . . . . . . . . . . . . . . . . . . . . . . . . .149
Jennifer B. Green, MD

Pharmacologic Approaches to the Patient With Nonalcoholic Fatty Liver Disease.. . . . . .158
Diana Barb, MD

NEUROENDOCRINOLOGY AND PITUITARY


Pseudoacromegaly Syndromes. . . . . . . . . . . . . . . . . . . . . . . . . . . . . . . . . . . . . . . . . . . . . . . . . . .178
Marta Korbonits, MD, PhD, and Pedro Marques, MD, PhD

Prevention and Management of Carcinoid Crisis. . . . . . . . . . . . . . . . . . . . . . . . . . . . . . . . . . . .186


Electron Kebebew, MD

Surveillance and Management of Long-Term Complications of Acromegaly. . . . . . . . . . . .192


Robert D. Murray, MBBS, MD

Dopamine Agonists: Risks, Adverse Effects, Surveillance, and Intervention. . . . . . . . . . . . .197


Mark E. Molitch, MD

Treatment of Functioning Gonadotroph Pituitary Adenomas. . . . . . . . . . . . . . . . . . . . . . . . .204


Philippe Chanson, MD

iv  ENDO 2021 • Endocrine Case Management

chased by Dr. Khalid H. Seedahmed, Jr., MRCP PGDip, CertEndocrinology SA - ID 283


PEDIATRIC ENDOCRINOLOGY
Management of Pituitary Hormone Replacement Through Transition From
Adolescence to Young Adulthood . . . . . . . . . . . . . . . . . . . . . . . . . . . . . . . . . . . . . . . . . . . . . . . . . . .214
Mehul T. Dattani, MBBS, DCH, FRCPCH, FRCP, MD

Pediatric Bone Fragility: When to Worry and What to Do?. . . . . . . . . . . . . . . . . . . . . . . . . . .223


Laura K. Bachrach, MD

Diabetes Insipidus in Children. . . . . . . . . . . . . . . . . . . . . . . . . . . . . . . . . . . . . . . . . . . . . . . . . . . .233


Craig A. Alter, MD

Screening of the Newborn for Adrenoleukodystrophy . . . . . . . . . . . . . . . . . . . . . . . . . . . . . . .241


Molly O. Regelmann, MD

Youth-Onset Type 2 Diabetes Mellitus. . . . . . . . . . . . . . . . . . . . . . . . . . . . . . . . . . . . . . . . . . . . .250


Philip Zeitler, MD, PhD, and Petter Bjornstad, MD

What Pediatric Endocrinologists Need to Know About Dyslipidemia . . . . . . . . . . . . . . . . . .259


Don P. Wilson, MD, FNLA

REPRODUCTIVE ENDOCRINOLOGY
Diagnosis and Management of Functional Hypogonadism in the Male Patient. . . . . . . . . .270
Mathis Grossmann, MD, PhD, FRACP

Evaluation of Female Fertility: Antimullerian Hormone and Ovarian Reserve Testing. . . .282
Marcelle I. Cedars, MD

Management of Hormone Replacement Therapy in Post–Reproductive-Aged Women . . .291


Genevieve Neal-Perry, MD, PhD

Detection and Management of Abuse of Androgenic Performance-Enhancing Drugs. . . .297


Bradley D. Anawalt, MD

Evaluation of Male Infertility. . . . . . . . . . . . . . . . . . . . . . . . . . . . . . . . . . . . . . . . . . . . . . . . . . . . .306


Channa Jayasena, MD, PhD

THYROID
Management of Patients With Medullary Thyroid Cancer. . . . . . . . . . . . . . . . . . . . . . . . . . . .316
Rossella Elisei, MD

When Thyroid Function Test Results Don’t Make Sense. . . . . . . . . . . . . . . . . . . . . . . . . . . . . .324


Mark Gurnell, MD, PhD

Surveillance of the Patient Who Has Had Therapeutic Hemithyroidectomy for


Thyroid Cancer. . . . . . . . . . . . . . . . . . . . . . . . . . . . . . . . . . . . . . . . . . . . . . . . . . . . . . . . . . . . . . . . .331
Whitney Goldner, MD

ENDO 2021   v

chased by Dr. Khalid H. Seedahmed, Jr., MRCP PGDip, CertEndocrinology SA - ID 283


Management of Thyroid Dysfunction Before and During Pregnancy. . . . . . . . . . . . . . . . . . .341
Kristien Boelaert, MD, PhD, FRCP

MISCELLANEOUS
A Little Help From My Friends: The Endocrinology–Oncology Interface—
How Do We Help Each Other?. . . . . . . . . . . . . . . . . . . . . . . . . . . . . . . . . . . . . . . . . . . . . . . . . . . .352
Claire Higham, DPhil, FRCP, Sacha Howell, PhD, FRCP, and Peter Trainer, MD, FRCPE

Endocrine Effects of Immune Checkpoint Inhibitors: How to Screen and Treat . . . . . . . . . .364
Simon H. S. Pearce, MD, FRCP

A Little Help From My Friends: The Endocrinology–Psychiatry Interface—


How Do We Help Each Other?. . . . . . . . . . . . . . . . . . . . . . . . . . . . . . . . . . . . . . . . . . . . . . . . . . . .371
Elizabeth J. Murphy, MD, DPhil, and Kewchang Lee, MD

A Little Help From My Friends: The Endocrinology–Dermatology Interface—


How Do We Help Each Other?. . . . . . . . . . . . . . . . . . . . . . . . . . . . . . . . . . . . . . . . . . . . . . . . . . . .380
Joy Y. Wu, MD, PhD, and Matthew A. Lewis, MD, MPH

vi  ENDO 2021 • Endocrine Case Management

chased by Dr. Khalid H. Seedahmed, Jr., MRCP PGDip, CertEndocrinology SA - ID 283


ENDO 2021
FACULTY
2021 Endocrine Case Management: Meet the Professor Faculty

Craig A. Alter, MD Mehul T. Dattani, MBBS, Mark Gurnell, MD, PhD


Children’s Hospital of Philadelphia DCH, FRCPCH, FRCP, MD University of Cambridge and
Pearlman School of Medicine at UCL GOS Institute of Child Health Addenbrooke’s Hospital
the University of Pennsylvania United Kingdom Wellcome Trus-MRC Institute
of Metabolic Science
Bradley D. Anawalt, MD Richard Eastell, MD United Kingdom
University of Washington University of Sheffield
School of Medicine United Kingdom Claire E. Higham, DPhil, FRCP
Christie Hospital
Richard J. Auchus, MD, PhD Ghada El-Hajj Fuleihan, MD, MPH United Kingdom
University of Michigan American University of Beirut
Lebanon Sacha J. Howell, PhD, FRCP
Laura K. Bachrach, MD Manchester University
Stanford University Rossella Elisei, MD United Kingdom
School of Medicine University of Pisa
Italy Channa N. Jayasena, MD, PhD
Irina Bancos, MD Imperial College London
Mayo Clinic – Rochester Neil J. Gittoes, BSc, PhD, FRCP United Kingdom
University of Birmingham,
Diana Barb, MD University Hospitals Birmingham Michael D. Jensen, MD
University of Florida United Kingdom Mayo Clinic - Rochester

Daniel H. Bessesen, MD Whitney S. Goldner, MD Electron Kebebew, MD


University of Colorado University of Nebraska Stanford University
School of Medicine Medical Center
Marta Korbonits, MD, PhD
Petter Bjornstad, MD Jennifer B. Green, MD Barts and the London
University of Colorado Duke University Medical Center School of Medicine
School of Medicine United Kingdom
Ashley Grossman, BA BSc,
Kristien Boelaert, MD, PhD, FRCP MD, FRCP, FMedSci Ronald M. Krauss, MD
University of Birmingham Churchill Hospital, University of California –
United Kingdom University of Oxford San Francisco
Barts and the London School of
Marcelle I. Cedars, MD Kewchang Lee, MD
Medicine, University of London
University of California – University of California –
United Kingdom
San Francisco San Francisco
Mathis Grossmann,
Philippe Chanson, MD, MS E. Michael Lewiecki, MD
MD, PhD, FRACP
Paris-Saclay University Hospital University of New Mexico
University of Melbourne,
France Health Sciences Center
Austin Health
Australia
Andrea D. Coviello, MD, Matthew Lewis, MD, MPH
MSc, MMCi, FACE Stanford University
Duke University School of Medicine

ENDO 2021 • Faculty  vii

chased by Dr. Khalid H. Seedahmed, Jr., MRCP PGDip, CertEndocrinology SA - ID 283


Mark E. Molitch, MD Simon H.S. Pearce, MD, FRCP Peter Trainer, MD, FRCPE
Northwestern University Newcastle University The Christie NHS Foundation Trust
Feinberg School of Medicine United Kingdom United Kingdom
Feinberg Medical School
Molly O. Regelmann, MD Thomas J. Weber, MD
Elizabeth J. Murphy, MD, DPhil Children’s Hospital at Montefiore Duke University Medical Center
University of California – Albert Einstein Medical College
San Francisco Don P. Wilson, MD, FNLA
San Francisco General Hospital Ian R. Reid, MD Cook Children’s Medical Center
University of Auckland
Robert D. Murray, MBBS, MD New Zealand Steven D. Wittlin, MD
St. James’s University Hospital University of Rochester School
United Kingdom Micol S. Rothman, MD of Medicine and Dentistry
University of Colorado
Rochelle N. Naylor, MD School of Medicine Joy Y. Wu, MD, PhD
University of Chicago Stanford University
Peter J. Tebben, MD School of Medicine
Genevieve S. Neal-Perry, MD, PhD Mayo Clinic – Rochester
University of Washington William F. Young, Jr., MD, MSc
School of Medicine David J. Torpy, MBBS, PhD, FRACP Mayo Clinic – Rochester
Royal Adelaide Hospital,
University of Adelaide Philip S. Zeitler, MD, PhD
Australia University of Colorado
School of Medicine

Annual Meeting Steering Committee (AMSC)

Felix Beuschlein, MD – Bulent O. Yildiz, MD – Megan R. Haymart, MD –


AMSC Chair Clinical Practice Chair Clinical Science Chair
University Hospital Zurich Hacettepe University University of Michigan
School of Medicine

Annual Meeting Steering Committee Clinical Peer Reviewers


Adda Grimberg, MD Jennifer Sipos, MD Matthew Freeby, MD
Andrew Bauer, MD Lauren Fishbein, MD, PhD Megan R. Haymart, MD
Bulent O. Yildiz, MD Margareta Pisarska, MD Nicola Napoli, MD, PhD
Daniel Dumesic, MD Marie Demay, MD Niki Karavitaki, FRCP, PhD
Dawn Davis, MD, PhD Marta Korbonits, MD, PhD Robert Wermers, MD
Jennifer Sherr, MD, PhD Massimiliano Caprio, MD, PhD Selma Witchel, MD

viii  ENDO 2021 • Endocrine Case Management

chased by Dr. Khalid H. Seedahmed, Jr., MRCP PGDip, CertEndocrinology SA - ID 283


ENDO 2021
OVERVIEW

OVERVIEW • Use existing resources pertaining to clinical


Endocrine Case Management: Meet the Professor is designed guidelines and treatment recommendations for
to provide physicians with a concise and high-quality endocrine and related metabolic disorders to guide
review of more than 45 common and rare endocrine diagnosis and treatment.
disorders to help you keep your practice current. It
consists of case-based clinical vignettes and rationales by TARGET AUDIENCE
experts in all areas of endocrinology, diabetes, and Endocrine Case Management: Meet the Professor provides
metabolism. case-based education to clinicians interested in
improving patient care.
As a result of ENDO 2020 not occurring in-person, the
2021 e-book includes chapters by speakers from ENDO
2020 who are also speaking at ENDO 2021. Several new STATEMENT OF INDEPENDENCE
chapters and clinical case vignettes have also been added. As a provider of CME accredited by the Accreditation
The new content is clearly delineated in the e-book. Council for Continuing Medical Education, the
Endocrine Society has a policy of ensuring that the
content and quality of this educational activity are
ACCREDITATION STATEMENT
The Endocrine Society is accredited by
the Accreditation Council for
Continuing Medical Education to ,mJ
provide continuing medical education ACCREDITED
for physicians. The Endocrine Society
WITH
COMMENDATION
balanced, independent, objective, and scientifically
rigorous. The scientific content of this activity was
developed under the supervision of the Endocrine
Society’s Annual Meeting Steering Committee.

has received Accreditation with Commendation. DISCLOSURE POLICY


The faculty, committee members, and staff who are in
The Endocrine Society designates this enduring material position to control the content of this activity are
for a maximum of 30.0 AMA PRA Category 1 Credits™. required to disclose to the Endocrine Society and to
Physicians should claim only the credit commensurate learners any relevant financial relationship(s) of the
with the extent of their participation in the activity. individual or spouse/partner that have occurred within
the last 12 months with any commercial interest(s)
whose products or services are related to the CME
LEARNING OBJECTIVES
content. Financial relationships are defined by
Endocrine Case Management: Meet the Professor will allow
remuneration in any amount from the commercial
learners to assess their knowledge of all aspects of
interest(s) in the form of grants; research support;
endocrinology, diabetes, and metabolism.
consulting fees; salary; ownership interest (eg, stocks,
Upon completion of this educational activity, learners stock options, or ownership interest excluding
will be able to: diversified mutual funds); honoraria or other payments
for participation in speakers’ bureaus, advisory boards,
• Recognize clinical manifestations of endocrine and or boards of directors; or other financial benefits. The
metabolic disorders and select among current intent of this disclosure is not to prevent CME planners
options for diagnosis, management, and therapy. with relevant financial relationships from planning or
delivering content, but rather to provide learners with
• Identify risk factors for endocrine and metabolic information that allows them to make their own
disorders and develop strategies for prevention. judgments of whether these financial relationships may
have influenced the educational activity with regard to
• Evaluate endocrine and metabolic manifestations of
exposition or conclusion. The Endocrine Society has
systemic disorders.

ENDO 2021 • Overview  ix

chased by Dr. Khalid H. Seedahmed, Jr., MRCP PGDip, CertEndocrinology SA - ID 283


reviewed all disclosures and resolved or managed all Health, Inc; Speaker: Alexion Pharmaceuticals, Inc,
identified conflicts of interest, as applicable. Radius Health, Inc; Advisory Board Member (Spouse):
Radius Health, Inc, Alexion Pharmaceuticals, Inc,
The following faculty reported relevant financial Sandoz. Mark E. Molitch, MD, Advisory Board
relationships, as identified below, as of January 1, 2021: Member: Jansen Pharmaceuticals, Sanofi; Consulting
Craig A. Alter, MD, Advisory Board Member: Novo Fee: Merck, Novartis Pharmaceuticals, Pfizer, Inc; Grant
Nordisk; Consulting Fee: Pfizer, Inc. Bradley D. Recipient: Novartis Pharmaceuticals, Bayer, Inc,
Anawalt, MD, UpToDate. Richard J. Auchus, MD, Strongbridge, Novo Nordisk. Robert D. Murray, MD,
PhD, Consulting Fee: Novartis Pharmaceuticals, FRCP, BSc, Grant Recipient: Sandoz, Takeda, Pfizer,
Corcept Therapeutics, Jansen Pharmaceuticals, Adrenas Inc. Genevieve S. Neal-Perry, MD, PhD, Advisory
Therapeutics, Selenity Therapeutics, Quest Diagnostics; Board Member: Astellas; Grant Recipient: NICHD.
Research Investigator: Novartis Pharmaceuticals, Simon H. S. Pearce, MD, FRCP, MBBS, Research
Corcept Therapeutics, Strongbridge Biopharma, Investigator: Apitope; Speaker: Sanofi-Aventis, Quidel.
Neurocrine Biosciences, Spruce Biosciences. Irina Ian Reid, MD, Advisory Board Member: Amgen, Inc;
Bancos, MD, Advisory Board Member: Corcept. Petter Research Investigator: Amgen, Inc; Speaker: Eli Lilly &
M. Bjornstad, MD, Advisory Board Member: Company, Amgen, Inc. Peter J. Tebben, MD, Research
Boehringer Ingelheim, XORTX; Consulting Fee: Bayer, Investigator: Ultragenyx. Steven D. Wittlin, MD,
Inc, Boehringer Ingelheim, Bristol-Myers Squibb, Novo Advisory Board Member: Senseonics; Consulting Fee:
Nordisk, Sanofi, Lilly USA, LLC; Grant Recipient; Novo; Speaker: Medtronic-Diabetes. Philip S. Zeitler,
Horizon Pharma. Philippe Chanson, MD, MS, MD, PhD, Consulting Fee: Merck, Eli Lilly & Company,
Advisory Board Member: Tiburio; Research Boehringer Ingelheim, Novo Nordisk; Research
Investigator: Tiburio. Mehul T. Dattani, MD, DCH, Investigator: Jansen Pharmaceuticals.
FRCPCH, FRCP, MBBS, Advisory Board Member:
Novo Nordisk, Pfizer, Inc, Ipsen; Consulting Fee: Ipsen, The following faculty reported no relevant financial
Novo Nordisk, Sandoz; Speaker: Novo Nordisk, Ipsen, relationships as of January 1, 2021: Diana Barb, MD;
Pfizer, Inc. Richard Eastell, MD, MBChB, Consulting Laura K. Bachrach, MD; Daniel H. Bessesen, MD;
Fee: Roche Diagnostics, Immunodiagnostic Systems, Kristien Boelaert, MD, PhD; Marcelle I. Cedars,
Nittobo; Grant Recipient: Roche Diagnostics, MD; Andrea D. Coviello, MD, MSc, MMCi; Ghada
Immunodiagnostic Systems, Nittobo. Rossella Elisei, El-Hajj Fuleihan, MD, MPH; Ashley Grossman, MD,
MD, Advisory Board Member: Bayer, Inc, Loxo, Ipsen; FRCP; Mark Gurnell, MD, PhD; Claire E. Higham,
Consulting Fee: Genzyme Corporation, Eli Lilly & DPhil, MBBS; Sacha J. Howell, PhD, FRCP; Michael
Company, Exelixis, Inc. Neil J. Gittoes, PhD, FRCP, D. Jensen, MD; Electron Kebebew, MD; Matthew
MBCB, BSc, Advisory Board Member: Shire, Amgen, Lewis, MD, MPH; Elizabeth J. Murphy, MD, DPhil;
Inc; Consulting Fee: Shire, Takeda; Speaker: Shire. Rochelle N. Naylor, MD; Molly O. Regelmann, MD;
Whitney S. Goldner, MD, Research Investigator: Micol S. Rothman, MD; David J. Torpy, PhD,
Roche Diagnostics. Jennifer B. Green, MD, Advisory FRACP, MBBS; Peter J. Trainer, MD, FRCP, MBChB,
Board Member: AstraZeneca, Boehringer Ingelheim; BSc; Thomas Weber, MD; Joy Y. Wu, MD, PhD;
Consulting Fee: Novo Nordisk; Research Investigator: William F. Young, Jr., MD, MSc
Boehringer Ingelheim, Sanofi. Mathis Grossmann,
MD, PhD, FRACP, Speaker: Besins Health Care. The following AMSC peer reviewers reported relevant
Channa N. Jayasena, MD, PhD, Grant Recipient: financial relationships as of January 1, 2021: Andrew
Investigator-led grant from Logixx Pharma Ltd; Bauer, MD, Speaker: Hexal, AG. Massimiliano
Speaker: BES 2016: Speaking against motion for Caprio, MD, PhD, Research Grant Recipient: Bayer
androgen therapy in older men, during debate AG. Dawn Davis, MD, PhD, Employee: Department of
sponsored by Besins healthcare. Marta Korbonits, Veterans Affairs; Grant Recipient: Department of
MD, PhD, Advisory Board Member: ONO-Pharma, Veterans Affairs, NIH; Site PI for clinical trial, Eiger
Novo Nordisk; Research Investigator: ONO-Pharma; Pharmaceuticals; Grant Reviewer: NIH. Marie Demay,
Speaker: Ipsen, Pfizer, Inc. Ronald M. Krauss, MD, MD, Reviewer: NIH; Grantee: NIH; (Spouse) CMO:
Advisory Board Member: Virta Health, DayTwo, Syros Pharmaceuticals. Daniel Dumesic, MD, NIHP50
Seraphina, Saliogen Therapeutics; Grant Recipient: Grantee: NIH. Adda Grimberg, MD, Consultant:
Quest Diagnostics; Employee: JumpstartMD. Pediatric Endocrine Society Growth Hormone
E. Michael Lewiecki, MD, Advisory Board Member: Deficiency Knowledge Center (Sponsored by Sandoz);
Amgen, Inc; Grant Recipient: Amgen, Inc, Radius Grant Recipient: Eunice Kennedy Shriver National

x  ENDO 2021 • Endocrine Case Management

chased by Dr. Khalid H. Seedahmed, Jr., MRCP PGDip, CertEndocrinology SA - ID 283


Institute of Child Health and Human Development. in this educational activity may not be the same as those
Megan R. Haymart, MD, Grantee: NIH, AHRQ. indicated in product labeling approved by the Food and
Marta Korbonits, MD, PhD, Advisory Board Member: Drug Administration (FDA). The Endocrine Society
Pfizer; Scientific Advisor: ONO, NorvoNordisk, requires that any discussions of such “off-label” use be
Heptare. Nicola Napoli, MD, PhD, Advisory Board based on scientific research that conforms to generally
Member: UCB, Eli Lilly; Consultant: Amgen, Lilly; accepted standards of experimental design, data
Speaker: Lilly. Jennifer Sherr, MD, PhD, Consultant: collection, and data analysis. Before recommending or
Medtronic Diabetes, Sanofi; Advisory Board Member: prescribing any therapeutic agent or device, learners
Bigfoot Biomedical, Lilly, Insulet, Cecelia Health; Grant should review the complete prescribing information,
Recipient: JDRF; Scientific Advisory Board Member: including indications, contraindications, warnings,
Metabolic Control Pituitary Area. Robert Wermers, precautions, and adverse events.
MD, Research Grant Recipient: Alexion
Pharmaceuticals.
PRIVACY AND CONFIDENTIALITY
The following AMSC peer reviewers reported no relevant STATEMENT
financial relationships as of January 1, 2021: Lauren The Endocrine Society will record learner’s personal
Fishbein, MD, PhD; Matthew Freeby, MD; Niki information as provided on CME evaluations to allow
Karavitaki, FRCP, PhD; Margareta Pisarska, MD; for issuance and tracking of CME certificates. The
Jennifer Sipos, MD; Selma Witchel, MD; Bulent O. Endocrine Society may also track aggregate responses to
Yildiz, MD questions in activities and evaluations and use these data
to inform the ongoing evaluation and improvement of
The Endocrine Society staff associated with the its CME program. No individual performance data or
development of content for this activity reported no any other personal information collected from
relevant financial relationships. evaluations will be shared with third parties.

DISCLAIMERS ACKNOWLEDGMENT OF
The information presented in this activity represents COMMERCIAL SUPPORT
the opinion of the faculty and is not necessarily the This activity is not supported by educational grant(s) or
official position of the Endocrine Society. other funds from any commercial supporter.

USE OF PROFESSIONAL JUDGMENT:


AMA PRA CATEGORY 1 CREDIT
The educational content in this enduring activity relates
(CME) INFORMATION
to basic principles of diagnosis and therapy and does not
To receive a maximum of 30.0 AMA PRA Category 1
substitute for individual patient assessment based on the
Credits™, participants must complete the online
health care provider’s examination of the patient and
interactive module and activity evaluation located at
consideration of laboratory data and other factors
https://education.endocrine.org/MTP2021.
unique to the patient. Standards in medicine change as
Participants must achieve a minimum score of 70% to
new data become available.
claim CME credit. After initially completing the module,
if participants do not achieve a minimum score of 70%,
DRUGS AND DOSAGES:
they have the option to change their answers and make
When prescribing medications, the physician is advised
additional attempts to achieve a passing score. Learners
to check the product information sheet accompanying
also have the option to clear all answers and start over.
each drug to verify conditions of use and to identify any
changes in drug dosage schedule or contraindications.
METHOD OF PARTICIPATION
This enduring material is presented online and in print
POLICY ON UNLABELED/OFF-LABEL USE
format. The estimated time to complete this activity,
The Endocrine Society has determined that disclosure of
including review of material, is 30 hours. Participants
unlabeled/off-label or investigational use of commercial
must achieve a minimum score of 70% to claim CME
product(s) is informative for audiences and therefore
credit. After initially completing the module, if
requires this information to be disclosed to the learners
participants do not achieve a minimum score of 70%, they
at the beginning of the presentation. Uses of specific
have the option to change their answers and make
therapeutic agents, devices, and other products discussed

ENDO 2021 • Overview  xi

chased by Dr. Khalid H. Seedahmed, Jr., MRCP PGDip, CertEndocrinology SA - ID 283


additional attempts to achieve a passing score. Participants LAST REVIEW DATE: March 2021
also have the option to clear all answers and start over.
ACTIVITY RELEASE DATE: March 20, 2021
SYSTEM REQUIREMENTS
To complete this activity, participants must have access ACTIVITY EXPIRATION DATE:
to a computer or mobile device with an Internet March 30, 2022 (date after which this enduring material
connection and use an up to date version of any major is no longer certified for AMA PRA Category 1 Credits™)
Web browser, such as Internet Explorer 10+, Firefox
32+, Safari, or Google Chrome 37+. In addition, cookies For questions about content or obtaining CME credit,
and Javascript must be enabled in the browser’s options. please contact the Endocrine Society at
http://education.endocrine.org/contact.

COMMON ABBREVIATIONS
ACTH = corticotropin FSH = follicle-stimulating hormone NPH insulin = neutral protamine
ACE inhibitor = angiotensin-converting GH = growth hormone Hagedorn insulin
enzyme inhibitor GHRH = growth hormone–releasing PCSK9 inhibitor = proprotein
ALT = alanine aminotransferase hormone convertase subtilisin/kexin 9 inhibitor
AST = aspartate aminotransferase GLP-1 receptor agonist = glucagonlike PET = positron emission tomography

BMI = body mass index peptide 1 receptor agonist PSA = prostate-specific antigen
GnRH = gonadotropin-releasing PTH = parathyroid hormone
CNS = central nervous system
hormone PTHrP = parathyroid hormone–related
CT = computed tomography
hCG = human chorionic gonadotropin protein
DHEA = dehydroepiandrosterone
HDL = high-density lipoprotein SGLT-2 inhibitor = sodium-glucose
DHEA-S = dehydroepiandrosterone cotransporter 2 inhibitor
sulfate HIV = human immunodeficiency virus
SHBG = sex hormone–binding globulin
DNA = deoxyribonucleic acid HMG-CoA reductase inhibitor =
3-hydroxy-3-methylglutaryl coenzyme A T3 = triiodothyronine
DPP-4 inhibitor = dipeptidyl-peptidase
reductase inhibitor T4 = thyroxine
4 inhibitor
IGF-1 = insulinlike growth factor 1 TPO antibodies = thyroperoxidase
DXA = dual-energy x-ray
LDL = low-density lipoprotein antibodies
absorptiometry
LH = luteinizing hormone TRH = thyrotropin-releasing hormone
FDA = Food and Drug Administration
MCV = mean corpuscular volume TRAb = thyrotropin-receptor antibodies
FGF-23 = fibroblast growth factor 23
MIBG = meta-iodobenzylguanidine TSH = thyrotropin
FNA = fine-needle aspiration
MRI = magnetic resonance imaging VLDL = very low-density lipoprotein

xii  ENDO 2021 • Endocrine Case Management

chased by Dr. Khalid H. Seedahmed, Jr., MRCP PGDip, CertEndocrinology SA - ID 283


ADIPOSE TISSUE,
APPETITE, OBESITY, AND
LIPID METABOLISM

chased by Dr. Khalid H. Seedahmed, Jr., MRCP PGDip, CertEndocrinology SA - ID 283


 New Content

When Should You Order


Advanced Lipoprotein Testing?
Ronald M. Krauss, MD Departments of Pediatrics and Medicine, UCSF, San Francisco, CA;
E-mail: Ronald.Krauss@ucsf.edu

Learning Objectives triglyceride-rich lipoprotein remnants, which are


proatherogenic, and large HDL particles, which
As a result of participating in this session, learners
are associated with reduced ASCVD risk. Among
should be able to:
methods for measurement of these particles,
• Describe the key features of advanced the most commonly used are nuclear magnetic
lipoprotein tests. resonance spectroscopy and ion mobility. Another
atherogenic particle that is not included in the
• Explain why standard cholesterol
standard lipid panel, lipoprotein (a) (Lp[a]), is
measurements may not identify abnormal
most commonly measured by immunoassay.
levels of lipoprotein particles associated with
Collectively, these measurements, as well as
cardiovascular disease risk.
plasma levels of total LDL particles and/or
apolipoprotein B, comprise advanced lipoprotein
testing. Consideration should be given to
obtaining such testing for ASCVD risk assessment
Main Conclusions in individuals with borderline dyslipidemia or
Among individuals identified by current American estimated risk using current algorithms, features
College of Cardiology/American Heart Association of metabolic syndrome, and/or significant family
guidelines as candidates for treatment of blood history of ASCVD. Testing can also be useful as
cholesterol to reduce risk of atherosclerotic an adjunct to standard measurements in assessing
cardiovascular disease (ASCVD) are those without efficacy of lipid-lowering therapy.
ASCVD or diabetes who have a 10-year risk of
developing ASCVD of at least 7.5%. This calculation
is based on an algorithm including total and
HDL cholesterol, as well as other established risk
Significance of the
factors. While the application of a calculator based Clinical Problem
on this algorithm has been shown to identify a Despite the efficacy of LDL-lowering therapy for
high proportion of patients who would benefit reducing ASCVD risk, there remains considerable
from statin or other lipid-lowering therapy, the residual risk. This may not be reflected by the
algorithm can fail to identify individuals at high risk plasma LDL-cholesterol concentration due
for ASCVD. Thus, candidates for lipid-lowering to increased levels of lipoprotein particles, in
on the basis of plasma levels of lipoprotein particle particular small, dense LDL and Lp(a), that impact
subclasses may not be identified. The lipoprotein ASCVD to an extent disproportionate to their
particle subclasses include small, dense LDL and cholesterol content. Application of methods for

2  ENDO 2021 • Endocrine Case Management

chased by Dr. Khalid H. Seedahmed, Jr., MRCP PGDip, CertEndocrinology SA - ID 283


measuring these particles can identify individuals subclasses ranging from larger, more buoyant and
at heightened ASCVD risk that is not revealed by cholesterol-enriched particles to smaller, more
standard lipid measurements and provide a basis dense, and cholesterol-depleted LDL species, and
for more targeted risk reduction in these patients. the relative proportions of these particles can
differ significantly among individuals.3 Thus,
while there is generally a correlation between LDL
Barriers to Optimal Practice cholesterol and LDL particle concentrations, any
Without obtaining measurements of lipoprotein given level of LDL cholesterol may overestimate
particles that comprise advanced lipid testing, the level of larger LDL particles or underestimate
clinicians may not recognize the advisability of the level of smaller LDL particles. It has been
more aggressive lipid management for a significant shown that when there is significant discordance
subset of their patients. There are, however, between LDL cholesterol and total LDL particle
some challenges for implementing such testing concentrations, the latter provide a more
in practice, including differing methodological reliable measure of ASCVD risk.4 This is also
platforms and the lack of standardized criteria reflected by levels of apolipoprotein B, which
for assigning ASCVD risk. However, clinical is a measure of the total number of atherogenic
experience with the methodologies, and familiarity lipoproteins in plasma.4 Consistent with these
with the studies supporting their use, can lead to observations is evidence that levels of smaller,
effective implementation of these measurements cholesterol-depleted LDL subclasses are more
in appropriately selected patients. strongly predictive of ASCVD than equivalent
concentrations of larger LDL particles, and
Strategies for Diagnosis, that smaller LDL particles have properties that
would be expected to have a greater adverse
Therapy, and/or Management impact on the development and progression of
Current American College of Cardiology/ atherosclerosis.2,5 Chief among these is reduced
American Heart Association guidelines specify exposure of the LDL receptor–binding domain
4 groups of candidates for cholesterol-lowering of apolipoprotein B, leading to impaired LDL
therapy: (1) clinical ASCVD; (2) LDL cholesterol receptor–mediated plasma clearance and hence
≥190 mg/dL (≥4.92 mmol/L); (3) diabetes mellitus greater plasma residence time than that of
(patients aged 40 to 75 years); and (4) individuals larger LDL. The resulting increased exposure
aged 40 to 75 years without ASCVD or diabetes of LDL to arterial tissue is accompanied by
who have a 10-year risk of ASCVD of at least 7.5% greater binding, uptake, and retention in the
based on an assessment tool including age, sex, subendothelial space, and enhanced susceptibility
total and HDL cholesterol, blood pressure, treated to oxidative modification that in turn can promote
hypertension, diabetes, and cigarette smoking. inflammatory responses in the artery wall.
Consideration may also be given to intermediate Another important feature of a lipoprotein
10-year risk of 5% to 7.5%.1 Lipid management for profile characterized by increased levels of
at-risk individuals in these categories is primarily small, dense LDL is its strong relationship with
focused on dietary and pharmacologic reduction of components of metabolic syndrome, particularly
LDL-cholesterol levels. elevated plasma triglyceride and reduced HDL
While there is no doubt that LDL particles cholesterol.3 High plasma triglyceride can in
play a central causal role in the development and turn be indicative of increased levels of partially
progression of ASCVD,2 measurement of total catabolized triglyceride-rich lipoproteins, or
and LDL cholesterol can lead to overestimation or remnants, and both of these measures have
underestimation of LDL particle concentrations. been shown to be independently associated with
This is because there is a spectrum of LDL ASCVD. Low levels of HDL cholesterol, however,

ENDO 2021 • Adipose Tissue, Appetite, Obesity, and Lipid Metabolism  3

chased by Dr. Khalid H. Seedahmed, Jr., MRCP PGDip, CertEndocrinology SA - ID 283


are commonly due to reduced concentrations analysis.5 The former is based on mathematical
of larger, cholesterol-enriched HDL particles, deconvolution of a single lipid-generated signal
and while both of these measures are inversely from plasma into multiple components of differing
associated with ASCVD risk, their causality size, while the latter yields direct measurement of
in the disease process, and hence their roles as concentrations of lipoprotein particle subclasses
treatment targets, has not been established. The within defined particle size intervals. In part due
cluster of increased triglyceride, reduced HDL to these different analytic principles, the reference
cholesterol, and a predominance of small, dense ranges for particle concentrations determined
LDL particles (designated LDL subclass pattern B), by these 2 methodologies differ. In the case of
termed atherogenic dyslipidemia, is the most ion mobility, ASCVD risk has been found to be
common lipid phenotype associated with obesity, associated with concentrations of medium-sized,
insulin resistance, and coronary artery disease.3 as well as small, LDL particles.6 Levels of these
Consistent with these observations are reports particles, and in particular small LDL, can vary
from a number of large prospective cohort studies, greatly at any given plasma concentration of LDL
using several different analytic methodologies, that cholesterol or triglyceride, and while correlations
have shown a much stronger association of small are stronger for non-HDL cholesterol (total
vs large particles with incidence of ASCVD that is minus HDL cholesterol) and apolipoprotein B,
independent of LDL-cholesterol concentration.5 there can still be a high degree of divergence.
While some analyses have led to the conclusion Thus, advanced lipid testing is preferred over
that all apolipoprotein B particles are equally these other measurements for precise delineation
atherogenic, there are analytic, statistical, and of LDL subclass profiles, and hence for more
biologic reasons for challenging this conclusion. In refined assessment of ASCVD risk and response
particular, the argument has been made that large to LDL-lowering therapies. Although remnant
LDL particles are highly atherogenic due to the fact (or intermediate-density) lipoprotein particles
that patients with familial hypercholesterolemia, are also measured by the ion mobility and nuclear
who are unquestionably at heightened risk for magnetic resonance techniques, their levels are
ASCVD, generally have a predominance of these not yet established for application in ASCVD risk
particles in plasma. However, consideration should assessment. An estimate of remnant cholesterol,
be given to the premise that prolonged plasma calculated as the value obtained after subtracting
residence time is a key determinant of the impact LDL and HDL cholesterol from total plasma
of LDL (and other lipoproteins) on ASCVD risk, cholesterol, has been suggested as an alternative.
by virtue of both increased arterial exposure and Another key component of advanced
greater potential for undergoing intravascular lipoprotein testing is Lp(a). This particle is
modifications. Hence, reduced LDL-receptor level, not readily discriminated by nuclear magnetic
which is the basis for the LDL elevation in familial resonance or ion mobility techniques and thus is
hypercholesterolemia, can lead to pathologic most commonly measured by immunoassay. It
behavior of all LDL particles, whereas, as noted consists of an LDL particle that is attached to the
above, in the absence of LDL-receptor deficiency, apolipoprotein (a) protein via covalent linkage
which is the case for the great majority of the to apolipoprotein B. Apolipoprotein (a) includes
population, the structure of small LDL particles a variable number of units termed kringles that
renders them less susceptible than larger LDL are homologous to those in plasminogen, and
particles to plasma clearance. this protein confers a number of pathologic
Among techniques for measuring lipoprotein properties to Lp(a) beyond those due to its LDL
particles, the 2 that are currently most commonly component that confer heightened risk for stroke
used in clinical practice are nuclear magnetic and coronary artery disease.7 These include a
resonance spectroscopy and ion mobility prothrombotic effect due to its competitive

4  ENDO 2021 • Endocrine Case Management

chased by Dr. Khalid H. Seedahmed, Jr., MRCP PGDip, CertEndocrinology SA - ID 283


inhibition of plasminogen-induced clot lysis, in the case of small and medium LDL particle
as well as increased susceptibility to undergo subclasses since no unique genetic markers have
oxidation due to a high content of disulfide bonds. yet been identified for levels of these particles.
Moreover, plasma Lp(a) is also associated with risk Despite these limitations, there is a case for using
for calcific aortic stenosis. Lp(a) levels are under advanced lipoprotein testing for making specific
much stronger genetic control than other plasma dietary recommendations to improve LDL particle
lipoproteins, due primarily to polymorphisms in levels in patients with LDL subclass pattern B.
the LPA gene locus that confer wide variability Lowering body weight and reducing intake of
in plasma Lp(a) levels, as well as variations in sugars and processed carbohydrates, rather than
the number of kringle repeats that can influence total or saturated fat, can selectively reduce plasma
blood levels and pathologic properties of Lp(a). levels of small, dense LDL particles. Regarding
Notably, plasma Lp(a) concentrations in more pharmacologic therapy, while statins act to lower
than one-third of the population are higher than LDL-cholesterol levels primarily by increasing
those associated with increased ASCVD risk LDL-receptor activity (resulting in a lesser
(>75 nmol/L), with relative risk rising at least reduction of smaller vs larger LDL particles), most
3-fold in the 95th population percentile.7 Due to patients experience a significant reduction in levels
the strong genetic basis of elevated Lp(a), cascade of small LDL particles.
screening of first-degree family members for this Finally, the strong genetic regulation of
condition has value for directing more aggressive Lp(a) results in marked resistance to dietary and
efforts at ASCVD reduction in the 50% of such pharmacologic intervention, with the exception of
relatives who may be affected. high-dosage nicotinic acid and PCSK9 inhibitors,
A number of considerations arise from use of which can lead to moderate (up to one-third)
advanced lipoprotein testing for the management of Lp(a) reduction. Emerging therapeutic prospects,
ASCVD risk. Overall, the application of evidence- including an LPA antisense oligonucleotide, can
based criteria for therapeutic decision-making has lower levels by more than 80%.10 In the absence of
been limited by the lack of randomized control trials effective Lp(a) reduction, affected patients should
aimed at specifically testing whether reductions be considered candidates for relatively aggressive
in lipoprotein particle subspecies are superior to lowering of LDL-cholesterol levels and increased
change in LDL cholesterol for predicting lower attention to other ASCVD risk factors as a means
incidence of ASCVD. Nevertheless, a number for reducing absolute risk, and hence attenuating
of analyses have provided support for the power the impact of the relative increase in risk conferred
of total LDL particle concentrations,8 as well as by Lp(a) elevation.
apolipoprotein B9 in this regard. While for the
majority of the population small- and medium-sized
LDL particles account for much of the relationship
Clinical Case Vignettes
of total LDL particles with ASCVD, and hence Case
present more specific therapeutic targets, changes A generally healthy 56-year-old woman has had
in small LDL particles in particular are strongly “high cholesterol” for 8 years. Her father and
correlated with changes in triglyceride-rich brother are on statin therapy, and a paternal
lipoproteins and HDL. Thus, a benefit specifically grandfather died of myocardial infarction. She
attributable to reduction of this LDL subclass follows a diet that is low in meat, dairy fat, and
is difficult to tease out. Moreover, the genetic processed carbohydrates and sugar. She walks
technique of Mendelian randomization, which approximately 10,000 steps per day, does not smoke
has been used to infer ASCVD causality for LDL- cigarettes, and has not had high blood pressure or
cholesterol and triglyceride levels, as well as Lp(a) hyperglycemia. Her hemoglobin A1c levels have
(but not HDL-cholesterol) levels, is not applicable ranged from 5.4% to 5.8% (36 to 40 mmol/mol).

ENDO 2021 • Adipose Tissue, Appetite, Obesity, and Lipid Metabolism  5

chased by Dr. Khalid H. Seedahmed, Jr., MRCP PGDip, CertEndocrinology SA - ID 283


On physical examination, her BMI is Answer: D) A statin
22.7 kg/m2 and blood pressure is 120/78 mm Hg.
There are no notable physical findings. Based on their well-established and general
efficacy, statins remain the first-line
Initial laboratory test results: pharmacologic intervention for all patients
considered to be at increased risk of ASCVD. This
Total cholesterol = 206 mg/dL (<200 mg/dL)
(SI: 5.34 mmol/L [<5.18 mmol/L]) patient was initially treated with rosuvastatin,
LDL cholesterol = 136 mg/dL (<100 mg/dL) 10 mg daily. After 3 months, there was a 45%
(SI: 3.52 mmol/L [<2.59 mmol/L]) reduction in small LDL and a 37% reduction
HDL cholesterol = 49 mg/dL (≥50 mg/dL) in medium LDL. However, the resulting levels
(SI: 1.27 mmol/L [<1.30 mmol/L])
were above optimal, and elevated Lp(a) and LDL
Triglycerides = 96 mg/dL (<150 mg/dL)
(SI: 1.08 mmol/L [<1.70 mmol/L]) subclass pattern B persisted. Therefore, further
Non-HDL cholesterol = 157 mg/dL (<130 mg/dL) lipid management was felt to be warranted and is
(SI: 4.07 mmol/L [<3.37 mmol/L]) underway.
Limitation of carbohydrate rather than
Based on the American College of Cardiology/ saturated fat (Answer A) is most appropriate for
American Heart Association risk calculator, her reducing elevated levels of small LDL particles,
estimated 10-year risk for ASCVD is 5.8%. which is a prominent feature of this patient’s
On the basis of her intermediate ASCVD risk lipoprotein profile.
score and family history of hypercholesterolemia Nicotinic acid (Answer B), while potentially
and coronary heart disease, an advanced lipid beneficial for reducing Lp(a), as well as
panel is ordered, and the results (as reported by triglycerides and small LDL, has failed to produce
Quest Diagnostics) are shown (see Table). clinical benefit in recent large trials when added
to a regimen yielding maximal LDL reduction by
Based on these values, assuming the
statins ± ezetimibe. Nevertheless, if this patient
laboratory values are corroborated by
does not achieve substantial further improvement
repeated testing, and based on the patient’s
overall clinical status, what treatment in LDL levels, nicotinic acid may be subsequently
should be recommended initially? considered for a trial of adjunctive therapy.
While the combination of substantial LDL
A. Diet only, with further limitation of lowering with the possibility of significant Lp(a)
saturated fat
reduction is an attractive option, PCSK9 inhibitors
B. Nicotinic acid (Answer C) are currently indicated only for
C. A PCSK9 inhibitor addition to maximal statin therapy if LDL
D. A statin lowering is not sufficient or if there is intolerance
to multiple statins.

Test Result Optimal Moderate Risk High Risk

Total LDL particles 1676 nmol/L <1138 nmol/L 1138-1409 nmol/L >1409 nmol/L

Small LDL particles 394 nmol/L <142 nmol/L 142-219 nmol/L >219 nmol/L

Medium LDL particles 457 nmol/L <215 nmol/L 215-301 nmol/L >301 nmol/L

Large HDL particles 4086 nmol/L >6729 nmol/L 6729-5353 nmol/L <5353 nmol/L

LDL subclass pattern B A N/A B

LDL peak size 216.8 Angstrom >222.9 Angstrom 229-217.4 Angstrom <217.4 Angstrom

Lipoprotein (a) 173 nmol/L … … >75 nmol/L

6  ENDO 2021 • Endocrine Case Management

chased by Dr. Khalid H. Seedahmed, Jr., MRCP PGDip, CertEndocrinology SA - ID 283


References
1. Grundy SM, Stone NJ, Bailey AL, et al. 2018 AHA/ACC/AACVPR/ 5. Krauss RM. All low-density lipoprotein particles are not created equal.
AAPA/ABC/ACPM/ADA/AGS/APhA/ASPC/NLA/PCNA guideline on Arterioscler Thromb Vasc Biol. 2014;34(5):959-961. PMID: 24740188
the management of blood cholesterol: executive summary: a report of the 6. Musunuru K, Orho-Melander M, Caulfield MP, et al. Ion mobility analysis of
American College of Cardiology/American Heart Association Task Force lipoprotein subfractions identifies three independent axes of cardiovascular
on Clinical Practice Guidelines. Circulation. 2019;139(25):e1046-e1081. risk. Arterioscler Thromb Vasc Biol. 2009;29(11):1975-1980. PMID: 19729614
PMID: 30565953 7. Nordestgaard BG, Chapman MJ, Ray K, et al. Lipoprotein(a) as a
2. Boren J, Chapman MJ, Krauss RM, et al. Low-density lipoproteins cardiovascular risk factor: current status. Eur Heart J. 2010;31(23):2844-2853.
cause atherosclerotic cardiovascular disease: pathophysiological, genetic, PMID: 20965889
and therapeutic insights: a consensus statement from the European 8. Toth PP, Grabner M, Punekar RS, Quimbo RA, Cziraky MJ, Jacobson TA.
Atherosclerosis Society Consensus Panel. Eur Heart J. 2020;41(24):2313-2330. Cardiovascular risk in patients achieving low-density lipoprotein cholesterol
PMID: 32052833 and particle targets. Atherosclerosis. 2014;235(2):585-591. PMID: 24956532
3. Berneis KK, Krauss RM. Metabolic origins and clinical significance of LDL 9. Thanassoulis G, Williams K, Ye K, et al. Relations of change in plasma levels
heterogeneity. J Lipid Res. 2002;43(9):1363-1379. PMID: 12235168 of LDL-C, non-HDL-C and apoB with risk reduction from statin therapy:
4. Davidson MH, Ballantyne CM, Jacobson TA, et al. Clinical utility of a meta-analysis of randomized trials. J Am Heart Assoc. 2014;3(2):e000759.
inflammatory markers and advanced lipoprotein testing: advice from PMID: 24732920
an expert panel of lipid specialists. J Clin Lipidol. 2011;5(5):338-367. 10. Boffa MB. Emerging therapeutic options for lowering of lipoprotein(a):
PMID: 21981835 implications for prevention of cardiovascular disease. Curr Atheroscler Rep.
2016;18(12):69. PMID: 27761705

ENDO 2021 • Adipose Tissue, Appetite, Obesity, and Lipid Metabolism  7

chased by Dr. Khalid H. Seedahmed, Jr., MRCP PGDip, CertEndocrinology SA - ID 283


 New Content

Challenging Cases in
Nutritional and Medical
Treatment of Obesity
Michael D. Jensen, MD Division of Endocrinology, Mayo Clinic, Rochester, MN; E-mail:
jensen@mayo.edu

Learning Objectives Significance of the


As a result of participating in this session, learners Clinical Problem
should be able to: Obesity is the most common nutritional problem
• Identify and estimate the various components in the United States, and it contributes to a host of
of daily energy expenditure in the context of a diseases. Too many patients believe that they must
weight management program. endure highly restrictive diets in order to succeed
with weight loss and weight-loss maintenance.
• Determine whether medications are This can lead to nihilism when it comes to
potentially contributing to weight gain or attempting to change behaviors that will improve
making it more difficult to lose weight. health in the long-term. By being more familiar
with the components and how to estimate daily
energy expenditure, providers can help patients
avoid repeated, futile diet attempts and help them
Main Conclusions select approaches that, by being more moderate,
Many patients are unaware of how many are more sustainable. Providers are encouraged
calories they consume each day to maintain to avoid supporting patient contentions that only
their weight and often believe they must endure very low-calorie diets are successful. It is possible
overly restrictive diets in order to lose weight to use online resources to make very reasonable
and keep it off. This can lead to frustration and estimates of a patient’s daily energy requirements
despair, making it even harder to commit to the and to direct a patient towards cognitive strategies
long-term behavior changes needed to have a that are most likely to promote long-term success.
healthier lifestyle. Providers who are able to make Many medications have weight gain as an
a reasonable estimate of a patient’s daily energy adverse effect, and it is often possible to substitute
expenditure can recommend more sustainable medications that have similar therapeutic benefit
changes in eating habits. When patients take without weight gain.
medications that increase their appetite, it makes it
doubly difficult to lose weight and keep it off.

8  ENDO 2021 • Endocrine Case Management

chased by Dr. Khalid H. Seedahmed, Jr., MRCP PGDip, CertEndocrinology SA - ID 283


Barriers to Optimal Practice Clinical Case Vignettes
• Quick access to online tools to estimate resting
Case 1
metabolic rate and energy expenditure of daily A 44-year-old woman presents with inability to
activity. lose weight. Her height is 67 in (170 cm), and
weight is 231.5 lb (105 kg) (BMI = 36.3 kg/m2).
• Routine review of medications for the adverse
She has tried many diets and cannot lose weight
effect of weight gain.
without consuming less than 1200 kcal/day. She is
euthyroid. Although she has a sedentary job, she
walks 30 minutes (1.5 miles) each way to work,
Strategies for Diagnosis, does her own housework, and is active gardening
Therapy, and/or Management on weekends.
Diagnosing obesity is seldom a clinical challenge.
However, it is important to have a strategy for Which of the following is the most likely
bringing up the topic that will not harm the cause of this patient’s inability to lose
doctor-patient relationship. It is also necessary to weight except on very low-calorie diets?
help patients identify areas where they can and A. Constitutionally low basal metabolic rate
will be able to make lifestyle changes that will B. High exercise efficiency phenotype (expends
accomplish the weight-loss goal or support the fewer calories for any given amount of work)
success of medication/procedural treatments. In C. Poor skills in self-monitoring
a patient with adiposity-related comorbidities,
D. Very low thermic effect of food
obtaining a brief history of the patient’s eating
and activity habits can lead to a discussion about E. Night eating syndrome
how lifestyle choices can predispose to diabetes, Answer: C) Poor skills in self-monitoring
hyperlipidemia, hypertension, etc. If the patient
is willing to make certain lifestyle changes, this is It is virtually impossible for an adult of this height
likely to improve health with the side benefit of and weight to expend less than 1200 kcal/day.
losing weight. Pointing out that lifestyle choices are Several formulas can be used to estimate basal
leading to health problems is often less threatening metabolic rate. The Harris-Benedict formula
to the patient than implying that they are unhealthy (available through numerous online calculators)
because they are “too fat” (not what we say, but predicts basal metabolic rate on the basis of height,
what they hear). If the patient is unwilling to weight, age, and sex and is accurate to within 10%
consider lifestyle changes, it is best to defer the in approximately basal metabolic rate for 80% to
conversation for another day when they are open to 90% of adults with BMIs of 18.5 to 45 kg/m2. This
the topic. Most physicians do not have time to take patient’s Harris-Benedict predicted basal metabolic
a diet history or provide activity recommendations, rate is 1768 kcal/day. Even if her basal metabolic
and therefore rely on dietitians. Dietitians with rate were 20% below predicted (consistent
additional training in motivational interviewing, with severe hypothyroidism), it would still be
behavioral strategies, and obesity management are 1430 kcal/day (thus, Answer A is incorrect).
best suited for this task. Finally, regarding good- Physical activity can be divided into
quality nutritional and behavioral interventions, 3 categories: (1) exercise (fitness- and sports-
some commercial programs have adequate, published related activities); (2) work-related physical
data. Weight Watchers provides lower cost, activity; and (3) nonexercise, nonemployment
moderately energy-restricted options, whereas Jenny (spontaneous) activity. This patient does not
Craig, Medifast, and OPTIFAST are more expensive participate in exercise per se, but despite her
and may employ severe energy-restricted approaches. sedentary job, she must walk to get to work.

ENDO 2021 • Adipose Tissue, Appetite, Obesity, and Lipid Metabolism  9

chased by Dr. Khalid H. Seedahmed, Jr., MRCP PGDip, CertEndocrinology SA - ID 283


Humans have a relatively narrow range of been estimated that the prevalence of night eating
exercise efficiency, ranging from 20% to 30% of syndrome is 1.5% in the general population in the
oxygen consumption relative to work output United States. Because this patient did not report
(thus, Answer B is incorrect).1 The online these experiences, we can rule out night eating
calculators that predict energy expenditure of syndrome.
walking as a function of the distance traveled The cognitive behavioral technique most
and the weight of the person are therefore quite strongly associated with long-term, successful
accurate. This woman walks at least 3 miles per weight-loss maintenance is self-monitoring.
day to get to and from work, which will expend Although self-monitoring of walking activity has
300 kcal/day. In addition, her housework, which become quite easy with the availability of high-
may be characterized as spontaneous physical quality motion sensors, self-monitoring food
activity, can contribute as little as 100 to as much intake still requires near-constant attention to
as 700 kcal/day depending on the amount and one’s food intake—both the types and amounts of
intensity of the work. food. Dietary recall is notoriously inaccurate (thus,
The thermic effect of food (TEF) (Answer D) Answer C is correct). Most people underestimate
averages only 10% of ingested energy,2 with their food intake, and those who are overweight
a range from 2% to 20% in our hands. Even or obese underestimate to a greater extent than do
if this patient had a basal metabolic rate of normal-weight adults. Thus, real-time monitoring
1430 kcal/day and were eating only 1800 kcal/day, is needed. Whether one uses simple pen and
her anticipated TEF (at 10% of energy intake) paper or sophisticated electronic methods, many
would be 180 kcal/day (possible range, patients find it nearly impossible to record their
36-360 kcal/day). At the very lowest end of food intake over extended periods. I find it helpful
possible TEF, she would expend only 94 kcal less to ask patients to record everything they eat for
per day than we would predict based on the 10% 2 weeks as a way for them to understand the value
TEF rule, which cannot account for her failure to of doing so. Not uncommonly, the mere act of
lose weight on a perceived intake of 1200 kcal/day. trying to record what they are eating causes them
Night eating syndrome (Answer E) was first to become more aware of their behavior and
described by Stunkard who reported a group of they lose weight—this can be a very enlightening
patients who consumed most of their daily energy experience. This exercise may also serve to identify
intake (25% or more) at a time when those without patterns of eating behavior that can more readily
obesity would not be eating, especially at night; be modified to reduce energy intake. While most
these patients also experienced insomnia and patients are able to self-monitor food intake
morning anorexia.3 These recurrent episodes of for brief periods, some cannot or will not keep
night eating are described as either excessive food accurate records. In either case, these patients will
consumption in the evening (after dinnertime [ie, find it exceeding difficult to succeed with changes
evening hyperphagia]) or eating after awakening in eating habits because it is difficult to change that
from sleep (ie, nocturnal ingestion). Night eating which cannot be measured.
syndrome is also diagnosed if there are at least 3 This patient may not have tried to self-
of the following symptoms: (1) morning anorexia; monitor food intake and thus should be provided
(2) a strong urge to eat between dinner and sleep the opportunity to do so. Because it is so common
and/or during the night; (3) sleep onset and/or for patients to believe they are eating far less than
maintenance insomnia; (4) depressed mood or they actually do, asking patients to self-monitor or
mood worsening in the evening; (5) believing having them work with a dietitian to keep a food
that one cannot get back to sleep without eating.4 diary is worthwhile. However, I have found that
The diagnosis requires that the patient be aware patients may have a desire to please the dietitian or
of and be able to recall the eating episodes. It has physician and therefore record what they think the

10  ENDO 2021 • Endocrine Case Management

chased by Dr. Khalid H. Seedahmed, Jr., MRCP PGDip, CertEndocrinology SA - ID 283


other wants to see rather than what they actually of its sedating properties) (Answer C) are also
eat. I try to emphasize that the self-monitoring associated with weight gain.
is primarily for the patient’s benefit, not to show Hypothalamic histamine suppresses feeding
someone else. behavior via the histamine-1 receptor (H1R)
in the ventromedial hypothalamus and the
Case 2 paraventricular nucleus. Histamine inhibits
appetite rather than increasing satiety. H1R
A 34-year-old woman is referred to you for antagonists/inverse agonists (older over-the-
evaluation of possible Cushing syndrome counter antiallergy medications) are associated
because she has gained 40 lb (18 kg) over the past with increased food intake and weight gain.
2 years, developed hypertriglyceridemia, and had The potencies of these drugs as H1 antagonists
worsening acne. She changed her contraceptive correlate with their orexigenic potencies. Of
program 3 years ago and has frequent insomnia. interest, neuroleptics with highest affinity for the
Her other medical problems include mild neuralgia H1R (olanzapine and clozapine) are associated
and depression. Her laboratory studies indicate a with the greatest weight gain.
normal cortisol axis. The mechanism by which corticosteroids
increase appetite is relatively well understood.
Which of the following medications would α-MSH is the ligand for the hypothalamic MC3/4
you consider changing to assess whether it is receptor, which suppresses appetite. In the
contributing to this patient’s weight gain? hypothalamus, proopiomelanocortin is processed
A. Tricyclic antidepressant amitriptyline she takes to β-lipotropin, γMSH, and ACTH; γMSH is
to treat neuralgia and depression processed to α-MSH and CLIP. Exogenous
B. Depot medroxyprogesterone she uses for corticosteroids suppress proopiomelanocortin,
contraception and thereby α-MSH production, resulting in
C. Tylenol PM she takes 5 times per week to help an unopposed neuropeptide Y/agouti-related
fall asleep peptide effect to stimulate appetite. The opposite
D. None of the above occurs with Addison disease, where cortisol
deficiency results in massive overproduction
Answer: A, B, and C) Tricyclic antidepressant of proopiomelanocortin, ACTH, and α-MSH,
amitriptyline she takes to treat neuralgia and resulting in extreme anorexia.
depression and depot medroxyprogesterone Whether medroxyprogesterone acts via this
she uses for contraception and Tylenol PM she pathway to increase appetite is unknown. There
takes 5 times per week to help fall asleep have been studies suggesting that nesfatin-1, a
hypothalamic peptide involved in the control of
Iatrogenic weight gain is not uncommon. Most appetite and energy metabolism is regulated by
endocrinologists are familiar with the weight ovarian sex steroid hormones. Specifically, in
gain potential of several antidiabetes medications ovariectomized mice, progesterone increased the
(pioglitazone, insulin, and sulfonylureas) and expression of the gene encoding nesfatin-1 in the
corticosteroids, as well as some notorious pituitary gland, and in vitro experiments showed
antipsychotic medications such as mirtazapine. mRNA expression was significantly decreased with
However, some of the older antidepressant progesterone alone in cultured pituitary tissue.7
medications that are also used for pain relief, It is possible that progesterone may act through
such as amitriptyline (Answer A), are associated the hypothalamus-pituitary axis via a nesfatin-1
with significant weight gain.5 Likewise, depot mechanism.
medroxyprogesterone (Answer B) is associated Unfortunately, merely stopping these
with fat gain.6 The older antihistamines, including medications is no guarantee of weight loss.
diphenhydramine (present in sleep aids because

ENDO 2021 • Adipose Tissue, Appetite, Obesity, and Lipid Metabolism  11

chased by Dr. Khalid H. Seedahmed, Jr., MRCP PGDip, CertEndocrinology SA - ID 283


However, if it is possible to substitute medications weight-loss program. We find it very important
that have weight loss as a side effect, that may to review both prescribed and over-the-counter
help. More importantly, if patients are taking medications when making plans for a weight-loss
medications that increase appetite, it is more intervention.
difficult to succeed with a comprehensive lifestyle

References
1. Brooks GA. Bioenergetics of exercising humans. Compr Physiol. 5. Serretti A, Mandelli L. Antidepressants and body weight: a comprehensive
2012;2(1):537-562. PMID: 23728979 review and meta-analysis. J Clin Psychiatry. 2010;71(10):1259-1272.
2. Du S, Rajjo T, Santosa S, Jensen MD. The thermic effect of food is reduced in PMID: 21062615
older adults. Horm Metab Res. 2014;46(5):365-369. PMID: 24155251 6. Dal’Ava N, Bahamondes L, Bahamondes MV, Bottura BF, Monteiro I. Body
3. Stunkard AJ, Grace WJ, Wolff HG. The night-eating syndrome; a pattern weight and body composition of depot medroxyprogesterone acetate users.
of food intake among certain obese patients. Am J Med. 1955;19(1):78-86. Contraception. 2014;90(2):182-187. PMID: 24780631
PMID: 14388031 7. Chung Y, Kim J, Im E, Kim H, Yang H. Progesterone and 17β-estradiol
4. McCuen-Wurst C, Ruggieri M, Allison KC. Disordered eating and obesity: regulate expression of nesfatin-1/NUCB2 in mouse pituitary gland. Peptides.
associations between binge-eating disorder, night-eating syndrome, and 2015;63:4-9. PMID: 25451467
weight-related comorbidities. Ann N Y Acad Sci. 2018;1411(1):96-105.
PMID: 29044551

12  ENDO 2021 • Endocrine Case Management

chased by Dr. Khalid H. Seedahmed, Jr., MRCP PGDip, CertEndocrinology SA - ID 283


 New Content

Managing Weight Effects


of Medications to Optimize
Health in Patients With
Overweight/Obesity
Andrea D. Coviello, MD, MSc, MMCi, FACE. Department of Medicine, Division of
Endocrinology, Metabolism, and Nutrition, Duke University School of Medicine, Durham, NC;
E-mail: andrea.coviello@duke.edu

Learning Objectives disease, heart rhythm disorders, and hypertension


and antihistamines used to treat seasonal allergies,
As a result of participating in this session, learners
allergic rhinitis, as well as rashes and urticarial
should be able to:
conditions. Polypharmacy can also be a problem
• Identify the variety of medication classes when a patient is prescribed multiple medications,
that are weight promoting, as well as the each of which may have a relatively small
differential weight effects of medications individual effect, but which may result in greater
within each group. weight gain when used concomitantly.
While the impact of the long-term use of
• Identify strategies to help counteract
many medications on weight gain is appreciated
medication-induced weight gain using
by clinicians, strategies to mitigate those effects
alternative medications that are weight neutral
are less well appreciated.1 A prudent approach is to
or promote weight loss.
scan medication lists for weight-promoting drugs
and substitute weight-neutral or weight loss–
promoting medications when feasible.
Main Conclusions
Many commonly used medications promote
weight gain and their use over long periods is
Significance of the
associated with overweight and obesity, conditions Clinical Problem
now affecting 75% of persons in the United Obesity (BMI ≥30 kg/m2) now affects 40% of
States. Weight gain is associated with many Americans and another third are overweight
classes of medications, including those that are (BMI 25-29.9 kg/m2). The obesity epidemic is
now well recognized such as glucocorticoids, fueling the epidemic of chronic diseases, including
antidepressants, and antipsychotics. However, prediabetes (88 million Americans), type 2 diabetes
other classes of medication are less well recognized (10.5 million Americans), and the evolving
such as β-adrenergic blockers used to treat heart epidemic of nonalcoholic fatty liver disease. Obesity

ENDO 2021 • Adipose Tissue, Appetite, Obesity, and Lipid Metabolism  13

chased by Dr. Khalid H. Seedahmed, Jr., MRCP PGDip, CertEndocrinology SA - ID 283


is also a significant risk factor for cardiovascular Strategies for Diagnosis,
disease, the leading cause of death in both men and
Therapy, and/or Management
women in the United States and around the world.
Obesity-related disorders include reproductive Many medications have off-target effects, including
disorders (eg, polycystic ovary syndrome, metabolic effects with alterations in carbohydrate
oligomenorrhea, amenorrhea, menorrhagia, and and lipid metabolism. Many promote weight gain
infertility) and breast and uterine cancer. In men, through a variety of mechanisms. The following
obesity is associated with hypogonadism, erectile classes of medications exert significant effects on
dysfunction, and prostate cancer. weight: antidepressants (particularly tricyclics),
Weight loss requires a multifaceted approach hormonal agents (including contraceptives),
for long-term success with weight reduction and antipsychotics (particularly second-generation
then weight maintenance. The 2013 American or atypical antipsychotics), anticonvulsants,
Heart Association/The Obesity Society guidelines glucocorticoids, diabetes medications (particularly
recommend lifestyle modification with changes in insulin and insulin secretagogues such as
diet composition and increases in physical activity sulfonylureas), and β-adrenergic blockers used to
as a first-line foundational approach with the treat cardiovascular disease.1 One of the first steps
addition of weight-loss medications as appropriate in approaching a treatment plan for patients with
based on individual patient characteristics overweight or obesity is to review their medication
and bariatric surgery as an option in select list to identify medications that may be promoting
individuals.2 The Endocrine Society Guidelines weight gain through excess adiposity, causing
on Pharmacologic Management of Obesity (2015) increased risk for obesity-related disorders.
also recommend lifestyle modification as an initial The most problematic weight-promoting
approach with the added component of medication medications are those that are used to treat chronic
management. This includes assessment of weight diseases, which means the patient is exposed to
effects of a patient’s current medications with weight-promoting medications over long periods.
modification of the regimen to change weight For example, a short course of steroids, 5 to 7 days,
gain–promoting medications to medications that may be prescribed to treat a patient’s asthma,
promote weight loss or are weight neutral.1 The effectively managing their condition in the short
Endocrine Society then recommends the addition term, but repeated courses of glucocorticoids
of weight-loss medications for appropriately over months to years can cause significant weight
selected patients with the goal of reducing gain. Once a patient’s condition has stabilized,
adiposity to improve health outcomes, including steroids may be continued with the perception that
the multitude of obesity-related chronic diseases.1 the glucocorticoids are controlling the patient’s
condition but the negative adverse effect is weight
gain, which may in fact exacerbate the underlying
Barriers to Optimal Practice condition. Steroid use can help control asthma,
but weight gain and obesity can then make asthma
• Lack of awareness of the effect of medications worse. Patients with asthma and obesity frequently
on weight. see an improvement in their asthma with weight
• Lack of knowledge of appropriate changes loss. Many other chronic conditions have been
in medications to those that would promote shown to improve with weight loss including type
weight loss or be weight neutral, particularly 2 diabetes, hypertension, dyslipidemia, metabolic
in patients with overweight or obesity. syndrome, cardiovascular disease, osteoarthritis,
• Time to address weight management during nonalcoholic fatty liver disease, and sleep apnea.
clinic visits for other conditions in patients Weight loss also decreases the risk of obesity-
with overweight or obesity. related cancers.1

14  ENDO 2021 • Endocrine Case Management

chased by Dr. Khalid H. Seedahmed, Jr., MRCP PGDip, CertEndocrinology SA - ID 283


Glucocorticoids (Oral, Topical, agents can be particularly problematic for women
Inhaled, Intra-articular Injections) and include progesterone-only formulations such
as medroxyprogesterone acetate injections and
Glucocorticoid medications stimulate weight gain levonorgestrel containing intrauterine devices. In
with increased adipose tissue mass accumulation. men, hypogonadism or low testosterone is actually
Accumulation of adipose tissue in the abdominal associated with excess adiposity, and treatment
region, called central adiposity, is a feature of with testosterone can improve body composition
insulin resistance, metabolic syndrome, and type with loss of adipose tissue and gain of lean muscle
2 diabetes. Significant use of exogenous steroids mass. In contrast, androgen-deprivation therapy
can even cause Cushing syndrome with obesity, for prostate cancer is associated with gain of fat
insulin resistance, diabetes, hypertension, and mass and subsequent increased risk for metabolic
mood disorders. Weight gain is most common disorders, including metabolic syndrome, type 2
with oral or intravenous glucocorticoid diabetes, and cardiovascular disease.
administration, but other formulations can also
contribute to weight gain with chronic use.
For example, long-term use of even low-dose Diabetes Medications
glucocorticoids for rheumatoid arthritis in a Diabetes is caused by an absolute (type 1) or
randomized controlled trial was associated with a relative (type 2) insulin deficiency (ie, sufficient
4% to 8% weight gain.3 Some inhalational agents insulin to keep blood glucose within the normal
and enteral steroids such as budesonide are touted range). Both types of diabetes have traditionally
as safer due to less systemic absorption, yet all have been treated with insulin, an anabolic hormone
been implicated in secondary adrenal insufficiency. that promotes weight gain. Insulin use in type
This suggests sufficient systemic absorption to 2 diabetes, which is strongly associated with
suppress the hypothalamic-pituitary-adrenal axis overweight/obesity and insulin resistance, can
and therefore sufficient absorption to contribute be particularly detrimental. The high doses
to weight gain and insulin resistance. The use of of insulin typically required to control blood
glucocorticoids for asthma, chronic obstructive glucose promote weight gain which in turn has a
pulmonary disease, rheumatologic disorders, negative impact on metabolism, typically fueling
osteoarthritis, and joint inflammation should be greater insulin resistance. Other medications
balanced against their negative adverse effects such such as insulin secretagogues (eg, sulfonylureas)
as weight gain. From an endocrine perspective, the can cause weight gain. Thiazolidinediones,
least amount of steroid necessary should be used currently pioglitazone, stimulate adipocytes
to treat the underlying condition. For intermittent in fat tissue and promote weight gain through
problems such as seasonal allergies, consideration increased fat mass, although glycemic control
should be given to episodic treatment when may be improved. Newer thiazolidinediones in
needed as opposed to chronic use for “suppression” development may have less weight-promoting
when feasible. effect. Fortunately, new classes of medications
for type 2 diabetes have been developed that
Reproductive Hormones are not only weight neutral but also promote
weight loss such as GLP-1 receptor agonists and
(Hormonal contraceptives,
SGLT-2 inhibitors. Metformin is regarded as an
Hormone Replacement insulin sensitizer that improves insulin resistance
Therapy with Sex Steroids) and lowers hepatic gluconeogenesis. Although
Hormonal agents including hormonal-based metformin is not FDA approved for weight loss,
contraceptives may cause weight gain with it is commonly associated with mild weight loss
increased adiposity. Hormonal contraceptive (3%-4%) in most patients and with moderate

ENDO 2021 • Adipose Tissue, Appetite, Obesity, and Lipid Metabolism  15

chased by Dr. Khalid H. Seedahmed, Jr., MRCP PGDip, CertEndocrinology SA - ID 283


weight loss in a subset of individuals (20% to 30% weight-loss effects. Weight effects of common
when used over time). Metformin does ameliorate medications used to treat metabolic disorders are
the metabolic derangements associated with described in Table 1.
newer psychotropic agents, particularly atypical
antipsychotics. GLP-1 receptor agonists (eg, Table 1. Weight Effects of Common Medications
exenatide, liraglutide, dulaglutide, semaglutide) are Used to Treat Metabolic Disorders

associated with weight loss when used to lower Weight Gain Weight Loss
blood glucose in the setting of diabetes, although Promoting Weight Neutral Promoting

to varying degrees. Liraglutide 3.0 mg daily has Insulin/insulin Metformin GLP-1 receptor
been FDA approved for weight loss specifically, analogues agonists
DPP-4 inhibitors
and others are in development for this indication. Sulfonylureas
α-Glucosidase
SGLT-2 inhibitors

SGLT-2 inhibitors have also been associated Meglitinides inhibitors

with weight loss when used to lower glucose Thiazolidinediones Statins (HMG
CoA reductase
levels in type 2 diabetes, with typical weight loss inhibitors)
of 5 to 10 lb (2.3 to 4.5 kg). Given the advent of Fibrates
newer choices of medications, most professional PCSK9 inhibitors
societies now recommend metformin as first-line
treatment for type 2 diabetes followed by a GLP-1
receptor agonist or SGLT-2 inhibitor given their
Table 2. Weight Effects of Psychotropic Medications Including Antidepressant and Antipsychotic Medications

Antidepressant Medications

Weight Gain Promoting Weight Neutral Weight Loss Promoting

Paroxetine (SSRI) Imipramine (tricyclic) Fluoxetine (SSRI)


Amitriptyline (tricyclic) Sertraline (SSRI) Bupropion (SSRI)
Nortriptyline (tricyclic) Citalopram (SSRI) Buspirone (serotonin receptor agonist)
Mirtazapine (tetracyclic, NaSSA) Escitalopram (SSRI)
Duloxetine (SNRI)
Venlafaxine (SNRI)

Antipsychotic Medications

Weight Gain Promoting Weight Neutral Weight Loss Promoting

Olanzapine Ziprasidone None


Quetiapine Aripiprazole
Risperidone
Cariprazine
Aripiprazole (in some)

Mood Stabilizers/Anticonvulsants

Weight Gain Promoting Weight Neutral Weight Loss Promoting

Gabapentin Lithium Topiramatea


Divalproex Lamotrigine Zonisamidea
Carbamazepine Phenytoin

Abbreviations: SSRI, serotonin reuptake inhibitor; SNRI, serotonin-norepinephrine reuptake inhibitor; NaSSA, noradrenergic and specific serotonergic
antidepressant.
a
Commonly used off-label for weight-loss effects.

16  ENDO 2021 • Endocrine Case Management

chased by Dr. Khalid H. Seedahmed, Jr., MRCP PGDip, CertEndocrinology SA - ID 283


Antidepressants and Antihypertensive and
Antipsychotic Medications Cardiovascular Medications
Antidepressants and antipsychotic medications The one category of blood pressure medication
have long been recognized to cause significant associated with weight gain is β-adrenergic
weight gain in general, but not all psychotropic blockers.8 Most classes of antihypertensive
medications have an adverse effect on weight medications are not associated with weight gain,
(Table 2). Antipsychotic medications are associated such as ACE inhibitors (eg, lisinopril, ramipril,
with significant weight gain, particularly second- captopril), angiotensin-receptor blockers (eg,
generation or atypical antipsychotics that have losartan, irbresartan), and calcium-channel
become more commonly used. Olanzapine is blockers (eg, amlodipine), although amlodipine is
associated with the most weight gain overall: associated with distal peripheral edema in the lower
2.4 kg (5 lb) in 3 months in clinical trials, but extremities, which can add mildly to weight gain
frequently much more in clinical practice in through fluid retention but not excess adiposity.
patients who have been on the medication for
longer periods. Some individuals, particularly
younger patients, may gain up to 45 kg
Clinical Case Vignettes
(100 lb).4 Quetiapine and risperidone are also Case 1
associated with significant weight gain. The A 66-year-old woman with class III obesity
weight gain caused by atypical antipsychotic (BMI 40.2 kg/m2 [height 68 in (173 cm), weight
agents is associated with the development of or 264 lb (120 kg)]) presents for management of
exacerbation of insulin resistance and onset of type secondary adrenal insufficiency in the context
2 diabetes. Initiation of metformin for patients of multiples sources of exogenous steroids. She
taking second-generation atypical antipsychotics has type 2 diabetes treated with metformin
has been shown to help by promoting mild (hemoglobin A1c 7.4% [57 mmol/mol]); familial
weight loss and improving glucose metabolism, combined hyperlipidemia treated with fenofibrate
insulin resistance, and the metabolic syndrome.5 and simvastatin; hypertension treated with
Topiramate, FDA approved for use in seizure lisinopril, triamterene hydrochlorothiazide, and
disorders and migraine prophylaxis, has also spironolactone; allergic rhinitis treated with
been shown to promote weight loss and improve fluticasone twice daily; and allergic asthma treated
glucose metabolism when used in conjunction with fluticasone-salmeterol inhalation twice
with antipsychotic medications. daily and oral steroid bursts as needed for acute
exacerbations of asthma. She receives intermittent
Antihistamines steroid injections for degenerative spine disease.
She was able to slowly wean her daily steroids
Long-term use of antihistamines, including over-
over the course of a year and had a normal result
the-counter antihistamines, has been associated
on cosyntropin-stimulation testing 3 months after
with weight gain in observational studies, but
being steroid-free:
adequate quantitative data from controlled studies
on their weight effects are lacking.6,7 The weight Baseline ACTH = 33 pg/dL (SI: 7.3 pmol/L)
gain–promoting effects of some antihistamines are Cortisol = 12.5 µg/dL (SI: 345 nmol/L)
thought to be mediated through their affinity for 60-minute cortisol = 23.5 µg/dL (SI: 648 nmol/L)
the H1-histamine receptor.7 Antihistamines are She loses 11 lb (5 kg) once off all steroid
thought to increase appetite. The antihistamine treatments. She wants to lose more weight but
cyproheptadine is potentially weight neutral.4 is limited in activity by her degenerative spine
disease and chronic back pain. Her insurance plan
does not cover weight management medications.

ENDO 2021 • Adipose Tissue, Appetite, Obesity, and Lipid Metabolism  17

chased by Dr. Khalid H. Seedahmed, Jr., MRCP PGDip, CertEndocrinology SA - ID 283


Which of the following medications Case 2
should be started as the best next
A 40-year-old woman presents with worsening
step to promote weight loss?
type 2 diabetes in the setting of weight regain after
A. Sitagliptin bariatric surgery 2 years ago. She was diagnosed
B. Pioglitazone with bipolar disorder in her mid-20s, and she was
C. Semaglutide treated with olanzapine. She gained 70 lb (32 kg)
D. Empagliflozin to a peak weight of 305 lb (138 kg). She developed
type 2 diabetes, nonalcoholic fatty liver disease
Answer: C) Semaglutide (which progressed to nonalcoholic steatohepatitis
with advanced fibrosis on liver biopsy),
This patient has class III obesity and multiple
hypertension, and an idiopathic cardiomyopathy.
obesity-related comorbidities including type
Olanzapine was switched to aripiprazole, and
2 diabetes, hypertension, and hyperlipidemia.
she lost some weight before plateauing at 265 lb
Obesity also has a negative effect on her asthma,
(120 kg) (BMI 40.3 kg/m2).
and steroid treatment for her asthma caused more
She tried multiple weight-loss medications,
weight gain, creating a vicious cycle. Weaning
including phentermine and liraglutide 3.0 mg
off steroids allowed her to lose some weight, but
daily, in addition to metformin but was unable to
additional weight loss would improve her multiple
tolerate them. Due to her liver disease and type
obesity-related conditions. Her hemoglobin A1c
2 diabetes, she underwent Roux-en-Y gastric
level of 7.4% (57 mmol/mol) is above target and
bypass at age 38 years and lost approximately 65 lb
allows for the addition of a second diabetes agent.
(30 kg) (weight 200 lb [91 kg]; BMI 30.4 kg/m2).
Multiple diabetes medications are associated
She had significant regression in her diabetes
with weight loss, including GLP-1 receptor agonists
with a decrease in hemoglobin A1c from 9.6% to
and SGLT-2 inhibitors. Injectable GLP-1 receptor
6.5% (81 to 48 mmol/mol) and reduction in her
agonists are associated with more weight loss
medications, eventually stopping insulin.
(5%-10% of body mass) than the oral SGLT-2
Two years after gastric bypass, she
inhibitors such as empagliflozin (Answer D),
regained approximately 44 lb (20 kg) (weight
which are associated with 5 to 10 lb (2.3-4.5 kg)
251 lb [114 kg]; BMI 38 kg/m2), and her diabetes
weight loss on average. The addition of semaglutide
worsened with an increase in hemoglobin A1c to
(Answer C) would improve her glycemic control
10.6% (92 mmol/mol). She restarted basal-bolus
and stimulate weight loss. The oral DPP-4
insulin in addition to liraglutide and metformin.
inhibitors such as sitagliptin (Answer A) work on
Current medications are insulin glargine,
the same pathway as GLP-1 receptor agonists, but
50 units twice daily; insulin lispro, 25 units with
they are not associated with significant weight loss
meals; liraglutide, 1.8 mg daily; metformin;
and have less effect on glycemic control. Pioglitazone
lisdexamfetamine; buspirone; cariprazine; and
(Answer B) has been shown to improve fatty liver
fexofenadine.
disease as well as glycemic control, but it promotes
weight gain, not weight loss. Which of the following medications is
After switching to semaglutide and titrating most likely contributing to her weight
up to a dose of 1 mg subcutaneously once weekly regain after bariatric surgery?
in combination with metformin, the patient lost
A. Cariprazine
another 13 lb (6 kg) for a total of approximately
25 lb (11 kg) over 1.5 years with a decrease in B. Lisdexamfetamine
BMI from 40.2 to 36.3 kg/m2. Her hemoglobin C. Buspirone
A1c improved to 6.2% (44 mmol/mol) without D. Fexofenadine
hypoglycemia.

18  ENDO 2021 • Endocrine Case Management

chased by Dr. Khalid H. Seedahmed, Jr., MRCP PGDip, CertEndocrinology SA - ID 283


Answer: A) Cariprazine to diet and activity levels, which helped her lose
weight after her second child. Two months ago,
Most atypical antipsychotic agents are associated her sertraline dose was increased to 100 mg daily
with weight gain and obesity. Olanzapine and and her propranolol dose was increased from
clozapine are associated with the most weight 20 to 40 mg daily. She also takes levothyroxine,
gain, up to 100 lb (45 kg) in some individuals. 125 mcg daily; atorvastatin, 20 mg daily; and a
Some agents are classified as weight neutral, such multivitamin. Current BMI is 29.3 kg/m2 (height
as aripiprazole, but these agents are associated 62 in [157 cm]; weight 160 lb [73 kg]). Her thyroid
with mild weight gain of approximately 9 to function tests are within normal limits.
10 lb (4-4.5 kg) in general and significantly more
in some individuals. Cariprazine (Answer A) is Which of the following is the best next
known to cause weight gain, although less than step in her medical management?
olanzapine.
A. Switch sertraline to escitalopram
This patient appears to be very sensitive
to weight gain and insulin resistance with use B. Switch propranolol to topiramate
of atypical antipsychotics. Lisdexamfetamine C. Increase her levothyroxine dosage
(Answer B) is used to treat binge eating disorder, D. Add liothyronine
which is characterized by high caloric intake,
particularly at night, which this patient has, but Answer: B) Switch propranolol to topiramate
it is not associated with weight gain. Buspirone This woman reports a more than 10% weight gain
(Answer C) is an anxiolytic medication that has in less than a year. Notably, she had returned to
been associated with weight loss in some patients a normal BMI after delivering her second child
and is weight neutral in most. Fexofenadine and then started to gain weight during a period of
(Answer D) is an antihistamine than can cause stress, which is a common environmental factor
weight gain in some patients, but the amount of that leads to weight gain. However, she has been
weight gain is minor relative to the weight gain managing her diet and activity level as she had
seen with atypical antipsychotic medications. done to successfully lose the weight that she gained
She is already on metformin, which has with her second pregnancy. She was started on 2
been used to mitigate weight gain with the use new mediations, one of which is associated with
of atypical antipsychotics. Topiramate would be weight gain. Propranolol, a β-adrenergic blocker,
another option. is commonly used to reduce the frequency of
migraines, but this medication class is associated
Case 3 with weight gain. The escalation in weight gain
coincides with the increase in her propranolol
A 37-year-old woman with hypothyroidism
dose from 20 to 40 mg daily. Topiramate
treated with levothyroxine presents with weight
(Answer B) is FDA approved for migraine
gain over the last 6 to 8 months. Her BMI was
prophylaxis and is also associated with weight loss.
in the normal range at 24.7 kg/m2 (height 62 in
It would be a better agent for this patient who is
[157 cm]; weight 135 lb [61.2 kg]) 1 year after
now approaching an obese BMI. Topiramate in
delivering her second child. She relates that she has
combination with phentermine is FDA approved
been under considerable stress and notes increased
for weight loss and the combination is associated
anxiety and a return of migraines. Her primary
with the most weight loss when use long-term
care provider started sertraline, 50 mg daily, and
(~10%). As a reproductive-aged woman, it is
propranolol, 20 mg daily, for migraine prophylaxis
important that she use reliable contraception if
with sumatriptan as needed. She gained 15 lb
she were to start topiramate, as this drug can have
(6.8 kg) over 6 months. She gained another 5 to
teratogenic effects on a developing fetus.
6 lb (2.3-2.7 kg) in the last month despite attention

ENDO 2021 • Adipose Tissue, Appetite, Obesity, and Lipid Metabolism  19

chased by Dr. Khalid H. Seedahmed, Jr., MRCP PGDip, CertEndocrinology SA - ID 283


Both sertraline and escitalopram are her levothyroxine dosage (Answer C) or
considered to be weight neutral, so switching adding liothyronine (Answer D) would not be
one for the other (Answer A) would not be indicated. Additionally, increasing her thyroid
expected to help her lose weight. Her thyroid hormone replacement could cause iatrogenic
function tests are normal on her current thyroid hyperthyroidism, worsen her anxiety, and cause
hormone replacement regimen, so increasing other adverse effects.

References
1. Apovian CM, Aronne LJ, Bessesen DH, et al; Endocrine Society. 5. Lee YJ, Jeong JH. A systematic review of metformin to limit weight-
Pharmacological management of obesity: an Endocrine Society clinical gain with atypical antipsychotics. J Clin Pharm Ther. 2011;36(5):537-545.
practice guideline. J Clin Endocrinol Metab. 2015;100(2):342-362. PMID: 21418264
PMID: 25590212 6. Ratliff JC, Barber JA, Palmese LB, Reutenauer EL, Tek C. Association
2. Apovian CM, Aronne LJ. The 2013 American Heart Association/American of prescription H1 antihistamine use with obesity: results from the
College of Cardiology/The Obesity Society guideline for the management of National Health and Nutrition Examination Survey. Obesity (Silver Spring).
overweight and obesity in adults: what is new about diet, drugs, and durgery 2010;18(12):2398-2400. PMID: 20706200
for obesity? Circulation. 2015;132(16):1586-1591. PMID: 26481564 7. Salvi V, Mencacci C, Barone-Adesi F. H1-histamine receptor affinity
3. Da Silva JAP, Jacobs JWG, Kirwan JR, et al. Safety of low dose glucocorticoid predicts weight gain with antidepressants. Eur Neuropsychopharmacol.
treatment in rheumatoid arthritis: published evidence and prospective trial 2016;26(10):1673-1677. PMID: 27593622
data. Ann Rheum Dis. 2006;65(3):285-293. PMID: 16107513 8. Sharma AM, Pischon T, Hardt S, Kunz I, Luft FC. Hypothesis: Beta-
4. Domecq JP, Prutsky G, Leppin A, et al. Clinical review: drugs commonly adrenergic receptor blockers and weight gain: A systematic analysis.
associated with weight change: a systematic review and meta-analysis. J Clin Hypertension. 2001;37(2):250-254. PMID: 11230280
Endocrinol Metab. 2015;100(2):363-370. PMID: 25590213

20  ENDO 2021 • Endocrine Case Management

chased by Dr. Khalid H. Seedahmed, Jr., MRCP PGDip, CertEndocrinology SA - ID 283


 New Content

What an Endocrinologist
Should Know About the
Surgical Treatment of Obesity
Daniel H. Bessesen, MD. Division of Endocrinology, Metabolism, and Diabetes, University
of Colorado, School of Medicine, Aurora, CO; E-mail: Daniel.Bessesen@cuanschutz.edu

Learning Objectives cholecystectomy. Preoperative evaluation includes


screening for preexisting vitamin deficiencies;
As a result of participating in this session, learners
optimizing medical conditions such as T2DM
should be able to:
and sleep apnea; and fully informing patients of
• Identify patients who would benefit from the need for ongoing monitoring of nutritional/
bariatric surgery and feel confident that they vitamin status; and avoiding medications such
can clearly communicate the risks and benefits as nonsteroidal antiinflammatory agents and
of bariatric surgery to these patients. cigarette smoking that can predispose to surgical
complications. Pregnancy should be avoided
• Assess and manage common medical problems
for 12 to 18 months following the procedure.
and nutritional deficiencies preoperatively
Postoperative monitoring of vitamins B12, D, A,
and postoperatively in patients undergoing
K; iron; copper; zinc; PTH; and other measures
bariatric surgery.
of bone health including bone densitometry can
help prevent long-term complications. Mechanical
complications such as anastomotic leaks,
strictures, and marginal ulcers may present to the
Main Conclusions endocrinologist. Weight regain following initial
Endocrinologists commonly encounter patients weight loss may be a sign of a mechanical problem
who would benefit from, as well as those who with the surgery, changes in lifestyle habits, or
have already had, bariatric surgery. Bariatric simply a failure of the surgery to maintain weight
surgery is the most effective treatment currently loss. New or worsening gastroesophageal reflux
available for weight loss, and it dramatically disease along with weight regain may be an
improves weight-related comorbidities, especially indication for endoscopic or surgical revision of
type 2 diabetes mellitus (T2DM). Evidence is the original surgery.
accumulating that bariatric surgery reduces
mortality from cancer and cardiovascular disease
as well. Mortality and morbidity associated
with the procedure have steadily fallen, and
mortality rates are lower than those seen with
commonly performed surgical procedures such as

ENDO 2021 • Adipose Tissue, Appetite, Obesity, and Lipid Metabolism  21

chased by Dr. Khalid H. Seedahmed, Jr., MRCP PGDip, CertEndocrinology SA - ID 283


liraglutide, 3 mg), and bariatric surgery. Lifestyle
Significance of the treatment alone typically provides 3% to 8%
weight loss depending on the intensity of
Clinical Problem treatment and is appropriate for all those with a
Obesity is one of the most common health BMI greater than 25 kg/m2. Antiobesity
problems in the United States. The most recent medications currently give an additional 3% to 10%
data from the National Health and Nutrition weight loss over that provided by lifestyle
Examination Survey (NHANES), which directly treatment alone and are appropriate to consider in
measures height and weight annually in a those with a BMI greater than 30 kg/m2 or in
nationally representative sample of 5000 people of patients with a BMI greater than 27 kg/m2 who
all ages, demonstrate that the prevalence of obesity also have a weight-related comorbidity. Bariatric
among US adults aged 20 years and older increased surgery (adjustable gastric banding [AGB], vertical
from 26.4% in 2006 to 42.4% in 2018. A recent sleeve gastrectomy [VSG], Roux-en-Y gastric
analysis used a statistical approach to correct bypass [RYGB], and biliopancreatic diversion with
the Behavioral Risk Factor Surveillance System duodenal switch [BPD/DS]) provides 20% to 35%
data for the limitations of self-reported height weight loss and is appropriate to consider in
and weight. The results of this analysis predict patients with a BMI greater than 40 kg/m2 or in
that by 2030, 48.9% of adults in the United States those with a BMI greater than 35 kg/m2 who also
will have obesity and 24.2% of adults will have have a weight-related comorbidity (see Figure).
severe obesity. In fact, severe obesity is poised to
Figure. Most Commonly Performed Types
become the most common BMI category (among of Bariatric Surgical Procedures
4: normal weight, overweight, obesity, severe
obesity) affecting women (27.6%), nonHispanic Roux-en-Y Gastric Bypass
black adults (31.7%), and low-income adults
(31.7%). Obesity predisposes to T2DM; cancers
of the breast uterus, colon, esophagus, and others;
obstructive sleep apnea; nonalcoholic fatty liver
disease; infertility; hypertension; coronary artery
disease; degenerative joint disease; depression; and
reduced functional capacity. While many patients
and even health care professionals continue to
believe that obesity is the result of poor lifestyle
choices that can be remedied by education,
obesity is now generally accepted by virtually
all professional organizations to be a disease
of body weight regulation. As such, clinicians Sleeve Gastrectomy
should be comfortable counseling patients on the
available options for treating obesity. Because
of the strong association of obesity with T2DM,
endocrinologists are often in a position to discuss
obesity treatment with their patients.1
There are currently 3 main treatment options
for obesity that can be considered individually or
in combination: lifestyle treatment, antiobesity
medications (phentermine; orlistat; phentermine/
topiramate ER; bupropion/naltrexone; and

22  ENDO 2021 • Endocrine Case Management

chased by Dr. Khalid H. Seedahmed, Jr., MRCP PGDip, CertEndocrinology SA - ID 283


There is increasing evidence of the benefits of Strategies for Diagnosis,
bariatric surgery in persons with T2DM and a
Therapy, and/or Management
BMI 30 to 35 kg/m2 in whom hyperglycemia
is not well controlled despite aggressive Risks and Benefits of
medical management. Each of these treatments Bariatric Surgery
demonstrates marked interindividual variability When seeing a patient who asks about bariatric
in weight loss, with some individuals losing surgery, it is important for the endocrinologist
much more than the mean predicted and some to convey the benefits and risks of surgery in a
individuals losing nothing or even gaining weight. way that will help that patient make an informed
In 2018, more than 252,000 patients had bariatric decision about the best next step. There is a good
surgery. This number represents a 60% increase deal of misinformation about both the risks and
over the number of procedures performed in benefits of bariatric surgery. What follows are
2011. VSG is now the most common operation some brief points that can be used in clinic to help
performed, accounting for 61% of all bariatric inform patients as they consider surgery. The
surgeries. The number of RYGB procedures has goal is not to advocate for surgery but to provide
declined over time, but there has been a slight balanced information on risks and benefits.2
increase in the number performed over the last
few years; it accounts for 17% of all procedures
performed. AGB procedures are rarely performed Weight Loss
any more, as efficacy is lower and complications While randomized controlled trials do not
are higher than had initially been appreciated. show significant differences, observational
BPD/DS procedures are associated with the studies generally show that patients undergoing
greatest weight loss but also the greatest long-term RYGB operations lose more weight than those
risk of nutritional deficiencies and are only rarely undergoing either VSG or AGB. The large
performed. PCORnet Bariatric Study, which used data from
electronic medical records to monitor weight loss
Barriers to Optimal Practice in more than 60,000 patients who underwent
bariatric surgery in 41 different health care systems
• Lifestyle treatment often provides only limited across the United States, found that 5-year weight
success in helping individuals with severe loss was 25.5% in those who had RYGB, 18.8% in
obesity lose enough weight to significantly those who had VSG, and 11.7% in those who had
improve health. Additionally, both lifestyle AGB. While most patients experience some weigh
treatments and antiobesity medications are regain between years 2 and 4 following surgery,
rarely paid for by health care insurers. a number of studies have now followed patients
for 10 to 20 years after surgery and found that
• While bariatric surgery is the most successful most maintain more than half of the initial weight
treatment currently available for severe lost. Many patients do not think that weight
obesity, most endocrinologists have not loss provided by bariatric surgery is durable, but
been formally trained in the evaluation and existing evidence suggests it is for most patients.
management of patients undergoing bariatric
surgery.
Type 2 Diabetes
There is now clear and compelling evidence of
the benefits of bariatric surgery in persons with
T2DM. To date, there have been 12 randomized
controlled trials of bariatric surgery for the

ENDO 2021 • Adipose Tissue, Appetite, Obesity, and Lipid Metabolism  23

chased by Dr. Khalid H. Seedahmed, Jr., MRCP PGDip, CertEndocrinology SA - ID 283


treatment of T2DM. These trials have enrolled Microvascular and Macrovascular Disease
a total of 874 patients (38-150 patients per trial). A meta-analysis that combined data from 3
All of these studies except for one that tested randomized controlled trials with observational
AGB have shown surgery to be superior to medical studies that collected data on more than 17,000
therapy in inducing remission and controlling patients with T2DM showed that bariatric surgery
glucose levels while requiring less medication. A reduced the risk of microvascular complications
number of these studies have found that initial high by 74% compared with medical therapy. Another
levels of remission are not sustained on long-term meta-analysis of 7 observational studies of more
follow-up, but surgery remains far superior to than 29,000 patients with T2DM showed a 48%
medical therapy in addressing glucose homeostasis. reduction in macrovascular complications and 79%
The Stampede Trial randomly assigned 150 patients lower mortality at 5 years of follow-up in those
with T2DM to either medical therapy, VSG, or who underwent bariatric surgery as compared
RYGB. Weight loss at 1 year was 5.4% in the medical with those undergoing medical therapy.
group, 25.1% in the VSG group, and 29.4% in the
RYGB group. The primary endpoint of this study Hypertension
was the fraction of patients with a hemoglobin A1c Most patients seeking bariatric surgery have a
value less than 6.0%. At 1 year of follow-up, only history of hypertension. Studies examining the
12% of patients in the medical therapy group had effects of weight-loss surgery on hypertension
a hemoglobin A1c level less than 6.0%, while 42% have shown that surgery results in remission of
of the RYGB group and 37% of the VSG group hypertension in 43% to 83% of affected individuals
achieved this endpoint. Mean hemoglobin A1c at 1 year. As is true with T2DM, the beneficial
levels in the 3 groups were 7.5% (medical therapy), effects of surgery are not always durable, and as
6.4% (RYGB), and 6.6% (VSG). After 5 years, the many as 44% of patients who initially experience
primary endpoint of a hemoglobin A1c level less remission of hypertension have a recurrence
than 6% was achieved by 5% (medical therapy), 29% requiring reinstitution of medications within
(RYGB), and 23% (VSG). The fraction of patients 10 years following surgery.
taking no glucose-lowering medication at 5 years
was 2% (medical therapy), 45% (RYGB), and 25% Cancer
(VSG). These data, which in general are confirmed Obesity is associated with increased risk for a
by findings in other randomized controlled trials, number of cancers. If the relationship is causal,
support the ideas that bariatric surgery induces then weight reduction should result in lower
remission in a substantial number of patients rates of cancer. Indeed, data from 8 observational
with T2DM, but that over time carbohydrate studies involving more than 630,000 patients
metabolism tends to deteriorate. This suggests that suggest that bariatric surgery is associated with
the significant weight loss produced by the surgery a 28% reduced risk for developing all types of
“turns back the clock” on the progression of T2DM. cancer, a 45% reduced risk of obesity-associated
The data also support the idea that RYGB is more cancers, and a 50% reduction in breast cancer.
effective than VSG at producing weight loss and The prospective observational Swedish Obese
improving glucose levels in persons with T2DM. Subjects trial found reduced overall mortality in
Bariatric surgery really is the most effective form of participants who underwent bariatric surgery
treatment for patients with T2DM and obesity. 15 years earlier. The greatest reduction in
mortality came not from cardiovascular disease
but from reduced cancer-related mortality. This
Other Health Issues information may be most important for women
The substantial weight loss produced by bariatric with obesity and a family history of breast cancer.3
surgery reduces morbidity and mortality from a
number of other weight-related comorbidities:

24  ENDO 2021 • Endocrine Case Management

chased by Dr. Khalid H. Seedahmed, Jr., MRCP PGDip, CertEndocrinology SA - ID 283


Sleep Apnea registry that allows for monitoring of adverse events
In a pooled analysis of 27 studies involving more occurring in patients undergoing bariatric surgery.
than 1100 patients with sleep apnea, bariatric With modern laparoscopic surgical methods
surgery was associated with a reduction in the and certification of metabolic surgery programs
apnea-hypopnea index from 39.3 events per hour by MBSAQIP, there has been a steady decline in
to 12.5 events per hour. Epworth Sleepiness surgical morbidity. Currently, when performed
Scores fell from 11.1 to 5.6 with a score greater by an accredited program, bariatric surgery
than 10 indicating clinically significant sleepiness. has a lower mortality rate than that seen with
While bariatric surgery can help many patients laparoscopic cholecystectomy or hip replacement
with sleep apnea, many are still left with clinically (0.1%, 0.7%, and 0.93%, respectively). Likewise,
significant apnea. 30-day complication rates for RYGB (3.4%) are
comparable to or lower than those seen with
Overall Mortality other commonly performed surgical procedures
Short-term mortality is slightly higher in those such as laparoscopic hysterectomy (3.5%),
undergoing bariatric surgery than in those who do laparoscopic cholecystectomy (3.7%), laparoscopic
not because of the modest rates of perioperative appendectomy (4.5%), and total knee arthroplasty
mortality. However, by 5 to 10 years following (5.2%). The message to convey to patients is that
surgery, several observational studies show while bariatric surgery is major surgery, the short-
reduced all-cause mortality in surgically treated term perioperative risks are no greater than those
patients as compared with matched patients who seen with other commonly performed procedures.
did not have surgery. In one study of more than
10,000 individuals with obesity living in Utah,
Preoperative Evaluation
7-year all-cause mortality was 2.7% in those who
had surgery compared with 4.1% in matched and Management
control participants. In another study of patients Education
cared for by the Veterans Administration health It is important that individuals considering
care system, 10-year mortality was 13.8% in those bariatric surgery have a realistic view of the risks
who had surgery compared with 23.9% in matched and benefits. A conversation with their physician
control participants.4 is a start. This should be supplemented with
informational seminars from the surgical group
that will be performing the surgery. Physicians can
Risks
help patients find a good local surgical program
Because of increased risk and concerns over through the MBSAQIP website (https://www.
reduced effectiveness, bariatric surgery is facs.org/search/bariatric-surgery-centers). This
contraindicated in patients with severe heart web-based tool can be used in conjunction with
failure, unstable coronary artery disease, end- the American Society for Metabolic and Bariatric
stage lung disease, active cancer, cirrhosis with Surgery website (https://asmbs.org/patients/
portal hypertension, uncontrolled drug or alcohol find-a-provider) to find a local program that has
dependency, Crohn disease, severely impaired been accredited by MBSAQIP. It is often useful
intellectual capacity, or current or planned for patients to attend a support group associated
pregnancy within the next 1 to 2 years. The with the surgical program, so that they can meet
Metabolic and Bariatric Surgery Accreditation other people who have had surgery and get
and Quality Improvement Program (MBSAQIP), honest, firsthand information directly from these
which is a joint effort between the American individuals about their personal experiences. It is
College of Surgeons (ACS) and the American important for prospective patients to understand
Society of Metabolic and Bariatric Surgery, has the need for long-term nutritional monitoring and
played a central role in developing a national data

ENDO 2021 • Adipose Tissue, Appetite, Obesity, and Lipid Metabolism  25

chased by Dr. Khalid H. Seedahmed, Jr., MRCP PGDip, CertEndocrinology SA - ID 283


ongoing vitamin supplementation. Patients should that psychological challenges are common,
be informed of the potential need for plastic that they can get in the way of a successful
surgery to address the possible issue of excess skin outcome following surgery, and that establishing
following weight loss involving the pannus, inner a relationship with a psychological health
thighs, and/or upper arms. Insurance coverage professional before surgery will be important
for surgery to address excess skin is sometimes in determining whether surgery is right for the
an issue. Patients should understand that actual person and ensuring the best long-term outcomes.
weight loss can be quite variable from person
to person. MBSAQIP has developed a bariatric Management of Associated Health Conditions
surgery decision tool/risk/benefit calculator Bariatric surgery is elective surgery, so the job of
(https://www.facs.org/quality-programs/ the endocrinologist is to optimize patient health
mbsaqip/calculator) that can also provide before the operation. Ideally, hemoglobin A1c
prospective patients with information that may be levels should be less than 8.0% before surgery
useful to them as they consider having surgery. in those with T2DM. Because weight-loss
surgery may not be as effective in patients with
Nutritional Evaluation Prader-Willi syndrome, genetic testing could be
All patients considering surgery will be evaluated considered in patients with a history suggesting
by a registered dietician nutritionist. While this diagnosis. Patients should be screened for
many insurance plans require adherence to 6 sleep apnea with a screening questionnaire such as
months of a supervised weight-loss program, the STOP-BANG questionnaire (Table 1).
there is no evidence that this practice improves
long-term outcomes. The most important aspect Table 1. STOP-BANG Questionnaire
of the presurgical nutritional evaluation is to
Scoring Use the following point tabulation
help patients understand the parameters of the for each question
0-2: Low risk
diet that they will need to consume following 0 = No
3-4: Intermediate risk
surgery with a particular focus on ensuring that 1 = Yes
5-8: High risk
in the immediate postoperative period they S: Snore loudly?
consume sufficient protein, fluid, vitamins, and T: Feel tired during the day?
minerals to sustain health. They will also benefit O: Observe apnea while sleeping?
from developing a personal relationship with a P: Treated for high blood pressure?
registered dietician nutritionist who can be an B: Body mass index >35 kg/m2?
important resource for them when questions arise A: Age >50 years?
in the postoperative period. N: Neck circumference
Male >17 inches?
Psychological Evaluation Female >16 inches?
Many individuals with obesity experience G: Gender, male?
depression, anxiety, social isolation, and
other psychological problems. In the past, the If symptoms are present, the patient should be
preoperative psychological evaluation was tested with polysomnography and treated with
often used to screen out individuals who were positive airway pressure for at least 1 month
perceived as being “noncompliant.” More recently, before surgery. Patients should be screened
accumulating data have revealed that suicide for vitamin deficiencies listed in Table 2 and
risk may increase following bariatric surgery, as supplemented to achieve levels in the normal
does the risk for substance/alcohol abuse. The range before surgery. It is easier to normalize
important messages for the endocrinologist vitamins D and B12 and iron before an operation
to convey to patients considering surgery are than to deal with more severe deficiencies

26  ENDO 2021 • Endocrine Case Management

chased by Dr. Khalid H. Seedahmed, Jr., MRCP PGDip, CertEndocrinology SA - ID 283


postoperatively. Because it is not advisable as an intrauterine device might be considered.
for women to become pregnant in the year Gastroesophageal reflux may develop or worsen
following surgery, discussing reproductive following VSG. Some centers routinely obtain an
plans with women preoperatively is important. upper endoscopy study preoperatively to assist in
Some women with obesity may assume that the selection of procedures, as well as to establish
they are infertile and be sexually active without a baseline for long-term follow-up. Testing for
contraception and yet not desiring pregnancy. and treating Helicobacter pylori if present might be
Many of these women will experience improved considered in areas of high prevalence. Those with
fertility following surgery and should plan for degenerative joint disease should be transitioned
effective contraception in the postoperative off nonsteroidal antiinflammatory agents
period. Oral contraceptives may be less effective preoperatively, as these are contraindicated in the
and may predispose to thromboembolic disease postoperative period.5
following bariatric surgery, so other options such

Table 2. Signs and Symptoms of Deficiencies of Vitamins and Minerals

Vitamin Signs and Symptoms


and Mineral
Deficiencies

Vitamin B1 Muscle weakness, ataxia, loss of reflexes


(thiamine)
Oculomotor dysfunction (nystagmus/ophthalmoplegia), paresthesia, confusion, heart failure/edema (wet beriberi)
Wernicke encephalopathy triad: mental status change, oculomotor changes, gait ataxia

Vitamin B12 Sensory distal neuropathy


(cobalamin)
Loss of proprioception, ataxia
Macrocytic anemia
Dementia, depression
Glossitis

Folate Macrocytic anemia, neural tube defects


Mild pancytopenia

Iron Microcytic anemia, fatigue, decrease work performance, glossitis, nail spooning or ridges, pica, restless legs

Calcium Bone disease, secondary hyperparathyroidism

Vitamin D Osteomalacia, muscle weakness and pain, cramps, tingling sensation, bone pain, hypocalcemia, tetany

Vitamin A Bitot spots, night blindness


Skin hyperkeratosis
Blindness

Vitamin E Neuropathy (sensor and motor)


Ataxia
Hemolysis

Vitamin K Impaired coagulation


Osteoporosis

Zinc Decreased taste/smell, impaired healing


Rash, hair loss, glossitis, diarrhea

Copper Hypochromic anemia, neutropenia, ataxia, paresthesia, myeloneuropathy

ENDO 2021 • Adipose Tissue, Appetite, Obesity, and Lipid Metabolism  27

chased by Dr. Khalid H. Seedahmed, Jr., MRCP PGDip, CertEndocrinology SA - ID 283


Immediate Postoperative Abdominal imaging shows extravasation
Management and Potential of contrast from the gastric lumen into the
peritoneum. This complication is a surgical
Complications
emergency, and all physicians who might be called
Medication Management by patients following bariatric surgery should be
Bariatric surgery may alter the absorption of aware of this presentation.
medications. As a result, in the postoperative
period, it is best to administer all medications as Thiamine Deficiency
either a liquid or as crushed tablets. Sustained Another serious complication that may be seen
or extended-release formulations should be shortly after surgery is Wernicke encephalopathy.
avoided. In patients with T2DM, glucose levels This typically presents 1 to 3 weeks following
improve very quickly even before substantial surgery in a patient who has several days of
weight loss occurs. For this reason, in the severe nausea and vomiting and inability to
immediate postoperative period it is best to stop keep anything down. The patient then develops
scheduled insulin and just use correction-factor ataxia, nystagmus, and confusion. These features
insulin for high glucose levels. Sulfonylureas are manifestations of thiamine deficiency,
should be avoided because of the risk of inducing which, if unrecognized and untreated, can lead
hypoglycemia. SGLT-2 inhibitors avoid the to irreversible central nervous system damage.
issue of hypoglycemia but may promote volume Treatment consists of parenteral thiamine
depletion, which can be a problem postoperatively administration of 250 to 600 mg daily depending
because of difficulties getting sufficient fluid on the severity of the symptoms. To prevent
intake. Metformin is a reasonable option, deficiency, patients who have had bariatric
but some clinicians prefer GLP-1 receptor surgery should take daily multivitamins. Because
agonists if glucose-lowering medications are the postsurgical requirement for thiamine is
needed. Blood pressure also tends to fall in the higher than the dose typically contained in over-
immediate postoperative period. As a result, the-counter multivitamins, patients should be
antihypertensive medications can be cut in advised to take 2 high-quality over-the-counter
half and diuretics stopped postoperatively to multivitamins or a multivitamin formulated
minimize the risk of hypovolemia. As mentioned specifically for individuals who have had bariatric
above, nonsteroidal antiinflammatory agents surgery.6
should not be used because of the risk of causing
anastomotic ulcerations. For most patients, lipid
levels fall postoperatively especially acutely when
Long-Term Follow-Up and
food intake is markedly reduced. Thus, most Management Issues
lipid-lowering medications can be stopped, and Monitoring for and Treatment of
consideration for restarting can be addressed at Vitamin/Mineral Deficiencies
follow-up based on measured lipid levels. Endocrinologists seeing patients who have had
bariatric surgery should routinely ask if the patient
Anastomotic Leak is taking vitamin and mineral supplements and, if
One of the most concerning complications of not, perform appropriate screening for signs and
bariatric surgery is an anastomotic leak. This symptoms of vitamin deficiencies (Table 2) and
problem typically presents 3 to 4 days following obtain scheduled testing to measure serum levels
surgery. The complication is more common of vitamins and minerals (Table 3).7
following RYGB but can be seen after VSG.
Affected patients present with shoulder or
abdominal pain, fever, and shortness of breath and
have tachycardia and tachypnea on examination.

28  ENDO 2021 • Endocrine Case Management

chased by Dr. Khalid H. Seedahmed, Jr., MRCP PGDip, CertEndocrinology SA - ID 283


Table 3. Suggested Schedule for Testing to Monitor for Nutritional Deficiencies in Patients Following Bariatric Surgery

3 Months 6 Months
Deficiency Screening Test Preoperatively Postoperatively Postoperatively Annually

Vitamin B1 Whole blood thiamine  Any time Any time Any time
nausea/vomiting nausea/vomiting nausea/vomiting

Vitamin B12 Serum B12 (methylmalonic acid,  RYGB, SG, BPD/ RYGB, SG, BPD/ 
homocysteine) DS DS

Folate Folate (red blood cell folate, homocysteine,    


methylmalonic acid)

Vitamin A Plasma retinol (retinol-binding protein)  BPD/DS 

Vitamin D 25-Hydroxyvitamin D    

Vitamin E/K Plasma α-tocopherol/plasma phylloquinone  


(PT-prothrombin time)

Zinc Plasma or serum zinc (red blood cell zinc,  RYGB, SG,
urine zinc)
BPD/DS

Copper Serum copper (ceruloplasmin, erythrocyte  RYGB, SG,


superoxide dismutase)
BPD/DS

Iron Ferritin (serum iron, transferrin saturation,    


transferrin and iron-binding capacity,
complete blood cell count)

PTH Serum PTH, serum calcium    

Calcium Combination: 25-hydroxyvitamin D, alkaline    


phosphatase, serum phosphate, 24-hour
urinary calcium

Vitamin B12 Iron


Because vitamin B12 comes primarily from meat Iron comes from meat and absorption is facilitated
(often in the postoperative period patients are by gastric acid. Alterations in gastrointestinal
not able to eat much meat) and requires intrinsic anatomy can further disrupt absorption. For these
factor that is secreted by the stomach, deficiency reasons, iron deficiency is a common complication
can develop following VSG or RYGB. Because of bariatric surgery, especially in women of
body stores are high, it may take a year or more reproductive age due to blood loss during
for patients to become deficient. Symptoms menstruation. The prevalence of iron deficiency
include macrocytic anemia, glossitis, and a has been reported to be 20% to 50% following
variety of neurologic disorders that may not be surgery. It is important to screen for and treat
reversible if not treated early. For these reasons, all iron deficiency preoperatively and to continually
patients who have had VSG, RYGB, or BPD/DS assess patients postoperatively. Because systemic
should take vitamin B12 supplements for life. inflammation can alter iron absorption, screening
Simply testing serum vitamin B12 levels may for iron deficiency is best done with a combination
not be sufficiently sensitive to detect deficiency. of measures, including C-reactive protein, ferritin,
Methylmalonic acid levels are a more sensitive and transferrin saturation. Treatment can start
measure of vitamin B12 deficiency. Options with oral iron supplements in dosages of 150 to
for supplementation include 300 to 500 mcg 200 mg daily, increasing to 300 mg 2 to 3 times
oral/sublingual tablet/liquid preparations daily if needed. Iron should be given in divided
daily, 1000 mcg intramuscularly monthly, or an doses separately from calcium supplements, and
intranasal preparation. the addition of ascorbic acid can help facilitate

ENDO 2021 • Adipose Tissue, Appetite, Obesity, and Lipid Metabolism  29

chased by Dr. Khalid H. Seedahmed, Jr., MRCP PGDip, CertEndocrinology SA - ID 283


absorption. Some patients require parenteral underwent bariatric procedures found that the
supplementation. weight-loss nadir occurred at a median of 2 years
following surgery. They looked for a cut-point
Vitamin D that clinicians could use to determine if there had
It is important to screen for vitamin D deficiency been significant weight regain as defined by an
preoperatively and treat if deficiency is identified. increased risk for the recurrence of weight-related
It is much easier to treat vitamin D deficiency comorbidities. The value that they determined was
before surgery than after. All patients who have return of 20% of the lost weight. For example, if
had bariatric surgery should consume 3000 IU of a person lost 100 lb, then a regain of 20 lb would
vitamin D daily to maintain 25-hydroxyvitamin be considered significant. In clinical practice,
D levels greater than 30 ng/mL (>85.9 nmol/L). however, insufficient weight loss or significant
Higher dosages may be needed to keep PTH levels weight regain is typically defined by the patient.
in the normal range. Some patients require as When seeing a patient with significant
much as 50,000 units 1 to 3 times per week. postbariatric surgery weight regain, the first
question is whether there is a mechanical problem
Calcium with the original surgery. This would be more
Many patients consume very little dairy following likely if the original procedure was an AGB or
bariatric surgery. Combined with the common if the person had a sustained weight loss for
problem of vitamin D deficiency and risk for many years followed by a relatively abrupt return
metabolic bone disease following weight loss, of appetite and increase in weight. An upper
it is important for patients to strive to consume gastrointestinal study and upper endoscopy
1200 to 1500 mg calcium daily following surgery. would be the best next step in a patient such as
Those who have had BPD/DS should consume this, with a referral to a gastroenterologist or
1800 to 2400 mg daily. Because calcium carbonate surgeon for consideration of a repair or revision.
is not well absorbed in the absence of stomach Also, it is important to ask about changes in diet,
acid, calcium citrate is the preferred form for physical activity habits, and new medications
supplementation. that could be associated with weight gain.
Referral to a registered dietician nutritionist or
Copper; Zinc; and Vitamins A, E, K exercise physiologist may be helpful. Antiobesity
Tables 2 and 3 describe the signs and symptoms of medications can be used both preoperatively and
deficiencies of these vitamins and micronutrients, postoperatively to facilitate weight loss. There
as well an approach to the diagnostic testing for are very few studies that have looked at the use of
each. The bariatric surgery guidelines from the antiobesity medications in patients who have had
American Association of Clinical Endocrinologists bariatric surgery, but in general, the data show
cited in the references provide detailed guidance that these medications perform much like they do
on these conditions.5,8 in patients who have not had surgery. One might
wonder if GLP-1 receptor agonists would work
Weight Regain Following in patients who had bariatric surgery since GLP-1
Weight-Loss Surgery levels rise following surgery. Studies done to date
Endocrinologists may encounter patients who suggest GLP-1 receptor agonist medications work
have had bariatric surgery who either did not just as well in patients following bariatric surgery
lose much weight or have regained a substantial as they do in nonsurgically treated patients.
amount of weight years after having had bariatric
surgery. Given the tremendous variability in
weight loss and regain, one might ask what
degree of regain would prompt evaluation and
treatment. One study of 2458 individuals who

30  ENDO 2021 • Endocrine Case Management

chased by Dr. Khalid H. Seedahmed, Jr., MRCP PGDip, CertEndocrinology SA - ID 283


Post–Bariatric Surgery Hypoglycemia Clinical Case Vignettes
Post–bariatric surgery hypoglycemia is an
increasingly recognized problem following Case 1
bariatric surgery. It typically presents with A 42-year-old man that you follow for T2DM
symptoms of hypoglycemia 1 to 3 hours following and severe obesity decided to have RYGB. For
a meal associated with a serum glucose value less financial reasons, he had the procedure done in
than 54 mg/dL (<3.0 mmol/L) with symptoms Mexico. He did well and returned home 3 weeks
being relieved by the ingestion of carbohydrate. ago, 1 week after his surgery. Over the last
This problem typically presents 1 to 4 years week, he has developed progressive nausea with
after surgery, most commonly following RYGB. vomiting and an inability to keep any solid food
An oral glucose tolerance test is not a good down over the last few days. Since yesterday, his
diagnostic test, as many postsurgical patients family has noticed experienced confusion. Today,
will have abnormal values on this test and not he was unsteady on his feet and fell. He comes to
have post–bariatric surgery hypoglycemia. the hospital and is found to be ataxic and confused,
A mixed-meal test is preferable, although simply and he has abnormal extraocular movements.
having patients check their glucose levels with
symptoms or use a continuous glucose monitor A deficiency of which of the following
often provides sufficient information to feel vitamins/minerals is likely?
confident of the diagnosis. First-line therapy is A. Vitamin B12
frequent small mixed meals with low–glycemic B. Vitamin A
index carbohydrates that emphasize protein. If
C. Thiamine
symptoms persist, acarbose can be used. Early
studies advocated partial pancreatectomy, but this D. Copper
is almost never done now. If severe and disabling, Answer: C) Thiamine
the problem may be an indication for reversing the
surgery.9 This presentation is typical for Wernicke
encephalopathy from acute thiamine deficiency
Indications for Re-Operation (Answer C). This is a medical emergency, and the
Up to 14% of bariatric surgery operations person should be treated with parenteral thiamine
performed currently are revision operations. (200 mg 3 times daily for 3-5 days).
These are most commonly done for mechanical Vitamin B12 deficiency (Answer A) can
problems with an AGB such as band slippage, band cause neurologic symptoms, but because
erosion into the stomach, port or tubing problems, of the large storage capacity of the body,
or migration of the band over time. Another deficiency usually does not appear until a year
common indication for revision of VSG to RYGB or 2 following surgery. Vitamin A deficiency
is worsening or intractable gastroesophageal reflux (Answer B) causes eye problems, including night
disease. Indications for revision of VSG to RYGB blindness and patches of hyperkeratinization
also include staple line leaks, weight regain in the of the conjunctiva called Bitot spots. However,
presence of pouch or anastomoses dilation, or deficiency of vitamin A typically does not occur
gastric stenosis leading to persistent nausea and quickly after surgery, again because of the fat
vomiting. Revisions to RYGB can be done either stores of the vitamin. Vitamin A deficiency,
endoscopically or surgically. The data on weight when present with corneal involvement,
loss following revisions done for weight regain are should be treated with 50,000 to 100,000 IU
mixed. Complications are increased with second administered intramuscularly daily for 3 days.
operations, and these procedures should be done Copper deficiency (Answer D) is uncommon
by experienced, high-volume surgeons. and occurs most frequently after BPD/DS. Signs

ENDO 2021 • Adipose Tissue, Appetite, Obesity, and Lipid Metabolism  31

chased by Dr. Khalid H. Seedahmed, Jr., MRCP PGDip, CertEndocrinology SA - ID 283


and symptoms of copper deficiency include bariatric surgical procedures in the past, and most
anemia, neutropenia, pancytopenia, neuropathy, of these patients are no longer seeing a surgeon.
paresthesia, spasticity, hypopigmentation of the It therefore may fall on the endocrinologist to
hair, and hypercholesterolemia. Treatment for identify a mechanical complication of one of
mild deficiency is 3 to 8 mg oral copper daily until these procedures. AGB is not used much anymore
levels normalize or, when neurologic symptoms because it is a mechanical device that can fail over
are present, 2 to 4 mg intravenously daily for 6 time. This presentation is typical for erosion of
days or until neurologic symptoms resolve. The the band (Answer B) through the wall and into the
patient’s nausea can be managed with antiemetic lumen of the stomach. Stomach contents then can
agents such as ondansetron, 4 mg orally, and track up the inflation cannula causing skin redness
hydration. If it does not resolve, consultation in the area of the port.
with a gastroenterologist to determine whether Bowel obstruction (Answer A) can be a
imaging studies or endoscopy might be indicated is complication of AGB, but it would present as not
warranted. just nausea but vomiting and would not have the
signs of inflammation at the port site. There are
Case 2 no surgical anastomoses or staple lines in AGB. An
anastomotic leak (Answer D) is an early surgical
You have followed a 48-year-old woman with complication occurring within days or weeks
T2DM for 7 years. Before she was in your care, of the initial surgery presenting as shoulder/
she had an AGB procedure. It initially produced abdominal pain, tachycardia, shortness of breath,
some weight loss, but she stopped seeing her and fever. It is a surgical emergency. A staple line
surgeon and has regained most of the weight dehiscence (Answer C) is a complication of an
that she previously lost. She recently developed old-fashioned open gastric bypass where, instead
abdominal pain and some nausea. During her of forming a pouch that is separate from the
clinic appointment, you note some redness and remaining stomach, the surgeon simply made a
tenderness at the site of her port. gastric pouch by stapling across the upper part of
the stomach. Over time, this staple line can break
Which of the following is the down, thereby undoing the surgery. This is a cause
most likely diagnosis? for late weight gain after an open RYGB.
A. Bowel obstruction The key point is that when you are seeing a
B. Band erosion patient who has had a bariatric surgical procedure
C. Stable line dehiscence in the past and now has unexplained weight
D. Anastomotic leak regain, abdominal pain, nausea, or signs of an
intraabdominal infection, think anatomically and
Answer: B) Band erosion get an upper gastrointestinal series, abdominal
CT, or consultation from a gastroenterologist who
Endocrinologists are not surgeons and we often has experience with patients who have undergone
do not think “anatomically” as a surgeon might. bariatric surgery.
And yet, we are seeing patients who have had

References
1. Bray GA, Heisel WE, Afshin A, et al. The science of obesity management: 3. Sjöström L. Bariatric surgery and reduction in morbidity and mortality:
an Endocrine Society scientific statement. Endocr Rev. 2018;39(2):79-132. experiences from the SOS study. Int J Obes (Lond). 2008;32(Suppl 7):S93-S97.
PMID: 29518206 PMID: 19136998
2. Arterburn DE, Telem DA, Kushner RF, Courcoulas AP. Benefits and 4. Arterburn DE, Olsen MK, Smith VA, et al. Association between bariatric
risks of bariatric surgery in adults: a review. JAMA. 2020;324(9):879-887. surgery and long-term survival. JAMA. 2015;313(1):62-70. PMID: 25562267
PMID: 32870301

32  ENDO 2021 • Endocrine Case Management

chased by Dr. Khalid H. Seedahmed, Jr., MRCP PGDip, CertEndocrinology SA - ID 283


5. Mechanick JI, Kushner RF, Sugerman HJ, et al. American Association of 8. Mechanick JI, Apovian C, Brethauer S, et al. Clinical practice guidelines for
Clinical Endocrinologists, The Obesity Society, and American Society for the perioperative nutrition, metabolic, and nonsurgical support of patients
Metabolic & Bariatric Surgery Medical Guidelines for Clinical Practice for the undergoing bariatric procedures - 2019 update: cosponsored by American
perioperative nutritional, metabolic, and nonsurgical support of the bariatric Association of Clinical Endocrinologists/American College of Endocrinology,
surgery patient [published correction in Surg Obes Relat Dis. 2010;6(1):112]. The Obesity Society, American Society for Metabolic & Bariatric Surgery,
Surg Obes Relat Dis. 2008;4(Suppl 5):S109-S184. PMID: 18848315 Obesity Medicine Association, and American Society of Anesthesiologists -
6. Aasheim ET. Wernicke encephalopathy after bariatric surgery: a systematic executive summary. Endocr Pract. 2019;25(12):1346-1359. PMID: 31682518
review. Ann Surg. 2008;248(5):714-720. PMID: 18948797 9. Salehi M, Vella A, McLaughlin T, Patti ME. Hypoglycemia after gastric
7. Bal BS, Finelli FC, Shope TR, Koch TR. Nutritional deficiencies after bypass surgery: current concepts and controversies. J Clin Endocrinol Metab.
bariatric surgery. Nat Rev Endocrinol. 2012;8(9):544-556. PMID: 22525731 2018;103(8):2815-2826. PMID: 30101281

ENDO 2021 • Adipose Tissue, Appetite, Obesity, and Lipid Metabolism  33

chased by Dr. Khalid H. Seedahmed, Jr., MRCP PGDip, CertEndocrinology SA - ID 283


chased by Dr. Khalid H. Seedahmed, Jr., MRCP PGDip, CertEndocrinology SA - ID 283
ADRENAL

chased by Dr. Khalid H. Seedahmed, Jr., MRCP PGDip, CertEndocrinology SA - ID 283


Selection, Dosing, and
Monitoring of Steroid
Replacement in the Patient
With Adrenal Failure
Irina Bancos, MD. Division of Endocrinology, Metabolism, and Nutrition, Mayo Clinic,
Rochester, MN; E-mail: bancos.irina@mayo.edu

Richard Auchus, MD, PhD. Division of Metabolism, Endocrinology, and Diabetes, University
of Michigan, Ann Arbor, MI; E-mail: rauchus@med.umich.edu

Learning Objectives glucocorticoid type, dose, and frequency of


administration. Both clinical and biochemical
As a result of participating in this session, learners
parameters are helpful in monitoring of optimal
should be able to:
replacement therapy.
• Illustrate the optimal approach to
glucocorticoid, mineralocorticoid, and
androgen replacement therapy and monitoring Significance of the
through a case-based approach.
Clinical Problem
• Review types of glucocorticoid replacement
Adrenal insufficiency is an endocrine disorder
therapies based on potency, pharmacokinetics,
characterized by lack of cortisol.1 Primary adrenal
delivery, and impact on clinical and
insufficiency occurs when the adrenal cortex is
biochemical outcomes.
destroyed by an autoimmune attack (Addison
disease) or infiltrative disease (tuberculosis,
bilateral fungal infection, or bilateral metastases).2
Secondary adrenal insufficiency occurs in pituitary
Main Conclusions dysfunction leading to insufficient production of
Management of adrenal insufficiency includes corticotropin, such as in pituitary macroadenomas,
glucocorticoid replacement therapy (for all types hypophysitis, Sheehan syndrome, or use of certain
of adrenal insufficiency) and mineralocorticoid medications (glucocorticoids and opioids).3
replacement (for primary adrenal insufficiency Prompt recognition, diagnosis, and management
only). Androgen replacement therapy could be of adrenal insufficiency are key to avoid poor
considered in women with adrenal insufficiency. quality of life, adrenal crisis, and even death.4,5
Replacement therapy may need to be adjusted in Optimal management of adrenal insufficiency
special situations such as shift work, pregnancy, includes education on glucocorticoid and
insulin-dependent diabetes mellitus, prolonged mineralocorticoid therapy, sick day rules, and
intense exercise, and end-stage renal disease. instruction in injection and situations requiring an
Glucocorticoid potency and circadian delivery injectable steroid. Selecting a regimen that works
need to be considered when deciding on the

36  ENDO 2021 • Endocrine Case Management

chased by Dr. Khalid H. Seedahmed, Jr., MRCP PGDip, CertEndocrinology SA - ID 283


with a patient’s lifestyle is important to ensure dexamethasone’s high potency, even the smallest
adherence and optimal quality of life.1 dose tablet currently available is excessive for most
patients, providing little opportunity for dose
adjustment and high likelihood of overtreatment
Barriers to Optimal Practice with development of iatrogenic Cushing syndrome.
Monitoring of glucocorticoid replacement
• Poor specificity of symptoms and signs of
therapy includes history and physical examination
adrenal insufficiency.
(Table 2). Manifestations of cortisol excess include
• Lack of biomarkers to monitor glucocorticoid weight gain, fatigue, and development of features
replacement therapy. consistent with iatrogenic Cushing syndrome.
• Poor access to certain therapies for adrenal Patients on insufficient glucocorticoid replacement
insufficiency. therapy may complain of fatigue, anorexia or
nausea, and weight loss (Table 2).
Androgen replacement therapy has been
Strategies for Diagnosis, described in women with either primary or
secondary adrenal insufficiency with a starting
Therapy, and/or Management
dehydroepiandrosterone (DHEA) dosage of
Glucocorticoid Replacement Therapy 25 mg daily. The goal of therapy is improved
The goal of glucocorticoid replacement therapy is libido, mood, and energy, as well as DHEA-S
providing physiological replacement therapy that concentrations in mid-normal range (Table 2).6
mimics circadian rhythm as closely as possible. Several approaches to initiation of DHEA may be
Choice of glucocorticoid therapy, including type considered. One approach is to consider initiation
and frequency of administration, must be balanced of DHEA only in women experiencing low libido
against individual preferences and circumstances. and sexual dissatisfaction (symptoms most likely
Currently available glucocorticoid preparations to improve with DHEA). Another approach is to
demonstrate different glucocorticoid potencies and discuss DHEA with every woman who has adrenal
half-lives that affect both dosing and frequency of insufficiency after stabilization of glucocorticoid and
administration (Table 1). mineralocorticoid therapy. Emphasis must be placed
Hydrocortisone is the most commonly used on review of current evidence, which describes
glucocorticoid, with a starting dosage of 15 to small but statistically significant improvement in
25 mg daily in divided (2 to 3) doses. Prednisone quality of life, fatigue, mood, and libido in women
and prednisolone are used less frequently at 4 to treated with DHEA, as well as highlights that
6 mg daily, each morning. Dexamethasone should younger women may experience a more substantial
not be used in the treatment of adrenal insufficiency benefit than postmenopausal women.6
for several reasons. It has a long half-life, which Mineralocorticoid replacement therapy
does not allow for diurnal replacement. Given consists of fludrocortisone, 100 mcg daily (usually

Table 1. Glucocorticoid Type and Potency1

Drug Hydrocortisone Equivalent Number of Doses per Day Total Daily Dose

Hydrocortisone = cortisol 1 2 to 3 15 to 25 mg

Cortisone acetate = cortisone 1.6 2 30 to 40 mg

Prednisolone, methylprednisolone 0.2 1 5 mg

Prednisone 0.25 1 5 mg

ENDO 2021 • Adrenal  37

chased by Dr. Khalid H. Seedahmed, Jr., MRCP PGDip, CertEndocrinology SA - ID 283


Table 2. Monitoring Steroid Replacement Therapy

Replacement Usual Starting Laboratory Manifestations of Manifestations of


Therapy Dosage Clinical Monitoring Monitoring Excessive Therapy Suboptimal Therapy

Glucocorticoid Hydrocortisone, Clinical exam: weight, No bloodwork is Weakness Fatigue


replacement 15 to 25 mg energy level, sleep needed for monitoring
Weight gain Weight loss
therapy daily, in divided quality of therapy, but can
doses (2 to 3) measure post-dose Metabolic Nausea
History: use of
cortisol to confirm abnormalities
stress dose steroids, Hyperpigmentation
exposure
hospital admissions, Decrease in bone
Adrenal crisis
circumstances around density
adrenal crises

Mineralocorticoid Fludrocortisone, Clinical exam: blood Potassium, sodium, Hypertension Hypotension


replacement 100 mcg daily pressure, orthostatic renin, or renin plasma
Lower-extremity Dizziness
therapy vitals, lower extremities activity
edema
(for edema) Salt craving
Hypokalemia
Hyperkalemia
Low renin/renin
Elevated renin/renin
plasma activity
plasma activity

Androgen DHEA, 25 mg Clinical exam: hirsutism, DHEA-S and Signs of androgen Low DHEA-S
replacement daily acne, other signs of testosterone excess
Nonspecific: no
therapy (women) androgen excess concentrations in the
Elevated DHEA-S symptoms vs fatigue,
morning, before taking
low sex drive
DHEA supplement

50-300 mcg daily), and is required in 95% of Adrenal crisis usually occurs during a viral
patients with primary adrenal insufficiency. The illness, such as gastroenteritis or an upper
goal of therapy is to achieve absence of orthostatic respiratory infection.4 However, nonadherence,
hypotension and normal electrolytes and renin improper glucocorticoid management in the
plasma activity. Monitoring includes assessment of perioperative period, and severe psychological
orthostatic vitals during each visit, electrolytes, and crisis have also been reported to lead to adrenal
renin plasma activity at least yearly or 3 months crisis.4 As such, comprehensive education,
after each dosage adjustment (Table 2). In a patient instruction on injection, and provision of proper
with hypertension and optimal mineralocorticoid supplies are important interventions for adrenal
replacement therapy, effective antihypertensive crisis prevention (Table 4).
agents include calcium-channel blockers and
α-adrenergic blockers (Table 3).
Glucocorticoid and mineralocorticoid therapy
Clinical Case Vignettes
should be adjusted in certain circumstances. For Case 1
example, in shift workers, the larger dose of A 31-year-old woman with a history of primary
glucocorticoid should be taken on waking, and the autoimmune adrenal insufficiency diagnosed at age
second dose 6 to 8 hours later. In pregnancy, given 22 years presents for follow-up. She shares that she
that physiologically cortisol production increases, is 8 weeks pregnant. Her medical history includes
consider increasing the hydrocortisone dosage by vitiligo and autoimmune primary hypothyroidism.
about 20% in each trimester (Table 3). The decision She has no concerns and no new symptoms. Her
to increase the dosage should be made depending medical therapy consists of hydrocortisone, 15 mg
on the starting baseline dosage and the clinical on waking, and 5 mg at noon; fludrocortisone,
evaluation. Other special situations are listed in 100 mcg daily; levothyroxine, 112 mcg daily; and
Table 3. a multivitamin. Over the last 12 months, she had
to increase the hydrocortisone dosage for several

38  ENDO 2021 • Endocrine Case Management

chased by Dr. Khalid H. Seedahmed, Jr., MRCP PGDip, CertEndocrinology SA - ID 283


days during an upper respiratory illness. She is The patient is counseled on adrenal crisis
comfortable self-administering her injectable prevention and her comfort level and knowledge
glucocorticoid and has sufficient supplies. are confirmed regarding self-injection.
Laboratory workup reveals normal electrolytes,
normal thyroid function, and renin plasma activity Which of the following adjustments
at goal. would also be recommended?
A. No adjustments in either hydrocortisone or
fludrocortisone are needed throughout the
Table 3. Special Circumstances in
Steroid Replacement Therapy1 pregnancy
B. Double the hydrocortisone dosage during third
Shift worker Change glucocorticoid and mineralocorticoid
replacement according to schedule (take the trimester, continue the same fludrocortisone
larger dose of hydrocortisone on waking, the dosage
second dose 6 to 8 hours after the first one)
C. Increase the hydrocortisone dosage to 25 mg
Pregnancy Glucocorticoids
daily in the second trimester and to 30 mg
• 2nd  3rd trimester: increase in cortisol-
binding globulin, total cortisol, and free daily in the third trimester; monitor the need
cortisol for fludrocortisone adjustment
• Consider an increase in hydrocortisone
dosage (20%-40% of total daily dose)
D. Increase the hydrocortisone and
Mineralocorticoids
fludrocortisone dosages by 20% now
• Consider change in mineralocorticoid Answer: C) Increase the hydrocortisone to
dosage in the last trimester, as progesterone
is anti-mineralocorticoid 25 mg daily in the second trimester and to
• Monitor with sodium and potassium 30 mg daily in the third trimester; monitor
(plasma renin is physiologically increased in the need for fludrocortisone adjustment
pregnancy and thus not useful)

Prolonged Small carbohydrate meal before exercising Cortisol production increases during pregnancy,
intense possibly due to increased placental production of
Additional salt requirement to replace sweat
exercise
losses, consider small dose increases in corticotropin-releasing hormone, which results in
glucocorticoid and mineralocorticoids when
undertaking extraordinarily strenuous exercise hypertrophy of the adrenal cortex and increased
(eg, consider 2.5 to 5 mg hydrocortisone every cortisol. To mimic this physiological increase
3 hours during a marathon run or triathlon)
in cortisol, the typical practice is to increase the
Hot Additional salt requirement to replace sweat
environment losses
Consider increasing fludrocortisone by 50 to
100 mcg daily to compensate for salt and Table 4. Adrenal Crisis Prevention
water loss
Identification Steroid emergency card AND/OR medical
Travel No routine increase, but be prepared for stress alert bracelet or necklace
dosing and emergency
Education Sick day rule 1: double daily glucocorticoid
Insulin- Consider longer-acting glucocorticoids (eg, dose in times of illness until well for 1 day
dependent prednisolone) to avoid cortisol-driven peaks
Sick day rule 2: inject glucocorticoid in times of
diabetes and troughs in blood glucose
inability to take oral glucocorticoid or severe
mellitus
illness AND proceed to emergency care
Dialysis Stop fludrocortisone (no longer needed) Special attention: explain reasons for dosage
Plan glucocorticoid replacement (substantial adjustment, discuss the situations needing
amounts of the morning glucocorticoid dose adjustment, teach self-administration of
might be lost through hemodialysis) injectable glucocorticoid

Hypertension Consider decreasing fludrocortisone Supplies Sufficient supply of glucocorticoid/


mineralocorticoid
Add calcium-channel blockers
Emergency injection kits/vials/syringes/needles
Note: ACE inhibitors and angiotensin-receptor
blockers are less effective Follow-up Reinforce the education during each visit

ENDO 2021 • Adrenal  39

chased by Dr. Khalid H. Seedahmed, Jr., MRCP PGDip, CertEndocrinology SA - ID 283


hydrocortisone dosage by 20% in second and Answer: E) Trial of DHEA supplementation
third trimesters (Answer C).7 The increased AND move the bedtime hydrocortisone
progesterone concentration during pregnancy dose to the early afternoon
has an antimineralocorticoid effect, and thus,
adjustment of fludrocortisone is occasionally This patient could benefit from a change in
needed in the third trimester.7 There is no clinical glucocorticoid replacement therapy to mimic
indication to increase her hydrocortisone dosage at circadian rhythm. Adding another 5 mg of
this time in early pregnancy (Answer D). Making hydrocortisone at noon (Answer B) may
no adjustments (Answer A) is unlikely to be improve her afternoon fatigue, but it would
correct and is premature to conclude. Doubling not help her insomnia and would increase
the hydrocortisone dosage in the third trimester her total glucocorticoid replacement leading
(Answer B) is an excessive increase. to potential signs and symptoms of cortisol
excess. Moving the bedtime hydrocortisone
dose to the early afternoon (Answer D) is most
Case 2 likely to both improve insomnia and help with
A 23-year-old woman with a history of primary afternoon fatigue.
autoimmune adrenal insufficiency diagnosed 2 DHEA replacement therapy (Answer C) in
years ago presents for evaluation of insomnia, women with adrenal insufficiency (especially
afternoon fatigue, and decreased libido. She has younger women)1,6 has been reported to
no other symptoms or signs of glucocorticoid potentially help with libido and could be
or mineralocorticoid underreplacement or tried for at least 6 months (usual dosage of
overreplacement. Her menstrual cycle is normal. 25 mg daily, with a goal of mid-normal DHEA-S
Her weight is stable, and she has no orthostasis. concentrations).
She is currently taking hydrocortisone, Referring the patient to sleep medicine
15 mg on waking and 5 mg at bedtime. She (Answer A) would be premature until the timing
takes fludrocortisone, 50 mcg each morning. of the bedtime hydrocortisone dose, which is the
On physical examination, she has normal sitting likely cause, is moved earlier.
and standing blood pressure. Her laboratory
workup demonstrates normal electrolytes, renin Case 3
plasma activity, and TSH. There is no evidence of
pernicious anemia or celiac disease. A 40-year-old man is status post bilateral
adrenalectomy for incurable pituitary Cushing
Which of the following changes in disease of 5 years’ duration following 2
therapy should be considered? unsuccessful transsphenoidal surgeries. He
also received pituitary radiation and is now
A. No adjustments in current therapy are needed;
panhypopituitary. He takes levothyroxine,
refer the patient to the sleep medicine team for
testosterone, and growth hormone in addition to
further evaluation
hydrocortisone, 15 mg on rising plus 5 mg after
B. Add another 5 mg of hydrocortisone at noon lunch, and fludrocortisone, 200 mcg at bedtime.
and reevaluate in 3 months His physical examination findings and laboratory
C. Trial of DHEA supplementation data confirm adequate adrenal replacement, but
D. Move the bedtime hydrocortisone dose to the he has concerns of chronic fatigue and mental
early afternoon clouding. He heard on an Internet chat group
E. Both C and D that he should try subcutaneous hydrocortisone
delivered by insulin infusion pump, and he would
like a prescription for this treatment.

40  ENDO 2021 • Endocrine Case Management

chased by Dr. Khalid H. Seedahmed, Jr., MRCP PGDip, CertEndocrinology SA - ID 283


Which of the following should be conveyed insufficiency and ordered testing to investigate this
to this patient regarding this treatment? diagnosis further. The patient was informed that
A. Subcutaneous hydrocortisone is superior to results of his cosyntropin-stimulation test were
oral hydrocortisone for quality of life normal; however, he presents for a second opinion.
B. Hydrocortisone sodium succinate is not FDA He denies history of exogenous glucocorticoid
approved for subcutaneous infusion use. Brain MRI performed 12 months earlier for
evaluation of headaches did not reveal pituitary
C. Cortisol exposure from subcutaneous
abnormalities. Physical examination findings are
hydrocortisone infusion is milligram-for-
unrevealing. His only medication is tramadol,
milligram identical to oral hydrocortisone
100 mg 4 times daily, which he has been taking
D. The data regarding patient satisfaction with consistently for the last 5 years.
subcutaneous hydrocortisone are mixed
E. Both B and D Laboratory test results:
Baseline cortisol (morning) = 3 µg/dL (7-24 µg/dL)
Answer: E) Hydrocortisone sodium succinate is (SI: 82.8 nmol/L [193.1-662.1 nmol/L])
not FDA approved for subcutaneous infusion Peak cortisol after 250 mcg cosyntropin = 18 µg/dL
AND the data regarding patient satisfaction (≥18 µg/dL) (SI: 496.6 nmol/L [≥496.6 nmol/L])
with subcutaneous hydrocortisone are mixed ACTH = 9 pg/mL (7-63 pg/mL) (SI: 2.0 pmol/L
[1.5-13.9 pmol/L])
Limited data on continuous subcutaneous TSH = 2.1 mIU/L (0.3-4.2 mIU/L)
hydrocortisone in patients with adrenal Free T4 = 1.2 ng/dL (0.9-1.7 ng/dL) (SI: 15.4 pmol/L
[11.6-21.9 pmol/L])
insufficiency showed some improvement in the DHEA-S = 21 µg/dL (57-522 µg/dL) (SI: 0.6 µmol/L
vitality domain on quality-of-life scores, but [1.5-14.1 µmol/L])
not sleep in one study.8 No differences were
observed in another small study (thus, Answer
A is incorrect).9 Problems related to pump Which of the following is the best next
gear can occur.8 Overall patient satisfaction step in this patient’s management?
with subcutaneous hydrocortisone was mixed A. Perform 1-mcg cosyntropin-stimulation test
(Answer D) in a study of 10 patients when B. Advise the patient that adrenal insufficiency
5 preferred subcutaneous hydrocortisone, was excluded
4 preferred oral hydrocortisone, and 1 was C. Repeat pituitary MRI
uncertain.9 Hydrocortisone is not FDA approved
D. Test for central hypogonadism
for subcutaneous infusion (Answer B), and
evidence on long-term improvement in quality Answer: D) Test for central hypogonadism
of life in patients with adrenal insufficiency is
scarce. The bioavailability of hydrocortisone is This patient has low cortisol, DHEA-S, and
usually high but incomplete and more prone to ACTH, concerning for secondary adrenal
first-pass metabolism via CYP3A4 and drug-drug insufficiency despite the normal response to
interactions than parenteral administration (thus, cosyntropin (thus, Answer B is incorrect). While
Answer C is incorrect). the result of the cosyntropin-stimulation test was
normal, it is important to remember suboptimal
sensitivity of cosyntropin-stimulation testing in
Case 4  New Content 
diagnosing secondary adrenal insufficiency (high
A 35-year-old man with history of chronic back number of false-negative results).10 Performing a
pain and headaches presents for evaluation 1-mcg cosyntropin-stimulation test (Answer A)
of fatigue, nausea, and depressive symptoms. in this case is unlikely to be more informative,
His primary care physician suspected adrenal as it performs similarly to the 250-mcg

ENDO 2021 • Adrenal  41

chased by Dr. Khalid H. Seedahmed, Jr., MRCP PGDip, CertEndocrinology SA - ID 283


cosyntropin-stimulation test in patients suspected glucocorticoid since the diagnosis of adrenal
to have adrenal insufficiency.10 Another MRI insufficiency.
(Answer C) is not necessary, as it was normal 12
months earlier and is unlikely to have changed. Which of the following is the correct
Opioid-induced adrenal insufficiency should be advice to give this patient?
considered in this patient due to long-term use A. She meets the Centers for Disease Control
of tramadol and the absence of other common definition of immunosuppressed because of
etiologies of secondary adrenal insufficiency.3,11 the hydrocortisone dosage
Opioid-induced adrenal insufficiency is an B. If she is concerned about exposure, she should
underrecognized condition.12 In a recent study start stress dosing now
of patients taking long-term opioid therapy,
C. She should review sick-day rules and
prevalence of opioid-induced adrenal insufficiency
ensure that she has access to an ample
was 9%, and it was associated with higher
supply of hydrocortisone and an alternative
cumulative opioid exposure.13 Notably, opioid-
glucocorticoid
induced adrenal insufficiency can develop when a
patient is taking a morphine milligram equivalent D. If she is admitted to the hospital for
(MME) dosage as low as 20 mg.13 This patient COVID-19 with respiratory distress, she
is taking a total daily tramadol dose of 400 mg, should refuse to take dexamethasone
which equals 40 MME mg. He should be tested Answer: C) She should review sick-day rules and
for central hypogonadism (Answer D), as it is the ensure that she has access to an ample supply of
most common opioid-induced endocrinopathy, hydrocortisone and an alternative glucocorticoid
seen in 63% of patients.14
Patients with adrenal insufficiency are at
increased risk of infections,15-18 possibly due
Case 5  New Content 
to decreased natural killer cell cytotoxicity, an
A 40-year-old woman with a 7-year history important component of the immune system
of autoimmune primary adrenal insufficiency responsible for fighting against viral infections.19
contacts her endocrinologist because she is Infection has been reported to be the most
concerned about her risk during the SARS- common reason for developing an adrenal
CoV-2 pandemic. She works as an elementary crisis.4 Respiratory infections in particular have
school teacher and is inquiring whether it is safe been reported to contribute to the increased
for her to return to work. She is concerned that hospitalizations and mortality in patients with
her diagnosis of primary adrenal insufficiency adrenal insufficiency.15,16,20 As such, patients with
places her at higher risk for a severe infection. adrenal insufficiency might be at increased risk of
In addition, she is wondering whether any SARS-CoV-2 infection, although no data currently
modifications to her glucocorticoid regimen are support this concern. The Centers for Disease
needed should she contract COVID-19. Control definition of “immunosuppressed,” however,
She is otherwise doing well regarding her is taking at least 20 mg daily of prednisone or
adrenal insufficiency. At her last appointment, equivalent (80-100 mg of hydrocortisone), so
her physical examination did not have any this patient does not meet that definition (thus,
features suggestive of overreplacement Answer A is incorrect). Nevertheless, patients with
or underreplacement with glucocorticoid adrenal insufficiency who become infected with
or mineralocorticoid therapy. She takes COVID-19 and develop a severe disease course are
hydrocortisone, 15 mg at 5:30 AM and 5 mg at risk for adrenal crisis. Furthermore, shortages
at 1 PM, plus fludrocortisone, 0.1 mg in the of hydrocortisone have been reported in several
morning. She has not had to use any injectable countries due to supply chain interruptions. She

42  ENDO 2021 • Endocrine Case Management

chased by Dr. Khalid H. Seedahmed, Jr., MRCP PGDip, CertEndocrinology SA - ID 283


should clearly understand her sick-day rules and up to 50 mg every 6 hours of hydrocortisone.21
have plenty of hydrocortisone plus a prescription Several studies have demonstrated a clinical benefit
for an alternative (methylprednisolone) of dexamethasone, 6 mg daily, in patients with
(Answer C) if her pharmacy cannot obtain COVID-19 and respiratory compromise because of
hydrocortisone. its antiinflammatory properties. In patients with
Management of patients with adrenal adrenal insufficiency, ongoing hydrocortisone
insufficiency during a pandemic should include therapy can generate unnecessary confusion about
assiduous adherence to protocols to prevent drug redundancy or futility, and the decision to
infection, including virtual, rather than in-person, use dexamethasone should be no different than
appointments when possible. Stress dosing should in patients without adrenal insufficiency (thus,
be commenced when the patient becomes ill Answer D is incorrect). If dexamethasone is
and should be proportionate to illness—just as not available, the National Institutes of Health
for any other sickness—but not based on worry recommend the following as alternatives:
or potential for contracting COVID-19 (thus, prednisone, 40 mg daily; methylprednisolone,
Answer B is incorrect). Optimal provision of oral 32 mg daily; or hydrocortisone, 160 mg daily.
and injectable glucocorticoid should never be These dosages of glucocorticoid more than
delayed or withheld from patients with adrenal adequately provide stress-dose coverage.
insufficiency, and especially, during a pandemic,

References
1. Bancos I, Hahner S, Tomlinson J, Arlt W. Diagnosis and management 12. Saeed ZI, Bancos I, Donegan D. Current knowledge and practices of heath
of adrenal insufficiency. Lancet Diabetes Endocrinol. 2015;3(3):216-226. care professionals on opioid-induced adrenal insufficiency. Endocr Pract.
PMID: 25098712 2019;25(10):1012-1021. PMID: 31170362
2. Herndon J, Nadeau AM, Davidge-Pitts CJ, Young WF, Bancos I. Primary 13. Li T, Cunningham JL, Gilliam WP, Loukianova L, Donegan DM, Bancos I.
adrenal insufficiency due to bilateral infiltrative disease. Endocrine. Prevalence of opioid-induced adrenal insufficiency in patients taking chronic
2018;62(3):721-728. PMID: 30178435 opioids. J Clin Endocrinol Metab. 2020;105(10):dgaa499. PMID: 32866966
3. Donegan D, Bancos I. Opioid-induced adrenal insufficiency. Mayo Clin Proc. 14. de Vries F, Bruin M, Lobatto DJ, et al. Opioids and their endocrine
2018;93(7):937-944. PMID: 29976376 effects: a systematic review and meta-analysis. J Clin Endocrinol Metab.
4. Hahner S, Loeffler M, Bleicken B, et al. Epidemiology of adrenal crisis in 2020;105(3):1020-1029. PMID: 31511863
chronic adrenal insufficiency: the need for new prevention strategies. Eur J 15. Bjornsdottir S, Sundstrom A, Ludvigsson JF, Blomqvist P, Kampe O, Bensing
Endocrinol. 2010;162(3):597-602. PMID: 19955259 S. Drug prescription patterns in patients with Addison’s disease: a Swedish
5. Hahner S, Spinnler C, Fassnacht M, et al. High incidence of adrenal crisis in population-based cohort study. J Clin Endocrinol Metab. 2013;98(5):2009-2018.
educated patients with chronic adrenal insufficiency: a prospective study. J PMID: 23543658
Clin Endocrinol Metab. 2015;100(2):407-416. PMID: 25419882 16. Smans LCCJ, Souverein PC, Leufkens HGM, Hoepelman AIM, Zelissen PMJ.
6. Alkatib AA, Cosma M, Elamin MB, et al. A systematic review and meta- Increased use of antimicrobial agents and hospital admission for infections in
analysis of randomized placebo-controlled trials of DHEA treatment effects patients with primary adrenal insufficiency: a cohort study. Eur J Endocrinol.
on quality of life in women with adrenal insufficiency. J Clin Endocrinol Metab. 2013;168(4):609-614. PMID: 23384710
2009;94(10):3676-3681. PMID: 19773400 17. Tresoldi AS, Sumilo D, Perrins M, et al. Increased infection risk in
7. Lebbe M, Arlt W. What is the best diagnostic and therapeutic management Addison’s disease and congenital adrenal hyperplasia. J Clin Endocrinol Metab.
strategy for an Addison patient during pregnancy? Clin Endocrinol (Oxf). 2020;105(2):418-429. PMID: 31532828
2013;78(4):497-502. PMID: 23153216 18. Stewart PM, Biller BMK, Marelli C, Gunnarsson C, Ryan MP, Johannsson
8. Oksnes M, Bjornsdottir S, Isaksson M, et al. Continuous subcutaneous G. Exploring inpatient hospitalizations and morbidity in patients with
hydrocortisone infusion versus oral hydrocortisone replacement for adrenal insufficiency. J Clin Endocrinol Metab. 2016;101(12):4843-4850.
treatment of addison’s disease: a randomized clinical trial. J Clin Endocrinol PMID: 27623069
Metab. 2014;99(5):1665-1674. PMID: 24517155 19. Bancos I, Hazeldine J, Chortis V, Hampson P, et al. Primary adrenal
9. Gagliardi L, Nenke MA, Thynne TR, et al. Continuous subcutaneous insufficiency is associated with impaired natural killer cell function: a
hydrocortisone infusion therapy in Addison’s disease: a randomized, placebo- potential link to increased mortality. Eur J Endocrinol. 2017;176(4):471-480.
controlled clinical trial. J Clin Endocrinol Metab. 2014;99(11):4149-4157. PMID: 28223394
PMID: 25127090 20. Chantzichristos D, Persson A, Eliasson B, et al. Mortality in patients with
10. Ospina NS, Al Nofal A, Bancos I, et al. ACTH stimulation tests for the diabetes mellitus and Addison’s disease: a nationwide, matched, observational
diagnosis of adrenal insufficiency: systematic review and meta-analysis. J Clin cohort study. Eur J Endocrinol. 2017;176(1):31-39. PMID: 27928991
Endocrinol Metab. 2016;101(2):427-434. PMID: 26649617 21. Arlt W, Baldeweg SE, Pearce SHS, Simpson HL. Endocrinology in the
11. Li T, Donegan D, Hooten MW, Bancos I. Clinical presentation and outcomes time of COVID-19: management of adrenal insufficiency. Eur J Endocrinol.
of opioid-induced adrenal insufficiency. Endocr Pract. 2020;26(11):1291-1297. 2020;183(1):G25-G32. PMID: 32379699
PMID: 33471659

ENDO 2021 • Adrenal  43

chased by Dr. Khalid H. Seedahmed, Jr., MRCP PGDip, CertEndocrinology SA - ID 283


Complexities in the
Diagnosis and Treatment
of Cushing Syndrome
Ashley B. Grossman, BA BSc, MD, PhD (Hon.causa), FRCP, FMedSci. Oxford Centre for
Diabetes, Endocrinology and Metabolism, Churchill Hospital, University of Oxford; Centre
for Endocrinology, Barts and the London School of Medicine, University of London, United
Kingdom; E-mail: Ashley.grossman@ocdem.ox.ac.uk

Learning Objectives abnormalities should be dealt with as a matter


of urgency. If necessary, serum cortisol levels
As a result of participating in this session, learners
should be lowered to normal safe concentrations.
should be able to:
The most appropriate long-term management
• Describe the problems and complexities is surgery. It cannot be overemphasized that the
in diagnosing Cushing syndrome and in management of these patients requires experience
localizing its source. and expertise of a dedicated clinical team, and the
investigation and management often benefit from
• Explain the various therapeutic approaches to
the input of a center of excellence.
the treatment of Cushing syndrome, including
surgery, radiotherapy, and medical therapy,
and describe their advantages, disadvantages,
and sequential use when required. Significance of the
Clinical Problem
Cushing syndrome is a rare condition, once
exogenous sources of Cushing syndrome have
Main Conclusions been excluded. However, it is almost always
Cushing syndrome is a rare but important eminently treatable, as long as certain basic rules
condition that can be problematic in both and considerations are accepted in both making
diagnosis and successful management. The original the diagnosis and identifying the source of the
diagnostic tests were devised for the most obvious problem. It is vital to assess any test procedures
cases, but the endocrinologist is increasingly faced in the light of a clinical suspicion of Cushing
with patients who have mild disease, which is syndrome, and then to proceed in that direction.
difficult to distinguish from a variety of common The diagnosis is based on the demonstration
clinical conditions. The optimal approach is to of an autonomous source of excess cortisol
(1) use a series of biochemical tests, ranging from production, most apparent and demonstrable
those with high sensitivity to those with increasing by loss of the normal circadian rhythmicity and
specificity, (2) determine whether the disorder failure to show adequate feedback regulation.
is ACTH-dependent, and (3) when appropriate, Thus, the usual investigative techniques are
locate the source of ACTH. Imaging is part of based on the recognition of an increase in the
this paradigm, but biochemical assessment is 24-hour production of cortisol (urinary free
paramount. Finally, in severe cases, the metabolic cortisol), a failure to show a fall in cortisol after

44  ENDO 2021 • Endocrine Case Management

chased by Dr. Khalid H. Seedahmed, Jr., MRCP PGDip, CertEndocrinology SA - ID 283


the corticosteroid dexamethasone is administered • Late-night salivary cortisol necessitates a well-
(dexamethasone-suppression test), and an absence validated salivary cortisol assay with extensive
of the usual decease in hypothalamic-pituitary- normal ranges.
adrenal axis activity at night (midnight cortisol • Low levels of plasma ACTH with adrenal
measurement, late-night salivary cortisol). autonomy may overlap with mild cases of
Unfortunately, all of the investigatory techniques Cushing disease.
have pitfalls and require careful interpretation.
Once the diagnosis of Cushing syndrome
is established, effective treatment depends on
Strategies for Diagnosis,
demonstration of the source. Around 80% of
cases are ACTH-dependent, which effectively Therapy, and/or Management
means either a pituitary corticotroph tumor While the classic clinical presentation of
(Cushing disease) or an ectopic source. Contrary Cushing syndrome is usually beyond doubt, the
to earlier publications, such ectopic sources may endocrinologist is increasingly faced with mild
not be grossly obvious. The classic ectopic ACTH cases where patients may present with obesity,
syndrome due to small cell lung carcinoma, depression, hypertension, or diabetes, problems
presenting in an older patient with severe widespread in the general population. Various
metabolic defects and an obvious malignancy, attempts have been made to screen at-risk
is now being increasingly replaced in practice populations, but it has generally been concluded
by small bronchial, thymic, pancreatic, or other that the rarity of Cushing syndrome precludes
neuroendocrine tumors occurring at almost any any special screening program, at least in most
age. In the presence of low or undetectable ACTH situations. However, it should be emphasized that
levels, an adrenal source should be sought, usually in Cushing syndrome the catabolic changes are
an adrenal adenoma or carcinoma, although there most useful in the differential diagnosis; these
are the very rare patients with bilateral adrenal include thinning of the skin with associated easy
hyperplasia (sometimes part of Carney complex) bruising, myopathy, osteoporosis, and, in children,
or bilateral macronodular hyperplasia. short stature and/or a fall in growth velocity.
Thus, the clinician will see many patients with a
Cushing-like presentation, and some may have
Barriers to Optimal Practice mildly abnormal biochemistry, so-called pseudo-
Cushing states, including patients with anorexia,
• Many urinary free cortisol assays are based on
depression, and obstructive sleep apnea (although
plasma assays and lack sufficient sensitivity for
this term of pseudo-Cushing is not generally
mild cases of Cushing syndrome.
recommended). Every type of investigation will
• Dexamethasone-suppression tests are varied have a significant false-positive and false-negative
in application, and dose regimens depend on rate. The approach and one’s interpretation of
the effective absorption and metabolism of test results depend on the a priori likelihood of
dexamethasone. the diagnosis.
• Loss of circadian rhythmicity of serum cortisol It is therefore essential to investigate the
requires an in-patient admission. diagnosis in stages, beginning with a test that
• MRI may fail to detect a corticotroph tumor is simple to use and has very high sensitivity.1
even with optimal imaging protocols; Then, if this test indicates possible Cushing
especially in older patients, incidental pituitary syndrome, more specific tests can be used.
lesions may be mistaken for a corticotroph Many clinicians start with the overnight
tumor. dexamethasone-suppression test, administering
1 mg of dexamethasone orally at midnight

ENDO 2021 • Adrenal  45

chased by Dr. Khalid H. Seedahmed, Jr., MRCP PGDip, CertEndocrinology SA - ID 283


followed by a 9-AM serum cortisol measurement. and sensitive assays (with a much lower limit of
A cortisol concentration less than 1.8 µg/dL “normal”), but it is probably best to reserve urinary
(<50 nmol/L) has a greater than 95% sensitivity free cortisol measurement for those patients
and approximately 80% specificity. With all whose other test results are indeterminate,
dexamethasone-suppression tests, it is important although it may be more useful in children.3
to be aware that false-positive results may occur The loss of circadian rhythmicity is a critical
with any form of poor absorption or the presence feature of Cushing syndrome, but it can also be
of drugs that increase dexamethasone catabolism, difficult to ascertain. For many years, a sleeping
such as rifampicin or phenytoin. midnight cortisol measurement was used, but it
With a clinical suspicion and some required hospitalization for at least 2 days and
biochemical evidence in favor of hypercortisolism, required the patient to be asleep, at least when
many clinicians move on to an extended low- the sample was beginning to be drawn. Again, a
dose 48-hour dexamethasone-suppression test, concentration less than 1.8 µg/dL (<50 nmol/L)
using dexamethasone, 0.5 mg given strictly at excludes Cushing syndrome, while a concentration
6-hour intervals for 48 hours. A 48-hour serum greater than 7.5 µg/dL (>208 nmol/L) renders
cortisol concentration less than 1.8 µg/dL the diagnosis highly probable. Unfortunately, the
(<50 nmol/L) excludes Cushing syndrome, with “grey” area (1.8-7.5 µg/dL [50-208 nmol/L]) is
approximately 95% probability and very high indeterminate. There is now increasing evidence
specificity. Alternative tests include measurement that a midnight salivary cortisol measurement
of 24-hour urinary free cortisol and late-night can also readily differentiate Cushing syndrome
salivary cortisol. from noncushingoid states, and the sample can
Urinary free cortisol has classically been be collected at home and then brought or mailed
the screening test most used, but it has many to the laboratory. With an appropriate assay and
drawbacks.2 The collection is complex and reliable normative values, this test approaches
sometimes difficult for patients, and incomplete greater than 95% sensitivity and specificity.
collections are not uncommon. Furthermore, it However, the normal ranges tend to be assay
relies on the fact that only 5% of serum cortisol specific, and there is some evidence for lesser
is excreted in the urine and, at least in the past, reliability in older patients, so experience in its use
most assays were simply based on assays for is important.
serum samples, which were then applied to Assuming that a firm clinical and biochemical
urine. In general, 3 to 4 completely normal urine diagnosis of Cushing syndrome has been made,
collections can (almost) always be interpreted to the next step is to identify a source: approximately
exclude Cushing syndrome, while a cortisol level 20% of patients have an adrenal source, and
greater than 4 times the upper normal limit is very this can be confirmed with a 9-AM plasma
confirmatory of Cushing syndrome. However, it ACTH measurement. It is important to collect
is possible, but not yet definitively shown, that the sample correctly, and in such a situation, a
when using the newer mass spectrometric assay, concentration less than 10 pg/mL (<2.2 pmol/L)
any elevated urinary free cortisol measurement indicates an adrenal source, a concentration
would be more suggestive of the diagnosis, and a greater than 30 pg/mL (>6.6 pmol/L) indicates
value more than 2 times the upper normal limit ACTH dependence. Two to three samples should
would be diagnostic. The difficulty in diagnosis be collected to ensure the reliability of the results.
remains the very large number of patients whose If the ACTH concentration is persistently in
values lie slightly above normal or fluctuate in the range of 10 to 30 pg/mL (2.2-6.6 pmol/L),
and out of the normal range. The introduction of then many clinicians perform a corticotropin-
more accurate assays, especially those based on releasing hormone test (100 mcg of human or
mass spectrometry, may allow for more specific ovine-sequence corticotropin-releasing hormone

46  ENDO 2021 • Endocrine Case Management

chased by Dr. Khalid H. Seedahmed, Jr., MRCP PGDip, CertEndocrinology SA - ID 283


intravenously, with sampling every 15 minutes petrosal sinus sampling, where an experienced
for 2 hours). A clear rise above 30 pg/mL interventional neuroradiologist inserts catheters
(>6.6 pmol/L) confirms ACTH-dependence. into the petrosal veins directly draining the
When the test suggests ACTH-independence, pituitary; corticotropin-releasing hormone (or
imaging of the adrenals with CT or occasionally desmopressin if corticotropin-releasing hormone
MRI is used to identify an adrenal tumor (benign is unavailable) is injected intravenously, and
or malignant), bilateral macronodular hyperplasia the gradient in ACTH between the central and
(bilaterally enlarged adrenal glands), or primary simultaneous peripheral venous samples is
pigmented nodular adrenal dysplasia (a disorder compared. A central-to-peripheral ACTH gradient
with often entirely normal or slightly nodular greater than 3 after corticotropin-releasing
adrenal glands, usually seen in young adults and hormone is 95% sensitive and almost 100% specific
frequently in the context of Carney complex). If for the diagnosis of Cushing disease. If the bilateral
corticotropin-releasing hormone is unavailable, inferior petrosal sinus sampling shows no central
then one could proceed directly to adrenal gradient, then the probability of Cushing disease
imaging, with the caveat that longstanding moves from a 90% to a 50% likelihood, and more
Cushing disease may lead to asymmetric detailed imaging should be pursued. Most ectopic
adrenal enlargement and a false diagnosis of an sources are in the neck or chest, and CT of the
adrenal tumor. neck, chest, and abdomen often reveals a bronchial
With ACTH-dependent Cushing syndrome, carcinoid (ie, lung neuroendocrine tumor, the
the differential diagnosis consists of a corticotroph most common source), a thymic neuroendocrine
tumor, Cushing disease, or an ectopic source. tumor, medullary thyroid carcinoma, a pancreatic
Generally, the higher the levels of ACTH and islet-cell tumor, or a pheochromocytoma (other
cortisol, the more severe the metabolic upset and sites have also been identified in large case series).
the presence of hypokalemia, and the more likely Radionuclide scanning with 68Ga-DOTATATE
there is an ectopic source, but no biochemical PET can be very useful,5 although tumors can
or clinical feature is pathognomonic.4 A young still be missed: overt sources are those revealed on
woman with a long history of symptoms and mild initial cross-sectional scanning, and covert sources
Cushing syndrome is much more likely to have are those that eventually are diagnosed with more
Cushing disease, while an older man with severe detailed imaging. Occult sources are those that defy
biochemistry is more likely to have an ectopic identification, even with the most sophisticated
source, but there is considerable overlap, and imaging techniques.
such features merely alter the a priori probability Not infrequently, the biochemical changes
of the cause. For this reason, many dynamic tests in Cushing syndrome are highly fluctuant, and
have been introduced to differentiate among these variability by more than 100% in 15% of patients
possibilities, but none has a major clinically useful with all causes of Cushing syndrome has been
discriminatory power. Many clinicians use the noted.6 This is—probably incorrectly—known as
responses to a low-dose 48-hour dexamethasone- periodic or cyclic Cushing syndrome. Rarely, there
suppression test (a fall in cortisol >20%) and the may indeed be a regular intermittency of cortisol
corticotropin-releasing hormone test (a rise in secretion. However, for Cushing syndrome to
cortisol >20%) to be very highly suggestive of truly “switch off” for prolonged periods, while
Cushing disease. In the presence of a clear pituitary recognized, is very rare. In such situations, in
lesion on MRI, this may be enough to mandate the face of uncertainty, it may be appropriate to
pituitary surgery. Equally, extreme biochemistry in wait and then reassess when the disease is in an
an older man with a mass lesion in the chest would active phase.
suggest thoracic surgery. However, in almost Finally, while in most patients a careful
all cases, it is best to consider bilateral inferior and analytic approach will lead to the correct

ENDO 2021 • Adrenal  47

chased by Dr. Khalid H. Seedahmed, Jr., MRCP PGDip, CertEndocrinology SA - ID 283


diagnosis and appropriate management, it must should be treated with intravenous etomidate,
be emphasized that the diagnosis of Cushing which rapidly and effectively lowers cortisol when
syndrome and the identification of its source administered with due care.9
remains essentially probabilistic and not In all but the most straightforward case,
algorithmic. In cases of ongoing uncertainty, patients should be treated in a center of excellence.
a pragmatic wait-and-see approach may be The diagnosis and treatment of Cushing syndrome
reasonable, while the lack of identification of a remains one of the most satisfying, as well as
source may require medical or surgical (bilateral frustrating, problems in endocrinology.
adrenalectomy) therapy to at least diminish the
harm from ongoing cortisol excess. In addition,
great attention should be given to the metabolic
Clinical Case Vignettes
complications, such as hypertension and diabetes Case 1
and the ever-present risk of sepsis. Many clinicians A 26-year-old man with a 5-year history of
advise anticoagulation with subcutaneous heparin significant symptoms and signs highly suggestive of
as soon as the diagnosis is confirmed. Cushing syndrome is referred for evaluation. Several
For confirmed Cushing disease, serum cortisol measurements have all been greater
transsphenoidal surgery by an experienced than 63 µg/dL (>1750 nmol/L), with plasma
surgeon should have a high cure rate, approaching ACTH levels of 300 to 350 pg/mL (66-77 pmol/L).
90%, while a repeat surgery when initially CT chest shows left lower collapse/consolidation.
unsuccessful can still lead to cure in around 50%
of patients.7,8 Failed surgery can be followed by Which of the following should
external beam radiotherapy if there is no obvious be the next investigation?
tumor, or radiosurgery or proton-beam therapy A. Low-dose dexamethasone-suppression test
when there is a discrete lesion, but normalization
B. Corticotropin-releasing hormone test
of serum cortisol levels may take several years
(faster in children, often <1 year). Medical therapy C. Bilateral inferior petrosal sinus sampling
with metyrapone, ketoconazole, or mifepristone D. Bronchoscopy
may be used while awaiting normalization. A E. 68Ga-DOTATATE PET scanning
minority of patients with Cushing disease respond
to cabergoline, while those with mild disease may Answer: D) Bronchoscopy
respond to the somatostatin analogue pasireotide,
The diagnosis of Cushing syndrome in this
although with a substantial risk of diabetes
patient is clear and does not require further
mellitus and long-term suppression of growth
confirmation. There are very high levels of both
hormone. Ectopic sources should be removed
cortisol and ACTH, so there is no need to confirm
where possible, or the patient should undergo
Cushing syndrome with a dexamethasone-
bilateral laparoscopic adrenalectomy when cure
suppression test (Answer A). The differential
is not possible, and this option should also be
diagnosis is an ectopic source or Cushing disease.
considered for occasional patients with residual or
The corticotropin-releasing hormone test
recurrent Cushing disease.
(Answer B) can sometime add useful information,
Adrenal tumors causing hypercortisolemia
but with such high levels of cortisol and ACTH,
should generally be removed laparoscopically, and
ectopic ACTH is more likely and responses to
patients with bilateral disease should be treated
corticotropin-releasing hormone will not shift
with bilateral adrenalectomy, although unilateral
this to any extent. The 68Ga-DOTATATE PET
adrenalectomy has been proposed in milder cases.
scan (Answer E) can be positive with any sort of
Patients with overwhelming metabolic
inflammation, so in this instance, finding positive
complications of Cushing syndrome of any cause
uptake in the lung would be unhelpful. Bilateral

48  ENDO 2021 • Endocrine Case Management

chased by Dr. Khalid H. Seedahmed, Jr., MRCP PGDip, CertEndocrinology SA - ID 283


inferior petrosal sinus sampling (Answer C) is biochemical data are indicative of an ectopic
often extremely useful, but as there is already source, and many healthy patients have minor
evidence for a lung abnormality with an abnormal pituitary abnormalities on MRI, such that pituitary
CT scan, bronchoscopy (Answer D) to look for a exploration (Answer E) would be premature.
bronchial tumor is the most useful and expeditious Bilateral inferior petrosal sinus sampling
investigation at this stage. (Answer A) should clarify whether the high levels
of ACTH are likely to be ectopic or pituitary in
origin.
Case 1 (Continued)
Indeed, bilateral inferior petrosal sinus
Bronchoscopy demonstrates a combined sampling in this patient confirmed a central
bacterial, fungal, and cytomegalovirus infection. source, mainly lateralized to the left side. During
This considerably improves with appropriate transsphenoidal surgery, a left-sided corticotroph
antibiotics, antiviral treatment, and antifungal tumor was identified and removed, and the patient
treatment. There is no abnormal cytology, and the was cured by surgery. Postoperative 9-AM serum
patient experiences complete resolution. Pituitary cortisol concentrations were less than 1.8 µg/dL
MRI shows slight enlargement of the right side of (<50 nmol/L).
the pituitary, while a 68Ga-DOTATATE PET scan
shows no abnormal uptake.
Case 2  New Content 
Which of the following should A 55-year-old woman is referred by her
be the next investigation? family practitioner because of weight gain and
A. Bilateral inferior petrosal sinus sampling hypertension. Her family physician suspects that
B. Thoracic surgery there is no major endocrine problem, but the
patient is keen to seek an endocrine opinion.
C. A combined dexamethasone–corticotropin-
Her medical history is unremarkable, going back
releasing hormone test
many years. There is no drug history of note, and
D. A high-dose dexamethasone-suppression test the clinical signs are compatible with, but not
E. Pituitary exploration diagnostic of, mild Cushing syndrome.
Answer: A) Bilateral inferior petrosal sinus sampling Which of the following should be
There is no obvious ectopic source, and further the first-line investigation?
dynamic tests are unlikely to be helpful at this A. Late-night salivary cortisol measurement
stage. The dexamethasone–corticotropin-releasing B. One urinary free cortisol measurement
hormone test (Answer C) is a cumbersome test C. Bilateral inferior petrosal sinus sampling
to diagnose Cushing syndrome and would not
D. Overnight dexamethasone-suppression test
add useful information, while the high-dose
dexamethasone-suppression test (Answer D) has E. 48-Hour low-dose dexamethasone-suppression
significant false-positive and false-negative results, test
plus the added danger of adding a further steroid Answer: A or D) Late-night salivary cortisol
load to an already compromised patient. The measurement OR overnight dexamethasone-
source remains uncertain, and there is no clear dexamethasone-suppression test
evidence for a thoracic origin, so chest surgery
(Answer B) is not indicated. Patients with Cushing In this case, the likelihood of Cushing syndrome
syndrome are at considerable risk of infections is low, so a screening test of high sensitivity
with a variety of organisms, as in this case, due and convenience is required. Both late-night
to profound immunosuppression. Much of the salivary cortisol measurement (Answer A) and

ENDO 2021 • Adrenal  49

chased by Dr. Khalid H. Seedahmed, Jr., MRCP PGDip, CertEndocrinology SA - ID 283


an overnight dexamethasone-suppression test to 30 pg/mL (2.2-6.6 pmol/L), a corticotropin-
(Answer D) would be reasonable, but the late- releasing hormone–stimulation test (Answer B)
night salivary cortisol measurement should can be helpful. In this case, the most rapid and
only be used when the local laboratory has a useful investigation would be CT of the adrenal
reliable assay and good normative values. In glands (Answer C) to look primarily for an adrenal
this case, the endocrinologist did not have this adenoma or carcinoma.
available, so the overnight dexamethasone-
suppression test was ordered. A single urinary
Case 2 (Continued)  New Content 
free cortisol measurement (Answer B) is of
limited sensitivity, is less convenient, and may CT of the adrenal glands shows 2 greatly enlarged,
be normal with low-grade Cushing syndrome. slightly nodular adrenals (maximal diameter of
The low-dose dexamethasone-suppression test 5 cm [left] and 3.5 cm [right]). A corticotropin-
(Answer E) has high sensitivity and specificity, releasing hormone–stimulation test is performed,
but it is more cumbersome and less useful as a which shows all plasma ACTH measurements to
first-line screening test. Bilateral inferior petrosal be less than 10 pg/mL (<2.2 pmol/L).
sinus sampling (Answer C) is only of value in
the differential diagnosis of ACTH-dependent Which of the following is the best
Cushing syndrome. treatment recommendation?
A. Unilateral adrenalectomy for the left-sided
lesion
Case 2 (Continued)  New Content 
B. Bilateral adrenalectomy
The patient’s serum cortisol concentration
C. Medical therapy with metyrapone
at 9 AM is 20 µg/dL (552 nmol/L), and
it is 18.5 µg/dL (510 nmol/L) after 1 mg D. Medical therapy with ketoconazole
dexamethasone given the preceding midnight. E. Medical therapy with mifepristone
A predexamethasone plasma ACTH concentration
Answer: A or B) Unilateral adrenalectomy for the
is 4 pg/mL (0.88 pmol/L).
left-sided lesion OR bilateral adrenalectomy
Which of the following would be the This patient clearly has bilateral macronodular
most useful next investigation? hyperplasia. Many such patients have aberrant
A. MRI of the pituitary gland tumor receptors for hormones such as gastric
B. Corticotropin-releasing hormone–stimulation inhibitory peptide or for the β-adrenoceptor.
test While of great theoretical interest, they are rarely
C. CT of the adrenal glands of long-term utility. Medical therapy requires
long-term treatment and frequent reassessments
D. CT of the thorax
and is not definitive. Metyrapone (Answer C) may
E. Octreotide scan also cause virilization. Mifepristone (Answer E)
Answer: C) CT of the adrenal glands is hard to monitor and is extremely expensive.
Ketoconazole (Answer D) is probably the most
Unless there was a problem with the collection or suitable, but it requires frequent monitoring of
assay of plasma ACTH, the very low ACTH level liver function.
indicates a primary adrenal pathology. Thus, MRI Many would advise bilateral adrenalectomy
of the pituitary gland (Answer A), octreotide scan (Answer B), with the caveat that this will require
(Answer E), and CT of the thorax (Answer D) lifelong adrenocortical replacement therapy.
to look for an ectopic source are unnecessary. Conversely, unilateral adrenalectomy (Answer A)
When ACTH levels are in the grey zone of 10 of the larger lesion may “normalize” cortisol levels

50  ENDO 2021 • Endocrine Case Management

chased by Dr. Khalid H. Seedahmed, Jr., MRCP PGDip, CertEndocrinology SA - ID 283


in many patients, but it is likely that the residual laparoscopically and required a 2-day hospital
adrenal will continue to grow and Cushing stay. On further questioning, she reported that she
syndrome will eventually recur.10 thought her sister had a similar condition. This
The patient decided, after discussion with was confirmed, and genetic analysis identified a
her endocrinologist, to undergo bilateral germline ARMC5 pathogenic variant in both the
adrenalectomy, which was performed patient and her sister.11

References
1. Nieman LK, Biller BM, Findling JW, et al. The diagnosis of Cushing’s 7. Nieman LK, Biller BM, Findling JW, et al; Endocrine Society. Treatment of
syndrome: an Endocrine Society clinical practice guideline. J Clin Endocrinol Cushing’s syndrome: an Endocrine Society clinical practice guideline. J Clin
Metab. 2008;93(5):1526-1540. PMID: 18334580 Endocrinol Metab. 2015;100(8):2807-2831. PMID: 26222757
2. Alexandraki KI, Grossman AB. Is urinary free cortisol of value in the 8. Pivonello R, De Leo M, Cozzolino A, Colao A. The treatment of Cushing’s
screening of patients with Cushing’s syndrome? Curr Opin Endocrinol Diabet. disease. Endocr Rev. 2015;36(4):385-486. PMID: 26067718
2011;18(4):259-263. PMID: 21681089 9. Preda V, Sen J, Karavitaki N, Grossman AB. Etomidate in the management
3. Shapiro L, Elahi S, Riddoch F, et al. Investigation for paediatric Cushing’s of hypercortisolaemia in Cushing’s syndrome [published correction appears
syndrome using twenty-four-hour urinary free cortisol determination. Horm in Eur J Endocrinol. 2013;168(2):X1]. Eur J Endocrinol. 2012;167(2):137-143.
Res Paediatr. 2016;86(1):21-26. PMID: 27287747 PMID: 22577107
4. Hayes RA, Grossman AB. The ectopic adrenocorticotropic hormone 10. Osswald A, Quinkler M, Di Dalmazi G, et al. Long-term outcome
syndrome: rarely easy, always challenging. Endocrinol Metab Clin North Am. of primary macronodular bilateral adrenocortical hyperplasia after
2018;47(2):409-425. PMID: 29754641 unilateral adrenalectomy. J Clin Endocrinol Metab. 2019;104(7):2985-2993.
5. Isidori AM, Sbardella E, Zatelli MC, et al; ABC Study Group. Conventional PMID: 30844071
and nuclear medicine imaging in ectopic Cushing’s syndrome: a systematic 11. Gagliardi L, Schreiber AW, Hahn CN, et al. ARMC5 mutations are common
review. J Clin Endocrinol Metab. 2015;100(9):3231-3244. PMID: 26158607 in familial bilateral macronodular adrenal hyperplasia. J Clin Endocrinol Metab.
6. Alexandraki KI, Kaltsas GA, Isidori AM, et al. The prevalence and 2014;99(9):E1784-E1792. PMID: 24905064
characteristic features of cyclicity and variability in Cushing’s disease. Eur J
Endocrinol. 2009;160(6):1011-1018. PMID: 19289537

ENDO 2021 • Adrenal  51

chased by Dr. Khalid H. Seedahmed, Jr., MRCP PGDip, CertEndocrinology SA - ID 283


Diagnosis and Management
of Endocrine Hypertension:
Deciding Who to Screen
for Genetic Causes
William F. Young, Jr., MD, MSc. Division of Endocrinology, Diabetes, Metabolism,
and Nutrition, Department of Internal Medicine, Mayo Clinic, Rochester, MN; E-mail:
young.william@mayo.edu

Learning Objectives disorders (Table).1 An accurate diagnosis of


endocrine hypertension provides clinicians with
As a result of participating in this session, learners
the opportunity to recommend a surgical cure
should be able to:
or treat with effective targeted pharmacologic
• Recognize the key signs, symptoms, therapy. During this Meet the Professor session,
biochemical testing, and adrenal imaging we will focus on the 2 main causes of adrenal-
findings that should raise suspicion for dependent hypertension: pheochromocytoma
pheochromocytoma. and primary aldosteronism. We will also include
commentary on genetic testing in patients who
• Consider the role for genetic testing in patients
have these disorders.
with pheochromocytoma.
• Recognize the key signs, symptoms, and
clinical circumstances that should trigger case-
detection testing for primary aldosteronism. Significance of the
• Consider the role for genetic testing in patients
Clinical Problem
with primary aldosteronism. Major problems related to the diagnosis of
adrenal-dependent hypertension:
• Clinicians think about and test for
pheochromocytoma a lot! However,
Main Conclusions because of the false-positive rate of
Hypertension affects approximately 25% of all plasma normetanephrine and the rarity of
adults. In most, hypertension is primary (essential pheochromocytoma, 97% of patients with
or idiopathic), but a subgroup of approximately elevated plasma normetanephrine do not have
15% of patients has secondary hypertension. a pheochromocytoma. This leads to additional
More than 50% of children and 30% of young downstream testing and, occasionally, surgical
adults (<40 years) have secondary hypertension. misadventures.
The secondary causes of hypertension include
• Even though primary aldosteronism affects
renal causes (eg, renal parenchymal disease) and
5% to 10% of all persons with hypertension,
endocrine causes. Hypertension may be the initial
clinicians do not think about it very much
clinical presentation for at least 15 endocrine

52  ENDO 2021 • Endocrine Case Management

chased by Dr. Khalid H. Seedahmed, Jr., MRCP PGDip, CertEndocrinology SA - ID 283


and do not test for it. The delayed or missed • Not recognizing that 3% to 5% of all
diagnosis of primary aldosteronism leads to adrenal incidentalomas prove to be
suboptimal quality of life and cardiac- and pheochromocytomas and that 60% of
renal-related morbidities. pheochromocytomas are discovered as
incidental adrenal masses on computed cross-
sectional imaging.
Barriers to Optimal Practice • Unawareness of the key role of imaging
Clinical practice barriers related to phenotype when determining whether an
pheochromocytoma: adrenal mass may be a pheochromocytoma
and the concept of “prebiochemical
• Lack of awareness of the medications pheochromocytoma.”
associated with false-positive biochemical
• Determining which patients with
testing.
pheochromocytoma or paraganglioma should
have genetic testing for germline pathogenic
Table. Endocrine Causes of Hypertension variants.
Etiology Clinical practice barriers related to primary
Adrenal-dependent causes aldosteronism:
1. Pheochromocytoma and sympathetic paraganglioma
2. Primary aldosteronism
• Lack of case-detection testing worldwide—
3. Hyperdeoxycorticosteronism
primary aldosteronism is one of the most
underdiagnosed endocrine disorders.
a. Congenital adrenal hyperplasia
i. 11β-Hydroxylase deficiency • Clinicians thinking that primary aldosteronism
ii. 17a-Hydroxylase deficiency is rare or difficult to diagnose.
b. Deoxycorticosterone-producing tumor • Confusion regarding which medications a
c. Primary cortisol resistance patient can or cannot be taking when case-
4. Cushing syndrome detection testing is performed for primary
Apparent mineralocorticoid excess/11β-hydroxysteroid aldosteronism.
dehydrogenase deficiency
1. Genetic
• Determining which patients with primary
2. Acquired
aldosteronism should have genetic testing for
a. Licorice or carbenoxolone ingestion
germline pathogenic variants.
b. Cushing syndrome
Parathyroid-dependent causes
1. Hyperparathyroidism Strategies for Diagnosis,
Pituitary-dependent causes Therapy, and/or Management
1. Acromegaly
Pheochromocytoma
2. Cushing syndrome
Secondary hyperaldosteronism Pheochromocytoma is a frequently sought, but
1. Renovascular hypertension
rarely found, catecholamine-secreting tumor
Thyroid-dependent causes
usually localized to the adrenal gland.2 When
1. Hypothyroidism
pheochromocytoma is correctly diagnosed and
2. Hyperthyroidism
properly treated, it is curable; when undiagnosed
Complex effects
or improperly treated, it can be fatal. The
1. Obstructive sleep apnea
prevalence of pheochromocytoma in the
hypertensive population is 0.01% to 0.1% and it

ENDO 2021 • Adrenal  53

chased by Dr. Khalid H. Seedahmed, Jr., MRCP PGDip, CertEndocrinology SA - ID 283


occurs equally in men and women—primarily in and catecholamines in a 24-hour urine collection.
the third through fifth decades of life. In 2020, If clinical suspicion is high, then plasma
a symptomatic presentation occurs in less than fractionated metanephrines are also measured.
40% of patients because the mode of diagnosis Although it is preferred that patients not be
has changed dramatically over the past 90 on any medications at the time of laboratory
years.3 In the most recent series of patients with testing, treatment with most medications may be
pheochromocytoma reported from Mayo Clinic continued (all blood pressure–related medications
(271 patients from 2005 to 2016), 61% were are OK!). Tricyclic antidepressants (TCAs)
detected as adrenal incidentalomas and 12% were interfere most frequently with the interpretation
detected because of genetic or family testing.3 of 24-hour urinary fractionated catecholamines
Whereas in the initial Mayo Clinic series (138 and metanephrines. Cyclobenzaprine is a TCA.
patients from 1926 to 1970), pheochromocytoma Treatment with TCAs and antipsychotic agents
was diagnosed based on paroxysmal symptoms should be tapered and discontinued at least
or refractory hypertension in 90% of patients.3 4 weeks before testing. Frequently this is not
Clinicians should suspect pheochromocytoma in possible, and in that setting, testing should proceed
the following clinical scenarios: and if the results are normal, no further testing
is needed. It is also important to recognize that
• Hyperadrenergic spells (eg, episodes of forceful catecholamine secretion may be appropriately
palpitations, diaphoresis, headache, tremor, increased in situations of physical stress or
pallor)—but recognizing that most patients illness (eg, stroke, myocardial infarction, acute
with spells do not have pheochromocytoma congestive heart failure). Medications that may
• Treatment-resistant hypertension increase measured levels of catecholamines
• A familial syndrome that predisposes to and metanephrines include TCAs (including
pheochromocytoma or paraganglioma cyclobenzaprine); levodopa; drugs containing
(eg, multiple endocrine neoplasia type 2, adrenergic receptor agonists (eg, decongestants);
neurofibromatosis type 1, von Hippel–Lindau amphetamines; buspirone and most psychoactive
syndrome, pathogenic variants in the SDHx genes) agents (but not selective serotonin reuptake
inhibitors); selective norepinephrine reuptake
• A family history of pheochromocytoma
inhibitors (may cause 2-fold increases above
• An incidentally discovered adrenal mass (3% upper limit of reference range); prochlorperazine;
to 5% of all adrenal incidentalomas prove to be reserpine withdrawal from clonidine and other
pheochromocytoma) drugs (eg, illicit drugs); and ethanol. However,
• A pressor response to anesthesia, surgery, with current assay methodology (tandem
angiography, high-dose corticosteroid, or mass spectrometry, high-performance liquid
β-adrenergic blocker chromatography), antihypertensive medications
and acetaminophen do not interfere with testing.
• Onset of hypertension at a young age
Genetic testing should be considered
(<30 years)
in all patients with pheochromocytoma,
Case-detection testing can be accomplished by especially if a patient has 1 or more of the
measurement of fractionated catecholamines following: paraganglioma; bilateral adrenal
(dopamine, norepinephrine, and epinephrine) and pheochromocytoma; unilateral adrenal
fractionated metanephrines (metanephrine and pheochromocytoma and a positive family history
normetanephrine) by high-performance liquid for pheochromocytoma; unilateral adrenal
chromatography or tandem mass spectrometry. pheochromocytoma and young age (<60 years);
At Mayo Clinic, the most reliable case-detection or other clinical findings suggestive of a related
strategy is measuring fractionated metanephrines syndromic disorder. Genetic testing can be

54  ENDO 2021 • Endocrine Case Management

chased by Dr. Khalid H. Seedahmed, Jr., MRCP PGDip, CertEndocrinology SA - ID 283


complex, and testing of one family member has with primary aldosteronism compared with the
implications for related individuals. Genetic prevalence in those with essential hypertension.
counseling is recommended to help families Longstanding, undiagnosed primary aldosteronism
understand the implications of genetic test results, frequently leads to chronic kidney disease. In
to coordinate testing of at-risk individuals, and a recent meta-analysis of 31 studies, including
to help families work through the psychosocial 3838 patients with primary aldosteronism and
issues that may arise before, during, or after 9284 patients with essential hypertension,
the testing process. An asymptomatic person at patients with aldosterone-producing adenomas
risk for disease on the basis of family history of and bilateral hyperplasia had an increased risk of
pheochromocytoma or paraganglioma should have stroke (odds ratio [OR], 2.58), coronary artery
genetic testing only if an affected family member disease (OR, 1.77), atrial fibrillation (OR, 3.52),
has a known pathogenic variant. and heart failure (OR, 2.05).4 In addition, the
diagnosis of primary aldosteronism increased the
risk of diabetes (OR, 1.33), metabolic syndrome
Primary Aldosteronism
(OR, 1.53), and left ventricular hypertrophy
Primary aldosteronism is usually diagnosed in (OR, 2.29).4 Thus, the cardiovascular toxicity
patients between 20 and 60 years of age. Current in primary aldosteronism extends beyond
prevalence estimates for primary aldosteronism hypertension; there is an aldosterone-specific
are 5% to 10% of all patients with hypertension. toxicity. In addition, several studies have
There is no reliable clinical phenotype to guide demonstrated the negative impact of primary
the clinician on which patients should be aldosteronism on quality of life.
tested for primary aldosteronism. The degree Before 1981, primary aldosteronism was
of hypertension is typically moderate to severe thought to be a rare cause of hypertension. Over
and may be resistant to usual pharmacologic time, it has been shown that most patients with
treatments. In general, patients with aldosterone- primary aldosteronism are not hypokalemic and
producing adenomas tend to have higher that case-detection testing can be completed
aldosterone levels and higher blood pressures than without stopping antihypertensive medications.
do patients with bilateral idiopathic hyperplasia. Case-detection testing is performed with a
Because hypokalemia is present in only 28% of morning venipuncture for the measurement of
patients with primary aldosteronism, all patients plasma aldosterone concentration and plasma
with hypertension are potential candidates for renin activity or plasma renin concentration.
this disorder. There is a unique subset of young Familial primary aldosteronism should
patients (typically <35 years) who present with be considered when primary aldosteronism is
marked hypokalemia but are not technically diagnosed before age 20 years or when primary
hypertensive with systolic/diastolic blood aldosteronism is diagnosed in more than one
pressures of 130s/80s mm Hg. These patients family member. In nearly all cases, patients with
usually have prior baseline blood pressures that familial primary aldosteronism have bilateral
average 100/60 mm Hg. Thus, although they adrenal disease and would not be cured with
do not meet the criteria for hypertension, there unilateral adrenalectomy.
is a clinically significant change from baseline,
and presumably their young age and blood
pressure counter-regulatory mechanisms prevent Clinical Case Vignettes
hypertension, at least in the first year or two of Case 1
the disease.
You see two 50-year-old women with adrenal
The prevalence of target-organ damage to
incidentalomas. Axial CT images of their adrenal
the heart and kidney is increased in patients
masses are shown (see images).

ENDO 2021 • Adrenal  55

chased by Dr. Khalid H. Seedahmed, Jr., MRCP PGDip, CertEndocrinology SA - ID 283


Patient 1 (HU). In a multicenter retrospective study of
533 patients with 548 histologically confirmed
pheochromocytomas, among the 376
pheochromocytomas for which unenhanced
CT attenuation data were available, 374 had an
attenuation value greater than 10 HU (99.5%).5
In the 2 exceptions (0.5%), the unenhanced
CT attenuation was exactly 10 HU. An
adrenal incidentaloma with an unenhanced
CT attenuation of 1.2 HU simply cannot be
a pheochromocytoma. The smart clinician
can avoid the conundrum of false-positive
biochemical testing in Patient 1 by not testing
for pheochromocytoma, as biochemical testing
is not needed if the unenhanced CT attenuation
is less than 10 HU. The CT image for Patient 2 is
Patient 2 highly suspicious for pheochromocytoma, and this
patient should be screened (Answer B).

Case 2
You are asked to see a 49-year-old man for an
incidentally discovered, lipid-poor, vascular, 1.6-cm
left adrenal mass. The patient has been troubled by
intermittent right upper-quadrant pain, which was
thought to be related to gallbladder dysfunction.
In the process of imaging, the 1.6-cm mass was
incidentally discovered (see images, arrows). The
unenhanced CT attenuation is 40.5 HU.
Biochemical testing documents normal plasma
concentrations of fractionated metanephrines:
Metanephrine = 39 pg/mL (<99 pg/mL)
Which patient(s) should be screened (SI: <0.2 nmol/L [<0.5 nmol/L])
for pheochromocytoma? Normetanephrine = 101 pg/mL (<165 pg/mL)
A. Patient 1 (SI: 0.55 nmol/L [<0.9 nmol/L])
B. Patient 2 In addition, 24-hour urine shows normal excretion
C. Both patients of the following:
D. Neither patient Metanephrine = 151 µg/24 h (<400 µg/24 h)
(SI: 766 nmol/d [<2028 nmol/d])
Answer: B) Patient 2 Normetanephrine = 447 µg/24 h (<900 µg/24 h)
(SI: 2441 nmol/d [<4914 nmol/d])
In this clinical vignette, you are provided with Epinephrine = 7.6 µg/24 h (<21 µg/24 h)
the unenhanced CT images from 2 patients (SI: 41 nmol/d [<115 nmol/d])
with nearly identical 2.8-cm right adrenal Norepinephrine = 61 µg/24 h (<80 µg/24 h)
masses. The one difference is the unenhanced (SI: 361 nmol/d [<473 nmol/d])
CT attenuation measured in Hounsfield units Dopamine = 201 µg/24 h (<400 µg/24 h)
(SI: 1311 nmol/d [<2610 nmol/d])

56  ENDO 2021 • Endocrine Case Management

chased by Dr. Khalid H. Seedahmed, Jr., MRCP PGDip, CertEndocrinology SA - ID 283


Do the normal plasma fractionated Answer: D) Next-generation
metanephrines and normal 24-hour urine sequencing panel for 12 genes
results for fractionated metanephrines
and catecholamines exclude the diagnosis The pathogenesis of pheochromocytoma can be
of pheochromocytoma in this patient? separated into 2 general pathways:
A. Yes • Hypoxic signaling pathway—“Cluster 1”
B. No (noradrenergic tumors): SDHx (SDHA,
C. Umm, maybe SDHAF2, SDHB, SDHC, SDHD); VHL; FH; EPAS1
(HIF2A); EGLN1 (PHD2), EGLN2 (PDH1); KIF1B;
Answer: B) No IDH1; MDH2; SLC25A11; DNMT3A
At laparoscopic left adrenalectomy, this • Kinase signaling pathway—“Cluster 2”
patient proved to have a 2.1 × 1.7 × 1.3-cm (adrenergic tumors): RET; NF1; MAX;
pheochromocytoma. The key learning point TMEM127
here is that all pheochromocytomas are
The prevalence of a detectable germline
“prebiochemical” when small. Pheochromocytomas
pathogenic variant decreases with each decade
need a “big factory” (typically >1.5 cm in diameter)
of life. For example, when pheochromocytoma
to be biochemically detectable. Lipid-poor and
is diagnosed in the first decade of life, 90% of
vascular adrenal masses should be considered
children have a detectable pathogenic variant
as suspicious for pheochromocytoma or small
(most commonly in the VHL gene).2 When
adrenocortical carcinoma. They should either be
pheochromocytoma is diagnosed in the seventh
resected (assuming you have an expert adrenal
to ninth decades of life, only 20% of patients
surgeon) or followed closely.
have a detectable pathogenic variant.2 The
patient in this vignette is 49 years old, and he
Case 3 has a 32% probability of carrying a germline
With regard to genetic testing in the pathogenic variant. Due to that risk, genetic
49-year-old man in Case 2, which of testing is indicated. Multigene testing with next-
the following is the best next step? generation sequencing (Answer D) is indicated
A. SDHx panel in this case because of the small size of this
patient’s pheochromocytoma and thus normal
B. RET gene testing biochemistry, one does not know if it is adrenergic
C. NF1 gene testing or noradrenergic and selective or targeted testing
D. Next-generation sequencing panel for 12 genes would be costly and time consuming.
E. Genetic testing is not indicated in this patient

ENDO 2021 • Adrenal  57

chased by Dr. Khalid H. Seedahmed, Jr., MRCP PGDip, CertEndocrinology SA - ID 283


Genetic testing should be considered for primary aldosteronism; most of the 75 million
in all patients with pheochromocytoma, people with primary aldosteronism go undetected.
especially if a patient has one or more of the Primary care providers think that primary
following: paraganglioma; bilateral adrenal aldosteronism is an uncommon and complicated
pheochromocytoma; unilateral adrenal disorder and do not test for it or treat it. Primary
pheochromocytoma and a positive family history care providers (and probably most physicians—
for pheochromocytoma; unilateral adrenal including endocrinologists!) are not following
pheochromocytoma and young age (<60 years); the Endocrine Society guidelines on primary
or other clinical findings suggestive of one of aldosteronism—where high-probability subsets of
the syndromic disorders. Genetic testing can patients are recommended to have case-detection
be complex, and testing of one family member testing. It is frustrating to see patients who were
has implications for related individuals. Genetic not tested for primary aldosteronism when they
counseling is recommended to help families were first diagnosed with hypertension, but rather
understand the implications of genetic test results, only after they have developed irreversible stage
to coordinate testing of at-risk individuals, and 4 to 5 chronic kidney disease. Clinical practice
to help families work through the psychosocial guidelines have not been effective in driving more
issues that may arise before, during, or after clinicians to consider case-detection testing for
the testing process. An asymptomatic person at primary aldosteronism.7,8 Could the guidelines
risk for disease on the basis of family history of be too complicated with regard to rules on
pheochromocytoma or paraganglioma should have medications and by focusing on recommending
genetic testing only if an affected family member subsets of patients for testing? The diagnostic
has a known pathogenic variant. algorithm should be simplified, and case-detection
testing for primary aldosteronism should be
Case 4 recommended at least once for all patients with
hypertension.8 In this clinical vignette, it would
Which of the following patients should be a shame to miss the diagnosis of primary
be screened for primary aldosteronism? aldosteronism in a 23-year-old woman with
A. 56-year-old man with hypertension controlled new-onset hypertension. The lack of hypokalemia
with 3 drugs and a serum potassium is meaningless—70% of patients with primary
concentration of 3.3 mEq/L (3.3 mmol/L) aldosteronism have a normal serum potassium
B. 23-year-old woman with new-onset concentration. Thus, both patients should be tested
hypertension controlled with 2 medications (Answer C).
and a serum potassium concentration of
4.0 mEq/L (4.0 mmol/L) Case 5
C. Both patients
You see a 56-year-old man with 20-year history of
D. Neither patient hypertension, poorly controlled for the past year
Answer: C) Both patients on 5 drugs: lisinopril, 20 mg daily; spironolactone,
50 mg daily; hydrochlorothiazide, 25 mg daily;
Answer A is technically correct because it follows metoprolol, 100 mg daily; and amlodipine, 10 mg
the 2016 Endocrine Society clinical practice daily. Despite those 5 medications, his current
guideline on primary aldosteronism.6 However, blood pressure is 165/110 mm Hg.
several studies have documented that clinicians
are not testing for primary aldosteronism.7,8 It is
likely that less than 2% of the 1.5 billion people
in the world with hypertension are being tested

58  ENDO 2021 • Endocrine Case Management

chased by Dr. Khalid H. Seedahmed, Jr., MRCP PGDip, CertEndocrinology SA - ID 283


Which medication(s) should be Should this patient have genetic testing
stopped before screening for primary for familial hyperaldosteronism?
aldosteronism in this patient? A. Yes
A. Lisinopril (ACE inhibitor) B. No
B. Spironolactone (mineralocorticoid receptor C. Umm, maybe
antagonist)
C. Hydrochlorothiazide (thiazide diuretic) Answer: B) No
D. Metoprolol (β-adrenergic blocker) There are 6 familial forms of primary aldosteronism:8
E. All of the above
• Glucocorticoid-remediable aldosteronism
F. None of the above (GRA) or familial hyperaldosteronism type I—
Answer: F) None of the above CYP11B1/CYP11B2 germline chimeric gene
• Familial hyperaldosteronism type II—germline
In this patient with severe hypertension, no reasonable CLCN2 pathogenic variants
clinician should stop these medications and switch
to doxazosin and verapamil. Most clinicians simply • Familial hyperaldosteronism type III—
would not test this patient for primary aldosteronism germline KCNJ5 pathogenic variants
because they think they cannot due to the “interfering” • Familial hyperaldosteronism type IV—
medications. The facts are these: (1) no medication germline CACNA1H pathogenic variants
causes false-positive testing if you use a cutoff for the • Primary aldosteronism with seizures and
plasma aldosterone concentration (eg, >10 ng/dL neurologic abnormalities (PASNA)—germline
[>277 pmol/L]); (2) in most patients with primary CACNA1D pathogenic variants
aldosteronism, you cannot budge plasma renin
activity (or plasma renin concentration) with • Familial hyperaldosteronism and ARMC5
these medications; (3) most patients treated with pathogenic variants, causing bilateral
mineralocorticoid receptor antagonists are on macronodular adrenal hyperplasia
subtherapeutic dosages and if the patient has However, all forms of familial hyperaldosteronism
primary aldosteronism, plasma renin activity will are rare; type II is the most common. Familial
be suppressed; and (4) endocrinologists have created hyperaldosteronism should be considered when
a barrier to diagnosing primary aldosteronism, primary aldosteronism is diagnosed before age
leading to increased morbidity.9 Patients can be on 20 years or when primary aldosteronism is diagnosed
any sodium diet and any blood pressure medication, in more than one family member. In nearly all cases,
including spironolactone and eplerenone, when patients with familial hyperaldosteronism have
performing case-detection testing for primary bilateral adrenal disease and would not be cured
aldosteronism.8 If renin is not suppressed in that with unilateral adrenalectomy. Thus, there is no
setting and the clinician is still suspicious of primary role for genetic testing in the patient described in
aldosteronism, then simply stop what are viewed this clinical vignette. What would be recommended
as the offending medications, keep blood pressure to patients such as this one is to encourage any
controlled, and repeat case-detection testing. blood relatives who have hypertension to be
screened for primary aldosteronism by measuring
Case 6 plasma aldosterone and plasma renin activity.
The 56-year-old patient from Case 5 proved to
have primary aldosteronism due to a unilateral
aldosterone-producing adenoma and was cured
with unilateral adrenalectomy.

ENDO 2021 • Adrenal  59

chased by Dr. Khalid H. Seedahmed, Jr., MRCP PGDip, CertEndocrinology SA - ID 283


Case 7  New Content  Her family history is remarkable. Her paternal
grandfather died at age 47 years after returning
During her first pregnancy, a 32-year-old
from World War II with hypertension and a
woman is noted to have hyperglycemia in the
known pheochromocytoma. Her paternal aunt had
first trimester and abnormal liver function
a pheochromocytoma removed at age 38 years.
tests in the third trimester. Her obstetrician
A first cousin (offspring of her paternal aunt) had
recommends inducing labor. During induction,
a pheochromocytoma removed at age 18 years. In
her blood pressure rises to 180/90-100 mm Hg.10
addition, 3 children of her paternal grandfather’s
She has no history of hypertension or diabetes.
brother had pheochromocytoma. There is no
Labor lasts 24 hours, and her blood pressure is
family history of medullary thyroid carcinoma,
elevated throughout and treated with magnesium
hyperparathyroidism, cerebellar tumors, spinal
sulfate. Her baby boy’s delivery is complicated by
cord tumors, retinal angiomas, pancreatic tumors,
pneumothorax and hypoglycemia, and he is kept
kidney tumors, neurofibromas, or any other
in the neonatal intensive care unit for 5 days.
familial neoplasms. Her father has a history of
Treatment of the mother’s hypertension with
hypertension and coronary artery disease, but he
magnesium sulfate is continued for 24 hours after
has not been screened for pheochromocytoma.
delivery. However, she develops fast and forceful
The patient has 2 brothers and 1 sister, and none
palpitations, headache, and nausea, and the poorly
has been screened for pheochromocytoma.
controlled hypertension persists. Because of nausea
Her medications include phenoxybenzamine,
and emesis, she is treated with metoclopramide.
10 mg twice daily, and carvedilol, 6.25 mg
Shortly after receiving metoclopramide, she
twice daily.
has a sense of impending doom and is diffusely
On physical examination, her BMI is
diaphoretic. Electrocardiography shows findings
24.1 kg/m2, blood pressure is 102/67 mm Hg, and
consistent with acute myocardial infarction.
pulse rate is 100 beats/min. Findings on retinal
She is transferred from the obstetric floor to the
examination are normal, and no Lisch nodules
coronary care unit where echocardiography shows
are evident on examination of the iris bilaterally.
an ejection fraction of 45% and hypokinesis of the
The thyroid gland is normal to palpation.
cardiac apex. Troponin I is elevated at 10 ng/mL
The skin is normal with no café-au-lait spots,
(10 µg/L) (normal <0.04 ng/mL [<0.04 µg/L]).
axillary or inguinal freckling, or subcutaneous
Coronary angiography shows normal coronary
neurofibromas.
arteries.11,12 Treatment with carvedilol is initiated.
While in the coronary care unit, she mentions that
some family members have been diagnosed with Investigations
pheochromocytoma. This triggers a 24-hour urine Laboratory studies at Mayo Clinic reconfirm a
collection for fractionated metanephrines and noradrenergic catecholamine-secreting tumor.
catecholamines, with the following results: Findings on echocardiography are normal with
Norepinephrine = 1555 µg/24 h (<80 µg/24 h) no regional wall motion abnormalities, and the
(SI: 9196 nmol/d [<473 nmol/d]) ejection fraction is normal at 61%. The exercise
Normetanephrine = 13,000 µg/24 h (<900 µg/24 h) sestamibi study is normal with no evidence of
(SI: 70,980 nmol/d [<4914 nmol/d]) provokable ischemia.
Treatment with phenoxybenzamine is initiated, The patient undergoes laparoscopic right
and CT of the abdomen detects a 4.5-cm right adrenalectomy, and the pheochromocytoma is
adrenal mass. She is referred to Mayo Clinic resected intact with great care to avoid tumor capsule
at 5 weeks post partum. With the addition of rupture. The tumor measures 5.3 × 5.0 × 2.0 cm.
phenoxybenzamine, 10 mg twice daily, her blood
pressure averages 100/60 mm Hg.

60  ENDO 2021 • Endocrine Case Management

chased by Dr. Khalid H. Seedahmed, Jr., MRCP PGDip, CertEndocrinology SA - ID 283


Outcome and Follow-Up 4 DNA bases: adenine (A), guanine (G), cytosine
(C), and thymine (T). The codon AGC, for
The patient recovers well from the operation
example, codes the amino acid serine. Since there
and is discharged from the hospital on the
are 4 bases in 3–base pair combinations, there are
second postoperative day. Plasma fractionated
64 possible codons; 61 specify amino acids and
metanephrines measured 2 days postoperatively
3 specify stop signals. Thus, most of the 20 amino
are normal. Blood pressure is normal without
acids are coded by more than one codon. For
medications. Germline genetic testing does not
example, proline is coded by codons CCT, CCC,
detect a pathogenic variant in any of the genes
CCA, and CCG. In our patient’s case, in codon
known to be associated with pheochromocytoma
138, instead of CCA, adenine (A) was replaced
or paraganglioma. She is followed with annual
by guanine (G), so the new codon became CCG.
biochemical testing.
Both CCA and CCG code for proline, so the amino
acid did not change. Synonymous variants are
The Genetic Mystery Solved often assumed to be neutral or functionally silent,
Fifteen years after presentation, this patient’s since there is no change in the product protein
annual biochemical testing is still normal. Given sequence. However, synonymous variants have
the known family history of pheochromocytoma the potential to disrupt splice sites, transcription,
but negative results from clinically available or mRNA transport or translation, which can
germline genetic testing that was available in 2005 potentially alter phenotype.
(VHL, RET, SDHB, SDHD, SDHC), we advised that Sanger sequencing of germline DNA
the patient’s parents and siblings have biochemical from 16 of the patient’s relatives showed that
testing for pheochromocytoma. Her 72-year-old the c.414A>G VHL variant segregated with
father proved to have a 2.8-cm right adrenal gland pheochromocytoma in this family.13 Despite
pheochromocytoma. Her 34-year-old brother had its location away from the canonical splice-site
a 1.8-cm right adrenal gland pheochromocytoma. regions, the c.414A>G VHL variant was found
Subsequently, we discussed her family with to promote exon 2 skipping.13 This same VHL
Patricia Dahia, MD, PhD (Professor of Medicine pathogenic variant was identified in 4 additional
at the University of Texas Health Science Center families.13 With the availability of germline genetic
at San Antonio). Her career has been dedicated to testing for this pathogenic variant, our patient’s
research on the genetics of cancer, with emphasis children were tested. Her son, who was born at
on inherited endocrine tumors and discovery of the time of her pheochromocytoma discovery and
cancer susceptibility genes. Dr. Dahia completed is now 14 years old, had inherited the pathogenic
exome sequencing of our proband’s germline variant. The 24-hour urine excretion of
DNA, and it revealed no missense or truncating normetanephrine (2392 µg/24 h [13,060 nmol/d];
mutations in known pheochromocytoma or normal <900 µg/24 h [<4814 nmol/d]) was
paraganglioma susceptibility genes. However, a diagnostic of a catecholamine-secreting tumor.
heterozygous, synonymous variant was detected in Abdominal MRI detected a left periadrenal 2.7-cm
exon 2 of the VHL gene (c.414A>G, p.Pro138Pro). mass, which proved to be a paraganglioma.
A synonymous variant is similar to a missense
mutation in that a single nucleotide is substituted
for another, but because of the redundancy of the
genetic code, the amino acid remains the same.
The genetic code is composed of units known as
codons, which are sequences of 3 nucleotides. Each
codon corresponds to a specific amino acid, which
are the building blocks of proteins. There are

ENDO 2021 • Adrenal  61

chased by Dr. Khalid H. Seedahmed, Jr., MRCP PGDip, CertEndocrinology SA - ID 283


References
1. Young WF Jr, Calhoun DA, Lenders JWM, et al. Screening for endocrine 8. Mulatero P, Monticone S, Burrello J, Veglio F, Williams TA, Funder J.
hypertension: an Endocrine Society scientific statement. Endocr Rev. Guidelines for primary aldosteronism: uptake by primary care physicians in
2017;38(2):103-122. Europe. J Hypertens. 2016;34(11):2253-2257. PMID: 27607462
2. Neumann HPH, Young WF Jr, Eng C. Pheochromocytoma and 9. Young WF Jr. Diagnosis and treatment of primary aldosteronism: practical
paraganglioma. N Engl J Med. 2019;381(6):552-565. PMID: 31390501 clinical perspectives. J Intern Med. 2019;285(2):126-148. PMID: 30255616
3. Gruber LM, Hartman RP, Thompson GB, et al. Pheochromocytoma 10. Bancos I, Atkinson E, Eng C, Young WF Jr, Neumann HPH; International
characteristics and behavior differ depending on method of discovery. J Clin Pheochromocytoma and Pregnancy Study Group. Maternal and fetal
Endocrinol Metab. 2019;104(5):1386-1393. PMID: 30462226 outcomes in phaeochromocytoma and pregnancy: a multicentre retrospective
4. Monticone S, D’Ascenzo F, Moretti C, et al. Cardiovascular events and cohort study and systematic review of literature. Lancet Diabetes Endocrinol.
target organ damage in primary aldosteronism compared with essential 2021;9(1):13-21. PMID: 33248478
hypertension: a systematic review and meta-analysis. Lancet Diabetes 11. Melson E, Amir S, Shepherd L, Kauser S, Freestone B, Kempegowda
Endocrinol. 2018;6(1):41-50. PMID: 29129575 P. Myocardial infarction with non-obstructed coronaries - atypical
5. Canu L, Van Hemert JAW, Kerstens MN, et al. CT characteristics of presentation of pheochromocytoma. Endocrinol Diabetes Metab Case Rep.
pheochromocytoma: relevance for the evaluation of adrenal incidentaloma. 2019;2019:EDM190089. PMID: 31634865
J Clin Endocrinol Metab. 2019;104(2):312-318. PMID: 30383267 12. Boulkina LS, Newton CA, Drake AJ 3rd, Tanenberg RJ. Acute
6. Funder JW, Carey RM, Mantero F, et al. The management of primary myocardial infarction attributable to adrenergic crises in a patient
aldosteronism: case detection, diagnosis, and treatment: an Endocrine Society with pheochromocytoma and neurofibromatosis 1. Endocr Pract.
clinical ppractice guideline. J Clin Endocrinol Metab. 2016;101(5):1889-1916. 2007;13(3):269-273. PMID: 17599858
PMID: 26934393 13. Flores SK, Cheng Z, Jasper AM, et al. A synonymous VHL variant in exon 2
7. Rossi E, Perazzoli F, Negro A, Magnani A. Diagnostic rate of primary confers susceptibility to familial pheochromocytoma and von Hippel-Lindau
aldosteronism in Emilia-Romagna, Northern Italy, during 16 years disease. J Clin Endocrinol Metab. 2019;104(9):3826-3834. PMID: 3094646.
(2000-2015). J Hypertens. 2017;35(8):1691-1697. PubMed PMID: 28410304

62  ENDO 2021 • Endocrine Case Management

chased by Dr. Khalid H. Seedahmed, Jr., MRCP PGDip, CertEndocrinology SA - ID 283


Difficulties in Diagnosis
and Management of
Adrenal Insufficiency in
the Critically Ill Patient
David J. Torpy, MBBS, PhD, FRACP. Endocrine and Metabolic Unit, Royal Adelaide Hospital
and University of Adelaide, Adelaide, SA, Australia; E-mail: david.torpy@sa.gov.au

Learning Objectives catecholaminergic vasopressors, has been studied


extensively over the past 2 decades from the point
As a result of participating in this session, learners
of view of glucocorticoid therapy. Earlier studies
should be able to:
of high-dose, short-term synthetic glucocorticoids
• Describe the current status of adjunctive for septic shock showed increased mortality.
hydrocortisone therapy in critical illness. Continuing clinical observations of benefit from
hydrocortisone in septic shock led to a succession
• Explain the clinical options for management
of small trials revealing salutary effects such as
of adrenal insufficiency in the light of evidence
a pressor response and reduced inflammatory
from physiological studies and clinical trials in
cytokines, ultimately leading to progressively
critical illness.
larger multicenter randomized controlled trials
of “low-dose” hydrocortisone. The general
conclusion is that randomized hydrocortisone,
generally 200 mg daily, leads to reduced time-to-
Main Conclusions withdrawal of vasopressor support (1-2 days) and a
Patients with critical illness are subject to possible small benefit in mortality. Sicker patients
profound physiological stress with multiple stimuli and curiously those treated with fludrocortisone
such as fever, elevation of inflammatory cytokines, may benefit more in terms of mortality. It may
pain, and hypovolemia leading to activation be that a subset of patients benefits more from
of the hypothalamic-pituitary-adrenal (HPA) hydrocortisone. A state of reduced cortisol
axis. Cortisol has a pivotal role in maintaining response to cosyntropin, 250 mcg (<9 µg/dL
homeostasis and in immunomodulation in [<248 nmol/L] at 60 minutes) may identify
critical illness. Routine measurements of serum a state of relative adrenal insufficiency, later
cortisol may not reflect end-organ glucocorticoid referred to as critical illness–related corticosteroid
sufficiency due to tissue resistance to cortisol and insufficiency (CIRCI). Initial studies suggested
reduced concentrations of the principal carrier CIRCI predicted mortality benefit from parenteral
proteins for cortisol, corticosteroid-binding hydrocortisone, but this was not confirmed in
globulin (CBG) and albumin. larger trials. At present, hydrocortisone, 200 mg
Septic shock, a common state and important daily, 50 mg every 6 hours or by intravenous
cause of mortality, with a 30% to 40% mortality infusion for 7 days without taper, perhaps
rate despite standard antibiotics, fluids, and with fludrocortisone, 0.05 mg enterally via

ENDO 2021 • Adrenal  63

chased by Dr. Khalid H. Seedahmed, Jr., MRCP PGDip, CertEndocrinology SA - ID 283


nasogastric tube daily, is indicated in patients Barriers to Optimal Practice
with hypotension despite fluid resuscitation and
vasopressor treatment. Other conditions for • Conventional measurements of serum cortisol,
which hydrocortisone therapy may be clinically performed on a random or postcosyntropin
indicated include community-acquired pneumonia basis, are not strong predictors of response
requiring hospitalization, acute respiratory distress to exogenous glucocorticoids, although they
syndrome, and bacterial meningitis. A place for may still identify patients with conventional
hydrocortisone in severe trauma is not established. adrenal insufficiency.
The decisions regarding hydrocortisone in
• Immediate response to exogenous
these conditions are generally not made by
glucocorticoids may not reflect mortality or
endocrinologists, but awareness of the trials and
ultimate pressor benefit in critical illness.
limitations of the evidence is of value, particularly
among endocrinologists in hospital practice.
Strategies for Diagnosis,
Therapy, and/or Management
Significance of the
Clinical Problem Definitions
The use of glucocorticoids in critical illness has Adrenal insufficiency is defined as the
been controversial for decades. Endocrinologists inability to produce sufficient glucocorticoids
will find the area challenging, as the notion and mineralocorticoids for tissue
of corticosteroid insufficiency (ie, a lack of requirements.1 Biochemically, it is defined by
glucocorticoid sufficiency at the tissue level a suboptimal cortisol response to cosyntropin,
without obvious deficits in random or 250 mcg (eg, peak cortisol at 30 minutes
cosyntropin-stimulated cortisol) requires <18 µg/dL [<500 nmol/L] [assay dependent])
reference to clinical trial data and consideration and subclassified by ACTH level into primary
of patient status where many variables may affect and secondary forms. Primary forms may have
the primary feature of CIRCI (ie, hypotension aldosterone deficiency as determined by plasma
unresponsive to fluids and vasopressors). Previous aldosterone and renin measurements. CIRCI is
operational definitions of CIRCI, including the a concept, developed in critical care medicine, of
60-minute rise in serum cortisol of less than impaired HPA response to the stress of critical
9 µg/dL (<248 nmol/L), remain controversial. illness.2 CIRCI is operationally defined as a
A measure of tissue glucocorticoid sufficiency serum cortisol increment of less than 9 µg/dL
relevant to CIRCI is not available. Even the initial (<248 nmol/L) 60 minutes after administration of
pressor response to glucocorticoids is not known 250 mcg cosyntropin.
to be predictive of benefits such as mortality and
time to vasopressor withdrawal. Endocrinologists Critical Illness Stress Response
may be asked to comment on serum cortisol values Critical illness such as sepsis is associated with
and the use of hydrocortisone in the intensive care central stimulation of the HPA axis as a response
unit, and a working knowledge of the current data to multiple stimuli, including the following:
is useful. inflammatory cytokines IL-1, TNF-α, and IL-6
acting at all HPA levels; pain; psychic stress
of many stimuli; reduced cortisol metabolism
through inhibition of 5α/5β reductases3 and
11β-hydroxysteroid dehydrogenase type 2
(11BHSD2),4 explaining the prolonged serum

64  ENDO 2021 • Endocrine Case Management

chased by Dr. Khalid H. Seedahmed, Jr., MRCP PGDip, CertEndocrinology SA - ID 283


half-life of cortisol in sepsis, only moderately or immature neutrophils (bands >10%). The
elevated cortisol production rate, and lack of criteria for organ dysfunction or hypoperfusion
elevation of ACTH in many cases; reductions in abnormality are 1 of the following: altered
the concentrations of cortisol-binding proteins in mental status, hypoxemia (PaO2 <75 mm Hg),
plasma (CBG and albumin); redirection of adrenal oliguria (urine output <30 mL/h), and elevated
steroidogenesis from androgens towards cortisol; plasma lactate (>2 mmol/L). The definition
tissue resistance to cortisol based on the activation for hypotension is systolic blood pressure less
of NFκB pathways, which antagonize intracellular than 90 mm Hg or a reduction of systolic blood
pathways used by the glucocorticoid receptor pressure by 40 mm Hg or more from baseline.
and reduced levels and transcription of GRα and More recent criteria have used definitions that
increased levels of the dominant negative GRβ;5,6 incorporate sepsis-related organ failure assessment
corticostatins reducing ACTH receptor activity;7 (SOFA) scores.9
and drug effects on the HPA axis such as the The chief vasopressor used in septic
inhibitory effects of opioids, benzodiazepines, and shock is noradrenaline, which acts through
in some cases etomidate. In addition, concomitant vasoconstriction to elevate blood pressure.
activation of the sympathetic nervous system at Reducing the time-to-withdrawal of vasopressor
the central level is augmented by cortisol’s action, support reduces costs and complications from
increasing conversion of norepinephrine to prolonged intensive care unit stays. Anecdotally,
epinephrine at the adrenal medulla and cortisol’s patients with septic shock may experience a
augmentation of norepinephrine’s effects on the beneficial rise in blood pressure when given
vasculature to sustain and redirect blood flow. exogenous glucocorticoid, resulting in shock
Given the complexity of these influences on the reversal, confirmed by systematic analyses.10,11
HPA axis, especially the variable degree of tissue However, early studies of high-dosage, short-
resistance that accompanies systemic inflammation course synthetic glucocorticoids (equivalent to
and the fall in serum binding proteins that alter hydrocortisone, 40,000 mg daily, often in a single
the relationship between total and free cortisol, dose) in septic shock showed increased mortality.12
serum cortisol concentrations may not reflect
tissue glucocorticoid status. Effects of Stress-Dose
Sepsis is a life-threatening organ dysfunction
caused by a dysregulated host response to
Hydrocortisone in Septic Shock
infection. The American College of Chest Hydrocortisone, 10 mg hourly, increases plasma
Physicians and Society of Critical Care Medicine cortisol levels around 4-fold, to a mean of
Consensus Conference criteria in 1992 defined 109 µg/dL (3000 nmol/L) after the first day,
septic shock as the presence of systemic which is above the upper end of the normal range
inflammatory response syndrome, documented for patients with septic shock at diagnosis.13,14
infection or positive blood culture and the The effects of cortisol concentrations at this
presence of organ dysfunction, hypoperfusion level include attenuation of the inflammatory
abnormality, or sepsis-induced hypotension cytokines IL-6 and IL-8; soluble E-selectin, a
refractory to adequate fluid resuscitation or factor important in endothelial-granulocyte
vasopressor agents.8 The systemic inflammatory interactions; and expression of the neutrophil
response was manifested by 2 or more of the activation markers CD11b and CD64, while the
following: fever (temperature >100.4oF [>38oC]) antiinflammatory cytokines IL-10 and soluble
or hypothermia (temperature <95.9oF [<35.5oC]), TNF receptor I and II are also attenuated.
tachycardia (>90 beats/min), tachypnea (>20 Markers of monocyte function such as HLA-DR
breaths/min), leukocytosis or leukopenia (white antigen expression and in vitro phagocytosis are
blood cell count >12,000 or <4000 cells/µL), not affected, nor are the Th1 function markers,

ENDO 2021 • Adrenal  65

chased by Dr. Khalid H. Seedahmed, Jr., MRCP PGDip, CertEndocrinology SA - ID 283


IL-12 and IFN-γ.13 Overall, these hydrocortisone trauma.22 A 2002 randomized controlled trial
responses are considered antiinflammatory rather of hydrocortisone, 50 mg every 6 hours, and
than immunosuppressive. fludrocortisone, 0.05 mg daily, vs placebo revealed
Longer treatment (7 days) with lower- a 28-day mortality benefit for treatment in the 229
dosage hydrocortisone approximating the nonresponders to cosyntropin, 250 mcg (cortisol
normal adrenocortical cortisol output in severe response at 30 and 60 minutes less than 9 µg/dL
stress (200-300 mg daily) shortens the need for [<248 nmol/L], indicative of relative adrenal
vasopressor support. Basal cortisol values in insufficiency; confidence interval, 0.47-0.95; P =
sepsis/septic shock are not useful predictors of .02), but no mortality benefit in 70 responders.23
mortality except at extremely high cortisol levels Similarly a reduced time-to-vasopressor-
(>45 µg/dL [>1242 nmol/L]).15 The term relative withdrawal in the treatment group was apparent
adrenal insufficiency was developed to describe in nonresponders (expressed as withdrawal rates
a deleteriously attenuated cortisol response to of 57% vs 40% at 28 days), but not in responders.
exogenous ACTH without a deficient basal cortisol This study was criticized, as 72 of 99 patients
level.16 In the Rothwell study of 32 patients with received etomidate, an adrenal 11-hydroxylase
septic shock, 13 had a cortisol response to ACTH inhibitor that reduces cortisol synthesis, and 68
of less than 9 µg/dL (<250 nmol/L), of whom of these had relative adrenal insufficiency.24 The
none (0/13) survived, compared with 13 of 19 use of etomidate may explain the high rate of
survivors among those with cortisol responses relative adrenal insufficiency in the Annane et
greater than 9 µg/dL (>250 nmol/L). Hence, a al study (77%) compared with only 33%. There
total plasma cortisol response to cosyntropin, was also a relatively high mortality rate of 55%
250 mcg, of less than 9 µg/dL (<248 nmol/L) to 61%; most studies have lower mortality rates
empirically defines relative adrenal insufficiency. (30%-40%) despite similar selection criteria. The
A single center, double-blind, randomized study of results of this study increased interest and use
40 patients with septic shock patients found that of hydrocortisone in patients with septic shock,
hydrocortisone, 0.18 mg/kg per h after a 100-mg especially those in whom shock is refractory to
loading dose reduced the time-to-withdrawal fluids and catecholaminergic vasopressors.
of vasopressor support from 7 to 2 days (P = The CORTICUS trial, however, did not reveal
.005).17 Forty-one patients with early septic any 28-day mortality benefit from hydrocortisone,
shock given a 50-mg loading dose of intravenous 50 mg every 6 hours in 499 patients with septic
hydrocortisone followed by 0.18 mg/kg per h shock (233 nonresponders and 266 responders
had a reduced time-to-withdrawal of vasopressor to cosyntropin, 250 mcg at 60 minutes; 251
from a median 120 to 53 hours (P = .02) with treated patients, 248 patients placebo) relative to
lowered serum cytokines.18 A relationship randomized placebo.25 Hydrocortisone, however,
between relative adrenal insufficiency and was associated with more rapid resolution
survival was noted in 189 patients with septic of shock. Risks of hydrocortisone included
shock with the lowest survival rate seen in those hyperglycemia, hypernatremia, and new infection.
with a basal cortisol concentration greater than The CORTICUS trial was under-enrolled (500 vs
34 µg/dL (>935 nmol/L) and a cortisol response planned 800 patients) and was criticized for being
to cosyntropin at 60 minutes less than 9 µg/dL underpowered and had a relatively low mortality,
(<248 nmol/L).19 Relative adrenal insufficiency is suggesting less sick patients were enrolled.26 The
associated with a blunted response to vasopressors relevance of nonadministration of fludrocortisone
that can be reversed by hydrocortisone.20,21 in the CORTICUS trial is still debated, although
These precepts were tested over 2 decades hydrocortisone in dosages of 200 mg daily has full
in clinical trials of critical illness, mostly in mineralocorticoid traditional effects on fluid and
septic shock but also in sepsis and major electrolyte status, effects of fludrocortisone on

66  ENDO 2021 • Endocrine Case Management

chased by Dr. Khalid H. Seedahmed, Jr., MRCP PGDip, CertEndocrinology SA - ID 283


central or immune function, perhaps particularly cortisol may protect tissues from immune damage,
relevant in sepsis, are mooted based on animal thereby reducing the risk of multiorgan failure, the
studies.27 After these and other trials (totaling sine qua non of septic shock. Depletion of high-
33 randomized controlled trials of 4268 patients), affinity CBG is a feature of sepsis, with the extent
a Cochrane systematic review concluded that of depletion relating to categorical sepsis illness
corticosteroids significantly reduced mortality at severity and mortality.33,34 Routine measurements
28 days compared with placebo.28 Most recently, of serum cortisol do not allow assessment of free
2 large trials have further defined the utility of cortisol or that bound to specific carrier proteins,
hydrocortisone in patients with septic shock.29,30 especially that fraction bound to high-affinity
Venkatesh et al administered 200 mg intravenous CBG. Hence, routine cortisol measurements may
hydrocortisone via continuous infusion for 7 days, not reflect that portion of circulating cortisol most
no taper) to 3658 ventilated patients with septic relevant to tissue action in the setting of sepsis.
shock (1832 hydrocortisone, 1826 placebo). No
difference in 28-day mortality was observed, but Current Intensive Care Practice
a 1-day reduction in time to shock resolution
(3 vs 4 days) and a 1-day reduction in initial The major critical care medicine societies
time of mechanical ventilation (5 vs 7 days) was guidelines regarding CIRCI22,35 appeared before
noted, with no increase in rates of bacteremia or the recent published large trials; however, these
fungemia.29 In contrast, Annane et al administered trials still suggest that there are positive pressor
hydrocortisone, 50 mg intravenously every 6 hours, effects of hydrocortisone, perhaps augmented
and fludrocortisone, 50 mcg daily via nasogastric by fludrocortisone, in septic shock.29,30 The
tube, or placebo to 1241 patients with septic shock effects of hydrocortisone administration on
and achieved a lower 90-day (but not 28-day) mortality (relative risk 0.91-0.96; upper limit
mortality in the treatment group (43% vs 49.1%), as 95% confidence interval, 0.98-1.03) are modest if
well as fewer days requiring vasopressor support. deployed randomly to patients with septic shock
There were similar adverse events, apart from after fluid resuscitation and vasopressor therapy.36
increased hyperglycemia, in the treatment group.30 Current total cortisol measurements at baseline
or after corticotropin are not considered reliably
predictive of a response to adrenal corticosteroids.
Potential for Plasma Free Cortisol The Society of Critical Care Medicine and
and CBG Measurements to European Society of Intensive Care Medicine22,35
Improve Diagnosis of CIRCI include the following points regarding CIRCI:
Serum cortisol is approximately 80% bound to • No recommendation for routine use of the
high-affinity CBG, 15% bound to low-affinity 250-mcg cosyntropin test
albumin, and 5% free. Cortisol levels above 20 to
25 µg/dL (>552 to 690 nmol/L) saturate CBG, • Suggest against use of free cortisol (very low-
leading to an exponential rise in free cortisol quality evidence) or salivary cortisol (very low-
concentration.31 In sepsis, TNFα and IL-6 reduce quality evidence) for diagnosis
hepatic CBG synthesis. When comparing control • Use of 250 mcg over 1 mcg cosyntropin (low-
patients with patients who have sepsis and septic quality evidence) for diagnosis
shock, free cortisol increments correspond to • Suggested use of cosyntropin 250 mcg
illness severity more closely than total cortisol.14 over a hemodynamic response test to bolus
CBG may be cleaved at inflammatory sites by hydrocortisone 50-300 mg (very low-quality
neutrophil elastase, reducing the CBG to cortisol evidence)
binding affinity by 90%.32 This liberates cortisol to
• Suggest against use of ACTH levels for CIRCI
tissues where the immunomodulatory effects of
diagnosis (low-quality evidence)

ENDO 2021 • Adrenal  67

chased by Dr. Khalid H. Seedahmed, Jr., MRCP PGDip, CertEndocrinology SA - ID 283


These guidelines make the following treatment severe postural hypotension, anorexia, and
recommendations: profound fatigue. Routine blood tests reveal
mild hyponatremia, normal serum potassium,
• Suggest against use of corticosteroids for adults and a serum cortisol concentration of 4 µg/dL
with sepsis without shock (110 nmol/L).
• Suggest use of corticosteroids in adults septic
shock unresponsive to fluid and moderate- to Which of the following statements is true?
high-dosage vasopressor therapy (low-quality A. Cosyntropin-stimulation testing is needed
evidence) to determine whether he has adrenal
• Use of long course, low-dosage intravenous insufficiency or CIRCI
hydrocortisone (<400 mg daily, >3 days) (low- B. Current use of synthetic glucocorticoid therapy
quality evidence) may be the cause of this clinical situation
• Use of corticosteroids in patients with early C. Past use of synthetic glucocorticoid therapy is a
acute respiratory distress syndrome (moderate- likely cause of this situation
quality evidence) D. Checkpoint inhibitor–related adrenal
• Suggest against use of corticosteroids in major insufficiency is the likely cause of the
trauma (low-quality of evidence) presentation, and parenteral glucocorticoids
should be administered without delay
• Recommendations were made for the use
of corticosteroids in hospitalized patients E. Most patients with checkpoint inhibitor–
with early moderate to severe acute related adrenal insufficiency recover with
respiratory distress syndrome (conditional time and this process is aided by high-dosage
recommendation [ie, benefits of adherence corticosteroid administration
probably outweighs risks]) (moderate-quality Answer: D) Checkpoint inhibitor–related
evidence); community-acquired pneumonia adrenal insufficiency is the likely cause of the
(<400 mg hydrocortisone or equivalent daily, presentation, and parenteral glucocorticoids
5-7 days) (conditional, moderate-quality should be administered without delay
evidence)
• Recommend corticosteroid use in bacterial Baseline cortisol and clinical features are
meningitis (strong recommendation, low- sufficiently diagnostic of adrenal insufficiency.
quality evidence) Synthetic glucocorticoid therapy would not
be consistent with clinical features. Past
• Suggest use in coronary bypass surgery glucocorticoid therapy rarely produces clinical
(conditional recommendation, moderate- features of this severity. Ipilimumab (CTLA-4
quality evidence) inhibitor) produces hypophysitis in up to 20% of
• Suggest use in cardiac arrest (conditional patients. HPA axis recovery is rare, and there is no
recommendation, very low-quality evidence) recovery benefit from high-dosage glucocorticoids.

Case 2
Clinical Case Vignettes
A 75-year-old man is admitted to the hospital with
Case 1 septic shock 2 weeks after a 3-vessel coronary
A 58-year-old man with metastatic melanoma bypass graft procedure. Fluid resuscitation is
being treatment with the immune checkpoint administered, as well as a norepinephrine infusion
inhibitor ipilimumab presents with low-grade at a rate of 2.0 mcg/kg per min. However, there
fever, hypotension (blood pressure 80/50 mm Hg), is persistent hypotension. A random serum

68  ENDO 2021 • Endocrine Case Management

chased by Dr. Khalid H. Seedahmed, Jr., MRCP PGDip, CertEndocrinology SA - ID 283


cortisol measurement is 18 µg/dL (497 nmol/L) Case 3
and a 250-mcg cosyntropin test reveals a baseline
A 33-year-old woman with Addison disease has
cortisol value of 14 µg/dL (386 nmol/L) with a rise
a cystoscopy for the investigation of microscopic
to 17 µg/dL (469 nmol/L) at 60 minutes.
hematuria (sterile urine). Twenty-four hours
later, she presents with fever 102ºF (39ºC), rigors,
Which of the following statements is true?
neutrophilia, and weakness. Her blood pressure is
A. Administration of intravenous hydrocortisone 110/70 mm Hg. Antimicrobial therapy directed at
is not indicated, as the evidence in trials does urosepsis is initiated.
not show a consistent mortality benefit and the
risks of treatment are excessive Which of the following statements is true?
B. This patient meets the criteria for treatment A. Glucocorticoid stress dosing (hydrocortisone,
with hydrocortisone for presumed CIRCI and 100 mg intravenously every 6 hours, and
cosyntropin testing was mandatory before 200 mg daily for 2 days) was necessary
consideration of intravenous hydrocortisone perioperatively to prevent adrenal crisis
therapy
B. No glucocorticoid stress doses were required
C. The results of testing may reflect low cortisol- with the procedure
binding proteins (corticosteroid-binding
C. Given the circumstances, full glucocorticoid
globulin and albumin) and these should be
stress dosing (50 mg every 6 hours) should be
measured to clarify the pathophysiology and
implemented to prevent adrenal crisis, as well
guide treatment
as appropriate antibiotic therapy
D. Salivary free cortisol measurement may assist
D. The patient has sepsis and even without a
with diagnosis
history of Addison disease, hydrocortisone,
E. The clinical circumstances warrant the 200 mg daily, is indicated given that sepsis
use of low-dosage hydrocortisone such as connotes a clear risk of progression to septic
intravenous 200 mg daily for 7 days without shock
taper
E. The administration of supraphysiologic
Answer: E) The clinical circumstances warrant dose glucocorticoids in this circumstance
the use of low-dosage hydrocortisone such as risks hyperglycemia and hypernatremia
intravenous 200 mg daily for 7 days without taper and may promote infection and is therefore
contraindicated
Hydrocortisone use is recommended in septic
shock unresponsive to fluid resuscitation and Answer: C) Given the circumstances, full
inotropes. Pressor effects are well demonstrated glucocorticoid stress dosing (50 mg every 6 hours)
to reduce vasopressor requirement and time to should be implemented to prevent adrenal crisis,
withdrawal of vasopressor. The CIRCI operational as well as appropriate antibiotic therapy
definition is not required for hydrocortisone
This patient has sepsis with severe manifestations.
treatment. Risks of 7-day hydrocortisone
She is at risk of adrenal crisis and stress-dose
treatment do not outweigh benefit. Cortisol-
hydrocortisone is required. Sepsis without shock
binding protein measurements have not been
in a patient without Addison disease would not
shown to be useful to guide treatment to date.
mandate use of hydrocortisone. Hydrocortisone,
100 mg intravenously/intramuscularly with the
procedure and double-dose oral glucocorticoids
for 48 hours was required.

ENDO 2021 • Adrenal  69

chased by Dr. Khalid H. Seedahmed, Jr., MRCP PGDip, CertEndocrinology SA - ID 283


Case 4 E. Hydrocortisone administration may be
indicated for the community-acquired
A 24-year-old man had a motorcycle crash
pneumonia on clinical grounds alone and
with a head injury resulting in unconsciousness
reassessment of the hypothalamic-pituitary
for 4 days, requiring several months of
axis can be performed outside of the setting
inpatient rehabilitation. He was treated with
of acute illness where testing and clinical
dexamethasone, 4 mg every 6 hours, for 2 weeks
assessment is likely to be more relevant to
after the head injury. Six months after his injuries,
long-term therapy considerations
he presents with severe dyspnea and hypoxemia
that have been present for several hours. He is Answer: E) Hydrocortisone administration may be
hypotensive (blood pressure 90/60 mm Hg), and indicated for the community-acquired pneumonia
chest x-ray shows lobar pneumonia. Tests of on clinical grounds alone and reassessment of the
pituitary function document the following: hypothalamic-pituitary axis can be performed
Serum cortisol = 16 µg/dL (SI: 440 nmol/L) outside of the setting of acute illness where
Serum total testosterone = 173 ng/dL testing and clinical assessment is likely to be more
(230-865 ng/dL) (SI: 6 nmol/L [8-30 nmol/L]) relevant to long-term therapy considerations
LH = 2.0 mIU/mL (1.0-10.0 mIU/mL) (SI: 2.0 IU/L
[1.0-10.0 IU/L]) This patient needs standard fluid resuscitation,
Free T4 = 0.9 ng/dL (0.8-1.6 ng/dL) (SI: 11 pmol/L antibiotics, and supportive measures. His serum
[10-20 pmol/L]) cortisol measurement does not indicate severe
Free T3 = 1.8 pg/mL (2.0-3.5 pg/mL) (SI: 2.8 pmol/L
[3.1-5.4 pmol/L] adrenal insufficiency; however, he has community-
TSH = 0.2 mIU/L (0.4-4.0 mIU/L) acquired pneumonia requiring hospitalization.
Low TSH and free T3 may reflect illness, and
levothyroxine is not indicated. Increased cortisol
Which of the following statements is true? metabolism may result from higher doses if used.
A. The serum cortisol is adequate; administration Dexamethasone use was too remote to influence
of hydrocortisone is not indicated serum cortisol. Low testosterone may reflect
B. Thyroid hormone replacement is required, illness and does not necessarily indicate a defect
and this may reduce cortisol concentrations of hypothalamic-pituitary-gonadal function.
through increased cortisol metabolism Hydrocortisone at a dosage less than 400 mg
C. The previous dexamethasone administration daily or the equivalent for 5 to 7 days is suggested
may explain the relatively low cortisol in the for community-acquired pneumonia based on
setting of acute respiratory distress syndrome results from 13 trials, 12 of which showed reduced
mortality. Most of the trials also showed reduced
D. The low testosterone level makes structural
ventilator use and reduced incidence of acute
hypothalamic-pituitary injury from trauma
respiratory distress syndrome. The principal
a likely diagnosis, hence reinforcing the
adverse effect was more frequent hyperglycemia.
likelihood that the cortisol response to illness
is likely to be inadequate

References
1. Bornstein SR, Allolio B, Arlt W, et al. Diagnosis and treatment of primary 3. Boonen E, Vervenne H, Meersseman P, et al. Reduced cortisol
adrenal insufficiency: an Endocrine Society clinical practice guideline. J Clin metabolism during critical illness. N Engl J Med. 2013;368(16):1477-1488.
Endocrinol Metab. 2016;101(2):364-389. PMID: 26760044 PMID: 23506003
2. Marik PE, Pastores SM, Annane D et al. Recommendations for the 4. Tomlinson JW, Moore J, Cooper MS, et al. Regulation of the expression
diagnosis and management of corticosteroid insufficiency in critically ill of 11beta-hydroxysteroid dehydrogenase type 1 in adipose tissue: tissue-
adult patients: consensus statements from an international task force by the specific induction by cytokines. Endocrinology. 2001;142(5):1982-1989.
American College of Critical Medicine. Crit Care Med. 2008;36(8):1937-1949. PMID: 11316764
PMID: 1896365

70  ENDO 2021 • Endocrine Case Management

chased by Dr. Khalid H. Seedahmed, Jr., MRCP PGDip, CertEndocrinology SA - ID 283


5. Ledderose C, Mohnle P, Limbeck E, et al. Corticosteroid resistance in sepsis 21. Bellissant E, Annane D. Effect of hydrocortisone on phenylephrine--mean
is influenced by microRNA-124-induced downregulation of glucocorticoid arterial pressure dose-response relationship in septic shock. Clin Pharmacol
receptor-α. Crit Care Med. 2012;40(10):2745-2753. PMID: 22846781 Ther. 2000;68(3):293-303. PMID: 11014411
6. Annane D, Pastores SM, Arlt W, et al. Critical illness-related corticosteroid 22. Annane D, Pastores SM, Rochwerg B, et al. Guidelines for the diagnosis
insufficiency (CIRCI): a narrative review from a multispeciality task force of and management of critical illness related corticosteroid insufficiency
the Society of Critical Care Medicine (SCCM) and the European Society of (CIRCI) in critically ill patients (Part 1): Society of Critical Care Medicine
Intensive Care Medicine (ESICM). Crit Care Med. 2017;45(12):2089-2098. (SCCM) and European Society of Intensive Care Medicine (ESICM) 2017.
PMID: 28938251 2017;45(12):2078-2088. PMID: 28938253
7. Tominaga T, Fukata J, Naito Y, et al. Effects of corticostatin-I on rat adrenal 23. Annane D, Sebille V, Charpentier C, et al. Effect of treatment with low doses
cells in vitro. J Endocrinol. 1990;125(2):287-292. PMID: 2165121 of hydrocortisone and fludrocortisone on mortality in patients with septic
8. American College of Chest Physicians/Society of Critical Care Medicine shock. JAMA. 2002;283(7):862-871. PMID: 12186604
Consensus Conference Committee. American College of Chest Physicians/ 24. Anonymous. Corticosteroids for patients with septic shock. JAMA.
Society of Critical Care Medicine Consensus Conference Committee: 2003;289(1):41-44.
definitions for sepsis and organ failure and guidelines for the use of 25. Sprung CL, Annane D, Keh D, et al; CORTICUS Study Group.
innovation therapies in sepsis. Crit Care Med. 1992;20(6):864-874. Hydrocortisone therapy for patients with septic shock. N Engl J Med.
PMID:1597042 2008;358(2):111-124. PMID: 18184957
9. Singer M, Deutschman CS, Seymour CW, et al. The third international 26. Mason PE, Al-Khafaji A, Milbrandt EB, Suffoletto BP, Huang DT.
consensus defiintions for sepsis and septic shock (Sepsis-3). JAMA. CORTICUS: the end of unconditional love for steroid use. Crit Care.
2016;315(8):801-810. PMID: 26903338 2009;13(4):309. PMID: 19691813
10. Annane D, Bellissant E, Bollaert PE, et al. Corticosteroids in the treatment 27. Heming N, Sivavandamoorthy S, Meng P, Bounab R, Annane D.
of severe sepsis and septic shock in adults: a systematic review. JAMA. Immune effects of corticosteroids in sepsis. Front Immunol. 2018;9:1736.
2009;301(22):2362-2375. PMID: 19509383 PMID: 30105022
11. Sligl WI, Milner DA Jr, Sundar S, Mphatswe W, Majumdar SR. Safety and 28. Annane D, Bellissant E, Bollaert PE, Briegel J, Keh D, Kupfer Y.
efficacy of corticosteroids for the treatment of septic shock: a systematic Corticosteroids for treating sepsis. Cochrane Database Syst Rev.
review and meta-analysis. Clin Infect Dis. 2009;49(1):93-101. PMID: 19489712 2015;12:CD002243. PMID: 26633262
12. Minneci PC, Deans KJ, Banks SM, Eichacker PQ, Natanson C. Meta-analysis: 29. Venkatesh B, Finfer S, Cohen J, et al; ADRENAL trial investigators
the effect of steroids on survival and shock during sepsis depends on the dose. and the Australian--New Zealand Intensive Care Society Trials Group.
Ann Intern Med. 2004;141(1):47-56. PMID: 15238370 Adjunctive glucocorticoid therapy in patients with septic shock. N Engl J Med.
13. Keh D, Boehnke T, Weber-Cartens S, et al. Immunologic and hemodynamic 2018;378(9):797-808. PMID: 29347874
effects of “low-dose” hydrocortisone in septic shock: a double-blind, 30. Annane D, Renault C, Brun-Buisson B, et al. Hydrocortisone plus
randomized, placebo-controlled, crossover study. Am J Respir Crit Care Med. fludrocortisone for adults with septic shock. N Engl J Med. 2018;378(9):809-
2003;167(4):512-520. PMID: 12426230 818. PMID: 29490185
14. Ho JT, Al-Musalhi H, Chapman MJ, et al. Septic shock and sepsis: a 31. Ballard PL. Delivery and transport of glucocorticoids to target cells. Monogr
comparison of total and free plasma cortisol levels. J Clin Endocrinol Metab. Endocrinol. 1979;12:25-48. PMID: 386085
2006;91(1):105-114. PMID: 16263835 32. Pemberton PA, Stein PE, Pepys MB, Potter JM, Carrell RW. Hormone
15. Sam S, Corbridge TC, Mokhlesi B, Comellas AP, Molitch ME. Cortisol binding globulins undergo serpin conformational change in inflammation.
levels and mortality in severe sepsis. Clin Endocrinol (Oxf). 2004;60(1):29-35. Nature. 1988;336(6196):257-258. PMID: 314075
PMID: 14678284 33. Nenke MA, Rankin W, Chapman MJ, et al. Depletion of high-affinity
16. Rothwell PM, Udwadia ZF, Lawler PG. Cortisol response to corticotropin corticosteroid-binding globulin corresponds to illness severity in sepsis and
and survival in septic shock. Lancet. 1991;337(8741):582-583. PMID: 1671944 septic shock: clinical implications. Clin Endocrinol (Oxf). 2015;82(6):801-807.
17. Briegel J, Forst H, Haller M, et al. Stress doses of hydrocortisone reverse PMID: 25409953
hyperdynamic septic shock: a prospective, randomized, double-blind, single- 34. Meyer EJ, Nenke MA, Rankin W, et al. Total and high affinity corticosteroid-
center study. Crit Care Med. 1999;27(4):723-732. PMID: 10321661 binding globulin depletion in septic shock is associated with mortality. Clin
18. Oppert M, Schindler R, Husung C, et al. Low-dose hydrocortisone improves Endocrinol (Oxf). 2019;90(1):232-240. PMID: 30160799
shock reversal and reduces cytokine levels in early hyperdynamic septic 35. Pastores SM, Annane D, Rochwerg B; Corticosteroid Guideline Task Force
shock. Crit Care Med. 2005;33(11):2457-2464. PMID: 16276166 of SCCM and ESICM. Guuidelines for the diagnosis and management of
19. Annane D, Sebille V, Troche G, Raphael JC, Gajdos P, Bellissant E. A 3-level critical illness related corticosteroid insufficiency (CIRCI) in critically ill
prognostic classification in septic shock based on cortisol levels and cortisol patients (Part II): Society of Critical Care Medicine (SCCM) and European
response to corticotropin. JAMA. 2000;283(8):1038-1045. PMID: 10697064 Society of Intensive Care Medicine (ESICM) 2017. 2018;46(1):146-148.
20. Annane D, Bellissant E, Sebille V, et al. Impaired pressor sensitivity to PMID: 29095205
noradrenaline in septic shock patients with and without impaired adrenal 36. Annane D. Why my steroid trials in septic shock were “positive.” Crit Care
function reserve. Br J Clin Pharmacol. 1998;46(6):589-597. PMID: 9862249 Med. 2019;47(12):1789-1793. PMID: 31259754

ENDO 2021 • Adrenal  71

chased by Dr. Khalid H. Seedahmed, Jr., MRCP PGDip, CertEndocrinology SA - ID 283


chased by Dr. Khalid H. Seedahmed, Jr., MRCP PGDip, CertEndocrinology SA - ID 283
BONE AND MINERAL
METABOLISM

chased by Dr. Khalid H. Seedahmed, Jr., MRCP PGDip, CertEndocrinology SA - ID 283


Evaluation of the Patient
With Hypophosphatemia
Peter J. Tebben, MD. Division of Endocrinology, Diabetes, Metabolism, and Nutrition,
Department of Internal Medicine and Department of Pediatrics and Adolescent Medicine,
Mayo Clinic, Rochester, MN; E-mail: tebben.peter@mayo.edu

Learning Objectives antibody therapy for those with X-linked


hypophosphatemic rickets.
As a result of participating in this session, learners
should be able to: • Patients treated with active vitamin D and
phosphorus require close monitoring to
• Identify clinical manifestations of acute and avoid complications of therapy including
chronic hypophosphatemia. secondary/tertiary hyperparathyroidism and
• Explain phosphate homeostasis and identify nephrocalcinosis/nephrolithiasis.
an approach for evaluation of patients with
hypophosphatemia.
• Describe therapeutic options for the
management of various causes of
Significance of the
hypophosphatemia. Clinical Problem
Hypophosphatemia is a common clinical
abnormality identified in hospitalized patients and
has been associated with poor outcomes.1 Chronic
Main Conclusions hypophosphatemia is less common and requires a
systematic approach to determine the underlying
• Measure serum phosphate and apply the age- etiology and formulate an effective treatment
appropriate reference range when evaluating plan. The biological role of phosphorus is diverse,
patients with unexplained musculoskeletal and it is essential for the proper function of a
complaints. multitude of systems, including having a critical
• History and limited laboratory evaluation role in skeletal mineralization.2 Disorders that lead
can establish the underlying etiology of to chronic hypophosphatemia are associated with
hypophosphatemia. abnormal mineralization and manifest as rickets in
children or osteomalacia in adults.3,4
• Treatment for hypophosphatemia due to poor
Symptoms of acute hypophosphatemia can
intake or malabsorption includes phosphate
be nonspecific and generally reflect the severity
supplementation, dietary changes, and/
and acuity of the decline of the serum phosphate
or treatment of underlying malabsorption
concentration.5 Muscle weakness and fatigue
disorders.
are common symptoms, particularly in those
• Treatment options for hypophosphatemia with acquired hypophosphatemia. Cardiac and
due to renal phosphate wasting includes respiratory dysfunction, as well as rhabdomyolysis
active vitamin D and phosphorus replacement and hemolysis, may occur with severe, acute
or anti-fibroblast growth factor 23 hypophosphatemia.5 However, patients with

74  ENDO 2021 • Endocrine Case Management

chased by Dr. Khalid H. Seedahmed, Jr., MRCP PGDip, CertEndocrinology SA - ID 283


chronic hypophosphatemia due to heritable causes adult reference range is applied to the pediatric
such as X-linked hypophosphatemic rickets (XLH) population.
generally do not experience these severe symptoms • Understanding the goals and potential
and typically present with skeletal manifestations adverse effects of therapy for chronic
of rickets and osteomalacia. Serum phosphorus is hypophosphatemia are not always appreciated
an important mineral that is frequently overlooked and can lead to adverse outcomes.
in the evaluation of patients with musculoskeletal
complaints, which can lead to significant delays in
diagnosis and treatment.6,7
Strategies for Diagnosis,
Therapy, and/or Management
Barriers to Optimal Practice The majority of total body phosphate is found
in the skeleton, with the remainder distributed
• Symptoms of hypophosphatemia can
in other tissues and the extracellular space.
be nonspecific and serum phosphate
Many factors affect intestinal absorption and
concentrations are not part of routine
renal reabsorption of phosphorus that influence
chemistry panels. Therefore, the diagnosis
concentrations measured in the in the blood
of hypophosphatemic conditions may go
(Figure 1). The major hormones involved in
unrecognized or delayed.
phosphate homeostasis include 1α-25(OH)2D3
• Serum phosphate concentrations are and PTH. However, phosphaturic peptides such
significantly higher in infants and younger as fibroblast growth factor 23 (FGF-23) also
children than in adults. As a result, have an important role in disorders of phosphate
hypophosphatemia may be overlooked if the metabolism and this has provided a target for

Figure 1. Major Factors in Phosphate Homeostasis

Intake Phosphate Homeostasis

Absorption
Extracellular Formation

Pool
Excretion Resorption

Reabsorption Filtered
NaPi 2b

NaPi 2a
PTH NaPi 2c FGF23
Increase urine phos
Increase urine phos
Increase 1,25(OH)2D
Decrease 1,25(OH)2D
Bone resorption

Urine Phosphorus excretion

ENDO 2021 • Bone and Mineral Metabolism  75

chased by Dr. Khalid H. Seedahmed, Jr., MRCP PGDip, CertEndocrinology SA - ID 283


therapeutic intervention in patients with excess These 3 categories provide a useful framework
FGF-23 due to XLH.8-12 for evaluation and management of patients with
Hypophosphatemia is defined as a serum hypophosphatemia (Figure 2).
phosphate concentration below the lower
end of normal and is dependent on age. The Decreased Intake and/or
normal range of serum phosphate in adults is
approximately 2.5 to 4.5 mg/dL (0.8-1.5 mmol/L).
Malabsorption
Infants and children have significantly higher Intestinal absorption of phosphorus is largely
serum phosphate concentrations than adults. It is dependent on the amount of phosphorus
imperative to use the appropriate reference range consumed. Absorption takes place primarily in the
in this younger population or the diagnosis of proximal small bowel with both active and passive
hypophosphatemia may be overlooked. mechanisms.13 Decreased intake or malabsorption
Hypophosphatemia can be divided into 3 main should be considered in patients with primary
categories (Table): malabsorption disorders such as inflammatory
bowel disease or short gut syndrome. In addition,
1. Decreased intake and/or malabsorption patients receiving phosphate binders such as
2. Redistribution within the body calcium, sevelamer, or lanthanum carbonate
3. Renal phosphate wasting can develop hypophosphatemia due to reduced
intestinal phosphate availability. Metabolic
• FGF-23 mediated bone disease (fractures and/or rickets) has been
• Non–FGF-23 mediated identified in some children receiving an elemental-
based infant and child formula thought to be
ѓ Hyperparathyroidism related to low phosphate bioavailability.14,15
ѓ Other

Table. Conditions Associated with Hypophosphatemia

Low Intake or Impaired Absorption Redistribution Renal Phosphate Wasting

Low phosphate intake • Insulin therapy for PTH mediated


diabetic ketoacidosis
Vitamin D deficiency • Hyperparathyroidism
• Refeeding syndrome
• Nutritional • Vitamin D deficiency
• Acute respiratory
• 1α-hydroxylase deficiency ѓ Nutritional
alkalosis
• Vitamin D receptor pathogenic variant ѓ 1α-hydroxylase deficiency
• Hungry bone syndrome
Malabsorption ѓ Vitamin D receptor pathogenic variant
• Inflammatory bowel disease FGF-23 mediated
• Short bowel syndrome • X-linked hypophosphatemic rickets
• Chronic diarrhea • Autosomal dominant hypophosphatemic rickets
Phosphate binders • Fibrous dysplasia
• Calcium • Tumor-induced osteomalacia
• Sevelamer • Iron infusions
• Lanthanum carbonate • DMP1 pathogenic variant
• ENPP1 pathogenic variant
Non–FGF-23 mediated
• Renal Fanconi syndrome
• Renal sodium-phosphate cotransporter
pathogenic variant

76  ENDO 2021 • Endocrine Case Management

chased by Dr. Khalid H. Seedahmed, Jr., MRCP PGDip, CertEndocrinology SA - ID 283


Laboratory Assessment diet or reducing/discontinuing phosphate binders
Figure 2 outlines an approach to the clinical will resolve the hypophosphatemia. Patients
investigation of patients with hypophosphatemia. with malabsorption due to severe inflammatory
Most patients can be categorized based on widely bowel disease or short bowel syndrome may
available tests, including measurement of serum require phosphate supplementation if the
calcium, phosphate, PTH, creatinine, and vitamin underlying gastrointestinal pathology cannot
D metabolites. Urinary calcium, phosphate, be fully corrected. Caution must be exercised in
and creatinine should also be measured. Low this group, and a low initial dose of phosphorus
urine phosphate with a low fractional excretion should be used to avoid worsening gastrointestinal
of phosphate (less than 5%) is a hallmark of symptoms. Stomach upset and diarrhea are the
hypophosphatemia caused by inadequate intake most common adverse effects of phosphate
or malabsorption. Elevated 1α,25(OH)2D3 supplementation.
concentration is also expected in this category
of patients. Redistribution
Treatment Hypophosphatemia due to redistribution is
Treatment for this group of patients is primarily typically apparent based on history alone and
directed at identification and correction of the generally does not require extensive evaluation to
underlying pathology. If dietary restriction or establish the etiology. Diabetic ketoacidosis (DKA)
phosphate binders are the culprit, adjusting the and severe malnutrition are states of whole-body

Figure 2. Algorithm for Evaluation of Patients With Hypophosphatemia

Low Serum Phosphorus

History Redistribution

Assess urine phosphorus

Low Urine Phos High urine phos


(TmP/GFR) (TmP/GFR)

Normal PTH High PTH

High
Low/Nl 1,25(OH)2D
1,25(OH)2D

Not enough in… Non-FGF23


FGF23 mediated PTH/Ca/Vit D disorder
mediated

ENDO 2021 • Bone and Mineral Metabolism  77

chased by Dr. Khalid H. Seedahmed, Jr., MRCP PGDip, CertEndocrinology SA - ID 283


phosphate depletion. Serum phosphate is phosphate is required, serum calcium must be
often normal before initiation of treatment closely monitored due to the risk of hypocalcemia.
for these conditions. However, when insulin Although hypophosphatemia occurring with
is administered to patients with DKA or when treatment of DKA can be severe, it is not clear that
feeding is introduced in malnourished patients, replacement improves outcomes.18 Despite this,
cellular uptake of phosphorus ensues, causing many DKA protocols include potassium phosphate
hypophosphatemia that is sometimes severe. in the intravenous fluid therapy.
Hungry bone syndrome is a consequence of Treatment of hungry bone syndrome is
calcium and phosphate uptake into bone following focused primarily on correction of hypocalcemia.
surgery for primary hyperparathyroidism. Active vitamin D metabolites can be helpful not
Although most patients do not experience hungry only to address hypocalcemia, but they also have a
bone syndrome following parathyroidectomy, salutary effect on serum phosphate concentration
larger parathyroid adenoma volume, higher via increased intestinal absorption. Phosphorus
alkaline phosphatase, and older age are risk factors administration will exacerbate hypocalcemia and
for this condition.16 should only be considered in patients with severe
Acute respiratory alkalosis can cause transient hypophosphatemia.
hypophosphatemia due to transcellular shifts,
is self-limited, and does not require phosphate Renal Phosphate Wasting
replacement therapy.
Renal phosphate wasting is caused by a wide
Laboratory Assessment variety of conditions (Table). These disorders
Limited laboratory testing is needed to can be further divided into FGF-23–mediated
establish the cause of hypophosphatemia due to and non–FGF-23–mediated disease. Renal
redistribution. However, management of these reabsorption of phosphorus is sodium-
patients may require more extensive laboratory dependent and is mediated by sodium-phosphate
monitoring. In addition to hypophosphatemia, cotransporters (NaPi IIa and NaPi IIc).19,20
refeeding syndrome and treatment of DKA are PTH and FGF-23 both inhibit renal phosphate
associated with other electrolyte abnormalities reabsorption via reduced sodium-phosphate
that require monitoring and replacement. Vitamin transporter abundance in the proximal tubule.20
D status should be evaluated in all malnourished Vitamin D deficiency can lead to
patients. Patients with hungry bone syndrome hypophosphatemia via 2 mechanisms: decreased
require close monitoring of serum calcium, intestinal absorption and increased urinary output
phosphate, and magnesium, as well as assessment due to the phosphaturic effect of secondary
for vitamin D deficiency. hyperparathyroidism that often ensues in a state of
vitamin D deficiency.
Treatment
Patients with severe malnutrition require a team Laboratory Assessment
experienced with refeeding syndrome to identify Measurement of serum calcium, phosphate,
those at risk and to implement strategies to PTH, creatinine, and vitamin D metabolites,
minimize the ensuing electrolyte disturbances. as well as urinary calcium, phosphate, and
Correction of baseline electrolyte abnormalities creatinine narrows the differential diagnosis
before initiating the refeeding process is substantially. Although FGF-23 concentrations
recommended.17 Oral phosphate replacement is can be measured, the assay is clinically available
preferred when tolerated. Intravenous phosphate in a limited number of reference laboratories.
replacement is reserved for patients with severe However, patients with hypophosphatemia due
symptomatic hypophosphatemia. If intravenous to renal phosphate wasting have characteristic
biochemical patterns that distinguish

78  ENDO 2021 • Endocrine Case Management

chased by Dr. Khalid H. Seedahmed, Jr., MRCP PGDip, CertEndocrinology SA - ID 283


FGF-23–mediated from non–FGF-23–mediated bone pain, and healing/prevention of fractures.
disease: It should be noted that not all adults with XLH
require pharmacologic intervention.
• FGF-23 mediated: Treatment with calcitriol and phosphate
ѓ Normal serum calcium supplementation requires vigilant monitoring
to avoid complications such as secondary/
ѓ Normal or low urinary calcium
tertiary hyperparathyroidism, hypercalciuria,
ѓ Low or inappropriately normal nephrocalcinosis, and nephrolithiasis. A
1α,25(OH)2D3 comprehensive review of conventional therapy
for patients with XLH has previously been
• Non–FGF-23 mediated
published.24
ѓ Normal or elevated serum calcium Burosumab treatment results in reduced renal
ѓ Elevated 1α,25(OH)2D3 phosphate wasting and increased 1α,25(OH)2D3
concentrations. Burosumab is given as a
ѓ Hypercalciuria subcutaneous injection every 2 weeks in children
and every 4 weeks in adults and improves
Treatment radiographic changes of rickets in children
FGF-23–Mediated Disease and improves healing of fractures in adults.11,25
Optimal treatment for patients with tumor- Monitoring of serum phosphate is recommended
induced osteomalacia includes identification with the dosage adjusted to maintain serum
of the offending tumor and surgical removal. phosphate within the age-appropriate reference
Tumors are notoriously difficult to locate and range. Burosumab has not been approved for
often require extensive advanced whole-body treatment of other FGF-23–mediated conditions.
imaging techniques.6 After surgical intervention,
patients with tumor-induced osteomalacia require PTH-Mediated Disease
ongoing monitoring for recurrence. When tumor Treatment of primary hyperparathyroidism is
location is elusive or not resectable, calcitriol and surgical. Secondary hyperparathyroidism is most
phosphate supplementation is required. Although commonly caused by vitamin D and or calcium
not part of usual treatment, improved phosphate deficiency. Treatment is focused on replacement
balance with use of the calcimimetic agent of vitamin D and optimizing calcium intake,
cinacalcet has been described in small numbers which will lead to correction of the secondary
of patients with FGF-23–mediated disease hyperparathyroidism and restoration of phosphate
(tumor-induced osteomalacia and XLH).21-23 The balance. Secondary hyperparathyroidism
presumed mechanism is reduced PTH-mediated related to genetic abnormalities in vitamin D
phosphaturia. production (1α-hydroxylase deficiency) or action
Until recently, the most effective treatment (vitamin D receptor pathogenic variant) is rare.
for XLH was calcitriol and phosphate 1α-Hydroxylase deficiency is treated with active
supplementation. Phosphate must be given in 3 vitamin D metabolites and calcium. Therapy in
to 5 divided doses. Calcitriol can be given daily or patients with a vitamin D receptor pathogenic
twice daily. More recently, anti-FGF-23 antibody variant is more challenging and requires high-
therapy (burosumab) has become available and has dosage calcium supplementation.
reduced the treatment burden. Treatment goals in
children with XLH include improvement in rickets Other
(radiographic and biochemical), improved linear Patients with renal Fanconi syndrome or
growth, straightening of legs, and avoidance of the rare patient with a sodium-phosphate
complications due to therapy. In adults, treatment cotransporter defect are treated with phosphate
goals include healing of osteomalacia, reduced

ENDO 2021 • Bone and Mineral Metabolism  79

chased by Dr. Khalid H. Seedahmed, Jr., MRCP PGDip, CertEndocrinology SA - ID 283


supplementation. Because these are not FGF-23– Which of the following is the
mediated conditions, calcitriol is not indicated as best initial management?
it would worsen the hypercalciuria and lead to A. Restart phosphorus supplementation to
nephrocalcinosis and nephrolithiasis. correct the hypophosphatemia and reduce
serum calcium
Clinical Case Vignettes B. Continue furosemide to prevent worsening
hypercalcemia
Case 1
C. Order genetic testing for multiple endocrine
A 21-year-old woman with XLH presents for neoplasia given her hyperparathyroidism at a
evaluation of recent-onset hypercalcemia. She young age
was diagnosed with XLH at age 2 years based
D. Consult a parathyroid surgeon
on typical radiographic findings and laboratory
abnormalities. She had been treated with Answer: D) Consult a parathyroid surgeon
calcitriol, 0.5 mcg twice daily, and phosphorus,
5 g daily in divided doses. She presented This patient has developed tertiary
to her local emergency department with a hyperparathyroidism as a result of overzealous
chief concern of abdominal pain. Laboratory phosphate supplementation. Hyperparathyroidism
studies at that time identified hypercalcemia from any cause will worsen the renal phosphate
(13.2 mg/dL [3.3 mmol/L]) with an elevated wasting and contribute to hypophosphatemia.
PTH concentration. Calcitriol and phosphorus Surgery is therefore indicated (Answer D).
supplements were discontinued and furosemide Restarting phosphorus supplementation before
was initiated. Her medical history is notable addressing her hyperparathyroidism (Answer A)
for nephrolithiasis with nephrocalcinosis and would only exacerbate the hyperparathyroidism.
abscessed teeth. Her laboratory studies 1 month Furosemide (Answer B) should only be used
after discontinuation of phosphorus and calcitriol transiently in treating severe hypercalcemia and
are shown: would be inappropriate for long-term treatment.
In addition, a loop diuretic would worsen her
Calcium = 11.2 mg/dL (8.2-10.2 mg/dL) hypercalciuria and nephrocalcinosis.
(SI: 2.8 mmol/L [2.1-2.6 mmol/L])
Phosphate = 2.2 mg/dL (2.5-4.5 mg/dL)
(SI: 0.7 mmol/L [0.8-1.5 mmol/L]) Case 1 (Continued)
Creatinine = 0.8 mg/dL (0.6-1.1 mg/dL)
(SI: 70.2 µmol/L [53.0-97.2 µmol/L]) The patient’s calcium and PTH concentrations
25-Hydroxyvitamin D = 35 ng/mL (30-80 ng/mL normalize after 3 and 1/2 gland parathyroidectomy.
[optimal]) (SI: 87.4 nmol/L [74.9-199.7 nmol/L]) She has completed linear growth, has no recent
1,25-Dihydroxyvitamin D = 46 pg/mL (16-65 pg/mL) fractures, and has no bone pain.
(SI: 119.6 pmol/L [41.6-169.0 pmol/L])
Bone-specific alkaline phosphatase = 13 µg/L
(<22 µg/L) Which of the following would be
PTH = 244 pg/mL (10-65 pg/mL) (SI: 244 ng/L the best treatment of her XLH?
[10-65 ng/L]) A. Resume phosphate and calcitriol therapy
Urinary calcium = 380 mg/24 h (100-300 mg/24 h)
(SI: 9.5 mmol/d [2.5-7.5 mmol/d]) B. Start burosumab, 1 mg/kg every 4 weeks
C. Observe without specific bone targeted
therapy
Answer: A, B, or C) Resume phosphate and calcitriol
therapy, or start burosumab, 1 mg/kg every 4 weeks,
or observe without specific bone targeted therapy

80  ENDO 2021 • Endocrine Case Management

chased by Dr. Khalid H. Seedahmed, Jr., MRCP PGDip, CertEndocrinology SA - ID 283


All 3 options could be considered. Not all adults Case 2
with XLH are symptomatic and therefore not all
A 40-year-old man presents for evaluation of
require pharmacologic intervention. After linear
progressive pelvic, lower-extremity, back, and
growth is completed, it is difficult to predict
rib pain. Imaging identifies several rib fractures
which patients will benefit from ongoing therapy.
and stress fractures of his pelvis and bilateral
Significant clinical variability exists even within
femur. Bone density assessment reveals T-scores
the same family.
in the osteoporosis range. He is of average height
and weight with no lower-extremity deformity.
Case 1 (Continued) Examination findings are notable for an antalgic
The patient elects to remain off therapy due to gait, muscle weakness, and a 2.5 × 3-cm firm
previous adverse effects and lack of symptoms of lesion at the base of his neck on his left shoulder.
hypophosphatemia or osteomalacia. She returns to He has no family history of bone disorders. His
clinic 2 years later with multiple fractures after a medications include calcium, 500 mg once daily;
fall on ice. She develops 2 abscessed teeth and mild vitamin D, 5000 IU once daily; and over-the-
pain related to enthesopathy and osteoarthritis counter pain medication.
in her hips. Radiographs identify pseudofractures Initial laboratory test results:
in both femurs. She elects to reinitiate treatment
with burosumab. Calcium = 9.7 mg/dL (8.2-10.2 mg/dL)
(SI: 2.4 mmol/L [2.1-2.6 mmol/L])
Creatinine = 0.9 mg/dL (0.7-1.3 mg/dL)
Which of the following is expected to occur (SI: 79.6 µmol/L [53.0-97.2 µmol/L])
as a result of therapy with anti-FGF-23 25-Hydroxyvitamin D = 69 ng/mL
antibody treatment (burosumab)? (30-80 ng/mL [optimal]) (SI: 172.2 nmol/L
A. Reduced bone pain and healing of her [74.9-199.7 nmol/L])
Phosphate = 1.8 mg/dL (2.5-4.5 mg/dL)
pseudofractures
(SI: 0.6 mmol/L [0.8-1.5 mmol/L])
B. Improvement in pain related to osteoarthritis Bone-specific alkaline phosphatase = 2-fold upper
C. Improvement in pain related to enthesopathy normal limit
PTH = 69 pg/mL (10-65 pg/mL) (SI: 69 ng/L
D. Reduction in frequency of abscessed teeth [10-65 ng/L])

Answer: A) Reduced bone pain and


healing of her pseudofractures Which of the following is the best next step
in this patient’s evaluation and management?
A reasonable expectation from treatment with
anti-FGF-23 antibody therapy (or conventional A. Initiate alendronate, 70 mg once weekly, to
therapy) includes healing of pseudofractures treat his low bone density and fractures
and reduced bone pain related to osteomalacia B. Refer to medical genetics to assess for heritable
(Answer A). Currently, there is no evidence that hypophosphatemic disorders
anti-FGF-23 antibody therapy improves pain C. Obtain urine studies to assess tubular
associated with osteoarthritis (Answer B) or reabsorption of phosphorus
enthesopathy (Answer C) or reduces the frequency D. Perform parathyroid scan due to possible
of dental abscess (Answer D). In one randomized eucalcemic primary hyperparathyroidism
controlled trial, burosumab-treated children with
E. Schedule a tetracycline-labeled bone biopsy
XLH developed dental abscess more frequently
than those treated with conventional therapy.26 Answer: C) Obtain urine studies to assess
tubular reabsorption of phosphorus

ENDO 2021 • Bone and Mineral Metabolism  81

chased by Dr. Khalid H. Seedahmed, Jr., MRCP PGDip, CertEndocrinology SA - ID 283


This patient has clinical and biochemical evidence Which of the following is the best next
of osteomalacia with low serum phosphate, step in this patient’s management?
fractures, and elevated alkaline phosphatase. His A. Initiate treatment with calcitriol and
low bone density is related to osteomalacia and phosphorus supplementation
not osteoporosis. Assessing urine phosphate B. Start anti-FGF-23 antibody therapy (burosumab)
status (Answer C) is the most important next and monitor serum phosphate concentration
step in identifying the underlying cause of his
C. Perform whole-body imaging such as FDG-
hypophosphatemia. The mild elevation in PTH
PET or 68Ga DOTATATE PET scan
with normal serum calcium would not be expected
to result in this degree of hypophosphatemia and D. Perform biopsy of skin lesion at the base of his
alkaline phosphatase elevation. neck
Answer: D) Perform biopsy of skin
Case 2 (Continued) lesion at the base of his neck
His fractional excretion of phosphorus is Lesions causing tumor-induced osteomalacia are
inappropriately elevated. Urinalysis does not notoriously difficult to locate and can occur in
reveal excess protein or glucose. You suspect an nearly any part of the body. Cutaneous lesions
FGF-23–mediated process such as tumor-induced causing tumor-induced osteomalacia have been
osteomalacia. described, and this patient has such a lesion at the
base of his neck. Biopsy of the lesion (Answer D)
Which of the following laboratory is the best next step. Excision of this lesion was
parameters is most consistent with not only diagnostic but also therapeutic. Calcitriol
hypophosphatemia due to elevated FGF-23?
and phosphate (Answer A) can be used in
A. Elevated 1,25-dihydroxyvitamin D patients for whom the tumor is not identified or
B. High urinary calcium not resectable. Anti-FGF-23 antibody therapy
C. Low 1,25-dihydroxyvitamin D (Answer B) is not approved for treatment of
D. Elevated serum creatinine tumor-induced osteomalacia. If the cutaneous
lesion was not the offending tumor or if metastatic
Answer: C) Low 1,25-dihydroxyvitamin D disease was suspected, whole-body imaging
(Answer C) would be appropriate.
FGF-23 not only reduces renal phosphate
reabsorption but also inhibits 1α-hydroxylase
activity. This results in low or inappropriately Case 2 (Continued)
normal 1,25-dihydroxyvitamin D concentrations The offending tumor was identified and removed,
(Answer C). Elevated 1,25-dihydroxyvitamin D which led to correction of his serum phosphate,
(Answer A) and hypercalciuria (Answer B) is seen alkaline phosphatase, and osteomalacia.
in non-FGF23–mediated renal phosphate wasting
disorders. Elevated creatinine (Answer D) due to Regarding follow-up, which of the
chronic kidney disease is associated with elevated following is the best recommendation?
FGF-23; however, chronic kidney disease leads to A. Return to clinic if symptoms recur
hyperphosphatemia, not hypophosphatemia. B. Periodic serum phosphate measurement
C. Annual whole-body imaging with FDG-PET
Case 2 (Continued) or 68Ga DOTATATE PET scan
This patient’s history and laboratory studies are D. No follow-up is required after surgical cure
consistent with the diagnosis of tumor-induced
osteomalacia. Answer: B) Periodic serum phosphate measurement

82  ENDO 2021 • Endocrine Case Management

chased by Dr. Khalid H. Seedahmed, Jr., MRCP PGDip, CertEndocrinology SA - ID 283


Patients with tumor-induced osteomalacia should Which of the following is the best next
be monitored proactively after apparent surgical step in this patient’s evaluation?
cure. The best recommendation is periodic A. Perform 68Ga DOTATATE PET scan
serum phosphate measurement (Answer B). B. Measure 1,25-dihydroxyvitamin D
When disease recurs, it is frequently local, but
C. Obtain arterial or venous blood gas
it may also be found at distant sites if malignant
transformation with metastases occurs. Several D. Measure urine phosphorus and creatinine to
years after his initial surgery, this patient determine the fractional excretion of phosphate
developed recurrence of hypophosphatemia with Answer: C) Obtain arterial or venous gas
sarcomatous transformation and metastases.
Hypophosphatemia is often present before This patient experienced symptoms of short
symptoms occur or osteomalacia develops. It duration that are consistent with acute anxiety.
would therefore be inappropriate to only reassess The best next step in her evaluation is to confirm
if symptoms recur. she has an acute respiratory alkalosis with an
arterial or venous blood gas measurement
Case 3  New Content  (Answer C). Acute alkalosis leads to a rapid shift
of phosphate to the intracellular space, thereby
A 27-year-old woman presents to the emergency causing hypophosphatemia that can be severe and
department with shortness of breath, palpitations, symptomatic. Measuring urine phosphorus and
chest tightness, and muscle cramps. Her symptoms creatinine for calculation of the fractional excretion of
started 1 hour ago. Her medical history is notable phosphate (Answer D) would be the appropriate next
for depression treated with fluoxetine. She takes step for patients with chronic hypophosphatemia
no other medications or supplements and has not related to acute intracellular shift. Likewise,
otherwise been healthy. Growth and development 1,25-dihydroxyvitamin D measurement (Answer B)
during childhood were normal, and there is and whole-body imaging (Answer A) are helpful in
no family history of metabolic bone disease or evaluating patients with renal phosphate wasting but
disorders of mineral metabolism. She has no bone would not be useful in managing hypophosphatemia
pain or fracture history. due to acute respiratory alkalosis.
On physical examination, she appears
anxious, with a pulse rate of 114 beats/min and
respiratory rate of 24 breaths/min. The rest of her Case 3 (Continued)  New Content 
examination findings are unremarkable. Venous blood gas reveals a respiratory alkalosis.
Her history and laboratory findings are consistent
Initial laboratory test results: with an acute intracellular shift of phosphate to
Calcium = 9.7 mg/dL (8.6-10.0 mg/dL) the intracellular space.
(SI: 2.4 mmol/L [2.2-2.5 mmol/L])
Creatinine = 0.72 mg/dL (0.59-1.04 mg/dL) Which of the following is the best
(SI: 63.6 µmol/L [52.2-91.9 µmol/L]) next step in management?
25-Hydroxyvitamin D = 14 ng/mL (30-80 ng/mL
[optimal]) (SI: 34.9 nmol/L [74.9-199.8 nmol/L]) A. Administer a benzodiazepine
Phosphate = 1.4 mg/dL (2.4-4.5 mg/dL) B. Treat her vitamin D deficiency with 50,000 IU
(SI: 0.5 mmol/L [0.8-1.5 mmol/L])
cholecalciferol once weekly for 4 to 6 weeks
Total alkaline phosphatase, normal
PTH = 53 pg/mL (10-65 pg/mL) (SI: 53 ng/L C. Change fluoxetine to another selective
[10-65 ng/L]) serotonin reuptake inhibitor
D. Administer intravenous phosphate
Answer: A) Administer a benzodiazepine

ENDO 2021 • Bone and Mineral Metabolism  83

chased by Dr. Khalid H. Seedahmed, Jr., MRCP PGDip, CertEndocrinology SA - ID 283


Treatment of this patient’s anxiety (Answer A) will Case 4  New Content 
reverse her acute respiratory alkalosis and quickly
A 70-year-old man presents with a 7-year history
restore serum phosphate to a normal range.
of musculoskeletal symptoms. Pain originated in his
Documentation of rapid normalization of serum
right knee, then progressed to his left knee, bilateral
phosphate after resolution of the acute alkalosis
hips, and feet. Previously, he had been quite active,
precludes the need for further investigation for
but he now uses an electric scooter and cane as
a chronic hypophosphatemic disorder. Whole-
mobility aids. Multiple fractures and stress fractures
body phosphate depletion is not the cause of
have been documented on radiographs. Numerous
hypophosphatemia and therefore phosphate
investigations did not identify the cause of his pain,
administration (Answer D) is not required. Her
weakness, and fractures until serum phosphate was
moderate vitamin D insufficiency is unlikely to
measured 1 year ago (6 years after symptom onset).
produce this degree of hypophosphatemia, and
Tumor-induced osteomalacia was suspected,
treatment with cholecalciferol (Answer B) is
although imaging findings on an octreotide scan
not needed. In the long term, she may benefit
were unremarkable. MRI identified numerous
from an alternative medication for her anxiety
fractures and a complex fluid collection with
and depression. However, switching to another
synovitis in the suprapatellar bursa of the left leg.
selective serotonin reuptake inhibitor (Answer C)
would not resolve this episode of acute anxiety. Initial laboratory test results:
Calcium = 9.1 mg/dL (8.6-10.0 mg/dL)
Case 3 (Continued)  New Content  (SI: 2.3 mmol/L [2.2-2.5 mmol/L])
Creatinine = 0.72 mg/dL (0.59-1.04 mg/dL)
After treatment, her symptoms improve and her (SI: 63.6 µmol/L [52.2-91.9 µmol/L])
serum phosphate concentration returns to normal. 25-Hydroxyvitamin D = 38 ng/mL (30-80 ng/mL
[optimal]) (SI: 94.8 nmol/L [74.9-199.8 nmol/L])
Which of the following clinical features 1,25-Dihydroxyvitamin D = 39 pg/mL (18-64 pg/mL)
in this patient were INCONSISTENT with (SI: 101.4 pmol/L [46.8-166.4 pmol/L])
Phosphate = 1.5 mg/dL (2.5-4.5 mg/dL)
a chronic hypophosphatemic disorder?
(SI: 0.5 mmol/L [0.8-1.5 mmol/L])
A. Acute-onset symptoms Total alkaline phosphatase = 298 U/L (45-115 U/L)
B. Normal growth and development as a child (SI: 4.98 µkat/L [0.75-1.92 µkat/L])
PTH = 39 pg/mL (10-65 pg/mL) (SI: 39 ng/L
C. Normal alkaline phosphatase [10-65 ng/L])
D. Lack of bone pain or fractures Carboxyterminal FGF-23 = 632 RU/mL
(<180 RU/mL)
E. All of the above Urinary fractional excretion of phosphorus = 38%
(<5% in the presence of hypophosphatemia)
Answer: E) All of the above
Normal results were documented for urinalysis,
Several clinical and biochemical findings can help
serum bicarbonate, AST, ALT, and serum protein
distinguish between an acute vs longstanding
electrophoresis.
disorder of hypophosphatemia. Chronic
hypophosphatemia leads to chronic symptoms
Which of the following is the best next step
that can have an insidious onset. The absence of in the treatment of this patient’s symptoms?
an elevated alkaline phosphatase concentration
and lack of bone pain or fractures are additional A. Refer to physical therapy and pain clinic
clues that hypophosphatemia is acute or of recent B. Initiate medical therapy with calcitriol and
onset. Signs or symptoms of hypophosphatemia phosphate or anti-FGF-23 antibody therapy
during childhood and/or a family history of C. Start a bisphosphonate
hypophosphatemia, if present, would suggest an D. Start teriparatide
inherited disorder of phosphate metabolism.

84  ENDO 2021 • Endocrine Case Management

chased by Dr. Khalid H. Seedahmed, Jr., MRCP PGDip, CertEndocrinology SA - ID 283


Answer: B) Initiate medical therapy with calcitriol if the tumor can be located and removed. Because
and phosphate or anti-FGF-23 antibody therapy of the wide anatomic distribution of tumors
in patients with tumor-induced osteomalacia,
Physical therapy and pain management whole-body functional imaging (Answer B)
(Answer A) may be helpful in recovery from (eg, FDG-PET, octreotide, 68Ga DOTATATE
tumor-induced osteomalacia but would not address PET, or whole-body sestamibi), followed by
the underlying cause of this patient’s symptoms anatomic imaging (MRI or CT) of any suspicious
(chronic hypophosphatemia and osteomalacia). lesions should be undertaken. A 68Ga DOTATATE
In patients whose tumor is not localized or is not PET scan may be superior to other functional
resectable, medical management should be initiated. imaging for patients with tumor-induced
Options for medical management include calcitriol osteomalacia and should be considered even if
and phosphate replacement or the recently previous imaging was unrevealing. The chronicity
approved anti-FGF-23 antibody burosumab of this patient’s symptoms argues strongly against a
(Answer B). Despite having multiple fractures, transient intracellular shift of phosphorus caused by
this patient does not have osteoporosis. He has acute respiratory alkalosis (Answer C). Symptoms
osteomalacia and therefore a bisphosphonate of hypophosphatemia developed after the patient
(Answer C) is not appropriate and may worsen his was 60 years old, making a genetic cause (Answer A)
disease. Likewise, teriparatide (Answer D) would unlikely. Measurement of the bone fraction of
not be appropriate treatment for osteomalacia and alkaline phosphatase (Answer D) will not refine
may exacerbate renal phosphate loss. the diagnosis or help localize the tumor.

Case 4 (Continued)  New Content 


Case 4 (Continued)  New Content 
Six months of medical therapy with calcitriol and Another left knee MRI to follow-up on the
phosphate supplementation improves but does not previously discovered synovial abnormality
resolve his symptoms. Unhealed fractures remain identifies a suspicious tumor. Biopsy of the lesion
and he is unable to ambulate without assistance. is consistent with a phosphaturic mesenchymal
tumor. With these findings, additional functional
Which of the following is the best next
whole-body imaging is deferred, and he is referred
step in this patient’s management?
for surgical removal.
A. Recommend genetic testing for inherited
hypophosphatemic disorders Which of the following would be
B. Obtain additional functional whole-body expected after complete removal
imaging (eg, 68Ga DOTATATE PET) of the offending tumor?
C. Obtain arterial or venous blood gas to evaluate A. Fractures and bone pain may take several
for acute respiratory alkalosis months to resolve after surgical cure
D. Measure bone-specific alkaline phosphatase to B. Serum phosphate will remain low for several
confirm the cause of his elevated total alkaline weeks after surgery due to “hungry bone
phosphatase syndrome”
C. Medical therapy for persistent hypophosphatemia
Answer: B) Obtain functional whole-body
will be required for several weeks after surgery
imaging (eg, 68Ga DOTATATE PET)
due to the long half-life of FGF-23
After the diagnosis of tumor-induced osteomalacia D. Muscle weakness is irreversible and will persist
is suspected or established, the best next step is to after normalization of serum phosphate
localize the offending tumor. Surgical excision is E. Surgical resection is definitive therapy with no
curative and obviates the need for medical therapy risk of future recurrence

ENDO 2021 • Bone and Mineral Metabolism  85

chased by Dr. Khalid H. Seedahmed, Jr., MRCP PGDip, CertEndocrinology SA - ID 283


Answer: A) Fractures and bone pain may take Hungry bone syndrome may occur after
several months to resolve after surgical cure parathyroidectomy for hyperparathyroidism;
however, serum phosphate typically returns
Although surgical removal is curative, it is to normal within days after surgical cure of
important to set appropriate expectations for tumor-induced osteomalacia (thus, Answer B is
recovery after severe, prolonged osteomalacia. incorrect). Recurrent hypophosphatemia can occur
Muscle weakness can resolve quickly; however, years after apparent surgical cure in some patients
pain and fractures may take weeks to months to with tumor-induced osteomalacia, necessitating
resolve (thus, Answer A is correct and Answer D long-term monitoring after resection (thus,
is incorrect). FGF-23 has a short, not long, half- Answer E is incorrect).
life, and medical therapy after complete resection
is not required (thus, Answer C is incorrect).

References
1. Brunelli SM, Goldfarb S. Hypophosphatemia: clinical consequences and 15. Gonzalez Ballesteros LF, Ma NS, Gordon RJ, et al. Unexpected widespread
management. J Am Soc Nephrol. 2007;18(7):1999-2003. PMID: 17568018 hypophosphatemia and bone disease associated with elemental formula use in
2. Neuman WF. Bone material and calcification mechanisms. In: Urist MR, infants and children. Bone. 2017;97:287-292. PMID: 28167344
ed. Fundamental and Clinical Bone Physiology. Philadelphia, PA: Lippincott; 16. Brasier AR, Nussbaum SR. Hungry bone syndrome: clinical and biochemical
1980:83-107. predictors of its occurrence after parathyroid surgery. Am J Med.
3. Berry JL, Davies M, Mee AP. Vitamin D metabolism, rickets, and 1988;84(4):654-660. PMID: 2400660
osteomalacia. Semin Musculoskelet Radiol. 2002;6(3):173-182. 17. Boateng AA, Sriram K, Meguid MM, Crook M. Refeeding syndrome:
PMID: 12541194 treatment considerations based on collective analysis of literature case
4. Econs MJ, McEnery PT. Autosomal dominant hypophosphatemic rickets/ reports. Nutrition. 2010;26(2):156-167. PMID: 20122539
osteomalacia: clinical characterization of a novel renal phosphate-wasting 18. Fisher JN, Kitabchi AE. A randomized study of phosphate therapy in the
disorder. J Clin Endocrinol Metab. 1997;82(2):674-681. PMID: 9024275 treatment of diabetic ketoacidosis. J Clin Endocrinol Metab. 1983;57(1):177-
5. Imel EA, Econs MJ. Approach to the hypophosphatemic patient. J Clin 180. PMID: 6406531
Endocrinol Metab. 2012;97(3):696-706. PMID: 22392950 19. Murer H, Hernando N, Forster I, Biber J. Molecular aspects in the regulation
6. Boland JM, Tebben PJ, Folpe AL. Phosphaturic mesenchymal tumors: what of renal inorganic phosphate reabsorption: the type IIa sodium/inorganic
an endocrinologist should know. J Endocrinol Invest. 2018;41(10):1173-1184. phosphate co-transporter as the key player. Curr Opin Nephrol Hypertens.
PMID: 29446010 2001;10(5):555-561. PMID: 11496046
7. Hodgson SF, Clarke BL, Tebben PJ, Mullan BP, Cooney WP 3rd, Shives 20. Gattineni J, Bates C, Twombley K, et al. FGF23 decreases renal NaPi-2a and
TC. Oncogenic osteomalacia: localization of underlying peripheral NaPi-2c expression and induces hypophosphatemia in vivo predominantly
mesenchymal tumors with use of Tc 99m sestamibi scintigraphy. Endocr Pract. via FGF receptor 1. Am J Physiol Renal Physiol. 2009;297(2):F282-F291.
2006;12(1):35-42. PMID: 16524861 PMID: 19515808
8. Imel EA, Econs MJ. Fibroblast growth factor 23: roles in health and disease. J 21. Geller JL, Khosravi A, Kelly MH, Riminucci M, Adams JS, Collins MT.
Am Soc Nephrol. 2005;16(9):2565-2575. PMID: 16033853 Cinacalcet in the management of tumor-induced osteomalacia. J Bone Miner
9. Schiavi SC, Kumar R. The phosphatonin pathway: new insights in phosphate Res. 2007;22(6):931-937. PMID: 17352646
homeostasis. Kidney Int. 2004;65(1):1-14. PMID: 14675031 22. Alon US, Levy-Olomucki R, Moore WV, Stubbs J, Liu S, Quarles LD.
10. Berndt TJ, Schiavi S, Kumar R. “Phosphatonins” and the Calcimimetics as an adjuvant treatment for familial hypophosphatemic
regulation of phosphorus homeostasis. Am J Physiol Renal Physiol. rickets. Clin J Am Soc Nephrol. 2008;3(3):658-664. PMID: 18256372
2005;289(6):F1170-F1182. PMID: 16275744 23. Yavropoulou MP, Kotsa K, Gotzamani Psarrakou A, et al. Cinacalcet in
11. Carpenter TO, Whyte MP, Imel EA, et al. Burosumab therapy in children hyperparathyroidism secondary to X-linked hypophosphatemic rickets: case
with X-linked hypophosphatemia. N Engl J Med. 2018;378(21):1987-1998. report and brief literature review. Hormones (Athens). 2010;9(3):274-278.
PMID: 29791829 PMID: 20688626
12. Insogna KL, Briot K, Imel EA, et al; AXLES 1 Investigators. A 24. Carpenter TO, Imel EA, Holm IA, Jan de Beur SM, Insogna KL. A clinician’s
randomized, double-blind, placebo-controlled, phase 3 trial evaluating guide to X-linked hypophosphatemia [J Bone Miner Res. 2015;30(2):394]. J
the efficacy of burosumab, an anti-FGF23 antibody, in adults with Bone Miner Res. 2011;26(7):1381-1388. PMID: 21538511
X-linked hypophosphatemia: week 24 primary analysis. J Bone Miner Res. 25. Portale AA, Carpenter TO, Brandi ML, et al. Continued beneficial effects of
2018;33(8):1383-1393. PMID: 29947083 burosumab in adults with X-linked hypophosphatemia: results from a 24-
13. Hernando N, Wagner CA. Mechanisms and regulation of intestinal week treatment continuation period after a 24-week double-blind placebo-
phosphate absorption. Compr Physiol. 2018;8(3):1065-1090. PMID: 29978897 controlled period. Calcif Tissue Int. 2019;105(3):271-284. PMID: 31165191
14. Creo AL, Epp LM, Buchholtz JA, Tebben PJ. Prevalence of metabolic bone 26. Imel EA, Glorieux FH, Whyte MP, et al. Burosumab versus conventional
disease in tube-fed children receiving elemental formula. Horm Res Paediatr. therapy in children with X-linked hypophosphataemia: a randomised, active-
2018;90(5):291-298. PMID: 30497080 controlled, open-label, phase 3 trial. Lancet. 2019;393(10189):2416-2427.
PMID: 31104833

86  ENDO 2021 • Endocrine Case Management

chased by Dr. Khalid H. Seedahmed, Jr., MRCP PGDip, CertEndocrinology SA - ID 283


Challenging Cases in Bone
and Mineral Disease
Neil J. L. Gittoes, BSc, PhD, FRCP. Centre for Endocrinology, Diabetes and Metabolism,
University of Birmingham and University Hospitals Birmingham, Birmingham, West
Midlands, United Kingdom; E-mail: n.j.gittoes@bham.ac.uk

Learning Objectives Significance of the


As a result of participating in this session, learners Clinical Problem
should be able to: Disorders of bone and mineral include prevalent
• Recognize key clinical and biochemical public health conditions such as osteoporosis and
parameters signifying that a case is challenging vitamin D deficiency, as well as ultra-rare diseases
and requires specific attention. of the skeleton and mineral handling. Although
most clinical presentations of these conditions
• Select appropriate investigations to are subacute and managed through outpatient
differentially diagnose challenging cases in pathways, acute, severe, and life-threatening
bone and mineral metabolism. presentations of mineral homeostasis also occur.
Given the spectrum of prevalence and acuity of
bone and mineral diseases, there is great scope for
challenge to clinicians managing such patients,
Main Conclusions within endocrinology and beyond. Challenging
cases may arise through difficulties in diagnosis
• Pathologies in bone and mineral metabolism and acute or long-term management.
are common and may require a formulaic Recognizing that a case is challenging is the
approach, such as protocolized assessment of first step to addressing that challenge. Some rare
facture risk in osteoporosis. conditions in this arena have a poor evidence base,
• Assessment and management of rarer bone and management can be largely experiential. It is
and mineral diseases, or rare events in important to concentrate experience for such cases
common conditions, require a considered and and to use clinical networks to share best practice
individualized approach. for the benefit of patients. Rapidly expanding
• Evaluation of bone and mineral metabolism developments in genomics, new treatments, and
disorders relies on a limited set of technologies in bone and mineral diseases offer
investigations; with clear understanding and new challenges, as well as opportunities.1,2
appropriate interpretation of these, most
conditions can be reliably diagnosed and Barriers to Optimal Practice
managed.
• Cases are often challenging because of rarity
and lack of objective data and high-quality
evidence. Experience is required to extrapolate
information from analogous clinical situations
but always using the principle of first do

ENDO 2021 • Bone and Mineral Metabolism  87

chased by Dr. Khalid H. Seedahmed, Jr., MRCP PGDip, CertEndocrinology SA - ID 283


no harm. Individualized care can be highly Mineral Disease
variable and opinion based.
Evaluation of mineral metabolism disorders
• Lack of robust discriminatory investigations relies on a limited set of investigations and most
may hinder diagnosis or management conditions can be reliably diagnosed and managed
such as in the case of familial hypocalciuric using these. Serum calcium adjusted for albumin,
hypercalcemia and tumor-induced phosphate, alkaline phosphatase, PTH, and
osteomalacia (localization), respectively. Use of 25-hydroxyvitamin D usually provide pointers
genomic techniques may be engaged later than to underlying disorders of mineral homeostasis.
ideal in the diagnostic odyssey.3 Plasma magnesium measurement and assessment
• Awareness of thresholds of intervention of urinary calcium excretion also provide useful
for treatment and rare adverse effects of additional information.
commonly used drugs may be overlooked, Traditionally, estimating the ratio of renal
potentially causing patient harm, such as clearance of calcium to that of creatinine in the
bisphosphonate-associated atypical femoral fasting state has been central to distinguishing
fractures and rebound vertebral fractures with familial hypocalciuric hypercalcemia (FHH)
denosumab.4 from primary hyperparathyroidism. However,
with genetic panel testing for FHH being more
• Lack of awareness of complexities in cases
readily accessible, this approach can also be used
with multiple comorbidities may promote less
to help distinguish the condition from primary
appropriate prescribing such as inappropriate
hyperparathyroidism.6
use of bisphosphonates in the setting of
Measurement of the active form of vitamin D,
chronic kidney disease–mineral and bone
1,25-dihydroxyvitamin D, is very rarely indicated,
disorder due to low bone turnover.5
but it occasionally can be useful in rare conditions
of extrarenal synthesis of 1,25-dihydroxyvitamin
D, such as sarcoidosis. Similarly, measurement
Strategies for Diagnosis, of PTHrP in serum is very rarely indicated. The
Therapy, and/or Management utility of calcitonin measurement is confined to the
Initial assessment of patients with potential bone diagnosis and monitoring of medullary carcinoma
and mineral disorders should include a detailed of the thyroid. There is no role for calcitonin
history that incorporates early-life developmental measurement in the routine investigation of
milestones and clinical events. Skeletal, dental, calcium and bone metabolism.
soft-tissue, and systemic features should be
explored and particular attention should be paid Bone Disease
to family history of illnesses. Findings on physical
Biochemical markers of bone turnover (resorption
examination may be unremarkable but in some
markers such as CTX and NTX and formation
instances they may offer subtle or even obvious
markers such as P1NP and osteocalcin) may be
clues to an underlying diagnosis. The expression
useful in assessing overall risk of osteoporotic
of bone and mineral pathologies is not confined to
fracture and accelerated bone loss and in judging
musculoskeletal features but can be diverse, cutting
response to treatments for osteoporosis and hence
across all major systems (eg, renal calcification as a
poor adherence to therapy. However, the clinical
manifestation of hypercalciuria, neuropsychiatric
utility of routine measurements of bone turnover
presentations in chronic hypoparathyroidism,
markers is yet to be established.
and blue sclerae in some forms of osteogenesis
Bone imaging using basic skeletal radiology
imperfecta).
is still useful to detect fractures, diagnose specific
diseases such as Paget disease of bone, and identify

88  ENDO 2021 • Endocrine Case Management

chased by Dr. Khalid H. Seedahmed, Jr., MRCP PGDip, CertEndocrinology SA - ID 283


bone dysplasias. Basic x-rays are not useful for disc on the left. Cranial MRI shows no pathology,
assessing bone density, but the appearance of and cerebrospinal fluid examination documents a
“osteopenia” on a plain film is an indication for a normal opening pressure and normal composition.
DXA scan.
Isotope bone scans are useful to characterize Which of the following is the most likely
the entire skeleton for evidence of localized areas cause for this patient’s presentation?
of bone disease, such as fractures, Paget disease, A. Anterior ischemic optic neuropathies
and metastases. Uptake on such imaging is not B. Hypocalcemia
selective, however, and is not diagnostic of specific
C. Primary affective disorder
bone diseases. Therefore, further triangulation
with additional clinical, biochemical, and imaging D. Pheochromocytoma
information is required to reach a diagnosis. E. Normal pressure hydrocephalus
Measurement of bone mass using DXA
Answer: B) Hypocalcemia
scanning is a reliable measure of bone strength
that allows prediction of fracture risk. It is
important not to consider T-score values in This patient’s laboratory test results were as
isolation and use them as isolated thresholds for follows:
treatment; DXA-determined data should always Serum adjusted calcium (repeated measurements)
be contextualized to the full fracture risk profile of = 5.3 mg/dL (8.2-10.2 mg/dL) (SI: 1.3 mmol/L
the patient. [2.1-2.6 mmol/L])
Bone biopsy is not a routine test, but it can be Phosphate = 4.7 mg/dL (2.3-4.7 mg/dL)
(SI: 1.5 mmol/L [0.7-1.5 mmol/L])
helpful in patients with complex metabolic bone Alkaline phosphatase = 159 U/L (73-330 U/L)
diseases, particularly when there is unexplained (SI: 2.66 µkat/L [1.22-5.51 µkat/L])
osteomalacia or a need for detailed assessment of Magnesium = 1.7 mg/dL (1.5-2.3 mg/dL)
renal osteodystrophy. Bone biopsies must only (SI: 0.7 mmol/L [0.6-0.9 mmol/L])
be undertaken in highly specialized centers with PTH = 2.0 pg/mL (10-65 pg/mL) (SI: 2.0 ng/L
[10-65 ng/L])
appropriate expertise. Urinary calcium = 56 mg/24 h (100-300 mg/24 h)
(SI: 1.4 mmol/d [2.5-7.5 mmol/d])
Clinical Case Vignettes Results of other investigations were normal
Case 1 and an autoantibody screen was negative.
Chvostek and Trousseau signs were both absent
A 42-year-old man presents with acute disturbance
and electrocardiography documented a normal
of vision bilaterally. His mood has become very
QT interval. Idiopathic hypoparathyroidism
low, and there has been substantial change in his
was diagnosed, and the patient was treated
personality. In the emergency department, he has
with calcium carbonate and alfacalcidol. With
hyperventilation, paraesthesia, and carpopedal
correction of his serum calcium, there was a
spasm and is diagnosed with panic attacks. There
marked improvement in vision, paraesthesia,
is no relevant medical or family history, and
and personality traits. Fundoscopy showed full
his only medication is low-dosage aspirin and
resolution of optic disc swelling. His visual fields
sertraline.
enlarged dramatically. Sertraline was discontinued
On physical examination, he is an anxious but
and his premorbid personality returned.
well-appearing man with no features of systemic
Neuropsychiatric presentations, including
disease. Visual acuity is 6/12 on the right side
mood disturbance, are recognized features
and 6/6 on the left side. Visual fields show gross
of hypocalcemia. There is also a rare but
constriction of both fields. Fundoscopy reveals a
recognized association between hypocalcemia and
pale optic disc on the right and a grossly swollen

ENDO 2021 • Bone and Mineral Metabolism  89

chased by Dr. Khalid H. Seedahmed, Jr., MRCP PGDip, CertEndocrinology SA - ID 283


papilledema. The mechanism is unclear, but it may Which of the following is the
be linked to reversible reduction in cerebrospinal most appropriate next step in
fluid absorption due to hypocalcemia. pharmacologic management?
Classic signs of hypocalcemia (eg, Chvostek sign) A. Continue zoledronic acid
may be absent despite significant hypocalcemia. B. Continue calcium and vitamin D only
Optimal care in hypoparathyroidism is still to be
C. Switch to denosumab
defined.7
Anterior ischemic optic neuropathies D. Switch to romosozumab
(Answer A) do not explain the breadth of E. Switch to teriparatide
symptoms at presentation. A primary affective
Answer: E) Switch to teriparatide
disorder (Answer C) does not account for
chronic changes in visual acuity and fields. Further bisphosphonate treatment was withheld.
Pheochromocytoma (Answer D) may have Teriparatide (Answer E), a bone anabolic agent,
complex presentations, but persistent changes in was commenced to promote fracture healing (non-
vision make this an unlikely unifying explanation licensed indication).
of all the presenting features. The absence of Osteogenesis imperfecta is typically associated
headaches in the history make normal pressure with frequent fractures in childhood and
hydrocephalus (Answer E) unlikely. adolescence. However, with age-associated decline
in bone density, affected patients may enter a
Case 2 second period of heightened fracture risk. This
patient had a genetic predisposition to fractures and
A 64-year-old woman with osteogenesis
was treated with bisphosphonates with an intention
imperfecta type 4 was commenced on
to ameliorate fracture risk. Paradoxically, in this
bisphosphonates after a low-trauma fracture of
case, such treatment is linked to the evolution of
her ankle. After having received pamidronate
bilateral incomplete subtrochanteric fractures. Even
for 3 years and zoledronic acid for 2 years, she
in patients with unequivocally increased fracture
describes a 10-month history of right lateral
risk (such as in osteogenesis imperfecta), close
thigh and groin pain. She is able to bear weight
clinical scrutiny and follow-up are required when
with only minimal discomfort, and there is
using bisphosphonates to provide individualized
no limitation of movement. Plain radiographs
treatment. Careful and sequential reassessments of
demonstrate an incomplete subtrochanteric
risks and benefits must be made when managing all
fracture of the lateral cortex of the right femur
patients taking bisphosphonates.
and a similar finding at the mid-diaphysis on
Persisting with an antiresorptive agent such
the left femur. Nuclear medicine bone scanning
as zoledronic acid (Answer A) or denosumab
with single-photon emission CT combined
(Answer C) is incorrect, as such treatments
with CT reveals intense uptake in the regions
exacerbate suppression of bone turnover, thought
corresponding to those identified on plain x-rays.
to be causally linked to atypical femoral fractures.
Orthopedic opinion is that the risk of completed
Continuing calcium and vitamin D (Answer B)
fractures is low, and because of the shape of the
is reasonable, but given the patient’s predicament
femora, nailing would be technically challenging.
of impending complete fractures, this would be
DXA scan lumbar spine T-score is –2.7, and
regarded as rather noninterventional drug therapy.
total hip T-score is –1.7.
Romosozumab (Answer D) may have a role in
situations such as that presented, but there are no
data to support its use in this scenario. Limited data
exist for teriparatide (Answer E) in this setting, but it
is the most appropriate pharmacologic therapy listed.

90  ENDO 2021 • Endocrine Case Management

chased by Dr. Khalid H. Seedahmed, Jr., MRCP PGDip, CertEndocrinology SA - ID 283


Case 3 that, following successful excision, histologically
revealed a hemangiopericytoma. Immediately
A 42-year-old man presents with a 3-year history
following surgery, the patient experienced a
of progressive pain in his feet and ankles. He
dramatic improvement in symptoms, and all
describes progressive difficulty walking due to
biochemical parameters rapidly normalized.
pain and muscle weakness. He has been evaluated
Hypophosphatemic conditions are rare.8
by multiple specialists. The progressive neurogenic
Tumor-induced osteomalacia (Answer B) is a
or myogenic weakness has affected all 4 limbs to
potentially reversible form of such pathology
the extent that he stopped working. Plain x-rays,
that has a typical biochemical pattern. Some
MRI, and electromyelography have revealed
laboratories do not include phosphate as part
no pathology, and routine laboratory tests have
of a standard blood panel, so the diagnosis can
documented mildly elevated alkaline phosphatase.
be missed and symptoms can be profound and
At today’s appointment, he is severely myopathic.
progressive. While surgical excision of the causal
Current laboratory test results: tumor (usually benign and of mesenchymal
origin) provides relief and full reversal of the
Alkaline phosphatase = 495 U/L (73-330 U/L)
(SI: 8.27 µkat/L [1.22-5.51 µkat/L])
manifestations of tumor-induced osteomalacia,
Phosphate = 1.6 mg/dL (2.3-4.7 mg/dL) localizing the tumor can be particularly
(SI: 0.5 mmol/L [0.7-1.5 mmol/L]) challenging, although multiple imaging
Serum adjusted calcium = 9.1 mg/dL (8.2-10.2 mg/dL) modalities have been explored. Medical therapy
(SI: 2.3 mmol/L [2.1-2.6 mmol/L]) with phosphate and activated vitamin D can be
Serum urea nitrogen = 10.9 mg/dL (8-23 mg/dL)
(SI: 3.9 mmol/L [2.9-8.2 mmol/L])
effective in ameliorating symptoms, but phosphate
Creatinine = 1.0 mg/dL (0.7-1.3 mg/dL) supplements are often poorly tolerated. Surgical
(SI: 88.4 µmol/L [61.9-114.9 µmol/L]) removal should be pursued if feasible.
25-Hydroxyvitamin D = 15 ng/mL (30-80 ng/mL The biochemical picture does not substantiate
[optimal]) (SI: 37.4 nmol/L [74.9-199.7 nmol/L]) a diagnosis of secondary hyperparathyroidism
PTH = 68.5 pg/mL (10-65 pg/mL) (SI: 68.5 ng/L
[10-65 ng/L])
(Answer A) that may be causally linked
Urine phosphate excretion = 0.94 g/24 h (0.9-1.3 g/24 h) to the clinical presentation. While the
(SI: 30.4 mmol/d [29.1-42.0 mmol/d]) 25-hydroxyvitamin D level is low, PTH is not
1,25-Dihydroxyvitamin D = correspondingly elevated and phosphate is
14.6 pg/mL (7.7-46.2 pg/mL) (SI: 38 pmol/L disproportionately low. For the duration and
[20-120 pmol/L])
FGF-23 = 251 U/L (<100 U/L)
degree of symptomatology, one may also expect
to see x-ray bone changes consistent with
osteomalacia if vitamin D deficiency (Answer C)
Which of the following is the most likely or resistance (Answer D) were the primary cause.
diagnosis causing this patient’s symptoms? The biochemical picture is compatible with
A. Secondary hyperparathyroidism X-linked hypophosphatemic rickets (Answer E),
B. Tumor-induced osteomalacia but this condition typically presents clinically
much earlier in life.
C. Vitamin D deficiency
D. Vitamin D resistance
E. X-linked hypophosphatemic rickets
Case 4
A 31-year-old woman from South East Asia has
Answer: B) Tumor-induced osteomalacia been diagnosed with celiac disease. She has iron
This patient had little symptomatic benefit from deficiency anemia with an adjusted calcium value
treatment with phosphate and activated vitamin of 8.68 mg/dL (8.2-10.2 mg/dL) (SI: 2.2 mmol/L
D. MRI showed a right ethmoid sinus mass [2.1-2.6 mmol/L]) and an alkaline phosphatase

ENDO 2021 • Bone and Mineral Metabolism  91

chased by Dr. Khalid H. Seedahmed, Jr., MRCP PGDip, CertEndocrinology SA - ID 283


value of 1607 U/L (73-330 U/L) (SI: 26.84 µkat/L Which of the following is the most
[1.22-5.51 µkat/L]) with otherwise normal likely cause of the x-ray appearance?
liver function. She commences a gluten-free A. Brown tumor
diet with ferrous sulfate and calcium/vitamin B. Aneurysmal bone cyst
D supplementation. She next presents to an
C. Fibrous dysplasia
orthopedic clinic when she is 21 weeks pregnant,
with a large swelling over her left proximal tibia D. Giant-cell tumor
following minor trauma 3 months previously. E. Simple bone cyst
X-ray of the leg shows a 5 × 2-cm destructive
lesion in the anterior tibia with an associated soft- Answer: A) Brown tumor
tissue mass (see left image). MRI shows localized
destruction with extension into the soft tissues. In this case, repeated investigations showed the
Referral is made to a specialized orthopedic following:
oncology unit. Needle biopsy show loose fibrous Adjusted calcium = 7.8 mg/dL (8.4-10.4 mg/dL)
tissue containing several osteoclast-type giant cells. (SI: 1.95 mmol/L [2.25-2.60 mmol/L])
A month later, she develops a pathologic fracture Alkaline phosphatase = 2011 U/L (73-330 U/L)
through the lesion (see right image). (SI: 33.58 µkat/L [1.22-5.51 µkat/L])
PTH = 482 pg/mL (12-65 pg/mL) (SI: 482 ng/L
[12-65 ng/L])
25-Hydroxyvitamin D = <6 ng/mL (20-68 ng/mL)
(SI: <15 nmol/L [50-170 nmol/L])

Osteomalacia with secondary hyperparathyroidism


caused brown tumors (Answer A) (a contralateral
lesion also developed). Vitamin D with calcium was
initiated and recalcified the “bone tumors,” but she
had 2 cm loss of leg length. Biochemical markers
returned to normal, suggesting poor prior adherence
to treatment. Celiac disease and pregnancy had
contributed to the severe expression of osteomalacia.
Frank osteomalacia is uncommon but is
seen more frequently in developing countries
throughout the world. Brown tumors are
very rare pseudotumoral manifestations of
hyperparathyroidism. High PTH levels (can
also be seen in primary hyperparathyroidism)
increase osteoclast activity and produce irregular
bone resorption resulting in microfractures
and hemorrhage. Microscopically, these lesions
are indistinguishable from giant-cell tumors or
aneurysmal bone cysts and a diagnosis of brown
tumor of hyperparathyroidism is based on
abnormal biochemistry. Most brown tumors due
to secondary hyperparathyroidism are seen in
renal osteodystrophy, and other causes are rare.
The biochemical picture provides context to
the radiograph. Aneurysmal bone cyst (Answer B)
or giant-cell tumor (Answer D) are within the

92  ENDO 2021 • Endocrine Case Management

chased by Dr. Khalid H. Seedahmed, Jr., MRCP PGDip, CertEndocrinology SA - ID 283


x-ray differential diagnosis, but the biochemical Creatinine = 0.9 mg/dL (0.6-1.1 mg/dL)
tests point clearly to a metabolic origin as a (SI: 79.6 µmol/L [53.0-97.2 µmol/L])
25-Hydroxyvitamin D = 14 ng/mL (30-80 ng/mL
brown tumor. One would not expect this degree
[optimal]) (SI: 34.9 nmol/L [74.9-199.7 nmol/L])
of biochemical abnormalities in a patient with PTH = 85 pg/mL (10-65 pg/mL) (SI: 85 ng/L
fibrous dysplasia (Answer C) or a simple bone cyst [10-65 ng/L])
(Answer E). The rapidly progressive nature would
also not support the latter 2 diagnoses. The cystic On more direct questioning, she has a 3-year
area is too large and dynamic for a simple bone cyst. history of osmotic symptoms, constipation, low
mood, headaches, and exhaustion. She has no
history of fractures or renal stones, and there is
Case 5  New Content 
no family history of endocrine diseases. Primary
A 25-year-old woman with a history of hyperparathyroidism is diagnosed, and she is
hypertension and asthma presents with watering prescribed supplemental vitamin D. She is also
in the right eye and slowly progressive, firm referred to a maxillofacial surgeon due to the threat
swelling in the lateral wall of the orbit. Vision to her vision, and she undergoes right orbital
is subjectively normal, and she is referred to osteotomy. Bone biopsy shows features compatible
ophthalmology. There is proptosis with superior with a brown tumor. At parathyroidectomy, a
and medial displacement of the right globe. There is plum-sized parathyroid adenoma is removed, and
conjunctival chemosis, and visual acuity on the right postoperatively she has a short duration of “hungry
side is 6/18, correcting to 6/12. Fundoscopy shows bones” but thereafter remains eucalcemic. Findings
significant macular choroidal folds. CT of the orbits on histopathologic examination of the parathyroid
identifies a large cystic tumor in the lateral wall of adenoma are consistent with parathyroid adenoma.
the right orbit (see image).
Which of the following is NOT a
recognized risk factor for hungry
bone syndrome in this case?
A. Elevated alkaline phosphatase
B. Younger age
C. Radiographic evidence of parathyroid bone
disease
D. Large parathyroid adenoma
E. Untreated vitamin D deficiency
Answer: B) Younger age
Hungry bone syndrome is uncommon, and
preoperative risk factors, as long as they are
identified, can often be mitigated to an extent.
Hungry bone syndrome can be seen after
parathyroidectomy for hyperparathyroidism or
Current laboratory test results:
following thyroidectomy for hyperthyroidism.
Alkaline phosphatase = >900 U/L (73-330 U/L) Upon removal of the stimulus for mineral release
(SI: >15.03 µkat/L [1.22-5.51 µkat/L]) from the skeleton, in this case PTH hypersecretion,
Phosphate = 1.6 mg/dL (2.3-4.7 mg/dL)
there is intense remineralization that can cause
(SI: 0.5 mmol/L [0.7-1.5 mmol/L])
Serum adjusted calcium = 12.7 mg/dL (8.2-10.2 mg/dL) hypocalcemia and hypophosphatemia, which can be
(SI: 3.2 mmol/L [2.1-2.6 mmol/L]) long-lasting for many weeks. Management requires

ENDO 2021 • Bone and Mineral Metabolism  93

chased by Dr. Khalid H. Seedahmed, Jr., MRCP PGDip, CertEndocrinology SA - ID 283


large dosages of calcium and activated vitamin D.
PTH levels are usually normal, but they eventually
rise in the presence of the hypocalcemia, as long
as residual parathyroid reserve is intact. Following
suppression of the parathyroid glands by the
dominant pathologic gland, physiologic responses
of the remaining parathyroids can take some
time. The biochemical and radiographic markers
of bone disease (Answer C) indicate an increased
likelihood of postoperative hungry bone syndrome,
and untreated vitamin D deficiency (Answer E)
can perpetuate this. Biochemically severe primary
hyperparathyroidism (Answer A) in association
with a large parathyroid adenoma (Answer D)
raises the possibility of postoperative hungry bone
syndrome. Older, rather than younger age, is more
often associated with hungry bone syndrome (thus,
Answer B is correct).

Case 6  New Content 


A 28-year-old woman is admitted to the hospital
with severe and progressive back pain 3 months
after the delivery of her first child. She has had
Which of the following is appropriate/
no trauma, but the pain is exacerbated by tasks
true regarding pregnancy and
involved in caring for her new baby. She has a
lactation-associated osteoporosis?
history of gastroesophageal reflux and has taken
omeprazole on occasion. She fractured her proximal A. The patient should be advised that the risk
ulna as a child after a fall, but she has no symptoms of fractures in subsequent pregnancies is not
of ligament laxity, hearing problems, or dental increased
issues. There is no family history of fractures. B. Bone active drugs should be continued long-
On physical examination, her BMI is 21 kg/m2. term because fracture risk remains high
She has pain with minimal movement. Vertebral C. Bisphosphonates should be avoided if possible
fracture assessment confirms MRI findings of to avoid later potential teratogenicity
vertebral fractures at T3, T8, T10, T12, and L3. D. Nonvertebral fractures are usually observed
The T10, T12, and L3 fractures show bone edema
E. Fractures typically first become apparent during
on MRI. Extensive biochemical testing reveals no
the second trimester
secondary cause of the multiple vertebral fractures.
DXA scan documents Z-scores of –4.3 at the lumbar Answer: C) Bisphosphonates should be avoided if
spine and –2.5 at the total hip. possible to avoid later potential teratogenicity
Pregnancy and lactation-associated osteoporosis
is diagnosed, and due to the severity and active Pregnancy and lactation-associated osteoporosis is
progression of the vertebral fractures, she is treated very rare and was first documented in the 1940s.
with teriparatide. Lumbar spine Z-score improves The clinical presentation is usually characterized
to –3.1 after 2 years. She has no additional fractures, by symptomatic spontaneous or minimum trauma-
pain is resolved, and functional status is excellent. related vertebral fractures (thus, Answer D is
incorrect). There is no identified cause for this rare

94  ENDO 2021 • Endocrine Case Management

chased by Dr. Khalid H. Seedahmed, Jr., MRCP PGDip, CertEndocrinology SA - ID 283


but potentially devastating condition. Increased due to potential teratogenicity (thus, Answer C is
calcium demands during pregnancy and lactation correct), but their use has been described in case
may be a small contributor, but this does not reports and case series. There are also a number
provide a convincing cause in most cases. Fractures of case reports describing the use of teriparatide.
in subsequent pregnancies occur in approximately There are no clinical guidelines for management,
20% to 30% of cases, and it is prudent to warn and each case must be assessed on an individual
patients about this risk (thus, Answer A is basis. Fractures typically occur in the postpartum
incorrect). Management is highly pragmatic, period (thus, Answer E is incorrect), and there is
ensuring adequate calcium and vitamin D intake. generally a substantial decrease in fracture risk in
Depending on severity, extent, and progression, the longer term beyond acute postpartum fractures
treatment may be indicated. Bisphosphonates are (thus, Answer B is incorrect).
not ideal in women during their childbearing years

References
1. Kishnani PS, Rockman-Greenberg C, Rauch F, et al. Five-year efficacy 5. Damasiewicz MJ, Nickolas TL. Rethinking bone disease in kidney disease.
and safety of asfotase alfa therapy for adults and adolescents with JBMR Plus. 2018;2(6):309-322. PMID: 30460334
hypophosphatasia. Bone. 2019;121:149-162. PMID: 30576866 6. Hannan FM, Kallay E, Chang W, Brandi ML, Thakker RV. The calcium-
2. Carpenter TO, Whyte MP, Imel EA, et al. Burosumab therapy in children sensing receptor in physiology and in calcitropic and noncalcitropic diseases.
with X-linked hypophosphatemia. N Engl J Med. 2018;378(21):1987-1998. Nat Rev Endocrinol. 2018;15(1):33-51. PMID: 30443043
PMID: 29791829 7. Mannstadt M, Bilezikian JP, Thakker RV, et al. Hypoparathyroidism. Nat Rev
3. Hannan FM, Newey PJ, Whyte MP, Thakker RV. Genetic approaches Dis Primers. 2017;3:17080. PMID: 28980621
to metabolic bone diseases. Br J Clin Pharmacol. 2019;85(6):1147-1160. 8. Christov M, Jüppner H. Phosphate homeostasis disorders. Best Pract Res Clin
PMID: 30357886 Endocrinol Metab. 2018;32(5):685-706. PMID: 30449549
4. Black DM, Abrahamsen B, Bouxsein ML, Einhorn T, Napoli N. Atypical
femur fractures: review of epidemiology, relationship to bisphosphonates,
prevention, and clinical management. Endocr Rev. 2019;40(2):333-368.
PMID: 30169557

ENDO 2021 • Bone and Mineral Metabolism  95

chased by Dr. Khalid H. Seedahmed, Jr., MRCP PGDip, CertEndocrinology SA - ID 283


Bone Turnover Markers in the
Management of Osteoporosis
Richard Eastell, MD. Department of Oncology and Metabolism, University of Sheffield,
Sheffield, South Yorkshire, England; E-mail: r.eastell@sheffield.ac.uk

Learning Objectives (BMD), the prior occurrence of a fracture (such


as spine or hip), or a high 10-year fracture
As a result of participating in this session, learners
risk.1 The most commonly used drugs are oral
should be able to:
bisphosphonates, particularly alendronate. Up
• Identify poor response to treatment using to 10% of older women in the United Kingdom
bone turnover markers. take alendronate. However, the adherence to the
medication is poor and after 1 year of treatment,
• After stopping treatment, identify offset of
only about 50% of patients continue the drug.
effect using bone turnover markers.
There is a big need to assess adherence. A working
group of the International Osteoporosis
Foundation and the European Calcified Tissue
Society proposed that the measurement of BTMs
Main Conclusions before and then 3 months after starting treatment
Bone turnover markers (BTMs) can be measured could be a successful strategy to identify poor drug
reliably in the serum or urine and are widely response, most likely caused by poor adherence.2
available. BTMs respond rapidly and by a lot after After 5 years of an oral bisphosphonate such as
starting the usual treatments for osteoporosis, such alendronate, there is an increased risk of atypical
as bisphosphonates. The greater the response, the femur fractures. A “drug holiday” at this time
greater the reduction in fracture risk. The goal in could be considered if the patient is at low risk
the individual patient is for the change in BTMs to of further fracture (hip BMD T-score above –2.5
exceed the least significant change into the lower and no prior spine fracture).3 There is not a clear
half (or below) of the reference interval for healthy approach to restarting treatment, but one method
young women. to determine no further antiresorptive effect of the
BTMs increase after stopping bisphosphonates, bisphosphonate is to measure BTMs.
but not to the levels found before starting them. An
increase beyond the least significant change or to a
level in the upper half (or above) of the reference Barriers to Optimal Practice
range for healthy young women indicates the need
• How should clinicians provide personalized
to restart therapy.
care for the individual patient to monitor
treatment response? One could count the
number of fractures a patient suffers, but these
Significance of the are uncommon events and can occur even with
Clinical Problem effective treatment. One could measure BMD,
but the changes are small and slow to occur.
The decision to treat a patient with osteoporosis
is based on their future risk of fracture and this • How should clinicians provide personalized
is usually assessed based on bone mineral density care for the individual patient to monitor

96  ENDO 2021 • Endocrine Case Management

chased by Dr. Khalid H. Seedahmed, Jr., MRCP PGDip, CertEndocrinology SA - ID 283


when it is time to restart treatment? One could menopause, women’s levels are about twice as
measure BMD, but the changes are small and high as those of premenopausal women. A recent
slow to occur. fracture results in a large increase in BTMs with
a peak after 3 months and levels not returning to
normal for a year. This is particularly striking for
Strategies for Diagnosis, the bone formation markers.
Therapy, and/or Management Drugs can affect BTMs. The most commonly
encountered problem is the use of glucocorticoids.
Bone Turnover Markers These suppress BTM, especially bone formation
BTMs can be measured in the serum and the markers, so if the dose has been increased in
urine, and they can reflect bone resorption or bone the last few days BTMs would be reduced.
formation. The bone resorption markers include Thus, BTMs are only helpful in patients with
degradation products of type I collagen such as glucocorticoid-induced osteoporosis if the
serum CTX (C-telopeptide) or urinary NTX glucocorticoid dosage remains unchanged.
(N-telopeptide) or the enzyme tartrate resistant
acid phosphatase type 5b. The bone formation Clinical Uses of BTMs—Identification
markers include matrix proteins such as PINP of Treatment Response
(procollagen I N-propeptide) or osteocalcin, or the
enzyme bone-specific alkaline phosphatase. There is some evidence to support the idea that
The International Osteoporosis Foundation higher BTM levels are associated with higher rates
and the International Federation of Clinical of bone loss, greater fracture risk, the presence
Chemistry have proposed using CTX and PINP of secondary osteoporosis, and better response to
as the reference markers.4 When conducting any forms of osteoporosis treatment. However,
research, these 2 BTMs should always be included. the evidence is not strong enough for use in the
This does not mean that NTX, TRACP5b, individual patient.6
osteocalcin, and bone alkaline phosphatase are not The best use of BTMs is in the identification of
useful in clinical practice. treatment response and in monitoring the offset of
These BTMs are easy to measure using manual treatment.
(ELISA) or automated immunoassays, and the latter The TRIO study was set up to evaluate
method has low precision error (less than 5%) and the clinical utility of BTM in postmenopausal
gives consistent results. Automated immunoassays osteoporosis.7 The study included a formal
are available for most of the BTMs worldwide and assessment of the 2 treatment targets:
are to be recommended. The only exception is that 1. The least significant change. This is the least
in the United States the automated assays for PINP change that can be considered to be significant
have not been approved by the FDA. (at P<.05) based on 2 measurements in an
individual patient. This was estimated to be
Sources of Variability in BTMs around 10 µg/L for PINP and 120 ng/L for CTX.
The most common reason for confusion in the use
2. The median value for BTM in a healthy young
of BTMs is their variability.5 BTM levels change
woman. This second target is useful if there
throughout the day. For example, CTX is twice
is only 1 BTM measurement documented
as high in the morning as in the afternoon, so it is
while on treatment. If the BTM is below the
recommended to draw the patient’s blood in the
premenopausal median value (or below the
fasting state, first thing in the morning.
reference interval), then the patient is considered
Different reference ranges should be used
to have responded. These mean values are
based on age. Children often have BTM levels
35 µg/L for PINP and 280 ng/L for CTX.
about 10-times higher than those of adults. After

ENDO 2021 • Bone and Mineral Metabolism  97

chased by Dr. Khalid H. Seedahmed, Jr., MRCP PGDip, CertEndocrinology SA - ID 283


The TRIO study included 3 commonly used oral The principles also apply to anabolic
bisphosphonates (alendronate, ibandronate, and agents such as teriparatide, abaloparatide, and
risedronate), and the percentage of responders romosozumab. These drugs all cause an early
using least significant change in this research study and large increase in BTMs, particularly PINP.
at 12 weeks of treatment was around 90% for CTX This marker can be measured as early as a month
and 85% for PINP. The percentage of responders after treatment initiation to identify response and
using a value below the premenopausal mean in it should increase. An increase above the least
this research study at 12 weeks of treatment was significant change (>10 µg/L) to above the upper
around 95% for CTX and 90% for PINP. Of course, limit of the reference range (>65 µg/L) indicates
in clinical practice fewer patients respond. good response.6 There is no role for monitoring
offset of effect, as all these agents should be
followed by treatment with antiresorptive drugs.
Clinical Uses of BTMs—
Identification of Treatment
Offset of Effect Clinical Case Vignettes
Patients taking oral bisphosphonates are Case 1
recommended to stop treatment after 5 years A 70-year-old woman has osteopenia noted
if they are at relatively low fracture risk (BMD in spinal radiographs. She is treated with
T-score above –2.5 at the hip and no recent alendronate, 70 mg once weekly, calcium, and
fracture) to reduce the risk of atypical femur vitamin D. The BMD T-score at the total hip and
fractures. The rate of bone loss after stopping lumbar spine is –3. BTMs are measured at baseline
these drugs is very low, as the drugs continue to and after 6 months of treatment. The patient takes
prevent bone loss for several years after stopping alendronate for 5 years (60 months) and BTMs are
them, particularly in the spine. BTMs are more measured again and then 1 year later.
useful than BMD, as they reflect the way in which
these drugs work.8 An increase in BTM above Time Point CTX PINP
the least significant change or a BTM level (off 0 months 500 ng/L 60 µg/L
treatment) above the premenopausal median
6 months 120 ng/L 20 µg/L
signify offset of response and therefore treatment
would be recommended to restart. 60 months 120 ng/L 20 µg/L

72 months 400 ng/L 40 µg/L

Clinical Uses of BTMs—Monitoring


Drugs Other Than Bisphosphonates Did the patient respond to treatment?
The principles shown here for bisphosphonates Has the treatment effect worn off?
can apply to other antiresorptive drugs such as A. She responded to treatment and the effect has
denosumab and raloxifene. The response rate worn off
will be even higher for denosumab, but lower for B. She did not respond to treatment and the effect
raloxifene, as denosumab is much more potent. has not worn off
Indeed, it could be argued that everyone responds
C. She responded to treatment and the effect has
to denosumab, so there is no need to monitor
not worn off
efficacy.9 However, after stopping denosumab,
there is a rebound increase in BTMs and the D. The results cannot be interpreted as there is
timing of the rebound might help dictate the too much variability
optimal time to give a bisphosphonate to prevent Answer: A) She responded to treatment
accelerated bone loss. and the effect has worn off

98  ENDO 2021 • Endocrine Case Management

chased by Dr. Khalid H. Seedahmed, Jr., MRCP PGDip, CertEndocrinology SA - ID 283


For CTX, she responded, as she exceeded the least She is at risk for further fractures, so
significant change (decrease by more than 120) and watching and waiting (Answer A) would be a
the value fell below the median for premenopausal high-risk strategy. Anabolic therapy (Answer B)
women (280 ng/L). For PINP, she responded, as is not an unreasonable approach, as the BMD
she exceeded the least significant change (decrease response to anabolic drugs is greater if BTMs
by more than 10) and the value fell below the are high. It is more expensive than zoledronate
median for premenopausal women (35 µg/L). or denosumab, though. The use of the licensed
She had evidence for offset of effect for dosage of alendronate (for osteoporosis) is not
CTX, as she exceeded the least significant change an unreasonable approach. However, low-dosage
(increase by more than 120) and the value treatment (Answer C) for such high bone turnover
increased above the median for premenopausal is unlikely help this patient reach the BTM target.
women (280 ng/L). She had evidence for offset
of effect for PINP, as she exceeded the least
Case 3  New Content 
significant change (increase by more than 10)
and the value increased above the median for A 78-year-old woman was referred to the
premenopausal women (35 µg/L). metabolic bone clinic with back pain and
She exceeds the least significant change and height loss 8 years ago. She was intolerant of
this takes into account any month-by-month oral bisphosphonates, so denosumab, 60 mg
variability, so the results can indeed be interpreted. subcutaneously every 6 months, was initiated.
Imaging at that time documented vertebral
fractures of T8, T9, T10, L1, and L2.
Case 2 She has now been taking denosumab for
An 87-year-old woman had previous fractures of 8 years. At today’s appointment, BMD assessment
the left femur, left shoulder, right wrist, and the documents T-scores of –0.7 at the lumbar spine
spine (vertebrae L3, L4). She was treated with and –1.5 at the total hip. Imaging shows a new
alendronate, 70 mg once weekly for 10 years, vertebral fracture at L3 (no history of trauma).
which was stopped 2 years ago. She is now The concentration of the bone turnover
receiving calcium and vitamin D alone. A vertebral marker PINP is 10 µg/L, and all other laboratory
fracture assessment reveals a fracture at L2, test results are normal.
confirmed by radiographs. Workup for secondary
causes of osteoporosis is negative. Her PINP Which of the following actions
concentration is 140 µg/L. should be recommended?
A. Continue treatment with denosumab
Which of the following would B. Change treatment to teriparatide
be the best treatment now?
C. Change treatment to zoledronate
A. No treatment, just watch and wait
D. Stop treatment
B. Initiate anabolic therapy (eg, teriparatide,
abaloparatide, romosozumab) Answer: A) Continue treatment with denosumab
C. Restart alendronate at half the usual dosage
Treatment with denosumab was effective (as
D. Initiate zoledronate or denosumab assessed by the PINP level), so this therapy should
Answer: D) Initiate zoledronate or denosumab be continued (Answer A). She has had 1 new
vertebral fracture over 8 years, which is not
This patient is no longer having suppression of bone considered treatment failure. According to the
turnover from the prior alendronate therapy. She has International Osteoporosis Foundation, treatment
high bone turnover and is likely to have a good BMD failure is defined as having 2 fractures while on
response to zoledronate or denosumab (Answer D). treatment. One cannot know from the BMD

ENDO 2021 • Bone and Mineral Metabolism  99

chased by Dr. Khalid H. Seedahmed, Jr., MRCP PGDip, CertEndocrinology SA - ID 283


whether she has responded, although her BMD Which of the following would
indicates only osteopenia. The PINP concentration be the best treatment now?
is well below the geometric mean for young A. No treatment, just watch and wait
women (<35 µg/L). Zoledronate (Answer C) B. Initiate anabolic therapy (eg, teriparatide,
is rarely effective if denosumab is not effective. abaloparatide, romosozumab)
Teriparatide (Answer B) could be effective, but she
C. Restart zoledronate with a single dose
has not failed to respond to denosumab. The BMD
increase is attenuated when giving teriparatide D. Initiate denosumab
after denosumab. Stopping treatment (Answer D) Answer: C) Restart zoledronate with a single dose
would not be appropriate.
This patient had accelerated bone loss and a bone
Case 4  New Content  turnover level above the desirable range after she
had stopped zoledronate for 3 years. The best
A 73-year-old woman was diagnosed with severe option now would be zoledronate (Answer C),
osteoporosis 11 years ago. She was given a 6-year as she has high turnover and rapid bone loss and
course of annual zoledronate infusions, 5 mg. Two she was able to tolerate this medication in the
years ago, she was evaluated to determine whether past. She has had a treatment gap of 3 years, so
bone loss had resumed after stopping zoledronate atypical femur fractures should not be a problem.
3 years earlier. Over this 3-year period, she Denosumab (Answer D) would be a reasonable
had lost 5% in total hip BMD, and her PINP option, but this author would only use it if
concentration was 50 µg/L. there were a reason to not give zoledronate.
She would most likely respond to an anabolic
drug (Answer B), but these agents are usually
reserved for patients with vertebral fractures and
thus severe osteoporosis. Watching and waiting
(Answer A) would be inappropriate, as she has
both accelerated bone loss and an unacceptably
high bone turnover.

References
1. Eastell R, Rosen CJ, Black DM, Cheung AM, Murad MH, Shoback D. 5. Szulc P, Naylor K, Hoyle NR, Eastell R, Leary ET; National Bone Health
Pharmacological management of osteoporosis in postmenopausal women: Alliance Bone Turnover Marker Project. Use of CTX-I and PINP as bone
an Endocrine Society clinical practice guideline. J Clin Endocrinol Metab. turnover markers: National Bone Health Alliance recommendations to
2019;104(5):1595-1622. PMID: 30907953 standardize sample handling and patient preparation to reduce pre-analytical
2. Diez-Perez A, Naylor KE, Abrahamsen B, et al; Adherence Working variability. Osteoporos Int. 2017;28(9):2541-2556. PMID: 28631236
Group of the International Osteoporosis Foundation and the European 6. Eastell R, Pigott T, Gossiel F, Naylor KE, Walsh JS, Peel NFA. Diagnosis of
Calcified Tissue Society. International Osteoporosis Foundation and endocrine disease: bone turnover markers: are they clinically useful? Eur J
European Calcified Tissue Society Working Group. Recommendations Endocrinol. 2018;178(1):R19-R31. PMID: 29046326
for the screening of adherence to oral bisphosphonates. Osteoporos Int. 7. Naylor KE, Jacques RM, Paggiosi M, et al. Response of bone turnover
2017;28(3):767-774. PMID: 28093634 markers to three oral bisphosphonate therapies in postmenopausal
3. Adler RA, El-Hajj Fuleihan G, Bauer DC, et al. Managing osteoporosis in osteoporosis: the TRIO study. Osteoporos Int. 2016;27(1):21-31.
patients on long-term bisphosphonate treatment: report of a task force PMID: 25990354
of the American Society for Bone and Mineral Research. J Bone Miner Res. 8. Kim TY, Bauer DC, McNabb BL, et al. Comparison of BMD changes and
2016;31(1):16-35. PMID: 26350171 bone formation marker levels 3 years after bisphosphonate discontinuation:
4. Vasikaran S, Eastell R, Bruyere O, et al; IOF-IFCC Bone Marker Standards FLEX and HORIZON-PFT Extension I Trials. J Bone Miner Res.
Working Group. Markers of bone turnover for the prediction of fracture risk 2018;34(5):810-816. PMID: 30536713
and monitoring of osteoporosis treatment: a need for international reference 9. Eastell R, Christiansen C, Grauer A, et al. Effects of denosumab on bone
standards. Osteoporos Int. 2011;22(2):391-420. PMID: 21184054 turnover markers in postmenopausal osteoporosis. J Bone Miner Res.
2011;26(3):530-537. PMID: 20839290

100  ENDO 2021 • Endocrine Case Management

chased by Dr. Khalid H. Seedahmed, Jr., MRCP PGDip, CertEndocrinology SA - ID 283


Using the Best Available
Evidence to Personalize
Osteoporosis Treatment
E. Michael Lewiecki, MD. University of New Mexico Health Sciences Center, Albuquerque,
NM; E-mail: mlewiecki@gmail.com

Micol S. Rothman, MD. University of Colorado School of Medicine, Aurora, CO; E-mail:
micol.rothman@cuanschutz.edu

Learning Objectives the applicability of the studies for some patients.


Guidelines developed by experts interpreting the
As a result of participating in this session, learners
best available medical evidence may be simpler,
should be able to:
more intuitive, and easier to apply in clinical
• Recognize the variability and limitations of practice, but are open to criticism that they are
clinical practice guidelines for osteoporosis. overly subjective and based on a lower level
of evidence. The application of any guideline
• Apply the best available medical evidence
when making decisions for individual patients
to the care of individual patients for whom
must consider all available clinical information,
guidelines may not be appropriate.
including comorbidities, previous experiences, and
patient preferences.

Main Conclusions
Many clinical practice guidelines address the
Significance of the
management of patients with osteoporosis.1-4 Clinical Problem
These are often developed by societies or Osteoporosis is a common disorder characterized
organizations that address specific patient by low bone density and poor bone quality
populations using divergent approaches to analyze resulting in increased risk of fractures. Despite the
and interpret the medical evidence, with or availability of excellent tools to assess fracture risk
without consideration of cost-effectiveness, and and approved medications that reduce fracture
differing views on inclusion of expert opinion. risk, most patients at risk for fractures are not
Guidelines that rely exclusively on data derived being treated.6 The large treatment gap is now
from large, randomized, placebo-controlled recognized as a global crisis in the care of patients
clinical trials are scientifically rigorous but with osteoporosis.7
tend to be restrictive, complex, and lacking the
flexibility required to address the needs of some
patients. Many or most patients seen in clinical Barriers to Optimal Practice
practice would not qualify for participation in Many factors contribute to the treatment gap,
the clinical trials supporting the approval of the including fear of medication adverse effects, poor
medications used to treat them,5 casting into doubt understanding of the balance of benefits and

ENDO 2021 • Bone and Mineral Metabolism  101

chased by Dr. Khalid H. Seedahmed, Jr., MRCP PGDip, CertEndocrinology SA - ID 283


risks with treatment, inadequate communication Which of the following is the best
skills to quantify risk, lack of appreciation of the option to reduce the risk of another
potentially serious consequences of fractures, vertebral fracture in this patient?
failure to recognize that a prior fracture is due to A. Calcium and vitamin D with no other
osteoporosis, reductions in reimbursement for pharmacologic agent since her DXA shows
bone density testing (United States) resulting in osteopenia
the closure of many office-based DXA facilities, B. Kyphoplasty to reduce her risk of adjacent
conflicting clinical practice guidelines, limited level fracture
available time for physician-patient encounters,
C. Teriparatide for 18 to 24 months followed by
and competing health care priorities.
antiresorptive therapy
D. Oral alendronate for 5 years followed by a drug
Strategies for Diagnosis, holiday
Therapy, and/or Management
Answer: C) Teriparatide for 18 to 24 months
Accordingly, many strategies to reduce the followed by antiresorptive therapy
treatment gap have been proposed,8 including
technology-enabled collaborative learning, the This patient has the diagnosis of osteoporosis
prototype of which is Bone Health TeleECHO.9 based on a low-trauma fracture2 and has a very
Tools and algorithms have been developed to high risk for future fracture; thus, pharmacologic
help patients and providers alike better quantify therapy is indicated (thus, Answer A is incorrect).
the risks and benefits of osteoporosis therapy. There has been controversy surrounding the use
However, providers must recognize that there of kyphoplasty (Answer B) for painful vertebral
is not a “one size fits all” answer for many fractures. A recent American Society for Bone
patients. At times, more than one visit and a and Mineral Research task force advocated
variety of materials may be needed for patients against routine use of vertebral augmentation for
to understand their diagnosis of osteoporosis and vertebral fractures based on a review of multiple
feel comfortable with initiation of medication. trials focusing on 1-year outcomes that showed
Clinical judgment, patient preference, and other no differences in pain and disability.10 However,
factors are key aspects of managing complex cases the VAPOUR trial showed vertebroplasty to be
of osteoporosis. superior to placebo for pain reduction in patients
with fractures of less than 6 weeks duration.
There was particular benefit for patients with
Clinical Case Vignettes severely painful fractures requiring high doses of
Case 1 pain medication and/or hospitalization (where
A 62-year-old woman presents after a painful length of stay was reduced in the treatment arm).11
vertebral compression fracture. She has a history However, vertebral augmentation would not
of chronic obstructive pulmonary disease with be expected to reduce the rate of adjacent level
frequent courses of oral glucocorticoids and long- fracture. In fact, there have been concerns for an
term inhaled steroids. She undergoes bone mineral increased risk of adjacent level fracture, but at this
density testing that shows osteopenia based on point, this risk remains unclear.10
lowest T-score. Secondary workup is otherwise As far as the best pharmacologic option,
negative with adequate vitamin D levels and an 18-month randomized head-to-head trial
normal renal function. in 414 patients comparing teriparatide with
alendronate in patients with glucocorticoid-
induced osteoporosis showed larger gains in
spine BMD (7.2% vs 3.4%), as well as lower rates

102  ENDO 2021 • Endocrine Case Management

chased by Dr. Khalid H. Seedahmed, Jr., MRCP PGDip, CertEndocrinology SA - ID 283


of vertebral compression fracture (0.6% vs 6.1%) of bisphosphonates are unique in that they
in the teriparatide-treated group.12 Although are slowly released from the bone, providing
oral alendronate (Answer D) is a reasonable continuing protection for an uncertain period
choice if teriparatide is not financially viable, (but not forever) after administration is stopped.1
is contraindicated, or is otherwise not desired This does not exist for any other osteoporosis
by the patient, the strongest data for secondary therapy. Stopping denosumab can lead to rapid
fracture prevention in this scenario would be for rise in bone turnover markers, rapid decrease
teriparatide (Answer C). Although the American in BMD, return of fracture risk to baseline, and
College of Rheumatology guidelines would possible increase in the risk of multiple vertebral
advocate for first-line alendronate based on cost, fractures.14 However, some patients treated with
other guidelines from Europe would advocate for denosumab may reach a treatment target at which
first-line teriparatide in a high-risk patient.13 fracture risk has reached an acceptable level and
modification of the treatment plan could be
considered. Recent data suggest that improvement
Case 2
in hip T-score on denosumab to a level of about
A 70-year-old man presents after treatment –2.0 to –1.5 is associated with maximum reduction
with denosumab for 8 years, with gains in bone of fracture risk.15 As to how to bridge to another
density and no fractures. His estimated glomerular therapy, recent data/case reports can help guide
filtration rate is 60 mL/min per 1.73 m2. DXA our thinking, but no guidelines exist to counsel
now shows bone mineral density in the osteopenia patients who do not want to commit to lifelong
range. He requests to stop treatment and wants denosumab (thus, Answer D is incorrect). Giving
to know what medications will give him the best intravenous zoledronic acid (Answer B) is a very
chance at avoiding bone loss. attractive option with its antifracture benefits and
ease of administration; however, data have raised
Which of the following is the best advice? concern about the optimal timing. Administering
A. Stop denosumab; it is time for a drug zoledronic acid at 6 months (ie, right when the
holiday since denosumab will provide lasting next shot of denosumab is due), when reviewed
protection in a small case series, did not appear to protect the
B. Give 5 mg intravenous zoledronic acid when patient fully from bone loss.16 Thus far, the data
his next injection is due (ie, 6 months after the suggest that administration of oral alendronate
last denosumab dose) (Answer C) gives the best ongoing protection.17
C. Start oral alendronate when his next injection The most recent Endocrine Society guidelines
is due suggest that denosumab be followed with an
antiresorptive agent, although which one and the
D. Explain that this has not been studied and
optimal timing of treatment are uncertain.1
there is no other option than to remain on
denosumab for life
Case 3
Answer: C) Start oral alendronate
when his next injection is due A 73-year-old woman has osteoporosis. She has
a history of breast cancer with lumpectomy and
There has been much recent discussion of how radiation followed by treatment with an aromatase
and when to stop/bridge of denosumab if needed/ inhibitor. She wants to know about romosozumab
desired. First, it must be emphasized that there because she is worried about adverse effects of
is no role for a drug holiday with denosumab bisphosphonates and teriparatide.
(thus, Answer A is incorrect). After stopping
denosumab, the treatment effect will be rapidly
lost if another agent is not started. The properties

ENDO 2021 • Bone and Mineral Metabolism  103

chased by Dr. Khalid H. Seedahmed, Jr., MRCP PGDip, CertEndocrinology SA - ID 283


Which of the following statements is true? 4093 postmenopausal women was at higher risk
A. There were no cases of osteonecrosis of the of fracture and older than the FRAME group.
jaw or atypical femoral fractures in any of the At 12 months, there were vertebral fractures in
romosozumab trials 4.0% of the romosozumab group and 6.1% of the
B. There is a boxed warning against using alendronate group (a relative risk reduction of
romosozumab in anyone who has had a 37%). Nonvertebral fractures were not different
myocardial infarction in the preceding 12 at 1 year. At the end of the second year, the risk
months of nonvertebral fractures and hip fractures was
lower in the romosozumab to alendronate group
C. Romosozumab should not be given to patients than the alendronate to alendronate group.
at high risk for osteosarcoma However, there was a numerically increased risk
D. Sclerostin acts to increase bone formation via of cardiovascular events in the romosozumab
the Wnt pathway group with (50/2014) 2.5% in the first year
compared with 38/2014 (1.9%) in the alendronate
Answer: B) There is a boxed warning against
group. There is no clear biological plausibility
using romosozumab in anyone who has had a
for a possible increase in cardiovascular events
myocardial infarction in the preceding 12 months
with romosozumab and no established causality,
Romosozumab is a humanized monoclonal although the presence of sclerostin in blood vessels
antibody to sclerostin, a cytokine expressed by and vascular smooth muscle is a possible factor.20
osteocytes. Sclerostin reduces osteoblastic bone It has also been postulated that alendronate may
formation by binding to LRP5/6, inhibiting provide cardiovascular protection, but this is
Wnt signaling and stimulating bone resorption, not clear from other studies. Due to the ARCH
in part by up-regulating RANKL, a stimulator data, there is a boxed warning that romosozumab
of osteoclast formation, activity, and survival. may increase the risk of myocardial infarction,
Romosozumab was approved by the US FDA in stroke, and cardiovascular death and should not
April 2019 for treatment of postmenopausal women be initiated in patients who have had a myocardial
with osteoporosis who are at high risk of fracture. infarction or stroke within the preceding year
The mechanism of action is novel: by inhibiting (Answer B). The package label also advises to
sclerostin, it has a dual action of stimulating “monitor for signs and symptoms of myocardial
bone formation and reducing bone resorption. infarction and stroke and instruct patients to seek
Bone formation is increased when sclerostin is prompt medical attention if symptoms occur,”
inhibited (thus, Answer D is incorrect). The but there are no guidelines as to how to do so.
phase 3 Fracture Study in Postmenopausal Since there is no evidence of increased risk of
Women with Osteoporosis (FRAME) trial osteosarcoma with sclerostin inhibition in rats
examined romosozumab for 12 months followed or humans, there is no warning against its use in
by denosumab for 12 months vs placebo for patients previously receiving radiation therapy
12 months followed by denosumab for 12 months or otherwise at high risk of osteosarcoma (thus,
in 7180 postmenopausal women.18 In the first year, Answer C is incorrect). Atypical femoral fractures
the romosozumab group had a 73% reduction in and osteonecrosis of the jaw were reported during
vertebral fracture incidence but had no significant both the FRAME and ARCH studies, thought
differences in nonvertebral fractures. Another to be related to the antiresorptive action (thus,
phase 3 trial, Active-Controlled Fracture Study Answer A is incorrect). Romosozumab is given
in Postmenopausal Women with Osteoporosis subcutaneously by a health care professional once
at High Risk (ARCH), compared romosozumab monthly for 12 months and should be followed
to alendronate for a year followed by a year of up with an additional osteoporosis treatment to
alendronate in both groups.19 This group of maintain or enhance its effects.

104  ENDO 2021 • Endocrine Case Management

chased by Dr. Khalid H. Seedahmed, Jr., MRCP PGDip, CertEndocrinology SA - ID 283


References
1. Eastell R, Rosen CJ, Black DM, Cheung AM, Murad MH, Shoback D. 11. Clark W, Bird P, Gonski P, et al. Safety and efficacy of vertebroplasty for
Pharmacological management of osteoporosis in postmenopausal women: acute painful osteoporotic fractures (VAPOUR): a multicentre, randomised,
an Endocrine Society clinical practice guideline. J Clin Endocrinol Metab. double-blind, placebo-controlled trial. Lancet. 2016;388(10052):1408-1416.
2019;104(5):1595-1622. PMID: 30907953 PMID: 27544377
2. Cosman F, de Beur SJ, LeBoff MS, et al; National Osteoporosis Foundation. 12. Saag KG, Shane E, Boonen S, et al. Teriparatide or alendronate in
Clinician’s guide to prevention and treatment of osteoporosis. Osteoporos Int. glucocorticoid-induced osteoporosis. N Engl J Med. 2007;357(20):2028-2039.
2014;25(10):2359-2381. PMID: 25182228 PMID: 18003959
3. Camacho PM, Petak SM, Binkley N, et al. American Association of 13. Rothman MS, Olenginski TP, Stanciu I, Krohn K, Lewiecki EM. Lessons
Clinical Endocrinologists and American College of Endocrinology clinical learned with Bone Health TeleECHO: making treatment decisions when
practice guidelines for the diagnosis and treatment of postmenopausal guidelines conflict. Osteoporos Int. 2019;30(12):2401-2406. PMID: 31471665
osteoporosis - 2016--executive summary. Endocr Pract. 2016;22(9):1111-1118. 14. Anastasilakis AD, Polyzos SA, Makras P, Aubry-Rozier B, Kaouri S, Lamy O.
PMID: 27643923 Clinical features of 24 patients with rebound-associated vertebral fractures
4. Qaseem A, Forciea MA, McLean RM, Denberg TD; Clinical Guidelines after denosumab discontinuation: systematic review and additional cases. J
Committee of the American College of Physicians. Treatment of low bone Bone Miner Res. 2017;32(6):1291-1296. PMID: 28240371
density or osteoporosis to prevent fractures in men and women: a clinical 15. Ferrari S, Libanati C, Lin CJF, et al. Relationship between bone mineral
practice guideline update from the American College of Physicians. Ann Intern density T-score and nonvertebral fracture risk over 10 years of denosumab
Med. 2017;166(11):818-839. PMID: 28492856 treatment. J Bone Miner Res. 2019;34(6):1033-1040. PMID: 30919997
5. Dowd R, Recker RR, Heaney RP. Study subjects and ordinary patients. 16. Reid IR, Horne AM, Mihov B, Gamble GD. Bone loss after denosumab: only
Osteoporos Int. 2000;11(6):533-536. PMID: 10982170 partial protection with zoledronate. Calcif Tissue Int. 2017;101(4):371-374.
6. Miller PD. Underdiagnosis and undertreatment of osteoporosis: the battle to PMID: 28500448
be won. J Clin Endocrinol Metab. 2016;101(3):852-859. PMID: 26909798 17. Freemantle N, Satram-Hoang S, Tang ET, et al; DAPS Investigators. Final
7. Khosla S, Shane E. A crisis in the treatment of osteoporosis. J Bone Miner Res. results of the DAPS (Denosumab Adherence Preference Satisfaction)
2016;31(8):1485-1487. PMID: 27335158 study: a 24-month, randomized, crossover comparison with alendronate in
8. Khosla S, Cauley JA, Compston J, et al. Addressing the crisis in the treatment postmenopausal women. Osteoporos Int. 2012;23(1):317-326. PMID: 21927922
of osteoporosis: a path forward. J Bone Miner Res. 2017;32(3):424-430. 18. Cosman F, Crittenden DB, Adachi JD, et al. Romosozumab treatment in
PMID: 28099754 postmenopausal women with osteoporosis. N Engl J Med. 2016;375(16):1532-
9. Lewiecki EM, Boyle JF, Arora S, Bouchonville MF 2nd, Chafey DH. 1543. PMID: 27641143
Telementoring: a novel approach to reducing the osteoporosis treatment gap. 19. Saag KG, Petersen J, Brandi ML, et al. Romosozumab or alendronate
Osteoporos Int. 2017;28(1):407-411. PMID: 27439373 for fracture prevention in women with osteoporosis. N Engl J Med.
10. Ebeling PR, Akesson K, Bauer DC, et al. The efficacy and safety of vertebral 2017;377(15):1417-1427. PMID: 28892457
augmentation: a second ASBMR Task Force report. J Bone Miner Res. 20. Rosen CJ. Romosozumab - promising or practice changing? N Engl J Med.
2019;34(1):3-21. PMID: 30677181 2017;377(15):1479-1480. PMID: 28892459

ENDO 2021 • Bone and Mineral Metabolism  105

chased by Dr. Khalid H. Seedahmed, Jr., MRCP PGDip, CertEndocrinology SA - ID 283


Is There an Ideal Vitamin D
Level and Dosage for All
Circumstances?
Ghada El-Hajj Fuleihan, MD, MPH. American University of Beirut, Beirut, Lebanon;
E-mail: gf01@aub.edu.lb

Learning Objectives vitamin D nutritional status). Higher dosages


of 1000 to 2000 IU daily may be needed in
As a result of participating in this session, learners
populations at high risk and are safe.
should be able to:
• Vitamin D given in pregnancy probably
• Describe the clinical manifestations of reduces the risk of preeclampsia (± calcium),
hypovitaminosis D. gestational diabetes, low neonatal birth weight
• Explain differences in vitamin D assays and the (<2500 g), and severe postpartum bleeding.
impact on practice. • There is no benefit of vitamin D
• Determine efficacy of vitamin D supplementation alone on fractures or falls in
supplementation across the lifetime. ambulatory adults younger than 65 years.
• Determine a desirable serum • There is no clear benefit of vitamin D in
25-hydroxyvitamin D range based on major combination with calcium on fractures or falls
musculoskeletal outcomes. in ambulatory adults younger than 65 years.
• Discuss evidence regarding the putative non- • Vitamin D co-administered with calcium
skeletal benefits of vitamin D supplementation. reduces the risk of hip fractures and any
fractures.
• Discuss the evidence regarding the role of
vitamin D supplementation in the COVID-19 • Vitamin D supplementation has no beneficial
pandemic. effect on nonclassic outcomes.
• Underscore knowledge gaps and future • To date there is no good evidence of a
directions. beneficial effect of vitamin D on COVID-19–
related complications.
• Routine measurement of serum
25-hydroxyvitamin D (25-OHD) is not
Main Conclusions indicated in the general population.
• Hypercalcemia and hypercalciuria are unlikely
• Vitamin D, at dosages of 400 IU daily, prevents
to occur at dosages of up to 2000 IU daily. The
rickets in infants.
upper tolerable level is 4000 IU daily, and the
• Modest dosages of 400 to 600 IU daily VITAL trial demonstrated an increased risk of
are safe in children, adolescents, and kidney stones at such dosages.
adults. Such dosages prevent secondary
hyperparathyroidism (in case of fluctuations in

106  ENDO 2021 • Endocrine Case Management

chased by Dr. Khalid H. Seedahmed, Jr., MRCP PGDip, CertEndocrinology SA - ID 283


• Evidence gaps exist regarding children, Of the circulating vitamin D metabolites,
pregnant women, and community-dwelling serum 25-OHD is the most abundant, has the
elderly individuals with low vitamin D. longest half-life (2-3 weeks), and reflects both skin
• Individual patient meta-analyses and synthesis and dietary intake. It is the metabolite
randomized controlled trials are needed to of interest relating vitamin D nutritional status to
shed light on areas of unclear evidence. outcomes.

Figure. Vitamin D Metabolic Pathway

Significance of the Supplement DIET


Clinical Problem Vitamin 02-03 Vitamin 03

.. ~~ ~
Background
Normal Physiology ~~
Vitamin D, a steroid hormone that controls
more than several hundreds of genes, amounting
to around 3% of mouse and human genomes,
impacts a wide range of molecular and cellular
functions.1 It is a pre-hormone derived from diet
or skin (sun exposure) whose active metabolite, 24, 25(0H)i03
1,25-dihydroxyvitamin D (1,25[OH]2D), has a 1,24, 2S(0Hh03
(Inactive)

critical role in calcium and mineral homeostasis,


bone modeling, and remodeling. The dietary
intake is usually low, as few food items contain or
are fortified with vitamin D; therefore, synthesis
in the skin is the main source of vitamin D.
The vitamin D metabolic pathway consists of
With permission from El-Hajj Fuleihan, et al. J Bone Miner Res.
3 major steps mediated by hydroxylases (Figure).1 2015;30(7). © John Wiley and Sons.
It starts in the skin, where ultraviolet B radiation
initiates conversion of 7‑dehydrocholesterol Genetic Polymorphisms and Environmental
(pre-vitamin D3) to cholecalciferol or D3, Predictors of Serum 25-OHD Levels
which is biologically inactive, and undergoes Polymorphisms in key genes of the vitamin
2 hydroxylation steps before the active 1α-25- D pathway (CYP2R1, CYP27B1, CYP24A1, and
dihydroxyvitamin D3 1,25(OH)2D3 is formed. The 7-dehydrocholesterol reductase DHCR7), the
vitamin D–binding protein, also called GC, has vitamin D–binding protein, and the vitamin D
a major role in the transport of D metabolites, receptor—even when combined—only explain
including vitamin D to the liver, 25-OHD to the less than 5% of variations in 25-OHD levels.
kidneys and other organs, and 1,25(OH)2D to Conversely, environmental and lifestyle factors
target tissues. Several cytochrome P450 (CYP) account for more substantial variations, estimated
enzymes are reported as human hepatic vitamin up to 50%, in vitamin D status or serum 25-
D 25-hydroxylases (25-hydroxylase [CYP2R1], OHD levels, amounting to 3 to 15 ng/mL
1-hydroxylase [CYP27B1], 24-hydroxylase (7.5-37.4 nmol/L). Consistent predictors of
[CYP24A1]) (Figure).2,3 As cellular concentrations low 25-OHD levels are extremes of age, gender,
of 1,25(OH)2D3 increase, it activates the pregnancy in Middle East, UVB/sun exposure,
cytoplasmic vitamin D receptor and turns on a season, pollution, clothing style, high BMI, lower
number of genes the receptor transcribes. socioeconomic status, skin pigmentation, race,

ENDO 2021 • Bone and Mineral Metabolism  107

chased by Dr. Khalid H. Seedahmed, Jr., MRCP PGDip, CertEndocrinology SA - ID 283


and ethnicity.2 CYP2R1 was initially considered World Health Organization action plans to correct
to be constitutively expressed and therefore the micronutrient deficiencies in the world.3
concentration of serum 25-OHD was thought Milder deficiency (hypovitaminosis D or vitamin
to be substrate dependent. Recent studies reveal D insufficiency) is more prevalent worldwide.
that CYP2R1 is under tight control of metabolic A systematic review of 195 studies involving more
signals.3 Indeed, obesity, type 1 and type 2 diabetes than 168,000 participants from 44 countries revealed
mellitus, fasting, and glucocorticoid therapy are considerable variation in mean 25-OHD values.
associated with lower serum 25-OHD levels. Around 37% of studies reported mean values
Compared with nonobese mice, obese mice fed a below 20 ng/mL (<50 nmol/L), with proportions
high-fat diet have been shown to have lower levels being higher in the Middle East and Asia.8
of CYP2R1 mRNA protein expression. Variations
in CYP2R1 expression may explain the variability
in serum 25-OHD levels in patients with similar
Barriers to Optimal Practice
lifestyles, as well as the wide variability observed Major organizations and societies have developed
in response of serum 25-OHD levels to similar guidelines for recommended desirable 25-OHD
vitamin D dosage regimens.3 levels and age-specific vitamin D dosages. Their
implementation and applicability across age groups
and populations are variable for the following reasons:
Overview
The implementation of vitamin D • Poor recognition of the problem, as
supplementation for all infants and small hypovitaminosis D is in large part a subclinical
children largely eliminated nutritional rickets disease.
in Europe and North America.4 Nutritional • Multitude of guidelines with contradicting
rickets, from simple vitamin D deficiency, or recommendations.
combined calcium and vitamin D deficiency, • Knowledge gaps in various age groups
however, persists in Asian countries and in the (children, pregnant women, community-
Middle East, while calcium deficiency rickets dwelling adults).
dominates in many African and some Asian
countries. Refugees, immigrants, and persons • Concerns regarding safety and fear of
with a darker skin complexion are at risk for this cardiovascular diseases.
disease. Reported rates for rickets vary between
1% and 25% (Mongolia) worldwide; however,
such estimates are derived from older studies Strategies for Diagnosis,
that were, for the most part, not population- Therapy, and/or Management
based studies.3 The prevalence of severe vitamin
Diagnosis
D deficiency across the globe (levels less than
10-12 ng/mL [25-30 nmol/L]) in these high risk Nutritional rickets presents with the classic
groups is estimated at 7%.4-7 Osteomalacia, the phenotype of musculoskeletal manifestations.
adult equivalent of rickets, also persists in these Once the infant starts crawling and walking,
high-risk populations/regions. The vitamin D bowing, fractures, and knock knees become
community unanimously supported an action apparent.4 X-rays reveal fraying of the
plan to prepare a dedicated memorandum to ask metaphyses and cupping of the epiphyseal
the World Health Organization to approve and plates.4,7 Osteomalacia in adults presents with
implement an action plan “to eradicate nutritional long bone pain, proximal myopathy, bowing, and
rickets by 2030” (23rd Vitamin D Workshop, New pseudofractures or Looser zone radiographically.9
York, May 2019). This would align with other Severe cases can present as cardiomyopathy. The
classic biochemical profile of both conditions

108  ENDO 2021 • Endocrine Case Management

chased by Dr. Khalid H. Seedahmed, Jr., MRCP PGDip, CertEndocrinology SA - ID 283


includes low levels of calcium, phosphate, and in terms of the desirable 25-OHD level, set at
25-OHD (below 10-12 ng/mL [25-30 nmol/L]). 30 ng/mL (75 nmol/L) by the Endocrine Society
If levels are higher, then rickets is exclusively due and 20 ng/mL (50 nmol/L) by the Institute of
to calcium deficiency or combined calcium and Medicine. They also differ in recommended
vitamin D insufficiency. Permanent consequences dosages to achieve these levels, varying between
include hypomineralized teeth and stunted growth 600 and 800 IU for the Institute of Medicine
in children. Bone histomorphometry is the gold and between 600 and 2000 IU for the Endocrine
standard to confirm the diagnosis, and double Society.17,18 The differences in desirable 25-OHD
tetracycline labeling reveals unmineralized osteoid levels derived by the Institute of Medicine and the
bone and increased mineralization lag time. Endocrine Society may only in part be explained
Radiologic studies allow for early diagnosis and by different target populations they addressed:
scoring of the disease severity (Thatcher score, 4). the general public (Institute of Medicine) vs
While it is universally agreed that prevention individuals at high risk (Endocrine Society). We
and treatment of rickets and osteomalacia are clearly have provided a detailed synthesis of the approach
indicated, the approach to vitamin D insufficiency used by these organizations, and the basis for their
(12-20 ng/mL [30-50 nmol/L]) is quite variable. recommendations by outcome of interest.2 Finally,
In most cases, vitamin D insufficiency may have while most, if not all, current vitamin D guidelines
no deleterious impact on health, while in others consider age, gender, and reproductive status, none
it could present initially as subclinical disease, has factored in any of the other modulators of
progressing from no to subtle biochemical vitamin D into their recommendations for desirable
abnormalities. Serious potential concerns are silent levels, with the exception of the Australian
adverse outcomes on maternal-neonatal health, guidelines that take season into account.15 Patients
bone mass in infants and adolescents, and chronic with obesity require higher dosages.19
outcomes such as secondary hyperparathyroidism
and fractures (including hip fractures in elderly Impact of Assay Variation on
high-risk individuals). Therefore, examination of
the evidence for treatment efficacy and adequate Management Strategies
supplementation is indicated. Relating the Although serum 25-OHD is the best index of
desirable serum 25-OHD level to outcomes and nutritional vitamin D status, its measurement to
recommended dosages has been quite challenging define desirable vitamin D levels is limited by large
because of the scarce evidence relating 25-OHD to variations incurred by the various methodologies
outcomes and the wide variability in achieved 25- used to date. Such variations by far exceed the
OHD levels with identical dosages.10-13 differences in desirable 25-OHD levels defined
by the Institute of Medicine and the Endocrine
Society. DEQAS (Vitamin D External Quality
Management Strategies/ Assessment Scheme, http://www.deqas.org) is
Vitamin D Guidelines an international quality assurance program that
The Endocrine Society, the Institute of evaluates performance of various 25-OHD assays,
Medicine, and many organizations and societies including the older radioimmunoassays, platform
have developed guidelines.14-18 The Endocrine automated assays, liquid chromatography, and
Society and Institute of Medicine guidelines are the gold standard liquid chromatography/
based on an extensive review of the literature, tandem mass spectrometry. Each quarter,
but their applicability across age groups and DEQAS sends 5 unknown 25-OHD samples to
populations worldwide are still debated. They more than 1100 laboratories partaking in its
use similar predefined outcomes of mineral quality assurance program. These unknown
metabolism, yet reach differing conclusions samples are run by participating laboratories

ENDO 2021 • Bone and Mineral Metabolism  109

chased by Dr. Khalid H. Seedahmed, Jr., MRCP PGDip, CertEndocrinology SA - ID 283


based on their specific method and the results Should We Screen by Measuring
are mailed back to DEQAS. Once all results are Serum 25-OHD Levels?
received, DEQAS generates a quarterly general
report with details on method-specific mean The Institute of Medicine defines a desirable
25-OHD values obtained for each method 25-OHD level to be 20 ng/mL (50 nmol/L), and
separately and specifies the National Institute of the Endocrine Society defines a desirable level
Standards and Technology target value for each to be 30 ng/mL (75 nmol/L). Desirable levels
unknown sample. We had previously reported by other organizations fall, for the most part,
that serum values measured in parallel with 2 in between these 2 cutoff values. All guidelines
assays could vary between –38 to +19 ng/mL, a concur that screening for vitamin D deficiency is
bias that was independent of the serum 25-OHD not recommended for the general population and
level.2 Furthermore, certain clinical situations should be reserved for high-risk individuals. High
affect how different assays perform. Examples risk is defined slightly differently by the various
include pregnancy, hemodialysis, acute organizations.2,5 Measuring serum 25-OHD does
inflammatory states, COVID-19, and illness not fulfill the World Health Organization criteria
severity necessitating hospitalization in the for a screening test.
intensive care unit. The implications of such
wide variability on the interpretation of results Treatment of Symptomatic Patients
from trials and the relevance and applicability Prevention and Treatment of Rickets
of guidelines and recommendations, at the Nutritional rickets is treated with calcium and
public health or individual level, are substantial. vitamin D, and dosages are modified according to
Mandatory requirement for vitamin D quality patient age and adherence. High dosages of 1000 to
assurance protocols in all clinical laboratories, 6000 IU daily for 3 months followed by 400 to
standardization, and cross-calibration to a 600 IU daily are preferred to intermittent therapy,
universal NIST standard (National Institute of but both can heal rickets.
Standards and Technology) are pressing priorities.

Table 1. Recommended Vitamin D Dosages in Children by Organization

Desirable 25-OHD = 20 ng/mL (50 nmol/L) Desirable 25-OHD = 30 ng/mL (75 nmol/L)
Institute of Medicine Recommendations Endocrine Society Recommendations

Age Group AI EAR RDA Upper Limit Daily Requirement Upper Limit

Infants

0-6 months 400 IU 1000 IU 400-1000 IU 2000 IU

6-12 months 400 IU 1500 IU 400-1000 IU 2000 IU

Children

1-3 years 400 IU 600 IU 2500 IU 600-1000 IU 4000 IU

4-8 years 400 IU 600 IU 3000 IU 600-1000 IU 4000 IU

9-13 years 400 IU 600 IU 4000 IU 600-1000 IU 4000 IU

14-18 years 400 IU 600 IU 4000 IU 600-1000 IU 4000 IU

Abbreviations: AI, adequate intake; EAR, estimated average requirement; RDA, recommended dietary allowance. AI is the amount of
a nutrient that will meet an individual’s daily requirements. It is used when an RDA cannot be determined. EAR is the intake level for a
nutrient at which the needs of 50% of the population are met. RDA is the intake at which 97.5% of the population needs are met.

110  ENDO 2021 • Endocrine Case Management

chased by Dr. Khalid H. Seedahmed, Jr., MRCP PGDip, CertEndocrinology SA - ID 283


Table 2. Recommended Vitamin D Dosages in Adults by Organization

Endocrine Society Recommendations


Institute of Medicine Recommendations (for patients at risk for vitamin D deficiency)

Life Stage Group EAR RDA Upper Limit Daily Upper Limit

Adults

19-30 years 400 IU 600 IU 4000 IU 1500-2000 IU 10,000 IU

31-50 years 400 IU 600 IU 4000 IU 1500-2000 IU 10,000 IU

51-70 years 400 IU 600 IU 4000 IU 1500-2000 IU 10,000 IU

>70 years 400 IU 600 IU 4000 IU 1500-2000 IU 10,000 IU

Abbreviations: EAR, estimated average requirement; RDA, recommended dietary allowance. EAR is the intake level for a nutrient at
which the needs of 50% of the population are met. RDA is the intake at which 97.5% of the population needs are met.

Treatment of Osteomalacia has diffuse musculoskeletal tenderness, tenderness


Several regimens are used and all can heal over the shins, waddling gait, and muscular
osteomalacia effectively. A widely used regimen weakness. She is unable to stand from a sitting
consists of giving 50,000 IU weekly for 6 to 8 position. Deep tendon reflexes are +4. Chvostek
weeks followed by 800 IU daily maintenance and Trousseau signs are both positive.
therapy.
Laboratory test results (sample drawn at
admission):
Treatment of Asymptomatic
Calcium = 6.9 mg/dL (8.5-10.5 mg/dL)
Individuals (SI: 1.7 mmol/L [2.1-2.6 mmol/L])
The treatment of asymptomatic individuals Phosphate = 2.2 mg/dL (2.9-5.2 mg/dL)
is described in Tables 1 and 2. These (SI: 0.7 mmol/L [0.9-1.7 mmol/L])
Alkaline phosphatase = 820 U/L (35-120 U/L)
recommendations apply to Caucasian populations. (SI: 13.7 µkat/L [0.6-2.0 µkat/L])
Albumin = 3.8 g/dL (3.6-5.3 g/dL)
(SI: 38 g/L [36-53 g/L])
Clinical Case Vignettes Creatinine = 0.9 mg/dL (0.6-1.2 mg/dL)
Case 1 (SI: 79.6 µmol/L [53.0-106.1 µmol/L])
Urinary calcium = 30 mg/24 h (0-250 mg/24 h)
A 51-year-old woman presents in a wheelchair (SI: 0.75 mmol/d [0-6.3 mmol/d])
with a 1-year history of low back pain, hip and Urinary phosphate = 770 mg/24 h
pelvic pain upon walking, difficulty getting up (400-1300 mg/24 h) (SI: 248.7 mmol/d
[129.2-419.9 mmol/d])
from a chair, and inability to climb stairs. The Urinary creatinine = 1.4 g/24 h (0.6-2.0 g/24 h)
pain and weakness have been progressive over the (SI: 12.4 mmol/d [5.3-17.7 mmol/d])
past year, interfering with daily activities, until 25-OHD = <2 ng/mL (desirable >20 ng/mL, not to
few weeks before admission when she became exceed 60 ng/mL) (SI: <5 nmol/L [>50 nmol/L,
completely homebound. She does not like dairy not to exceed 150 nmol/L])
PTH = 250 pg/mL (8-76 pg/mL) (SI: 250 ng/L
products, spends little time outdoors, and uses a [8-76 ng/L])
concealed clothing style. She has had no diarrhea,
no bowel surgery, and no intake of anticonvulsant D-xylose test is negative, stool test for Sudan
agents. She had total thyroidectomy 20 years ago stain is negative, assessment for endomysial and
for multinodular goiter. transaminase antibodies is negative, and intestinal
On physical examination, she appears healthy biopsy shows no flattening of the villi. X-ray of the
and is in no distress. Her BMI is 31 kg/m2. She pelvis shows a generalized decrease in bone density

ENDO 2021 • Bone and Mineral Metabolism  111

chased by Dr. Khalid H. Seedahmed, Jr., MRCP PGDip, CertEndocrinology SA - ID 283


and bilateral pseudofractures (Looser zones) in Answer: C) Months
the pelvic rami. Bone mineral density T-scores are
–2.6 in the spine and –2.4 in the hip (NHANES).
Question 5: How do you expect her bone
Question 1: Which of the following is mineral density will be affected after
this patient’s most likely diagnosis? replacement with calcium and vitamin D?
A. Obesity A. No change
B. Malabsorption B. Decrease
C. Nutritional osteomalacia C. Increase by 3% to 5% at the spine and 1% to 2%
at the hip
D. X-linked hypophosphatemic rickets
D. Substantial increments with normalization
E. Hypoparathyroidism
Answer: D) Substantial increments with normalization
Answer: C) Nutritional osteomalacia
This vignette is based on a real case report.9
The patient has the classic clinical, biochemical,
Question 2: Does this patient
radiologic, and densitometric presentation of
have osteoporosis?
osteomalacia. This condition, almost eradicated
A. Yes in western populations due to vitamin D
B. No supplementation, is still prevalent in immigrants
and certain populations (Middle East, India,
Answer: B) No
Asia). She was treated with calcium and vitamin
D, 50,000 IU weekly for 1.5 years. Her symptoms
Question 3: In addition to daily resolved and she was able to walk within few
calcium, which of the following days. Serum calcium and phosphate normalized
treatments would you recommend? within days, PTH and alkaline phosphatase took
A. Vitamin D at 1000 IU daily more than a year.9 DXA does not distinguish
B. Vitamin D at 50,000 IU daily osteomalacia from osteoporosis. She experienced
substantial increments in bone mineral density
C. Vitamin D at 50,000 IU weekly for several within 4 months, both at the spine and hip,
weeks followed by 600 to 1000 IU daily with steady increments thereafter. Her T-score
D. Stoss therapy, 600,000 IU once now and every normalized completely by 14 months. Such
6 to 12 months changes reflect healing of osteomalacia and not
cure of osteoporosis.
Answer: C) Vitamin D at 50,000 IU weekly for
several weeks followed by 600 to 1000 IU daily
Case 2
Question 4: When do you expect A 32-year-old woman comes to clinic for routine
normalization of her serum PTH and check-up and advice. She is otherwise healthy
alkaline phosphatase after treatment? and takes no medication. She does consume any
A. Days vitamin D–fortified food, uses sunscreen, and does
not sit in the sun. She has 2 children (ages 10 years
B. Weeks
and 5 years) and is planning another pregnancy
C. Months soon. Her son’s friend had an arm fracture after
D. Years falling off a swing. She would like your advice
E. Never regarding vitamin D supplementation dosages/
regimens for her and family members.

112  ENDO 2021 • Endocrine Case Management

chased by Dr. Khalid H. Seedahmed, Jr., MRCP PGDip, CertEndocrinology SA - ID 283


Question 1: Are there differences between Question 5: Does vitamin D supplementation
the following regimens given as daily during pregnancy improve neonatal
(1500 IU daily), weekly (10,500 IU weekly), outcomes such as birth height or
or monthly (45,000 IU daily) in adults? neonatal bone mineral content?
A. Yes A. Yes
B. No B. Possibly
C. No
Answer: B) No
Answer: B) Possibly
Question 2: Are vitamin D3 based
supplements superior to D2 Question 6: This patient plans to breastfeed
supplements in terms of efficacy? her third child and would like to know
A. Yes the recommended dosage of vitamin D
B. No to prevent rickets in her newborn. The
universally recommended dosage in
C. Unclear
breastfed infants is which of the following?
Answer: A) Yes A. 200 IU daily
B. 400 IU daily
Question 3: What is the recommended total C. 600 IU daily
daily intake of vitamin D during pregnancy? D. 1000 IU daily
A. 100 IU daily
Answer: B) 400 IU daily
B. 600 IU daily
C. 5000 IU daily
Question 7: What is the recommended
D. 10,000 IU daily
total daily recommended vitamin D
Answer: B) 600 IU daily intake for the 5-year-old and 10-year-
old going into adolescence?
A. 400-600 IU daily
Question 4: Does vitamin D supplementation
during pregnancy decrease the risk of B. 800 IU daily
pregnancy-associated complications, C. 2500 IU daily
such as preeclampsia, gestational D. 4000 IU daily
diabetes, and cesarean delivery rates? E. None of the above
A. Yes
B. Possibly Answer: A) 400-600 IU daily
C. No
Question 8: Does vitamin D supplementation
Answer: B) Possibly decrease the risk of fracture in
apparently healthy children?
A. Yes
B. No
C. No data
Answer: C) No data

ENDO 2021 • Bone and Mineral Metabolism  113

chased by Dr. Khalid H. Seedahmed, Jr., MRCP PGDip, CertEndocrinology SA - ID 283


Prevention of rickets in infants is through early that vitamin D probably reduces the risk of
supplementation with 400 IU daily in infancy preeclampsia (same for calcium/vitamin D),
to be increased to 600 IU daily after age 1 year. gestational diabetes, and the risk of having a baby
While the efficacy of vitamin D treatment in with a birth weight less than 2500 g compared
infants and children suffering from rickets is with no intervention or placebo. High-quality
unequivocal, the effect of supplementation in research is needed, especially in high-risk
improving bone mineral in apparently healthy populations/ethnic groups. To date, the Institute
children and adolescents is yet to be determined.2 of Medicine recommends a recommended daily
A meta-analysis of 6 randomized controlled trials allowance of 600 IU of vitamin D daily, and the
with a total of 884 healthy adolescents, mostly Endocrine Society recommends 1500 to 2000 IU
girls (mean age 10 to 13 years), on vitamin D daily. The World Health Organization 2012
supplementation at dosages of 132 IU to 400 IU vitamin D guidelines in pregnancy recommend
daily (one study included a 2000 IU daily arm) against routine vitamin D supplementation.2,23,24
had no statistically significant effects on total One umbrella review had previously reported
body bone mineral content or on bone mineral a consistent effect of maternal vitamin D
density of the hip or forearm.20 There was a supplementation on neonatal birth weight,
trend to a small effect on lumbar spine bone whether considering observational studies or
mineral density. The investigators concluded randomized controlled trials.25
that it is unlikely that vitamin D supplements are
beneficial to bone mineral density in children and Case 3
adolescents with normal mean serum 25-OHD
levels. The planned subgroup analyses suggest that A 65-year-old woman presents to the osteoporosis
vitamin D supplementation of deficient children clinic for advice. She is active and has a healthy
and adolescents could result in clinically useful diet. She has no personal history of fracture, but
beneficial effect on lumbar spine bone mineral her mother had a hip fracture at age 75 years.
content and total body in a subgroup of children FRAX estimates her risks of major osteoporotic
with lower serum 25-OHD levels.20 One of the fracture and hip fracture (with bone mineral density)
trials compared placebo, low-dosage treatment to be below the country-specific intervention
(200 IU daily), and high-dosage treatment (2000 IU threshold. She has a history of osteoarthritis. She
daily), on bone density at 1 year in adolescent girls, takes calcium, 1000 mg daily, and vitamin D, 600 IU
with a mean baseline 25-OHD level of 14 ng/mL daily, prescribed by her rheumatologist. She read on
(35 nmol/L). Lean mass increased significantly in the Internet that vitamin D may decrease the risk
both treatment groups, and bone area and total of cardiovascular events and cancer. However, she
hip bone mineral density increased in the high- was told by her primary care physician at her last
dosage group.21 To date, recommended dosages check-up to discontinue her calcium and vitamin
range from various organizations vary between D supplementation because of concerns of an
600 to 2000 IU daily depending on organization increased risk of cardiovascular events.
and region, and dosage would probably depend
on ethnicity, latitude, and lifestyle (Table 1).14 An Question 1: Does vitamin D administered
ongoing individual patient meta-analysis should with calcium decrease the risk of hip fracture
shed light on the efficacy of vitamin D on bone in community-dwelling individuals?
mineral density in apparently healthy children A. Yes
and adolescents, at a critical time of bone mass B. No
accretion.22 There are no data on fractures. C. No evidence available
The most recent Cochrane systematic
review included 30 trials (n = 7033) and revealed Answer: B) No

114  ENDO 2021 • Endocrine Case Management

chased by Dr. Khalid H. Seedahmed, Jr., MRCP PGDip, CertEndocrinology SA - ID 283


Question 2: Does vitamin D co- decreased gastrointestinal absorption and renal
administered with calcium increase function all contribute to decreased 25-OHD
the risk of cardiovascular events? levels. Recommended dosages vary between 600
A. Yes to 2000 IU daily (Table 2). There is no evidence
B. No that higher dosages are beneficial in any of the
outcomes studies. Intermittent high dosages could
C. Possibly
be harmful, increasing fall risk. It is noteworthy
Answer: B) No that all pivotal phase 3 osteoporosis trials that led
to drug approval by the US FDA co-administered
calcium and vitamin D in their treatment arms.2
Question 3: Does vitamin D Despite the long, staggering list of observational
decrease the risk of cancer? studies relating low 25-OHD levels to chronic
A. Yes diseases, several randomized controlled trials
B. No conducted in adults (VITAL [n = 25,871; 2000 IU
C. Possibly daily; median follow-up 5.3 years], D2D [n =
2435; prediabetes; 3.3 years; 4 000 IU daily;
Answer: B) No median 2.5 years], ViDA [n = 5110; 50-84 years;
bolus 200,000 IU then 100,000 monthly; median
Vitamin D combined with calcium reduces the
3.3 years]) reveal no beneficial effect of vitamin
risk of hip fracture by 16% to 33% and of any
D compared with placebo on the incidence of
fracture by 5% to 19%,14 while the effect of vitamin
cardiovascular events, invasive cancer, diabetes,
D supplementation on falls is less inconsistent.
falls, and nonvertebral fractures.27-30 There is
The effect on fractures is only demonstrated
also no evidence that vitamin D increases the
when combining trials in community-dwelling
risk of cardiovascular events or blood vessel
and institutionalized individuals and is potentially
calcification in elderly patients. Mean 25-OHD
driven by data from institutionalized individuals
levels at study entry were 20 ng/mL or greater
in 3 large systematic reviews and meta-analyses.15
(>50 nmol/L) in all 3 trials. Ongoing mega-trials
The best evidence is from a Cochrane meta-
include D-Health in Australia, DO-HEALTH in
analysis.26 The administration of calcium with
Europe, and FIND in Finland. These trials are
vitamin D reduced the risk of hip fractures
evaluating the impact of vitamin D at various
(relative risk, 0.84; 95% CI, 0.74-0.96) and any
dosages on all-cause mortality, nonvertebral
fracture (relative risk, 0.95; 95% CI, 0.90-0.99) in
fractures, cognitive decline, functional decline,
elderly individuals. The derived number needed
blood pressure, rate of infections, cardiovascular
to treat is 54/1000 persons per year in individuals
disease, and cancer as primary outcomes. A 2019
at high risk and 1/1000 persons per year in
meta-analysis that included the VITAL and
individuals at low risk. An example of a population
ViDA trials showed that compared with placebo,
at high risk would be older institutionalized
vitamin D supplementation did not reduce the
individuals who are likely to have low 25-OHD
risk of major adverse cardiovascular events.31 A
levels and secondary hyperparathyroidism. In all
Cochrane systematic review and meta-analysis
3 meta-analyses, the evidence is limited by major
of 18 randomized controlled trials of mostly
shortcomings and heterogeneity, including the
elderly community-dwelling women revealed
population studied, dosages, and types of vitamin
that vitamin D did not have any significant effect
D used. The impact of routine supplementation
on cancer incidence.32 Finally, a Cochrane meta-
on musculoskeletal outcomes in community-
analysis of 56 randomized controlled trials with
dwelling individuals remains unclear. With aging,
95,286 participants consisting mostly of women
changes in diet and lower sun exposure, decreased
older than 70 years revealed that vitamin D,
ability of the skin to synthesize vitamin D, and

ENDO 2021 • Bone and Mineral Metabolism  115

chased by Dr. Khalid H. Seedahmed, Jr., MRCP PGDip, CertEndocrinology SA - ID 283


administered over 4 years, decreased mortality Answer: C) No evidence available
(relative risk, 0.97; 95% CI, 0.94-0.99).33
The dosages recommended in both tables have Disease severity and mortality due to COVID-19
been shown to be safe. Higher dosages do not incur infection are greater in elderly and chronically
superior efficacy and may actually have an adverse ill patients, populations at high risk for vitamin
effect on health outcomes.31-34 Mild hypercalcemia D deficiency. Several reports have suggested a
was reported in 3% and 9% of patients receiving possible association between vitamin D deficiency
4000 and 10,000 IU daily in the D2D and Calgary (25[OH]D levels <20 ng/mL [<50 nmol/L]) and
studies, respectively.28,34 The mean 25-OHD levels COVID-19 susceptibility.38 Vitamin D has a role in
were 28 and 32 ng/mL at entry and reached 54 and cytokine release and inflammation and modulation
80 ng/mL in these 2 trials, respectively. Dosages of innate and adaptive immunity. It may decrease
greater than 1000 IU daily may be needed in the risk of infections via several mechanisms.39
patients with low baseline levels; dosages greater Vitamin D supplementation was shown to
than 2000 IU daily are rarely necessary. decrease the risk of acute respiratory infections,
including influenza infection.40
We have just completed a systematic review
Case 4  New Content  and meta-analysis on vitamin D and COVID-19–
A 70-year-old man with obesity and diabetes related outcomes. The protocol is available
mellitus develops symptoms of fever, headache, online on PROSPERO (registration number
myalgias, and cough and subsequently tests positive CRD42020203960).41 The overwhelming evidence
for COVID-19. He has never smoked cigarettes. to date, based on these studies, mostly of low
His breathing is comfortable, oxygen saturation quality, reveals no association between vitamin
is 96%, and he is advised to self-quarantine and to D deficiency and COVID-19–related health
be managed by telemedicine. His vitamin D level outcomes. The certainty of the evidence is very
is low, and he is started on vitamin D, 50,000 IU low. We identified 32 registered ongoing clinical
weekly, by an outside physician, with the goal of trial protocols, but only 2 have published results.
preventing COVID-19–related adverse outcomes. A pilot trial suggests that calcifediol may have a
The patient would like to know whether his family protective effect regarding COVID-19–related
members who were exposed to him should take intensive care unit admissions. In conclusion, the
vitamin D and, if so, at what dosages. use of high dosages of vitamin D is unwarranted
and awaits evidence from ongoing trials.
Question 1: Does vitamin D decrease the risk Meanwhile, it would be prudent to use dosages
of COVID-19–related adverse outcomes? below the upper tolerable level of 4000 IU daily
A. Yes until forthcoming evidence becomes available.42-45
B. No Eight ongoing mega-trials are investigating
efficacy of vitamin D supplementations in
C. Weak, low-quality evidence nonhospitalized individuals who are COVID-19
D. No evidence available positive or negative. Evidence to prove the
Answer: D) No evidence available putative beneficial effect of vitamin D on immune
modulation and respiratory function, which
would inform best practices on optimal dosing
Question 2: Does vitamin D decrease for various populations, is hopefully forthcoming.
the risk of developing COVID-19? We will discuss the rapidly evolving evidence to
A. Yes answer the 2 questions above.
B. No
C. No evidence available

116  ENDO 2021 • Endocrine Case Management

chased by Dr. Khalid H. Seedahmed, Jr., MRCP PGDip, CertEndocrinology SA - ID 283


References
1. Bouillon R, Marcocci C, Carmeliet G, et al. Skeletal and extraskeletal actions 20. Winzenberg T, Powell S, Shaw KA, Jones G. Effects of vitamin D
of vitamin D: current evidence and outstanding questions. Endocr Rev. supplementation on bone density in healthy children: systematic review and
2019;40(4):1109-1151. PMID: 30321335 meta-analysis. BMJ. 2011;342:c7254. PMID: 21266418
2. El Hajj Fuleihan G, Bouillon R, Clarke B, et al. Serum 25-hydroxyvitamin D 21. El-Hajj Fuleihan G, Nabulsi M, Tamim H, et al. Effect of vitamin D
levels: variability, knowledge gaps, and the concept of a desirable range. J Bone replacement on musculoskeletal parameters in school children: a randomized
Miner Res. 2015;30(7):1119-1133. PMID: 25952470 controlled trial. J Clin Endocrinol Metab. 2006;91(2):405-412. PMID: 16278262
3. Bouillon R, Bikle D. Vitamin D metabolism revised: fall of dogmas. J Bone 22. Winzenberg T, Lamberg-Allardt C, El-Hajj Fuleihan G, et al. Does
Miner Res. 2019;34(11):1985-1992. PMID: 31589774 vitamin D supplementation improve bone density in vitamin D-deficient
4. Bouillon R, Antonio L. Nutritional rickets: historic overview and plan children? Protocol for an individual patient data meta-analysis. BMJ Open.
for worldwide eradication. J Steroid Biochem Mol Biol. 2019;198:105563. 2018;8(1):e019584. PMID: 29362271
PMID: 31809867 23. Palacios C, Kostiuk LK, Pena-Rosas JP. Vitamin D supplementation for
5. Zhang W, Stoecklin E, Eggersdorfer M. A glimpse of vitamin D status in women during pregnancy. Cochrane Database Syst Rev. 2019;7:CD008873.
mainland China. Nutrition. 2013;29(7-8):953-957. PMID: 23594582 PMID: 31348529
6. Arabi A, El Rassi R, El-Hajj Fuleihan G. Hypovitaminosis D in developing 24. Harvey NC, Holroyd C, Ntani G, et al. Vitamin D supplementation in
countries-prevalence, risk factors and outcomes. Nat Rev Endocrinol. pregnancy: a systematic review. Health Technol Assess. 2014;18(45):1-190.
2010;6(10):550-561. PMID: 20852586 PMID: 25025896
7. Chakhtoura M, Rahme M, Chamoun N, El-Hajj Fuleihan G. Vitamin D in the 25. Theodoratou E, Tzoulaki I, Zgaga L, Ioannidis JP. Vitamin D and multiple
Middle East and North Africa. Bone Rep. 2018;8:135-146. PMID: 29955632 health outcomes: umbrella review of systematic reviews and meta-analyses
8. Hilger J, Friedel A, Herr R, et al. A systematic review of vitamin D status in of observational studies and randomised trials. BMJ. 2014;348:g2035.
populations worldwide. Br J Nutr. 2014;111(1):23-45. PMID: 23930771 PMID: 24690624
9. Al-Ali H, Fuleihan GE. Nutritional osteomalacia: substantial clinical 26. Avenell A, Mak JC, O’Connell D. Vitamin D and vitamin D analogues for
improvement and gain in bone density posttherapy. J Clin Densitom. preventing fractures in post-menopausal women and older men. Cochrane
2000;3(1):97-101. PMID: 10745306 Database Syst Rev. 2014;(4):CD000227. PMID: 24729336
10. Cashman KD, Fitzgerald AP, Kiely M, Seamans KM. A systematic review and 27. Scragg R. The vitamin D assessment (ViDA) study - design and main
meta-regression analysis of the vitamin D intake-serum 25-hydroxyvitamin findings. J Steroid Biochem Mol Biol. 2019;198:105562. PMID: 31809866
D relationship to inform European recommendations. Br J Nutr. 28. Pittas AG, Dawson-Hughes B, Sheehan P, et al. Vitamin D supplementation
2011;106(11):1638-1648. PMID: 22000709 and prevention of type 2 diabetes. N Engl J Med. 2019;381(6):520-530.
11. Gallagher JC, Sai A, Templin T 2nd, Smith L. Dose response to vitamin D PMID: 31173679
supplementation in postmenopausal women: a randomized trial [published 29. Manson JE, Cook NR, Lee IM, et al. Vitamin D supplements and prevention
correction appears in Ann Intern Med. 2012;156(9):672]. Ann Intern Med. of cancer and cardiovascular disease. N Engl J Med. 2019;380(1):33-44.
2012;156(6):425-437. PMID: 22431675 PMID: 30415629
12. Zhao LJ, Zhou Y, Bu F, et al. Factors predicting vitamin D response variation 30. Bouillon R. Vitamin D and cardiovascular disorders. Osteoporos Int.
in non-Hispanic white postmenopausal women. J Clin Endocrinol Metab. 2019;30(11):2167-2181. PMID: 31402402
2012;97(8):2699-2705. PMID: 22585090 31. Barbarawi M, Kheiri B, Zayed Y, et al. Vitamin D supplementation
13. Waterhouse M, Tran B, Armstrong BK, et al. Environmental, personal, and and cardiovascular disease risks in more than 83 000 individuals in 21
genetic determinants of response to vitamin D supplementation in older randomized clinical trials: a meta-analysis. JAMA Cardiol. 2019;4(8):765-776.
adults. J Clin Endocrinol Metab. 2014;99(7):E1332-E1340. PMID: 24694335 PMID: 31215980
14. Bouillon R. Comparative analysis of nutritional guidelines for vitamin D. Nat 32. Bjelakovic G, Gluud LL, Nikolova D, et al. Vitamin D supplementation for
Rev Endocrinol. 2017;13(8):466-479. PMID: 28387318 prevention of cancer in adults. Cochrane Database Syst Rev. 2014;(6):CD007469.
15. Chakhtoura M, Chamoun N, Rahme M, Fuleihan GE. Impact of vitamin D PMID: 24953955
supplementation on falls and fractures-a critical appraisal of the quality of the 33. Bjelakovic G, Gluud LL, Nikolova D, et al. Vitamin D supplementation
evidence and an overview of the available guidelines. Bone. 2020;131:115112. for prevention of mortality in adults. Cochrane Database Syst Rev.
PMID: 31676406 2014;(1):CD007470. PMID: 24414552
16. Institute of Medicine Committee to Review Dietary Reference Intakes for 34. Maalouf J, Nabulsi M, Vieth R, et al. Short- and long-term safety of weekly
Vitamin D and Calcium. Dietary reference intake for calcium and vitamin D. high-dose vitamin D3 supplementation in school children. J Clin Endocrinol
Washington, DC: National Academies Press. Available at: http://www.nal. Metab. 2008;93(7):2693-2701. PMID: 18445674
usda.gov/fnic/DRI/DRI_Calcium_Vitamin_D/FullReport.pdf 35. Gallagher JC, Smith LM, Yalamanchili V. Incidence of hypercalciuria and
17. Holick MF, Binkley NC, Bischoff-Ferrari HA, et al; Endocrine Society. hypercalcemia during vitamin D and calcium supplementation in older
Evaluation, treatment, and prevention of vitamin D deficiency: an Endocrine women. Menopause. 2014;21(11):1173-1180. PMID: 24937025
Society clinical practice guideline [published correction appears in J Clin 36. Bouillon R. Safety of high dose vitamin D supplementation. J Clin Endocrinol
Endocrinol Metab. 2011;96(12):3908]. J Clin Endocrinol Metab. 2011;96(7):1911- Metab [Epub ahead of print] PMID: 31858106
1930. PMID: 21646368 37. Billington EO, Burt LA, Rose MS, et al. Safety of high dose vitamin D
18. Lips KD, Cashman C, Lamberg-Allardt C, et al. Current vitamin D status supplementation: secondary analysis of a randomized controlled trial. J Clin
in European and Middle East countries and strategies to prevent vitamin D Endocrinol Metab. 2020;105(4):dgz212. PMID: 31746327
deficiency: a position statement of the European Calcified Tissue Society. Eur 38. Grant WB, Lahore H, McDonnell SL, et al. Evidence that vitamin D
J Endocrinol. 2019;180(4):23-54. PMID: 30721133 supplementation could reduce risk of influenza and COVID-19 infections and
19. Bassatne A, Chakhtoura M, Saad R, Fuleihan GE. Vitamin D supplementation deaths. Nutrients. 2020;12(4):988. PMID: 32252338
in obesity and during weight loss: a review of randomized controlled trials. 39. Tay MZ, Poh CM, Rénia L, MacAry PA, Ng LFP. The trinity of COVID-19:
Metabolism. 2019;92:193-205. PMID: 30615949 immunity, inflammation and intervention. Nat Rev Immunol. 2020;20(6):363-
374. PMID: 32346093

ENDO 2021 • Bone and Mineral Metabolism  117

chased by Dr. Khalid H. Seedahmed, Jr., MRCP PGDip, CertEndocrinology SA - ID 283


40. Martineau AR, Jolliffe DA, Hooper RL, et al. Vitamin D supplementation 43. Chakhtoura M, Napoli N, El Hajj Fuleihan G. Commentary: myths and facts
to prevent acute respiratory tract infections: systematic review and meta- on vitamin D amidst the COVID-19 pandemic. Metabolism. 2020;109:154276.
analysis of individual participant data. BMJ. 2017;356:i6583. PMID: 28202713 PMID: 32470350
41. Bassatne A, Basbous M, Chakhtoura M, Rahme M, El-Hajj Fuleihan G. 44. Tramontana F, Napoli N, El-Hajj Fuleihan G, Strollo R. The D-side of
The link between corona viruses and vitamin D (VIVID): an extensive COVID-19: musculoskeletal benefits of vitamin D and beyond. Endocrine.
and rigorous systematic review and meta-analysis. PROSPERO 2020 2020;69(2):237-240. PMID: 32632722
CRD42020203960. Available at: https://www.crd.york.ac.uk/prospero/ 45. Lanham-New SA, Webb AR, Cashman KD, et al. Vitamin D and SARS-
display_record.php?ID=CRD42020203960 CoV-2 virus/Covid-19 disease. BMJ Nutr Prev Health. 2020;3(1):106-110.
42. National Institute for Health and Care Excellence. COVID-19 rapid guideline: PMID: 33230499
vitamin D. NICE guideline [NG187]. Available at: https://www.nice.org.uk/
guidance/ng187

118  ENDO 2021 • Endocrine Case Management

chased by Dr. Khalid H. Seedahmed, Jr., MRCP PGDip, CertEndocrinology SA - ID 283


Stopping Bone-Active
Medications
Ian R. Reid, MD. Department of Medicine, University of Auckland, New Zealand;
E-mail: i.reid@auckland.ac.nz

Learning Objectives climbs rapidly to substantially exceed the


pretreatment levels, and all the BMD gained
As a result of participating in this session, learners
is lost by 1 year. Up to 15% of patients
should be able to:
stopping long-term denosumab experience
• Describe the different patterns of offset of the vertebral fractures, so transition to another
various classes of osteoporosis drugs. antiresorptive agent should always occur.
Oral or intravenous bisphosphonates appear
• Develop strategies for coping with offset of the
to be adequate in patients who have received
various classes of osteoporosis drugs.
denosumab for only a few years. In patients
who have been treated for more than 5 years,
prevention of rebound bone loss appears
to be more difficult, so more intensive
Main Conclusions bisphosphonate therapy may be required with
Suggested drug discontinuation strategies are as close monitoring of bone resorption markers
follows: and BMD. Patients starting denosumab should
be told of this phenomenon, so they know that
• Oral Bisphosphonate If a drug holiday is
they must not delay injections or discontinue
judged to be indicated, 6 months is usually
them without transition to a bisphosphonate.
appropriate for risedronate and 12 to 24
months for alendronate.
• Zoledronate Zoledronate is usually given
at intervals of 1 to 2 years. Clinicians often Significance of the
increase the dose interval in patients on long-
term treatment to 2 to 4 years. Other clinicians
Clinical Problem
give 3 to 6 doses at annual intervals, then have Osteoporosis is a chronic degenerative condition
a 3-year gap. that usually requires lifelong management.
However, treatment will not necessarily
• Anabolics (teriparatide, abaloparatide,
be continuous since it is now common to
romosozumab) Bone mineral density (BMD)
provide drug holidays (particularly when using
gains during treatment are lost in the months
bisphosphonates) to reduce the risk of atypical
following cessation. Therefore, for each of
femoral fractures. Also, patients sometimes wish
these agents, it is important to lock in the
to discontinue medications for personal reasons
BMD gains with the use of an antiresorptive
or because of adverse effects. Bisphosphonates
agent (bisphosphonate or denosumab) at the
bind tightly to bone mineral and continue to exert
time of anabolic withdrawal.
antiresorptive effects for months to years after
• Denosumab In the months following their discontinuation. The FLEX study showed
cessation of denosumab, bone resorption residual antifracture efficacy for up to 5 years

ENDO 2021 • Bone and Mineral Metabolism  119

chased by Dr. Khalid H. Seedahmed, Jr., MRCP PGDip, CertEndocrinology SA - ID 283


after a 5-year treatment period with alendronate.1 Bisphosphonates
Zoledronate inhibits bone resorption for more
Bisphosphonates have a very high affinity for bone
than 5 years,2 so annual dosing is unnecessary
mineral, to which they bind and remain bound for
in most circumstances. Anabolics are typically
some years. Their inhibition of bone resorption is
used for periods of 1 to 2 years. Therefore, it is
not quite so long-lived, and it varies among agents
necessary to understand that different drugs have
according to their affinity for bone mineral. Thus,
different rates of offset, and some demonstrate a
zoledronate has the longest duration of action,
rebound phenomenon whereby there is excessive
whereas the effects of risedronate and ibandronate
bone loss that can result in multiple vertebral
are more transient. Duration of effect is also
fractures. This has been most clearly demonstrated
dosage dependent.
with denosumab.3 Up to 15% of patients stopping
Several studies of rates of offset of antiresorptive
long-term denosumab experience vertebral
activity have been published. McClung et al
fractures, and multiple fractures are seen in 10%.4
demonstrated that the effects of alendronate on both
This is a potentially catastrophic outcome that
bone turnover and BMD lasted considerably longer
must be anticipated by patient counseling at
than those of estrogen.5 The FLEX study assessed
the time of treatment initiation and vigorously
multiple endpoints in women already treated
prevented by transition to bisphosphonates if
before the trial with alendronate for 5 years, who
denosumab is stopped.
were then randomly assigned to continue on
full-dosage alendronate, half-dosage alendronate
Barriers to Optimal Practice (equivalent to 70 mg every 2 weeks), or placebo.1
Nonvertebral fractures were not different in those
• Failure to anticipate drug offset when randomly assigned to either dosage of alendronate
initiating an osteoporosis treatment and thus or placebo, although in women entering FLEX
to have an a priori plan. with femoral neck T-scores less than –2.5, there
• Failure to discuss drug offset with patients, so were fewer fractures in those on alendronate. For
they are aware of the dangers of discontinuing the FLEX cohort as a whole, clinical vertebral
medication. fractures were reduced by about one-half in those
who continued to take alendronate. This study
has been very influential, creating the T-score
Strategies for Diagnosis, threshold of –2.5 for determining whether
ongoing therapy is indicated. Its other important
Therapy, and/or Management message, often overlooked, is that after 5 years
In most areas of medicine, physicians and their treatment, the dosage of alendronate can be
patients assume that medications only act for as reduced by one-half without loss of efficacy. It
long as they are taken. For bone, this has been should be remembered that there was continuing
less clear. Certainly, bone turnover markers prevention of clinical vertebral fractures in those
show rapid responses to starting and stopping who stayed on treatment for 10 years.
some interventions, but BMD always responds Studies of risedronate demonstrate a more
slowly. Also, we have grown accustomed to using rapid offset of bone turnover suppression, within
bisphosphonates, the principal bone medications, 12 months of drug discontinuation. Within
and have accepted their very long duration of this period, femoral neck BMD also returned
action without regarding it as being unique to this to baseline, in contrast to either alendronate or
group of medications. Now that the pharmacology zoledronate.6 Despite the transience of effect, in
of bone is becoming more diverse, it is important the first year after risedronate discontinuation
to recognize differences in rates of offset of the new vertebral fractures were reduced by about
various medications now available. one-half.

120  ENDO 2021 • Endocrine Case Management

chased by Dr. Khalid H. Seedahmed, Jr., MRCP PGDip, CertEndocrinology SA - ID 283


Zoledronate has the highest affinity for fractures were any more common than in those
bone mineral, and it is also the most potent participants discontinuing placebo. However,
bisphosphonate in terms of its inhibition of in the FREEDOM extension study, where
the target enzyme in the mevalonate pathway. denosumab use was up to 10 years in duration,
When given in the conventional intravenous 15% of women discontinuing the drug developed
dose of 5 mg, its inhibition of bone turnover vertebral fractures in the following months, and
lasts for more than 5 years after a single dose, in 10% of discontinuing participants, there were
and hip BMD is still 2% above baseline at that multiple vertebral fractures. This is a major cause
time point.2 Extensions to the pivotal phase 3 for concern, since multiple vertebral fractures are
trial demonstrated residual antifracture efficacy likely to lead to height loss, spinal deformity, and
after the drug had been discontinued, but optimal long-term morbidity. Also, observational studies
fracture prevention was seen in those who suggest that more than half of patients treated
continued treatment annually for 6 years. A post for osteoporosis discontinue their medications
hoc analysis of the phase 3 trials found that among within 2 years of initiation. Thus, it is likely that
women who received only 1 injection of trial many patients treated with denosumab receive
medication, fracture prevention over 3 years was no benefit from this intervention if they stop it
similar to that observed in those receiving 3 doses without transitioning to some other antiresorptive
of either zoledronate or placebo. This provides agent. Indeed, if they sustain multiple vertebral
some evidence that the long duration of the effects fractures, they are likely to be significantly worse
of this drug on bone turnover and BMD might off than if they had never started the medication.
also be reflected in longevity of antifracture effects. This problem has led to a number of
The recent trial of 18-month dosing of zoledronate observational studies assessing the effectiveness
also showed substantial fracture prevention and of transitioning from denosumab to other
provides further evidence for this.7 antiresorptive agents. After 1 year of denosumab
treatment, transitioning to oral alendronate
Denosumab appears to provide good prevention of bone
boss.8 Similarly, some studies have demonstrated
The issue of offset of action of osteoporosis satisfactory outcomes from giving zoledronate at
medications has come to the forefront with the the time of denosumab discontinuation,9 but after
increasing use of denosumab, a monoclonal long-term denosumab treatment, zoledronate is
antibody directed against RANKL. It suppresses only partially effective in preventing subsequent
bone turnover by about 90% for a period of bone loss.10 When denosumab treatment has
6 months after each subcutaneous injection. continued for more than 5 years, it appears that
The phase 2 trial program demonstrated that more intensive therapy is necessary to maximize
offset after the first missed dose is dramatic, BMD preservation. Following bone resorption
with bone resorption rising to about 50% above markers such as serum CTX may be an aid in
pretreatment levels within months of drug optimizing post-denosumab bone preservation.
discontinuation.3 Most of the bone gained during
treatment is lost in the first 6 months after drug
discontinuation and BMD is back to baseline at Anabolics
12 months. In 2016, case reports and case series Bone loss resumes following discontinuation
describing multiple vertebral fractures following of teriparatide. One year after conclusion of a
denosumab discontinuation began to appear and 1-year treatment course, spine BMD is still several
now represent a substantial literature. Analysis percent above baseline, but hip BMD is only about
of patients discontinuing from the 3-year 0.5% above baseline. Therefore, the convention is
FREEDOM study did not suggest that vertebral to transition to a bisphosphonate, which achieves

ENDO 2021 • Bone and Mineral Metabolism  121

chased by Dr. Khalid H. Seedahmed, Jr., MRCP PGDip, CertEndocrinology SA - ID 283


continued positive trajectories in bone density. Either continuing current therapy (Answer A)
Similarly, transition to denosumab results in or transitioning to a bisphosphonate (Answer D)
significant further gains in bone density.11 would be acceptable answers. Cessation of therapy,
Romosozumab is a monoclonal antibody whether temporary or permanent (Answers B
directed against the osteocyte protein, sclerostin. and C), would expose her to risk of rebound
This has both anabolic and antiresorptive effects vertebral fractures. Whether she needs to remain
and results in substantial increases in BMD. Its on osteoporosis treatment cannot be determined
discontinuation is followed by substantial BMD from the history provided. Her fracture risk
loss over the first year off treatment, and bone could be high, despite her T-score, if she were
resorption markers show a rebound similar to elderly or had an extensive history of fractures.
that following denosumab.12 However, if patients However, if she were fracture-free and in her 50s,
transition to denosumab or to alendronate, transitioning her regimen to a bisphosphonate and
positive changes in BMD are observed and then weaning therapy would seem the appropriate
fracture rates remain low.13,14 course.
Strategies for treatment discontinuation are
described in the Main Conclusions section. In Case 2
addition to those specific measures, it is critically
important to counsel patients about the offset A patient has been taking alendronate for 5 years,
characteristics of osteoporosis drugs when they and she has been fracture-free during that time.
are first being prescribed, so that this can be Her current hip T-score is –2.2.
incorporated into the decision to embark upon
a particular therapeutic strategy. Just as patients Which of the following is the
starting β-adrenergic blockers for ischemic best recommendation?
heart disease are warned not to discontinue A. Continuation of current therapy
their medications without medical consultation, B. A 1-year drug holiday
the same warning must be provided to those C. A 5-year drug holiday
starting nonbisphosphonate osteoporosis drugs, D. Permanent cessation of treatment
particularly denosumab.
E. Transition to an intravenous bisphosphonate

Clinical Case Vignettes Answer: B or C) A 1-year drug holiday


or a 5-year drug holiday
Case 1
The preferred responses are a 1-year drug holiday
A patient has been receiving denosumab injections
(Answer B) or a 5-year drug holiday (Answer C).
every 6 months for 5 years, and she has been
Continuing therapy in the next year (Answer A)
fracture-free during that time. Her current hip
is probably not necessary in light of her current
T-score is –2.2.
BMD and recent fracture history. The FLEX
Which of the following is the study does indicate, however, that after 5 years
best recommendation? of treatment, the alendronate dosage can be
halved (eg, to 70 mg every 2 weeks) without
A. Continuation of current therapy
loss of efficacy. A 1-year drug holiday would
B. A 1-year drug holiday be sensible to reduce the small risk of atypical
C. Permanent cessation of treatment femoral fractures. The FLEX study indicates that
D. Transition to a bisphosphonate a 5-year drug holiday will not increase her risk
of nonvertebral fractures, but there may be a rise
Answer: A or D) Continuation of current in the incidence of clinical vertebral fractures.
therapy or transition to a bisphosphonate

122  ENDO 2021 • Endocrine Case Management

chased by Dr. Khalid H. Seedahmed, Jr., MRCP PGDip, CertEndocrinology SA - ID 283


If there was a reasonable indication for the Case 4  New Content 
initiation of alendronate in the first place, then
A patient has been receiving annual infusions of
permanent cessation of treatment (Answer D) is
zoledronate for 3 years, and she has been fracture-
likely to be inappropriate. She could transition
free during that time. Her current hip T-score is –2.2.
to an intravenous bisphosphonate (Answer E) if
she finds that more convenient, but there is no Which of the following is the
other specific indication to do so, since her clinical best recommendation?
course receiving oral alendronate appears to have
A. Continuation of annual zoledronate infusions
been satisfactory.
B. A 3-year drug holiday
C. Permanent cessation of treatment
Case 3  New Content 
D. Transition to an oral bisphosphonate or
A patient has been receiving daily teriparatide
denosumab
injections for 2 years, and she has been fracture-
free during that time. Her current hip T-score Answer: B or D) A 3-year drug holiday or transition
is –2.2. to an oral bisphosphonate or denosumab

Which of the following is the Either a 3-year drug holiday (Answer B) or


best recommendation? transition to an oral bisphosphonate or denosumab
A. Continuation of current therapy (Answer D) could be appropriate responses,
depending on the clinical situation. Zoledronate
B. A 1-year drug holiday can certainly be used beyond 3 annual infusions
C. Permanent cessation of treatment (Answer A), and in the extension to the phase
D. Transition to a bisphosphonate or denosumab 3 trial, patients received 9 annual infusions
without evidence of problems. However, it was
Answer: D) Transition to a found that after 3 annual infusions, those who
bisphosphonate or denosumab had not had incident fractures and no longer had
Results of studies in rats demonstrate that high- osteoporotic BMD maintained low fracture rates
dosage, long-term use of teriparatide increases in a subsequent 3-year period off treatment.
the risk of osteosarcoma. As a result, the use of If treatment with zoledronate was justified in
teriparatide is restricted to 18 to 24 months across the first place, it is unlikely that this patient should
various jurisdictions. Therefore, this patient permanently stop treatment (Answer C), since the
should not continue teriparatide (Answer A) antiresorptive effect of zoledronate will wear off in
any longer, even though a decade of clinical due course and her fracture risk will then rise.
surveillance has not produced evidence that this She could transition to either an oral
is a problem in humans. Bone loss commences bisphosphonate or denosumab if the zoledronate were
once teriparatide is stopped, so neither a drug no longer acceptable to her, although this would be
holiday (Answer B) nor permanent cessation of unusual. Some patients do have a marked flulike illness
treatment (Answer C) is appropriate. Therefore, after their first injection and choose not to continue,
the convention is to transition to a bisphosphonate but this does not develop de novo subsequently.
or denosumab (Answer D), which results in Zoledronate exerts antiresorptive effects for
continued positive trajectories in BMD. more than 5 years after infusion. Therefore, in this
case, it would be reasonable to recommend further
infusions of zoledronate at intervals of 2 to 3 years,
depending on bone turnover markers and the
evolution of BMD.

ENDO 2021 • Bone and Mineral Metabolism  123

chased by Dr. Khalid H. Seedahmed, Jr., MRCP PGDip, CertEndocrinology SA - ID 283


References
1. Black DM, Schwartz AV, Ensrud KE, et al; FLEX Research Group. Effects of 8. Freemantle N, Satram-Hoang S, Tang ET, et al; DAPS Investigators. Final
continuing or stopping alendronate after 5 years of treatment: the Fracture results of the DAPS (Denosumab Adherence Preference Satisfaction)
Intervention Trial Long-Term Extension (FLEX): a randomized trial. JAMA. study: a 24-month, randomized, crossover comparison with alendronate in
2006;296(24):2927-2938. PMID: 17190893 postmenopausal women. Osteoporos Int. 2012;23(1):317-326. PMID: 21927922
2. Grey A, Bolland MJ, Horne A, Mihov B, Gamble G, Reid IR. Duration 9. Horne AM, Mihov B, Reid IR. Effect of zoledronate on bone loss
of antiresorptive activity of zoledronate in postmenopausal women after romosozumab/denosumab: 2-year follow-up. Calcif Tissue Int.
with osteopenia: a randomized, controlled multidose trial. CMAJ. 2019;105(1):107-108. PMID: 31087124
2017;189(36):E1130-E1136. PMID: 28893975 10. Reid IR, Horne AM, Mihov B, Gamble GD. Bone loss after denosumab: only
3. Bone HG, Bolognese MA, Yuen CK, et al. Effects of denosumab treatment partial protection with zoledronate. Calcif Tissue Int. 2017;101(4):371-374.
and discontinuation on bone mineral density and bone turnover markers PMID: 28500448
in postmenopausal women with low bone mass. J Clin Endocrinol Metab. 11. Leder BZ, Tsai JN, Uihlein AV, et al. Denosumab and teriparatide
2011;96(4):972-980. PMID: 21289258 transitions in postmenopausal osteoporosis (the DATA-Switch study):
4. Cummings SR, Ferrari S, Eastell R, et al. Vertebral fractures after extension of a randomised controlled trial. Lancet. 2015;386(999):1147-1155.
discontinuation of denosumab: a post hoc analysis of the randomized PMID: 26144908
placebo-controlled FREEDOM trial and its extension. J Bone Miner Res. 12. McClung MR, Brown JP, Diez-Perez A, et al. Effects of 24 months of
2018;33(2):190-198. PMID: 29105841 treatment with romosozumab followed by 12 months of denosumab or
5. McClung MR, Wasnich RD, Hosking DJ, et al; Early Postmenopausal placebo in postmenopausal women with low bone mineral density: a
Intervention Cohort Study. Prevention of postmenopausal bone loss: six-year randomized, double-blind, phase 2, parallel group study. J Bone Miner Res.
results from the Early postmenopausal Intervention Cohort Study. J Clin 2018;33(8):1397-1406. PMID: 29694685
Endocrinol Metab. 2004;89(10):4879-4885. PMID: 15472179 13. Cosman F, Crittenden DB, Adachi JD, et al. Romosozumab treatment in
6. Kim TY, Bauer DC, McNabb BL, et al. Comparison of BMD changes and postmenopausal women with osteoporosis. N Engl J Med. 2016;375(16):1532-
bone formation marker levels 3 years after bisphosphonate discontinuation: 1543. PMID: 27641143
FLEX and HORIZON-PFT Extension I Trials. J Bone Miner Res. 14. Saag KG, Petersen J, Brandi ML, et al. Romosozumab or alendronate
2019;34(5):810-816. PMID: 30536713 for fracture prevention in women with osteoporosis. N Engl J Med.
7. Reid IR, Horne AM, Mihov B, et al. Fracture prevention with zoledronate 2017;377(15):1417-1427. PMID: 28892457
in older women with osteopenia. N Engl J Med. 2018;379(25):2407-2416.
PMID: 30575489

124  ENDO 2021 • Endocrine Case Management

chased by Dr. Khalid H. Seedahmed, Jr., MRCP PGDip, CertEndocrinology SA - ID 283


Modern Management of the
Patient With Paget Disease
Thomas J. Weber, MD. Division of Endocrinology, Metabolism and Nutrition, Duke
University Medical Center, Durham, NC; E-mail: thomas.weber2@dm.duke.edu

Learning Objectives to treatment than those with a longer duration


of symptoms. On the basis of available evidence,
As a result of participating in this session, learners
it is important to note that more advanced
should be able to:
complications of PDB, such as fractures, skeletal
• Explain the best current approach to the deformities, need for orthopedic interventions, or
biochemical and radiographic diagnosis of hearing loss, may or may not benefit from specific
Paget disease of bone (PDB). pharmacologic or surgical interventions.
• Explain the best therapeutic approach for
painful PDB.
• Describe current controversies regarding Significance of the
the management of asymptomatic PDB and Clinical Problem
specific disease-related complications. PDB is a common metabolic bone disorder that
affects approximately 2% to 3% of individuals
older than 55 years in the United States, although
the incidence of the disease may be declining
Main Conclusions for unclear reasons. Most patients are identified
PDB is the second most common metabolic incidentally because of an elevated alkaline
bone disease after osteoporosis. This condition phosphatase measurement or characteristic
is typically identified through routine laboratory findings on radiology studies obtained for other
testing, based on elevated serum alkaline indications, but a substantial number of affected
phosphatase, or identified incidentally on patients present with bone pain. Additionally,
radiographs that are obtained for another patients may develop skeletal deformity, fractures,
indication. PDB most often involves one site or neurologic sequelae related to encroaching
(mono-ostotic) or is less commonly multicentric effects of pagetic lesions, such as cranial nerve
(polyostotic), with the most common sites of palsies or hearing loss. Limited plain radiographic
involvement being the pelvis, femur, spine, studies (anteroposterior abdomen and pelvis,
skull, and tibia. Most patients with PDB are bilateral tibia, and skull) are recommended
asymptomatic; the most common symptom is for initial radiologic confirmation in patients
bone pain, which is generally contingent on site with asymptomatic PDB who have elevated
of involvement and effect on local structures. alkaline phosphatase, although whole-body bone
Painful PDB is responsive to treatment with scintigraphy is recommended for confirmation of
bisphosphonates, with a greater and more diagnosis and/or characterization of the extent of
sustained effect conferred by intravenous than oral disease involvement. While bisphosphonates are
therapy. In addition, patients with recent onset of proven to reduce bone pain in patients with PDB,
symptoms due to PDB are more likely to respond treatment appears to be more effective in patients

ENDO 2021 • Bone and Mineral Metabolism  125

chased by Dr. Khalid H. Seedahmed, Jr., MRCP PGDip, CertEndocrinology SA - ID 283


with a more proximate onset of symptoms. roughly doubles in incidence for each decade after
Importantly, there is insufficient evidence of age 50 years.1 Interestingly, the disease prevalence
other disease benefit (ie, prevention of deformity, appears to be declining, as does the degree and
fractures, orthopedic surgery, hearing loss) severity of the disease,3,4 although the reasons for
with bisphosphonates or other pharmacologic/ this are unclear.
nonpharmacologic interventions, such that PDB may be sporadic or familial, with
expectant management and treatment of patients evidence that up to 30% of affected patients have a
with PDB who have bone pain is currently positive family history of the disorder. Within the
advised. United States, studies suggest that a first-degree
relative of a patient with PDB is 7 times more
likely to develop the disease than someone with an
Barriers to Optimal Practice unaffected relative.1 The most frequent pathogenic
variant linked to PDB is in the sequestosome
• PDB may be underrecognized in patients with
1 gene (SQSTM1), which encodes a ubiquitin-
an alkaline phosphatase level within but at
binding protein important for cellular signaling
the upper limit of the normal range, in which
events in osteoclast activation. Patients with
case measurement of bone-specific alkaline
the SQSTM1 pathogenic variant appear to have
phosphatase and/or other bone biomarkers
more severe disease at an earlier age of onset.5 In
(P1NP, C-telopeptide) may be helpful.
the New England cohort of patients with PDB,
• Biochemical relapse (ie, increase in alkaline pathogenic variants in SQSTM1 accounted for 21%
phosphatase above the normal range) does of cases, resulting in early-onset disease and spinal
not necessarily predict clinical relapse (ie, stenosis in exclusively male patients.6 There is
recurrence of bone pain), thereby presenting also evidence of an environmental contribution to
a challenge in the long-term management of PDB, based on earlier animal and human studies
patients with PDB. suggesting that paramyxovirus infections such as
• Although somewhat intuitive, bisphosphonate measles may precipitate osteoclast activation and
treatment of patients with PDB has not been transformation, although studies to date have not
proven to reduce the risk for orthopedic and been conclusive.
neurologic outcomes. The initial lesion in PDB, which may
be solitary (mono-ostotic) or multicentric
(polyostotic), is lytic in nature and due to increased
Strategies for Diagnosis, number and activity of osteoclasts. Subsequently,
osteoblasts are recruited to the skeletal site,
Therapy, and/or Management
resulting in accelerated bone formation that results
Prevalence and Pathogenesis in woven or immature bone that is incompletely
PDB is the second most prevalent metabolic bone remodeled and subsequently infiltrated by
disease after osteoporosis, affecting approximately excessive fibrous connective tissue and blood
1% of adults older than 50 years in the United vessel deposition. The result of this process is an
Kingdom and approximately 2% to 3% of adults enlarged, hypervascular and structurally inferior
older than 55 years in the United States.1,2 The skeletal site. Over time, hypercellularity typically
incidence of the disease varies significantly, diminishes, resulting in slower skeletal turnover
however, by geography and ethnicity/race, with and “burned out” sclerotic pagetic lesions.
a much greater prevalence in persons of Anglo-
Saxon or European heritage than in persons of
Asian or African ancestry. The disease is more
common in males than in females (~1.4:1), and it

126  ENDO 2021 • Endocrine Case Management

chased by Dr. Khalid H. Seedahmed, Jr., MRCP PGDip, CertEndocrinology SA - ID 283


Clinical Presentation In some patients, it may be useful to assess markers
of bone resorption, including C- and N-terminal
Most patients with PDB are asymptomatic and
telopeptides of collagen (CTX and NTX), as well
are identified through an incidentally identified
as markers of bone formation, namely procollagen
elevated alkaline phosphatase level on routine
type 1 N-terminal propeptide, to confirm elevated
chemistry testing or by radiologic studies
skeletal turnover due to PDB and provide baseline
performed for other indications. Patients with
data to evaluate therapeutic response.10
symptomatic PDB generally have localized bone
As noted above, PDB is often suspected by
pain at the site of pagetic involvement that may
radiologic studies (plain x-ray or CT) performed
occur with movement or at rest, although the
for other indications. If initially suspected because
mechanism of pain is poorly understood. Bone
of biochemical findings, a plain radiograph of
deformities, particularly when involving the lower
a suspected skeletal site based on clinical signs
limbs, can exacerbate mechanical stress on joints
or symptoms is reasonable. In patients without
and accelerate the development and progression of
localizing complaints, a limited x-ray survey
osteoarthritis. Less commonly, bony enlargement
of the skull and facial bones, abdomen (which
from PDB may cause symptomatic compression
includes the pelvis and proximal femurs), and
of adjacent structures resulting in cranial nerve
tibia is recommended given the high prevalence
palsies and hearing loss, among other symptoms.
of involvement of these sites. In the absence of
There is also evidence that hearing loss can be
plain radiographic involvement, whole-body bone
significant and subclinical, although screening
scintigraphy is recommended based on its superior
with a specific tool may be helpful in identifying
sensitivity for detecting PDB compared with the
affected patients.7,8 Rarely, hydrocephalus or spinal
sensitivity of plain x-rays, although whole-body
stenosis may result from basilar invagination of
bone scintigraphy may be negative at sites of
the skull and vertebral involvement, respectively.
inactive PDB. Whole-body bone scintigraphy
Fractures may also occur through pagetic bone,
is recommended as well, in addition to targeted
particularly in the long bones at sites of lytic
radiographs, as a means to fully and accurately
involvement. Hypervascularity of affected bone
define the extent of metabolically active disease in
may exacerbate bleeding risk with orthopedic
patients with PDB.11
procedures that involve pagetic sites and has
been associated, rarely, with high-output cardiac
failure. Very rarely, malignant transformation Therapeutic Management
to osteosarcoma (<1%) or giant-cell tumor Based on the accelerated bone turnover
development occurs. Despite the risk, albeit that underlies the pathobiology of PDB,
rare, for osteosarcoma, there is no evidence for bisphosphonates would be an expected appropriate
premature mortality in patients with PDB.9 therapeutic agent. However, although PDB may
result in several clinically significant adverse
Diagnosis outcomes, existing evidence for bisphosphonate
use is robust only for reduction in skeletal pain. A
An elevated total alkaline phosphatase
Cochrane meta-analysis of existing randomized,
concentration is a hallmark of PDB, although it
placebo-controlled trials confirmed that
may not be consistent with the diagnosis in some
bisphosphonate treatment is more than 3 times
patients. Specifically, total alkaline phosphatase
more likely than placebo to eliminate bone pain
reflects both skeletal and hepatic activity, such
in PDB (31% vs 9% [relative risk (RR) = 3.42; 95%
that a low hepatic fraction may result in a normal
confidence interval (CI), 1.31-8.90]) and twice as
total alkaline phosphatase value. Additionally, the
likely to reduce pain compared with placebo (RR,
degree of skeletal involvement and activity may
1.97; 95% CI, 1.29-3.01), with a number needed to
not be sufficient to raise total alkaline phosphatase.

ENDO 2021 • Bone and Mineral Metabolism  127

chased by Dr. Khalid H. Seedahmed, Jr., MRCP PGDip, CertEndocrinology SA - ID 283


treat of only 5 patients to show significant benefit such that these findings may not necessarily apply
(95% CI, 2-15).12 There is also evidence that to younger patients who are identified earlier in
patients with a shorter disease duration are more the disease course.
likely to respond to treatment.13 There is insufficient evidence that
Regarding specific bisphosphonates, there bisphosphonate treatment prevents hearing loss,
is randomized controlled trial evidence that bone deformity, or progression of associated
zoledronic acid, 5 mg intravenously, is superior neurologic symptoms or osteoarthritis.
to risedronate, 30 mg orally for 2 months, in Nonetheless, patients with skull/spinal
relieving bone pain (RR, 1.36; 95% CI, 1.06-1.74).14 involvement or those undergoing major
Patients treated with zoledronic acid are also orthopedic surgery that is proximate to areas
less likely than those treated with risedronate to of pagetic involvement have historically been
experience clinical relapse (recurrence of bone considered candidates for treatment based on
pain) (9.2% vs 25.2%), although lack of biochemical concerns about neurologic or hemorrhagic
relapse (increase in alkaline phosphatase level) complications, respectively, the lack of definitive
was not helpful in defining which patients were evidence for benefit notwithstanding. As noted
at risk of clinical relapse when treated with above, patients with PDB are at risk for orthopedic
zoledronic acid. There are limited comparative complications of their disease, including fractures
data on other oral and parenteral bisphosphonates and requirement for joint replacement, although
in PDB. Low level evidence indicates that there is insufficient evidence to support a benefit
calcitonin may improve bone pain in patients of intensive pharmacologic therapy on these
with PDB, although it may be considered when outcomes, as well as inadequate evidence to
bisphosphonate use is contraindicated. Finally, support a specific type of surgical intervention.
denosumab is not recommended for PDB because Nonetheless, hip or knee arthroplasty is
of insufficient evidence. recommended in patients in whom medical
The apparent distinct nature between therapy is inadequate. Although only a conditional
biochemical and clinical relapse in patients recommendation, osteotomy may also be
treated with zoledronic acid vs risedronate does considered for correction of bone deformity.
call into question a “treat to target” approach
of normalizing alkaline phosphatase in PDB.
Consequently, the PRISM and PRISM-EZ trials
Clinical Case Vignettes
sought to determine whether symptomatic or Case 1
intensive treatment with bisphosphonates was A 64-year-old man is referred for evaluation after
more effective in the short- and long-term a recent abnormal abdominal and pelvic CT scan,
management of PDB. The PRISM trial revealed which he recently underwent for evaluation of left
that intensive bisphosphonate treatment did not lower abdominal pain of 2 months’ duration. He
reduce bone pain, fractures, or need for orthopedic has no obvious abdominal or pelvic findings to
surgery compared with symptom-driven explain his pain, although there is fairly extensive
treatment in patients with PDB over an average of cortical and trabecular thickening involving
3 years.15 Furthermore, long-term normalization the right iliac and right superior and inferior
of alkaline phosphatase over more than 7 years pubic rami. The patient has no pain, stiffness, or
did not improve bone pain, improve quality of numbness in these areas. His medical history is
life, or reduce the need for orthopedic procedures, noncontributory, and family history is negative for
and it may actually be associated with a higher “bone disease.”
fracture risk.16 It is important to note that these On physical examination, he has no tenderness
patients were older (mean age 76 years) and had to palpation over the right lateral or posterior
longstanding disease (~12 years since diagnosis), hip. Right hip passive range of motion is normal

128  ENDO 2021 • Endocrine Case Management

chased by Dr. Khalid H. Seedahmed, Jr., MRCP PGDip, CertEndocrinology SA - ID 283


without elicited pain. Weber and Rinne tests of Case 2
hearing are normal.
A 52-year-old man presents with a 2-year history
Laboratory test results: of gradually worsening left posterior-lateral
hip pain. He describes gradual onset of the pain
Alkaline phosphatase = 114 U/L (24-110 U/L)
without antecedent trauma or injury. He describes
(SI: 1.90 μkat/L [0.40-1.84 μkat/L])
AST, normal the pain as deep and dull and exacerbated by
AST, normal weight bearing. He has also developed left groin
Total bilirubin, normal pain within the past 6 months that is worse with
25-Hydroxyvitamin D, normal movement. He has known degenerative disc and
On the basis of this patient’s clinical spine disease involving the lumbar spine, although
presentation, which of the following is his pain has not responded to oral nonsteroidal
the best next step in his management? antiinflammatory agents, steroid dose-packs,
gabapentin, or epidural steroid injections. His
A. Zoledronic acid, 5 mg intravenously
medical history is notable for osteoarthritis of both
B. Alendronate, 40 mg daily for 6 months knees and shoulders. He does not smoke cigarettes
C. Technetium 99Tc whole-body bone or drink alcohol. Family history is negative for
scintigraphy osteoporosis or other bone disorders.
D. Fasting serum C-telopeptide measurement On physical examination, there is mild
E. Reassurance, as no additional workup or tenderness to palpation of the left posterior hip/
treatment is indicated pelvic region. Passive range of motion of the left
hip is also restricted due to elicited pain. The
Answer: C) Technetium 99Tc whole- oropharynx is normal to inspection.
body bone scintigraphy
Laboratory test results:
This patient has biochemical and radiographic
Calcium, normal
evidence for PDB, although he is currently Creatinine, normal
asymptomatic. On the basis of this information 25-Hydroxyvitamin D, normal
and current evidence, there is not an indication Alkaline phosphatase = 114 U/L (24-110 U/L)
to treat him with pharmacologic therapy, (SI: 1.90 μkat/L [0.40-1.84 μkat/L])
including bisphosphonates. However, he may have Bone-specific alkaline phosphatase = 33 μg/L
(0-20 μg/L [males])
multifocal disease that includes the spine, which
could position him as a candidate for treatment Plain radiograph of the pelvis shows thickening
because of concerns regarding disease progression and sclerosis of the left iliac wing cortex extending
and possible neurologic complications. Given inferiorly to the medial aspect of the acetabular
this, whole-body bone scanning with technetium wall and inferior pubic ramus. Subsequent
99
Tc (Answer C) is indicated. Treatment with an technetium 99Tc whole-body bone scan shows
oral or intravenous bisphosphonate (Answers A diffusely increased tracer activity involving most
and B) is not appropriate without the results of of the left hemipelvis, including the acetabulum,
the bone scan. Measurement of bone turnover inferior pubic ramus, superior pubic ramus,
with C-telopeptide (Answer D) is not needed to ischium, and iliac wing, which correlates with the
support the diagnosis of PDB in this patient, and it areas of cortical thickening and bony sclerosis on
also has limited utility in predicting or confirming the pelvic radiograph.
response to pharmacologic therapy. Finally,
reassurance (Answer E) should not be given to this
patient until the extent of disease is established.

ENDO 2021 • Bone and Mineral Metabolism  129

chased by Dr. Khalid H. Seedahmed, Jr., MRCP PGDip, CertEndocrinology SA - ID 283


On the basis of this patient’s clinical In the course of her workup, she exhibits a
presentation, which of the following is mild elevation of alkaline phosphatase of 133 U/L
the best next step in his management? (25-125 U/L) (SI: 2.22 μkat/L [0.42-2.09 μkat/L]),
A. Zoledronic acid, 5 mg intravenously with a repeated measurement (different assay)
B. Risedronate, 30 mg daily for 2 months of 102 U/L (24-110 U/L) (SI: 1.70 μkat/L [0.40-
1.84 μkat/L]). Bone-specific alkaline phosphatase
C. Calcitonin, 100 units subcutaneously 3 times
is measured and found to be elevated at 29 μg/L
weekly
(<22 μg/L [postmenopausal women]). Plain
D. Fasting serum C-telopeptide measurement radiographs of the abdomen, pelvis, and bilateral
E. Denosumab, 60 mg subcutaneously every tibia are negative, but subsequent technetium 99Tc
6 months whole-body bone scan shows mild, diffuse uptake
throughout the calvarium. Plain radiograph of
Answer: A) Zoledronic acid, 5 mg intravenously
the skull shows mild thickening of the calvarium.
This patient has unequivocal evidence for active Upon relaying this result to the patient, she states
and symptomatic PDB based on biochemical that she has no headaches or subjective hearing
findings, plain radiographs, and nuclear imaging. loss, reporting that her hearing is much better
Given the available evidence and the relatively than her husband’s.
recent onset of symptoms, he is an appropriate
candidate for pharmacologic therapy. Zoledronic On the basis of this patient’s clinical
acid intravenously (Answer A) is superior to presentation, which of the following is
oral risedronate (Answer B) based on a head-to- the best next step in her management?
head comparison and is the best treatment for A. Zoledronic acid, 5 mg intravenously
this patient. Calcitonin (Answer C) has limited B. Alendronate, 40 mg daily for 6 months
benefit based on available evidence. Denosumab C. Audiometry testing
(Answer E) may have benefit in giant-cell tumors
D. Fasting serum procollagen type 1 N-terminal
in the context of PDB because of its potent
propeptide measurement
antiresorptive effect, but it is not approved
specifically for either indication. In addition, the E. CT of the skull
potential consequences of the robust off effect of Answer: C) Audiometry testing
the drug, as is seen in patients with osteoporosis,
is unknown in patients with PDB. As in the first This patient has evidence for PDB with skull
case, measurement of bone turnover (Answer D) involvement that is asymptomatic. Despite
has limited utility in either the diagnosis or this, and not necessarily contingent on skull
therapeutic management of this patient. involvement, the incidence of hearing impairment
in patients with PDB is significantly higher than
in the general population.5 Given this, audiometry
Case 3
testing (Answer C) is indicated to determine
A 60-year-old woman is referred for evaluation whether hearing loss is present and whether she
of osteoporosis. She has a history of risedronate would potentially benefit from hearing aids or
use (2 years), but she stopped it secondary to other assistive technology. There is evidence for a
unacceptable gastroesophageal reflux symptoms. She benefit from screening for hearing loss in patients
is presently on raloxifene, 60 mg daily, which she has with PDB (Hearing Handicap Inventory for the
taken for 4 years, and she has evidence of stable bone Elderly-Screening), which also has established
mineral density on therapy, with estimated 10-year utility in elderly patients (without PDB).6 There
risks of hip and major osteoporotic fractures of is no evidence to date that treatment with
1.0% and 12%, respectively. bisphosphonates (Answers A and B) slows or

130  ENDO 2021 • Endocrine Case Management

chased by Dr. Khalid H. Seedahmed, Jr., MRCP PGDip, CertEndocrinology SA - ID 283


reverses hearing loss in patients with PDB. Finally, Which of the following is the best next
additional diagnostic studies of bone turnover step in this patient’s management?
(Answer D) and imaging (Answer E) are unlikely to A. Zoledronic acid, 5 mg intravenously
provide additional clinical guidance for this patient. B. Risedronate, 30 mg daily for 2 months
C. Calcitonin, 100 units subcutaneously 3 times
Case 4  New Content  weekly
A 75-year-old man is referred for evaluation D. MRI of the pelvis
of left posterior hip pain and elevated alkaline E. Technetium 99Tc whole-body bone
phosphatase. He was in his usual state of health scintigraphy
until 2 months ago, when he noted subacute onset
of pain in the left buttocks. He had no antecedent Answer: D) MRI of the pelvis
injury and has no history of significant back,
This patient presents with biochemical and
pelvis, or hip pain. His symptoms have progressed
radiographic evidence of PDB. However, the
over the past several weeks, such that he has
acuity of his presentation and degree of pain,
difficulty ambulating without incurring severe
which is somewhat disproportionate to the
pain in his left hip.
available clinical findings, combined with recent
His medical history is notable for hypertension
unintentional weight loss, raise a concern for
and hyperlipidemia, both of which are well
malignancy. The best next step in this patient’s
controlled with medical therapy. His medications
management is MRI of the pelvis (Answer D).
include lisinopril and atorvastatin. Family history
Indeed, he had MRI evidence of focal cortical
is negative for known bone disease. He has
destruction and a contiguous soft-tissue mass
somewhat poor appetite, but review of systems is
consistent with osteosarcoma. MRI is preferred
otherwise noncontributory.
to CT given superiority in defining the presence
On physical examination, he is normotensive.
of soft-tissue and/or neurovascular involvement.
His weight is 150 lb (68.2 kg), which is
Characteristic radiologic findings include
approximately 8 lb (3.6 kg) lighter than his weight
destruction of the normal trabecular bone pattern,
1 year ago. He has tenderness to palpation over
loss of cortical margins, and no endosteal bone
the left midsacral area, lateral to the midline. No
response. The involved bone is notable for a
erythema or warmth is noted over the affected
mixture of radiodense and radiolucent areas,
area. Findings on spinal examination are normal
cortical destruction, and incomplete periosteal
without palpable tenderness. The rest of his
new bone formation. Following pathologic
examination findings are normal.
confirmation by diagnostic biopsy, additional
Laboratory test results: staging by whole-body bone scintigraphy using
technetium 99Tc (Answer E) is recommended,
Alkaline phosphatase = 190 IU/L (50-120 U/L)
which may be superior to other imaging modalities
(SI: 3.17 µkat/L [0.84-2.00 µkat/L])
Bone-specific alkaline phosphatase = 42 µg/L such as PET/CT for detection of additional
(≤20 µg/L) osseous and pulmonary metastases. In light of
the diagnosis of osteosarcoma, treatment with
Anteroposterior radiograph of the pelvis shows bisphosphonates (oral or intravenous) (Answers A
asymmetric cortical thickening of the inferior and B) or calcitonin (Answer C) is not indicated
medial ilium with focal interruption of the cortex, for this patient.
loss of local trabecular pattern, and evidence of
partial callus formation.

ENDO 2021 • Bone and Mineral Metabolism  131

chased by Dr. Khalid H. Seedahmed, Jr., MRCP PGDip, CertEndocrinology SA - ID 283


Case 5  New Content  Which of the following is the best next
step in this patient’s management?
A 65-year-old woman is referred after moving to
the area to establish care for known PDB. PDB A. Zoledronic acid, 5 mg intravenously
was diagnosed approximately 3 years ago, based B. Alendronate, 40 mg daily for 6 months
initially on incidentally discovered asymptomatic C. Continued clinical and biochemical
elevation of alkaline phosphatase. Subsequent surveillance every 6 to 12 months
studies, including plain x-rays and technetium D. Fasting serum C-telopeptide measurement
99
Tc whole-body bone scintigraphy confirmed the
E. Technetium 99Tc whole-body bone
diagnosis of PDB involving the right proximal
scintigraphy
femur. She was treated with zoledronic acid,
5 mg, with subsequent normalization of alkaline Answer: C) Continued clinical and biochemical
phosphatase after 3 months. Follow-up showed surveillance every 6 to 12 months
consistently normal alkaline phosphatase levels,
with the most recent measurement 6 months This patient has a history of PDB confirmed
ago. On presentation today, she is without biochemically and radiographically, with an
musculoskeletal symptoms, including the absence appropriate response to zoledronic acid based
of right hip and thigh pain or stiffness. Her on normalization of alkaline phosphatase levels.
medical history is otherwise noncontributory. She Although alkaline phosphatase has increased
takes calcium and vitamin D but no prescription again above the upper normal limit, there is
medications. Her family history is negative for no evidence that this necessarily indicates or
PDB or other skeletal disorders. predicts an episode of painful PDB. Therefore,
On physical examination, she appears well. She continued clinical and biochemical surveillance
is normotensive and has had minimal height loss (Answer C) is indicated for this patient. As
since youth. Examination of the right proximal demonstrated by the PRISM trials, there is no
femur shows no localized tenderness. The right indication for pharmacologic treatment with an
hip has full range of motion without elicited pain. oral or intravenous bisphosphonate (Answers A
and B) at this time, based on the lack of skeletal
Laboratory test results: pain involving the affected pagetic site. Whole-
Alkaline phosphatase = 140 U/L (50-120 U/L) body bone scintigraphy with technetium 99Tc
(SI: 2.34 µkat/L [0.84-2.00 µkat/L]) (Answer E) is also not indicated given a known
Calcium, normal diagnosis of monostotic PDB. Finally, although
Creatinine, normal skeletal turnover markers (Answer D) may be of
Intact PTH, normal use in the diagnostic confirmation of PDB, the
25-Hydroxyvitamin D, normal
clinical history and alkaline phosphatase elevation
render this approach moot in this particular
patient.

References
1. van Staa TP, Selby P, Leufkens HG, Lyles K, Sprafka JM, Cooper C. Incidence 4. Corral-Gudino L, García-Aparicio J, Sánchez-González MD, et al. Secular
and natural history of Paget’s disease of bone in England and Wales. J Bone changes in Paget’s disease: contrasting changes in the number of new referrals
Miner Res. 2002;17(3):465-471. PMID: 11878305 and in disease severity in two neighboring regions of Spain. Osteoporos Int.
2. Altman RD, Bloch DA, Hochberg MC, Murphy WA. Prevalence of pelvic 2013;24(2):443-450. PMID: 22395312
Paget’s disease of bone in the United States. J Bone Miner Res. 2000;15(3):461- 5. Visconti MR, Langston AL, Alonso N, et al. Mutations of SQSTM1 are
465. PMID: 10750560 associated with severity and clinical outcome in paget disease of bone. J Bone
3. 3: Corral-Gudino L, Borao-Cengotita-Bengoa M, Del Pino-Montes J. Miner Res. 2010;25(11):2368-2373. PMID: 20499339
Epidemiology of Paget’s disease of bone: a systematic review and meta-
analysis of secular changes. Bone. 2013;55(2):347-352. PMID: 23643679

132  ENDO 2021 • Endocrine Case Management

chased by Dr. Khalid H. Seedahmed, Jr., MRCP PGDip, CertEndocrinology SA - ID 283


6. Seton M, Hansen M, Solomon DH. The implications of the sequestosome 12. Corral-Gudino L, Tan AJ, Del Pino-Montes J, Ralston SH. Bisphosphonates
1 mutation P392L in patients with Paget’s disease in a United States cohort. for Paget’s disease of bone in adults. Cochrane Database Syst Rev.
Calcif Tissue Int. 2016;98(5):489-496. PMID: 26713335 2017;12:CD004956. PMID: 29192423
7. Amilibia Cabeza E, Holgado Pérez S, Pérez Grau M, et al. Hearing in Paget›s 13. Tan A, Ralston SH. Clinical presentation of Paget’s disease: evaluation of a
disease of bone. Acta Otorrinolaringol Esp. 2019;70(2):89-96. PMID: 29880223 contemporary cohort and systematic review. Calcif Tissue Int. 2014;95(5):385-
8. Young CA, Fraser WD, Mackenzie IJ. Detection of hearing impairment and 392. PMID: 25160936
handicap in Paget’s disease of bone using a simple scoring system: a case 14. Reid IR, Miller P, Lyles K, et al. Comparison of a single infusion of zoledronic
control study. Bone. 2007;40(1):189-193. PMID: 16962839 acid with risedronate for Paget’s disease. N Engl J Med. 2005;353(9):898-908.
9. Wermers RA, Tiegs RD, Atkinson EJ, Achenbach SJ, Melton LJ 3rd. PMID: 16135834
Morbidity and mortality associated with Paget’s disease of bone: a population- 15. Langston AL, Campbell MK, Fraser WD, MacLennan GS, Selby PL, Ralston
based study. J Bone Miner Res. 2008;23(6):819-825. PMID: 18269308 SH; PRISM Trial Group. Randomized trial of intensive bisphosphonate
10. Singer FR, Bone HG 3rd, Hosking DJ, et al; Endocrine Society. Paget’s disease treatment versus symptomatic management in Paget’s disease of bone. J Bone
of bone: an Endocrine Society clinical practice guideline. J Clin Endocrinol Miner Res. 2010;25(1):20-31. PMID: 19580457
Metab. 2014;99(12):4408-4422. PMID: 25406796 16. Tan A, Goodman K, Walker A, et al; PRISM-EZ Trial Group. Long-term
11. Ralston SH, Corral-Gudino L, Cooper C, et al. Diagnosis and management randomized trial of intensive versus symptomatic management in Paget’s
of Paget’s disease of bone in adults: a clinical guideline. J Bone Miner Res. disease of bone: the PRISM-EZ study. J Bone Miner Res. 2017;32(6):1165-1173.
2019;34(4):579-604. PMID: 30803025 PMID: 28176386

ENDO 2021 • Bone and Mineral Metabolism  133

chased by Dr. Khalid H. Seedahmed, Jr., MRCP PGDip, CertEndocrinology SA - ID 283


chased by Dr. Khalid H. Seedahmed, Jr., MRCP PGDip, CertEndocrinology SA - ID 283
DIABETES MELLITUS
AND GLUCOSE
METABOLISM

chased by Dr. Khalid H. Seedahmed, Jr., MRCP PGDip, CertEndocrinology SA - ID 283


Closing the Loop With Pump
and Sensor Communication
in Diabetes Mellitus: How
to Digest Data and Develop
Treatment Recommendations
During a Clinic Visit
Steven D. Wittlin, MD. University of Rochester School of Medicine and Dentistry, Rochester,
NY; E-mail: steven_wittlin@urmc.rochester.edu

Learning Objectives Significance of the


As a result of participating in this session, learners Clinical Problem
should be able to: In 1993, the DCCT (Diabetes Control and
• Select a sensor-pump system. Complications Trial) demonstrated that reducing
hemoglobin A1c resulted in a reduced rate of
• Interpret pump-sensor downloads in the office microvascular diabetes complications. However,
or clinic. “intensive” diabetes control resulted in a tripling
of the occurrence of severe hypoglycemia.1 To
address this, improvements in insulin pump
therapy, and especially real-time continuous
Main Conclusions glucose monitoring (RT-CGM), have enabled
Use of sensors, smart pumps, and hybrid closed- intensification of control while reducing
loop pumps has improved diabetes control. hypoglycemia.2 There is now integration of RT-
With the use of hybrid closed-loop pumps, CGM with pumps and algorithms, allowing for the
despite the improvement in glycemic control, development of sensor-augmented pump therapy
regulating postmeal glucose concentrations is still and its evolution into varying degrees of “closing-
a challenge. Counting carbohydrates facilitates the-loop” and hybrid closed-loop systems.3-5
bolus calculation, but it is an imperfect and This technology has presented the clinician
unsatisfactory tool when patients eat a mixed with new challenges, such as (1) correlating
meal. Metrics have been developed to assess hemoglobin A1c and the multitude of data points
diabetes control that aid in making adjustments to provided by new sensors, (2) assessing accuracy of
regimens and interpreting pump downloads. sensors vs fingerstick blood glucose measurements,
(3) interpreting RT-CGM data, (4) selecting
an appropriate system for a given patient, (5)
trouble-shooting various data and systems, and (6)

136  ENDO 2021 • Endocrine Case Management

chased by Dr. Khalid H. Seedahmed, Jr., MRCP PGDip, CertEndocrinology SA - ID 283


handling downloads of data in an efficient way • Needle introducer
during a busy clinic visit. This session attempts to • “Running out of sites”
address many of these issues.
• Possible skin/site problems
• “Compression lows”
Barriers to Optimal Practice
• Not too close to infusion site
• Logistics of office download and interpretation
of data from pump/CGM and closed-loop Medtronic
systems.
• Rapid changes in available technology with • Current iteration: Guardian 3
slow changes in insurance coverage.
Pros
• Cost of new diabetes technologies.
• Need for expertise in using the growing • Alerts
number of and changing diabetes technologies. • MARD <10%
• Alerts identified individuals
• Sensor life = 7 days
Strategies for Therapy
• Excellent download display
and Management
• Has its own “pump partner”
Real-Time Continuous Glucose
• Predictive alerts
Monitors (Sensors)
RT-CGMs currently available in United States are Cons
presented below.
• Patient must insert every 7 days
DexCom • Needle introducer
• Current iteration: G6 • “Running out of sites”
• Possible skin/site problems
Pros
• Requires calibration
• Factory calibrated
• Mean absolute relative difference (MARD) <10% Eversense (Senseonics):
• Sensor life = 10 days Wired-Enzyme Technology
• No need for fingerstick confirmation Pros
• High and low alerts • Accurate: MARD <10%
• Alerts identified individuals • Sensor life = 3 months United States; 6 months
• Predictive alerts European Union
• Implanted, does not use up skin sites
Cons
• Improved adherence/use (it is implanted)
• Patient must insert every 10 days • Alerts including vibratory directly
• Study confirming no need for calibration had • Alerts identified individuals
multiple exclusions
• No needle

ENDO 2021 • Diabetes Mellitus and Glucose Metabolism  137

chased by Dr. Khalid H. Seedahmed, Jr., MRCP PGDip, CertEndocrinology SA - ID 283


Cons • Remaining 3 all work; if calibration error is an
issue, DexCom has an advantage
• Minor office procedure (time)
• Needs calibration
If the patient has hypoglycemia
• Occasional removal difficulties unawareness/severe hypoglycemia
• No “pump partner” and a pump or desire for a pump:
• Eversense does not have a “pump partner” at
Freestyle Libre: Flash Technology present, so would not use if threshold suspend
Pros or hybrid closed loop are desired
• If Medtronic System (vide infra) is desired,
• Easy to use then Guardian 3
• Less expensive • If desire Tandem System (vide infra) is desired,
• Accurate: MARD <10%-15% then DexCom
• Factory calibrated • NB: Omnipod may partner with DexCom/
• Sensor life = 14 days Libre in the near future pending FDA approval

Cons So When Eversense?6


• Older US version does not have alerts; Libre 2 • Body image issues/athletics
has optional alarms
• Adherence to sensor issues
• Requires patient to actively interrogate sensor
for result, raising adherence issues • Patient issues:
• Does not notify another person ѓ Such as hearing, seeing
• No current US “pump partners” ѓ Patient preference
• Needle implantation • “Running out of sites”
• No predictive alerts • Needle phobia
Clearly, choosing a sensor requires a conversation
Choosing a Sensor between the provider as a guide and the patient.
Insurance coverage plays a role. For example,
If the patient has no hypoglycemia at the time of this writing, Medicare is only
unawareness and/or type 2 diabetes: “covering” Freestyle Libre and Dexcom.

• All 4 sensors work; Libre may be easier, less


Selecting a Pump
costly, and at least as accurate as fingerstick
measurements and is factory calibrated, thus Again, patient preference is paramount. Below are
reducing calibration error several considerations:
• If a sensor-augmented system or closed-loop
If the patient has hypoglycemia system will be used, the sensor may dictate
unawareness/severe hypoglycemia: choice of pump

• Not Libre (no alerts), but consider Libre 2, • Omnipod is “tubeless” but has a smaller reservoir;
which does have alarms it requires a PDM (Personal Diabetes Manager)

138  ENDO 2021 • Endocrine Case Management

chased by Dr. Khalid H. Seedahmed, Jr., MRCP PGDip, CertEndocrinology SA - ID 283


• Medtronic has its own sensor • Timing: late boluses can result in both early
hyperglycemia and late hypoglycemia
ѓ Longest time on market
• Type of insulin
ѓ Has a meter that directly communicates
with the pump • Food content:

• Tandem has a sensor partner ѓ Traditional: insulin-to-carbohydrate ratio


ѓ Bell et al and others have shown a
ѓ Can do software updates without getting a
significant effect of fat and protein on
new pump glucose levels, resulting in a blunted early
ѓ More stylish glucose peak and a late rise in glucose; there
is no proven formula for this adjustment9
Sensor–Pump Interactions: • Bolus calculators prevent insulin “stacking”
Toward “Closing the Loop”
Closed-loop systems involve 3 basic elements:
Special Situations
1. Pump
2. Sensor • Exercise
3. Controller (an algorithm) • Pregnancy
• Hospitalization
The sophistication of these systems has progressed
over the past 2 decades:
• Low-glucose (threshold) suspend systems: the Reading a Sensor/Pump Download
pump shuts off if a low glucose value is used An international consortium and others have
• Predictive low glucose suspend: the algorithm created an Ambulatory Glucose Profile involving
turns the pump off if low value is impending metrics for assessing diabetes control when using
• Hybrid closed-loop systems: the algorithm RT-CGM.10 The standardized metrics for clinical
corrects the glucose toward a target/target care (2019) are as follows (recommendations in
range; the patient still determines meal boluses parentheses):

ѓ Medtronic 670 G7 and 770 1. Number of days worn (14 days)


ѓ Control-IQ (Tandem and DexCom): 2. Percentage of time CGM active (70%)
automated correction bolus8 3. Mean glucose
ѓ In France, Diabeloop (DBL G1) 4. Glucose management indicator

• DIY Systems (Do-It-Yourself): not FDA approved; 5. Glycemic variability (<36%)


success appears user-related 6. Time above range >250 mg/dL (>13.9 mmol/L)
7. Time above range 181-250 mg/dL
(10.0-13.9 mmol/L)
Successful Meal Bolus
8. Time in range
• Ensure the basal rate(s) is/are correct 9. Time below range: 54-69 mg/dL
(3.0-3.8 mmol/L)
• Appropriate mode (eg, regular vs square-wave
vs “combo bolus”) (see below) 10. Time below range <54 mg/dL (<3.0 mmol/L)

ENDO 2021 • Diabetes Mellitus and Glucose Metabolism  139

chased by Dr. Khalid H. Seedahmed, Jr., MRCP PGDip, CertEndocrinology SA - ID 283


Usual Target Recommendations The formula for GMI is as follows:
• Time in range >70% GMI (%) =3.31+0.02392 × mean CGM glucose
• Time below range <4% (mg/dL)
• Time above range <25% Each change of 25 mg/dL results in a GMI increase
of ~0.6%.
For older patients, looser time in range and time
above range are appropriate to avoid time below Using GMI
range. Pregnancy targets for time in range are • If GMI is lower than hemoglobin A1c, consider:
63 to 140 mg/dL (3.5-7.8 mmol/L).
ѓ Avoiding hypoglycemia
Regarding Time in Range 11
ѓ Perhaps increasing hemoglobin A1c target
Fourteen days suffice to correlate well with
hemoglobin A1c. • If GMI is higher than hemoglobin A1c,
consider:
Advantages of Time in Range
ѓ Addressing hyperglycemia
• It responds faster to interventions than does ѓ Perhaps lowering hemoglobin A1c target
hemoglobin A1c
• It reflects glycemic variability
• It is unaffected by factors that affect Clinical Case Vignettes
hemoglobin A1c (eg, red-cell half-life)
Case 1
• It correlates with microvascular complications
A 35-year-old woman with type 2 diabetes and a
• The relative measurement error of time in history of hearing loss (and a mother with hearing
range is approximately 4%; for hemoglobin A1c loss) comes to see you. She is 5 months pregnant.
approximately 5% She has had persistently elevated fasting blood
glucose levels (~110 mg/dL [6.1 mmol/L]). She
Using Time in Range is prescribed 12 units NPH insulin at bedtime.
She took metformin before pregnancy. She is
• If there is a lack of concordance between time now wearing a Freestyle Libre Sensor. Her sensor
in range and hemoglobin A1c download is shown (see images).
ѓ One needs to assess for any interferences in Summary
hemoglobin A1c
Average Time In Range Coefficient Standard
ѓ One might reset hemoglobin A1c target Glucose of Deviation
Above 180 mg/dl Variation (SO)
ѓ It may reflect a recent change in glycemic (abo-11 250 ffl'>'dL 0"1-.) 3% (CV)

control 103
mg/dL
29.h . 30
mg/dl
In Target Range
• CGM values are only as good as the “gold- 70-180 mg/dl 88o;,
88--116'
standard” to which the CGM is calibrated
19-25' 10·26'

Below 70 mgldl
lbelcm S4 ffl<"~dL. 1,l
g .,,, ('.
Glucose Management Indicator 12

Glucose management indicator (GMI) is a new


term for estimating hemoglobin A1c from CGM
readings—mainly to avoid confusion.

140  ENDO 2021 • Endocrine Case Management

chased by Dr. Khalid H. Seedahmed, Jr., MRCP PGDip, CertEndocrinology SA - ID 283


Ambula10fY Glucose Profile overnight glucose levels are on the low side, but
CUNeS/pfots rep(eseol glucose frequency distribtJIJons b)' time tegacdless of 08.fe
still within target range. Adding metformin might
300-,-- - - - - - - - - - - - - - - - - - ~
f119'dL decrease the overnight glucose values even further.
300

,..
Case 2
A 46-year-old man has had type 1 diabetes
. __________________
..._
1~ .....
)tjJ'II
__.
12,m
for 30 years. His hemoglobin A1c level is 8.1%
(65 mmol/mol). He has had frequent high and
low blood glucose values. He uses a 670G hybrid
closed-loop pump. He does not understand why
Which of the following is the best next step? his glucose values are so variable on a closed-loop
A. Continue the regimen; her time in range is 88% system. The image below shows 6 days of data
from his download.
B. Reduce her dosage of NPH; her time below
range is 9% Which of the following is the
C. Move her NPH insulin to the morning most likely explanation?
D. Re-introduce metformin A. Inaccurate carbohydrate counting
Answer: C) Move her NPH insulin to the morning B. High-fat meals
C. Inappropriate insulin-to-carbohydrate ratio
The target glucose range for pregnancy is
D. Late meal boluses
63 to 140 mg/dL (3.5-7.8 mmol/L). Thus, her
postlunch blood glucose values are elevated. Answer: D) Late meal boluses
Moving her NPH insulin to the morning
(Answer C) is therefore the best next step. Her As the figure illustrates, the blood glucose levels
start to rise BEFORE the meal bolus delivery.

_.. _.,,
....

_., I - - ~ ...rt I ,M- ~ "'7tn l ~ 1- ~ - ~ ...


..... .., G
e dldl di I
~
~ ~=1 .a, : 4)<1> ~

,......,. .. -··
~
,. ~
• a
£xJt Ro•a.on Oota11a

~Oi,;JnQt. . . . . . /1,U,o~Ol . . .~ -
~~CM'WCIICle~

- ...,.

·-·- - -· -- :- -

ENDO 2021 • Diabetes Mellitus and Glucose Metabolism  141

chased by Dr. Khalid H. Seedahmed, Jr., MRCP PGDip, CertEndocrinology SA - ID 283


Because of the delay, there is inadequate 23 years ago, added a real-time CGM 8 years ago,
insulin early after meals and excess insulin and has been on a hybrid closed-loop 670 G system
in the later postprandial period, resulting in for the past 9 months. She states that her glucose
late hypoglycemia. Thus, late meal boluses control has markedly improved while using her
(Answer D) is the most likely explanation. closed-loop system, but she still has occasional
postdinner high blood glucose values, as well as
troublesome low values. Her pump and sensor data
Case 3  New Content 
are shown (see images).
A 51-year-old woman has had type 1 diabetes
for approximately 35 years. She initially took
multidose insulin, “graduated” to an insulin pump

..,,Pe:rotfltii. ~.mot't

·- ®

Hypogtycemc PG""'1\S (sr• ti fl)bodes (per day): 0D ~ pettems (2) I ~ (per day)'. 0 I
@t--------------------+-------------------
O O
A A
0 ,s ~,
U :lAAN.• 1.:l&P"
V
1:2iPM -9'..DaPM
,, OCCUIT«alJ V
l:27AM•2:22AM
c1OCQ.lnl'Qt )
10 18PM·l2.'00l>M U.S-PW--A.'.M'P-t.11

@ AUloModeExltl @ © @ ©
.....,
Stalistlca
S'!I.
,..cauno,n
16'!1.
Hlgtl!;CAJil!lol,OOI-~
• •• l
• I
I •
0
5 ~MootCP«•H«l
Wll'Wltw.,odt(.Pt'WHI.)
'315'11_,I
11'!1.(td°"')
G'!I. !JdUOJ

!ier..otWt»(per-.1\ ml,(5d tot,) 11'!1. 1"'13111


• ,,... ss....,._
,...
• 1
8, AIIIOMOOt-ffl.u~ Alf~SOaS-l> US t S4.,.;dl
C CMr"
e AJMU,QOI N, ~ • I 0

• c.~~V1dl:ooo('4j ""
Xl'!I. 37A

--.._
0
E 90ntq!#Ntot'AMfrJWOO.
...,,,.soAIOfS(Jlor..,., :Llltl ISI OI
~ 1, ... 76'!1. ~~UnOtfft-JCI l •• •
• • 2 2 •• 6 A'l~;,taG
eGl ~ ( p o r . . ,J
IS5tS5m9"ft.
UIU
1:Ut&l-Odl.
u ru
..,,.,.
NoSGVA\ln 0
B __(pt!..,.,
T.... O>OJ ....f;>tr..,I 12...t> 7)-

-
05\lliffl M•J~)

.. ..
- ..
3'11.

--.
Ar.to M.~ cSJWOltd t,J ~

._ .._.,...,
• 1
0

l ••
0

ii
MIOl!aal,1 9-.al ~ ( p t f CliJ)1
SgCNtlgt

-CIW!9t
VUl (Pe<""J)
2711 lll'!I.I

E"'JOd.ay>
12'
40,&t62g
3CU\U'!I.I
b"tlOIDI"
E...-,20..._.
,a
llhMG

- , !211

III I r ~
■)
......... -- -·· M·M·
........... -""'"""' - I·- _..,,....,.., ......
L lJ
I Exit Reason Details

142  ENDO 2021 • Endocrine Case Management

chased by Dr. Khalid H. Seedahmed, Jr., MRCP PGDip, CertEndocrinology SA - ID 283


Which of the following is the most likely would like to switch to a different brand of CGM
reason for her low blood glucose values? due to the inaccuracy.
A. Overcorrecting her blood glucose
Which of the following is the
B. Sensor error
most likely explanation?
C. Premeal exercise
A. Not calibrating the sensor correctly
D. Incorrect insulin-to-carbohydrate ratio
B. Compression of the sensor in his sleep
Answer: A) Overcorrecting her blood glucose C. Excessive basal insulin overnight
This patient’s hypoglycemia is occurring after D. Inaccurate fingerstick technique
spikes in blood glucose. It happens at variable Answer: B) Compression of the sensor in his sleep
times of the day. Both findings make exercise-
induced hypoglycemia less likely. Moreover, Various physical forces may affect the accuracy of
she is taking “correction boluses” at a time that a sensor, including state of hydration, temperature
the closed-loop system is already correcting her changes, and compression of the sensor, which
blood glucose. Thus, she is in effect inadvertently change the interstitial milieu in which the sensor
“stacking” insulin and overcorrecting (Answer A). resides. The most likely explanation for the
observations in this vignette is compression of
the sensor while the patient sleeps (Answer B).
Case 4  New Content 
Sensors are generally least accurate during the
A 60-year-old man uses a DexCom G6 sensor. His first 2 to 24 hours after insertion, as they adjust
BMI is 23 kg/m2. He has had latent autoimmune to the interstitial milieu. When using a sensor
diabetes in adults (LADA) for 13 years. He recently that requires patient calibration (eg, Eversense
acquired a glucose sensor. He notes recurrent low or Guardian 3), it is crucial that calibration be
blood glucose values around 3 AM. These are not performed when the blood glucose is relatively
confirmed by fingerstick blood glucose checks. stable due to the lag time between blood and
He used fingerstick blood glucose to recalibrate interstitial compartment changes in glucose.
his sensor, with even worsening accuracy. He is
tired of being awakened at night, as is his wife. He

References
1. Hypoglycemia in the Diabetes Control and Complications Trial. The Diabetes 7. Garg SK, Weinzimer SA, Tamborlane WV, et al. Glucose outcomes with the
Control and Complications Trial Research Group. Diabetes. 1997;46(2):271- in-home use of a hybrid closed-loop insulin delivery system in adolescents
286. PMID: 9000705 and adults with type 1 diabetes. Diabetes Technol Ther. 2017;19(3):155-163.
2. Juvenile Research Foundation Continuous Glucose Monitoring Study Group, PMID: 28134564
Tamborlane WV, Beck RW, et al. Continuous glucose monitoring and 8. Brown SA, Kovatchev BP, Raghinaru D, et al; iDCL Trial Research Group.
intensive treatment of type 1 diabetes. N Engl J Med. 2008;359(14):1464-1476. Six-month randomized, multicenter trial of closed-loop control in type 1
PMID: 18779236 diabetes. N Engl J Med. 2019;381(18):1707-1717. PMID: 31618560
3. Roy RW, Bergenstal RM, Laffel LM, Pickup JC. Advances in technology for 9. Bell KJ, Fio CZ, Twigg S, et al. Amount and type of dietary fat, postprandial
management of type 1 diabetes. Lancet. 2019;394(10205):1265-1273. PMID: glycemia, and insulin requirements in type 1 diabetes: a randomized within-
31533908 subject trial. Diabetes Care. 2020;43(1):59-66. PMID: 31455688
4. Kovatchev B. A century of diabetes technology: signals, models, and artificial 10. Battelino T, Danne T, Bergenstal RM, et al. Clinical targets for continuous
pancrease control. Trends Endocrinol Metab. 2019;30(7):432-442. PMID: glucose monitoring data interpretation: recommendations from the
31151733 International Consensus on Time in Range. Diabetes Care. 2019;42(8):1593-
5. Tauschmann M, Hovorka R. Nat Rev Endocrinol. 2018;14(8):464-475. PMID: 1603. PMID: 31177185
29946127 11. Beck RW, Bergenstal RM, Cheng P, et al. The relationships between
6. Deiss D, Szadkowska A, Gordon D, et al. Clinical practice recommendations time in range, hyperglycemia metrics, and HbA1c. J Diabetes Sci Technol.
on the routine use of Eversense, the first long-term implantable continuous 2019;13(4):614-626. PMID: 30636519
glucose monitoring system. Diabetes Technol Ther. 2019;21(5):254-264. PMID: 12. Bergenstal RM, Beck RW, Close KL, et al. Glucose management indicator
31021180 (GMI): a new term for estimating A1C from continuous glucose monitoring.
Diabetes Care. 2018;41:2275-2280. 30224348

ENDO 2021 • Diabetes Mellitus and Glucose Metabolism  143

chased by Dr. Khalid H. Seedahmed, Jr., MRCP PGDip, CertEndocrinology SA - ID 283


Practical Approaches to
the Genetic Diagnosis and
Management of Patients
With Diabetes Mellitus
Rochelle N. Naylor, MD. Departments of Pediatrics and Medicine, Section of Adult and
Pediatric Endocrinology, Diabetes and Metabolism, University of Chicago, Chicago, IL;
E-mail: rnaylor@bsd.uchicago.edu

Learning Objectives HNF4A-MODY. GCK-MODY does not need


treatment outside of pregnancy. In pregnancies
As a result of participating in this session, learners
affected by GCK-MODY, treatment is based
should be able to:
on fetal genotype. HNF1B-MODY most often
• Identify the clinical and laboratory features requires insulin therapy, but accurate diagnosis
that suggest a diagnosis of monogenic diabetes. can allow for surveillance and management of the
renal and other extra-pancreatic features that are
• Describe features and appropriate
a common manifestation of heterozygous HNF1B
management for the most common forms of
pathogenic variants. Clinicians must consider
monogenic diabetes.
monogenic diabetes at initial diabetes classification
to ensure timely diagnosis and appropriate
treatment to improve diabetes outcomes.
Main Conclusions
While monogenic diabetes represents an
uncommon form of diabetes, accurate diagnosis
Significance of the
through genetic testing directs treatment and Clinical Problem
management and identifies affected and at-risk Monogenic diabetes, due to single gene pathogenic
family members. Several clinical and laboratory variants or chromosomal abnormalities, accounts
features can help identify a patient highly likely for approximately 2% of young-onset diabetes.
to have monogenic diabetes, who should be The main phenotypes are neonatal diabetes, which
referred for genetic testing. These include can be permanent or transient, and maturity-onset
negative pancreatic autoantibodies and continued diabetes of the young (MODY). The vast majority
endogenous insulin production more than 3 of monogenic diabetes cases are misdiagnosed as
years after diabetes diagnosis. The most common type 1 or type 2 diabetes. Studies estimate that
forms of monogenic diabetes have gene-directed more than 80% of cases go undiagnosed or are
therapies that may improve glycemic control and misdiagnosed for many years.1,2 Misdiagnosis
outcomes. This includes sulfonylureas for K-ATP of MODY leads to improper and unnecessary
channel–related neonatal diabetes, relapsed treatment. Misdiagnosis increases health care
6q24 neonatal diabetes, HNF1A-MODY, and expenditures due to unnecessary treatment or

144  ENDO 2021 • Endocrine Case Management

chased by Dr. Khalid H. Seedahmed, Jr., MRCP PGDip, CertEndocrinology SA - ID 283


inappropriate medications, including intensive Strategies for Diagnosis,
insulin therapy. Patients with MODY are also
Therapy, and/or Management
exposed to adverse medication effects as a result of
improper management.2 Several clinical features can raise suspicion
While there is considerable clinical overlap for monogenic diabetes. Diabetes diagnosis
between monogenic diabetes and both type 1 and in patients younger than 6 months nearly
type 2 diabetes, there are clinical and laboratory always implies a monogenic cause. MODY is
features that can help providers identify patients an autosomal dominant condition and thus is
who may have monogenic diabetes. Several studies typically characterized by a strong family history
have demonstrated the usefulness of biomarkers of diabetes over several generations. While obesity
to identify patients who should undergo genetic does not preclude monogenic diabetes, nonobese,
testing for monogenic diabetes.3-5 Additionally, young-onset, non–insulin-dependent diabetes,
there is a MODY probability calculator that uses particularly with a positive family history, is highly
clinical information to assess the probability of likely to be monogenic in nature.
a patient having MODY.6 Cost-effectiveness Laboratory testing can also help to identify
analyses suggest that genetic testing for monogenic patients likely to have monogenic diabetes,
diabetes in appropriate populations can be cost- with published studies showing the utility of
saving.7-9 Genetic diagnosis of certain forms using biomarkers to identify individuals for
of monogenic diabetes is clinically actionable, genetic testing.3-5 “Type 1 diabetes” with absence
allowing therapy selection based on the genetic of pancreatic autoantibodies or continued
cause, often with decreased burden of treatment endogenous insulin production, as measured
and stable or improved glycemia. Proband by serum or urine C-peptide, several years after
diagnosis also aids in earlier diagnosis and correct diabetes diagnosis should raise suspicion for
classification of diabetes in family members. monogenic diabetes.10,11 Studies in “type 2 diabetes”
have used anthropometric data (BMI, absence
of metabolic syndrome) and high-sensitivity
Barriers to Optimal Practice C-reactive protein levels to discriminate likely
type 2 diabetes from possible HNF1A-MODY.12
• Significant overlap between type 1 and type A prediction model for MODY, known as the
2 diabetes precludes definitive diagnosis of MODY calculator, based on clinical features and
monogenic diabetes solely on a clinical basis. hemoglobin A1c, was developed and validated in
Providers may be unaware of the clinical a United Kingdom population and is available
features that should prompt consideration of a without charge.6
monogenic diabetes diagnosis. Genetic testing for monogenic diabetes
• Navigating the process for genetic testing to is available in numerous certified clinical
confirm monogenic diabetes can be time- and laboratories. All individuals diagnosed with
labor-intensive for providers. Moreover, diabetes under 6 months of age should undergo
genetic testing reports may be difficult to comprehensive genetic testing using next-
interpret. generation sequencing strategies. It is reasonable
• Providers may be unaware of the benefits to consider testing in those diagnosed between 6
of gene-targeted therapy in certain forms of and 12 months of age, although rates of neonatal
monogenic diabetes. diabetes are markedly decreased compared with
rates in individuals diagnosed before 6 months
• Providers may be unaware of the extra- of age. In contrast, MODY lacks a single clinical
pancreatic features and complication risks in criterion to identify patients who should have
certain forms of monogenic diabetes. genetic testing, and screening the entire diabetes

ENDO 2021 • Diabetes Mellitus and Glucose Metabolism  145

chased by Dr. Khalid H. Seedahmed, Jr., MRCP PGDip, CertEndocrinology SA - ID 283


population for MODY is not economically feasible. disproportionally above the 75th percentile on
Thus, it is important to target genetic testing second trimester scans.14
to high-risk populations, using biomarkers as Other forms of MODY (HNF1A-MODY,
described above. Interpretation of genetic testing HNF4A-MODY, and HNF1B-MODY) are
reports can be confusing and lead to erroneous typically diagnosed in adolescence or early
diagnosis of monogenic diabetes when a variant adulthood (before age 35 years, and often under
of uncertain significance is detected. Consultation 25 years). HNF1A-MODY and HNF4A-MODY
with experts in monogenic diabetes is advised for exhibit a marked sensitivity to sulfonylureas,
any uncertainty after genetic testing. which bypass the major β-cell defect. A number
Genetic diagnosis of certain forms of of studies have demonstrated equal or improved
monogenic diabetes is clinically actionable, control as compared with insulin therapy.15,16 The
allowing therapy selection based on the genetic majority of people with HNF1B-MODY require
cause. Permanent neonatal diabetes owing to insulin therapy. Heterozygous HNF1B pathogenic
pathogenic variants in KCNJ11 or ABCC8, the variants cause a spectrum of abnormalities
genes encoding the ATP-sensitive potassium that can be assessed for and managed once an
channel in the β cell, respond to high-dosage accurate diagnosis is made. Heterozygous HNF1B
sulfonylureas with excellent glycemic control and pathogenic variants are the most common cause
improvement in neurologic outcomes. Those with of developmental renal disease, which often
transient neonatal diabetes often have relapse manifests as cysts but single kidneys and renal
of diabetes in adolescence or early adulthood. hypoplasia may also occur. Renal function is often
Relapse of transient neonatal diabetes due to 6q24 impaired. Additional features include pancreatic
abnormalities can be treated with low doses of hypoplasia that may include subclinical exocrine
sulfonylureas.13 pancreas dysfunction, genital tract abnormalities,
GCK-MODY is characterized by stable mildly abnormal liver function, magnesium wasting, and
elevated blood glucose due to a glucose-sensing hyperuricemia and gout.17
defect. Mild hyperglycemia is present at birth
but often diagnosed incidentally, such as during
a routine exam or while investigating another
Clinical Case Vignettes
medical complaint. GCK pathogenic variants cause
mild, regulated hyperglycemia, so treatment is not Case 1
needed outside of pregnancy. Studies have shown A 32-year-old woman with a known diagnosis
that discontinuation of therapy in GCK-MODY of GCK-MODY is currently on no treatment.
does not alter hemoglobin A1c. There is very She plans to conceive this year and is asking for
minimal deterioration of glucose tolerance over guidance on management of GCK-MODY during
the long term due to decreased insulin sensitivity pregnancy.
with insulin secretion remaining stable, as
also occurs in the nondiabetic population with Based on current recommendations
age. At this level of glycemic control, diabetes- and studies in GCK-MODY, which
related complications are rare. In pregnancies of the following should you tell
of women with GCK-MODY, the risk of fetal her regarding insulin therapy?
macrosomia is based on whether the fetus A. Early initiation of insulin therapy will reduce
inherits the heterozygous GCK pathogenic fetal macrosomia and has not been associated
variants from the mother. Insulin therapy is with adverse effects
recommended only when the fetus is known or
B. Initiation of insulin therapy should be based
suspected to not carry a GCK pathogenic variant,
on second-trimester maternal glycemia
based on fetal abdominal circumference rising

146  ENDO 2021 • Endocrine Case Management

chased by Dr. Khalid H. Seedahmed, Jr., MRCP PGDip, CertEndocrinology SA - ID 283


C. Initiation of insulin therapy should be based Case 2
on second-trimester fetal growth
A 26-year-old man has been referred to you
D. Fetal genotype should be assessed by for a recent genetic diagnosis of HNF1A-
amniocentesis or chorionic villus sampling to MODY. Monogenic diabetes was suspected
guide decisions on insulin therapy due to his ability to miss basal insulin and
E. Insulin therapy is not needed in pregnancies continued C-peptide production. He was initially
affected by GCK-MODY misdiagnosed with type 1 diabetes 6 years ago
and is on insulin therapy. He has no additional
Answer: C) Initiation of insulin therapy should
significant medical history, including no known
be based on second-trimester fetal growth
complications of diabetes. He is currently receiving
With the exception of pregnancy, treatment is not insulin glargine, 8 units daily, and takes 1 to 2
effective and not recommended in GCK-MODY. units of insulin aspart with meals. On physical
In pregnancies of women with GCK-MODY, the examination, his weight is 176 lb (80 kg), and
risk of fetal macrosomia is based on whether the height is 74 in (188 cm) (BMI = 22.6 kg/m2). Vital
fetus inherits the heterozygous GCK pathogenic signs are normal and examination findings are
variant from the mother. If the fetus carries a unremarkable. His hemoglobin A1c value is 7.2%
heterozygous GCK pathogenic variant, it will sense (55 mmol/mol).
mother’s mildly elevated blood glucose levels as
normal and grow normally. If the fetus does not In addition to monitoring blood glucose
carry the heterozygous GCK pathogenic variant, it closely via glucose meter, which of the
is at risk for macrosomia. following is the best plan for transitioning
While there is some debate in the literature, him to sulfonylurea therapy?
the largest consensus on management of GCK- A. Continue insulin at his current dosages and
MODY in pregnancy is to base therapy on known start glyburide ½ of the 1.25 mg tablet daily
or suspected fetal genotype. Amniocentesis or B. Discontinue meal-time insulin and start
chorionic villus sampling for the sole purpose glyburide ½ of the 1.25 mg tablet daily
of assessing fetal genotype for a heterozygous C. Discontinue basal insulin and start glyburide ½
GCK pathogenic variant (Answer D) is not of the 1.25 mg tablet daily
recommended due to the risk of pregnancy
D. Stop all insulin and start glyburide ½ of the
loss. For most cases where fetal genotype is
1.25 mg tablet daily
not known, it can be inferred based on second-
trimester fetal growth on ultrasonography. Answer: D) Stop all insulin and start
Current recommendations advise to consider glyburide ½ of the 1.25 mg tablet daily
insulin therapy if fetal abdominal circumference is
rising disproportionally above the 75th percentile There are several things to consider when
(Answer C) and to induce labor at 38 weeks. transitioning to sulfonylurea therapy, including
Women with GCK-MODY may experience current treatment (insulin vs noninsulin glucose-
hypoglycemia, including severe hypoglycemia, lowering therapy), duration of diabetes, and
when treated with insulin during pregnancy. weight (presence or absence of obesity). Patients
Additionally, there is a theoretical risk of growth should have evidence of ongoing endogenous
restriction in GCK-MODY affected fetuses whose insulin production. For those who are on insulin
mothers receive insulin therapy. In cases where therapy, this can be inferred from subreplacement
insulin is given for known or suspected unaffected doses or the ability to miss insulin, and can
fetuses, the effect on birthweight is limited due to be confirmed via measurement of C-peptide.
the difficulty in altering maternal glycemia (thus, Successful transfer off insulin is more likely in
Answer A is incorrect). patients younger than 40 years who have had

ENDO 2021 • Diabetes Mellitus and Glucose Metabolism  147

chased by Dr. Khalid H. Seedahmed, Jr., MRCP PGDip, CertEndocrinology SA - ID 283


a diabetes duration less than 20 years. Normal- duration of diabetes (>20 years), basal insulin can
weight patients are also more likely to successfully be decreased by half and bolus insulin discontinued
transition off insulin. upon starting a low-dosage sulfonylurea. Basal
Patients with HNF1A-MODY are often insulin should be decreased as the sulfonylurea
very sensitive to low dosages of sulfonylureas; dose is titrated up.
starting with one-half tablet of the lowest dose In this vignette, positive C-peptide indicates
and titrating up to achieve adequate glycemic that the patient continues to make insulin.
control is advised. Those who are on oral or other Additionally, he is on subreplacement doses of
noninsulin therapy or who are on subreplacement insulin (0.14-0.18 units per kg daily). He is young,
doses of insulin can be transitioned directly normal weight, and has a short diabetes duration.
to low-dosage sulfonylureas. For patients on Thus, he is very likely to successfully transition to
replacement doses of insulin or who have a long sulfonylurea therapy (Answer D).

References
1. Shields BM, Hicks S, Shepherd MH, Colclough K, Hattersley AT, Ellard 9. Goodsmith MS, Skandari MR, Huang ES, Naylor RN. The impact of
S. Maturity-onset diabetes of the young (MODY): how many cases are we biomarker screening and cascade genetic testing on the cost-effectiveness
missing? Diabetologia. 2010;53(12):2504-2508. PMID: 20499044 of MODY genetic testing. Diabetes Care. 2019;42(12):2247-2255. PMID:
2. Carmody D, Naylor RN, Bell CD, et al. GCK-MODY in the US National 31558549
Monogenic Diabetes Registry: frequently misdiagnosed and unnecessarily 10. McDonald TJ, Colclough K, Brown R, et al. Islet autoantibodies can
treated. Acta Diabetol. 2016;53(5):703-708. PMID: 27106716 discriminate maturity-onset diabetes of the young (MODY) from type 1
3. Pihoker C, Gilliam LK, Ellard S, et al; SEARCH for Diabetes in Youth Study diabetes. Diabet Med. 2011;28(9):1028-1033. PMID: 21395678
Group. Prevalence, characteristics and clinical diagnosis of maturity onset 11. Besser REJ, Shepherd MH, McDonald TJ, et al. Urinary C-peptide creatinine
diabetes of the young due to mutations in HNF1A, HNF4A, and glucokinase: ratio is a practical outpatient tool for identifying hepatocyte nuclear factor
results from the SEARCH for Diabetes in Youth. J Clin Endocrinol Metab. 1-{alpha}/hepatocyte nuclear factor 4-{alpha} maturity-onset diabetes of the
2013;98(10):4055-4062. PMID: 23771925 young from long-duration type 1 diabetes. Diabetes Care. 2011;34(2):286-291.
4. Shepherd M, Shields B, Hammersley S, et al; UNITED Team. Systematic PMID: 21270186
population screening, using biomarkers and genetic testing, identifies 2.5% 12. Thanabalasingham G, Shah N, Vaxillaire M, et al. A large multi-centre
of the U.K. pediatric diabetes population with monogenic diabetes. Diabetes European study validates high-sensitivity C-reactive protein (hsCRP) as
Care. 2016;39(11):1879-1888. PMID: 27271189 a clinical biomarker for the diagnosis of diabetes subtypes. Diabetologia.
5. Shields BM, Shepherd M, Hudson M, et al; UNITED study team. Population- 2011;54(11):2801-2810. PMID: 21814873
based assessment of a biomarker-based screening pathway to aid diagnosis of 13. Hattersley AT, Patel KA. Precision diabetes: learning from monogenic
monogenic diabetes in young-onset atients. Diabetes Care. 2017;40(8):1017- diabetes. Diabetologia. 2017;60(5):769-777. PMID: 28314945
1025. PMID: 28701371 14. Chakera AJ, Steele AM, Gloyn AL, et al. Recognition and management
6. Shields BM, McDonald TJ, Ellard S, Campbell MJ, Hyde C, Hattersley AT. of individuals with hyperglycemia because of a heterozygous glucokinase
The development and validation of a clinical prediction model to determine mutation. Diabetes Care. 2015;38(7):1383-1392. PMID: 26106223
the probability of MODY in patients with young-onset diabetes. Diabetologia. 15. Shepherd M, Shields B, Ellard S, Rubio-Cabezas O, Hattersley AT. A genetic
2012;55(5):1265-1272. PMID: 22218698 diagnosis of HNF1A diabetes alters treatment and improves glycaemic control
7. Greeley SAW, John PM, Winn AN, et al. The cost-effectiveness of in the majority of insulin-treated patients. Diabet Med. 2009;26(4):437-441.
personalized genetic medicine: the case of genetic testing in neonatal diabetes. PMID: 19388975
Diabetes Care. 2011;34(3):622-627. PMID: 21273495 16. Bacon S, Kyithar MP, Rizvi SR, et al. Successful maintenance on
8. Johnson SR, Carter HE, Leo P, et al. Cost-effectiveness analysis of routine sulphonylurea therapy and low diabetes complications rates in a HNF1A-
screening using massively parallel sequencing for maturity-onset diabetes MODY cohort. Diabet Med. 2016;33(7):976-984. PMID: 26479152
of the young in a pediatric diabetes cohort: reduced health system costs and 17. Clissold RL, Hamilton AJ, Hattersley AT, Ellard S, Bingham C. HNF1B-
improved patient quality of life. Diabetes Care. 2019;42(1):69-76. PMID: associated renal and extra-renal disease-an expanding clinical spectrum. Nat
30523035 Rev Nephrol. 2015;11(2):102-112. PMID: 25536396

148  ENDO 2021 • Endocrine Case Management

chased by Dr. Khalid H. Seedahmed, Jr., MRCP PGDip, CertEndocrinology SA - ID 283


Using Clinical Trial Results
to Optimize Patient Care:
Cardiovascular Outcomes
in Patients With Type 2
Diabetes Mellitus Treated
With GLP-1 Receptor Agonists
and SGLT-2 Inhibitors
Jennifer B. Green, MD. Department of Medicine, Division of Endocrinology, Duke University
Medical Center, Durham, NC; E-mail: jennifer.green@duke.edu

Learning Objectives atherosclerotic cardiovascular disease (ASCVD)


or multiple ASCVD risk factors. Findings from
As a result of participating in this session, learners
the outcomes trials conducted thus far suggest that
should be able to:
several GLP-1 receptor agonists primarily reduce
• Identify patients with type 2 diabetes mellitus the risk of atherosclerotic type complications
(T2DM) who have indications for GLP- and progression of albuminuria, while SGLT-2
1 receptor agonist or SGLT-2 inhibitor inhibitors consistently reduce the risks of heart
therapy to reduce the risk of cardio-renal failure and important renal outcomes. Trials
complications. of SGLT-2 inhibitor therapy in patients with
diabetes and chronic kidney disease and in
• Plan for modifications in already complex
patients with heart failure with reduced ejection
regimens of care to permit incorporation of
fraction with or without diabetes, have shown
SGLT-2 inhibitors or GLP-1 receptor agonists.
significant outcomes benefit in those patient
populations as well. These newer agents should
be preferentially used in patients at high risk for
cardio-renal complications, including those with
Main Conclusions established major comorbidities, likely irrespective
Patients with T2DM are at high risk for major of the need for additional glucose lowering or
cardio-renal complications. When used in addition preexisting antihyperglycemic therapy. In patients
to traditional modalities of cardiovascular risk who have already achieved an individually
reduction, numerous agents in the GLP-1 receptor appropriate degree of glycemic control, alteration
agonist and SGLT-2 inhibitor classes have been of background antihyperglycemic therapy may
shown to significantly reduce rates of important be indicated to reduce the risk of hypoglycemia
adverse outcomes in patients with T2DM and and/or mitigate regimen expense. In addition,

ENDO 2021 • Diabetes Mellitus and Glucose Metabolism  149

chased by Dr. Khalid H. Seedahmed, Jr., MRCP PGDip, CertEndocrinology SA - ID 283


anticipatory modification of diuretic or other Thus, preferential use of these medications
antihypertensive therapy may be needed to avoid is recommended for such patients, and may
significant volume depletion or hypotension with help to address the residual cardiovascular risk
the addition of an SGLT-2 inhibitor. that still affects those with diabetes. However,
implementation of these therapies has thus far
been quite limited, particularly in the populations
of patients most likely to derive an outcomes
Significance of the
benefit.8 There are many potential barriers to
Clinical Problem provider implementation of these guideline-
Although rates of morbidity and mortality in based practices into diabetes care, including lack
persons with diabetes have decreased over of familiarity with the rapidly emerging body of
time, individuals with diabetes still have an cardiovascular outcomes trials data. In addition,
approximately 2-fold excess risk of cardiovascular concerns about potential adverse effects of newer
outcomes (including coronary heart disease, drug classes and how they may best fit into
stroke, and vascular death) compared with existing complex medication regimens may pose
outcomes of persons without diabetes.1,2 This additional obstacles to optimization of care.
excess risk of vascular events in persons with
diabetes appears higher in younger individuals
and in women, and in those with a longer
Barriers to Optimal Practice
duration of diabetes and established microvascular
• Complexity of emerging data from
complications such as kidney disease. In addition,
cardiovascular outcomes trials and
diabetes increases the risk of heart failure by up
understanding applicability to individual at-
to 5-fold, and diabetes is strongly associated with
risk patients.
adverse outcomes in patients with heart failure,
particularly heart failure with reduced ejection • Difficulty in assessing the balance of
fraction.3,4 Intensive multifactorial therapy to overall benefits and risks with use of newer
manage cardiovascular risk factors is effective in antihyperglycemic agents in patients with
reducing the risk of vascular events in patients diabetes-related comorbidities.
with T2DM.5 However, progress in the control of • Challenges in introducing newer, beneficial
traditional cardiovascular risk factors is slowing. It antihyperglycemic agents into an already
is estimated that 33% to 49% of patients still do not complex medication regimen.
meet recommended targets for the management
of hemoglobin A1c, blood pressure, or lipids. In
fact, only 14% meet targets for glycemic and blood Strategies for Diagnosis,
pressure control and lipid management and do not
Therapy, and/or Management
smoke cigarettes.6
Findings from many recent large outcomes In 2008, the US FDA issued a guidance
trials designed to assess the cardiovascular effects recommending thorough evaluation of all new
of newer classes of diabetes medications have diabetes therapies for cardiovascular safety. Since
further altered recommendations for the care of that time, numerous cardiovascular outcomes
persons with T2DM. Numerous agents in the trials of drugs in the DPP-4 inhibitor, GLP-1
GLP-1 receptor agonist and SGLT-2 inhibitor receptor agonist, and SGLT-2 inhibitor classes
classes reduce the risk of important cardiovascular have been completed. Most of these trials were
and renal complications when included as part of designed to assess the cardiovascular impact of
usual care in patients with T2DM and established a particular new diabetes medication compared
ASCVD or multiple risk factors for ASCVD.4,7 with placebo when added to the otherwise usual

150  ENDO 2021 • Endocrine Case Management

chased by Dr. Khalid H. Seedahmed, Jr., MRCP PGDip, CertEndocrinology SA - ID 283


care of patients with T2DM. These trials primarily Cardiovascular Effects of
enrolled patients with established ASCVD Newer Medication Classes
(including coronary, cerebrovascular, or peripheral
vascular disease) or patients who were otherwise DPP-4 Inhibitors
at high risk for major adverse cardiovascular Trials of the DPP-4 inhibitor medications
events (MACE) such as cardiovascular death, completed thus far have consistently demonstrated
myocardial infarction, and stroke. The findings a neutral effect of these drugs on rates of MACE.
of these trials, including the effects of the studied Most of the DPP-4 inhibitors have also had a
medications on risks of MACE and hospitalization neutral effect on rates of hospitalization for heart
for heart failure, are summarized in the tables failure. However, the SAVOR TIMI-53 trial of
(Table 1 and Table 2). saxagliptin therapy unexpectedly found an increase
in hospitalization for heart failure in patients
assigned to that medication when compared
with placebo. This increase in risk has not been
explained or reproduced in other trials, but it
should be carefully considered when managing
patients with diabetes and existing heart failure or

Table 1. Effects of Newer Diabetes Medications: Major Adverse Cardiovascular Events

SAVOR-TMI-53 EXAMINE TECOS CARMELINA


Drug Class saxagliptin alogliptin sitagliptin linagliptin

DPP-4 inhibitor Neutral Neutral Neutral Neutral

LEADER ELIXA SUSTAIN-6 EXSCEL REWIND


linagliptin lixisenatide semaglutide injection exenatide once weekly dulaglutide

GLP-1 receptor Beneficial Neutral Beneficial Neutral Beneficial


agonist

EMPA-REG CANVAS DECLARE


empagliflozin canagliflozin dapagliflozin

SLGT-2 inhibitor Beneficial Beneficial Neutral*

MACE = Major Adverse Cardiovascular Events: CV death, MI, stroke


*One of two primary composite endpoints

Table 2. Effects of Newer Diabetes Medications: Heart Failure

SAVOR-TMI-53 EXAMINE TECOS CARMELINA


Drug Class saxagliptin alogliptin sitagliptin linagliptin

DPP-4 inhibitor Increased Risk Neutral Neutral Neutral

LEADER ELIXA SUSTAIN-6 EXSCEL REWIND


linagliptin lixisenatide semaglutide injection exenatide once weekly dulaglutide

GLP-1 receptor Neutral Neutral Neutral Neutral Neutral


agonist

EMPA-REG CANVAS DECLARE DAPA HF*


empagliflozin canagliflozin dapagliflozin dapagliflozin

SLGT-2 inhibitor Beneficial Beneficial Beneficial Beneficial

All trials listed other than DAPA-HF enrolled patients with type 2 diabetes and established atherosclerotic CV disease, or multiple risk factors for the same.
HF endpoints in most trials were hospitalizations due to heart failure. *DAPA-HF enrolled patients with HFrEF with or without diabetes. Primary outcome was
worsening HF event or cardiovascular death (worsening HF event = unplanned HF hospitalization or an urgent heart failure visit requiring intravenous therapy).

ENDO 2021 • Diabetes Mellitus and Glucose Metabolism  151

chased by Dr. Khalid H. Seedahmed, Jr., MRCP PGDip, CertEndocrinology SA - ID 283


high heart failure risk. DPP-4 inhibitors, although with reduced ejection fraction, with or without
largely found to be safe from a cardiovascular diabetes.9 In that trial, dapagliflozin therapy
perspective, should not be used to reduce significantly reduced the risk of worsening heart
cardiovascular risk in patients with T2DM. failure or death from cardiovascular causes
compared with placebo.
GLP-1 Receptor Agonists In the United States, empagliflozin is indicated
The GLP-1 receptor agonist class is quite diverse to reduce the risk of cardiovascular death in adult
with respect to molecular structure, duration patients with T2DM and established ASCVD;
of action, and potency. Thus, it is perhaps canagliflozin is indicated to reduce the risk of
unsurprising that the findings from cardiovascular MACE in the same population; and dapagliflozin
outcomes trials of these medications have is indicated to reduce the risk of hospitalization for
differed. These trials have shown that liraglutide, heart failure in adults with T2DM and established
semaglutide, and dulaglutide all reduce the ASCVD or multiple cardiovascular risk factors.
risk of MACE when compared with placebo in
patients with T2DM at high risk for such events.
Renal Effects of GLP-1 Receptor
Of note, the REWIND trial of dulaglutide also
enrolled patients at high risk, but in contrast to Agonists and SGLT-2 Inhibitors
the other cardiovascular outcomes trials, most Although not the primary outcomes of interest,
of the patients included had not had a prior the effects of the newer drug classes on a variety
cardiovascular event. A MACE benefit was also of renal outcomes were also analyzed in most
seen in the cardiovascular outcomes trial of of the cardiovascular outcomes trials completed
albiglutide; however, that medication is no longer to date. Meta-analyses of these data suggest that
available for use. As shown in the summary table, the renal outcomes benefit noted in the trials
the trials of GLP-1 receptor agonists have found of GLP-1 receptor agonists were primarily
a neutral effect of these agents on rates of heart attributable to a reduction in macroalbuminuria,
failure hospitalization. Of these agents, thus far in while SGLT-2 inhibitors were found to reduce
the United States liraglutide is indicated to reduce the risks of worsening estimated glomerular
the risk of MACE in adults with T2DM and filtration rate, end-stage kidney disease, or death
established ASCVD. due to renal causes.10 The CREDENCE trial
has since specifically evaluated the effects of
SGLT-2 Inhibitors SGLT-2 inhibitors in patients with T2DM and
Trials of agents in the SGLT-2 inhibitor class albuminuric chronic kidney disease.11 In that trial,
have had more consistent outcomes results. The canagliflozin, 100 mg daily, significantly reduced
cardiovascular outcomes trials of empagliflozin the risk of kidney failure and cardiovascular events
and canagliflozin demonstrated a significant at 2.62 years compared with placebo. As a result,
reduction in MACE with use of those agents. canagliflozin is now also indicated to reduce
The DECLARE trial did not find a reduction in the risk of end-stage kidney disease, doubling
MACE risk with dapagliflozin; however, a second of serum creatinine, cardiovascular death, and
primary heart failure outcome in that trial was hospitalization for heart failure in adults with
significantly reduced with active therapy compared T2DM and diabetic nephropathy with albuminuria
with placebo. All of the completed cardiovascular and an estimated glomerular filtration rate of
outcomes trials of SGLT-2 inhibitors have shown 30 mL/min per 1.73 m2 or higher.
a significant reduction in heart failure outcomes
with use of those agents. Most recently, the
DAPA heart failure trial evaluated the effects
of dapagliflozin in patients with heart failure

152  ENDO 2021 • Endocrine Case Management

chased by Dr. Khalid H. Seedahmed, Jr., MRCP PGDip, CertEndocrinology SA - ID 283


GLP-1 Receptor Agonists and SGLT- particularly effective in improving outcomes in
2 Inhibitors Outcomes Summary patients with T2DM and chronic kidney disease or
heart failure with reduced ejection fraction.
Meta-analyses of the cardiovascular outcomes
trials data suggest the following summary of drug
class effects: Therapeutic Issues
GLP-1 Receptor Agonists
1. Both GLP-1 receptor agonists and SGLT-2
The agents in this class with demonstrated
inhibitors decrease risk of MACE in patients
cardiovascular outcomes benefit are delivered via
with established ASCVD.
subcutaneous injection. Primary contraindications
a. In REWIND, the GLP-1 receptor agonist to use of these medications include a personal
dulaglutide reduced risk of MACE in patients or family history of medullary thyroid cancer
without a prior cardiovascular event. or multiple endocrine neoplasia type 2. Other
2. SGLT-2 inhibitors reduce hospitalization for potential contraindications include history of
heart failure and progression of kidney disease pancreatitis or bariatric surgery. Worsening of
in patients with established ASCVD or who retinopathy has been noted with semaglutide use.
are at high cardiovascular risk.10,12 These agents may cause nausea and vomiting,
particularly with dose initiation or up-titration.
The cardiovascular benefits of GLP-1 receptor Cautious dosage adjustment may be prudent.
agonist therapy may be due to beneficial
antiatherogenic effects of the medications SGLT-2 Inhibitors
mediated by improvements in cardiovascular These agents have a diuretic effect that may cause
risk factors such as weight and blood pressure. volume depletion or hypotension. Increased urinary
The mechanisms of benefit provided by SGLT-2 glucose excretion increases the risk of genital
inhibitors are likely different and may be in part mycotic infections, particularly in women, and
attributable to favorable hemodynamic and other some SGLT-2 inhibitor users have experienced
effects induced by glycosuria and natriuresis. Fournier gangrene. Good genitourinary/perineal
The cardiovascular benefits identified in the hygiene should be encouraged. An increased
cardiovascular outcomes trials do not appear amputation risk was seen with canagliflozin therapy
attributable to or dependent on glucose lowering. in the CANVAS trial, but not in CREDENCE,
In applying this information to clinical which implemented a rigorous strategy of foot care.
practice, it is important to recognize the variable Diabetic ketoacidosis with lower than expected
outcomes noted in cardiovascular outcomes trials blood glucose elevation may occur with use of
of the individual diabetes medications. Current any SGLT-2 inhibitor. Thus, the drug should be
diabetes care guidelines note that drugs with discontinued during significant illness, dehydration,
proven outcomes benefit should be specifically major procedures, or hospitalization.
selected when used to reduce cardiovascular risk
in patients with T2DM.4,7 Some guidelines suggest
preferential use of GLP-1 receptor agonists in
Strategies for Incorporation
patients with ASCVD, with SGLT-2 inhibitors as of Cardioprotective
a second choice if GLP-1 receptor agonist therapy Diabetes Medications
is not feasible or tolerated. However, there are no Patients with T2DM and high cardiovascular risk
head-to-head trials comparing the effects of the 2 are often already taking highly complex regimens
drug classes in this patient population; thus, choice of medication. This may make the addition of
of a proven agent from either class is reasonable, new agents with cardiovascular outcomes benefit
based on individual patient characteristics and particularly challenging. In addition to the
preferences. SGLT-2 inhibitors also appear above therapeutic concerns, the patient’s degree

ENDO 2021 • Diabetes Mellitus and Glucose Metabolism  153

chased by Dr. Khalid H. Seedahmed, Jr., MRCP PGDip, CertEndocrinology SA - ID 283


of glycemic control and risk of hypoglycemia This patient with T2DM has established ASCVD
should be considered before adding either a and indications for a medication to reduce her risk
GLP-1 receptor agonist or SGLT-2 inhibitor. In of future cardiovascular events (thus, Answer A
addition, if an SGLT-2 inhibitor is chosen, the is incorrect). She is currently on no medications
patient’s blood pressure and volume status should to manage glycemia and has a hemoglobin A1c
be carefully assessed.13 Adjustments to baseline level only slightly above 7% (53 mmol/mol).
medications should be considered as outlined in Although the cardiovascular outcomes trials of
the Figure. newer medications enrolled patients who were
often using metformin (Answer B) as background
Figure. Addition of Cardioprotective therapy, there is no evidence to suggest that the
Diabetes Medication beneficial effects of SGLT-2 inhibitors or GLP-
1 receptor agonists are restricted to metformin
Patient taking insulin or Patient taking diuretic or
sulfonylurea other antihypertensive users, or to patients who have hemoglobin A1c
values above a certain threshold. Although some

HbA1c at or near target, or

Blood pressure at or near guidelines still suggest that metformin therapy
hypoglycemia target, or volume depletion
should always be included as initial therapy for
• T2DM, metformin has no demonstrated efficacy

'-----------JI [.._____
Substitute SGLT2i or GLP1-RA
for sulfonylurea, or reduce
insulin dose
Substitute SGLT2i for diuretic,
or reduce other
antihypertensive therapy
in the reduction of cardiovascular risk in patients
with established ASCVD. Thus, the initiation of
empagliflozin (Answer C) would be the preferred
next step for this patient. Should additional glucose
lowering be needed in the future, metformin could
Clinical Case Vignettes be added to empagliflozin therapy. As this older
Case 1 patient is essentially normotensive on a regimen
that includes the diuretic hydrochlorothiazide,
A 76-year-old woman with T2DM complicated
that medication should be discontinued (Answer
by hyperlipidemia, hypertension, and coronary
D) when empagliflozin is started, and the patient’s
artery disease requiring stent placement is
volume status and blood pressure should be
seen for evaluation. Her current medications
reassessed at follow-up. A GLP-1 receptor agonist
include aspirin, clopidogrel, lisinopril,
might also be a reasonable consideration for this
hydrochlorothiazide, and atorvastatin. Her BMI is
patient to reduce her risk of future MACE.
33 kg/m2; blood pressure is 124/80 mm Hg; and
she appears euvolemic. Her hemoglobin A1c level
is 7.2% (55 mmol/mol) and estimated glomerular Case 2
filtration rate is 56 mL/min per 1.73 m2. A 57-year-old man with T2DM, peripheral
vascular disease, coronary artery disease,
Which of the following change(s) chronic kidney disease, obesity, hyperlipidemia,
to her medication regimen would and hypertension presents for evaluation. His
be recommended at this time? current medications include sitagliptin; insulin
A. No change glargine, 10 units once daily; aspirin; losartan;
B. Start metformin furosemide; and rosuvastatin. His BMI is
C. Start empagliflozin 40 kg/m2, and blood pressure is 144/86 mm Hg.
His hemoglobin A1c level is 8.4% (68 mmol/mol),
D. Discontinue hydrochlorothiazide
and estimated glomerular filtration rate is
Answer: C and D) Start empagliflozin 28 mL/min per 1.73 m2. He has had no problems
and discontinue hydrochlorothiazide with hypoglycemia on his current regimen.

154  ENDO 2021 • Endocrine Case Management

chased by Dr. Khalid H. Seedahmed, Jr., MRCP PGDip, CertEndocrinology SA - ID 283


Which of the following change(s) Case 3  New Content 
to his medication regimen would
A 67-year-old woman with insulin-treated T2DM
be recommended at this time?
complicated by nonproliferative retinopathy,
A. No change hypertension, chronic kidney disease, and heart
B. Add empagliflozin failure with preserved ejection fraction is seen
C. Add lixisenatide for evaluation. Her current medications include
D. Add liraglutide insulin glargine, 20 units once daily; insulin aspart,
5 units 3 times daily with meals; atorvastatin;
E. Substitute liraglutide for sitagliptin
furosemide; atenolol; and lisinopril.
Answer: E) Substitute liraglutide for sitagliptin On physical examination, her BMI is 37 kg/m2
and blood pressure is 126/68 mm Hg. She does not
This patient with T2DM and multiple appear to have significant volume overload.
manifestations of ASCVD has indications
for a medication to further reduce his risk Laboratory test results:
of cardiovascular events (thus, Answer A is Hemoglobin A1c = 8.2% (4.0%-5.6%) (66 mmol/mol
incorrect). His hemoglobin A1c is not well [20-38 mmol/mol])
controlled, even in light of his established Estimated glomerular filtration rate = 32.5 mL/min
comorbidities. He is on a low insulin dosage but per 1.73 m2 (>60 mL/min per 1.73 m2)
has not had issues with hypoglycemia. He is also Urine albumin-to-creatinine ratio = 840 mg/g
(<30 mg/g creat)
taking the DPP-4 inhibitor sitagliptin, which does
not increase, but also does not reduce, his risk of She does not report recent problems with
cardiovascular events. Empagliflozin (Answer B) hypoglycemia.
is not currently the correct choice for this patient
given his low estimated glomerular filtration rate, Which of the following changes
as this medication is at present only indicated to her medication regimen would
for use in patients with an estimated glomerular you recommend now?
filtration rate of 45 mL/min per 1.73 m2 or higher. A. No change
Although canagliflozin may be considered for B. Add linagliptin, 5 mg daily
use in patients with lower estimated glomerular
filtration rate values, this patient’s current C. Add low-dosage metformin (≤1000 mg daily)
estimated glomerular filtration rate is too low for D. Add canagliflozin, 100 mg daily
use of that drug as well. Therefore, introduction of E. Substitute dulaglutide for mealtime insulin
a GLP-1 receptor agonist would be an appropriate aspart doses
choice. Lixisenatide (Answer C) was not found
to reduce cardiovascular risk in the ELIXA trial, Answer: D) Add canagliflozin, 100 mg daily
so liraglutide is the appropriate choice for this This patient with insulin-treated T2DM has
patient. Careful use with slow up-titration and established chronic kidney disease with severe
monitoring for tolerability is needed in patients albuminuria, and is thus at high risk for significant
with significantly impaired renal function. This cardiovascular complications and progression
patient’s sitagliptin should be discontinued of renal dysfunction. She is currently treated
when liraglutide is started, as combined use of a with insulin for glycemic control, and she has
GLP-1 receptor agonist and a DPP-4 inhibitor a hemoglobin A1c level that is probably slightly
is not recommended (thus, Answer E is correct above the optimal range for someone of her age
and Answer D is incorrect). In addition, as both and with her established comorbidities. Some
medications may be expensive, this will minimize intensification of glycemic control could be
the cost associated with his diabetes care regimen. attempted if achievable without causing significant

ENDO 2021 • Diabetes Mellitus and Glucose Metabolism  155

chased by Dr. Khalid H. Seedahmed, Jr., MRCP PGDip, CertEndocrinology SA - ID 283


hypoglycemia. However, even if her glycemic Laboratory test results:
control were at goal, incorporation of a medication
Hemoglobin A1c = 5.9% (4.0%-5.6%) (41 mmol/mol
to attenuate the risk of cardio-renal complications [20-38 mmol/mol])
should be seriously considered as part of her Estimated glomerular filtration rate = 46 mL/min per
treatment regimen. Therefore, no change in 1.73 m2 (>60 mL/min per 1.73 m2)
treatment (Answer A) is not the best next step. B-type natriuretic peptide = 944 ng/L (<125 ng/L)
Linagliptin (Answer B) may be used in patients
with significant renal dysfunction, and it might Which of the following interventions
modestly improve her glycemic control, but it would you recommend now?
would not be expected to provide important
A. Engage in intensive lifestyle modification
outcomes benefits beyond that. The addition of
metformin (Answer C) might help with glycemic B. Begin dapagliflozin, 10 mg daily
control, but the addition of even low-dosage C. Begin liraglutide with gradual up-titration to
metformin is not recommended in patients such 3 mg daily
as this with an estimated glomerular filtration D. Begin metformin, 500 mg twice daily
rate between 30 and 44 mL/min per 1.73 m2.
Canagliflozin therapy (Answer D) has been shown Answer: B) Begin dapagliflozin, 10 mg daily
to significantly reduce the risk of progression to This patient with obesity, hypertension, and
end-stage kidney disease, doubling of the serum prediabetes has significant complications,
creatinine level, death from cardiovascular causes, including coronary disease and heart failure with
and hospitalization for heart failure in patients reduced ejection fraction. Both intensive lifestyle
with T2DM and diabetic nephropathy with modification (Answer A) and metformin therapy
albuminuria who have an estimated glomerular (Answer D) have been shown to reduce the risk
filtration rate of 30 mL/min per 1.73 m2 or of progression from prediabetes to diabetes and
greater. Therefore, this represents the preferred might be considered as options for this patient.
next step for this particular patient. If canagliflozin In addition, liraglutide at a dosage of 3 mg daily
is not tolerated, then use of a GLP-1 receptor (Answer C) may be used to promote weight
agonist such as dulaglutide (Answer E) could be loss in patients with obesity. However, based on
considered as an alternative option to reduce the the results of the DAPA-HF trial (Dapagliflozin
risk of major adverse cardiovascular events in this and Prevention of Adverse Outcomes in Heart
patient with multiple cardiovascular risk factors. Failure), this patient’s diagnosis of heart failure
with reduced ejection fraction makes the addition
Case 4  New Content  of dapagliflozin (Answer B) the preferred
A 56-year-old man with obesity, hypertension, next step. The DAPA-HF trial found that in
multivessel coronary artery disease, heart failure patients with heart failure with reduced ejection
with reduced ejection fraction, and prediabetes fraction with or without diabetes, dapagliflozin
presents for evaluation. His current medications significantly reduced the risk of worsening heart
include atorvastatin, torsemide, spironolactone, failure or death of cardiovascular causes. In
carvedilol, losartan, and clopidogrel. addition, although dapagliflozin is not indicated
On physical examination, his BMI is 38 kg/m2 for diabetes prevention, it reduced the risk of new-
and blood pressure is 128/85 mm Hg. onset diabetes in the DAPA-HF trial population.

156  ENDO 2021 • Endocrine Case Management

chased by Dr. Khalid H. Seedahmed, Jr., MRCP PGDip, CertEndocrinology SA - ID 283


References
1. Rawshani A, Rawshani A, Franzen S, et al. Mortality and cardiovascular 8. McCoy R, Dykhoff H, Sangaralingham L, Ross JS, Karaca-Mandic P, Montori
disease in type 1 and type 2 diabetes. N Engl J Med. 2017;376(15):1407-1418. VM, Shah ND. Adoption of new glucose-lowering medications in the U.S.-
PMID: 28402770 the case of SGLT2 inhibitors: nationwide cohort study. Diab Technol Ther.
2. Emerging Risk Factors Collaboration, Sarwar N, Gao P, et al. Diabetes 2019;21(12);702-712. PMID: 31418588
mellitus, fasting blood glucose concentration, and risk of vascular 9. McMurray JJV, Solomon SD, Inzucchi SE, et al; DAPA-HF Trial Committees
disease: a collaborative meta-analysis of 102 prospective studies. Lancet. and Investigators. Dapagliflozin in patients with heart failure and reduced
2010;375(9733):2215-2222. PMID: 20609967 ejection fraction. N Engl J Med. 2019;381(21):1995-2008. PMID: 31535829
3. Ghosh RK, Ghosh GC, Gupta M, et al. Sodium glucose co-transporter 2 10. Zelniker TA, Wiviott SD, Raz I, et al. Comparison of the effects of glucagon-
inhibitors and heart failure. Am J Cardiol. 2019;124(11):1790-1796. PMID: like peptide receptor agonists and sodium-glucose cotransporter 2 inhibitors
31627834 for prevention of major adverse cardiovascular and renal outcomes in type 2
4. Cosentino F, Grant PJ, Aboyans V, et al; ESC Scientific Document Group. diabetes mellitus. Circulation. 2019;139(17):2022-2031. PMID: 30786725
2019 ESC guidelines on diabetes, pre-diabetes, and cardiovascular diseases 11. Perkovic V, Jardine MJ, Neal B, et al; CREDENCE Trial Investigators.
developed in collaboration with the EASD. Eur Heart J. 2019;pii:ehz486. Canagliflozin and renal outcomes in type 2 diabetes and nephropathy. N Engl J
PMID: 31497854 Med. 2019;380(24):2295-2306. PMID: 30990260
5. Gaede P, Vedel P, Larsen N, Jensen GV, Parving HH, Pedersen O. 12. Zelniker TA, Wiviott SD, Raz I, et al. SGLT2 inhibitors for primary and
Multifactorial intervention and cardiovascular disease in patients with type 2 secondary prevention of cardiovascular and renal outcomes in type 2 diabetes:
diabetes. N Engl J Med. 2003;348(5):383-393. PMID: 12556541 a systematic review and meta-analysis of cardiovascular outcome trials. Lancet.
6. American Diabetes Association. Standards of medical care in diabetes. 1. 2019;393(10166):31-39. PMID: 30424892
Promoting health and reducing disparities in populations. Diabetes Care. 13. Gomez-Peralta F, Abreu C, Lecube A, et al. Practical approach to initiating
2017;40(Suppl 1):S6-S10. PMID: 27979888 SGLT2 inhibitors in type 2 Diabetes. Diabetes Ther. 2017;8(5):953-962. PMID:
7. American Diabetes Association. Cardiovascular disease and risk management: 28721687
standards of medical care in diabetes-2019. Diabetes Care. 2019;42(Suppl
1):S103-S123. PMID: 30559236

ENDO 2021 • Diabetes Mellitus and Glucose Metabolism  157

chased by Dr. Khalid H. Seedahmed, Jr., MRCP PGDip, CertEndocrinology SA - ID 283


Pharmacologic Approaches to
the Patient With Nonalcoholic
Fatty Liver Disease
Diana Barb, MD. Department of Medicine, Division of Endocrinology, Diabetes, and
Metabolism, University of Florida, Gainesville, FL; E-mail: diana.barb@medicine.ufl.edu

Learning Objectives and use of therapies proven to be effective in


randomized controlled trials.
As a result of participating in this session, learners
should be able to: • There are currently no FDA-approved
pharmacologic therapies for NAFLD or
• Recognize burden of disease and how to NASH. However, GLP-1 receptor agonists
diagnose nonalcoholic fatty liver disease and pioglitazone should be considered first-
(NAFLD) and nonalcoholic steatohepatitis line therapy in patients with NASH confirmed
(NASH). on liver biopsy and in patients with diabetes
• Describe optimal management of patients with and NAFLD/NASH. Vitamin E demonstrated
type 2 diabetes mellitus (T2DM) and NAFLD. efficacy in patients without diabetes, and
is only of marginal benefit in patients with
• Describe current therapeutic targets for
NASH and T2DM. GLP-1 receptor agonists
treating NAFLD/NASH.
promote weight loss and are promising
new agents for NASH that are undergoing
testing. SGLT-2 inhibitors also reduce
hepatic steatosis, but their ability to reverse
Main Conclusions steatohepatitis has not been explored.
• NAFLD is one of the silent, often • Recent progress in understanding the
unrecognized complications of obesity and pathophysiology of NASH has led to
T2DM. It is important to identify patients the development of a number of novel
with NAFLD at risk for disease progression. pharmacological agents that are in ongoing
clinical trials in an effort to develop safe and
• Mainstay of NAFLD treatment is weight
effective treatments.
loss and aggressive cardiovascular risk factor
management (ie, dyslipidemia, hypertension, • Finally, endocrinologists should become more
and glycemic control). involved in the management of NAFLD/
NASH given the significant overlap of NASH
• Patients with diabetes are considered a high-
with obesity and diabetes and the many clinical
risk population for this condition and should
implications of the disease.
be screened for the presence of NAFLD
and NASH even if concentrations of plasma
aminotransferases are normal. The presence of
NASH in a patient with T2DM should prompt
early treatment with lifestyle intervention

158  ENDO 2021 • Endocrine Case Management

chased by Dr. Khalid H. Seedahmed, Jr., MRCP PGDip, CertEndocrinology SA - ID 283


to cirrhosis and hepatocellular carcinoma.3 The
Significance of the global prevalence of NASH is estimated to be
3.5%,1 but this is likely grossly underestimated,
Clinical Problem as it is currently the second leading cause of liver
Despite remarkable progress in understanding the transplantation in the United States.3,6 More
pathogenesis of NAFLD/NASH and subsequently, importantly, NAFLD and NASH carry a high
in developing medications to target the disease, risk of diabetes and cardiovascular disease.3 The
currently there are no pharmacologic agents prevalence of diabetes is increased across fibrosis
approved specifically for this condition. Treatment stages in NASH anywhere from 21% in mild to
of NAFLD/NASH should include aggressive moderate fibrosis stages and up to 50% in NASH
management of hyperglycemia and cardiovascular cirrhosis.7 Conversely, and by mechanisms that are
risk factors. Agents tested with some success still incompletely understood, patients with T2DM
in patients with NASH without T2DM include are susceptible to more severe forms of NAFLD,
pioglitazone, liraglutide, and vitamin E. In patients have a 2- to 4-fold higher risk of progression
with T2DM and NASH, pioglitazone and GLP-1 from NAFLD to NASH, and a higher prevalence
receptor agonists (liraglutide and semaglutide) of cirrhosis and hepatocellular carcinoma due to
have been shown to significantly improve liver NASH.8,9 Presence of diabetes promotes NASH,
histology. This session focuses on available drugs and is one of the important factors associated with
that could potentially be useful for management advanced fibrosis (Figure 1).3 Prevalence of NASH
of patients with NAFLD/NASH. Many new in persons with T2DM is estimated at 37%, or
molecules targeting metabolic pathways, 10-fold higher than in the general population in
inflammation, or oxidative stress and fibrosis are a recent meta-analysis,2 with advanced fibrosis in
being studied, and understanding of the complex up to 17%.2 It is important to recognize that about
pathophysiology of NASH provides a rationale in half of obese patients with T2DM and normal
the future for using combination of drugs to treat concentrations of plasma aminotransferases
this condition. (AST/ALT levels ≤40 IU/L [≤0.67 µkat/L]) may
NAFLD is the most common chronic liver have NAFLD and about 28% may have NASH on
disease in developed countries. Its prevalence liver biopsy.10
depends on the population studied and diagnostic
methodology used. In a recent meta-analysis, Figure 1. Bidirectional relationship between NASH and
the global prevalence of NAFLD was estimated T2DM

to be 25% in the general population1 and up to


approximately 56% in patients with T2DM.2
However, due to a lack of reliable noninvasive
ifJ·)&l4---1~4ii=I
diagnostic tests, this prevalence of NAFLD is likely
T2DM is common in patients with proven NASH (~44%)
underestimated.1,3 When using a more sensitive
Presence of T2DM confers a higher risk of advanced
and specific imaging method, such as proton fibrosis (2- to 4-fold higher prevalence)8,9
magnetic resonance spectroscopy (1H-MRS), the NAFLD/NASH prevalence is ≥2-fold higher in T2DM vs
prevalence of NAFLD in the general population in patients without T2DM
has been reported to be as high as 34% and even Presence of NAFLD in T2DM increases cardiovascular
risk (promotes dyslipidemia, hyperinsulinemia, and
higher, or up to 70% in overweight or obese inflammation)8,9
adults with T2DM.4,5 NAFLD can progress from
simple steatosis to its more severe form, known
as NASH, and is characterized by the presence of Why should endocrinologists care about NAFLD?
inflammation and hepatocyte injury (ballooning Despite being the most common cause of liver
and hepatocyte necroinflammation). It can lead disease in patients with T2DM and estimated to

ENDO 2021 • Diabetes Mellitus and Glucose Metabolism  159

chased by Dr. Khalid H. Seedahmed, Jr., MRCP PGDip, CertEndocrinology SA - ID 283


be the number one reason for liver transplantation with NAFLD, with or without T2DM, include
by 2020, NAFLD/NASH is frequently overlooked poor patient education, needle anxiety, clinical
and commonly undiagnosed. Only a few affected inertia, and absence of time and resources.
patients receive a diagnosis of NASH or are ever
treated in clinic. The disease carries a greater
cardiovascular risk and unfortunately, when it Strategies for Diagnosis,
progresses to fibrosis, it can lead to cirrhosis, liver Therapy, and/or Management
failure, hepatocellular carcinoma, and premature
death. Although traditionally viewed as a disease Diagnosis of NAFLD and NASH
of hepatologists, NAFLD should become a major The cornerstone of medical management of
concern for endocrinologists and those caring for NAFLD and NASH remains early diagnosis and
patients with obesity and T2DM. This session will intervention, as treatment success is less likely
focus on available treatment options for patients in advanced stages of fibrosis.5 The diagnosis
with NAFLD and/or NASH. of NAFLD6,11 is based on the following: (1)
presence of hepatic steatosis in addition to (2)
Barriers to Optimal Practice lack of significant alcohol consumption (defined
as ongoing or recent alcohol consumption of
• Patients and clinicians rarely think of NAFLD/ >21 units/week for men and >14 units/week for
NASH as a potentially serious medical women), and (3) exclusion of other liver diseases.
condition. Hepatic steatosis is usually seen on radiologic
imaging (ie, liver ultrasonography, control
• The diagnosis of NAFLD and NASH is often
attenuation parameters, and CT or by the gold
missed due to a reliance on low-sensitivity
standard technique 1H-MRS) and is defined as
diagnostic tests (ie, plasma aminotransferase liver fat content above 5.56% when measured
measurements or liver ultrasonography); by 1H-MRS. In patients with T2DM, diagnosis
transient elastography and 1H-MRS are not should not solely rely on elevated plasma liver
being routinely used in clinical practice.
aminotransferases as up to 50% of patients with
• Definite diagnosis of NASH involves liver normal ALT levels (<40 U/L [<0.67 µkat/L]) may
biopsy, which has many limitations and it is have NAFLD. If available, vibration-controlled
rarely pursued by providers, even in patients transient elastography or magnetic resonance
who are at high risk of NASH. There is a need elastography can also be used to determine the
for better noninvasive tests in clinical practice. severity (stage) of fibrosis.
• More specific screening and treatment Noninvasive algorithms based on metabolic
recommendations are needed for patients and anthropometric parameters could be used
with NAFLD and T2DM, and there is a need in clinical practice to identify patients at risk for
for more pharmacologic agents with proven NAFLD (ie, fatty liver index) and advanced fibrosis
efficacy and favorable weight profile. (ie, NAFLD fibrosis score, FIB-4 index, AST-to-
platelet ratio index, and many others). However,
• Common “barriers” to using pioglitazone
these are usually less sensitive and specific than
more often in patients with NASH include
the “gold standard” of liver biopsy.4 Therefore,
unawareness about its efficacy (>50% have
current guidelines6 advise that liver biopsy is the
NASH resolution with pioglitazone), lack of
only way to diagnose NASH and that noninvasive
familiarity with prescribing the drug, and
techniques have not yet been fully validated for
concern and misinformation about weight gain
the diagnosis of this condition.4 In the future, as
or the risk of other adverse effects with its use.
better noninvasive diagnostic techniques develop,
“Barriers” to use of GLP-1 receptor agonists
it will become likely to shift toward noninvasive
or other agents for weight loss in patients

160  ENDO 2021 • Endocrine Case Management

chased by Dr. Khalid H. Seedahmed, Jr., MRCP PGDip, CertEndocrinology SA - ID 283


measures and not rely on liver biopsy before out other causes for liver disease, to assess for
treatment initiation for NASH. fibrosis. Another important category is patients
Although liver biopsy remains the gold with normal liver enzymes and a history of
standard for diagnosis of NASH, it has multiple longstanding uncontrolled diabetes (10 plus years
caveats: it is invasive, subject to interpretation in duration) or high triglycerides. Assessment of
errors, and difficult to apply to large populations.6,8 fibrosis can be estimated by applying a fibrosis
Definite NASH is determined on low- to medium- biomarker panel and/or if available, magnetic
power microscopy based on the presence of at least resonance elastography or vibration-controlled
grade 1 in each of the following 3 components: transient elastography (VCTE), which is a simple
(1) steatosis (5%-33% of parenchyma is involved and noninvasive test (similar to ultrasonography),
or grade 1, >33%-66% for grade 2, and >66% for validated to measure liver fat and estimate for the
grade 3), (2) lobular inflammation (<2 foci are presence of fibrosis. A combination or sequential
present per 200x field for grade 1, 2 to 4 foci for use of tests (eg, liver enzymes and VCTE)
grade 2, and >4 foci per 200x field are present will yield a higher positive predictive value in
for grade 3) and hepatocellular ballooning (few identifying at-risk patients with active NASH and
balloons cells are present per high-power field fibrosis. Patients with high or intermediate risk for
for grade 1 and many ballooning cells present fibrosis may need referral to hepatology for a liver
for grade 2). NAFLD Activity Score is the sum biopsy. Bril and Cusi4 propose an algorithm for
of scores of steatosis (0-3), lobular inflammation management of patients with NAFLD and T2DM.
(0-3), and hepatocellular ballooning (0-2).
Strategies for Therapy and
Selecting Candidates Management of NAFLD and NASH
for Liver Biopsy Current guidelines advise that patients
In clinical practice, physicians have to decide with biopsy-proven NASH should receive
whom to screen for advanced fibrosis and when pharmacological treatment along with lifestyle
to recommend liver biopsy. Patients with high intervention.6 It is also important to recognize
risk of having NASH include those who are older that NAFLD—through increased lipotoxicity and
(>45 years), have higher BMI, have T2DM, glucotoxicity—carries an increased cardiovascular
and have an AST-to-ALT ratio of 1 or greater. disease risk,4 and treatment of NAFLD should
Patients with T2DM are at increased risk for include aggressive cardiovascular risk-factor
more severe forms of NASH, and this has been management (ie, weight, lipids, blood pressure,
recently acknowledged by the American Diabetes and treatment of hyperglycemia). Statins are safe
Association in the 2019 Standard of Care guidelines, to use in patients with NAFLD/NASH even if
recommending that “patients with T2DM or liver enzymes are elevated, and should be used as
prediabetes and elevated liver enzymes or fatty first-line therapy for lipid lowering.5 In a few small
liver on ultrasonography should be evaluated randomized controlled trials, statins improved
for the presence of NASH and liver fibrosis” steatosis; however, there was no significant
(Recommendation 4.14, page S40). There is no change in inflammation or fibrosis.12 Fibrates
further guidance on how to evaluate for NASH or and fish oil can be added as second-line therapy
who should be considered for a liver biopsy. One when indicated for treatment of dyslipidemia (ie,
simple algorithm that could be applied in clinical elevated triglycerides and low HDL cholesterol);
practice is to screen patients with obesity and/or however, they do not have direct effects on
T2DM and elevated liver enzymes, after ruling steatohepatitis.4,5

ENDO 2021 • Diabetes Mellitus and Glucose Metabolism  161

chased by Dr. Khalid H. Seedahmed, Jr., MRCP PGDip, CertEndocrinology SA - ID 283


Table. Therapeutic Agents Under Evaluation for the Management of NAFLD and NASH
and Their Effects on Liver Steatosis, Inflammation, and Fibrosis in Clinical Trials

Effects on

Class Drug Metabolic effect Steatosis Inflammation Fibrosis

Biguanides Metfomin28 ↓ Hepatic glucose output ↓H, I ↔ ↔

PPAR agonists Pioglitazone14-16 ↑ Adipose, hepatic and muscle insulin sensitivity ↓H, I ↓ ↓

PPARγ Saroglitazar43* ↓ Lipids ↓I N/A N/A


PPARα/γ ↑ Insulin sensitivity
PPARα/δ Elafibranor21* ↓ Hepatic lipogenesis ↓I ↔ ↔
PPARα/γ/δ ↓ ↓
Lanifibranor18,19* ↓ Hepatic lipogenesis ↓H
↑ Insulin sensitivity

DPP-4 inhibitors Sitagliptin29-31 ↑ Endogenous level of GLP-1 ↓↔ H, I ↓↔ ↔

Vildagliptin32 ↓I N/A N/A

GLP-1 receptor Exenatide26 ↑ Glucose-mediated insulin secretion ↓I N/A N/A


agonists
Liraglutide17,26,27 ↓ Gastric emptying and appetite ↓H, I ↓ ↓
↓ Body weight
Semaglutide20 ↓H ↓ ↔

SGLT-2 inhibitors Canagliflozin33 ↓ Renal glucose reabsortion ↓I N/A N/A

Empagliflozin36-37 ↓ Body weight ↓I N/A N/A


↑ Hepatic insulin sensitivity
Ipragliflozin35 ↓I N/A N/A
↑ Glucagon and lipolysis
Dapagliflozin38 ↓I N/A N/A

Antioxidants Vitamin E14,15,25 ↓ Lipid peroxidation ↓H, I ↓ ↔

↓ Repletion of glutathione
↓ Cytokine production

Lipid-lowering Fish oil5 ↓ VLDL-triglycerides and apolipoprotein B ↔H, I ↔ ↔


drugs production

Fibrates5 ↓ Hepatic secretion of VLDL ↔I ↔ ↔

Statins12 ↓ Cholesterol biosynthesis by HMG CoA ↔H, I ↔ ↔


reductase

Lipase inhibitor Orlistat39,40 ↓ Absorption of dietary triglycerides ↓↔ H, I ↔ ↔


↓ Body weight

PDE inhibitors Pentoxiphiline5 ↓ TNFα ↓H, I ↓ ↔

FXR receptor Obeticholic ↑ Hepatic insulin sensitivity ↓H, I ↓ ↓


agonist* acid41,42*
↓ Hepatic gluconeogenesis

FGF21 analogue BMS-98603645* ↑ Hepatic insulin sensitivity and promote ↓I N/A N/A
weight loss

FGF19 analogue NGM-28246* ↑ Insulin sensitivity, weight loss ↓I N/A N/A


↓ Bile acid synthesis

TRβ agonist Resmetirom13* ↑ Hepatic fat metabolism ↓H, I ↓ ↔


↓ Lipid peroxidation

Abbreviations: H, NAFLD assessed by histology; I, NAFLD assessed by imaging (ultrasonography, CT, 1H-MRS, or liver proton density fat fraction); PDE,
phosphodiesterase inhibitor; PPAR, peroxisome proliferator–activated receptors
* Investigational

162  ENDO 2021 • Endocrine Case Management

chased by Dr. Khalid H. Seedahmed, Jr., MRCP PGDip, CertEndocrinology SA - ID 283


Presently, there are no FDA-approved agents with Although many agents show significant
a specific indication for the treatment of NASH. reduction in steatosis (either by histology or
The Table summarizes current pharmacologic imaging [Table]), it should be noted that only a
approaches under evaluation in patients with few resulted in a significant proportion of patients
NAFLD and NASH and their metabolic effects and achieving resolution of NASH when compared with
histologic effects, when available, in randomized placebo (with pioglitazone, vitamine E, liraglutide,
controlled trials. The endpoints usually evaluated and semaglutide being available in clinic).
in pharmacotherapy randomized controlled The proportions achieving NASH resolution
clinical trials are: (1) improvement in liver reported in clinical trials are approximately 51%
histology (each histologic component [ie, steatosis, with pioglitazone (30 or 45 mg daily),14-16 39% with
inflammation, ballooning, and fibrosis]), ≥2-point liraglutide (1.8 mg daily),17 and 36% with vitamin
improvement in NAFLD Activity Score; and/ E (800 IU daily)15 (Figure 2). Treatment with
or (2) NASH resolution without worsening of lanifibranor, an investigational pan-PPAR agonist,
fibrosis; and (3) improvement in liver fibrosis resulted in NASH resolution proportions that
of 1 or more stages with no worsening of were similar to that with pioglitazone (~45% in the
steatohepatitis, with the last 2 being endorsed 1200 mg treatment group vs 19% in the placebo
by the FDA. Other surrogate imaging endpoints group).18,19 In addition, investigational
are improvement in liver fat by 1H-MRS, or semaglutide, administered as a daily injection,
liver proton density fat fraction (PDFF) and/or exhibited a dose-response proportion of patients
improvement in fibrosis by magnetic resonance achieving NASH resolution with no worsening of
elastography. A relative reduction in liver fat by fibrosis: ~40% in the low-dose group (0.1 mg), 59%
MRI-PDFF of 30% or more correlated with an in the 0.4-mg group, and 17% in the placebo group
improvement in clinical trials.13 (P <.001 for semaglutide 0.4 mg vs placebo).20

Figure 2. Proportion of Patients Achieving Resolution of NASH (Drug vs Placebo) for Pharmacologic Agents Assessed in
Randomized Controlled Trials

Proportions (%) achieving NASH resolution (without fibrosis worsening)

Semaglutlde
0.4mg

Pioglltazone
45mg

lanlfibranor
1200mg

Uragl utlde
1.8mg

Vitamin E
SOOIU
••
Resmetlron
80mg
••
Elaflbranor
120mg

0 10 20 30 40 50 60 70

Data are presented from the references: semaglutide trial;20 lanifibranor (NATIVE trial);18,19 pioglitazone trials;14,16 liraglutide (LEAN trial);17 vitamin E (PIVENS
trial);15 resmetirom;13 and elafibranor (GOLDEN505 trial).21

ENDO 2021 • Diabetes Mellitus and Glucose Metabolism  163

chased by Dr. Khalid H. Seedahmed, Jr., MRCP PGDip, CertEndocrinology SA - ID 283


The placebo subtracted treatment differences (a secondary outcome) vs 19% in the placebo
were 42% for semaglutide (0.4 mg daily dose),20 group, with a placebo substracted treatment
32% for pioglitazone,14 30% for liraglutide17 and difference of 32% points (confidence interval,
lanifibranor,19 and less than 20% for other drugs 13%-51%; P<.001).14 This histologic improvement
(~15% for vitamin E15). was closely correlated with improvement in
adipose tissue insulin resistance and increased
Pioglitazone adiponectin level.14 In patients with NASH but
Pioglitazone is a thiazolidinedione that modulates without T2DM, positive but less striking results
the transcription factor peroxisome proliferator– were also reported, possibly because a lower
activated receptor γ (PPAR-γ) (and to a lesser dosage of pioglitazone was used in these studies
extent PPAR-α) and affects positively insulin or because of differences in the populations
sensitivity in adipose tissue, liver, and skeletal studied.15,22 In the study by Cusi et al, a mean
muscle, resulting in improved glucose and lipid reduction in fibrosis stage was significant only for
metabolism and redistribution of excess ectopic pioglitazone at the higher dosage of 45 mg daily.13
triglyceride accumulation from the liver (and Belfort et al also reported a trend for changes in
other tissues) to a “metabolically healthier” fibrosis compared with baseline, and Aithal et al
adipose tissue. Pioglitazone has also been shown reported a reduction in fibrosis in patients without
to decrease cardiovascular- and stroke-related diabetes treated with pioglitazone, 30 mg daily.16,22
mortality.5 Belfort et al reported improvement Concerns related to pioglitazone use remain:
in hepatic steatosis and necroinflammation weight gain of approximately 3 kg on average,
with a reduction in score of 2 or greater after most likely related to expansion of body fat and
6 months of treatment in 55 patients with biopsy- less often from fluid retention, peripheral edema,
proven NASH and prediabetes or T2DM.16 The congestive heart failure when treatment is initated
reduction in steatosis and necroinflammation were in a patient with unknown diastolic disfunction or
proportionally higher in the pioglitazone arm vs history of heart failure, and bone loss, especially in
the placebo group: 43% vs 0% for steatosis and women.5,23 The association between pioglitazone
46% vs 14% for necroinflammation. The NAFLD and bladder cancer remains controversial, and was
Activity Score improved in 73% of patients with negative in a long-term study.21 In conclusion,
pioglitazone versus 24% on placebo with a trend pioglitazone leads to resolution of NASH (>50%)
for less fibrosis (but not statistically significant in in patients with and without T2DM, and long-
this study). A longer-term study in 101 patients term (3-year) data show improvement in NAFLD
with prediabetes or T2DM and NASH also showed Activity Score in two-thirds of patients (75%).
robust improvement in insulin sensitivity, as well Given safety and efficacy shown by pioglitazone
as in histology on sequential liver biopsies at 18 and in nonalcoholic steatohepatitis, combined with its
36 months and confirmed the safety and efficacy proven cardiovascular benefit and relatively low
of pioglitazone in this setting. More patients in the cost, pioglitazone should be a first-line treatment in
pioglitazone group (58%) achieved the primary patients with confirmed NASH on liver biopsy and
outcome of a 2-point reduction or greater in NAFLD in patients with diabetes and NAFLD, and could
Activity Score (in 2 categories) without worsening be used empirically in patients with T2DM and
of fibrosis compared with the placebo group (17%). elevated liver enzymes when other causes of liver
When the same analysis was limited to patients disease are excluded.
with definite NASH at baseline, 67% achieved the
primary outcome with pioglitazone vs 17% with Vitamin E
placebo (treatment difference of 50% points). In Vitamin E is an antioxidant and has been
this trial (as shown in the Figure 2), 51% of patients evaluated in 2 randomized controlled trials with
in the pioglitazone arm had NASH resolution reported liver histology outcomes in adults.15,25

164  ENDO 2021 • Endocrine Case Management

chased by Dr. Khalid H. Seedahmed, Jr., MRCP PGDip, CertEndocrinology SA - ID 283


In the PIVENS trial, adults with biopsy-proven evidence supporting its effectiveness become
NASH without diabetes received vitamin E at a available, vitamin E is not recommend treating
dosage of 800 IU daily vs placebo for 96 weeks.15 NASH in diabetic patients.”6 Vitamin E has not
Vitamin E treatment led to a higher proportion been studied in patients with liver cirrhosis. Long-
of patients achieving the primary histologic term safety concerns of vitamin E administration
endpoint of improvement in NAFLD Activity include increased risk for the development of
Score of 2 or more grades, with no worsening hemorrhagic stroke and potentially prostate
of fibrosis (43% vs 19%; P = .001). However, cancer.5
resolution of NASH only reached borderline
statistical significance (36% with vitamin E vs GLP-1 Receptor Agonists
21% with placebo; P = .05), an endpoint reached In 25 patients with T2DM and NAFLD, exenatide
in the same study by pioglitazone (47% with (n = 19) and liraglutide (n = 6) showed 42%
pioglitazone vs placebo; P = .001). A recent reduction in liver fat assessed by 1H-MRS.26 A
randomized controlled trial in patients with meta-analysis of the LEAD program also reported
T2DM randomly assigned to receive placebo, positive results with liraglutide improving liver
vitamin E alone, or vitamin E plus pioglitazone, enzymes and showing a positive trend for hepatic
showed that vitamin E alone did not significantly steatosis by CT after 26 weeks of treatment.27 In
change the primary histologic outcome of 2-point the LEAN trial, liraglutide 1.8 mg daily, given
or greater reduction in NAFLD Activity Score for 48 weeks showed improvement in histology
(31% vs 19%; P = .26). However, in the treatment in 52 patients with biopsy-proven NASH.17
arm with vitamin E alone, more patients achieved Resolution of NASH occurred in 39% of patients
NASH resolution (33% vs 12%; P = .04) when treated with liraglutide compared with 9% of
compared with placebo,25 and this improvement patients in the placebo group, and fewer patients
was similar to what was reported in the PIVENS on liraglutide experienced worsening of fibrosis.
trial in patients without diabetes, with an overall These effects on the liver are related to significant
placebo subtracted treatment effect of 21% to reduction of body weight and improved glycemic
24%.15 The resolution of NASH reported in the control in patients with T2DM (about one-third),
combination arm (vitamin E plus pioglitazone) as well as improved insulin sensitivity. The
was 43% vs 12% in the placebo group (P = .005)25 relatively high rates of resolution of NASH, despite
and similar to, not superior to, pioglitazone alone nonsignificant changes in individual histologic
(51% vs 19% in the placebo group) in a different scores, may be a result of the small study size, with
randomized controlled trial reported by the same only 45 patients having paired biopsies. Finally,
investigators.14 Again, in the study by Bril and the results of a phase 2 clinical trial evaluating the
Cusi, only steatosis assessed by histology improved efficacy of semaglutide (0.1, 0.2, or 0.4 mg daily
with vitamin E alone (P = .018); no improvement administration) on histologic endpoints (with
in inflammation (P = .018), ballooning (P = .022), paired biopsy) in 320 patients with NASH (62%
or fibrosis scores were observed.25 These findings with T2DM) and advanced fibrosis were just
will have to be reproduced in a larger trial in published.20 NASH resolution with no worsening
patients with T2DM in order to establish whether of fibrosis was seen in a dose-response fashion
there is a potential role for vitamin E alone or in with higher proportions of patients achieving
combination with pioglitazone in patients with NASH resolution with no worsening of fibrosis
T2DM. Finally, in a pediatric population with in the treatment group: 40% in the 0.1-mg group,
NASH, vitamin E did not significantly improve 36% in the 0.2-mg group, and 59% in the 0.4-mg
histology.25 In line with these findings, the 2018 group vs 17% in the placebo group (P <.001 for
guidance from the American Association for the semaglutide 0.4 mg vs placebo). An improvement
Study of Liver Diseases states that “until further in fibrosis stage was not statistically significant

ENDO 2021 • Diabetes Mellitus and Glucose Metabolism  165

chased by Dr. Khalid H. Seedahmed, Jr., MRCP PGDip, CertEndocrinology SA - ID 283


and occurred in 43% of patients in the 0.4-mg group showed that treatment with canagliflozin,
group and in 33% of patients in the placebo 300 mg daily, improved hepatic insulin sensitivity
group (P = .48). The 0.4-mg group had a mean and showed a trend to decrease intrahepatic
percentage weight loss of 13% (vs 1% in the triglyceride measured by proton magnetic
placebo group) and experienced a higher incidence resonance spectroscopy compared with placebo
of nausea, vomiting, constipation, and gallbladder- (–6.9% vs –3.8%, respectively, although this did not
related disorders.20 Certainly, larger studies with reach statistical significance), with the reduction
GLP-1 receptor agonists are needed to establish in intrahepatic triglyceride depending on the
their future role in the treatment of NASH. Novel magnitude of weight loss, which was greater
GLP-1 and glucagon receptor agonists are also in and more often with the SGLT-2 inhibitor.33
development and may be promising in the future Positive results with a decrease in liver fat were
by inducing weight loss of at least 10%. In general, observed with luseogliflozin,34 ipragliflozin,35
GLP-1 receptor agonists are well tolerated and the empagliflozin,36,37 and dapagliflozin.38 Of note,
gastrointestinal adverse effects can be minimized there are no randomized controlled trials assessing
with careful titration. Risk of pancreatitis and the effect of SGLT-2 inhibitors on liver histology.
pancreatic cancer with long-term use of these agents The favorable weight profile and beneficial effects
remains to be determined, and routine monitoring on reduction in liver fat make these agents an
of lipase in clinical practice is not recommended. attractive second- or third-line treatment and could
possibly be used in combination with pioglitazone.
Other Antidiabetes Drugs In general, SGLT-2 inhibitors are well tolerated
The effects of other antidiabetes drugs on and potential risks, such as volume depletion due
NASH histologic outcomes are summarized in to osmotic diuresis, genital mycotic and urinary
the Table. Although metformin remains first-line tract infections, bone loss, and euglycemic diabetes
therapy for the management of hyperglycemia in ketoacidosis must be carefully monitored and
T2DM and may reduce liver fat, it is not effective avoided by educating patients and providers.
for the treatment of steatohepatitis, as it does not
significantly impact resolution of NASH or fibrosis.28 Antiobesity Medications Other
It is considered neutral by current guidelines. Than GLP-1 Receptor Agonists
Sitagliptin was evaluated in small studies with A positive effect in improving steatohepatitis
imaging and histology. In patients with NAFLD would be expected with any of these agents when
and T2DM or prediabetes (n = 50), sitagliptin inducing an 8% to 10% reduction in body weight;
was no better than placebo in reducing liver fat however, most of these agents have not undergone
as assessed by MRI- proton density fat fraction29 rigorous testing in patients with NASH. In a
or in improving liver fibrosis in patients with recent meta-analysis39 including 3 randomized
biopsy-proved NASH (n = 12) after 24 weeks of controlled trials and 4 single-arm trials in 330
treatment.30 In an open-labeled study of 50 patients participants with NAFLD or NASH, orlistat, a
with NASH and paired liver biopsies after 1 year reversible gastric and pancreatic lipase inhibitor,
treatment, sitagliptin improved NAFLD activity reduced plasma aminotransferases in patients with
score, steatosis, and ballooning.31 Treatment with NAFLD, but it failed to improve liver histology
vildagliptin for 6 months also showed a modest in NASH.40 Although orlistat does not appear
decrease hepatic steatosis (from 7.3% to 5.3% to have drug-specific effects in steatohepatitis,
measured by 1H-MRS; normal being ≤5.5%).32 improvement in insulin sensitivity and liver
Finally, significant interest has recently been histology is proportional to the magnitude of
given to SGLT-2 inhibitors because of recent weight loss. When stratified according to weight
evidence on reduction in cardiovascular mortality loss, patients who lost 9% or more of body weight
with these agents. Specifically, in NAFLD, our (n = 16) had improved liver steatosis, ballooning,

166  ENDO 2021 • Endocrine Case Management

chased by Dr. Khalid H. Seedahmed, Jr., MRCP PGDip, CertEndocrinology SA - ID 283


and inflammation (P <.001) compared with those Combined PPAR Agonists: Elafibranor,
who did not lose that much body weight (n = 25). Saroglitazar, and Lanifibranor
The combination of phentermine/topiramate Elafibranor (GFT505), a dual agonist of PPAR-α
(maximal dosage 15 mg/92 mg) induced 8% to and PPAR-δ, with positive effects on lipid glucose
11% weight loss when compared with placebo at metabolism, has been evaluated in the GOLDEN
1 year in about 30% to 40% of patients.5 Patients 505 randomized controlled trial at 2 dosages
treated with naltrexone/bupropion achieved 5% to (80 mg daily [n = 93] and 120 mg daily [n = 91])
6% weight loss on average at 12 months. In about against placebo (n = 92) for 52 weeks in patients
20% of patients who achieved more meaningful with biopsy-proven NASH.21 Elafibranor failed to
weight loss of greater than 10%, a modest decrease meet the primary endpoint of resolution of NASH
in plasma aminotransferases was observed at 56 and showed histologic benefit in patients with a
weeks with the higher drug dosage (16 mg/180 mg higher NAFLD Activity Score (≥4) at the higher
twice daily).5 dosage of 120 mg compared with placebo (20%
vs 11% NASH resolution; P = .018 [see Figure 2]).
Investigational Drugs A larger phase 3 trial (RESOLVE-IT) is currently
A number of investigational drugs are currently evaluating elafibranor vs placebo in patients with
being pursued for an indication in NASH. A few of biopsy-confirmed NASH and NAFLD Activity
them are briefly reviewed in this session. Score of 4 or higher and fibrosis.
The efficacy of obeticholic acid, a synthetic Saroglitazar is PPAR-α and PPAR-γ agonist
farnesoid X receptor agonist, was studied in that is currently approved in India and Mexico for
patients with NASH in a multicenter randomized treatment of dyslipidemia and hypertriglyceridemia
controlled trial (FLINT) by Tetri and colleagues.41 in T2DM that is uncontrolled by statins. A phase 2
Although treatment with obeticholic acid, randomized controlled trial in NASH is currently
25 mg daily, improved steatosis, hepatocellular ongoing. In 221 patients with T2DM and NAFLD,
ballooning, and lobular inflammation in 46% 24 weeks of treatment with saroglitazar, 4 mg
vs 21% of patients on placebo (P = .0002), and daily, significantly improved liver enzymes and
reduced fibrosis in 35% vs 19% of patients on elastography findings along with lipids and
placebo (P = .004), disappointingly, resolution glycemia.43
of NASH was not achieved in a substantial Lanifibranor is a pan-PPAR-αγδ agonist
proportion of patients (only 22% of patients with antiinflammatory and antifibrotic effects
treated with obeticholic acid vs 13% in the placebo in preclinical models of NASH. In a large phase
group; P = .08). In a larger (n = 931) phase 3 2b trial in patients with NASH with or without
multicenter study (REGENERATE)42 enrolling diabetes (NATIVE),18,19 the drug demonstrated
patients with stage 2 and 3 fibrosis over 18 beneficial effects on several histologic endpoints,
months, once-daily obeticholic acid, 25 mg, met including NASH resolution and improvement of
the primary endpoint of fibrosis improvement fibrosis. After 24 weeks of treatment, resolution of
(≥1 stage) with no worsening of NASH in 23% NASH (without worsening fibrosis) was observed
of patients compared with 12% of patients in the in approximately 45% of patients treated with
placebo group (P = .0002). However, the study did 1200 mg daily of lanifibranor vs in 19% in the
not meet the NASH resolution primary endpoint placebo group (P <.01). In addition, improvement
at the 18-month interim analysis.42 Common in fibrosis by at least 1 point (without worsening
adverse effects reported included pruritus (one- of steatohepatitis) was also observed in 42% vs 24%
third of patients), increased LDL cholesterol, and in the placebo group (P = .011). Consistent with
decreased HDL cholesterol, which raises concerns the histologic data, treatment with lanifibranor
regarding long-term risk for cardiovascular safety resulted in a reduction in serum biomarkers of
and the use of this agent in the future. liver necroinflammation and fibrosis.

ENDO 2021 • Diabetes Mellitus and Glucose Metabolism  167

chased by Dr. Khalid H. Seedahmed, Jr., MRCP PGDip, CertEndocrinology SA - ID 283


MSDC-0602K is a second-generation oral (n = 74) resulted in significant reduction in hepatic
insulin sensitizer designed to selectively modulate fat (–37.3% vs –8.5%) when compared with
the mitochondrial pyruvate carrier while minimizing placebo (n = 34) at week 36. Resolution of NASH,
direct PPAR-γ activation. It was studied in the a secondary endpoint in this study, was achieved
12-month EMMINENCE trial.44 In this trial, 392 in 24.7% in the resmetirom group vs 6.5% in
patients with biopsy-confirmed NASH (NAFLD the placebo group, with a higher proportion in
Activity Score ≥4 and fibrosis score F1-F3) were those who achieved greater than 30% reduction
randomly assigned to daily oral doses of MSDC- in hepatic fat fraction. The number of patients
0602K (62.5 mg daily, 125 mg daily, or 250 mg achieving at least 1-point reduction in fibrosis
daily) or placebo. Treatment with MSDC-0602K did not differ between groups. More importantly,
improved insulin sensitivity and liver enzyme treatment with resmetirom resulted in clinically
levels and showed a dose-dependent trend toward significant reductions in multiple atherogenic lipid
improvement in liver histopathology, which did not, fractions, including small, dense LDL particles and
however, reach statistical significance, for the primary large VLDL or chylomicrons. A larger ongoing
histologic outcome at any dosage. A phase 3 trial phase 3 clinical trial, MAESTRO-NASH, exploring
may be planned with the higher dosage in the future. the role of resmetirom is ongoing.
BMS-986036, a pegylated recombinant Many other new compounds targeting
FGF21 analogue, was evaluated in one metabolic homeostasis, oxidative stress,
randomized controlled trial in 74 patients with inflammation apoptosis, or fibrosis, not discussed
NASH at 2 dosages (10 mg daily and 20 mg here, are being evaluated with some undergoing
weekly) vs placebo (randomized 1:1:1).45 After 16 phase 2B/3 randomized controlled trials.
weeks, liver fat by MRI proton density fat fraction In conclusion, it is important to screen patients
decreased by 6.8% with the 10 mg daily dosage and with prediabetes and diabetes and identify those
by 5.2% with the 20 mg weekly dosage vs 1.3% in “at risk” for developing NASH-induced liver
placebo (both P <.01). fibrosis. Patients should be counseled regarding
NGM282, a novel variant of FGF19, was effective therapies, including lifestyle changes and
evaluated in a phase 2 randomized controlled trial weight loss, and medications currently available to
(n = 82) at 2 dosages (3 mg daily and 6 mg daily treat diabetes and proven to be effective in NASH
for 12 weeks vs placebo) in patients with NAFLD (ie, GLP-1 receptor agonists and pioglitazone).
with baseline liver fat by MRI proton density fat
fraction ≥8%.46 NGM282 decreased ALT (43+%
and 45% with 3 mg daily and 6 mg daily dosage,
Clinical Case Vignettes
respectively) and liver fat by more than 5% in 79% Case 1
of participants, with 34% achieving normal liver fat A 69-year-old man with history of T2DM presents
(mean reduction in liver fat was –9.7% and –11.9% with right upper-quadrant discomfort. He is
with 3 mg daily and 6 mg daily dosage, respectively). overall healthy except for longstanding T2DM
The selective thyroid hormone receptor β (>10 years), hypertension, and dyslipidemia. He
(TRβ) agonist resmetirom (MGL-3196) has been takes metformin, 1000 mg twice daily; lisinopril,
shown in preclinical data to increase hepatic fat 40 mg daily; atorvastatin, 40 mg daily; and aspirin,
metabolism and reduce lipotoxicity and fibrosis 81 mg daily. His family history is notable for
markers. It is highly targeted towards liver TRβ T2DM in his mother and sister and prostate
to avoid the toxicities associated with systemic cancer in his father. Liver ultrasonography ordered
thyroid hormone exposure (largely mediated by his primary care physician to evaluate the
via TRα). In a 36-week trial in patients with right upper-quadrant discomfort shows hepatic
confirmed NASH (fibrosis stages 1–3) and hepatic steatosis. Physical examination findings are normal
fat fraction of at least 10%,13 resmetirom treatment except for a BMI of 35 kg/m2.

168  ENDO 2021 • Endocrine Case Management

chased by Dr. Khalid H. Seedahmed, Jr., MRCP PGDip, CertEndocrinology SA - ID 283


Laboratory test results: hormone replacement with estrogens. Thus,
pursuing all of the assessments listed (Answer E) is
Fasting glucose = 127 mg/dL (70-99 mg/dL)
(SI: 7.0 mmol/L [3.9-5.5 mmol/L]) correct.
Hemoglobin A1c = 7.5% (4.0%-5.6%) (58 mmol/mol
[20-38 mmol/mol])
ALT = 52 U/L (7-40 U/L) (SI: 0.87 µkat/L
Case 1 (Continued)
[0.12-0.67 µkat/L]) You discuss the laboratory findings with the
AST = 39 U/L (7-40 U/L) (SI: 0.65 µkat/L patient and inform him that he has a fatty liver.
[0.12-0.67 µkat/L])
Albumin = 3.5 g/dL (3.4-5.4 g/dL) (SI: 35 g/L
[34-54 g/L]) Which of the following statements
Platelet count = 132 x 103/µL (150-450 x 103/µL) regarding why NAFLD is associated
(SI: 132 x 109/L [150-450 x 109/L]) with a 2- to 3-fold risk of developing
LDL cholesterol = 110 mg/dL (<100 mg/dL hyperglycemia and T2DM is correct?
[optimal]) (SI: 2.85 mmol/L [<2.59 mmol/L])
HDL cholesterol = 40 mg/dL (>60 mg/dL [optimal])
A. Individuals with NAFLD are more insulin
(SI: 1.04 mmol/L [>1.55 mmol/L]) resistant at the level of muscle, liver, and
Triglycerides = 290 mg/dL (<150 mg/dL [optimal]) adipose tissue than individuals without
(SI: 3.28 mmol/L [<1.70 mmol/L]) NAFLD, even if they are lean and without
diabetes
Which of the following assessments are B. In NAFLD, insulin resistance is present in the
indicated to further evaluate the etiology muscle and liver but not in adipose tissue
of transaminitis in this patient? C. In NAFLD, insulin resistance is present in the
A. Review CAGE questionnaire liver and adipose tissue but not in muscle
B. Order viral and autoimmune hepatitis D. Nonobese individuals are not insulin resistant
serologies and rarely develop NAFLD
C. Review current medications E. Obese individuals with diabetes without
D. Review herbal supplements NAFLD are usually insulin sensitive (about
30% of all patients with diabetes)
E. All of the above
Answer: A) Individuals with NAFLD are more
Answer: E) All of the above
insulin resistant at the level of muscle, liver, and
The diagnosis of NAFLD requires exclusion adipose tissue than individuals without NAFLD,
of other competing causes that could lead to even if they are lean and without diabetes
elevation of liver enzymes and fatty infiltration
As reviewed by Gastaldelli et al,47 individuals
of the liver.6 Since accumulation of intrahepatic
with NAFLD are more insulin resistant than
triglycerides can be due to alcoholic liver disease,
those without NAFLD, even if they are lean and
excessive alcohol use must be ruled out. Significant
without diabetes (Answer A). In patients with
alcohol consumption is defined as ongoing or
NAFLD, the insulin resistance is present at the
recent alcohol consumption of greater than 21
level of muscle, liver, and adipose tissue, and as a
units per week for men and greater than 14
direct consequence, hepatic glucose production
units per week for women. In addition, other
and adipose tissue lipolysis are increased (less
liver diseases that should be investigated include
suppressed by insulin), thus resulting in higher
chronic viral hepatitis, hemochromatosis, α1-
fasting glucose and free fatty acid concentrations.
antitrypsin deficiency, Wilson disease, and
This leads to increased risk of T2DM in these
autoimmune hepatitis. Medications that can cause
patients. Bril et al11 have shown that liver insulin
hepatic steatosis should also be excluded such as
resistance, assessed as suppression of endogenous
corticosteroids, methotrexate, amiodarone, and

ENDO 2021 • Diabetes Mellitus and Glucose Metabolism  169

chased by Dr. Khalid H. Seedahmed, Jr., MRCP PGDip, CertEndocrinology SA - ID 283


glucose production during a hyperinsulinemic risk of liver fibrosis, so additional workup is and
euglycemic insulin clamp, already develops with referral to hepatology are needed (Answer D).
intrahepatic triglyceride accumulation of only NASH is associated with liver damage in the
2% to 3% and that this progressively worsens at form of hepatocellular injury with hepatocyte
higher levels of accumulation. However, nonobese ballooning/necrosis and predominantly lobular
individuals can also be insulin resistant, and inflammation. Hepatic fibrosis has 4 stages based
particularly if they have T2DM, they are often on its severity: stage F0 (no fibrosis), mild (stage
insulin resistant and develop NAFLD. Individuals F1), moderate (stage F2, with zone 3 sinusoidal
with diabetes without NAFLD are usually not fibrosis plus periportal fibrosis), or severe with
insulin sensitive (about 30% of all patients with bridging fibrosis (stage 3) or cirrhosis (stage 4).
diabetes), but they have less insulin resistance and Moderate to severe fibrosis (F2-F3) has been
a more favorable metabolic profile.48 linked to higher mortality.1,2 Advanced liver
fibrosis and cirrhosis occur more often in obesity,
but in particular, in patients with T2DM.6 NAFLD
Case 1 (Continued)
is also associated with a 2- to 3-fold increased risk
You use the Fibrosis-4 (FIB-4) calculator to of diabetes and of cardiovascular disease. FIB-4 has
estimate this patient’s risk of having liver fibrosis. proven useful in identifying patients at high risk
of advanced fibrosis (≥F3) fibrosis.4,50 Noninvasive
Which of the following is your conclusion? plasma tests are not as good to diagnose early
A. The FIB-4 cannot be calculated in this patient stages of fibrosis, but they have been useful for
B. He has a very low risk of liver fibrosis, so no the diagnosis of advanced disease (≥F3). In other
additional workup is needed words, they are best used to rule out severe disease
C. He has a low to moderate risk of liver fibrosis, (good specificity/negative predictive value), but
so no additional workup is needed they have poor sensitivity (positive predictive
value) to diagnose or monitor the disease.
D. He has a moderate to high risk of liver
fibrosis, so additional workup and referral to
hepatology are needed Case 1 (Continued)
E. If positive (>2.67), the FIB-4 is unreliable to You order elastography to further evaluate
rule advanced liver fibrosis in patients with NAFLD. This shows CAP (controlled attenuation
diabetes parameter) of 300 dB/m and VCTE (vibration-
controlled transient elastography) of 10 kPa. You
Answer: D) He has a moderate to high risk
advise that liver biopsy is indicated based on above
of liver fibrosis, so additional workup and
results and that additional therapy to improve both
referral to hepatology are needed
his diabetes and NAFLD should be considered.
Calculating FIB-4 can help clinicians in the
management of NASH by ruling out those at the What second-line therapy after metformin
highest risk of having advanced liver fibrosis and for management of hyperglycemia has
can be easily computed with available clinical been shown to result in a significant
and routine laboratory information such as age, proportion of patients achieving resolution
AST, ALT, and platelet count.44 Several websites of NASH when compared with placebo?
offer calculators to assess the risk of having liver A. Liraglutide
fibrosis. Examples of such sites are https://www. B. Dulaglutide
hepatitisc.uw.edu/page/clinical-calculators/fib-4 C. Empagliflozin
or https://www.mdcalc.com/fibrosis-4-fib-4-
D. Sitagliptin
index-liver-fibrosis. The calculated FIB-4 in this
case is 2.87, which indicates a moderate to high E. Pioglitazone

170  ENDO 2021 • Endocrine Case Management

chased by Dr. Khalid H. Seedahmed, Jr., MRCP PGDip, CertEndocrinology SA - ID 283


Answer: A and E) Liraglutide and pioglitazone C. He should continue atorvastatin due to known
effects of statins to improve NASH and reduce
In the LEAN trial, liraglutide, 1.8 mg daily, fibrosis in clinical studies
given for 48 weeks showed improvement in
D. He should stop atorvastatin because his liver
histology in 52 patients with biopsy-proven
enzymes are elevated and statins have no effect
NASH.17 Resolution of NASH occurred in 39%
on liver histology
of patients with liraglutide compared with 9%
of patients in the placebo group, and fewer Answer: A) He should continue atorvastatin
patients on liraglutide experienced worsening of because of increased cardiovascular disease
fibrosis. Kuchay et al36 and others37 have showed risk from T2DM and NAFLD
reduction in liver fat with empagliflozin and
other SGLT-2 inhibitors; however, there are no Statins are safe to use in patients with T2DM and
studies reporting on NASH histologic outcomes. NASH.12 Given the high cardiovascular risk profile
The effects of DPP-4 inhibitors on NAFLD are in these patients, statin therapy should be standard
modest and variable, but with an overall neutral of care. Most prospective studies with 10 or more
impact on histology (see Table). In patients with patients evaluating statin effects on liver histology
prediabetes or T2DM, Belfort et al16 reported in NASH failed to show significant histologic
improvement with a reduction in score of 2 or improvement (or change in aminotransferase
greater in hepatic steatosis and necroinflammation levels) when compared with placebo. In the placebo
in the pioglitazone arm vs patients in the placebo arm of the pioglitazone trial by Cusi,12 there were
group after 6 months of treatment in 55 patients no significant changes in liver histology observed
with biopsy-proven NASH. In a longer-term in patients with NASH newly started on a statin.
randomized controlled trial, Cusi et al14 showed
that more patients in the pioglitazone group (58%) Case 1 (Continued)
achieved the primary outcome of a 2 or more
point reduction in NAFLD Activity Score than in
The patient is hesitant to start either
the placebo group (17%). Resolution of NASH, a
pioglitazone or liraglutide and wants to
secondary outcome in this trial, occurred in 51% know if you would recommend treatment
of pioglitazone-treated patients vs in 19% of those with vitamin E. Regarding the use of vitamin
receiving placebo (treatment difference of 32% E in patients with T2DM and NASH, which
points P <.001). of the following statements is INCORRECT?
A. Vitamin E at a dosage of 800 IU daily should
Case 1 (Continued) be recommended given the known benefit of
vitamin E in NASH reported in the PIVENS
The patient is concerned about trial
continuing atorvastatin in the setting B. Current guidelines do not recommend vitamin
of elevated liver enzymes. Which of the E for treatment of NASH in patients with
following should be recommended? T2DM
A. He should continue atorvastatin because of C. Current guidelines recommend vitamin E for
increased cardiovascular disease risk from treatment of NASH in patients without T2DM
T2DM and NAFLD D. Vitamin E may increase overall mortality and
B. He should stop atorvastatin since you are risk of prostate cancer in men
discussing starting pioglitazone and there is
potential for drug interaction Answer: A) Vitamin E at a dosage of 800 IU daily
should be recommended given the known benefit of
vitamin E in NASH reported in the PIVENS trial

ENDO 2021 • Diabetes Mellitus and Glucose Metabolism  171

chased by Dr. Khalid H. Seedahmed, Jr., MRCP PGDip, CertEndocrinology SA - ID 283


The findings from the PIVENS trial included Hemoglobin A1c = 5.9% (4.0%-5.6%) (41 mmol/mol
adults with biopsy-proven NASH without [20-38 mmol/mol])
ALT = 48 U/L (7-40 U/L) (SI: 0.80 µkat/L
T2DM.15 Vitamin E treatment led to a higher
[0.12-0.67 µkat/L])
proportion of patients achieving the primary AST = 30 U/L (7-40 U/L) (SI: 0.50 µkat/L
histologic endpoint of improvement in 2 or [0.12-0.67 µkat/L])
more points in NAFLD Activity Score, with no Viral hepatitis panel, normal
worsening of fibrosis (43% vs 19%; P = .001).
Abdominal MRI 2 years ago showed fatty
However, resolution of NASH only reached
infiltration of the liver with a normal spleen.
borderline statistical significance, an endpoint
reached in the same study by pioglitazone (47%
If trialed in this patient, which of the
with pioglitazone vs placebo [P = .001] and 36%
following antiobesity medication(s) in
with vitamin E vs 21% with placebo [P = .05]). combination with lifestyle intervention
A recent randomized controlled trial in patients would most likely result in achieving
with T2DM showed that vitamin E alone did a weight loss goal of 8% to 11% that is
not significantly change the primary histologic expected to improve steatohepatitis?
outcome in patients with NASH and T2DM.25
A. Orlistat
In line with these findings, 2018 guidance from
the American Association for the Study of Liver B. Phentermine/topiramate ER
Diseases states that, “until further evidence C. Naltrexone/bupropion SR
supporting its effectiveness become available, D. Liraglutide, 3.0 mg
vitamin E is not recommend treating NASH in
diabetic patients.”6 Caution should be taken when Answer: B or D) Phentermine/
prescribing vitamin E, as recent data suggest that it topiramate ER or liraglutide
may increase overall mortality and risk of prostate Phentermine/topiramate (Answer B), on average,
cancer in men. produces the most weight loss compared with
the other listed options. Given this patient’s
Case 2  New Content  postmenopausal status, it is a reasonable choice
A 54-year-old postmenopausal woman with for her, as there would be no need to monitor for
history of hypertension, hyperlipidemia, and pregnancy. There are no randomized controlled
prediabetes seeks help with weight loss. She does studies exploring direct effects of phentermine/
not drink sugar-sweetened beverages, but snacking topiramate on steatohepatitis, but the combination
at times has been a problem. She is sedentary. She of 15 mg phentermine and 92 mg topiramate has
was evaluated for bariatric surgery a few years been shown to induce 8% to 11% weight loss when
ago, and she decided not to proceed. She takes compared with placebo at 12 months in 30% to
lisinopril, 20 mg daily, and atorvastatin, 20 mg 40% of patients.5
daily. Her family history is notable for T2DM in Liraglutide (Answer D) is also a good choice
her father. She does not smoke cigarettes or drink given that this patient has prediabetes. The 3.0-mg
alcohol. Physical examination findings reveal daily dosage could induce up to 8% body weight
acanthosis nigricans and central and peripheral loss. In the SCALE trial,51 liraglutide 3.0 mg
adiposity. Her BMI is 39 kg/m2, and her blood induced, on average, weight loss of 8.4 ± 7.3 kg
pressure is well controlled. (–5.6 kg placebo subtracted), and 10% or more
weight loss occurred in about 33% of patients
Laboratory test results: (vs 10.6% in the placebo group). In the LEAN
trial (1.8-mg daily dosage),17 the mean weight
Fasting glucose = 102 mg/dL (70-99 mg/dL)
(SI: 5.7 mmol/L [3.9-5.5 mmol/L]) loss was 5.5% (vs 0.7% in the placebo group) at
48 weeks and was associated with significant

172  ENDO 2021 • Endocrine Case Management

chased by Dr. Khalid H. Seedahmed, Jr., MRCP PGDip, CertEndocrinology SA - ID 283


improvement in histology (both NASH resolution AST = 36 U/L (7-40 U/L) (SI: 0.60 µkat/L
and improvement in fibrosis) in 52 patients with [0.12-0.67 µkat/L])
HDL cholesterol = 35 mg/dL (>60 mg/dL [optimal])
biopsy-proven NASH.
(SI: 0.91 mmol/L [>1.55 mmol/L])
Patients treated with naltrexone/bupropion Triglycerides = 350 mg/dL (<150 mg/dL [optimal])
(Answer C) lost 5% to 6% body weight on average (SI: 3.96 mmol/L [<1.70 mmol/L])
at 12 months. In the subset of 20% who were Urinary albumin-to-creatinine ratio = 69 mg/g creat
able to achieve more meaningful weight loss (<30 mg/g creat)
Viral hepatitis panel, normal
of 10% or more, a modest decrease in plasma
aminotransferases was observed at 56 weeks with Abdominal CT 2 years ago showed fatty
the higher dosage of the drug.5 Weight loss with infiltration of the liver with a normal spleen.
orlistat (Answer A), a lipase inhibitor, is only 3% You optimize diabetes treatment by stopping
to 4%, and only 12% of patients achieve a weight glyburide, starting semaglutide weekly, and later
loss of 10% or greater. In a meta-analysis in 330 adding empagliflozin. Because of a high AST-to-
patients with NAFLD or NASH, orlistat reduced ALT ratio, you order elastography, and results
plasma aminotransferases but failed to improve show CAP (controlled attenuation parameter) of
liver histology in NASH.39 In the study by Harrison 330 dB/m and liver stiffness (VCTE) of 12.4 kPa.
et al52 in patients with NASH, improvement You refer the patient to hepatology. Liver biopsy
in liver histology in the orlistat group was confirms the presence of NASH (NAFLD activity
proportional to the magnitude of weight loss. In score of 5/8) with stage 3 fibrosis.
the same study, patients who lost 9% or greater
body weight (n = 16) also had improved liver Which of the following is NOT
steatosis, ballooning, and inflammation (P <.001) considered a risk factor for advanced
compared with those who did not (n = 25). fibrosis in this patient?
A. Poor glycemic control
Case 3  New Content  B. High BMI
A 61-year-old man with a history of longstanding C. “Normal” liver enzymes
(10-plus years) uncontrolled T2DM, hypertension, D. High triglycerides
sleep apnea, and hypertriglyceridemia seeks E. Modest red wine consumption
help with glycemic control. His medications are
metformin, 1000 mg daily; glyburide, 10 mg daily; Answer: E) Modest red wine consumption
atorvastatin, 20 mg daily; and lisinopril. His family
history is notable for T2DM and heart disease This case exemplifies a patient with longstanding
in his father. He drinks 1 to 2 glasses of red wine diabetes and “normal” concentrations of liver
occasionally. enzymes (less than upper normal limit). However,
Physical examination findings reveal after careful review, the AST-to-ALT ratio is
acanthosis nigricans and central adiposity. His suspicious for the presence of advanced fibrosis.
BMI is 39 kg/m2, and his blood pressure is well The prevalence of NASH in persons with T2DM
controlled. is estimated at 37%, or 10-fold higher than in
the general population,2 with advanced fibrosis
Laboratory test results: in up to 17%. A normal ALT concentration does
not exclude the presence of NAFLD/NASH,23
Fasting glucose = 250 mg/dL (70-99 mg/dL)
(SI: 13.9 mmol/L [3.9-5.5 mmol/L]) and a true normal cutoff for ALT should be
Hemoglobin A1c = 9.5% (4.0%-5.6%) (80 mmol/mol ≤19 U/L (≤0.31 µkat/L) for women and ≤30 U/L
[20-38 mmol/mol]) (≤0.50 µkat/L) for men (thus, Answer C is
ALT = 31 U/L (7-40 U/L) (SI: 0.52 µkat/L incorrect). In a retrospective study of 534 adults
[0.12-0.67 µkat/L]) with biopsy-proven NAFLD, 1 in 5 patients with

ENDO 2021 • Diabetes Mellitus and Glucose Metabolism  173

chased by Dr. Khalid H. Seedahmed, Jr., MRCP PGDip, CertEndocrinology SA - ID 283


normal levels of liver enzymes had significant with biopsy-proven NASH.20 In this study,
liver fibrosis, 19% had NASH with stage 2 or 3 approximately 43% of participants showed
fibrosis, and 7% had liver cirrhosis.53 Some of the improvement in fibrosis in the semaglutide group
risk factors known to be associated with advanced vs ~33% in the placebo group at the end of the
fibrosis are older age, white ancestry, presence of 72-week treatment period. Although this was
diabetes, and multiple components of metabolic not statistically significant, it is likely clinically
syndrome (including higher triglyceride levels, relevant. Of note, worsening of fibrosis occurred
high BMI, and hypertension) (thus, Answers A, B, in 5% of patients in the semaglutide 0.4-mg group
and D are incorrect). While alcohol consumption and in 19% of patients in the placebo group. These
should not be encouraged in patients with findings could be explained by the fact that most
NAFLD, modest red wine consumption (Answer of the patients had advanced fibrosis, which may
E), likely due to antioxidants like resveratrol, has take longer to improve or may not improve at all.
the potential to suppress hepatocellular injury and This brings up current challenges in the field in
fibrosis. In a cohort of 178 patients with NAFLD estimating liver fibrosis with liver biopsy and the
and liver biopsy, the light alcohol consumer group need for better linear tools in assessing fibrosis.
(≥20 g of ethanol/day) had significantly lower In the LEAN trial,17 fewer patients in the
ballooning (marker of inflammation) and fibrosis liraglutide group (Answer A) had progression of
scores when compared with findings in the group fibrosis (9% [2/23]) than in the placebo group
that did not consume alcohol.54 (36% [8/22]). Treatment with pioglitazone (at the
higher dose of 45 mg) showed a significant mean
Case 3 (Continued)  New Content  reduction in fibrosis.14 Belfort et al also reported
a trend for changes in fibrosis compared with
baseline, and Aithal et al reported a reduction
Which of the following pharmacotherapies in fibrosis in patients without diabetes treated
has NOT been shown in clinical trials to with pioglitazone, 30 mg daily (Answer C).16,22
significantly improve fibrosis by at least 1 Improvement in fibrosis by at least 1 point
point without worsening steatohepatitis
(without worsening of steatohepatitis) was
when compared with placebo?
recently reported in the NATIVE study in 42%
A. Liraglutide of patients treated with lanifibranor (Answer
B. Semaglutide D), an investigational pan-PPAR-αγδ agonist, vs
C. Pioglitazone 24% in the placebo group (P = .011).18,19 In a large
D. Lanifibranor phase 3 trial (REGENERATE), obeticholic acid
(Answer E) met the primary end point of fibrosis
E. Obeticholic acid
improvement (≥1 stage) with no worsening of
Answer: B) Semaglutide NASH (23% in the obeticholic acid group vs 12%
in the placebo group), despite the fact that the
Surprisingly, of all the drugs listed as answer study did not meet the NASH resolution primary
options, semaglutide (Answer B) failed to show end point.42 An increase in LDL cholesterol and
significant difference in fibrosis improvement high incidence of pruritus in 50% of participants
when compared with placebo in a recently on the higher dosage are significant limitations for
published large clinical trial in patients the use of this drug clinically.

174  ENDO 2021 • Endocrine Case Management

chased by Dr. Khalid H. Seedahmed, Jr., MRCP PGDip, CertEndocrinology SA - ID 283


References
1. Younossi Z, Anstee QM, Marietti M, et al. Global burden of NAFLD and NASH: results of the NATIVE phase IIb trial. Presented at: The Liver
NASH: trends, predictions, risk factors and prevention. Nat Rev Gastroenterol Meeting Digital Experience, November 13-16, 2020.
Hepatol. 2018;15(1):11-20. PMID: 28930295 20. Newsome PN, Buchholtz K, Cusi K, et al. A placebo-controlled trial of
2. Younossi ZM, Golabi P, de Avila L, et al. The global epidemiology of NAFLD subcutaneous semaglutide in nonalcoholic steatohepatitis. N Engl J Med. 2020
and NASH in patients with type 2 diabetes: A systematic review and meta- [Epub ahead of print] PMID: 33185364
analysis. J Hepatol. 2019;71(4):793-801. PMID: 31279902 21. Ratziu V, Harrison SA, Francque S, et al; GOLDEN-505 Investigator Study
3. Stefan N, Häring HU, Cusi K. Non-alcoholic fatty liver disease: causes, Group. Elafibranor, an agonist of the peroxisome proliferator-activated
diagnosis, cardiometabolic consequences, and treatment strategies. Lancet receptor-α and -δ, induces resolution of nonalcoholic steatohepatitis
Diabetes Endocrinol. 2019;7(4):313-324. PMID: 30174213 without fibrosis worsening. Gastroenterology. 2016;150(5):1147-1159.
4. Bril F, Cusi K. Management of nonalcoholic fatty liver disease in patients PMID: 26874076
with type 2 diabetes: a call to action. Diabetes Care. 2017;40(3):419-430. 22. Aithal GP, Thomas JA, Kaye PV, et al. Randomized, placebo-controlled trial
PMID: 28223446 of pioglitazone in nondiabetic subjects with nonalcoholic steatohepatitis.
5. Barb D, Portillo-Sanchez P, Cusi K. Pharmacological management Gastroenterology. 2008;135(4):1176-1184. PMID: 18718471
of nonalcoholic fatty liver disease. Metabolism. 2016;65(8):1183-1195. 23. Portillo-Sanchez P, Bril F, Lomonaco R, et al. Effect of pioglitazone on bone
PMID: 27301803 mineral density in patients with nonalcoholic steatohepatitis: a 36-month
6. Chalasani N, Younossi Z, Lavine JE, et al. The diagnosis and management clinical trial. J Diabetes. 2019;11(3):223-231. PMID: 30073778
of nonalcoholic fatty liver disease: practice guidance from the American 24. Lewis JD, Habel LA, Quesenberry CP, et al. Pioglitazone use and risk of
Association for the Study of Liver Diseases. Hepatology. 2018;67:328-357. bladder cancer and other common cancers in persons with diabetes. JAMA.
PMID: 28714183 2015;314(3):265-277. PMID: 26197187
7. Neuschwander-Tetri BA, Clark JM, Bass NM, et al; NASH Clinical 25. Bril F, Biernacki D, Kalavalapalli S, et al. Role of vitamin E for nonalcoholic
Research Network. Clinical, laboratory and histological associations in steatohepatitis in patients with type 2 diabetes: a randomized controlled trial.
adults with nonalcoholic fatty liver disease. Hepatology. 2010;52(3):913-924. Diabetes Care. 2019;42(8):1481-1488. PMID: 31332029
PMID: 20648476 26. Cuthbertson DJ, Irwin A, Gardner CJ, et al. Improved glycaemia correlates
8. Bril F, Cusi K. Nonalcoholic fatty liver disease: the new complication of type with liver fat reduction in obese, type 2 diabetes, patients given glucagon-
2 diabetes mellitus. Endocrinol Metab Clin North Am. 2016;45(4):765-781. like peptide-1 (GLP-1) receptor agonists. PLoS One. 2012;7(12):e50117.
PMID: 27823604 PMID: 23236362
9. Cusi K. Treatment of patients with type 2 diabetes and non-alcoholic 27. Armstrong MJ, Houlihan DD, Rowe IA, et al. Safety and efficacy of liraglutide
fatty liver disease: current approaches and future directions. Diabetologia. in patients with type 2 diabetes and elevated liver enzymes: individual
2016;59(6):1112-1120. PMID: 27101131 patient data meta-analysis of the LEAD program. Aliment Pharmacol Ther.
10. Portillo-Sanchez P, Bril F, Maximos M, et al. High prevalence of nonalcoholic 2013;37(2):234-242. PMID: 23163663
fatty liver disease in patients with type 2 diabetes mellitus and normal plasma 28. Loomba R, Lutchman G, Kleiner DE, et al. Clinical trial: pilot study of
aminotransferase levels. J Clin Endocrinol Metab. 2015;100(6):2231-2238. metformin for the treatment of non-alcoholic steatohepatitis. Aliment
PMID: 25885947 Pharmacol Ther. 2009;29(2):172-182. PMID: 18945255
11. Bril F, Barb D, Portillo-Sanchez P, et al. Metabolic and histological 29. Cui J, Philo L, Nguyen P, et al. Sitagliptin vs. placebo for non-alcoholic fatty
implications of intrahepatic triglyceride content in nonalcoholic fatty liver liver disease: a randomized controlled trial. J Hepatol. 2016;65(2):369-376.
disease. Hepatology. 2017;65(4):1132-1144. PMID: 27981615 PMID: 27151177
12. Barb D, Cusi K. Reply to “statins and non-alcoholic steatohepatitis.” 30. Joy TR, McKenzie CA, Tirona RG, et al. Sitagliptin in patients with non-
Metabolism. 2017;66:e3-e5. PMID: 278655560 alcoholic steatohepatitis: a randomized, placebo-controlled trial. World J
13. Harrison SA, Bashir MR, Guy CD, et al. Resmetirom (MGL-3196) for the Gastroenterol. 2017;23(1):141-150. PMID: 28104990
treatment of non-alcoholic steatohepatitis: a multicentre, randomised, 31. Alam S, Ghosh J, Mustafa G, Kamal M, Ahmad N. Effect of sitagliptin on
double-blind, placebo-controlled, phase 2 trial. Lancet. 2019;394(10213):2012- hepatic histological activity and fibrosis of nonalcoholic steatohepatitis
2024. PMID: 31727409 patients: a 1-year randomized control trial. Hepat Med. 2018;10:23-31.
14. Cusi K, Orsak B, Bril F, et al. Long-term pioglitazone treatment for patients PMID: 29740221
with nonalcoholic steatohepatitis and prediabetes or type 2 diabetes mellitus: 32. Macauley M, Hollingsworth KG, Smith FE, et al. Effect of vildagliptin
a randomized trial. Ann Intern Med. 2016;165(5):305-315. PMID: 27322798 on hepatic steatosis. J Clin Endocrinol Metab. 2015;100(4):1578-1585.
15. Sanyal AJ, Chalasani N, Kowdley KV, et al; NASH CRN. Pioglitazone, PMID: 25664602
vitamin E, or placebo for nonalcoholic steatohepatitis. N Engl J Med. 33. Cusi K, Bril F, Barb D, et al. Effect of canagliflozin treatment on hepatic
2010;362(18):1675-1685. PMID: 20427778 triglyceride content and glucose metabolism in patients with type 2 diabetes.
16. Belfort R, Harrison SA, Brown K, et al. A placebo-controlled trial of Diabetes Obes Metab [Epub ahead of print] PMID: 30447037
pioglitazone in subjects with nonalcoholic steatohepatitis. N Engl J Med. 34. Shibuya T, Fushimi N, Kawai M, et al. Luseogliflozin improves liver fat
2006;355(22):2297-2307. PMID: 17135584 deposition compared to metformin type 2 diabetes patients with non-
17. Armstrong MJ, Gaunt P, Aithal GP, et al; LEAN trial team. Liraglutide alcoholic fatty liver disease: a prospective randomized controlled pilot study.
safety and efficacy in patients with non-alcoholic steatohepatitis (LEAN): a Diabetes Obes Metab. 2018;20(2):438-442. PMID: 28719078
multicentre, double-blind, randomised, placebo-controlled phase 2 study. 35. Ito D, Shimizu S, Inoue K, et al. Comparison of ipragliflozin and pioglitazone
Lancet. 2016;387(10019):679-690. PMID: 26608256 effects on nonalcoholic fatty liver disease in patients with type 2 diabetes:
18. Francque SM, Bedossa P, Abdelmalek MF, et al. A randomised, double- a randomized, 24-week, open-label, active-controlled trial. Diabetes Care.
blind, placebo-controlled multi-centre, dose-range, proof-of-concept, 2017;40(10):1364-1372. PMID: 28751548
24-week treatment study of lanifibranor in adult subjects with non- 36. Kuchay MS, Krishan S, Mishra SK, et al. Effect of empagliflozin on liver
alcoholic steatohepatitis: design of the NATIVE study. Contemp Clin Trials. fat in patients with type 2 diabetes and nonalcoholic fatty liver disease: a
2020;98:106170. PMID: 33038502 randomized controlled trial (E-LIFT Trial). Diabetes Care. 2018;41(8):1801-
19. Francque S. The panPPAR agonist lanifibranor induces both resolution of 1808. PMID: 29895557
NASH and regression of fibrosis after 24 weeks of treatment in non-cirrhotic

ENDO 2021 • Diabetes Mellitus and Glucose Metabolism  175

chased by Dr. Khalid H. Seedahmed, Jr., MRCP PGDip, CertEndocrinology SA - ID 283


37. Kahl S, Gancheva S, Strassburger K, et al. Empagliflozin effectively lowers 46. Harrison SA, Rinella ME, Abdelmalek MF, et al. NGM282 for treatment
liver fat content in well-controlled type 2 diabetes: a randomized, double- of non-alcoholic steatohepatitis: a multicenter, randomized, double-blind,
blind, phase 4, placebo-controlled trial. Diabetes Care. 2020;43(2):298-305. placebo-controlled, phase 2 trial. Lancet. 2018;391(10126):1174-1185.
PMID: 31540903 PMID: 29519502
38. Latva-Rasku A, Honka MJ, Kullberg J, et al. The SGLT2 inhibitor 47. Gastaldelli A, Cusi K. From NASH to diabetes and from diabetes to NASH:
dapagliflozin reduces liver fat but does not affect tissue Insulin sensitivity: mechanisms and treatment options. JHEP Reports. 2019;1(4):312-328.
a randomized, double-blind, placebo-controlled study with 8-week 48. Lomonaco R, Bril F, Portillo-Sanchez P, et al. Metabolic impact of
treatment in type 2 diabetes patients. Diabetes Care. 2019;42(5):931-937. nonalcoholic steatohepatitis in obese patients with type 2 diabetes. Diabetes
PMID: 30885955 Care. 2016;39(4):632-638. PMID: 26861926
39. Wang H, Wang L, Cheng Y, Xia Z, Liao Y, Cao J. Efficacy or orlistat in non- 49. Sterling RK, Lissen E, Clumeck N, et al; APRICOT Clinical Investigators.
alcoholic fatty liver disease: a systematic review and meta-analysis. Biomed Development of a simple noninvasive index to predict significant fibrosis
Rep. 2018;9(1):90-96. PMID: 29930810 patients with HIV/HCV coinfection. Hepatology. 2006;43(6):1317-1325.
40. Harrison SA, Ramrakhiani S, Brunt EM, Anbari MA, Cortese C, Bacon PMID: 16729309
BR. Orlistat in the treatment of NASH: a case series. Am J Gastroenterol. 50. Castera L, Friedrich-Rust M, Loomba R. Noninvasive assessment of liver
2003;98(4):926-930. PMID: 12738478 disease in patients with nonalcoholic fatty liver disease. Gastroenterology.
41. Neuschwander-Tetri BA, Loomba R, Sanyal AJ, et al; NASH Clinical 2019;156(5):1264-1281. PMID: 30660725
Research Network. Farnesoid X nuclear receptor ligand obeticholic acid 51. Pi-Sunyer X, Astrup A, Fujioka K, et al; SCALE Obesity and Prediabetes
for non-cirrhotic, non-alcoholic steatohepatitis (FLINT): a multicentre, NN8022-1839 Study Group. A randomized, controlled trial of 3.0 mg
randomised, placebo-controlled trial [published correction appears in Lancet. of liraglutide in weight management. N Engl J Med. 2015;373(1)11-22.
2015;385(9972):946]. Lancet. 2015;385(9972):956-965. PMID: 25468160 PMID: 26132939
42. Younossi ZM, Ratziu V, Loomba R, et al; REGENERATE Study 52. Harrison SA, Fecht W, Brunt EM, Neuschwander-Tetri BA. Orlistat for
Investigators. Obeticholic acid for the treatment of non-alcoholic overweight subjects with nonalcoholic steatohepatitis: a randomized,
steatohepatitis: interim analysis from a multicentre, randomised, placebo- prospective trial. Hepatology. 2009;49(1):80-86. PMID: 19053049
controlled phase 3 trial. Lancet. 2019;394(10215):2184-2196. PMID: 31813633 53. Gawrieh S, Wilson LA, Cummings OW, et al; NASH Clinical Research
43. Joshi S, Ruby S, Saboo B, Chawla R, Bhandari S. Saroglitazar in non-alcoholic Network. Histologic findings of advanced fibrosis and cirrhosis in patients
fatty liver disease. Abstract presented at: American Association of Clinical with nonalcoholic fatty liver disease who have normal aminotransferase
Endocrinologists 25th Annual Scientific & Clinical Congress; May 25-29, levels. Am J Gastroenterol. 2019;114(10):1626-1635. PMID: 31517638
2016; Orlando, FL. 54. Yamada K, Mizukoshi E, Seike T, et al. Light alcohol consumption has the
44. Harrison SA, Alkhouri N, Davison BA, et al. Insulin sensitizer MSDC-0602K potential to suppress hepatocellular injury and liver fibrosis in non-alcoholic
in non-alcoholic steatohepatitis: a randomized, double-blind placebo- fatty liver disease. PLoS One. 2018;13(1):e0191026. PMID: 29342182
controlled phase IIb study. J Hepatol. 2020;72(4):613-626. PMID: 31697972
45. Sanyal A, Charles ED, Neuschwander-Tetri B, et al. Pegbelfermin (BMS-
986036) a PEGylated fibroblast growth factor 21 analogue, in patients with
non-alcoholic steatohepatitis: a randomised, double-blind, placebo-controlled,
phase 2a trial. Lancet. 2019;392(10165):2705-2717. PMID: 30554783

176  ENDO 2021 • Endocrine Case Management

chased by Dr. Khalid H. Seedahmed, Jr., MRCP PGDip, CertEndocrinology SA - ID 283


NEUROENDOCRINOLOGY
AND PITUITARY

chased by Dr. Khalid H. Seedahmed, Jr., MRCP PGDip, CertEndocrinology SA - ID 283


Pseudoacromegaly Syndromes
Marta Korbonits, MD, PhD. Department of Endocrinology, Barts and the London
School of Medicine, Queen Mary University of London, London, United Kingdom; E-mail:
m.korbonits@qmul.ac.uk

Pedro Marques, MD, PhD. Department of Endocrinology, Barts and the London School
of Medicine, Queen Mary University of London, London, United Kingdom; E-mail:
pedro.miguel.sousa.marques@gmail.com

Learning Objectives GH/IGF-1 axis. However, some conditions can


mimic acromegaly (at least for less trained eyes)
As a result of participating in this session, learners
but have no associated GH/IGF-1 anomalies:
should be able to:
these conditions are collectively termed
• Recognize the possibility that patients may pseudoacromegaly.1 Overlapping features can
have a pseudoacromegaly condition. include acromegaloid facies and acral enlargement,
prognathism, visceromegaly, hypertension,
• Differentiate among manifestations aiding
fatigue, headaches, arthralgias, paresthesia,
clinical and biochemical diagnosis.
hyperhidrosis, oily odorous skin, hypertrichosis,
• Explain genetic causes and molecular hyperpigmentation, and low-pitched voice.
mechanisms of the disease. Apart from hyperinsulinemia,
pseudoacromegaly states are rare conditions,
and general physicians and endocrinologists
rarely encounter them. Affected patients are
Main Conclusions often seen by pediatricians or specialists in other
disciplines and are referred to endocrinology for
• There are conditions that can mimic some differential diagnosis. Pseudoacromegaly covers
features of acromegaly or pituitary gigantism. a heterogeneous range of diseases where 1 or
• Awareness of these rare conditions will help 2 features may be reminiscent of acromegaly,
the differential diagnosis in patients with but the overall picture suggests a different
acromegaloid features but no GH excess. diagnosis. Examples include Sotos syndrome,
pachydermoperiostosis, hyperinsulinemia, non–
• Most pseudoacromegaly conditions have
islet-cell tumor-induced hypoglycemia syndrome,
a genetic origin, so referral for genetic
and even drug adverse effects such as prolonged
consultation is recommended if endocrine
use of phenytoin and minoxidil.
causes have been ruled out.

Barriers to Optimal Practice


Because pseudoacromegaly conditions are rare,
Significance of the most physicians have no experience with these
Clinical Problem diseases, thus hindering diagnosis. We have
Acromegaly-related physical abnormalities are reported a case in which a patient was referred
usually distinctive at clinical examination, which to endocrinology on 4 different occasions over
should lead to the prompt assessment of the 40 years, from age 10 to 50 years, by the time the

178  ENDO 2021 • Endocrine Case Management

chased by Dr. Khalid H. Seedahmed, Jr., MRCP PGDip, CertEndocrinology SA - ID 283


correct diagnosis was made.2 There is a need to minoxidil treatment, via its stimulating activity on
increase awareness of this entity and the associated the ATP potassium channel, produces symptoms
rare, usually genetic, disorders. similar to those observed in Cantú syndrome,
acromegaloid skin changes, and hypertrichosis.
Osteopetrosis can also produce bony changes
Strategies for Diagnosis, mimicking acromegaly. The differential diagnostic
Therapy, and/or Management algorithm is shown in Figure 1.
The differential diagnosis of pseudoacromegaly
can be challenging due to the long list of Tall Stature
sometimes overlapping and rare conditions.
Key facial features (the overall “gestalt”) may Tall stature is most often due to constitutive
characterize Sotos syndrome. Distinctive forehead tall stature, while endocrine conditions such as
skin folds and joint abnormalities assist in the hyperthyroidism, congenital adrenal hyperplasia,
diagnosis of pachydermoperiostosis. Severe accelerated puberty in a child, or delayed puberty,
hypoglycemia is typical in persons with IGF-2– and familial glucocorticoid resistance can also
secreting tumors, while generalized infant-onset result in advanced height. Some skeletal disease
hypertrichosis indicates Cantú syndrome. can produce extreme tall stature. The differential
To systematically review these conditions, the diagnostic algorithm is shown in Figure 2.
main features can be assigned to 3 categories: (1)
primarily acromegaloid features with tall, normal, Biochemical Alterations
or short stature; (2) primarily tall stature; and There are 2 physiological states where both
(3) biochemical abnormalities suggestive of high GH and IGF-1 are elevated: pregnancy and
GH or IGF-1 levels. In addition to acromegaloid adolescence. The complex interplay of pituitary
features and accelerated growth, parameters such and placental GH, increased levels of IGF-binding
as dysmorphism, proportionate/disproportionate protein cleaving proteases, and elevated estradiol
growth, pubertal anomalies, and intellectual and IGF-2 result in a unique metabolic scenario
disability can help in the differential diagnosis. promoting fetal growth. In early pregnancy, there
is a slight drop in IGF-1 followed by 2 to 3 times
Acromegaloid Features increased IGF-1 peaking at the 37th week of
Acromegaloid features with rapid growth in pregnancy. GH axis evaluation is challenging in
childhood are typical of several overgrowth adolescence. GH levels are increased, particularly
syndromes3 such as Sotos syndrome, Weaver at the peak of pubertal growth, and often do not
syndrome, Beckwith-Wiedemann syndrome, and fully suppress during an oral glucose tolerance
Tatton-Brown syndrome and are often associated test. GH nadir after oral glucose tolerance testing
with a typical gestalt and mental retardation. is gender- and pubertal stage-specific. GH nadir is
Marfanoid syndromes such as Marfan and Loeys- highest in Tanner stage 2 to 3 girls (1.57 ng/mL
Dietz syndromes, are not usually associated [1.57 µg/L]) and lower in other pubertal stages
with delayed development, while fragile X (0.64 ng/mL [0.64 µg/L]) or for boys (0.5 ng/mL
syndrome is. Endocrine or metabolic syndromes [0.5 µg/L]). Serum IGF-1 correlates better with
such as severe insulin resistance, IGF-2–related pubertal stage than chronological age. Delayed
hypoglycemia, Berardinelli-Seip congenital or accelerated puberty can result in a mismatch
generalized lipodystrophy, and even untreated between the measured hormone level and the
primary hypothyroidism can be associated with chronological IGF-1 reference range. Evaluation
acromegaloid clinical features. Phenytoin can of adolescents and young women can be further
induce some acromegaloid features. Longstanding complicated by the commonly seen increased
pituitary volume in this setting.

ENDO 2021 • Neuroendocrinology and Pituitary  179

chased by Dr. Khalid H. Seedahmed, Jr., MRCP PGDip, CertEndocrinology SA - ID 283


Figure 1. Differential Diagnostic Algorithm of Acromegaloid Features

Conditions with hypertrichosisl hirsutism


Berardinelli-Seip congenttal generalized lipodystrophy
cantu sy.
I Acromegaloid facies
I
C.Offin-Siris sy.
c.orne~ de Lange sy.
0-ugs: 11/inoxidl: Aienytoin YES
lnsulln-rrediated pseudoacromegafy
Marshal-Snith sy,
I GH/IGF-1 excess Acromegaly/ Pituitary gigantism

Ncolaides-Baraftser sy. I IGF-2 excess YES


Non-islet cell tumor-induced hypoglycemia I
NO

Acceler ated growth / tall stature


I I Normal growth / normal stature
I I Growth retardation I short stature I

Intellectua l Intellectual Intellectual


I disability I I disability I I disability I
YES NO YES NO YES NO
I I I I I I
fuumctkmilt1 Q'lt'.l:cgcgwtb ecggg[lhloabt ~[QWWlb l!J&.cal 1olacam1Dl e&Cill IDlilCQIWIDl tliDd DWl:il CCmDtiail1'2kl l:laod ogo~accmnrualgld
Berardinelli-Seip congenital Beckwith-Wiedmann sy. Pallister-Killian sy. Cantusy. dlfm:mllil& deformtttfts
lipodyslrophy Chromosome 11 Inversion Insulin-mediated Borjeson-Forssman- Scterosteosis
EEO-related sy. HMGA2 rearrangements- t:laod Dl2D:llCCWIHIQillli2id pseudoacromegaly Lehmannsy.
Malan sy. related overgrowth dafQl::mi:lill Mino>ddil Coffin-Lowry s y. ~m:mal baod
Mars hall-Smith s y. Coffin-Sins sy. Primaryh)1)0thyroidism Cornelia de Langesy. Osleopetrosls
M ·c roduplications 15q26 Nicolaides-Baraftsersy.
Phe.lan-McDermid sy. Haod DQD:iCCmDIQillhlid SETOS-related Sy.
SET02-related sy. deformities
Simpson-Golabi-Behmel sy. Klippel-Trenaunaysy.
Sotossy. Pach)<lermoperiostosis
Tatton-Brown-Rahman s y.
Weaver sy. Normal hand
X-lelrasomy Anti-retrmiirals
Barraquer-Simons sy.
D:lspcopoctio.oale Fabry disease
Q.Ye:CQCS2Wlb Phenytoin
FraglleX sy.

There is only one known disease condition in have lost their contour, furrowing of the forehead
which both high GH and IGF-1 can be seen skin, and clubbing of fingers and toes. His body
without the patient having clinical acromegaly: the habitus is lean rather than the bulky habitus typical
recently described IGSF1 deficiency syndrome.4,5 of patients with acromegaly.
Discrepant GH and/or IGF-1 levels can
be detected in several conditions (Table) and Although all are reasonable options,
considering these diagnoses will help to interpret which of the following is the best
unusual biochemical results. step to help in the diagnosis?
A. Take a family history, including questions
regarding consanguinity
Clinical Case Vignettes B. Perform baseline pituitary function tests
C. Refer for a genetic consultation
Case 1
D. Measure urinary prostaglandin E2
A 26-year-old man with tall stature is referred to
endocrinology for possible gigantism. The patient Answer: D) Measure urinary prostaglandin E2
is significantly taller than his midparental height,
has had painful joints since age 6 years, and has The key clinical diagnostic clue in this case
had excessive sweating since age 15 years. is the clubbing, with additional clues being
On physical examination, he has enlarged furrowing of forehead, profuse sweating,
hands and feet with wide wrists and ankles that young-onset joint pain, and skinny body

180  ENDO 2021 • Endocrine Case Management

chased by Dr. Khalid H. Seedahmed, Jr., MRCP PGDip, CertEndocrinology SA - ID 283


Figure 2. Differential Diagnostic Algorithm of Tall Stature

Prenatal overgrowth
Beckwith•Wieoemam sy. I Accelerated growth / Tall stature I
EEO-related sy.
Maternal diabetes
t.tcroduplications 15q26
Simpson-Golabi-Behmel sy.
V I GH/IGF-1 excess
YES
Acromegaly/ Pituitary gigantism
Sotossy.
Tatton•Brown.Rahman sy.
NO
Weawrsy.

YES I Dysmorphism I No
I I
YES Proportionate
overgrowth
I
I
NO ;es I Pubertal
anomalies J-;
I
Constitutional !all slature
Constitutional adwnce ofgrow
Obesity
Th}fflto>Ooos is

I Intellectual
disability I lntellec1ual
disability I
YES NO YES NO
I I I

Acc,un1:galg;ld facits &mmegalgld facias &cmm1galgld facles ~g as:m::unegafgJd facles euu;.a!;iau1 gub1r:1lt' Dela~ed g:ubem'.
Berardinelli-Seip congenital Beckwith-Wiedmann s y. Fragile X s y. Marian sy. NachnoldcyslS Nomalasedeficiency
lipodystrophy Chromosome 11 imers ion SHOX owrdosage (e.g. X• Eplphyseal chondrodysplasia CNS tumors Autoimmuneoophoritis
EEO-related sy. HMGA2 rearrangemenls· telrasomy) Mura tw<1 (CNP/NPR Congenital adrenal Estrogendeficiencyor
M:llan sy. related overgrowth signalopalhy) hyperplasia fnsensitl\Jfty
Cranial irradiation Gonadal damage by
Marshall-Smith sy. Hg a,mm1galgis::I u,11s Cranlopha'}<lglomas lrauma,surgery,
t.tcroduplications 15q26 lu- - - - Familialglucocorticoid
Phelan-McDermid sy. Encephalibs chemotherapyor
CATSHL sy. deficiency
SETD2-telated sy. Estrogen-se<::reting tumors radiotherapy
Klinefeltersy. Emgenous estrogens
SETD5-relaled sy, H)l)ogonadism
Familial testoto)Ocosis Klinefellor sy.
Simpson-Golabl-Behmel sy.
Sotossy. hCG-secreting tumors LHIFSH resistance sy.
Tetton--Srown•Rahmansy. Headlrauma Pol)Cystic ovary sy.
Woaversy. H)<lrocephalus Steroid bios)"lhetic
H)1)0111alam ic hamartomas defects
Kisspeptinsystem mutations Tumersy.
f:io aC!OmegalQid facies Le)<llg cell adenomas
flBP-related owrgrowlh sy.
M)elomenlngooele
Sarcold or tubercular
granulomas
Wlllzing adrenal tumors

Table. Conditions Associated With Discrepant GH and/or IGF-1 Levels

Normal GH and High IGF-1 High GH and Normal or Low IGF-1

• Adolescence • Adolescence
• Pregnancy • Pregnancy
• Increased tissue responsiveness to GH (eg, genotype D3 of the GH receptor) • Diabetes mellitus
• Supraphysiologic testosterone levels • Obesity
• Early or mild acromegaly • Malnutrition
• “Burnt out acromegaly” • Eating disorders
• Early postoperative period of acromegaly • Cystic fibrosis
• GH cutoff too high • Renal failure
• IGF-1 cutoff too low • Liver disease
• Technical issues with biochemical assays • Hypothyroidism
• Oral estrogen treatment
• “Burnt out acromegaly”
• GH cutoff too low
• IGF-1 cutoff too high
• Technical issues with biochemical assays

ENDO 2021 • Neuroendocrinology and Pituitary  181

chased by Dr. Khalid H. Seedahmed, Jr., MRCP PGDip, CertEndocrinology SA - ID 283


habitus—these are all characteristics of patients Case 2
with pachydermoperiostosis. The tall stature
A female patient is referred due to prominent
could be a red herring; although striking in
acromegaloid facial features with course skin,
this case,6 it is not typically described with
thickened fingers, and wide-spread hirsutism.
pachydermoperiostosis. Measuring baseline
There is a family history of infant-onset
pituitary hormones (Answer B) can help rule
generalized hirsutism in 5 members (2 males,
out acromegaly. This disease is due abnormally
3 females) of this 3-generational family and
high prostaglandin E2 (PGE2) levels in the blood
pericardial effusions in 1 member.
either caused by defective uptake into the lung
(biallelic pathogenic variants in the transporter Which of the following is the best
SLCO2A1 gene) where PGE2 is degraded, or due approach to this patient?
to defective enzymatic clearing of PGE2 caused
A. Measure GH and IGF-1
by biallelic pathogenic variants in the HPGD
gene (15-hydroxyprostaglandin-dehydrogenase). B. Measure gonadal hormones
A positive family history and consanguinity C. Type clinical features into Google
(Answer A) would support the diagnosis. Clinical
Answer: C) Type clinical features into Google
features include clubbing, joint problems, and
pachydermia. Arthralgia is present in 40% of This is a challenging diagnosis of a multisystem
cases, with synovial effusions, periosteal changes, disorder. This patient has hypertrichotic
acroosteolysis, and ligaments/interosseous osteochondrodysplasia or Cantú syndrome.
membranes ossifications. There are some Previously used names include acromegaloid
phenotypic differences between the 2 subtypes. facial appearance syndrome or hypertrichosis
Genetic consultation (Answer C) could acromegaloid facial features syndrome. There
identify the disease-causing pathogenic variants. is a long list of possible manifestations,7 which
There is male preponderance, hypocellular probably explains the various names. Typing
myelofibrosis, and more prominent cutis verticis clinical features into Google (Answer C) may
gyrata in patients with SLCO2A1 pathogenic be helpful. The most prominent manifestation
variants, while younger onset of disease is is the generalized hypertrichosis, in addition to
more characteristic of patients with HPGD the coarse facial features, bulbous nose, thick lips,
pathogenic variants. In patients with HPGD long philtrum, macroglossia, epicanthic folds,
pathogenic variants, there is elevated urinary cardiomegaly, patent ductus arteriosus, pericardial
PGE2 (Answer D), with low/undetectable effusions, broad ribs, hyperextensibility of joints,
PGE2 metabolites, while in patients with wrinkled skin, macrosomia at birth, and recurrent
SLCO2A1 pathogenic variants, both PGE2 and its infections. The coarse facial features and thickened
metabolites are high in the urine. fingers would raise the possibility of acromegaly
The patient in this vignette has homozygous (Answer A). Hypertrichosis in a female patient
frameshift mutations in the HPGD gene, and would raise the possibility of androgen excess
his younger brother is also affected. The most (Answer B). Hypertrichotic osteochondrodysplasia
obvious differential diagnosis for clubbing includes is caused by activating pathogenic variants in the
lung disease, and the mechanism is probably ABCC9 gene (sulfonylurea receptor 2) or, rarely,
linked, as lung is the primary site for PGE2 its partner the KCNJ8 gene (coding for Kir6.1),
degradation. Cutis verticis gyrata can be seen in resulting in overactive ATP-sensitive potassium
acromegaly, severe hyperinsulinemia, amyloidosis, channels. While these receptors are present in
hypothyroidism, and Noonan syndrome, etc. skeletal and heart muscle, probably explaining the
heart phenotype, the mechanism leading to the
abnormal skin changes and hirsutism is unclear. A

182  ENDO 2021 • Endocrine Case Management

chased by Dr. Khalid H. Seedahmed, Jr., MRCP PGDip, CertEndocrinology SA - ID 283


unique phenocopy of this disease is seen in patients • Are there biochemical features of abnormal
with prolonged use of minoxidil, as minoxidil gonadal and adrenal hormones?
activates this potassium channel and is of course • Is there a family history of tall stature or
used to induce hair growth. In this vignette, gigantism?
the coarse facial features and thickened fingers
raise the possibility of acromegaly (Answer A). • What do the patient’s toes look like?
Hypertrichosis in a female patient raises the The differential diagnosis of tall stature is broad
possibility of androgen excess (Answer B). and we refer to excellent reviews.8 This patient
has extreme tall stature at a young age. Of the
Case 3 endocrine causes, the only abnormality resulting
in this degree of SD in height is X-linked
A 4-year-old girl is referred because of extreme tall
acrogigantism (Answer A), characterized by
stature (SD +5). There is no family history of tall
infant-onset acromegaly. Overgrowth syndromes
stature or pituitary disease.
can cause tall stature in early childhood, although
Which of the following should be such individuals often have normal adult height
included in the differential diagnosis? due to advanced bone age. Marfan syndrome can
be associated with rapid growth in small children.
A. X-linked acrogigantism
For Miura type epiphyseal chondrodysplasia, the
B. McCune-Albright syndrome length of the toes could be diagnostic—a striking
C. Pathogenic variants in the AIP gene feature, although not described in every case.9
D. Neurofibromatosis type 1 If pituitary gigantism is suspected, serum
E. Carney complex GH, IGF-1, and prolactin measurement, as well
as pituitary MRI, would help to establish the
F. Multiple endocrine neoplasia type 1 or 4
diagnosis. Almost 80% of patients with X-linked
G. Congenital adrenal hyperplasia acrogigantism are female. Normal birth weight
H. Skeletal disease (Marfan syndrome, Loeys- with rapid growth under the age of 1 year is
Dietz syndrome, epiphyseal chondrodysplasia, typical. Associated clinical features include
Miura type, and homocystinuria) coarse facial features, widened hands and feet,
I. Overgrowth syndromes (Sotos syndrome, increased appetite, and increased BMI. X-linked
Weaver syndrome, Tatton-Brown syndrome) acrogigantism is due to duplication of the orphan
receptor GPR101 gene and results in excess of
Answer: H) Epiphyseal chondrodysplasia, Miura type GH/IGF-1 and usually prolactin due to pituitary
To explore the diagnosis, the following questions hyperplasia or a tumor. The genetic diagnosis
could be asked: can be confirmed by a comparative genomic
hybridization array, although a small duplication
• Does the patient have features compatible with may not be detected by this technique. Droplet
GH excess, such as acromegaloid body habitus, digital polymerase chain reaction or targeted
increased appetite, sweating, vision problems, array could be used if comparative genomic
and headache? hybridization array is negative. In male patients,
• What was the patient’s birthweight? mosaicism is associated with the disease; therefore,
ideally, DNA extracted from pituitary tissue can
• Is there evidence of intellectual disability or identify the GPR101 duplication. For overgrowth
central nervous system abnormalities? syndromes, clinical phenotyping is key, followed
• Does the patient have signs of early puberty, by targeted gene sequencing or overgrowth gene
skin lesions, or bony abnormalities? panel testing to verify the genetic diagnosis.

ENDO 2021 • Neuroendocrinology and Pituitary  183

chased by Dr. Khalid H. Seedahmed, Jr., MRCP PGDip, CertEndocrinology SA - ID 283


Epiphyseal chondrodysplasia, Miura type is ectopia lentis is the hallmark feature seen in nearly
caused by the upregulation of the natriuretic peptide 60% of cases. Patients should be evaluated for
C (abbreviated either as NPC or CNP) pathway. cardiovascular features such as dilation of the aorta,
NPC lacks natriuretic functions and via activation predisposition to aortic rupture, enlarged pulmonary
natriuretic peptide receptor type 2 (NPR2), it artery, and mitral valve prolapse. The Ghent criteria
stimulates endochondral bone growth, promoting could be used for diagnosis. Marfan-like syndromes
synthesis of cartilage matrix and stimulating include Beals syndrome due to pathogenic variants
chondrocyte differentiation and proliferation. The in the fibrillin-2 gene (FBN2) and Loeys-Dietz
most striking and practically pathognomonic feature syndrome due to pathogenic variants in genes part
is the significantly extended halluces. Long hands and of the TGFβ-pathway (SMAD2, SMAD3, TGFB2,
feet, arachnodactyly, camptodactyly, clinodactyly, TGFB3, TGFBR1, TGFBR2).
syndactyly, feet deformities, tibia and femur bowing,
unstable slipped capital femoral epiphysis, scoliosis, Does the patient have features compatible
hyperlordosis, dorsal dysmorphism, multiple hernias, with GH excess, such as acromegaloid
wide vertebral canal, and low bone mineral density body habitus, increased appetite, sweating,
can be present. Genetic testing includes searching vision problems, or headache?
for activating pathogenic variants in the NPC One could consider early-onset GH excess,
receptor NPR2 gene, upregulation/duplication X-linked acrogigantism, or possibly McCune-
of the gene coding for NPC (NPR1), or loss-of- Albright syndrome.
function pathogenic variants in the scavenger
receptor NPR3 gene.10 Biochemical assessment of What was the patient’s birthweight?
serum NTproCNP can be useful. In the setting Increased birth weight would be compatible
of NPC duplication/upregulation, NTproCNP with overgrowth syndromes or some Miura type
levels are high, while in the setting of activating abnormalities.
NPR2 pathogenic variants and loss-of-function
NPR3 pathogenic variants, NTproCNP levels are Is there evidence of intellectual disability
low. cGMP, the second messenger downstream of or central nervous system abnormalities?
NPR2, is elevated in all 3 cases. This would suggest overgrowth syndromes,
Marfan syndrome is due to pathogenic variants possibly fragile X syndrome.
in the fibrillin-1 gene (FBN1). Altered fibrillin-1
Does the patient have signs of early puberty
leads to upregulation of the TGFβ pathway.
with skin lesions and bony abnormalities?
Clinical features include tall stature, which could
This would suggest McCune-Albright syndrome.
be comparable to that observed in acromegalic
gigantism,11 and typically disproportionately long Are there biochemical features of abnormal
extremities for trunk size (dolichostenomelia) gonadal and adrenal hormones?
leading to an increased arm span-to-height This would suggest congenital adrenal hyperplasia.
ratio and lower-to-upper segment ratio. Pectus
carinatum or excavatum, scoliosis, pes planus or Is there a family history of tall
cavus, and long fingers are common. stature or gigantism?
Skeletal manifestations can be present in young This could be present in several diseases, such as
children and progress in periods of rapid growth. skeletal disorders, X-linked acrogigantism, AIP
Facial features include narrow and long face, pathogenic variants.
enophthalmos, downslanting palpebral fissures,
malar hypoplasia, and micro/retrognathia, although What do the patient’s toes look like?
prognathism may be present in 30% of patients. Extra-long halluces could be diagnostic for diseases
Myopia is the most common ocular finding, but affecting the natriuretic peptide C pathway.

184  ENDO 2021 • Endocrine Case Management

chased by Dr. Khalid H. Seedahmed, Jr., MRCP PGDip, CertEndocrinology SA - ID 283


References
1. Marques P, Korbonits M. Pseudoacromegaly. Front Neuroendocrinol. 7. Marques P, Spencer R, Morrison PJ, et al. Cantu syndrome with coexisting
2019;52:113-143. PMID: 30448536 familial pituitary adenoma. Endocrine. 2018;59(3):677-684. PMID: 29327300
2. Dahlqvist P, Spencer R, Marques P, et al. Pseudoacromegaly: a differential 8. Albuquerque EVA, Scalco RC, Jorge AAL. Management of endocrine
diagnostic problem for acromegaly with a genetic solution. J Endocr Soc. disease: diagnostic and therapeutic approach of tall stature. Eur J Endocrinol.
2017;1(8):1104-1109. PMID: 29264563 2017;176(6):R339-R353. PMID: 28274950
3. Kamien B, Ronan A, Poke G, et al. A clinical review of generalized 9. Hannema SE, van Duyvenvoorde HA, Premsler T, et al. An activating
overgrowth syndromes in the era of massively parallel sequencing. Mol mutation in the kinase homology domain of the natriuretic peptide
Syndromol. 2018;9(2):70-82. PMID: 29593474 receptor-2 causes extremely tall stature without skeletal deformities. J Clin
4. SunY, Bak B, Schoenmakers N, et al. Loss-of-function mutations in IGSF1 Endocrinol Metab. 2013;98(12):E1988-E1998. PMID: 24057292
cause an X-linked syndrome of central hypothyroidism and testicular 10. Boudin E, de Jong TR, Prickett TCR, et al. Bi-allelic loss-of-function
enlargement. Nat Genet. 2012;44(12):1375-1381. PMID: 23143598 mutations in the NPR-C receptor result in enhanced growth and connective
5. Joustra SD, Roelfsema F, van Trotsenburg ASP, Schneider HJ, et al. IGSF1 tissue abnormalities. Am J Hum Genet. 2018;103(2):288-295. PMID: 30032985
deficiency results in human and murine somatrotrope neurosecretory 11. Marques P, Collier D, Barkan A, Korbonits M. Coexisting pituitary and non-
hyperfunction. J Clin Endocrinol Metab. 2020;105(3):e70-e84. PMID: 31650157 pituitary gigantism in the same family. Clin Endocrinol (Oxf). 2018;89(6):887-
6. Marques P, Stelmachowska-Banas M, Collier D, Wernig F, Korbonits M. 888. PMID: 30223298
Pachydermoperiostosis mimicking the acral abnormalities of acomegaly.
Endocrine. 2020;67(2):499-500. PMID: 31916215

ENDO 2021 • Neuroendocrinology and Pituitary  185

chased by Dr. Khalid H. Seedahmed, Jr., MRCP PGDip, CertEndocrinology SA - ID 283


Prevention and Management
of Carcinoid Crisis
Electron Kebebew, MD. Stanford University, Stanford, CA; E-mail: kebebew@stanford.edu

Learning Objectives or surgery, interventional therapy, radionuclide


therapy, examination, medication (eg, serotonergic
As a result of participating in this session, learners
drugs such as clomipramine and amitriptyline),
should be able to:
and biopsy. Carcinoid crisis can also occur
• Recognize the symptoms and signs of spontaneously. The treatment for carcinoid crisis
carcinoid syndrome and crisis. and syndrome includes somatostatin analogues,
histamine-receptor blockers, vasopressin or
• Identify risk factors for carcinoid crisis.
phenylephrine for significant hypotension, and
• Treat carcinoid crisis. α-adrenergic and β-adrenergic receptor blockers
for hypertension.

Main Conclusions
Significance of the
The clinical hallmarks of classic carcinoid
syndrome include episodic flushing, diarrhea,
Clinical Problem
cardiac valvular disease, and, less frequently, The family of neuroendocrine tumors consists
bronchospasm. Most patients with classic of a spectrum of neoplasms that can arise from
carcinoid syndrome have metastatic small bowel the diffuse endocrine system throughout the
carcinoid. Extraintestinal carcinoids can cause the body. The incidence of neuroendocrine tumors
syndrome in the absence of metastatic disease, (commonly referred to carcinoid) is increasing, so
because the substances that they secrete gain many health care providers will encounter patients
direct access to the central venous circulation. with neuroendocrine tumors.1,2 The carcinoid
Carcinoid crisis is defined as a severe episode tumor, a member of this fascinating family of
of flushing, hypotension, tachyarrhythmia, tumors, is characterized by its ability to uptake (or
confusion, and respiratory distress. Some synthesize), store, and release a variety of biogenic
patients with functioning gastric or bronchial amines, polypeptides, and prostaglandins.3-4
carcinoids have clinical and biochemical variations Although carcinoid tumors have been reported in
from the classic carcinoid syndrome. It is very essentially all tissues, the overwhelming majority
important to understand the biochemical and of these neoplasms originate in the gastrointestinal
pathophysiological link between the carcinoid tract. Most carcinoid tumors have a relatively
tumor and the carcinoid syndrome, because the slow-growing, indolent natural history, but
physician may be easily misled into thinking the sometimes they can behave in a highly malignant
patient has isolated functional problems rather fashion, exhibiting metastatic behavior, most often
than a syndrome caused by a tumor. Risk factors to the liver and lungs.
for or precipitating events for carcinoid crisis The early symptoms of carcinoid tumor are
include, most common to least common, the nonspecific and may include abdominal pain,
presence of carcinoid heart disease, anesthesia diarrhea, intermittent intestinal obstruction,

186  ENDO 2021 • Endocrine Case Management

chased by Dr. Khalid H. Seedahmed, Jr., MRCP PGDip, CertEndocrinology SA - ID 283


and gastrointestinal bleeding. These often vague The incidence of classic carcinoid syndrome,
and generalized signs and symptoms produce a which is characterized by flushing, diarrhea,
delay in diagnosis in many patients. Although abdominal cramping, and, less often, wheezing,
symptoms may be caused by mechanical effects of heart-valve dysfunction, and pellagra, varies
the tumors, symptoms of carcinoid tumors are also depending on the study population. While studies
caused by the effects of amine and neuropeptide that include patients with localized and incidental
substances secreted into the gastrointestinal tumors report the frequency of carcinoid
tract or the systemic circulation. The effects of syndrome to be about 10% to 18%, the prevalence
these chemicals, collectively referred to as the may be as high as 50% among patients with
carcinoid syndrome, are diverse and include advanced and metastatic disease. The syndrome
cutaneous flushing, diarrhea, carcinoid heart results from synergistic interactions between
disease, bronchoconstriction, hypotension, biochemical substances secreted by the tumor, and
hypertension, and pellagra. Carcinoid crisis occurs it is often observed in patients with metastatic
with worsening of all of these symptoms and can disease or when the primary tumor site allows
result in cardiovascular collapse and death if not the secreted amines to escape enteral hepatic
recognized and treated early. The cornerstone circulation. The presence of carcinoid syndrome is
of avoiding carcinoid crisis is identifying at- associated with poor survival.
risk patients and ensuring rapid and effective In 1954, Pernow and Waldenstrom
treatment. demonstrated presence of serotonin in the blood
and urine of 2 patients with carcinoid tumor
during an attack of flushing. Originally, the whole
Barriers to Optimal Practice spectrum of symptoms was thought to be related
to the production and secretion of serotonin by the
• Carcinoid syndrome and crisis symptoms may
tumor and its metastases; however, since that time,
be nonspecific.
a number of hormonal substances contributing
• There is a great deal of heterogeneity to the signs and symptoms of carcinoid syndrome
regarding the clinical definition of carcinoid have been described, and the syndrome itself has
crisis for assessing treatment effectiveness. been more fully characterized. Carcinoid tumors
• The role of prophylactic somatostatin produce and secrete a great variety of bioactive
analogues in the treatment of carcinoid crisis is compounds, including serotonin, histamine,
unclear. bradykinin, tachykinin, motilin, substance P,
kallikrein, prostaglandins, catecholamines, and
in some cases ACTH. Although the contribution
Strategies for Diagnosis, of each of these mediators is not yet completely
understood, some of them, including serotonin,
Therapy, and/or Management
histamine, tachykinins, and kallikrein, are thought
Classic carcinoid syndrome does not develop until to have a major role in the development of the
a tumor has metastasized to the liver and/or lung, clinical picture of carcinoid syndrome and crisis.
and the hormonal products released by the tumor
reach the central venous circulation in substantial
concentrations. Some patients with functioning Clinical Presentation of Classic
gastric or bronchial carcinoids have clinical and Carcinoid Syndrome
biochemical variations from the classic syndrome, Carcinoid syndrome, the hormonal manifestation
as this tumor can directly drain into the central of carcinoid tumors, is generally characterized by
venous circulation without the metabolites from flushing, diarrhea, and, less commonly, wheezing,
the tumor being metabolized by the liver. heart-valve dysfunction, and pellagra (Table 1).

ENDO 2021 • Neuroendocrinology and Pituitary  187

chased by Dr. Khalid H. Seedahmed, Jr., MRCP PGDip, CertEndocrinology SA - ID 283


The frequency of carcinoid syndrome outside the gastrointestinal tract. In up to 90% of
correlates with the extent of the metastatic patients, the development of carcinoid syndrome
carcinoid disease, and it can be as low as 10% in is associated with metastatic tumors originating
localized cases, but it can occur in 40% to 50% in the midgut. In contrast, bronchial carcinoid
of patients with more advanced disease. The tumors are associated with carcinoid syndrome
presence of carcinoid syndrome is associated with in approximately 10% of cases, and nearly all
poor survival. Not all patients with carcinoid hindgut tumors are hormonally silent. As many
tumors develop carcinoid syndrome. Symptoms as 40 different substances have been identified
develop when the secretory products released in various carcinoid tumors.4 Some of these
from the tumors gain direct access to the tumor products are responsible for the carcinoid
systemic circulation, bypassing metabolism in syndrome, but the relative contributions of each
the liver. The conditions in which such a bypass and specificity of any for particular components
may develop include the presence of liver and of the syndrome are yet unknown (Table 2). Thus,
lung metastases draining directly into the caval the diagnosis of carcinoid syndrome and crisis is
circulation, retroperitoneal disease with drainage clinical based on the symptoms and signs.
into paravertebral veins, or primary sites of disease

Table 1. Common Symptoms in Patients With Carcinoid Syndrome: Frequency and Association With Secreted Compounds

Symptoms Putative Substances Frequency, %

Flushing Histamine, tachykinins, kallikrein, bradykinin >90

Diarrhea Serotonin, substance P, motilin, prostaglandins 70-85

Cardiac valvular disease Serotonin, 5-hydroxyindoleacetic acid, neurokinin A, substance P ~40-60

Telangiectasias Serotonin, bradykinin, prostaglandins 25

Bronchospasm Serotonin, bradykinin 15-19

Pellagra (dermatitis, diarrhea, dementia) Niacin deficiency due to excess tryptophan metabolism 5-7

Table 2. Association of Carcinoid Syndrome With Carcinoid Tumor Localization and Secreted Compounds

Site of Tumor Localization Secreted Substances Carcinoid Syndrome

Foregut Bronchus 5-hydroxytryptophan, histamine, peptide Atypical, rarely Cushing syndrome


hormones, ACTH (bronchial, thymus)
Thymus

Stomach

Duodenum

Midgut Jejunum Serotonin, peptide hormones, prostaglandins Classic

Ileum

Appendix

Ascending colon

Hindgut Transverse colon Variable peptide hormones Rare

Descending colon

Sigmoid colon

Rectum

188  ENDO 2021 • Endocrine Case Management

chased by Dr. Khalid H. Seedahmed, Jr., MRCP PGDip, CertEndocrinology SA - ID 283


Some patients with functioning gastric or Carcinoid Crisis
bronchial carcinoids have clinical and biochemical
Some patients with carcinoid tumors develop
variations from the classic carcinoid syndrome.
carcinoid crisis, a life-threatening form of
carcinoid syndrome, characterized by profound
Gastric Variant Syndrome
flushing, diarrhea, arrhythmias, extreme changes
Gastric carcinoid tumors have 2 important
in blood pressure, bronchospasm, and altered
biochemical features, which may explain at least in
mental status.1-3 This medical emergency may
part the pathophysiologic basis of variant gastric
occur spontaneously, after palpation of the tumor,
carcinoid syndrome. First, gastric carcinoids are
or during anesthesia, surgery, or chemotherapy.
usually unable to convert the dietary tryptophan
It has been hypothesized that these precipitating
into serotonin, because they are frequently
events may stimulate release of an overwhelming
deficient in the enzyme aromatic L-amino acid
amount of bioactive compounds by the tumor,
decarboxylase. Serotonin has been increasingly
leading to a multisystem functional collapse
implicated in the pathophysiology of diarrhea and
refractory to fluid resuscitation and administration
carcinoid heart disease. As a consequence, the
of vasopressors. A great deal of precaution
diarrhea and cardiac lesions, which are common
is needed prior to surgery, embolization, or
features of classic midgut carcinoid syndrome, are
chemotherapy, because carcinoid crisis may be
unusual in the gastric variant of the syndrome.
precipitated by anesthesia with intraoperative
Second, gastric carcinoids can produce large
complications occurring in 2% to greater than
amounts of histamine, which has been shown
50% of patients. Octreotide should be available
to have primary role in the atypical flushing and
for intraoperative use or given preemptively,
pruritus associated with these tumors. In patients
especially in patients with extensive tumor load.5
with the gastric carcinoid variant, the flushes may
During carcinoid crisis, an initial bolus dose of
be patchy, sharply demarcated, serpiginous, and
50 mg intravenously with further increments
cherry red; they are also intensely pruritic.
as required can be successful in reversing the
Bronchial Carcinoid Variant Syndrome condition. In addition, histamine-receptor
Bronchial carcinoids produce less serotonin than do blockers may be useful in reducing histamine
midgut carcinoids, accounting for a lower rate of release from gastric carcinoid tumors. Close
carcinoid syndrome. Although carcinoid syndrome hemodynamic monitoring may be required for
is encountered only rarely among patients with patients with carcinoid heart disease, who are
localized bronchial carcinoids (majority of typical at risk of developing arrhythmias and extreme
carcinoids), it can be present in more than 80% changes in blood pressure. Regional anesthesia
of patients with liver metastases. The symptoms and specific pharmacologic agents such as
of bronchial carcinoid syndrome may be atypical, morphine, suxamethonium, β-adrenergic blockers,
with episodes of severe and prolonged flushing, tubocurarine, halothane, and atracurium should
lasting hours to days. Other related manifestations be avoided. Additional supportive treatment
include disorientation, anxiety, tremor, lacrimation, in patients with a carcinoid crisis includes
salivation, hypotension, tachycardia, diarrhea, vasopressin or phenylephrine for significant
asthma, and edema. Approximately 1% to 2% of hypotension and α-adrenergic and β-adrenergic
bronchial carcinoids (both typical and atypical) are receptor blockers for hypertension and
associated with Cushing syndrome due to ectopic tachyarrhythmias.
production of ACTH.

ENDO 2021 • Neuroendocrinology and Pituitary  189

chased by Dr. Khalid H. Seedahmed, Jr., MRCP PGDip, CertEndocrinology SA - ID 283


Clinical Case Vignettes Case 2
Case 1 A 54-year-old man is having cryoablation of
liver neuroendocrine tumor metastases from an
A 68-year-old woman with flushing is found to
ileal neuroendocrine tumor (carcinoid). During
have an ileal neuroendocrine tumor (carcinoid)
the procedure he develops flushing and becomes
with regional involved lymph nodes and liver
hypotensive (70/30 mm Hg) and develops
metastasis (see image). Her 5-hydroxyindoleacetic
bronchospasm.
acid level is elevated.
Which of the following should be
administered as the most appropriate
management for this patient?
·~r•

A. β-adrenergic blockers
B. Octreotide bolus with continuous infusion,
antihistamine, and vasopressin
C. An antihistamine
D. A blood transfusion
E. Phentolamine
Answer: B) Octreotide bolus with continuous
infusion, antihistamine, and vasopressin
68
Gallium DOTATATE imaging showing small bowel
neuroendocrine tumor, mesenteric lymph node disease, and liver
metastasis. This patient has classic signs of carcinoid crisis that
is best managed with octreotide bolus and drip,
Her risk of having a carcinoid vasopressin for hypotension, and antihistamine
crisis at the time of her operation (Answer B), as some neuroendocrine tumors may
is which of the following? secrete histamine. The patient has hypotension,
A. None existent and β-adrenergic blockers (Answer A) would make
it worse. An antihistamine (Answer C) would only
B. Very low
block the effect of histamine and would not treat
C. 100% the hypotension. A blood transfusion (Answer D)
D. Intermediate is incorrect, as the patient is not having active
bleeding. Phentolamine (Answer E) is incorrect, as
Answer: D) Intermediate
it would further lower the patient’s blood pressure.
This patient has carcinoid symptoms and a
large tumor burden and will be having general
anesthesia, so she is at intermediate risk
(Answer D) of developing a carcinoid crisis. Some
providers give patients an octreotide drip (50-500
mcg/h) to prevent a carcinoid crisis during an
operation, but it is unclear if this reduces the risk
of a carcinoid crisis. Answers A and B are incorrect
because the patient has significant burden of
disease and already has carcinoid symptoms.
Answer C is incorrect because not all patients with
carcinoid symptoms will have a carcinoid crisis
during procedures.
190  ENDO 2021 • Endocrine Case Management
chased by Dr. Khalid H. Seedahmed, Jr., MRCP PGDip, CertEndocrinology SA - ID 283
References
1. Howe JR, Cardona K, Fraker DL, et al. The surgical management of small 4. Fottner C, Ferrata M, Weber MM. Hormone secreting gastro-entero-
bowel neuroendocrine tumors: consensus guidelines of the North American pancreatic neuroendocrine neoplasias (GEP-NEN): when to consider, how to
Neuroendocrine Tumor Society. Pancreas. 2017;46(6):715-731. PMID: diagnose? Rev Endocr Metab Disord. 2017;18(4):393-410. PMID: 29256148
28609357 5. Borna RM, Jahr JS, Kmiecik S, Mancuso KF, Kaye AD. Pharmacology of
2. Larouche V, Akirov A, Alshehri S, Ezzat S. Management of small bowel octreotide: clinical implications for anesthesiologists and associated risks.
neuroendocrine tumors. Cancers (Basel). 2019;11(9). Pii:E1395. PMID: Anesthesiol Clin. 2017;35(2):327-339. PMID: 28526153
31540509
3. Tapia Rico G, Li M, Pavlakis N, Cehic G, Price TJ. Prevention and
management of carcinoid crises in patients with high-risk neuroendocrine
tumours undergoing peptide receptor radionuclide therapy (PRRT): literature
review and case series from two Australian tertiary medical institutions.
Cancer Treat Rev. 2018;66:1-6. PMID: 29602040

ENDO 2021 • Neuroendocrinology and Pituitary  191

chased by Dr. Khalid H. Seedahmed, Jr., MRCP PGDip, CertEndocrinology SA - ID 283


Surveillance and Management
of Long-Term Complications
of Acromegaly
Robert D. Murray, MBBS, MD. Leeds Centre for Diabetes and Endocrinology, St James’s
University Hospital, Leeds, United Kingdom; E-mail: robertmurray@nhs.net

Learning Objectives improve most complications. Biochemical control


has less impact, however, on impaired quality
As a result of participating in this session, learners
of life and arthropathy, which remain areas for
should be able to:
improvement in management.
• Recognize the long-term complications of GH
and IGF-1 hypersecretion.
• Undertake a structured surveillance program Significance of the
for patients with acromegaly to identify Clinical Problem
complications.
Acromegaly is the clinical syndrome resulting
• Explain the importance of timely control from chronic excessive GH and IGF-1 secretion.
of GH and IGF-1 in reducing long-term In almost all cases, the cause is a GH-secreting
complications. adenoma. As a result of the low incidence (3 to 4
• Recognize those complications that still cases/106 per year) and nonspecific presentation
require addressing and mitigate their impact. early in the disease process, diagnosis is frequently
delayed. In the vast majority of individuals, the
initial management is selective hypophyseal
adenomectomy. Postoperative remission rates
Main Conclusions for microadenomas in specialist centers approach
90%. However, remission rates are significantly
In addition to an increase in mortality, patients
less for macroadenomas, particularly when there
with acromegaly have significant and diverse
is notable invasion of the cavernous sinus. In
morbidity reflecting the widespread expression
these latter cases, remission rates even in specialist
of the GH and IGF-1 receptor in multiple organ
centers are less than 50%. Most somatotroph
systems. Morbidity includes hypopituitarism,
adenomas are macroadenomas, and therefore
hypertension, diabetes mellitus, dyslipidemia,
overall remission rates postoperatively are in the
cardiovascular disease, obstructive sleep apnea,
range of 42% to 65%.1 In patients with persistent
arthropathy, vertebral fractures, carpal tunnel
disease following surgery, somatostatin analogue
syndrome, malignancy, and impaired quality of
therapy is generally favored as second-line
life. Patients require holistic assessment at time
management. Long-acting somatostatin analogue
of diagnosis, with surveillance for complications
therapy leads to control of both GH and IGF-
repeated after interventions to normalize GH
1 in 30% to 40% of patients not in remission
and IGF-1 levels and at least annually thereafter.
following surgery.2 Therefore, combined use of
Control of GH and IGF-1 levels can resolve or
surgery and somatostatin analogue therapy leads

192  ENDO 2021 • Endocrine Case Management

chased by Dr. Khalid H. Seedahmed, Jr., MRCP PGDip, CertEndocrinology SA - ID 283


to overall biochemical remission rates of around Barriers to Optimal Practice
70%. Approximately 30% of patients still require
further therapy to achieve optimal disease control. • Due to the low prevalence of acromegaly, the
It is clear, therefore, that remission in a significant diagnosis is frequently delayed by 8 to 10 years,
proportion of patients requires multimodality leading to potentially irreversible changes in
therapy (surgery, radiation therapy, and medical tissues by the time of diagnosis.
therapy) and takes time to achieve.3
• Rapid control of GH and IGF-1 excess is not
Almost all tissues express the GH and IGF-
always achievable, and many patients require
1 receptor, so there is the potential for adverse
multimodality therapy to meet biochemical
effects of excess secretion on multiple tissues and
targets.
biological systems. Acromegaly is associated with
increased mortality, which can be normalized by
control of GH and IGF-1 secretion.4,5 There is, in
addition, a notable associated long-term morbidity,
Strategies for Diagnosis,
including hypopituitarism, hypertension, diabetes Therapy, and/or Management
mellitus, dyslipidemia, cardiovascular disease, In the long-term, multimodality therapy achieves
obstructive sleep apnea, arthropathy, vertebral biochemical control of acromegaly in almost all
fractures, carpal tunnel syndrome, malignancy, individuals. Patients, however, achieve disease
and impaired quality of life.6,7 Furthermore, control at different time points during their
interventions used to achieve biochemical therapeutic journey. For example, surgery can
remission are not themselves without potential lead to immediate biochemical remission, whereas
early and late sequelae. Complications of surgery in patients who receive radiation therapy for
generally occur early, whereas those associated persistent disease, control of biochemical markers
with radiation therapy (hypopituitarism, second of disease activity can take many years to be
malignancies, vascular disease) usually take a realized. Furthermore, a number of individuals
number of years to be realized. Adverse events must remain on long-term medical therapy to
relating to medical therapy are specific to the maintain biochemical control either as secondary
medication used and add further complexity therapy after unsuccessful pituitary surgery or while
to long-term monitoring of patients with awaiting the effects of radiation therapy. Given the
acromegaly. heterogeneity in treatment regimens and degree
With the plethora of long-term complications of biochemical control, most specialist pituitary
of acromegaly, affected individuals require centers are likely to undertake surveillance on
prolonged follow-up and surveillance. Early patients with biochemical remission, GH/IGF-1
recognition and aggressive management of dichotomy, and mild biochemical disease activity
complications is essential, although management achieved after variable treatment duration.
strategies to reduce the frequency of long-term Baseline assessment of many putative
complications remain the ultimate goal. For complications should be done at the time of
a few outcomes (arthropathy, quality of life), presentation due to their impact on initial
management has had less of an impact and future management. Of most importance at diagnosis
research is needed.8,9 is assessment and management of ACTH
deficiency, hypertension, diabetes mellitus, cardiac
function, and obstructive sleep apnea. Once
acceptable GH and IGF-1 levels are achieved,
surveillance for morbidity in most individuals
is undertaken annually, with more frequent
visits when intervention to manage morbidity is

ENDO 2021 • Neuroendocrinology and Pituitary  193

chased by Dr. Khalid H. Seedahmed, Jr., MRCP PGDip, CertEndocrinology SA - ID 283


uncovered. Central to surveillance for long-term Clinical Case Vignettes
complications is a thorough history and listening
to patient concerns, followed by blood pressure Case 1
measurement and relevant laboratory tests and A 34-year-old man with acromegaly (GH =
imaging. A structured surveillance program must 24 ng/mL [24 µg/L]; IGF-1 = 2.3 times the
be personalized and should consider: upper normal limit) undergoes transsphenoidal
adenomectomy for a microadenoma and achieves
• Hypopituitarism postoperative control of GH and IGF-1.
• Blood pressure
• Vascular risk factors (lipid profile and glucose Which of the following complications
intolerance) is most likely to improve?
A. Diabetes mellitus
• Obstructive sleep apnea
B. Arthropathy
• Echocardiography
C. Vertebral fractures
• Colonoscopy
D. Sleep apnea
• Bone densitometry
Answer: A) Diabetes mellitus
• Spinal x-rays
• Joint x-rays/ultrasonography Biochemical control of acromegaly fails to
prevent progression of premature degenerative
• Thyroid ultrasonography
arthropathy (Answer B), which correlates most
• Quality of life closely to GH and IGF-1 levels at presentation.
The incidence of vertebral fractures (Answer C)
Not all of these screening tests require regular
does appear to slow with biochemical control;
repetition, and others potentially only performed
however, the incidence remains greater than
should the patient have symptoms or signs.
expected. Data on sleep apnea (Answer D) are
Whether, and when, surveillance regimens could
variable, although most studies fail to show
be discontinued or simplified needs consideration.
resolution of sleep apnea with biochemical control
This may be particularly relevant in elderly
of acromegaly. This most likely reflects that the
patients, very long-term survivors, and those who
soft-tissue changes observed in patients with
achieve remission immediately following primary
acromegaly frequently remain after achieving
pituitary surgery. Individualization of screening is
biochemical control. The excess GH secretion that
essential; however, all aspects should be considered
defines acromegaly antagonizes insulin action,
at the patient’s follow-up visits.
leading to glucose intolerance and diabetes. With
Control of GH and IGF-1 reduces the severity,
the fall in GH levels, insulin resistance and glucose
or prevalence, of most complications observed,
tolerance improve in almost all patients, with
and therefore biochemical control should be
resolution of diabetes in a number of individuals
pursued rapidly and aggressively.6,7 Exceptions
(thus, Answer A is correct).
to this rule may be sleep apnea, arthropathy, and
quality of life.8,9,10 Following biochemical control,
management of most complications of acromegaly Case 2
is not dissimilar to that of the same conditions A 46-year-old woman with a GH-secreting
when present within the general population. adenoma with persistent disease relating to
Management of many of the complications cavernous invasion fails to show GH suppression
requires multidisciplinary care, frequently with first-generation somatostatin analogues.
coordinated by the endocrinologist. Pasireotide is initiated with successful control of

194  ENDO 2021 • Endocrine Case Management

chased by Dr. Khalid H. Seedahmed, Jr., MRCP PGDip, CertEndocrinology SA - ID 283


the GH axis, but she develops diabetes mellitus as a phase, characterized by fissuring (Answer C)
complication. and degeneration of the cartilage (joint space
narrowing) (Answer D), subchondral bone cysts,
Which of the following would be and bone remodeling (osteophytosis) (Answer B).
the best class of medication to help
control glucose metabolism?
Case 4  New Content 
A. Sulfonylurea
A 54-year-old man was recently diagnosed with
B. Metformin
acromegaly. His BMI is 38.6 kg/m2. The patient
C. GLP-1 receptor agonist recounts a history of daytime sleepiness and
D. Insulin snoring suggestive of obstructive sleep apnea.
Answer: C) GLP-1 receptor agonist When during the course of his management
Pasireotide inhibits not only insulin secretion, but would it be important to assess/reassess
also gut hormones including the incretins. As a for the presence of sleep apnea?
consequence, GLP-1 receptor agonists (Answer C) A. Preoperatively
and DPP-4 inhibitors appear to be the most B. Immediately after surgery
appropriate treatment modality for pasireotide- C. If he has weight loss or weight gain
induced diabetes.
D. If his symptoms change

Case 3 Answer: A and C) Preoperatively AND


if he has weight loss or weight gain
A 58-year-old man with acromegaly who has had
control of GH and IGF-1 for 15 years presents Obstructive sleep apnea is highly prevalent in
with worsening joint pain and functional patients with acromegaly and should be identified
limitation. preoperatively (Answer A) to aid anesthetic
management. Data vary regarding improvement
Which of the following are features of sleep apnea following biochemical control
of acromegalic arthropathy? of acromegaly, with most studies failing to
A. Joint space widening show resolution. This likely reflects that the
B. Hyperosteophytosis soft-tissue changes observed in patients with
acromegaly frequently remain even after achieving
C. Fissuring of the cartilage
biochemical control. When improvement in soft
D. Joint space narrowing tissues does occur, this may take several months
E. All of the above to be appreciated. Thus, reassessing for sleep
apnea immediately after surgery (Answer B)
Answer: E) All of the above
would be premature. Similar to observations in
All of these are features of acromegalic arthropathy the general population, significant improvements
at different stages of the process (Answer E). in sleep apnea can be observed with weight
Acromegalic arthropathy appears to have 2 distinct loss (Answer C). Symptoms of sleep apnea are
phases. The initial phase is characterized by an usually insidious and therefore both development
increase in joint space (Answer A) resulting from and resolution may not be noted by the patient
hypertrophy of the cartilage and ligamentous (thus, Answer D is incorrect). Additionally, the
laxity. At this stage, the arthropathy may have a predominant symptom of sleep apnea is day-time
degree of reversibility. The inherent instability of lethargy; however, in patients with acromegaly,
such joints in the longer term leads to the second this is generally multifactorial in etiology, rather
than being specific to sleep apnea.

ENDO 2021 • Neuroendocrinology and Pituitary  195

chased by Dr. Khalid H. Seedahmed, Jr., MRCP PGDip, CertEndocrinology SA - ID 283


Case 5  New Content  Answer: D) At diagnosis and then every 5 years
until GH and IGF-1 levels are controlled
A 31-year-old woman is diagnosed with a large
GH-secreting macroadenoma with extension into Although the risk of malignancy in persons with
the cavernous sinus bilaterally. Transsphenoidal acromegaly appears to be slightly elevated based
surgery leads to a decrease in GH levels from on population studies of mortality, there remains
32 ng/mL (32 µg/L) to 1.4 ng/mL (1.4 µg/L) and significant debate as to whether this is a true
in IGF-1 levels from 3.8 times the upper normal finding and by how much any risk is elevated
limit to 1.6 times the upper normal limit. Initiation above that of the general population. The most
of a somatostatin analogue improves levels further: studied malignancies are colonic and thyroid.
GH = 0.7 ng/mL (0.7 µg/L) and IGF-1 = 1.4 times Overall, the data suggest there is an increased
the upper normal limit. The patient is awaiting prevalence of both colonic hyperplastic polyps
discussion of further treatment options. and malignancies. Therefore, recommending
no screening (Answer A) is likely inadequate.
Should this patient undergo screening for These abnormalities are present at presentation
colonic malignancies and, if so, when? and therefore screening should not wait until
A. No screening required age 50 years (Answer B), when screening is
B. First colonoscopy at age 50 years and then typically recommended for the general population.
every 5 years thereafter Screening thereafter is dependent on whether the
C. At diagnosis and then every 5 years thereafter disease remains active and findings of the previous
irrespective of control of GH and IGF-1 colonoscopy. The risk of future polyps is greater
if they are present on the initial colonoscopy.
D. At diagnosis and then every 5 years until GH
Risk also appears to be significantly abrogated
and IGF-1 levels are controlled
by control of GH and IGF-1 secretion (thus,
Answer D is correct and Answer C is incorrect).

References
1. Minniti G, Jaffrain-Rea ML, Esposito V, Santoro A, Tamburrano G, Cantore G. 6. Gadelha MR, Kasuki L, Lim DST, Fleseriu M. Systemic complications of
Evolving criteria for post-operative biochemical remission of acromegaly: can we acromegaly and the impact of the current treatment landscape: an update.
achieve a definitive cure? An audit of surgical results on a large series and a review Endocr Rev. 2019;40(1):268-332. PMID: 30184064
of the literature. Endocr Relat Cancer. 2003;10(4):611-619. PMID: 14713271 7. Giustina A, Barkan A, Beckers A, et al. A consensus on the diagnosis and
2. Murray RD, Melmed S. A critical analysis of clinically available somatostatin treatment of acromegaly comorbidities: an update. J Clin Endocrinol Metab.
analog formulations for therapy of acromegaly. J Clin Endocrinol Metab. Citation details now available: 2020;105(4):dgz096. PMID: 31606735
2008;93(8):2957-2968. PMID: 18477663 8. Claessen KM, Ramautar SR, Pereira AM, et al. Increased clinical symptoms
3. Katznelson L, Laws ER Jr, Melmed S, et al; Enodcrine Society. Acromegaly: of acromegalic arthropathy in patients with long-term disease control: a
an Endocrine Society clinical practice guideline. J Clin Endocrinol Metab. prospective follow-up study. Pituitary. 2014;17(1):44-52. PMID: 23344976
2014;99(11):3933-3951. PMID: 25356808 9. Kyriakakis N, Lynch J, Gilbey SG, Webb SM, Murray RD. Impaired quality
4. Holdaway IM, Bolland MJ, Gamble GD. A meta-analysis of the effect of of life in patients with treated acromegaly despite long-term biochemically
lowering serum levels of GH and IGF-I on mortality in acromegaly. Eur J stable disease: results from a 5-years prospective study. Clin Endocrinol (Oxf).
Endocrinol. 2008;159(2):89-95. PMID: 18524797 2017;86(6):806-815. PMID: 28316090
5. Orme SM, McNally RJ, Cartwright RA, Belchetz PE. Mortality and cancer 10. Parolin M, Dassie F, Alessio L, et al. Obstructive sleep apnea in acromegaly
incidence in acromegaly: a retrospective cohort study. United Kingdom and the effect of treatment: a systematic review and meta-analysis. J Clin
Acromegaly Study Group. J Clin Endocrinol Metab. 1998;83(8):2730-2734. Endocrinol Metab. Citations details now available: 2020;105(3):dgz116.
PMID: 9709939 PMID: 31722411

196  ENDO 2021 • Endocrine Case Management

chased by Dr. Khalid H. Seedahmed, Jr., MRCP PGDip, CertEndocrinology SA - ID 283


Dopamine Agonists: Risks,
Adverse Effects, Surveillance,
and Intervention
Mark E. Molitch, MD. Division of Endocrinology, Metabolism, and Molecular Medicine,
Northwestern University Feinberg School of Medicine, Chicago, IL; E-mail: molitch@
northwestern.edu

Learning Objectives using dopamine agonists have a risk of tumor


enlargement—2% of those with microadenomas
As a result of participating in this session, learners
and 18% of those with macroadenomas. Neither
should be able to:
drug has been shown to increase the risk of fetal
• List at least 3 risks of prescribing cabergoline malformations.
at high dosages.
• Recognize the consequences of tumor
shrinkage by dopamine agonists. Significance of the
• Explain the risks and benefits of dopamine Clinical Problem
agonist use in women who become pregnant. Prolactinomas are the most common hormone-
secreting pituitary tumors.1 Dopamine agonists
remain the treatment mainstay for patients
with prolactinomas because of their high degree
Main Conclusions of efficacy and low adverse effect profiles.2
Although dopamine agonists have been shown to Bromocriptine, in use since the mid-1970s,
be highly effective and remain the mainstay of the can normalize prolactin levels in 80% to 90%
treatment of patients with prolactinomas, their use of patients and shrink prolactinomas by more
does entail some risks and adverse effects. An unusual than 50% in about 50% of patients. Cabergoline,
adverse effect is the development of obsessive- in use since the mid-1990s, can achieve these
compulsive behavior, such as hypersexuality and goals in more than 90% of patients. Cabergoline,
gambling. This is usually seen with cabergoline. therefore, is more effective and better tolerated.
With either cabergoline or bromocriptine, shrinkage The common adverse effect of nausea can
of a large skull-based macroadenoma that invades the be lessened by starting with low dosages and
sellar floor can result in the leakage of cerebrospinal gradually increasing over weeks to months,
fluid around the now smaller tumor. Cerebrospinal monitoring the prolactin response. About 15% to
fluid rhinorrhea can be diagnosed by measuring 20% of patients are resistant to these medications,
β2-transferrin in the nasal fluid, and the leak must requiring more than standard dosing to achieve
be repaired to prevent meningitis. About 15% to prolactin normalization. Approximately 50%
20% of patients require larger than standard dosages of those resistant to bromocriptine respond to
of cabergoline to normalize prolactin levels, and cabergoline. Very high cabergoline dosages have
large dosages may cause cardiac valve abnormalities. been associated with cardiac valve abnormalities
Women who wish to become pregnant while in patients with Parkinson disease, but such valve

ENDO 2021 • Neuroendocrinology and Pituitary  197

chased by Dr. Khalid H. Seedahmed, Jr., MRCP PGDip, CertEndocrinology SA - ID 283


disease is not found in patients with prolactinomas weekly when needed to control prolactin secretion
treated with conventional dosages (≤2 mg weekly). and tumor size, but they must recognize the
The dosage at which the risk for valve disease need to perform periodic echocardiography to
increases is not known, and it is recommended monitor for cardiac valvular disease and switch to
that patients taking dosages greater than 2 mg bromocriptine or other modes of therapy if such
weekly be monitored with echocardiography. valve disease develops.
Of special concern is the woman with a Both bromocriptine and cabergoline are
prolactinoma who wishes to become pregnant. safe for the developing fetus when given
Since the dopamine agonists must be used to to women who wish to become pregnant.
allow ovulation, there is some finite period of fetal Although symptomatic monitoring is sufficient
exposure to the dopamine agonists, but fortunately for women with microadenomas because their
no adverse consequences to the mother or fetus risk for tumor enlargement is only about 2%,
have been found with such exposure. There is visual field testing each trimester in addition to
a small risk of tumor enlargement, especially symptomatic monitoring is necessary for those
macroadenomas, due to the high estrogen levels with macroadenomas, as their risk for tumor
present during pregnancy.2 enlargement is 18%.

Barriers to Optimal Practice Clinical Case Vignettes


• Adverse effects related to obsessive-
Case 1
compulsive disorders and tumor shrinkage are A 29-year-old man presents with decreased libido,
not well understood. erectile dysfunction, and headaches. His prolactin
concentration is 2904 ng/mL (126.3 nmol/L).
• The increased risks associated with more than
MRI shows a 2.4-cm macroadenoma with
conventional dosages of cabergoline are not
lateral, intrasellar, and suprasellar extension.
well understood.
His testosterone concentration is borderline-
• Clinicians often do not appreciate the low at 250 ng/dL (8.7 nmol/L), and the rest of
problems associated with dopamine agonist his pituitary function is normal. Cabergoline is
use in women with prolactinomas who initiated, and the dosage is gradually increased
become pregnant. to 3.5 mg weekly. As a result, his prolactin level
normalizes and MRI shows marked tumor
size reduction. His libido and erectile function
Strategies for Diagnosis, normalize and his headaches disappear. At a recent
Therapy, and/or Management clinic visit, his wife asks whether his cabergoline
dosage could be reduced.
The lowest dosages of dopamine agonists that
normalize prolactin levels should be used.
Which of the following adverse
Clinicians should query their patients and family
effects of cabergoline is she most
members about the possible development of likely concerned about?
obsessive-compulsive behavior.
Increased rhinorrhea can be due to A. Difficulty urinating
cerebrospinal fluid leakage. Measurement of β2- B. Hypersexuality
transferrin in the nasal mucus is diagnostic, paving C. Sleep apnea
the way for surgical repair to prevent meningitis. D. Restless legs
Endocrinologists should not hesitate to
increase dosages of cabergoline beyond 2 mg Answer: B) Hypersexuality

198  ENDO 2021 • Endocrine Case Management

chased by Dr. Khalid H. Seedahmed, Jr., MRCP PGDip, CertEndocrinology SA - ID 283


Impulse control disorders can be defined as “a Case 1 (Continued)
failure to resist an impulse, drive, or temptation
The patient’s cabergoline dosage is gradually
to perform an act that is harmful to the person or
reduced to 2 mg weekly. His prolactin
others.”3 Impulse control disorders include, but are
concentration increases to 45 ng/mL
not limited to, problem gambling, hypersexuality,
(2.0 nmol/L) after 1 month and then stabilizes
compulsive eating, compulsive shopping, and
with improvement in his hypersexuality. His
“punding.” Punding is characterized by compulsive
testosterone concentration, which had increased to
performance of and fascination with repetitive
570 ng/dL (19.8 nmol/L), remains in the reference
mechanical tasks; for example, assembling and
range (400-450 ng/mL [13.9-15.6 nmol/L]). MRI
disassembling household objects or collecting or
at 6 months shows a 50% reduction in tumor size.
sorting various items.
During treatment, he has some nasal stuffiness and
The mechanism of action behind impulse
at 7 months he notices a substantial increase in
control disorders seems to be an interaction
watery rhinorrhea.
between the dopamine agonists and the D3
receptors in the mesolimbic system, known to be Which of the following is the best
responsible for the processes governing behavior, next step in his management?
pleasure, and addiction.4 Several studies have
A. Switch to bromocriptine
shown that both cabergoline and bromocriptine
can cause compulsive behavior in 15% to 20% B. Send nasal fluid for glucose and protein
of treated patients.4,5 The effect appears to be measurements
somewhat dosage dependent, so lowering the C. Send nasal fluid for β2-transferrin
dosage may be helpful in some circumstances. measurement
In a recent cross-sectional multicenter study D. Start pseudoephedrine
in Turkey of 308 patients with prolactinomas
(289 on cabergoline, 19 on bromocriptine), 17% Answer C) Send nasal fluid for β2-
were found to have impulse control disorders transferrin measurement
(hypersexuality alone 6.5%, pathologic gambling Cerebrospinal fluid rhinorrhea can occur when
alone 0.6%, compulsive eating alone 2.9%, there is a large, invasive, skull-based prolactinoma
compulsive shopping 1%, more than 1 impulse that serves as a “cork” in the base of the skull.
control disorder 5.5%). Hypersexuality was more When the tumor size is reduced substantially
common in men, and compulsive eating was more through dopamine agonist use, cerebrospinal
common in women.6 fluid can leak around the tumor into the sphenoid
Clinicians should warn patients and their sinus and nasal passages. The development
significant others or family members about this of profuse rhinorrhea in this patient suggests
potential adverse effect when first prescribing that this is what is happening.7 To distinguish
dopamine agonists, but they should also ask about between cerebrospinal fluid and simple nasal
impulse control disorders at subsequent visits. mucus, the fluid should be sent to the laboratory
It is important to also ask the spouse, as some for measurement of β2-transferrin (Answer C),
impulse control disorders, such as gambling and which is an asialo-transferrin isoform found only
compulsive shopping, may be hidden from the in cerebrospinal fluid, ocular fluids, and perilymph
family. and an accepted marker of cerebrospinal fluid
leakage.8 Measurements of nasal fluid glucose and
protein levels (Answer B) are not specific. β-trace
protein (prostaglandin D synthase) can also be
used to distinguish between cerebrospinal fluid
and nasal mucus.

ENDO 2021 • Neuroendocrinology and Pituitary  199

chased by Dr. Khalid H. Seedahmed, Jr., MRCP PGDip, CertEndocrinology SA - ID 283


The major concern with such leaks is the risk (19.0 nmol/L), with some tumor shrinkage but no
of meningitis. Although a reduction in cabergoline beginning of menses.
dosage may cause the tumor to get larger, thus
plugging the leak, judging the exact dosage and Which of the following is the best next
tumor size change is difficult and not reliable step in this patient’s management?
and surgery is the best course once the diagnosis A. Perform transsphenoidal debulking
is made. Endonasal, endoscopic surgical repair B. Switch to bromocriptine
(urgent, but not emergent) to prevent meningitis
C. Increase the cabergoline dosage
generally recommended.7,9 Use of prophylactic
antibiotics pending surgery is controversial. D. Perform stereotactic radiosurgery
Lumbar drainage is usually not successful.7,9 Answer: C) Increase the cabergoline dosage
Switching to bromocriptine (Answer A) would not
help. Pseudoephedrine (Answer D) may decrease Fifteen to twenty percent of patients with
nasal congestion but will not affect cerebrospinal prolactinomas do not achieve normal prolactin
fluid leakage. levels with conventional dosages of cabergoline
(≤2 mg weekly).10,11 For such patients, there are
a number of alternatives. Occasionally, patients
Case 2
who do not respond to cabergoline do respond
An 18-year-old woman presents with primary to bromocriptine, but the numbers reported
amenorrhea but otherwise normal prior growth are too low to determine how common this
and development. She has also been experiencing is. Conversely, about 50% of patients who do
headaches and decreased visual acuity. not respond to bromocriptine do respond to
On physical examination, her height is 61 in cabergoline. However, most patients who have
(154.9 cm) and weight is 140 lb (63.6 kg) (BMI = some response to cabergoline will have a further
26.4 kg/m2). Blood pressure is normal. She has response if the dosage is increased. Dosage
galactorrhea with Tanner stage 3 breast and pubic increases should be done in a stepwise fashion,
hair development. documenting a reduction in prolactin levels
with each dosage increase. The major issue with
Laboratory test results:
increasing dosage is an increase in adverse drug
Karyotype, 46,XX effects. Studies in patients with Parkinson disease,
LH = 3.8 mIU/mL (SI: 3.8 IU/L) who may be treated with 3 to 5 mg daily, show
FSH = <2.0 mIU/mL (SI: <2.0 IU/L)
Estradiol = 34 pg/mL (SI: 124.8 pmol/L)
that adverse effects are uncommon.11
Free T4 = 0.8 mIU/L (0.8-1.8 ng/dL) The major concern has been the finding of
(SI: 10.30 pmol/L [10.30-23.17 pmol/L]) cardiac valve disease with these high cabergoline
Cortisol (8 AM) = 14 µg/dL (5-25 µg/dL) dosages in as many as 34% of patients with
(SI: 386.2 nmol/L [137.9-689.7 nmol/L]) Parkinson disease, with tricuspid regurgitation
Pregnancy test, negative
being the abnormality. Valve abnormalities
Her prolactin concentration is are not seen with bromocriptine. It is thought
29 ng/mL (1.3 nmol/L). On 1:100 dilution, it is that the cabergoline has action at the serotonin
12,840 ng/mL (558.3 nmol/L). MRI shows a 3.4- (5-HT) 2B receptors, which are present in
cm pituitary adenoma. valves, thereby activating a variety of mitogenic
Cabergoline is initiated at a dosage of pathways. This results in a plaquelike process
0.5 mg twice weekly, and the dosage is increased that extends along the leaflet surfaces and
at monthly intervals. On a dosage of 1 mg chordae tendineae and is similar to the lesions
twice weekly (the package insert maximum), seen in patients with carcinoid syndrome.12
her prolactin concentration is 438 ng/mL Almost all echocardiographic studies have not

200  ENDO 2021 • Endocrine Case Management

chased by Dr. Khalid H. Seedahmed, Jr., MRCP PGDip, CertEndocrinology SA - ID 283


shown an increase in valve abnormalities in Neither cabergoline nor bromocriptine should
hyperprolactinemic patients taking cabergoline be continued once the pregnancy is diagnosed,
compared with control patients when dosages are so as to limit the exposure of the developing
kept to a maximum of 2 mg weekly.13 However, fetus to either drug. The dopamine agonist
given the Parkinson disease data, there clearly should be stopped once pregnancy is diagnosed
is an increased risk with higher dosages, but (Answer C).14,15 When these drugs are stopped
exactly where that threshold is for increased within the first few weeks of pregnancy, the
risk is not known. Because of this, it has been frequency of major malformations is not greater
recommended that hyperprolactinemic patients than would be expected in the general population,
receiving greater than 2 mg weekly of cabergoline and an elective termination is not justified. The
have echocardiography performed when that databases establishing such safety are greater for
dosage threshold is crossed with then annually bromocriptine (5000 to 6000) than for cabergoline
thereafter.10 Limited data in patients with (900 to 1000),14 so some clinicians prefer
Parkinson disease show that if valve abnormalities bromocriptine to allow ovulation when women
develop, one-third have regression and half wish to become pregnant. Many clinicians think
have lack of further progression of valve disease that the cabergoline database is sufficiently large to
if the drug is stopped. However, if the drug is feel comfortable with the safety aspect. A concern
continued about 15% have progression of valve about stopping the dopamine agonist is the risk
abnormalities. of tumor enlargement. The chance of clinically
significant tumor growth related to the pregnancy
Case 3 is about 18% for patients with macroadenomas
and 2.5% for those with microadenomas.14 The
A 42-year-old physician who was treated 20 years data regarding safety for using dopamine agonists
ago for a 2-cm macroprolactinoma calls with a throughout pregnancy and, in this case, after
question. Cabergoline shrunk the prolactinoma, 20 weeks, are sparse. However, organogenesis
her prolactin levels normalized, and ovulatory is usually complete by 8 weeks’ gestation, so no
menses returned. She had tried going off additional malformations would be expected.
cabergoline several times over the years without A recent compilation of the literature does not
maintaining normal prolactin levels. Now she is suggest any late complications either.14 However,
20 weeks pregnant and asks whether she should Hurault-Delarue et al reported an increased risk
continue cabergoline. She notes that she feels of preterm birth and early pregnancy loss with
much better when taking cabergoline than when dopamine agonist use.16
she does not.

Which of the following is the best next Case 4  New Content 


step in this patient’s management? A 61-year-old man with a prolactin-secreting
A. Switch cabergoline to bromocriptine and macroadenoma has been treated with cabergoline,
continue throughout pregnancy 1.0 mg twice weekly, for 2 years. His prolactin
B. Continue cabergoline throughout the concentration has decreased from 2104 ng/mL
pregnancy (91.5 nmol/L) to 11 ng/mL (0.5 nmol/L), and
his adenoma has decreased in size from 2.3 cm
C. Stop cabergoline now
to 1.1 cm in maximum diameter. At his most
D. Stop cabergoline now but restart if she starts recent visit, he asks whether he has to take this
feeling badly medication forever.
Answer: C) Stop cabergoline

ENDO 2021 • Neuroendocrinology and Pituitary  201

chased by Dr. Khalid H. Seedahmed, Jr., MRCP PGDip, CertEndocrinology SA - ID 283


Which of the following courses of may be achieved in a significant proportion of
management should be tried in this patient? patients, even without irradiation (Answer D).
A. Stop cabergoline and repeat the prolactin
measurement and MRI in 3 months Case 5  New Content 
B. Taper the cabergoline dosage to 0.5 mg per A 35-year-old man presents with erectile
week and then stop it and measure prolactin in dysfunction and decreased libido. He has no
1 month headaches and otherwise feels well. Physical
C. Continue cabergoline because there is still examination findings are normal.
residual tumor visible
Laboratory test results:
D. Administer irradiation to the residual
tumor via stereotactic radiosurgery; then Testosterone = 153 ng/dL (300-900 ng/dL)
taper the cabergoline dosage over 6 months (SI: 5.3 nmol/L [10.4-31.2 nmol/L])
before stopping it and repeating prolactin LH = 1.3 mIU/mL (1.0-9.0 mIU/mL)
(SI: 1.3 IU/L [1.0-9.0 IU/L])
measurement and MRI 3 months later FSH = 1.1 mIU/mL (1.0-13.0 mIU/mL)
(SI: 1.1 IU/L [1.0-13.0 IU/L])
Answer: B) Taper the cabergoline dosage to 0.5 mg per Prolactin = 58.2 ng/mL (4-23 ng/mL)
week and then stop it and measure prolactin in 1 month (SI: 2.53 nmol/L [0.17-1.00 nmol/L])
The best management option is to taper the The rest of his pituitary function is normal. MRI
cabergoline dosage to 0.5 mg per week and then shows a large macroadenoma wrapped around
try to stop it (Answer B). In 2003, Dr. Colao the left internal carotid artery with minimal
and colleagues showed that 42% of patients with suprasellar extension. Because of the tumor’s large
macroprolactinomas whose tumors had decreased size, prolactin is measured again at 1:100 dilution
in size by more than 50% and who had normal and is still only minimally elevated. The tumor is
prolactin levels could maintain normal prolactin thought to be a nonfunctioning pituitary adenoma.
levels and experience no tumor reenlargement Transsphenoidal surgery results in incomplete
after tapering and withdrawal of cabergoline.17 In tumor removal with a substantial tumor remnant.
2018, a meta-analysis of 24 studies including 1106
patients found that the following factors were Which of the following is the most
more conducive of persistent normoprolactinemia likely result for immunostaining
after dopamine agonist withdrawal: cabergoline of the tumor specimen?
rather than bromocriptine; >2 years rather than A. Positive stain for α subunit
<2 years of treatment, and any decrease in tumor
B. Positive stain for prolactin
size rather than no decrease in tumor size.18
Prolactin should be monitored monthly after C. Positive stain for ACTH
dopamine agonist withdrawal rather than every D. Negative stain for all pituitary hormones
3 months, and if the prolactin concentration
Answer: A) Positive stain for α subunit
remains normal, there is no need to repeat the
MRI (Answer A). Even if tumor is still present This tumor specimen is most likely to stain
with no tumor decrease, the success rate for positively for α subunit (Answer A). Between 40%
persistent normoprolactinemia is still 27.1%, and and 75% of clinically nonfunctioning adenomas
if there is any decrease in tumor size, it is 40.5%,18 are actually gonadotroph adenomas, based on the
so cabergoline does not necessarily need to be finding that patients with such tumors produce
continued (Answer C). Success in maintaining intact gonadotropins or their glycoprotein
normoprolactinemia when off dopamine agonists subunits (α and/or β) in vivo or in vitro as
shown by measurement of these products in the

202  ENDO 2021 • Endocrine Case Management

chased by Dr. Khalid H. Seedahmed, Jr., MRCP PGDip, CertEndocrinology SA - ID 283


blood, staining on immunohistochemistry, or A negative stain for all pituitary hormones
measurement of the mRNA for these products in (Answer D) is uncommon.20,21
the tumors.19-21 Rarely, clinically nonfunctioning In this particular case, the patient’s tumor
tumors stain positively for ACTH (Answer C), stained positively not for gonadotropins but for
GH, prolactin (Answer B), or TSH, but they do prolactin, so he had a “silent” lactotroph adenoma.
not secrete these hormones in sufficient quantities Although there are no data in the literature to
so as to cause clinical syndromes. Such tumors are determine whether such tumors respond to
referred to as “silent” corticotroph, somatotroph, dopamine agonists, this patient has now been
lactotroph, or thyrotroph adenomas20,21 and are treated with cabergoline for many years and has
much less common than gonadotroph adenomas. had stabilization and no regrowth of residual
tumor.

References
1. Molitch ME. Diagnosis and treatment of pituitary adenomas. JAMA. 12. Roth BL. Drugs and valvular heart disease. N Engl J Med. 2007;356(1):6-9.
2017;317(5):516-524. PMID: 28170483 PMID: 17202450
2. Gillam MP, Molitch MP, Lombardi G, Colao A. Advances in the treatment of 13. Stiles CE, Tetteh-Wayoe ET, Bestwick J, Steeds RP, Drake WM. A meta-
prolactinomas. Endocr Rev. 2006;27(5):485-534. PMID: 16705142 analysis of the prevalence of cardiac valvulopathy in hyperprolactinemic
3. American Psychiatric Association. Diagnostic and Statistical Manual of Mental patients treated with cabergoline. J Clin Endocrinol Metab 2019;104(2):523-538.
Disorders—Text Revision (DSM-IV-TR). Washington, DC: American Psychiatric 14. Huang W, Molitch ME. Pituitary tumors in pregnancy. Endocrinol Metab Clin
Association; 2000. N Am. 2019;48(3):569-581. PMID: 31345524
4. Noronha S, Stokes V, Karavitaki N, Grossman A. Treating prolactinomas 15. Glezer A, Bronstein MD. Prolactinomas, cabergoline, and pregnancy.
with dopamine agonists: always worth the gamble? Endocrine. 2016;51(2):205- Endocrine. 2014;46(1):64-69. PMID: 24985062
210. PMID: 26336835 16. Hurault-Delarue C, Montastruc JL, Beau AB, Lacroix I, Damase-Michel
5. Bancos I, Nannenga MR, Bostwick JM, Silber MH, Erickson D, Nippoldt C. Pregnancy outcome in women exposed to dopamine agonists during
TB. Impulse control disorders in patients with dopamine agonist-treated pregnancy: a pharmacoepidemiology study in EFEMERIS database. Arch
prolactinomas and nonfunctioning pituitary adenomas: a case-control study. Gynecol Obstet. 2014;290(2):263-270. PMID: 24664257
Clin Endocrinol (Oxf). 2014;80(6):863-868. PMID: 24274365 17. Colao A, Di Sarno A, Cappabianca P, Di Somma C, Pivonello R, Lombardi G.
6. Dogansen SC, Cikrikcili U, Oruk G, et al. Dopamine agonist-induced impulse Withdrawal of long-term cabergoline therapy for tumoral and nontumoral
control disorders in patients with prolactinoma: a cross-sectional multicenter hyperprolactinemia. N Engl J Med. 2003;349(21):2023-2033. PMID: 14627787
study. J Clin Endocrinol Metab. 2019;104(7):2527-2534. PMID: 30848825 18. Xia MY, Lou XH, Lin SJ, Wu ZB. Optimal timing of dopamine agonist
7. Suliman SG, Gurlek A, Byrne JV, et al. Nonsurgical cerebrospinal fluid withdrawal in patients with hyperprolactinemia: a systematic review and
rhinorrhea in invasive macroprolactinoma: incidence, radiological, and meta-analysis. Endocrine. 2018;59(1):50-61. PMID: 29043560
clinicopathological features. J Clin Endocrinol Metab. 2007;92(10):3829-3835. 19. Young WF, Scheithauer BW, Kovacs KT, Horvath E, Davis DH, Randall RV.
PMID: 17623759 Gonadotroph adenoma of the pituitary gland: a clinicopathologic analysis of
8. Warnecke A, Averbeck T, Wurster U, Harmening M, Lenarz T, Stover T. 100 cases. Mayo Clin Proc. 1996;71(7):649-656. PMID: 8656706
Diagnostic relevance of beta2-transferrin for the detection of cerebrospinal 20. Yamada S, Kovacs K, Horvath E, Aiba T. Morphological study of clinically
fluid fistulas. Arch Otolaryngol Head Neck Surg. 2004;130(10):1178-1184. nonsecreting pituitary adenomas in patients under 40 years of age. J
PMID: 15492165 Neurosurg. 1991;75(6):902-905. PMID: 1941118
9. Lam G, Mehta V, Zada G. Spontaneous and medically induced cerebrospinal 21. Saeger W, Lüdecke DK, Buchfelder M, Fahlbusch R, Quabbe H-J, Petersenn
fluid leakage in the setting of pituitary adenomas: review of the literature. S. Pathohistological classification of pituitary tumors: 10 years of experience
Neurosurg Focus. 2012;32(6):E2. PMID: 22655691 with the German Pituitary Tumor Registry. Eur J Endocrinol. 2007;156(2):203-
10. Molitch ME. Management of medically refractory prolactinoma. J Neurooncol. 216. PMID: 17287410
2014;117(3):421-428. PMID: 24146188
11. Ono M, Miki N, Kawamata T, et al. Prospective study of high-dose
cabergoline treatment of prolactinomas in 150 patients. J Clin Endocrinol
Metab. 2008;93(12):4721-4727. PMID: 18812485

ENDO 2021 • Neuroendocrinology and Pituitary  203

chased by Dr. Khalid H. Seedahmed, Jr., MRCP PGDip, CertEndocrinology SA - ID 283


Treatment of Functioning
Gonadotroph Pituitary
Adenomas
Philippe Chanson, MD. Endocrinology and Reproductive Diseases, Reference Center for
Rare Pituitary Diseases, Bicêtre Hospital, and University Paris-Saclay, Le Kremlin-Bicêtre,
France; E-mail: philippe.chanson@bct.aphp.fr

Learning Objectives ovary syndrome). In most cases of OHSS, estradiol


levels are high or very high.
As a result of participating in this session, learners
Surgical removal of the pituitary adenoma is
should be able to:
the treatment of choice, as it leads to a rapid fall
• Diagnose functioning pituitary gonadotroph in FSH levels and thus, in case of OHSS, reversal
adenomas. of ovarian hyperstimulation that is rapidly
associated with a diminution of estradiol levels
• Treat gonadotroph pituitary adenomas.
and regression of ovarian cysts. Medical treatment
is limited to cabergoline, which in some cases has
been reported decrease FSH and estradiol levels.
However, its effect is generally transient and
Main Conclusions surgery must not be delayed. GnRH agonists are
More than 80% of clinically nonfunctioning not recommended, as they may be responsible for
pituitary adenomas are gonadotroph adenomas. pituitary apoplexy.
Very rarely, gonadotroph adenomas may be
clinically functioning. They preferentially secrete
FSH and are thus associated with unusual clinical
pictures. In women of reproductive age, such
Significance of the
functioning adenomas may induce spontaneous Clinical Problem
ovarian hyperstimulation syndrome (OHSS) Aside from prolactinomas (responsible for
with multiple large ovarian cysts, generally hyperprolactinemia), somatotropinomas
discovered on pelvic ultrasonography performed (acromegaly), corticotropinomas (Cushing
for menstrual disturbances or abdominal or disease), and thyrotropinomas (central
pelvic pain. In affected men, macroorchidism has hyperthyroidism), gonadotropinomas (or
been described, while children may present with gonadotroph adenomas) are generally clinically
precocious puberty. Mass effects related to the nonfunctioning and are discovered incidentally
pituitary adenoma may also occur. or because of mass effects. However, few of
In general, FSH is the gonadotropin that is them are functioning with excess production
oversecreted. In the case of OHSS, serum FSH levels of gonadotropins that stimulate the gonads,
are often increased, but they may be in the normal leading to clinical phenotypes such as OHSS in
range, while serum LH levels are invariably low (in premenopausal women, macroorchidism in men,
contrast to the gonadotropin pattern of polycystic and precocious puberty in children.

204  ENDO 2021 • Endocrine Case Management

chased by Dr. Khalid H. Seedahmed, Jr., MRCP PGDip, CertEndocrinology SA - ID 283


Barriers to Optimal Practice differentiation and availability of cytogenetic
markers, we now know that some null cell
Barriers to optimal practice include lack of
adenomas express steroidogenic factor 1 (SF-1),
knowledge about this potential diagnosis due to
a transcription factor specific to the gonadotroph
its rarity and lack of familiarity among physicians,
lineage, implying that they also belong to the
leading to failure to recognize clinical phenotypes
gonadotroph family.5
associated with gonadotroph adenomas. There
are often delays in diagnosis and neurosurgical
treatment of functioning gonadotroph adenomas. Gonadal Hyperstimulation
In the case of gonadotroph adenomas, it remains
Strategies for Diagnosis, largely unknown why the gonadotropins are
seldom secreted in the bloodstream and, when
Therapy, and/or Management they are, why they are exceptionally responsible
Background for clinical syndromes related to gonadotropin
Clinically nonfunctioning pituitary adenomas excess (functioning gonadotroph adenomas)
include all pituitary adenomas that are not but are much more likely to be associated with
hormonally active and are therefore not associated hypogonadism. Chromatofocusing analysis has
with clinical syndromes such as amenorrhea or shown that gonadotroph adenomas produce
galactorrhea (prolactinomas), acromegaly, Cushing more basic FSH isoforms,6 which, paradoxically,
disease, or hyperthyroidism (TSH-secreting are considered to be more biologically active.
adenomas). However, most such nonfunctioning As glycosylation of gonadotropins is essential
pituitary adenomas, which are “chromophobic” on for their biological activity, hypogonadism may
classic histology, in fact secrete gonadotropins or are thus be related to decreased biological activity of
actually gonadotroph pituitary adenomas as assessed gonadotropins related to abnormal glycosylation
by immunocytochemistry.1,2 Very rarely these of the isoforms produced by the adenoma.
gonadotroph adenomas are clinically functioning,
and the hypersecretion of gonadotropins are Epidemiology
responsible for stimulation of the gonads, The prevalence of pituitary adenomas is 80
particularly in women who may develop OHSS. to 100 per 100,000 persons, and 15% to 30%
The first case was described in 1995.3 of these adenomas are nonfunctioning.7-9 On
immunocytochemistry, greater than 80% of
Pathology them are gonadotroph. While nonfunctioning
The cells of gonadotroph adenomas are well gonadotroph adenomas are generally diagnosed
delineated, oval or polyhedral, and generally during the fifth or sixth decade of life and show
show no signs of secretion (small nuclei without a male predominance, functioning gonadotroph
nucleoli).4 The percentage of cells positive for adenomas are more often described in women of
gonadotropin antibodies ranges from 100% to a reproductive age (very rarely in men).10,11 They
few islands, but it is usually low (<20%-30%). Some represent a minority (3%-13%) of nonfunctioning
tumors contain cells positive for β-FSH, β-LH, and pituitary adenomas/gonadotroph adenomas.12,13
α-subunit, while others contain cells that are only
positive for β-FSH or, more rarely, β-LH or α-SU. Clinical Presentation
Previously, when less than 5% of the adenomatous Clinically nonfunctioning gonadotroph adenomas
cells immunostained positively, the tumor was are mostly revealed by mass effect (or incidentally),
considered to be a “null cell adenoma.” However, while functioning gonadotroph adenomas, by
thanks to improved knowledge of pituitary cell

ENDO 2021 • Neuroendocrinology and Pituitary  205

chased by Dr. Khalid H. Seedahmed, Jr., MRCP PGDip, CertEndocrinology SA - ID 283


definition, present with symptoms and signs of which are associated with much higher prolactin
gonad hyperstimulation. levels, proportional to tumor size.21

Hyperstimulation Syndromes Pituitary Apoplexy


Gonadotropin hypersecretion by the adenoma Pituitary apoplexy can be the presenting feature of
may stimulate the gonads and lead to OHSS in gonadotroph adenomas, with severe headaches of
premenopausal women with FSH-secreting sudden onset, meningismus, a variably depressed
tumors, macroorchidism in men, and precocious sensorium, and visual disturbances.22 Apoplexy
puberty in children. Spontaneous OHSS is may be triggered by stimulation tests or injection
the most typical picture in premenopausal of GnRH analogues.
women.14 Generally identified at time of
ovarian ultrasonography performed for Biochemical Evaluation
menstrual disorders such as amenorrhea,
oligomenorrhea, or irregular menses, it can Gonadotropin Secretion
also be revealed after ultrasonography is Baseline plasma dimeric FSH and/or LH levels are
performed to evaluate abdominal and pelvic rarely elevated in patients with nonfunctioning
pain or bloating. Ultrasonography shows the pituitary gonadotroph adenomas23,24 and
presence of multiple ovarian cysts, similar to elevation of free subunit levels (mainly α-LH,
those observed in OHSS in women undertaking more rarely β-LH) is more common but is
assisted reproductive technology using ovulation generally moderate. In patients with functional
induction. Ovarian MRI may demonstrate large gonadotroph adenomas, the gonadotropin
(>5 cm), multiseptated cysts. Sometimes OHSS oversecreted is generally FSH. FSH levels are
is complicated by adnexal torsion.11 In men, increased in more than half of OHSS cases due to
macroorchidism has been described in very few gonadotroph adenomas, but they can be also in
cases.6,15,16 Isosexual precocious puberty has also the normal range (in the other half). However,
very rarely been described in both sexes.17-20 LH levels are invariably low.10,14,25 This is the
opposite gonadotropin pattern of that observed in
Mass Effects polycystic ovary syndrome. In most cases of OHSS,
Most functioning gonadotroph adenomas are estradiol levels are high or very high.10,14,25 Free
macroadenomas. As in the case of nonfunctioning α-subunit levels may be increased or normal. In
gonadotroph pituitary adenomas,1 mass effects men, FSH levels are also generally increased and
on anatomic structures in the vicinity of the LH and testosterone levels are decreased.
pituitary (headache, optic chiasm compression, The hormonal pattern of increased FSH
occasional compression of cranial nerves III, IV, contrasting with low LH levels and generally low
and VI) and/or on pituitary hormonal function, levels of all pituitary hormones in postmenopausal
lead to hypopituitarism. Hypopituitarism can women can help with the preoperative diagnosis
be caused by anterior pituitary compression, of a gonadotroph adenoma.23
pituitary stalk interruption, or hypothalamic It was previously recommended to measure the
involvement. Pituitary stalk compression can also response of gonadotropins and their free subunits
produce hyperprolactinemia by disinhibiting the to thyrotropin-releasing hormone and GnRH
dopaminergic tone that normally acts at the level stimulation, but these tests are neither sensitive
of pituitary lactotrophs, causing amenorrhea and nor specific for indicating the gonadotroph nature
galactorrhea, but prolactin serum levels are always of nonfunctioning pituitary adenomas.24 Although
below 150 to 200 ng/mL (<6.5-8.7 nmol/L). This rare, stimulation tests can also trigger pituitary
distinguishes them from macroprolactinomas, apoplexy,22 so they are no longer recommended.24

206  ENDO 2021 • Endocrine Case Management

chased by Dr. Khalid H. Seedahmed, Jr., MRCP PGDip, CertEndocrinology SA - ID 283


Other Pituitary Hormones Clinical Case Vignettes
Patients with pituitary macroadenomas, whether Case 1
discovered fortuitously or revealed by a mass
A 29-year-old woman has a history of menstrual
effect, require assessment of secretion of the
irregularity thought to be related to polycystic
various pituitary hormones to detect deficiencies
ovary syndrome. A moderate increase in prolactin
that may require preoperative replacement
(40 ng/mL [1.7 nmol/L]) has prompted a
therapy.
pituitary MRI, which shows a 23-mm pituitary
macroadenoma without optic chiasm compression
Treatment or visual field or acuity defects (see images).
Surgical Treatment
Surgical removal of the culprit pituitary adenoma
is the treatment of choice, as not only it treats the
mass effect, but it also rapidly produces a fall in
FSH levels and thus, in the case of OHSS, decreases
ovarian stimulation, which is associated with rapid
diminution of estradiol levels and regression of
ovarian cysts, leading to clinical improvement.14
This is achieved in a few days postoperatively.
Pregnancy has been achieved shortly after the
operation in a few cases, emphasizing the excellent Pituitary MRI. T2-weighted coronal section.

gonadal outcome after treatment of a functioning


gonadotroph adenoma. If necessary, large invasive
or even aggressive tumors that cannot be totally
surgically removed may be treated by adjuvant
radiotherapy or radiosurgery.26 The occurrence
of pituitary apoplexy may also prompt surgical
removal of the lesion, particularly if there is visual
field and/or acuity impairment or a reduced level
of consciousness.
Pituitary MRI. T1-weighted sagittal section.
Medical Management
Medical treatment currently available for Surgical resection is planned, but the patient then
functional pituitary gonadotroph adenomas is presents with progressive abdominal and pelvic
limited to cabergoline, which, in some cases, pain. Pelvic ultrasonography shows multiple
may decrease FSH and estradiol levels and allow ovarian cysts.
pregnancy.27,28 However, the effect is generally
transient and surgery must not be delayed. GnRH Laboratory test results:
agonists are not recommended, as they may be Total testosterone = 8.6 ng/dL (8.6-86.5 ng/dL)
responsible for pituitary apoplexy.22,29 (SI: 0.3 nmol/L [0.3-3.0 nmol/L])
Estradiol = 3241.6 pg/mL (<163 pg/mL)
(SI: 11,900 pmol/L [<600 pmol/L])
LH = <0.1 mIU/mL (0.7-5.6 mIU/mL) (SI: <0.1 IU/L
[0.7-5.6 IU/L])
FSH = 11.6 mIU/mL (2.0-10.0 mIU/mL)
(SI: 11.6 IU/L (2.0-10.0 IU/L)

ENDO 2021 • Neuroendocrinology and Pituitary  207

chased by Dr. Khalid H. Seedahmed, Jr., MRCP PGDip, CertEndocrinology SA - ID 283


Three days after the workup, she is admitted Answer: B) OHSS related to FSH-secreting adenoma
to the hospital for severe acute pelvic pain.
Pelvic MRI reveals the presence of large ovarian While OHSS is a well-known complication of
cysts and suspected right ovarian torsion (see assisted reproductive procedures, particularly with
images). Bilateral adnexal detorsion is performed gonadotropin stimulation, spontaneous OHSS is a
laparoscopically, resulting in a favorable outcome. very unusual occurrence and its presence suggests
the diagnosis of an FSH-secreting adenoma
(Answer B), particularly when FSH levels are
increased. The finding of OHSS should lead to
pituitary MRI, even in the absence of pituitary
mass effects. MRI allows for the diagnosis of a
pituitary adenoma. It is important to remember
that the size of the adenoma may be quite variable.
With microadenomas, mass effects may not be
present at time of diagnosis. Macroadenomas can
vary in size and invasion has been described.
Menstrual disturbances, infertility, and
Pelvic MRI. T2-weighted sagittal section. multiple ovarian cysts may suggest polycystic
ovary syndrome (Answer A). However, multiple
follicles in women with polycystic ovary syndrome
are much smaller (<10 mm) and the hormonal
pattern is quite different in terms of LH and FSH
levels (LH is generally increased with an increased
LH-to-FSH ratio). Moreover, polycystic ovary
syndrome is often associated with
hyperandrogenism, but not with such high
estradiol levels. Such large ovarian cysts with very
high estradiol levels and low testosterone levels
Pelvic MRI. T2-weighted coronal section.
may be encountered in patients with McCune-
Albright syndrome (Answer C), but these patients
generally have a history of precocious puberty and
often display other manifestations of the syndrome
(café-au-lait skin lesions, fibrous dysplasia, or even
acromegaly). Moreover, their gonadotropins are
low, suppressed by the very high levels of
estrogens autonomously produced by the ovarian
cysts (related to the activating gsp mutation).
Ovarian granulosa-cell tumors (Answer D) often
Pelvic MRI. T2-weighted axial section. present at the time of premenopause or early
menopause and, if associated with high estradiol
Which of the following is this levels, also show suppressed levels of
patient’s most likely diagnosis? gonadotropins.
A. Polycystic ovary syndrome In this case, treatment with cabergoline
was initiated 7 days before pituitary surgery.
B. OHSS related to FSH-secreting adenoma
The patient then underwent endoscopic
C. McCune-Albright syndrome transsphenoidal removal of the pituitary
D. Ovarian granulosa-cell tumor adenoma. The outcome was excellent. Pathologic

208  ENDO 2021 • Endocrine Case Management

chased by Dr. Khalid H. Seedahmed, Jr., MRCP PGDip, CertEndocrinology SA - ID 283


Analyte Preoperative Day 3 Postoperative Reference Ranges

FSH 11.6 mIU/mL (SI: 11.6 IU/L) 1.4 mIU/mL (SI: 1.4 IU/L) 2.0-10.0 mIU/mL (SI: 2.0-10.0 IU/L)

LH <0.1 mIU/mL (SI: <0.1 IU/L) 0.6 mIU/mL (SI: 0.6 IU/L) 0.7-5.6 mIU/mL (SI: 0.7-5.6 IU/L)

Estradiol 3242 pg/mL (SI: 11,900 pmol/L) 98 pg/mL (SI: 360 pmol/L) <163 pg/mL (SI: <600 pmol/L)

Prolactin 79.4 ng/mL (SI: 3.5 nmol/L) 5.6 ng/mL (SI: 0.2 nmol/L) 4.8-23.4 ng/mL (0.2-1.0 nmol/L)

examination of the adenoma displayed FSH Case 2


immunostaining in 40% of the cells and diffuse
A 78-year-old man underwent 2 operations
expression of chromogranin A. Ki67 was positive
for prostate cancer, 7 years and 1 year ago,
in 2% of the cells. The course of gonadotropins
after which hormonal therapy was decided. A
and estradiol postoperatively is depicted below.
course of antiandrogen treatment (cyproterone
After 3 months, her hormonal workup was normal
acetate 200 mg/day) is started 1 month before
and the ovarian cysts had disappeared. No residual
the injection of the GnRH agonist. The long-
tumor was observed on MRI. Five months later,
acting GnRH agonist DTrp6-GnRH (triptorelin)
the patient achieved a spontaneous pregnancy,
(375 mg) is then administered intramuscularly.
which was uneventful, and the long-term outcome
A few minutes after the injection, the patient
was favorable.
complains of increasing retroorbital headache,
This case illustrates that OHSS can have severe
which is followed 24 hours later by the
consequences, which are fortunately rare. In
occurrence of postural dizziness and left partial
addition to hypovolemia and thromboembolism,
ophthalmoplegia caused by a third-nerve lesion.
OHSS can be complicated by ovarian torsion, as
Visual field testing is normal. CT and MRI of
seen in this case, justifying surgical detorsion. This
the skull disclose the presence of a large pituitary
case also illustrates that the treatment of choice is
mass with suprasellar extension and lateral
surgery and that it must not be delayed. Correction
invasion of cavernous sinuses, particularly on
of the FSH excess allows for rapid resolution of
the left side (see images). Cyproterone acetate and
ovarian cysts and pituitary mass effect if present.
GnRH agonist treatments are discontinued.

Pituitary MRI. T1-weighted sagittal and coronal sections following administration of gadolinium-DTPA,
demonstrating a large, hyperintense pituitary adenoma with suprasellar, infrasellar, and laterosellar extension.
Reproduced with permission from Chanson P, Schaison G. Pituitary apoplexy caused by GnRH-agonist
treatment revealing gonadotroph adenoma. J Clin Endocrinol Metab. 1995;80(7):2267-2268.29

ENDO 2021 • Neuroendocrinology and Pituitary  209

chased by Dr. Khalid H. Seedahmed, Jr., MRCP PGDip, CertEndocrinology SA - ID 283


In the following days, clinical symptoms Pituitary apoplexy, a rare complication of pituitary
progressively improve. Because of the absence adenomas, is characterized by sudden headache,
of visual field alteration and the spontaneous vision impairment, and ophthalmoplegia and
improvement of ophthalmoplegia, conservative is often associated with hemorrhagic infarction
management of the pituitary tumor is pursued. of the tumor. Usually, pituitary apoplexy occurs
Two months after the interruption of hormonal spontaneously and may thus reveal the adenoma.
treatment, hormonal testing shows the following: However, there have been occasional reports of
pituitary apoplexy following administration of
Total testosterone = 47.8 ng/dL (296.8-798.2 ng/dL)
(SI: 1.66 nmol/L [10.3-27.7 nmol/L]) a GnRH agonist. In this vignette, the sequence
LH = 0.16 mIU/mL (2.0-6.0 mIU/mL) (SI: 0.16 IU/L of events strongly suggests a causal relationship
[2.0-6.0 IU/L]) between the administration of the GnRH agonist
FSH = 9.2 mIU/mL (1.0-5.0 mIU/mL) (SI: 9.2 IU/L and subsequent apoplexy. While GnRH agonists
[1.0-5.0 IU/L])
are known to acutely stimulate both LH and
Free α-subunit = 2.3 ng/mL (0.20-6.0 ng/mL)
FSH and then to desensitize GnRH receptors
and decrease gonadotropin secretion in patients
Which of the following proposals concerning with a normal pituitary gland, they may have a
this patient is/are the correct one(s)? persistent stimulatory effect on tumoral secretion
A. There is no relationship between the injection in patients with FSH-secreting pituitary adenomas.
of the GnRH analogue and the pituitary mass Indeed, increased serum FSH levels with low
effect in this patient levels of LH and testosterone in a patient with a
B. The injection of GnRH analogue triggered pituitary macroadenoma who has no evidence of
pituitary apoplexy excess secretion of prolactin, GH, ACTH, or TSH
is typical of a pituitary gonadotroph adenoma.
C. The hormonal pattern in this patient is typical
Whether gonadotroph adenomas are more prone
of a gonadotroph adenoma
to develop apoplexy when stimulated by a GnRH
D. The hormonal pattern in this patient is related agonist than other types of adenomas is unknown.
to the flair-up effect of GnRH agonist Case 1 and Case 2 have been published.11,29
Answer: B and C) The injection of GnRH analogue
triggered pituitary apoplexy and the hormonal pattern
in this patient is typical of a gonadotroph adenoma

References
1. Chanson P, Lecoq A-L, Raverot G, et al. Physiopathology, diagnosis, and 5. Nishioka H, Inoshita N, Mete O, et al. The complementary role of
treatment of nonfunctioning pituitary adenomas. In: Casanueva FF, Ghigo E, transcription factors in the accurate diagnosis of clinically nonfunctioning
eds. Hypothalamic-Pituitary Diseases. Cham: Springer International Publishing pituitary adenomas. Endocr Pathol. 2015;26(4):349-355. PMID: 2641628
AG, Switzerland; 2018:1-37. 6. Pigny P, Henric B, Lahlou N, et al. A gonadotroph adenoma with a high
2. Chanson P, Raverot G, Castinetti F, et al; French Endocrinology Society proportion of basic FSH isohormones by chromatofocusing. J Clin Endocrinol
non-functioning pituitary adenoma work-group. Management of clinically Metab. 1996;81(6):2407-2408. PMID: 8964889
non-functioning pituitary adenoma. Ann Endocrinol (Paris). 2015;76(3):239- 7. Raappana A, Koivukangas J, Ebeling T, Pirila T. Incidence of pituitary
247. PMID: 26072284 adenomas in Northern Finland in 1992-2007. J Clin Endocrinol Metab.
3. Djerassi A, Coutifaris C, West VA, et al. Gonadotroph adenoma in a 2010;95(9):4268-4275. PMID: 20534753
premenopausal woman secreting follicle-stimulating hormone and causing 8. Daly AF, Rixhon M, Adam C, Dempegioti A, Tichomirowa MA, Beckers
ovarian hyperstimulation. J Clin Endocrinol Metab. 1995;80(2):591-594. A. High prevalence of pituitary adenomas: a cross-sectional study in the
PMID: 7852525 province of Liege, Belgium. J Clin Endocrinol Metab. 2006;91(12):4769-4775.
4. Asa SL. Pituitary adenomas. In: Asa SL, ed. Tumors of the Pituitary Gland. PMID: 16968795
Vol Fourth Series Fascicle 15. Washington, DC: Armed Forces Institute of 9. Fernandez A, Karavitaki N, Wass JA. Prevalence of pituitary adenomas: a
Pathology; 2011:55-172. community-based, cross-sectional study in Banbury (Oxfordshire, UK). Clin
Endocrinol (Oxf). 2010;72(3):377-382. PMID: 19650784

210  ENDO 2021 • Endocrine Case Management

chased by Dr. Khalid H. Seedahmed, Jr., MRCP PGDip, CertEndocrinology SA - ID 283


10. Ntali G, Capatina C, Grossman A, Karavitaki N. Clinical review: functioning 20. Ambrosi B, Bassetti M, Ferrario R, Medri G, Giannattasio G, Faglia G.
gonadotroph adenomas. J Clin Endocrinol Metab. 2014;99(12):4423-4433. Precocious puberty in a boy with a PRL-, LH- and FSH-secreting pituitary
PMID: 25166722 tumour: hormonal and immunocytochemical studies. 1990;122(5):569-576.
11. Graillon T, Castinetti F, Chabert-Orsini V, et al. Functioning gonadotroph PMID: 2112813
adenoma with severe ovarian hyperstimulation syndrome: a new emergency 21. Chanson P, Maiter D. The epidemiology, diagnosis and treatment of
in pituitary adenoma surgery? Surgical considerations and literature review. prolactinomas: the old and the new. Best Pract Res Clin Endocrinol Metab.
Ann Endocrinol (Paris). 2019;80(2):122-127. PMID: 30825998 2019;33(2):101290. PMID: 31326373
12. Caretto A, Lanzi R, Piani C, Molgora M, Mortini P, Losa M. Ovarian 22. Briet C, Salenave S, Bonneville JF, Laws ER, Chanson P. Pituitary apoplexy.
hyperstimulation syndrome due to follicle-stimulating hormone-secreting Endocr Rev. 2015;36(6):622-645. PMID: 26414232
pituitary adenomas. Pituitary. 2017;20(5):553-560. PMID: 28676954 23. Chanson P, Brochier S. Non-functioning pituitary adenomas. J Endocrinol
13. Kawaguchi T, Ogawa Y, Ito K, Watanabe M, Tominaga T. Follicle- Invest. 2005;28(11 Suppl International):93-99. PMID: 16625856
stimulating hormone-secreting pituitary adenoma manifesting as recurrent 24. Raverot G, Assie G, Cotton F, et al. Biological and radiological exploration
ovarian cysts in a young woman--latent risk of unidentified ovarian and management of non-functioning pituitary adenoma. Ann Endocrinol
hyperstimulation: a case report. BMC Res Notes. 2013;6:408. PMID: 24119690 (Paris). 2015;76(3):201-209. PMID: 26122495
14. Halupczok J, Kluba-Szyszka A, Bidzinska-Speichert B, Knychalski B. Ovarian 25. Cooper O, Geller JL, Melmed S. Ovarian hyperstimulation syndrome caused
hyperstimulation caused by gonadotroph pituitary adenoma--review. Adv by an FSH-secreting pituitary adenoma. Nat Clin Pract Endocrinol Metab.
Clin Exp Med. 2015;24(4):695-703. PMID: 26469116 2008;4(4):234-238. PMID: 18268519
15. Heseltine D, White MC, Kendall-Taylor P, de Kretser DM, Kelly W. 26. Chanson P, Dormoy A, Dekkers O. Use of radiotherapy after pituitary
Testicular enlargement and elevated serum inhibin concentrations occur surgery for non-functioning pituitary adenomas. Eur J Endocrinol.
in patients with pituitary macroadenomas secreting follicle-stimulating 2019;181(1):D1-D13. PMID: 31048560
hormone. Clin Endocrinol (Oxf). 1989;31(4):411-423. PMID: 2627747 27. Knoepfelmacher M, Danilovic DL, Rosa Nasser RH, Mendonca BB.
16. Dahlqvist P, Koskinen LO, Brannstrom T, Hagg E. Testicular enlargement in Effectiveness of treating ovarian hyperstimulation syndrome with cabergoline
a patient with a FSH-secreting pituitary adenoma. Endocrine. 2010;37(2):289- in two patients with gonadotropin-producing pituitary adenomas. Fertil Steril.
293. PMID: 20960265 2006;86(3):719.e15-e18. PMID: 16952513
17. Tashiro H, Katabuchi H, Ohtake H, Kaku T, Ushio Y, Okamura H. A 28. Paoletti AM, Depau GF, Mais V, Guerriero S, Ajossa S, Melis GB.
follicle-stimulating hormone–secreting gonadotroph adenoma with Effectivenesses of cabergoline in reducing follicle-stimulating hormone
ovarian enlargement in a 10-year-old girl. Fertil Steril. 1999;72(1):158-160. and prolactin hypersecretion from pituitary macroadenoma in an infertile
PMID: 10428166 woman. Fertil Steril. 1994;62(4):882-885. PMID: 7926104
18. Di Rocco C, Maira G, Borrelli P. Pituitary microadenomas in children. Childs 29. Chanson P, Schaison G. Pituitary apoplexy caused by GnRH-agonist
Brain. 1982;9(3-4):165-178. PMID: 7105883 treatment revealing gonadotroph adenoma. J Clin Endocrinol Metab.
19. Faggiano M, Criscuolo T, Perrone L, Quarto C, Sinisi AA. Sexual precocity in 1995;80(7):2267-2268. PMID: 7608291
a boy due to hypersecretion of LH and prolactin by a pituitary adenoma. Acta
Endocrinol (Copenh). 1983;102(2):167-172. PMID: 6681924

ENDO 2021 • Neuroendocrinology and Pituitary  211

chased by Dr. Khalid H. Seedahmed, Jr., MRCP PGDip, CertEndocrinology SA - ID 283


chased by Dr. Khalid H. Seedahmed, Jr., MRCP PGDip, CertEndocrinology SA - ID 283
PEDIATRIC
ENDOCRINOLOGY

chased by Dr. Khalid H. Seedahmed, Jr., MRCP PGDip, CertEndocrinology SA - ID 283


Management of Pituitary
Hormone Replacement Through
Transition From Adolescence
to Young Adulthood
Mehul T. Dattani, MBBS, DCH, FRCPCH, FRCP, MD. Genetics and Genomic Medicine
Programme, UCL GOS Institute of Child Health, London, United Kingdom; E-mail:
mdattani@ucl.ac.uk

Learning Objectives early. Reversibility of pituitary hormone deficits


may occur, even in the presence of a structurally
As a result of participating in this session, learners
abnormal pituitary gland. Nevertheless,
should be able to:
MRI remains an extremely useful tool in the
• Describe the dynamic nature of management of hypopituitarism. Careful
hypopituitarism and adapt to the needs of the hormone replacement with diligent monitoring
young person at all stages, but particularly at of auxology, general well-being, and biochemical
the time of adolescence and transition. profiles is critical to ensuring good outcomes in
terms of height, weight, and clinical well-being.
• Develop a holistic approach to the
Finally, hypopituitarism may be associated with
management of hypopituitarism at the time of
a range of other clinical phenotypes, including
transition.
morbid obesity, autism, learning difficulties,
and visual impairment, necessitating a careful
multidisciplinary approach to the problem.
A holistic approach involves a pediatric and adult
Main Conclusions (at transition and thereafter) endocrinologist,
Hypopituitarism is phenotypically a highly ophthalmologist, neurodevelopmental
variable condition that may encompass a single pediatrician, clinical nurse specialist, psychologist,
hormone deficiency or multiple pituitary hormone dietitian, and social worker.
deficiencies, including both anterior and posterior
pituitary hormones. The condition may evolve
throughout adolescence and childhood, and
hence there is a need for vigilance and regular
Significance of the
testing of pituitary function on a lifelong basis. Clinical Problem
The endocrine morbidity score may be a useful Hypopituitarism is a rare disorder and may be
marker, but it is highly variable in these children. congenital (1 in 4000 to 1 in 10,000 live births) or
Importantly, puberty is highly variable in children acquired due to tumors, inflammation, infiltration,
with hypopituitarism, and may be normal in infection, etc.1 Congenital hypopituitarism is
timing, delayed, or absent. Paradoxically, in a highly variable condition that evolves over
patients with septo-optic dysplasia, it may occur time. It may be associated with a range of other

214  ENDO 2021 • Endocrine Case Management

chased by Dr. Khalid H. Seedahmed, Jr., MRCP PGDip, CertEndocrinology SA - ID 283


abnormalities, including midline forebrain Strategies for Diagnosis,
abnormalities (eg, absence of the septum
Therapy, and/or Management
pellucidum and hypoplasia/absence of the corpus
callosum), eye abnormalities (eg, optic nerve Introduction
hypoplasia, coloboma, and microphthalmia/ Hypopituitarism may be due to congenital or
anophthalmia), cleft lip and/or palate, and acquired causes. Congenital hypopituitarism
sensorineural deafness.2 Rare cases may be can arise from pathogenic variants in any of the
associated with holoprosencephaly or other genes involved in pituitary development and
syndromes associated with hypopituitarism. has a reported incidence of 1 in 3000 to 4000
The disorder can have significant morbidity births. It is a highly heterogeneous disorder
and mortality if undiagnosed and managed that manifests either as an isolated hormone
suboptimally. Additionally, these children and deficiency, the most common being isolated GH
adolescents often have associated challenging deficiency, or as combined pituitary hormone
comorbidities such as sleep disturbance, obesity, deficiency when 2 or more pituitary hormones
learning difficulties, behavioral issues, and visual are affected. The clinical features vary in severity
impairment. Optimal management remains a and timing of presentation; its onset may be
challenge, particularly at the time of transition, early in the neonatal period or later in life.
when there are complex issues with respect to Congenital hypopituitarism can also be part of a
puberty and fertility, adherence, the need to syndrome where abnormalities in extrapituitary
conform with peers, and the emergence of high- structures that share a common embryologic
risk behaviors. Few studies have explored the origin with the pituitary gland (eg, eye, midline,
trajectory of these patients through puberty and and forebrain) occur in addition to pituitary
to adulthood. Hence, data are sparse in terms of hormone deficiencies. The etiology remains
optimal management. unknown in most patients.3 Recent data suggest
What is becoming increasingly clear is that that pathogenic variants in genes implicated
the need for a holistic, multidisciplinary approach in hypothalamic-pituitary development or GH
is never greater than at puberty. Significant input secretion can be identified in approximately 10%
is required from the clinician and the clinical of cases.2,3 Congenital forms of hypopituitarism are
nurse specialist/nurse practitioner. Challenges rare and account for only 4.5% of hypopituitarism
around adherence, optimal dosage, behavior, in adults.4 Acquired hypopituitarism may be due to
psychology, and emerging independence from tumors and their treatment, trauma, inflammation,
parents should be tackled with the help of a range infiltration, or infection. In the pediatric and
of multidisciplinary professionals. The need for a adolescent population, intracranial tumors are
pause of, for example, GH therapy, before possible probably the most frequent cause. In adult cohort
retesting needs to be addressed. studies, suprasellar tumors cause the majority of
hypothalamic-pituitary dysfunction (50%-60%).4,5
Barriers to Optimal Practice Other causes include surgery or radiotherapy,
traumatic brain injury, infection, autoimmune
• Lack of resources in terms of time and processes, infiltration by granulomatous disease,
management from the multidisciplinary team iron overload states, and vascular causes.
to provide optimal care.
• Lack of engagement from the young person. Diagnosis
• Lack of consensus with respect to optimal The diagnosis of hypopituitarism is based on a
management. combination of clinical assessment, looking for the
presence of not just endocrine-related symptoms

ENDO 2021 • Pediatric Endocrinology  215

chased by Dr. Khalid H. Seedahmed, Jr., MRCP PGDip, CertEndocrinology SA - ID 283


and signs (eg, frontal bossing and growth failure, stimulation test neither distinguishes the 2 nor
micropenis, and cryptorchidism in males), but changes clinical management.7
also for other features related to the presence The timing of basal and dynamic function tests
of a congenital syndrome or an acquired form may be important. Testing for hypogonadotropic
such as raised intracranial pressure, auxology hypogonadism is not useful outside of the mini-
(height, weight, BMI and their corresponding puberty (up to age 6 months) and pubertal
standard deviation scores), pubertal assessment, phases (boys aged >9 years, girls aged >8 years),
biochemistry, neuroradiology, and possibly as the hypothalamic-pituitary-gonadal axis is
genetics. If available, prior growth curves from quiescent outside these periods. In some cases,
community screening may indicate the presence of 24-hour (cortisol) or overnight (GH) hormone
growth failure or rapid weight gain in the months profiling to determine spontaneous hormone
preceding diagnosis. For instance, in patients with secretion may be necessary to detect more
craniopharyngiomas, early changes in weight and subtle endocrine deficits that would otherwise
BMI have been shown to precede the diagnosis appear to be normal based on values obtained
of a suprasellar mass by several months and may from artificial stimulation tests. The interaction
be predictive of future hypothalamic obesity.6 between the different hypothalamic-pituitary
The presence of precocious, delayed, or arrested axes is relevant to interpreting endocrine tests.
puberty supported by an inappropriately delayed For instance, cortisol sufficiency is required to
or advanced bone age can also help predict future permit renal free water clearance via inhibition
growth potential and determine if intervention is of antidiuretic hormone secretion, so ACTH
needed to achieve this. deficiency may mask coexistent central diabetes
Confirmation of hypopituitarism is a stepwise insipidus until glucocorticoid replacement is
process, usually beginning with a baseline initiated. If a suprasellar tumor is suspected or
pituitary function screen, followed by dynamic has already been confirmed by neuroimaging,
pituitary function tests. Normal hormone additional investigations should also be
secretion is dependent on the presence of an intact performed to determine whether the mass is
hypothalamic-pituitary-target gland axis. Dynamic responsible for secreting α-fetoprotein and/or
tests aim to stimulate the relevant axis if hormone β-hCG (germinomas), prolactin (prolactinomas),
deficiency is suspected. Basal pituitary hormone GH (somatotropinomas), or ACTH
screening helps risk-stratify patients who require (corticotropinomas). Occasionally, measurement
prioritizing for dynamic testing before definitive of cerebrospinal fluid α-fetoprotein and β-hCG
treatment, particularly to establish the status of the concentrations may be needed to support the
hypothalamic-pituitary-adrenal and antidiuretic diagnosis of a germinoma. Evidence of tumor
hormone axes in order to avoid the fatal secretion of various molecules can help avoid
consequences of uncorrected cortisol insufficiency the need for a diagnostic biopsy or unnecessary
and/or central diabetes insipidus (eg, during surgical resection, since such procedures
surgery). In this situation, early-morning cortisol carry significant risks of further damaging the
and ACTH concentrations should be measured, hypothalamic-pituitary axis.
particularly before administration of high- MRI is warranted in all patients with
dosage dexamethasone for peritumoral edema. documented hypopituitarism. In a retrospective
In some cases, dynamic function testing is not study of MRI findings and endocrine function
required—for instance, a low concentration of free in 170 children with or “at risk” for congenital
T4 in the presence of an inappropriately low or hypopituitarism (optic nerve hypoplasia),
normal concentration of TSH is sufficient for the holoprosencephaly, or multiple midline systemic
diagnosis of secondary or tertiary hypothyroidism, defects), the risk of hypopituitarism was 27 times
and an additional thyrotropin-releasing hormone greater in patients with an ectopic posterior

216  ENDO 2021 • Endocrine Case Management

chased by Dr. Khalid H. Seedahmed, Jr., MRCP PGDip, CertEndocrinology SA - ID 283


pituitary.8 Anterior pituitary hypoplasia and may predict prognosis, improve early diagnosis
pituitary stalk agenesis were also significantly and management, and assist with counseling
associated with hypopituitarism. With respect to the family. Identifying certain pathogenic
the type or severity of hypopituitarism, combined variants may assist in the differential diagnosis.
pituitary hormone deficiency was more often For example, PROP1 pathogenic variants are
associated with an abnormal corpus callosum and associated with suprasellar masses that undergo
stalk abnormalities. Overall, midline forebrain spontaneous resolution, and the presence of a
defects were 5.2 times more prevalent in patients PROP1 pathogenic variant may help to reassure
with combined pituitary hormone deficiency than families of the absence of a brain tumor and
in patients with isolated GH deficiency. The results circumvent the need for surgery. Given the
of this study confirm that midline forebrain defects variable penetrance of many dominant pathogenic
are associated with a higher risk of combined variants, prenatal diagnosis should not be offered
pituitary hormone deficiency, but also suggest that until understanding of the genetic basis of many of
in children at risk of developing hypopituitarism these conditions has advanced significantly. Whole
due to the presence of optic nerve hypoplasia and/ exome- and genome-sequencing technologies
or midline brain abnormalities, corpus callosum and are currently offered on a research basis and may
pituitary stalk abnormalities are strong predictors of further improve understanding of the genetic
combined pituitary hormone deficiency. etiology of these conditions.
The role of genetics in congenital
hypopituitarism remains to be established, and Evolution of Endocrinopathies
testing is offered on a research basis only. The
past decades have witnessed an explosion in our Children with isolated GH deficiency have
understanding of the development of the anterior a significant risk of developing additional
pituitary gland and of mechanisms that underlie the pituitary deficiencies.9 This risk ranges between
diagnosis of GH deficiency and combined pituitary 5.5% in childhood-onset idiopathic isolated
hormone deficiency. However, to date, pathogenic GH deficiency and 35% in adult-onset organic
variants have been identified in only a modest isolated GH deficiency. In a large prospective,
proportion of patients.1-3 It is clear that many genes multinational study (GeNeSIS), data were analyzed
remain to be identified and the characterization of in 5805 pediatric patients with idiopathic isolated
these genes will further elucidate the pathogenesis GH deficiency. Combined pituitary hormone
of these complex conditions. deficiency developed in 2.0% of the overall
Transcription factors encoded by the genes cohort and in 5.5% among children followed for
HESX1, PROP1, POU1F1, LHX3, LHX4, SOX2, and a minimum of 3.5 years. The relative frequency
SOX3 are involved in the etiology of combined of additional deficiencies was TSH > LH/FSH >
pituitary hormone deficiency, and they are antidiuretic hormone > ACTH. Median interval
associated with highly variable phenotypes from diagnosis of GH deficiency to additional
encompassing isolated GH deficiency, combined pituitary hormone deficiency was 1.9 years for
pituitary hormone deficiency, or more complex TSH, 2.4 years for both antidiuretic hormone and
disorders such as septo-optic dysplasia.2,3 PROP1 ACTH, and 3.3 years for LH/FSH.10 In 716 pediatric
pathogenic variants constitute the most common patients from the same database with isolated
genetic cause for combined pituitary hormone GH deficiency due to organic causes, there was a
deficiency, with a poor genotype-phenotype higher frequency of combined pituitary hormone
correlation and different pattern of evolution of deficiency (9.9% in the overall cohort, and 20.7%
endocrinopathies. Appropriate genetic testing in the subgroup followed up for a minimum
is an important adjunct to management because of 3.5 years). The most frequent additional
understanding the pathophysiologic process deficiencies were TSH and LH/FSH.11 In a

ENDO 2021 • Pediatric Endocrinology  217

chased by Dr. Khalid H. Seedahmed, Jr., MRCP PGDip, CertEndocrinology SA - ID 283


retrospective study of 83 patients initially diagnosed suggest that thyroid and adrenal function should
as having isolated GH deficiency in childhood, 45% be monitored during GH therapy.
developed combined pituitary hormone deficiency
after a median follow-up of 5.4 years.12 Treatment
Hypothalamic-pituitary, optic nerve, and
midline brain abnormalities predispose to a higher The mainstay of treatment is replacement of the
risk of developing additional pituitary deficits appropriate hormones. In both congenital and
(up to 45%-100% in children with pituitary stalk acquired hypopituitarism, it is crucial to recognize
interruption syndrome). Pituitary stalk agenesis the potential for evolution of endocrine deficits
as demonstrated on MRI after gadolinium over time, and all patients with documented
administration and an ectopic posterior pituitary endocrine dysfunction require lifelong follow-up
located at the median eminence and hypothalamic with appropriate transition into adulthood.
level are highly predictive of combined pituitary Growth and puberty should be monitored and
hormone deficiency. Empty sella, absent/ GH deficiency should be treated with recombinant
hypoplastic corpus callosum, and optic nerve human GH (rhGH) until linear growth ceases,
hypoplasia are also associated with a higher risk although there is increasing evidence for
of evolution of endocrinopathy. The type and continued GH treatment into adulthood due to its
age at occurrence of additional pituitary deficits possible metabolic effects on body composition
are highly variable and they cannot be easily and bone mineral density. In most countries,
predicted based on the etiology, severity, and age ongoing GH treatment is only indicated in those
at onset of isolated GH deficiency. Importantly, young people who have been shown to have
idiopathic isolated GH deficiency without permanent GH deficiency. Retesting of GH
structural hypothalamic-pituitary abnormalities secretion is indicated in those with childhood-
does not exclude the occurrence of additional onset GH deficiency, usually after the completion
pituitary deficits. A longer duration of follow- of statural growth. Current guidelines suggest
up is associated with a higher risk of additional that retesting is not required for those with more
pituitary deficits, suggesting that hypopituitarism than 2 pituitary hormone deficiencies, a confirmed
is a dynamic condition where new deficiencies genetic pathogenic variant accounting for the
can appear years after the initial diagnosis. hypopituitarism, and/or a specific hypothalamic-
Adrenal insufficiency can silently evolve years pituitary structural defect on MRI with the
after the onset of GH deficiency and the start of exception of an ectopic posterior pituitary.13 The
GH treatment in patients with GH deficiency can insulin-tolerance test or the GHRH-arginine
unmask central hypothyroidism. test are recommended for retesting. Between
To conclude, current available evidence 25% and 88% of patients reverse their GH
supports longstanding recommendations for secretion at the time of transition. The dosages
the need, in all patients diagnosed with isolated recommended are variable, but generally lower
GH deficiency, of careful and indefinite follow- than those used in childhood, and titration with
up for additional pituitary hormone deficiencies, IGF-1 concentrations is recommended. It has been
irrespective of the age at presentation, severity, recommended that GH be continued until full
etiology, and reversibility of GH deficiency in skeletal and muscle maturation has been attained.
adolescence. In patients with an initial diagnosis of In tumor survivors, GH therapy in replacement
isolated GH deficiency, particularly those with an dosages is not associated with an increased risk
ectopic posterior pituitary or other developmental of recurrence or progression, although some
abnormalities, the clinician should be alert to the evidence suggests that there may be a small
risk of the development of combined pituitary increased risk of second primary neoplasms, but
hormone deficiency. Similarly, adult guidelines this is not completely clear. Hence, minimum

218  ENDO 2021 • Endocrine Case Management

chased by Dr. Khalid H. Seedahmed, Jr., MRCP PGDip, CertEndocrinology SA - ID 283


effective dosages for GH replacement should be The dosage is gradually increased over a period of
used, aiming for normal IGF-1 concentrations and 24 months or so. A cyclical progestogen is added
an age-appropriate height velocity. once breakthrough bleeding occurs. Monitoring
Puberty may be precocious or occur at the growth, secondary sexual characteristics, uterine
normal time in those with hypopituitarism volume, blood pressure, and bone mineral density
associated with septo-optic dysplasia, but it is recommended. Pulsatile GnRH treatment or the
may be delayed or absent in those with isolated administration of gonadotropins is recommended
combined pituitary hormone deficiency.14 for the achievement of spermatogenesis and
Treatment of coexistent precocious puberty testicular enlargement in males and ovarian
with GnRH analogues can help restore adult follicular maturation in females.
height if commenced early, but the decision to TSH deficiency is easily replaced with
artificially delay puberty should be decided on levothyroxine supplementation, with dose titrations
an individual basis. Conversely, in the face of being based entirely on free T4 concentrations
coexistent hypogonadotropic hypogonadism, it is and not TSH. Free T4 concentrations should
not unusual to commence GH supplementation be maintained in the upper half of the normal
at least 6 months before sex steroid replacement, range, particularly given the risk of hypothalamic
the timing of which again should be tailored to obesity in many forms of hypopituitarism. Before
the individual child. Delaying commencement commencing replacement, preexisting ACTH
of the latter beyond the usual pubertal age deficiency should be detected to avoid the risk of
(~12 years in girls and 14-15 years in boys) is precipitating a hypocortisolemic crisis.
not generally recommended, given the long- The diagnosis of ACTH deficiency is
term benefits on bone mineral accretion.15 challenging during adolescence. Because fatigue
Precocious puberty does not preclude the is commonly observed in adolescence, this is not
evolution of subsequent hypogonadotropic a good discriminatory feature in the selection
hypogonadism, and children should continue to of adolescents for testing. Once the diagnosis is
be monitored carefully after cessation of GnRH made, replacement doses of hydrocortisone should
analogue therapy to ensure normal pubertal be given at least 3 times daily and titrated in the
progress. With respect to hypogonadotropic growing child and adolescent against trough
hypogonadism, treatment in males is commonly concentrations using a cortisol day curve. Doses
in the form of testosterone (parenteral, oral, or should be doubled or even tripled in the face
transdermal). This is induced slowly over the of illness, with patient education on how and
period of approximately 2.5 to 3 years, with when to administer emergency intramuscular
careful monitoring of growth, virilization, and hydrocortisone (doses <1 year, 25 mg; 1-5 years,
trough testosterone concentrations. Longer- 25-50 mg; >5 years, 100 mg) and correct
acting testosterone preparations may be used once hypoglycemia during crises. Input from the
growth has been completed. Monitoring of the full endocrine clinical nurse specialist is particularly
blood count is recommended, given the adverse critical at the time of transition. Discussions
effect of polycythemia. In females, 17β-estradiol around timing of hydrocortisone replacement are
is the preferred option, and is available as gel, critical, as adolescents may stay up late and wake
transdermal patch, and oral preparation. It has up late. Additionally, empowering adolescents
been suggested that the transdermal route is with critical information on how to prevent
superior to the oral route in terms of bone accrual adrenal crises, as well as addressing lifestyle
and uterine growth.16 Concentrations of genotoxic changes such as drinking alcohol, may save lives.
estrogens (mutagenic metabolites linked to Longer-acting corticosteroids such as prednisolone
breast carcinogenesis) are lower with the use of may provide better control on less frequent dosing
transdermal estrogen than with oral estrogen.17 once growth is complete.

ENDO 2021 • Pediatric Endocrinology  219

chased by Dr. Khalid H. Seedahmed, Jr., MRCP PGDip, CertEndocrinology SA - ID 283


Management of central diabetes insipidus, and mortality. Psychological issues may be a
particularly in patients with coexisting ACTH major feature of the disease phenotype, with
deficiency and/or hypothalamic adipsia the adolescent facing the prospect of a lifelong
(eg, in some patients with postsurgical suprasellar condition requiring ongoing medication. This
tumor), is complex and needs specialist care. Oral in turn may lead to isolation. The presence of
desmopressin may be preferred, as it is widely hypogonadism may raise concerns with respect
available, is generally well absorbed especially to future relationships and prospects for fertility.
with intercurrent respiratory tract infections, less Associated abnormalities such as vision impairment,
likely to lead to hyponatremia, and easier to use obesity, and the presence of varying degrees of
in patients with visual impairment and physical autism and other complications of the underlying
disability.18 Untreated patients with an intact thirst condition may add to difficulties in management.
axis and free access to fluid are able to maintain Transition is also a time for re-evaluation
euvolemia and eunatremia by adjusting their oral of the endocrinopathy and planning long-term
intake appropriately. DDAVP treatment aims to management. Transition clinics with input from
provide the patient with a relatively normal quality pediatric/adolescent endocrinologists, adult
of life by reducing the burden of polyuria and endocrinologists, and the endocrinology clinical
polydipsia. Doses should be titrated against trough nurse specialist are critical to the successful
paired plasma and urine osmolalities, and treatment transition of patients. In patients with learning
should ideally be started in the inpatient setting. disabilities, it is important that “best interest”
Treatment should generally err on the side of meetings take place to ensure that a suitable
underdosing, since overdosing DDAVP can result advocate is available to act on behalf of the patient.
in water intoxication, hyponatremia, and rapid fluid Appropriate arrangements must be implemented
shifts that are difficult to correct safely due to the for patients to function as independently as
risk of cerebral edema or even death. Patients with possible. Loss to follow-up by a physician may
hypothalamic adipsia require a strict fluid intake be associated with the risk of adrenal crisis, a
regimen to maintain euvolemia. Additionally, it is hypothyroid state, reduced bone mineral density,
important to remember that certain “recreational” and fatigue with reduced libido.
drugs such as MDMA (“Ecstasy”) may lead to
overdrinking, which could be extremely dangerous
in patients on desmopressin.
Clinical Case Vignettes
Case 1
Management at Transition A 5-month-old male infant presents with
cryptorchidism and poor growth. His birth weight
Transition presents significant challenges in the
was 5 lb 0 oz (2260 g) at 35 weeks’ gestation, and
management of hypopituitarism. It is defined
he required nasogastric feeding. At the age of
as the time between the completion of normal
1.5 years, he is noted to have a length of 22.8 in
puberty and the achievement of peak bone mass.15
(58 cm; -8 SDS) with a growth rate of 2 cm/y. He
The transition period is a highly complex stage
has excess skin folds. Basal investigations reveal a
of life, as adolescents complete their secondary
total T4 concentration of 2.95 μg/dL (38 nmol/L)
education, change lifestyles, demonstrate greater
with a TSH concentration of 1.0 mIU/L.
independence from their parents and thereby
Levothyroxine is initiated. He has a hypoglycemic
lose parental supervision, engage in high-risk
convulsion and undergoes a clonidine test,
behaviors, and prepare to move away from
which reveals all GH concentrations to be less
their parental homes. As with most conditions,
than 0.3 ng/mL (<0.3 μg/L). The 8-AM cortisol
adherence to treatment may be an issue, putting
concentration is 40.1 μg/dL (1105 nmol/L).
the young person at risk for significant morbidity

220  ENDO 2021 • Endocrine Case Management

chased by Dr. Khalid H. Seedahmed, Jr., MRCP PGDip, CertEndocrinology SA - ID 283


GH therapy is initiated at age 2.5 years, and by POU1F1 is a transcription factor that is implicated
he has an excellent response, with a growth in somatotroph, thyrotroph, and lactotroph
velocity of 20 cm/y. At age 3.5 years, CT reveals development. Pathogenic variants are associated
a hypoplastic anterior pituitary gland. His early- with GH, TSH, and prolactin deficiencies, but not
morning cortisol peak is 11.0 μg/dL (303 nmol/L). ACTH or gonadotropin deficiencies. MRI usually
At age 6.5 years, hydrocortisone is initiated at a reveals a small anterior pituitary with a normally
dosage of 12 mg/m2 per day in view of a perceived placed posterior pituitary.
suboptimal cortisol peak on glucagon-stimulation
testing (12.1 μg/dL [335 nmol/L] at baseline). Case 2
He gains weight rapidly with weight at the
75th percentile and height at the 3rd percentile. An 11-and-2/12-year-old girl presents with short
At age 12 years, intramuscular testosterone is stature (height, –2.5 SDS; weight, –1.69 SDS).
initiated. Although the testes are palpable, they are Birth weight was 4 lb 1 oz (1875 g) at 32 weeks’
thought to be small. Weight gain remains an issue, gestation (0.47 SDS). Her growth slowed down
with height between the 3rd and 10th percentiles after 3 years of age. She was evaluated at 8 years
and the weight around the 90th percentile. of age, and the results of basal investigations
Random cortisol concentrations are elevated. were unremarkable. It was thought that she had a
At age 16.44 years, his near-final height is probable diagnosis of constitutional growth delay.
60.1 in (152.6 cm) and weight is 152.1 lb (69 kg). The growth velocity is 5 cm/y, and she has a bone
Genetic screening confirms the presence of a age delay of 2 years. She is completely prepubertal.
pathogenic variant in a gene regulating GH Laboratory test results:
secretion. He is then retested off all treatment
at 18 years of age, and the results document IGF-1 = 53 ng/mL (218-965 ng/mL) (SI: 6.9 nmol/L
GH, prolactin, and TSH deficiencies with [28.6-126.4 nmol/L])
Free T4 = 1.05 ng/dL (0.93-1.7 ng/dL)
normal LH (15.1 mIU/mL [15.1 IU/L]), FSH (SI: 13.5 pmol/L [12-22 pmol/L])
(5.8 mIU/mL [5.8 IU/L]), and peak cortisol of LH = <0.7 mIU/mL (SI: <0.7 IU/L)
22.7 μg/dL (627 nmol/L) on insulin-tolerance FSH = 3.7 mIU/mL (SI: 3.7 IU/L)
testing. Hydrocortisone and testosterone treatment Celiac screen, negative
are then stopped. MRI confirms a hypoplastic Karyotype = 46,XX
anterior pituitary with a normal stalk and eutopic Skeletal survey and prepubertal pelvic
posterior pituitary. ultrasonography are normal.
Constitutional delay of growth and puberty
A mutation/pathogenic variant in is diagnosed, and ethinyl estradiol is initiated at a
which of the following genes most likely dosage of 2 mcg daily. She is then lost to follow-
accounts for the patient’s phenotype? up. She stops the estradiol 4 months after being
A. KAL1 seen in the clinic. She reappears in the clinic a year
B. LHX4 later, and her growth rate is 3 cm/y. A primed
C. PROP1 insulin-tolerance test is performed. This reveals a
D. POU1F1 peak GH concentration of 5.6 ng/mL (5.6 μg/L)
with a peak cortisol concentration of 33.5 μg/
E. GLI2 dL (924 nmol/L). The GnRH test reveals a peak
Answer: D) POU1F1 LH concentration of 19.4 mIU/mL (19.4 IU/L)
and a peak FSH concentration of 3.3 mIU/mL
The pathogenic variant was a de novo p.R271W (3.3 IU/L). MRI of the brain and pituitary is
in the POU1F1 gene (Answer D), which is a performed and reveals a hypoplastic anterior
dominant-negative mutation. The protein encoded pituitary, an ectopic or undescended posterior

ENDO 2021 • Pediatric Endocrinology  221

chased by Dr. Khalid H. Seedahmed, Jr., MRCP PGDip, CertEndocrinology SA - ID 283


pituitary, and an absent pituitary stalk. GH deficiencies. Additionally, if the GH dosage is
treatment is initiated, and she responds well with inadequate, it would need to be increased.
a growth velocity of 8 cm/y. At age 15.4 years,
she has progressed to breast Tanner stage 4/5, but Testing documented the following:
complains of fatigue. IGF-1 = 128 ng/mL (224-896 ng/mL)
(SI: 16.8 nmol/L [29.4-117.4 nmol/L])
Which of the following is the best next step? Free T4 = 0.9 ng/dL (0.9-1.7 ng/dL) (SI: 12.1 pmol/L
[12-22 pmol/L])
A. Re-start ethinyl estradiol TSH = 3.9 mIU/L (<6 mIU/L)
B. Check the adequacy of GH dosage Peak cortisol on cosyntropin-stimulation testing =
14.5 μg/dL (normal >20 μg/dL) (SI: 401 nmol/L
C. Assess her cortisol and thyroid function [>550 nmol/L]) with a mean 24-hour cortisol
D. Investigate for other associated conditions value of 3.9 μg/dL (normal >5.3 μg/dL) (SI:
E. B, C, and D 107 nmol/L [>145 nmol/L]) on 2 hourly profile

Answer: E) B, C, and D Low-dosage hydrocortisone and levothyroxine


were started, and the GH dosage was adjusted. She
With an ectopic posterior pituitary, this patient felt better and achieved menarche at 16.1 years
would most likely have developed other hormone with a final height of 61.4 in (156 cm).

References
1. Castinetti F, Reynaud R, Saveanu A, et al. Clinical and genetic aspects of 11. Child CJ, Blum WF, Deal C, et al. Development of additional pituitary
combined pituitary hormone deficiencies [article in French]. Ann Endocrinol hormone deficiencies in pediatric patients originally diagnosed with
(Paris). 2008;69(1):7-17. PMID: 18291347 isolated growth hormone deficiency due to organic causes. Eur J Endocrinol.
2. Gregory LC, Dattani MT. The molecular basis of congenital hypopituitarism 2016;174(5):669-679. PMID: 26888628
and related disorders. J Clin Endocrinol Metab. 2020;105(6):dgz184. 12. Otto AP, Franca MM, Correa FA, et al. Frequent development of combined
PMID: 31702014 pituitary hormone deficiency in patients initially diagnosed as isolated growth
3. Fang Q, George AS, Brinkmeier ML, et al. Genetics of combined hormone deficiency: a long term follow-up of patients from a single center.
pituitary hormone deficiency: roadmap into the genome era. Endocr Rev. Pituitary. 2015;18(4):561-567. PMID: 25315032
2016;37(6):636-675. PMID: 27828722 13. Molitch ME, Clemmons DR, Malozowski S, Merriam GR, Vance ML;
4. Tanriverdi F, Dokmetas HS, Kebapci N, et al. Etiology of hypopituitarism Endocrine Society. Evaluation and treatment of adult growth hormone
in tertiary care institutions in Turkish population: analysis of 773 patients deficiency: an Endocrine Society clinical practice guideline. J Clin Endocrinol
from Pituitary Study Group database. Endocrine. 2014;47(1):198-205. Metab. 2011;96(6):1587-1609. doi: PMID: 21602453
PMID: 24366641 14. Cerbone M, Guemes M, Wade A, Improda N, Dattani M. Endocrine
5. Regal M, Paramo C, Sierra SM, Garcia-Mayor RV. Prevalence and incidence morbidity in midline brain defects: differences between septo-optic dysplasia
of hypopituitarism in an adult Caucasian population in northwestern Spain. and related disorders. EClinical Medicine. 2020;19:100224. PMID: 32140665.
Clin Endocrinol (Oxf). 2001;55(6):735-740. PMID: 11895214 15. Sbardella E, Pozza C, Isidori AM, Grossman AB. Endocrinology and
6. Muller HL, Emser A, Faldum A, et al. Longitudinal study on growth and body adolescence: dealing with transition in young patients with pituitary
mass index before and after diagnosis of childhood craniopharyngioma. J Clin disorders. Eur J Endocrinol. 2019;181(4):R155-R171. PMID: 31370006
Endocrinol Metab. 2004;89(7):3298-3305. PMID: 165240606 16. Nabhan ZM, Dimeglio LA, Qi R, Perkins SM, Eugster EA. Conjugated oral
7. Mehta A, Hindmarsh PC, Stanhope RG, Brain CE, Preece MA, Dattani versus transdermal estrogen replacement in girls with Turner syndrome:
MT. Is the thyrotropin-releasing hormone test necessary in the diagnosis of a pilot comparative study. J Clin Endocrinol Metab. 2009;94(6):2009-2014.
central hypothyroidism in children. J Clin Endocrinol Metab. 2003;88(12):5696- PMID: 19318455
5703. PMID: 14671155 17. Mauras N, Torres-Santiago L, Santen R, et al. Impact of route of
8. Mehta A, Hindmarsh PC, Mehta H, et al. Congenital hypopituitarism: administration on genotoxic oestrogens concentrations using oral vs
clinical, molecular and neuroradiological correlates. Clin Endocrinol (Oxf). transdermal oestradiol in girls with Turner syndrome. Clin Endocrinol (Oxf).
2009;70(1):96-103. PMID: 19320653 2019;90(1):155-161. PMID: 30281805
9. Cerbone M, Dattani MT. Progression from isolated growth hormone 18. Kalra S, Zargar AH, Jain SM, et al. Diabetes insipidus: the other diabetes.
deficiency to combined pituitary hormone deficiency. Growth Horm IGF Res. Indian J Endocrinol Metab. 2016;20(1):9-21. PMID: 26904464
2017;37:19-25. PMID: 29107171
10. Blum WF, Deal C, Zimmermann AG, et al. Development of additional
pituitary hormone deficiencies in pediatric patients originally diagnosed
with idiopathic isolated GH deficiency. Eur J Endocrinol. 2014;170(1):13-21.
PMID: 24088548

222  ENDO 2021 • Endocrine Case Management

chased by Dr. Khalid H. Seedahmed, Jr., MRCP PGDip, CertEndocrinology SA - ID 283


Pediatric Bone Fragility: When
to Worry and What to Do?
Laura K. Bachrach, MD. Division of Endocrinology, Stanford University School of Medicine,
Stanford, CA; E-mail: lkbach@stanford.edu

Learning Objectives that the diagnosis of pediatric osteoporosis be


restricted to those with a significant fracture
As a result of participating in this session, learners
history.3 The armamentarium to treat pediatric
should be able to:
osteoporosis is still limited.4,5 Bisphosphonates
• Identify the risk factors for pediatric bone have proved helpful in reducing fractures in both
fragility. primary (genetic) and most forms of secondary
osteoporosis. Low-dosage estrogen has been
• Order appropriate laboratory and radiographic
shown to increase bone mineral density (BMD)
studies to assess fracture risk.
in hypothalamic amenorrhea (eating disorders,
• Discuss the indications for bisphosphonate or exercise-associated amenorrhea), but the impact
sex steroid therapy in pediatric patients. on fractures remains uncertain. In the absence of
randomized controlled trials, the choice of drug,
dosage, and duration of treatment for younger
patients remain based on expert opinion.
Main Conclusions
The foundation for lifetime bone strength is
established in the first 2 decades. Bone size, Significance of the
geometry, microarchitecture, and material
properties each contribute to resistance of bone to
Clinical Problem
fracture.1 The expected gains in bone strength can Pediatric providers are increasingly asked to
be compromised during this critical developmental address bone fragility in younger patients.
period by genetic and acquired disorders. Early Referrals are made for children who already
identification and treatment of younger patients at have had fractures and those considered “at risk”
risk for bone fragility is key to avoiding fractures because of conditions associated with osteoporosis
not only in childhood but throughout later life. (Box). Although attention to early skeletal health
Each 10% (1 SD) gain in peak bone strength has is encouraging, the provider faces persistent
been estimated to reduce an individual’s lifetime dilemmas related both to the evaluation and
fracture risk by 50%.2 treatment of bone fragility. There is no established
There have been several advances in pediatric bone “fracture threshold” based on DXA
the assessment and treatment of pediatric or peripheral quantitative CT. Additionally, there
bone fragility in the past 30 years. Despite is no tool comparable to FRAX used in adults to
improvements in DXA, peripheral quantitative weight the clinical factors that aid in calculating
CT, and high-resolution peripheral quantitative fracture risk. For these reasons, pediatric patients
CT, these have proven to be imperfect surrogate who have not yet sustained a fracture are usually
predictors of fracture risk. For this reason, a managed with general bone health measures only.
panel of pediatric bone experts recommended These include optimizing nutrition, calcium and

ENDO 2021 • Pediatric Endocrinology  223

chased by Dr. Khalid H. Seedahmed, Jr., MRCP PGDip, CertEndocrinology SA - ID 283


vitamin D intake, physical activity, and treatment functional endpoints (bone pain, fractures) are
of the underlying chronic diseases, while lacking.
minimizing exposure to osteotoxic drugs. • Randomized controlled trials of sex steroids
For patients meeting the criteria for pediatric for hypothalamic amenorrhea have been short
osteoporosis (based on fractures), there are still term with biochemical bone markers and
controversies regarding the choice of optimal BMD as outcome measures.
drug, dosage, and duration of treatment. Data
on the safety and efficacy of bisphosphonates are
inadequate to recommend these agents for routine
Strategies for Diagnosis,
use; offering these drugs on a compassionate use
basis to treat fractures and bone pain is accepted Therapy, and/or Management
as standard of care. Intravenous zoledronic acid Defining Pediatric Osteoporosis
has replaced pamidronate in many centers because
According to the International Society of Clinical
of the shorter infusion time and longer duration
Densitometry (ISCD) guidelines, the diagnosis
of action of this newer drug. Pediatric dosing of
of osteoporosis is reserved for patients with the
zoledronic acid ranges from 0.05 to 0.10 mg/kg
following:
per year delivered every 6 months; pamidronate
dosing ranges from 4 to 9 mg/kg per year divided • One or more vertebral compression fractures
every 2 to 4 months depending on age. The occurring without local disease or major
maximal gains from bisphosphonate therapy are trauma (with or without densitometry
achieved by 2 to 4 years. However, it appears risky measures)
to institute a “drug holiday” for growing patients
who face ongoing threats from chronic conditions OR
such as osteogenesis imperfecta or Duchenne • Low bone mineral content or density for age
muscular dystrophy. Fractures can occur at the (a standard deviation for age or Z-score less
interface between older “treated” bone and the than –2) plus 2 or more long bone fractures by
weaker, newly acquired distal bone. To avoid age 10 years or 3 or more by age 19 years
over-suppression of bone turnover, the dosage
of bisphosphonates used for the initial treatment These criteria reflect the fact that BMD Z-scores
is typically halved during the maintenance phase. vary depending on the normative reference
Data on safety and efficacy of newer agents, database used. They also account for the
including denosumab and sclerostin antibody, observation that fragility fractures can occur
remain quite limited. Anabolic agents, including at a relatively “better” bone mineral content
teriparatide, have a black box warning prohibiting or BMD, particularly in patients treated with
use in patients with open epiphyses. glucocorticoids. The guidelines emphasize the
differing clinical significance of fractures at various
skeletal sites. Fractures of long bones but not
Barriers to Optimal Practice fingers and toes are important in decision making
about treatment.
• Challenges to deciding when to use a
pharmacologic agent to treat bone fragility in
young patients. Evaluation
• Bone densitometry alone cannot be used to For Whom?
diagnose pediatric osteoporosis. A bone health assessment is appropriate for
• Uncertainty about choice of osteoporosis drugs patients with a diagnosis of the genetic disorders
because of randomized controlled trials with or acquired diseases associated with bone fragility,

224  ENDO 2021 • Endocrine Case Management

chased by Dr. Khalid H. Seedahmed, Jr., MRCP PGDip, CertEndocrinology SA - ID 283


Box. Causes of Osteoporosis several of which are listed in the Box. Other
candidates include seemingly healthy children with
Causes of Primary Osteoporosis
recurrent fractures. Deciding how much evaluation
• Osteogenesis imperfecta
is needed in the latter situation is challenging since
• Syndromes:
fractures are a common injury in children.6 Features
ѓ Bruck syndrome
of a concerning fracture history include low-trauma
ѓ Marfan syndrome
injuries (from standing height or less) or vertebral or
ѓ Ehlers-Danlos syndrome
femur fractures since injuries at these skeletal sites
ѓ Osteoporosis-pseudoglioma syndrome
are rare in healthy youth.
• Paget disease
• Metabolic syndromes: How?
ѓ Wilson disease
A detailed clinical history should be obtained
ѓ Homocystinuria about prior bone pain or fractures and how the
ѓ Menkes kinky hair syndrome injuries occurred. The family history should
ѓ Galactosemia explore if relatives have had recurrent fractures or
• Idiopathic juvenile osteoporosis hip fractures. Baseline laboratory studies include
measurement of serum 25-hydroxyvitamin D,
Causes of Secondary Osteoporosis
calcium, phosphate, creatinine, PTH, antibodies
• Chronic inflammatory disorders
ѓ Inflammatory bowel disease
for celiac disease, and urinary calcium excretion.
ѓ Juvenile idiopathic arthritis
Testing for thyroid, growth, sex hormone
ѓ Celiac disease
abnormalities, or underlying pathogenic
genetic variants may be appropriate depending
ѓ Cystic fibrosis
on the clinical history and physical exam.
• Immobilization
Urine and blood biochemical markers of bone
ѓ Cerebral palsy
turnover have proven less useful surrogate
ѓ Myopathic disease (eg, Duchenne muscular dystrophy)
measures of bone health because of their greater
ѓ Epidermolysis bullosa
interindividual and intraindividual variability in
• Endocrine disturbance
childhood and adolescence. These tests are not
ѓ Turner syndrome
recommended for diagnosis or decision making
ѓ Anorexia nervosa
for treatment, but they may be useful to monitor
ѓ Type 1 diabetes mellitus
for over-suppression of bone turnover during
• Cancer and therapies with adverse effects on bone health
pharmacologic therapy. The most commonly
ѓ Acute lymphoblastic leukemia
measured parameters are procollagen type I
ѓ Post chemotherapy for childhood cancer
N-terminal propeptide (PINP; a marker of bone
ѓ Post transplant (non-renal)
formation) and serum collagen type I cross-linked
• Hematologic disorders
C-telopeptide (CTx; a marker of bone resorption).
ѓ Thalassemia
Bone densitometry may be included as part
ѓ Sickle-cell disease
of the evaluation; in clinical practice this is
• Drug-induced typically done using DXA. The ISCD guidelines
ѓ Glucocorticoids
recommend performing an initial DXA when
ѓ Immunosuppressants the patient might benefit from intervention and
ѓ Anticonvulsant agents when the densitometry results would influence
ѓ Medroxyprogesterone management.3 There are now robust pediatric
reference data to allow calculation of standard
deviation Z-scores for age, sex, ethnicity, and
height.7 As discussed above, however, the

ENDO 2021 • Pediatric Endocrinology  225

chased by Dr. Khalid H. Seedahmed, Jr., MRCP PGDip, CertEndocrinology SA - ID 283


diagnosis of pediatric osteoporosis can be made daily) is recommended. The 25-hydroxyvitamin D
only in the presence of a significant fracture measurement should be repeated at the end of
history. For this reason, determining whether therapy since higher dosages may be needed in
a vertebral fracture has occurred is often an patients with obesity, cystic fibrosis, or other
important part of the evaluation. Since 40% of malabsorption disorders. Activity should be
vertebral fractures are asymptomatic, the absence encouraged as tolerated but may be limited in
of back pain does not rule out this injury. Expertise those with immobilization disorders. Control of
is required to differentiate vertebral fractures the underlying chronic disease is key to improving
from normal variants of pediatric vertebral bone health. The use of biologics instead of
development. Lateral thoracolumbar spine glucocorticoids to reduce inflammatory cytokines
radiographs using the Genant semiquantitative has proven effective in enhancing BMD.9 Sex
scoring method to calculate the spinal deformity steroid replacement is appropriate for patients
index are considered the “gold standard” to identify with gonadal failure from chemotherapy, Turner
vertebral fractures.8 Vertebral fractures can also syndrome, or other causes and appears to enhance
be detected using vertebral fracture assessment BMD. Less is known about the impact of hormone
software available on newer DXA equipment. replacement on fracture risk.

Drug Therapy
Management
Osteoporosis in pediatric patients often results
General Measures from a failure to gain bone strength rather than
A key first step is to address all risk factors for from the accelerated bone loss experienced by elderly
poor bone health. Although this seems obvious, and postmenopausal patients. For this reason,
attention to these general measures can be it would be ideal to treat with an anabolic agent
overlooked in the care of these complex disorders. to build bone. The most commonly prescribed
Nutritional goals include achieving a healthy BMI of these, synthetic parathyroid hormone, carries
since both overweight and underweight have a black box warning against its use in growing
been linked to pediatric fractures. Counseling patients.
should focus on ensuring adequate intake of Bisphosphonates are the most commonly
calories, protein, calcium, and vitamin D. The prescribed pharmacologic agents for both primary
recommended daily allowance for calcium is and secondary osteoporosis.4,5 None are FDA-
700 mg for ages 1 to 3 years, 1000 mg for ages approved for use in pediatric patients because of
4 to 8 years, and 1300 mg for ages 9 to 18 years. a lack of adequate randomized controlled trials in
An 8-oz serving of milk provides about 300 mg of younger patients. The outcome measures in most
calcium, and many children and teens fall short of studies have been surrogates (such as BMD) rather
the 2 to 4 servings needed to meet optimal intake. than the functional variables (bone fracture or
Oral calcium supplements are recommended to pain). Bisphosphonates appear to be effective in
fill the gap. Formulations vary in the amount most causes of primary and secondary osteoporosis
of elemental calcium contained. For example, with the exception of anorexia nervosa.
calcium carbonate is only 40% elemental calcium, The benefits of sex steroid therapy to increase
so a 100-mg chewable provides only 400 mg. BMD or reduce fractures to treat “hypothalamic
Although the definition of vitamin D “sufficiency” amenorrhea” (anorexia nervosa, exercise associated
is debated, supplementation with at least 600 IU amenorrhea) remain controversial. Bone fragility
daily is appropriate for all children older than in these disorders reflects nutritional (energy)
1 year of age. If the serum 25-hydroxyvitamin D deficits, as well as deficits in both estrogen and
concentration is less than 20 ng/mL (<50 nmol/L), the insulinlike growth factors (IGFs). Nutritional
a 6- to 12-week course of high-dosage vitamin D therapy is recommended as first-line treatment
supplementation (50,000 IU weekly or 4000 IU

226  ENDO 2021 • Endocrine Case Management

chased by Dr. Khalid H. Seedahmed, Jr., MRCP PGDip, CertEndocrinology SA - ID 283


with reduction in excessive activity if present.10 triad.” The diagnosis is one of exclusion, and it is
Supplementation with both sex steroids and important to exclude other causes of amenorrhea,
IGF-1 or use of low-dosage sex steroids have including primary ovarian insufficiency or
shown positive effects on BMD.11 These may be hyperprolactinemia (Answer C). Ruling out
reasonable to initiate if nutritional rehabilitation vitamin D insufficiency is reasonable, although
has been unsuccessful. More research is needed these athletes are often taking vitamins in an
to clarify the benefits of sex hormone therapy to effort to maintain a healthy body. Screening
reduce fracture risk for these patients. for celiac disease is always reasonable in the
setting of fractures. Performing a pregnancy test
(Answer E) is appropriate in any adolescent girl
Clinical Case Vignettes with amenorrhea, but it might not be done as a
Case 1 first-line evaluation given the history that her
A 14-year-old girl with primary amenorrhea hours in school and dance practice leave her no
comes with her parents to discuss bone health time for socializing.
after sustaining a stress fracture of her right foot. Perhaps the most controversial question is
She is a ballet dancer hoping to be accepted into the role for bone densitometry in the evaluation
an elite dance troupe. Her days are filled with of this patient. If DXA were to be performed,
school and dance practice, leaving little time for the preferred regions of interest would be
socializing with peers or even family. The foot whole body and spine (Answer B) and not spine
fracture occurred after 6 months of beginning toe and hip (Answer A). Whether DXA findings
dance maneuvers and appears to be healing slowly. would change management is not clear. If
On physical examination, she has Tanner results were normal, would they provide false
stage 4 breast and pubic hair development but has reassurance about the risks for future fractures?
not had menarche. Her height plots at the 40th If abnormal, would this increase adherence to
percentile and her BMI at the 10th percentile. recommendations for skeletal health?
Her parents request further evaluation and In this case, the parents insisted upon
treatment, so she can safely continue to return to DXA, which showed whole-body and spine
her training. BMD values of –0.8 and –0.7, respectively.
Laboratory test results were normal, including
Which of the following is the best a 25-hydroxyvitamin D value of 32 ng/mL
next step in her evaluation? (79.9 nmol/L) and negative celiac antibodies. She
A. DXA of spine and hip had a spontaneous period during the 4-month
hiatus from dancing while her stress fracture
B. DXA of whole body and spine
healed. She is being allowed to return to dance
C. Prolactin, LH, FSH, estradiol, and with a graded exercise program closely supervised
25-hydroxyvitamin D measurements and celiac by the sports medicine team.
screen
D. Lateral spine x-ray
Case 2
E. Serum pregnancy test
You are asked to see a 14-year-old boy with a
Answer: C) Prolactin, LH, FSH, estradiol, history of 7 long bone fractures. At age 3 years, he
25-hydroxyvitamin D measurements and celiac screen sustained a spiral femur fracture when an older
child landed on him at play. Six weeks later, he
This patient’s stress fracture represents an overuse fell backwards with his heavy leg cast and suffered
injury in the setting of intensive training, chronic a left arm greenstick fracture. At age 5 years, he
energy deficits, and hypogonadism. This clinical fractured both radius and ulna while sliding; at
scenario is often referred to as “the athletic

ENDO 2021 • Pediatric Endocrinology  227

chased by Dr. Khalid H. Seedahmed, Jr., MRCP PGDip, CertEndocrinology SA - ID 283


age 7 years, he fractured his collarbone playing had 1 fracture. This increase in fracture rates
football; at age 10 years, he broke his humerus persists after controlling for risk-taking behavior,
rollerblading; at age 12 years, he fractured his suggesting there may be underlying differences in
wrist rollerblading, and at 14 years, he had a radial bone strength among those with fractures.
fracture playing basketball. All injuries healed The concerning features in this teen’s history
normally and left no skeletal deformities. He has are the large number of fractures beginning before
no back pain, hearing deficits, or dental caries. He age 5 years, the occurrence of a femur facture, and
consumes a normal diet, has no gastrointestinal the low-trauma nature of the first arm fracture
complaints, and takes no medications. (fall from standing height). Two-thirds of the
On physical examination, he has normal height fractures in healthy youth involve the distal
at the 95th percentile, BMI at the 70th percentile, radius and hand. By contrast, femur, hip, and
and Tanner stage 4 genitalia. His sclerae are white spine fractures occurring without major trauma
and teeth are noncarious. There is no tenderness and suggest abnormal bone fragility. There are enough
no deformities on skeletal exam, including the spine. concerning features in this patient’s history to
His 8-year-old brother has had 3 fractures, but argue against reassurance alone (Answer A).
there is otherwise no family history of recurrent DXA of the whole body and spine (Answer B),
fractures or osteoporosis in his other 3 siblings, even if abnormal, does not establish a diagnosis
parents, or extended family. Prior laboratory of osteoporosis. DXA was indeed performed in
tests show normal levels of calcium, phosphate, this patient and showed spine and whole-body
PTH, 25-hydroxyvitamin D, complete blood cell BMD Z-scores of –1.7 and –1.1, respectively.
count, and erythrocyte sedimentation rate. Celiac These were not corrected for height and may
antibodies are negative. have overestimated his actual areal BMD. A
bone biopsy (Answer C) would be indicated only
Which of the following evaluations if noninvasive testing were nondiagnostic. His
(if any) should be performed now? growth, BMI, and Tanner staging are normal,
A. Reassurance only; fractures very common making assessment of sex steroids (Answer D) less
in boys and all of his fractures occurred with helpful. Genetic testing (Answer E) was performed
trauma in this patient, which confirmed a pathogenic
B. DXA of whole body and spine variant in the COL1A1 gene. He was treated with
bisphosphonates for 3 years, stopped playing
C. Bone biopsy
contact sports, and remained fracture-free during
D. LH, FSH, prolactin, and testosterone the subsequent 4 years.
measurements
E. Genetic studies for osteogenesis imperfecta
Case 3
Answer: E) Genetic studies for osteogenesis imperfecta A 9-year-old boy with Duchenne muscular
Fractures in otherwise healthy youth are a dystrophy (DMD) is referred for discussion of
common injury. By 16 years of age, up to 40% bisphosphonate therapy to prevent fractures.
of otherwise healthy girls and 64% of boys will DMD was diagnosed at age 5 years, and
have experienced at least 1 broken bone, with deflazacort, 0.8 mg/kg, was started at that time. He
a peak incidence in the peripubertal years.6 remains ambulatory at home but uses a wheelchair
Epidemiological studies have shown that these at school and participates in adaptive physical
injuries are not distributed evenly; some children education. He has never had a long bone fracture
fracture more than once, while others escape and does not complain of back pain. Serial DXAs
injury. The risk of a second fracture has been at another facility show a decline in spine BMD
estimated to double for a child who has already Z-scores from –1.7 to –2.1 over the past 2 years.

228  ENDO 2021 • Endocrine Case Management

chased by Dr. Khalid H. Seedahmed, Jr., MRCP PGDip, CertEndocrinology SA - ID 283


His parents are concerned about his fracture risk be treated and when? At present, the standard of
after reading posts on a DMD parents’ website, care has been to initiate pharmacologic therapy
and they want to start medication now to prevent only after a first fracture has occurred in the hopes
their son from having a broken bone. of preventing more. Bisphosphonates (Answers
A, B, and C) would not be recommended based
Which of the following do on changes in BMD alone. Furthermore, oral
you advise at this visit? bisphosphonates such as alendronate have not
A. Start once-weekly oral alendronate been shown to be as effective as intravenous agents
B. Start intravenous zoledronic acid in preventing vertebral fractures in children.4
Given the increased incidence of vertebral fracture
C. Start intravenous pamidronate
in patients receiving glucocorticoids, alendronate
D. Obtain lateral spine x-ray would not be a preferred drug.
E. Repeat DXA in 6 months and treat if Z-score Primary prevention drug trials are worthy
is less than –2.5 of future investigation if the population of
children to be targeted can be determined.
Answer: D) Obtain lateral spine x-ray
Valuable insights into predicting fractures have
It is well established that the reduced mobility, come from longitudinal observational studies
chronic inflammation, delayed growth and conducted by the Steroid-Associated Osteoporosis
puberty, and exposure to long-term, high-dosage in the Pediatric Population (STOPP) network,
glucocorticoids in patients with DMD create a a consortium of pediatric bone centers across
clinical “perfect storm” for bone fragility. Low- Canada. This research has identified several
impact fractures both in long bones and in the clinical and laboratory correlates of fragility
vertebrae (once glucocorticoids are initiated) are fractures in patients with acute lymphoblastic
common. The lack of skeletal loading contributes leukemia, nephrotic syndrome, and rheumatologic
to narrower bones with thinner cortices. disorders. At the time of diagnosis, 16% of children
Glucocorticoids appear to have a greater impact with acute lymphoblastic leukemia had at least 1
on trabecular bone in the spine. Similar risk fracture. The cumulative risk of incident vertebral
factors for osteoporosis complicate other chronic fracture and nonvertebral fracture was 32.5%
disorders characterized by inflammation, growth and 23%, respectively, over the next 6 years; 71%
delay, reduced activity, and pharmacologic dosages occurred in the first 2 years of treatment.12 The
of glucocorticoids. The decline in BMD Z-score presence of even 1 mild grade 1 vertebral fracture
in this patient is not unexpected and reflects the predicted future fractures, a phenomenon called
slower rate of growth and bone mineral accrual the “vertebral fracture cascade.” Cumulative,
in DMD, perhaps compounded by accelerated average daily dose, and short bursts of high-dose
bone loss from the glucocorticoids. However, the “pulse therapy” of glucocorticoids were strong
change in BMD Z-score alone is not a sufficient correlates of fracture. Other factors associated
indication for drug therapy (thus, Answer E is with vertebral fracture include younger age, lower
incorrect). The best next step is a lateral spine BMD, cranial and spinal radiation, endocrine
radiograph (Answer D) or vertebral fracture deficits, decreased physical activity, and vitamin D
assessment by DXA. Finding a vertebral fracture deficiency. Randomized controlled trials could be
would establish the diagnosis of osteoporosis and designed to target those at highest risk for incident
would be an indication to offer bisphosphonates. fracture and those least likely to exhibit healing
The question raised by this case is the (reshaping) of the vertebral deformities.
indication for bisphosphonates as primary
prevention for patients at high risk such as this
one. Are there sufficient data to guide who should

ENDO 2021 • Pediatric Endocrinology  229

chased by Dr. Khalid H. Seedahmed, Jr., MRCP PGDip, CertEndocrinology SA - ID 283


Case 4  New Content  Answer: D) Order chemistry panel and
25-hydroxyvitamin D measurement
A 6-month-old boy is seen for management
and obtain lateral spine x-ray
of osteogenesis imperfecta due to a known
pathogenic variant in the COL1A1 gene Close follow-up of this child is warranted because
(heterozygous c.1002delT), identical to that of his known pathogenic variant in the COL1A1
confirmed in his father and 5-year-old brother. gene. Initiation of bisphosphonate therapy in
Both of these family members have had repeated the first few months of life would be appropriate
low-trauma fractures, and his brother has been to reduce vertebral compression fractures (that
on bisphosphonate therapy since age 3 years. The compromise spine growth) and long bone fragility
patient had minimal lower-extremity bowing in fractures once they occur. However, given the
utero, and postpartum genetic testing confirmed variability in phenotype even within families
that he carried the COL1A1 pathogenic variant. His with the same genotype, it is reasonable to start
mother reports that she does not think he has had therapy only if fractures occur (thus, Answers B
any postnatal fractures. However, she is concerned and C are incorrect). This child has not had any
that he will not bear weight on his lower confirmed long bone fractures postnatally. His
extremities when held erect. He is not yet sitting hypotonia is concerning but does not appear to
or rolling over. Otherwise he is a healthy, happy be due to bone pain or deformities. His history
baby. He is exclusively breastfed every 2 hours, but of exclusive breastfeeding without vitamin D
he gags when he is offered solid foods. He takes supplementation is concerning for possible
no vitamins or other medications. He stools only vitamin D deficiency rickets. Lateral spine x-ray is
every 8 to 9 days and has foul smelling gas. His also indicated at this age to rule out asymptomatic
review of systems is otherwise negative. vertebral fractures. Thus, the best course of action
On physical examination, he is a smiling infant for this patient is to order a chemistry panel and
with a length and weight at the 25th percentile. 25-hydroxyvitamin D measurement and obtain
The anterior fontanelle is patent, and the posterior lateral spine x-ray (Answer D). Laboratory studies
fontanelle is closed. He has white sclerae and no confirmed profound vitamin D deficiency rickets
teeth. He has diffuse hypotonia and does not bear with a 25-hydroxyvitamin D concentration of 4
weight when held up. He has no obvious bowing ng/mL (10 nmol/L); elevated alkaline phosphatase
or spine deformities. concentration of 419 U/L (7.00 µkat/L) (reference
range, 38-126 U/L [0.63-2.10 µkat/L]); and
Which of the following is the normal calcium, magnesium, and phosphate
best course of action now? levels. If vertebral fractures are found, it would be
A. Refer for physical therapy and nutritional appropriate to initiate bisphosphonate therapy,
counseling but only after vitamin D stores have been repleted
B. Obtain skeletal survey, including lateral spine, (25-hydroxyvitamin D >20 ng/mL [>50 nmol/L])
and initiate intravenous pamidronate to reduce the risk of postinfusion hypocalcemia
C. Order chemistry panel and 25-hydroxyvitamin and exacerbation of osteomalacia. Simply referring
D measurement and initiate intravenous for physical therapy and nutritional counseling
pamidronate (Answer A) or ordering laboratory testing and
referring for physical therapy (Answer E) are
D. Order chemistry panel and 25-hydroxyvitamin
inadequate.
D measurement and obtain lateral spine x-ray
E. Order chemistry panel and
25-hydroxyvvitamin D measurement and start
physical therapy

230  ENDO 2021 • Endocrine Case Management

chased by Dr. Khalid H. Seedahmed, Jr., MRCP PGDip, CertEndocrinology SA - ID 283


Case 5  New Content  underscoring the importance of the muscle-
bone unit. These changes in bone geometry have
A 7-year-old boy with profound developmental
been confirmed by 3-dimensional imaging by
delay and epilepsy is referred for a bone health
peripheral quantitative CT. Two-dimensional
evaluation. He is nonambulatory and treated with
assessments of BMD by DXA will be low for age
anticonvulsant medication and a ketogenic diet to
as a result of the smaller, narrower bones. If DXA
control his seizures. He receives supplementation
were to be ordered, the distal lateral femur BMD
of 1000 IU vitamin D daily and has a
is recommended as the best region of interest
25-hydroxyvitamin D concentration of 32 ng/mL
to predict lower-extremity fracture. However,
(80 nmol/L). He has never had a fracture, but his
finding “low bone density for age” will most likely
parents and his pediatrician are very anxious he
add to parental concern but not necessarily guide
will break a bone. They request a bone density test
treatment. Bisphosphonates are generally reserved
and treatment plan to prevent injury.
for youth who have sustained a fragility fracture.
On physical examination, he is markedly
In contrast to DXA results (Answers B, C,
hypotonic and unable to sit or stand unassisted.
and D), a lateral spine x-ray (Answer E) would
His length is at the 5th percentile, and weight and
influence treatment. Youth with immobilization
BMI are at the 10th percentile. He has no obvious
disorders (cerebral palsy, congenital myopathies)
long bone or spine abnormalities or tenderness.
are at risk for vertebral fractures even if they
Which of the following diagnostic are not on glucocorticoid therapy. A lateral
studies should be recommended? spine radiograph is recommended by age 6 to 8
years and at intervals thereafter until the end of
A. No imaging
growth; an x-ray is indicated whenever bone pain
B. DXA of spine, total body, and distal lateral is suspected.4 Pursuing no imaging (Answer A)
femur would be inappropriate.
C. DXA of distal lateral femur and lateral spine In terms of treatment, physical therapy is
x-ray warranted to optimize skeletal loading as much
D. DXA of spine, total body, and hip as possible. Vibratory platform therapy has been
E. Lateral spine x-ray shown to increase bone mineral in small pilot
studies. Options for pharmacologic therapy are
Answer: E) Lateral spine x ray limited. There is no approved anabolic agent for
use in pediatrics to enhance the compromised
Patients with immobilization disorders are bone geometry; bisphosphonate therapy is offered
at increased risk for fragility fractures due to if low-trauma long bone or vertebral fractures
impaired gains in bone quality and quantity. occur.
The absence of skeletal loading results in long
bones that are narrower with a thinner cortex,

References
1. Gordon CM, Zemel BS, Wren TA, et al. The determinants of peak bone 4. Ward LM, Konji VN, Ma J. The management of osteoporosis in children.
mass. J Pediatr. 2016;180:261-269. PMID: 27816219 Osteoporosis Int. 2016;27(7):2147-2179. PMID: 27125514
2. Rizzoli R, Bianchi ML, Garabedian M, McKay HA, Moreno LA. Maximizing 5. Grover M, Bachrach LK. Osteoporosis in children with chronic illnesses:
bone mineral mass gain during growth for the prevention of fractures in the diagnosis, monitoring, and treatment. Curr Osteoporos Rep. 2017;15(4):271-
adolescent and the elderly. Bone. 2010;46(2):294-305. PMID: 19840876 282. PMID: 28620868
3. Gordon CM, Leonard MB, Zemel BS; International Society for Clinical 6. Clark EM. The epidemiology of fractures in otherwise healthy children. Curr
Densitometry. 2013 Pediatric Position Development Conference: executive Osteoporos Rep. 2014;12(3):272-278. PMID: 24973964
summary and reflections [published correction appears in J Clin Densitom. 7. Zemel BS, Leonard MB, Kelly A, et al. Height adjustment in assessing dual
2014;17(4):517]. J Clin Densitom. 2014;17(2):219-224. PMID: 24657108 energy x-ray absorptiometry measurements of bone mass and density in
children. J Clin Endocrinol Metab. 2010;95(3):1265-1273. PMID: 20103654

ENDO 2021 • Pediatric Endocrinology  231

chased by Dr. Khalid H. Seedahmed, Jr., MRCP PGDip, CertEndocrinology SA - ID 283


8. Genant HK, Wu CY, van Kuijk C, Nevitt MC. Vertebral fracture assessment 11. Ackerman KE, Singhal V, Baskaran C, et al. Oestrogen replacement improves
using a semiquantitative technique. J Bone Miner Res. 1993;8(9):1137-1148. bone mineral density in oligo-amenorrhoeic athletes: a randomised clinical
PMID: 8237484 trial. Br J Sports Med. 2019;53(4):229-236. PMID: 30301734
9. Griffin LM, Thayu M, Baldassano RN, et al. Improvements in bone density 12. Ward LM, Ma J, Lang B, Ho J, et al; Steroid-Associated Osteoporosis in the
and structure during anti-TNF-α therapy in pediatric crohn’s disease. J Clin Pediatric Population (STOPP) Consortium. Bone morbidity and recovery in
Endocrinol Metab. 2015;100(7):2630-2639. PMID: 25919459 children with acute lymphoblastic leukemia: Results of a six-year prospective
10. Southmayd EA, Hellmers AC, De Souza MJ. Food versus pharmacy: study. J Bone Miner Res. 2018;33(8):1435-1443. PMID: 29786884
assessment of nutritional and pharmacological strategies to improve
bone health in energy-deficient exercising women. Curr Osteoporos Rep.
2017;15(5):459-472. PMID: 28831686

232  ENDO 2021 • Endocrine Case Management

chased by Dr. Khalid H. Seedahmed, Jr., MRCP PGDip, CertEndocrinology SA - ID 283


Diabetes Insipidus in Children
Craig A. Alter, MD. Division of Endocrinology and Diabetes, Children’s Hospital of
Philadelphia, Pearlman School of Medicine at the University of Pennsylvania, Philadelphia,
PA; E-mail: alterc@email.chop.edu

Learning Objectives possibility of DI when there is clinical suspicion


for disease.
As a result of participating in this session, learners
Once the diagnosis of central DI is established,
should be able to:
an organized approach to diagnosing the cause is
• Determine when and when not to perform a crucial. The most common causes are idiopathic
water-deprivation study. (likely hypophysitis), tumor, histiocytosis, germ-
cell tumors, structural defects, and congenital
• Formulate a plan to determine the cause of
(genetic) defects.
central diabetes insipidus (DI).
Treatment may involve ensuring specific
• Describe the pros and cons of various therapies water intake, desmopressin (oral, intranasal, or
for DI. subcutaneous formulations), and occasionally
diuretics.

Main Conclusions
Significance of the
Although DI is considered as a possible
diagnosis when children have polyuria, other
Clinical Problem
diagnoses should be considered. In addition After establishing a diagnosis of central DI, MRI
to hyperglycemia, other conditions such as of the pituitary is mandatory if the etiology is not
thyrotoxicosis, hypercalcemia, and hypokalemia known. The pediatric endocrinologist is often
can lead to polyuria. Protein malnutrition can faced with various findings on MRI and must
produce dilute urine. Some medications such as decide when repeat scanning is needed or whether
lithium can lead to poor renal water reabsorption. a central nervous system biopsy is mandated.
Psychogenic polydipsia is the most common This chapter will outline an organized approach
state that must be distinguished from central to using MRI to formulate a plan. Correlations
DI. In children with polyuria and polydipsia, of MRI findings to clinical parameters will be
a serum osmolality exceeding 300 mOsm/kg reviewed. In addition, the pros and cons of
with a simultaneous urine osmolality less than choosing different formulations of desmopressin
300 mOsm/kg confirms the diagnosis of DI. The will be discussed.
threshold of urine osmolality used to exclude DI
ranges from 600 to 750 mOsm/kg. In clinical Barriers to Optimal Practice
practice, it has been observed that some patients
whose initial urine osmolality is between these • One of the challenges in establishing the
values are subsequently diagnosed with partial etiology of central DI is that the diagnosis is
DI. Thus, it is advised to use a urine osmolality usually not known after the initial MRI.
threshold of at least 750 mOsm/kg to assess the

ENDO 2021 • Pediatric Endocrinology  233

chased by Dr. Khalid H. Seedahmed, Jr., MRCP PGDip, CertEndocrinology SA - ID 283


• The most important decision is how often acid glycosylated peptide that is synthesized
to repeat MRI and when to obtain a central along with AVP and neurophysin II in the
nervous system biopsy. hypothalamus, may prove to be useful clinically
given its longer half-life. Copeptin measurement
after infusion of hypertonic saline may be an
Strategies for Diagnosis, alternative approach to water-deprivation testing
Therapy, and/or Management in diagnosing DI, but this has not been studied in
pediatric patients.2
Introduction
Arginine vasopressin (AVP) is crucial to The Child With Polydipsia
maintaining water and osmolar homeostasis. and Polyuria
The condition of DI may be due to inability of
the kidneys to respond to AVP or to a disordered When a child presents with increased thirst or
release or deficiency of AVP (central DI). urination, there are important clues to pursue not
AVP is released in response to dehydration, only by history, but also on physical examination.
along with activation of the renin-angiotensin New-onset or worsening enuresis should be
system and thirst. The neurons that synthesize questioned. Drinking from unusual sources
AVP are hypothalamic in origin, but they such as puddles, bathtub faucets, the toilet, or
convene at the posterior pituitary. AVP is stored other people’s drinks suggests abnormal thirst.
in granules in the posterior pituitary and are Trying to quantify the liquids can be useful.
seen as the posterior pituitary “bright spot” on Fatigue is common and may be due to poor sleep,
T1-weighted MRI imaging (a bright area in the dehydration, potentially concomitant central
posterior pituitary on a noncontrast film). AVP hypothyroidism, or another systemic process.
binds to the AVP V2 receptors in the collecting Occasionally, the degree of polydipsia decreases
ducts of the renal tubules. Through a cyclic AMP without therapy, but this should not be interpreted
signal transduction mechanism, this results in as reassurance that the underlying process has
activation, transport, and insertion of aquaporin-2 resolved. Primary polydipsia can be just as potent
water channels into the membranes of the as the polydipsia from central DI. Growth chart
collecting duct. As a result, there is increased analysis is crucial to the evaluation. Unusual rashes
permeability of the collecting duct to water and or bone abnormalities may support a diagnosis
reabsorption of water back into the circulation. of Langerhans cell histiocytosis. Weight loss can
AVP also stimulates V1 α receptors on blood signify a systemic process such as a germ-cell
vessels, causing vasoconstriction. AVP is involved tumor or can result from the marked polyuria.
in von Willebrand factor production, as well as While type 1 diabetes mellitus is at the top of the
factor VIII synthesis.1 differential diagnosis list in patients with polyuria,
Blood pressure and plasma osmolality are it has usually been ruled out by the time central
maintained by a combined effort of thirst, AVP, DI is considered. Asking questions about vision
the adrenal glands, and the kidneys. AVP increases change is important, as it is common for a child
in a linear fashion in response to even a 1% and their family to be unaware of vision changes.
increase in osmolality or a 10% or more decrease For example, it would be important to know that a
in intravascular volume. For more details, please child recently diagnosed with central DI had been
see Pituitary Disorders of Childhood (Humana Press, moved to the front row in the classroom due to
2019). concerns of mild blurriness.
Given the short half-life of AVP, it is difficult
to use it in the clinical workup of a child with
symptoms of DI. However, copeptin, a 39-amino

234  ENDO 2021 • Endocrine Case Management

chased by Dr. Khalid H. Seedahmed, Jr., MRCP PGDip, CertEndocrinology SA - ID 283


Polyuria: Differential Diagnosis a psychiatric diagnosis. Psychogenic polydipsia
is often mild, but in severe cases it may result in
Although central DI is considered a possible
hyponatremic seizures.
diagnosis when children have polyuria,
other diagnoses should be considered before
evaluating for central DI. These etiologies Central DI
include endocrine and metabolic disorders, as DI is diagnosed when the serum sodium
well as medications that can lead to increased concentration exceeds 145 mEq/L (>145 mmol/L),
urine output. Hyperglycemia is a prime example. urine osmolality is less than 300 mOsm/kg, urine
Hyperglycemia with a blood glucose concentration output is persistently greater than 4 mL/kg per h,
well above the renal threshold (145-160 mg/dL and other causes of polyuria have been excluded.
[8.0-8.9 mmol/L]) leads to an osmotic diuresis Making the formal diagnosis of DI can be
that resolves with initiation of diabetes therapy. challenging and involves confirming a dilute
Other endocrine disorders are associated with urine in the setting of plasma hyperosmolality.
polyuria. Thyrotoxicosis can produce polyuria Plasma osmolality can be calculated by measuring
due to decreased expression of aquaporin water the concentration of sodium, glucose, and serum
channel expression, despite adequate amounts urea nitrogen. Sodium is the major effective
of AVP, along with increased solute excretion. plasma solute. When serum glucose and serum
Hypercalcemia, regardless of etiology, can urea nitrogen are in the normal range, the sodium
produce deficits in renal concentrating capacity. concentration correlates closely with serum
Hypokalemia can lead to defects in renal osmolality.
concentrating capacity via mechanisms that are
not completely understood but may be related pOsm = 2[sodium] + (glucose in mg/dL/18) +
to accelerated autophagy-mediated degradation (serum urea nitrogen in mg/dL/2.8)
of aquaporin-2. Protein malnutrition can also
If the glucose value is in mmol/L, then there is no
produce increased output of dilute urine.
need to divide by 18.
Diuretics can make the assessment of
In children with polyuria and polydipsia,
polydipsia and polyuria difficult. Lithium is unique
a plasma osmolality exceeding 300 mOsm/kg,
because it reduces the renal responsiveness to
reflecting hyperosmolality, with a simultaneous
AVP, leading to defects in urinary concentrating
urine osmolality less than 300 mOsm/kg confirms
capacity.
the diagnosis of DI. Additional testing can
determine whether DI is central (responsive
Hypernatremia (Hyperosmolality) to AVP) or nephrogenic (nonresponsive to
Hypernatremia in the absence of polyuria most exogenous forms of AVP). In contrast, if a
often reflects hypernatremic dehydration, as may simultaneous serum osmolality is less than
occur in the setting of febrile illness, excessive 270 mOsm/kg and the urine osmolality is
losses of body free water (eg, diarrhea, burns), or maximally concentrated, the diagnosis of DI is
impaired fluid intake. Hypernatremia can occur excluded. The threshold of urine osmolality used
when thirst is impaired or there is inability to to exclude DI ranges from 600 to 750 mOsm/kg.
request increased fluids, which may be seen in In our clinical practice, we have cared for patients
those with neurological conditions, including who were subsequently diagnosed with partial DI
brain trauma. Polydipsia in the absence of whose initial urine osmolality was between these
hypernatremia is less often related to a central values. Thus, it is advised to use a urine osmolality
lesion and may rather indicate a psychiatric threshold of at least 750 mOsm/kg to assess the
disorder. In these cases, persistently low or low- possibility of DI when there is strong clinical
normal serum sodium should raise suspicion for suspicion for disease.

ENDO 2021 • Pediatric Endocrinology  235

chased by Dr. Khalid H. Seedahmed, Jr., MRCP PGDip, CertEndocrinology SA - ID 283


Water-deprivation testing carries associated bright spot can be used to provide additional
risks (most notably dehydration) and should be evidence supporting the diagnosis of DI—either
performed in a carefully monitored setting. Water- central DI or nephrogenic DI.
deprivation studies in patients with longstanding A child who has an ectopic posterior pituitary
partial DI may not reveal classic results. In such gland recognized on T1-weighted images as an
patients, the urine may only concentrate partially ectopic posterior pituitary bright spot does not
with the addition of pharmacological AVP, even typically have central DI but is at risk for anterior
though they have central DI.3 pituitary hormone deficiencies.6,7
When there is a sellar or suprasellar mass
on MRI, an important diagnosis to consider is
Establishing the Etiology
craniopharyngioma. Calcifications strongly suggest
of Central DI craniopharyngioma. CT is more reliable than MRI
Once the diagnosis of central DI is established, in detecting calcifications. Other possibilities of
it is important, yet challenging, to determine its masses that can cause central DI include arachnoid
etiology. Any infiltrative process that involves the cyst, meningioma, teratoma, or lymphoma.
hypothalamic-pituitary axis can cause central DI, Inflammatory and infiltrative disorders including
as well as anterior pituitary hormone deficiencies. Langerhans cell histiocytosis, germ-cell tumor,
Embryologic neurodevelopmental disorders hypophysitis, or sarcoidosis can present with
presenting with anterior pituitary hormone pituitary stalk thickening (germ-cell tumors
deficiencies, such as septo-optic dysplasia, can include not only germinomas, but also teratomas,
result in central DI, even if not present clinically yolk sac tumors, choriocarcinoma, and embryonal
at birth. Central DI may occur in conjunction with carcinoma).
multiple anterior pituitary hormone deficiencies; MRI in children with central DI shows a
hence, anterior pituitary testing and surveillance thickened stalk in approximately one-third of
is warranted. The appearance of anterior pituitary cases. Of the children with a thickened stalk,
hormone deficits may occur years later, especially one study showed that 17% were eventually
if the cause is Langerhans cell histiocytosis. diagnosed with a germ-cell tumor and 17% were
Genetic defects of either AVP production or diagnosed with Langerhans cell histiocytosis. In
of its complex with neurophysin II can lead to the absence of bone lesions and a clear diagnosis
central DI. of Langerhans cell histiocytosis, serum and
After establishing a diagnosis of central DI, cerebrospinal fluid tumor markers for quantitative
MRI of the pituitary gland and hypothalamus is hCG and α-fetoprotein should be obtained at
the first step in identifying the etiology.4,5 MRI initial presentation to assess for the possibility of a
of the whole brain is insufficient to evaluate the central nervous system germ-cell tumor.
pituitary gland. A dedicated MRI of the pituitary It is recommended to measure tumor markers
should be requested with and without gadolinium and anterior pituitary hormone levels in children
contrast to optimally visualize the pituitary presenting with central DI and a thickened
gland. MRI of the pituitary without contrast infundibulum. Pituitary MRI and measurement
should show a bright appearance of the posterior of serum tumor markers and anterior pituitary
pituitary, termed the posterior pituitary “bright hormones should be performed every 3 months for
spot” on T1-weighted imaging. This bright spot is 1 to 2 years, then yearly for 2 years. Resolution of
often, although not invariably, absent in central pituitary stalk thickening is more often associated
DI of any cause. Many clinicians do not advise with lymphocytic hypophysitis and confers a
diagnosing DI based on the presence or absence of positive prognosis. Sometimes lesions from
the posterior pituitary bright spot alone. However, Langerhans cell histiocytosis can also decrease, or
in select cases, absence of the posterior pituitary increase, in size. In one study, an infundibular stalk

236  ENDO 2021 • Endocrine Case Management

chased by Dr. Khalid H. Seedahmed, Jr., MRCP PGDip, CertEndocrinology SA - ID 283


greater than 4.5 mm in thickness was correlated remains the main treatment. Desmopressin is a
with a greater likelihood of developing multiple synthetic analogue of the hormone 8-arginine
anterior pituitary hormone deficiencies. AVP and has a longer half-life than AVP. The
A normal-appearing stalk and normal anterior antidiuretic action is more specific than that of
pituitary hormone levels at the time central DI AVP, and it has less pressor activity. The duration
is diagnosed is generally reassuring, but these of action of desmopressin is variable; therefore,
findings do not always exclude an underlying the dosage and schedule are titrated to each child.
pathology. For example, the stalk was normal in Desmopressin is available for outpatient therapy
appearance in one child who was subsequently as a pill, nasal spray, or subcutaneous injection.
diagnosed with a germ-cell tumor. In children The adverse effects of desmopressin include fluid
presenting with central DI, a normal pituitary retention and hyponatremia.
stalk on MRI, and no additional central nervous
system risk factors (eg, headaches, seizures, or
Central DI Following Neurosurgery:
vision concerns), it is recommended to perform a
repeated MRI in 3 to 6 months, then every 6 to 12 The Triphasic Response
months for the next 2 years.8 Complete or transient DI can develop in children
While most cases of central DI are acquired, following neurosurgical procedures involving the
some may be genetic. Genetic cases of central DI pituitary/hypothalamus region. A well-described
are typically associated with an autosomal dominant triphasic response of DI/SIADH/DI is seen in
inheritance pattern, although rarely an autosomal the immediate postoperative period following
recessive or X-linked pattern is observed. On MRI, complete stalk resection, as observed in many
the bright spot is absent and the stalk size is normal. children following surgery for craniopharyngioma.
More than 55 pathogenic variants that cause central
DI have been described; most are associated with The Infant With Central DI
autosomal dominant inheritance. Pathogenic
variants have been described in the gene that Management of neonatal DI is particularly
encodes the AVP peptide, resulting in reduced AVP difficult, as infants cannot communicate thirst and
biological activity. are dependent on caretakers for providing fluids.
Other cases of central DI result from Often, fluids alone are used to treat DI, although
congenital or acquired anatomic disruption to the adding a thiazide class of diuretics with a low
posterior pituitary. In one series of 147 children solute intake can be effective. If desmopressin is
with central DI, 24% had an associated central added, typically a low dosage is chosen.9,10
nervous system malformation.5 Congenital
malformations include septo-optic dysplasia, Summary
holoprosencephaly, vascular malformations, and The management of DI in children involves
encephalocele. Central DI can occur following several areas of focus. The first challenge is in
traumatic brain injury or pituitary-hypothalamic establishing the diagnosis, which in itself, has risks.
surgery. Transient central DI can be seen after In addition, those with partial or evolving central
transsphenoidal pituitary surgery. DI can have reassuringly normal laboratory results
and may require prolonged water deprivation
Treatment of the Child to prove there is DI. After the diagnosis of
With Central DI central DI is established, the search for the cause
is vital. Germ-cell tumors and Langerhans cell
The approach to the treatment of children with
histiocytosis may only be found after a few years
central DI should be tailored to each family.
of frequent MRI assessments. The treatment of
Desmopressin was introduced in the 1970s and
central DI should be tailored to each patient.

ENDO 2021 • Pediatric Endocrinology  237

chased by Dr. Khalid H. Seedahmed, Jr., MRCP PGDip, CertEndocrinology SA - ID 283


Clinical Case Vignettes Laboratory test results:

Case 1 Serum sodium = 152 mEq/L (136-142 mEq/L)


(SI: 152 mmol/L [136-142 mmol/L])
A 12-year-old boy presents with excessive Serum urea nitrogen = 8 mg/dL (8-23 mg/dL)
thirst and urination of 6 months duration. He is (SI: 2.9 mmol/L [2.9-8.2 mmol/L])
otherwise well, with no central nervous system Blood glucose = 80 mg/dL (70-99 mg/dL)
(SI: 4.4 mmol/L [3.9-5.5 mmol/L])
concerns. He has started puberty (Tanner stage 2),
Urine osmolality = 193 mOsm/kg
and his growth chart is unremarkable other than (150-1150 mOsm/kg) (SI: 190 mmol/kg
some weight loss in the past few months. School [150-1150 mmol/kg])
performance has been fine, but he reports fatigue
out of the ordinary. His sodium concentration is
143 mEq/L (143 mmol/L), and his urine is dilute. Which of the following is true regarding
the diagnosis of central DI in this patient?
Which of the following statements is true? A. A low copeptin concentration would establish
A. MRI is indicated to look for a pituitary tumor the diagnosis of DI of any cause
B. The weight loss and fatigue suggest that tumor B. A water-deprivation study is indicated but
markers should be obtained should be done on an inpatient basis
C. Careful examination is indicated to look for C. Therapy with desmopressin can be initiated
a rash D. Serum and urine osmolality should be
D. His age rules out familial DI measured

Answer: C) Careful examination is Answer: C) Therapy with desmopressin can be initiated


indicated to look for a rash This child has a calculated serum osmolality
About 15% to 20% of children presenting with already above 304 mOsm/kg (twice the sodium),
central DI have Langerhans cell histiocytosis. plus the additional component from the
Langerhans cell histiocytosis can affect the bone glucose (5 mOsm/kg). This finding, coupled
and skin and lead to a rash on the arms and groin. with the urine, establishes the diagnosis of
Thus, careful examination for a rash (Answer C) DI. A low copeptin concentration in the face
is correct. MRI (Answer A) is incorrect because of hyperosmolality might imply central DI
MRI is typically performed after the diagnosis (Answer A); however, this remains to be proven
of central DI is established. It is not indicated in children. In addition, it would be a test only
for nephrogenic DI. The patient’s symptoms are for central DI, not nephrogenic DI. The main
typical of polyuria/polydipsia and by themselves point of this vignette is to show that sometimes
do not imply there is a malignancy. Therefore, a water-deprivation study (Answer B) is not
obtaining tumor markers (Answer B) is not needed, as the diagnosis is already established. The
indicated now. Tumor markers are indicated if the calculated osmolality is well above 305 mOsm/kg.
diagnosis of central DI is confirmed, even if those It would have been nice to measure the osmolality
symptoms are not present. Familial DI can present (Answer D), but the diagnosis of DI is already
even in older teenagers, so his age does not rule established. This patient could have nephrogenic
out familial DI (Answer D). DI, so a trial of desmopressin (Answer C) and MRI
(if there is a response to desmopressin) should be
the next steps.
Case 2
A 5-year-old girl with behavioral issues is found to
be drinking from unusual sources such as puddles.
She has new-onset enuresis.

238  ENDO 2021 • Endocrine Case Management

chased by Dr. Khalid H. Seedahmed, Jr., MRCP PGDip, CertEndocrinology SA - ID 283


Case 3  New Content  development is affected, examination may show
small testicular size for Tanner stage. In this case,
A 14-year-old boy presents with a 6-month
findings on pubertal examination were normal
history of excessive thirst and urination. He has
(testicular volume of 16 mL and Tanner stage 4)
marked fatigue. He has no headaches or vision
(thus, Answer B is incorrect). The thickened
concerns and is growing well. Several laboratory
stalk implies an increased risk for an infiltrating
studies over the past 6 months have documented
disorder, such as a germ-cell tumor, Langerhans
normal glucose and sodium levels. He has no
cell histiocytosis, or hypophysitis. MRI should
hypercalcemia.
be done every 3 months (not yearly) for the first
On physical examination, he is well into
couple years (thus, Answer D is incorrect). Germ-
puberty (Tanner stage 4), with a testicular size
cell tumors can be diagnosed by detecting tumor
of 16 mL bilaterally. A water-deprivation study
markers (hCG and α-fetoprotein); however,
confirms he has central DI. MRI of the pituitary
cerebrospinal fluid is more sensitive than serum
gland shows an absent bright spot of the posterior
samples. Thus, the correct answer is to obtain a
pituitary and a thickening of the infundibulum,
cerebrospinal fluid sample (Answer C).
but no tumors. No serum tumor markers (hCG or
α-fetoprotein) are identified.
Case 4  New Content 
Which of the following statements is true? A 4-year-old boy is found to be drinking from
A. The absent bright spot confirms an increased unusual sources such as puddles and bathroom
risk of central nervous system malignancy faucets, as well as from others’ water bottles. He
B. Findings on pubertal examination suggest has new-onset enuresis. On physical examination,
stalling of puberty and are a sign of he looks dehydrated.
panhypopituitarism
Laboratory test results:
C. Cerebrospinal fluid sample should be obtained
D. Because of a risk of central nervous system Serum sodium = 144 mEq/L (136-142 mEq/L)
(SI: 144 mmol/L [136-142 mmol/L])
malignancy, annual MRI studies are needed for Serum urea nitrogen = 14 mg/dL (8-23 mg/dL)
at least 5 years (SI: 5.0 mmol/L [2.9-8.2 mmol/L])
Blood glucose = 72 mg/dL (70-99 mg/dL)
Answer: C) Cerebrospinal fluid (SI: 4.0 mmol/L [3.9-5.5 mmol/L])
sample should be obtained Urine osmolality = 184 mOsm/kg
(150-1150 mOsm/kg) (SI: 184 mmol/kg
This boy has central DI that was confirmed [150-1150 mmol/kg])
by a water-deprivation study. Some children
with DI initially present with central nervous
system symptoms, including headache and vision Which of the following statements is true?
concerns, if there is mass effect. In this case, MRI A. A high copeptin concentration would establish
showed no mass, which correlates with the lack the diagnosis of central DI
of central nervous system symptoms. Fatigue B. A water-deprivation study is indicated and
can be caused by hypothyroidism or simply by should be done on an inpatient basis
poor sleep due to marked nocturia. The absent C. MRI should be performed prior to further
bright spot is seen in all causes of central DI, so diagnostic evaluation
it is not a specific finding for a tumor etiology D. Therapy with desmopressin should be initiated
(thus, Answer A is incorrect). This patient is at
risk for panhypopituitarism, and workup should Answer: B) A water-deprivation study is indicated
include an assessment of each gland. If pubertal and should be done on an inpatient basis

ENDO 2021 • Pediatric Endocrinology  239

chased by Dr. Khalid H. Seedahmed, Jr., MRCP PGDip, CertEndocrinology SA - ID 283


To put the laboratory data in context, it is study is typically done as an inpatient procedure
necessary to calculate the serum osmolality: given the risk of dehydration and hypovolemia,
as well as the need to monitor compliance with
Serum osmolality mOsm/kg ≈ 2 × Na mEq/L avoiding oral fluid intake. If the water-deprivation
+ glucose (mg/dL) / 18 + serum urea nitrogen study confirms central DI, then MRI (Answer C)
(mg/dL) / 2.8 is indicated. One exception is if a child has central
This child has a calculated serum osmolality of nervous system symptoms, in which case the
297 mOsm/kg [(2 × 144) + (72 / 18) + (14 / 2.8)]. central nervous system workup may take priority.
Ideally, serum osmolality should also be measured, A low copeptin concentration, not high
but in this case the calculated osmolality is below (Answer A), in the face of hyperosmolality might
300 mOsm/kg. This serum osmolality, despite imply central DI. However, this remains to be
the dilute urine, is not high enough to obviate proven in children. Recall that copeptin is the
the need for a water-deprivation study. Thus, C-terminal portion of pre-provasopressin and is
therapy with desmopressin (Answer D) should thus cosecreted with AVP. In addition, copeptin
not be initiated yet. In this vignette, a water- would only help identify central DI, but it would
deprivation study (Answer B) is needed, as the not be low in cases of nephrogenic DI where the
diagnosis has not been established based on the issue is with the end organ.
presented laboratory studies. A water-deprivation

References
1. Majzoub JA, Srivatsa A. Diabetes insipidus: clinical and basic aspects. Pediatr 7. Chen S, Leger J, Garel C, Hassan M, Czernichow P. Growth hormone
Endocrinol Rev. 2006;4(Suppl 1):60-65. PMID: 17261971 deficiency with ectopic neurohypophysis: anatomical variations and
2. Fenske W, Refardt J, Chifu I, et al. A copeptin-based approach in the relationship between the visibility of the pituitary stalk asserted by magnetic
diagnosis of diabetes insipidus. N Engl J Med. 2018;379(5):428-439. resonance imaging and anterior pituitary function. J Clin Endocrinol Metab.
PMID: 30067922 1999;84(7):2408-2413. PMID: 10404812
3. Di Iorgi N, Napoli F, Allegri AE, et al. Diabetes insipidus--diagnosis and 8. Alter CA, Bilaniuk LT. Utility of magnetic resonance imaging in the
management. Horm Res Paediatr. 2012;77(2):69-84. PMID: 22433947 evaluation of the child with central diabetes insipidus. J Pediatr Endocrinol
4. Maghnie M, Cosi G, Genovese E, et al. Central diabetes insipidus in children Metab. 2002;15(Suppl 2):681-687. PMID: 12092681
and young adults. N Engl J Med. 2000;343(14):998-1007. PMID: 11018166 9. Korkmaz HA, Demir K, Kilic FK, et al. Management of central diabetes
5. Werny D, Elfers C, Perez FA, Pihoker C, Roth CL. Pediatric central diabetes insipidus with oral desmopressin lyophilisate in infants. J Pediatr Endocrinol
insipidus: brain malformations are common and few patients have idiopathic Metab. 2014;27(9-10):923-927. PMID: 24854529
disease. J Clin Endocrinol Metab. 2015;100(8):3074-3080. PMID: 26030323 10. Rivkees SA, Dunbar N, Wilson TA. The management of central diabetes
6. Murray PG, Hague C, Fafoula O, et al. Associations with multiple pituitary insipidus in infancy: desmopressin, low renal solute load formula, thiazide
hormone deficiency in patients with an ectopic posterior pituitary gland. Clin diuretics. J Pediatr Endocrinol Metab. 2007;20(4):459-469. PMID: 17550208
Endocrinol (Oxf). 2008;69(4):597-602. PMID: 18331606

240  ENDO 2021 • Endocrine Case Management

chased by Dr. Khalid H. Seedahmed, Jr., MRCP PGDip, CertEndocrinology SA - ID 283


Screening of the Newborn
for Adrenoleukodystrophy
Molly O. Regelmann, MD. Division of Pediatric Endocrinology and Diabetes, Department
of Pediatrics, Children’s Hospital at Montefiore, Albert Einstein Medical College, Bronx, NY;
E-mail: moregelm@montefiore.org

Learning Objectives Significance of the


As a result of participating in this session, learners Clinical Problem
should be able to: ALD is a heterogeneous disease that frequently
• Describe the various presentations and causes primary adrenal insufficiency in affected
pathophysiology of adrenoleukodystrophy males.2,3 Unrecognized and untreated adrenal
(ALD). insufficiency is associated with morbidity and
mortality.1,4 Following the passage of Aidan’s Law,
• Explain the newborn screening process and New York was the first state to initiate newborn
apply adrenal insufficiency surveillance and screening for ALD on December 30, 2013.
treatment recommendations to boys identified ALD was added to the Recommended Uniform
with ALD. Screening Panel in 2016.1,5 As of October 2019,
newborn screening for ALD has been expanded to
13 states and the District of Columbia with plans
to be added to several more states’ screening panels
Main Conclusions in the next 2 years. Adrenal insufficiency develops
ALD is the most common peroxisomal disorder in most males with ALD, but predicting onset
and presents variably with adrenal insufficiency, of and diagnosing adrenal insufficiency remain
neurologic disease, and testicular dysfunction. clinical challenges given the limited knowledge of
Recently, ALD was added to state newborn the natural history of adrenal insufficiency in ALD
screening panels. Initial newborn screening data and the subtle and variable clinical presentations.1-3
suggest that ALD may be more prevalent than
previously described and adrenal insufficiency may
Barriers to Optimal Practice
develop in some affected boys during early infancy.
Early monitoring, diagnosis, and treatment of • Infants do not have established predictable
adrenal insufficiency can prevent life-threatening diurnal variation in ACTH and cortisol
adrenal crisis. The Pediatric Endocrine Society secretion.
developed an algorithm to guide clinicians
caring for boys with ALD identified by newborn • Normal reference ranges for ACTH and
screening. The publication suggests a monitoring cortisol during infancy are not well established.
schedule for adrenal insufficiency, addresses the • There is no known predictable time course
ambiguities in cortisol and ACTH test results, and for when adrenal insufficiency will develop in
guides therapeutic decisions for boys with ALD.1 males with ALD.

ENDO 2021 • Pediatric Endocrinology  241

chased by Dr. Khalid H. Seedahmed, Jr., MRCP PGDip, CertEndocrinology SA - ID 283


Strategies for Diagnosis, In the adrenal cortex, VLCFA preferentially
accumulate in the zona fasciculate and zona
Therapy, and/or Management
glomerulosa, leading to glucocorticoid and
Pathophysiology and Epidemiology androgen deficiencies, most commonly. However,
ALD has 6 clinical presentations (Table) and mineralocorticoid deficiency is described in some
is caused by pathogenic variants in the ABCD1 patients with ALD. VLCFA are thought to be
gene, located on chromosome Xq28. ABCD1 directly cytotoxic to adrenocortical cells. In vitro
encodes adrenoleukodystrophy protein, an ATP- studies also support VLCFA disrupting ACTH
binding cassette protein that normally forms receptor function and leading to adrenocortical
a transmembrane channel responsible for the atrophy.1
transport of very long-chain fatty acids (VLCFA) Before newborn screening, the estimated
into the peroxisome for degradation.2,3 Pathogenic incidence of ALD in the United States was 1 in
variants in ABCD1 (more than 800 described 21,000 males and 1 in 14,000 females. The sex
variants - https://adrenoleukodystrophy. discrepancy has been proposed possibly to be
info/mutations-and-variants-in-abcd1) lead to secondary to unrecognized adrenal insufficiency.1
elevations in VLCFA. Eighty percent of women Lower than expected diagnoses rates have
heterozygous for and all men hemizygous for also been reported in ethnic minorities.7 In a
pathogenic ABCD1 variants have elevations in prospective study of 49 asymptomatic male
VLCFA.2 In males, the excess VLCFA accumulate patients (mean age 4.5 ± 3.5 years) identified
in the adrenal cortex, nervous system, and to have ALD based on family history, 80% had
testicular Leydig cells. There is no correlation abnormal or borderline abnormal adrenal function
between the ABCD1 variant and the degree of tests at baseline and 86% did by the end of the
VLCFA elevation. Furthermore, there is no study period (mean follow-up 2 ± 1.7 years). None
genotype-phenotype correlation, and the degree of the 18 patients tested had mineralocorticoid
of VLCFA elevation is not predictive of onset of deficiency.8 In a retrospective review of 159 male
adrenal insufficiency or neurologic symptoms.2,6

Table. Clinical Presentations of Adrenoleukodystrophy2,3

Type of ALD (Frequency) Description Age of Onset

Childhood cerebral ALD Typically presents with symptoms similar to First decade (typically not before
(~31%-35% of affected males) attention-deficit hyperactivity disorder and then 2 years)
rapidly progresses to impaired cognition and
Adolescent cerebral ALD motor dysfunction, followed by complete disability Second decade
(~4%-7% of affected males) (without supportive measures, fatal within 2
to 4 years of symptom onset). Most patients
Adult cerebral ALD with cerebral ALD have adrenal insufficiency at After second decade
(~2%-5% of affected males) diagnosis.

Adrenomyeloneuropathy (AMN) Progressive stiffness/weakness in the legs Third to fourth decade


(~40%-46% of affected males) (spasticity), bowel and bladder incontinence,
sexual dysfunction, mood disturbance.
Approximately 70% have adrenal insufficiency at
diagnosis.

Adrenal insufficiency only Primary adrenal insufficiency symptoms First decade (peak reported between
(decreases with age) (hyperpigmentation, weight loss/growth failure, 3 and 10 years)
nausea/anorexia/abdominal pain, fatigue, lack of
pubic/axillary hair, and less often salt-wasting if
mineralocorticoid deficiency is present).

Asymptomatic No symptoms of ALD but at high risk for both Newborn


adrenal insufficiency and neurologic disease.

242  ENDO 2021 • Endocrine Case Management

chased by Dr. Khalid H. Seedahmed, Jr., MRCP PGDip, CertEndocrinology SA - ID 283


patients with ALD in 2 large neurology clinics, Surveillance for Adrenal
there was an estimated 80% lifetime prevalence Insufficiency
of adrenal insufficiency, with most developing
adrenal insufficiency during childhood and Initial recommendations for adrenal insufficiency
adolescence (median age of diagnosis, 14 years). surveillance did not address the unpredictable
There was an average delay in diagnosis of adrenal secretory pattern or lack of robust reference
insufficiency of 3.5 years (range, 0.25-21 years) data for ACTH and cortisol during the first
based on reported symptoms, and more than two- few years of life.5 The Pediatric Endocrine
thirds had endocrine symptoms at the time of ALD Society Drug and Therapeutics/Rare Diseases
diagnosis. A significant number of patients were Committee recognized the gap in knowledge
treated with mineralocorticoid, but the median and the need for clinical guidance when caring
time to therapy initiation was significantly older at for boys with ALD.1 Other than case reports
56 years.6 and the referenced studies above,6,8 there is little
published about the natural history of adrenal
insufficiency associated with ALD. The youngest
Newborn Screening patients reported to have biochemical evidence
ALD is considered an ideal condition for of adrenal dysfunction were 5 months old,8 7
screening, as there is a presymptomatic phase and months old,6 and, in a recent case series, 5 weeks
life-saving interventions are available to treat both and 4.5 months old.9 Given the limited experience
the primary adrenal insufficiency and cerebral with male infants known to have ALD and case
ALD. State programs have varying protocols, but reports found in the literature, as well as clinician
all measure VLCFA from filter paper specimens. experience, the recommendation was made to start
In some states, after confirmation of elevated screening shortly after ALD diagnosis. Follow-
VLCFA, ABCD1 gene sequencing is available. up surveillance for adrenal insufficiency was
The sensitivity and specificity of the filter paper recommended every 3 to 4 months for children
VLCFA measurements are reported to be excellent younger than 2 years and every 4 to 6 months for
for males hemizygous for pathogenic ABCD1 children older than age 2 years (more frequent
variants. As 20% of females heterozygous for screening was recommended in the younger
ABCD1 pathogenic variants do not have VLCFA age group due to the challenge of identifying
elevations, they are not expected to be identified symptoms of adrenal insufficiency in patients
by newborn screening. VLCFA are also elevated younger than 2 years). The algorithm developed
in other, rarer peroxisomal disorders, including for adrenal insufficiency surveillance in males
but not limited to Zellweger syndrome, acyl with ALD was not only designed to address the
CoA oxidase deficiency, and D-bifunctional unpredictable nature of adrenal insufficiency
protein deficiency, which do not have good associated with ALD, but also to consider
therapeutic interventions for neurologic disease the potential pitfalls of ACTH and cortisol
at this time. Once a patient is identified to have interpretation during the first few years of life
elevated VLCFA on newborn screening, most (Figure).1
state protocols refer infants and families to a
metabolic center for confirmatory testing and
genetic counseling. Because ALD is X-linked, it
is common to identify additional family members
at risk for ALD, some of whom may require
urgent evaluations for adrenal insufficiency or
cerebral ALD.5

ENDO 2021 • Pediatric Endocrinology  243

chased by Dr. Khalid H. Seedahmed, Jr., MRCP PGDip, CertEndocrinology SA - ID 283


Figure. Suggested Algorithm for Surveillance of Adrenal Function1

I Male Identified with ALD <2 years of age


!.
I Urgent Referral to Pediatric Endocrinology I
J.
ACTH and Cortisol
l !
ACTH <100 pg/mL ACTH 100-299 pg/mL ACTH ≥300 pg/mL

I I
l l l l l
Cortisol Cortisol Cortisol Cortisol Cortisol Cortisol
≥5 µg/dL <5 µg/dL 11 ≥10 µg/dL I I <10 µg/dL I I ≥18 µg/dL I <18 µg/dL

I
l
Consider high dose (15 µg/kg or
Repeat ACTH Consider Start daily and
125 µg) cosyntropin stimulation Perform high dose confirmatory ACTH
and cortisol every stress dose
testing versus repeat ACTH and (15 µg/kg or 125 µg) & cortisol prior to
3-4 months for glucocorticoid
cortisol as clinically indicated cosyntropin starting daily and
the first 2 years replacement
(ACTH & cortisol should be stimulation testing stress dose
of life therapy plans
repeated at least every 3-4 glucocorticoid
months for the first 2 years of life) replacement
If cosyntropin stimulation therapy plans

For children 2 years of age and older,


7 testing is performed

an 8 am ACTH and cortisol should be Peak cortisol <18 µg/dL  start stress
performed every 4-6 months glucocorticoid replacement and consider daily To convert cortisol to nmol/L
following the same algorithm for glucocorticoid replacement as clinically indicated multiply by 27.588
high-dose ACTH stimulation testing
(using 250 µg cosyntropin for those
Peak cortisol ≥18 µg/dL  repeat ACTH and To convert ACTH to pmol/L
≥2 years) and treatment. cortisol every 3-4 months for the first 2 years of life multiply by 0.2222

Management of Adrenal 6 to 8 hours in times of stress) and to administer


Insufficiency injectable hydrocortisone when unable to
take oral stress doses. All caregivers should be
Treatment of adrenal insufficiency associated provided training for administration of injectable
with ALD is the same as treatment for most hydrocortisone. Patients with adrenal insufficiency
other causes of primary adrenal insufficiency.10 should be instructed to wear medical identification
Hydrocortisone is the preferred glucocorticoid alerting emergency medical care providers to the
and, for ALD, the starting dosage is need for steroid therapy.4
8 to 12 mg/m2 daily divided into 3 doses.
Doses should be increased with stress, and a
standardized “Adrenal Insufficiency Action Neurologic Disease: Cerebral ALD
Plan” has recently been published to provide and Adrenomyeloneuropathy
guidance for safe outpatient and emergency An extensive review of the pathophysiology,
department management of adrenal insufficiency. surveillance, and treatment of neurologic disease
It is recommended that all patients receive is beyond the scope of this chapter. Surveillance
an individualized treatment plan, including MRI protocols for cerebral ALD vary, but
instructions to increase oral hydrocortisone (2 generally, MRI of the brain is initiated around
to 3 times physiologic doses administered every 12 months of age and performed annually until

244  ENDO 2021 • Endocrine Case Management

chased by Dr. Khalid H. Seedahmed, Jr., MRCP PGDip, CertEndocrinology SA - ID 283


age 3 years, when the risk for cerebral ALD On physical examination, his length and
increases. Surveillance MRI of the brain is then weight are at the 50th percentile and consistent
repeated every 6 months until age 10 years. with the pediatrician’s parameters from the
After 10 years of age, brain MRI is performed 3-month-old well-child visit. He seems to have
annually, as risk for cerebral ALD decreases. a skin tone that is between his mother’s and
Brain MRI is performed more frequently when father’s (both present for the visit), but it is
concerning lesions are noted. Cerebral ALD, when possible that the scrotum and areolae are slightly
detected in its earliest stages, can be treated with hyperpigmented. His tone is normal, and findings
hematopoietic stem-cell therapy. Gene therapy has on neurologic examination are normal.
also been reported to halt cerebral ALD, although
this therapy is still considered investigational. At Which of the following is the best next
present, only supportive therapies are available to step in this patient’s evaluation?
manage adrenomyeloneuropathy.2,3 A. Draw sample for ACTH measurement today
B. Draw sample for cortisol measurement today
Additional Resources for C. Draw sample for both ACTH and cortisol
Providers, Patients, and Families measurement today
D. Have the baby return for an 8-AM draw for
• ALD Info – https://adrenoleukodystrophy.info ACTH and cortisol measurement
• ALD Connect – https://aldconnect.org E. Perform a high-dose cosyntropin-stimulation
• ALD Alliance – https://aldalliance.org test today
• Aidan Jack Seeger Foundation – Answer: C) Draw sample for both ACTH
https://aidanhasaposse.org and cortisol measurement today
• Leukodystrophy Network (Hunter’s Hope) – This baby is at high risk for adrenal insufficiency
https://www.huntershope.org/family-care/ due to ALD. After taking a detailed history
leukodystrophies/adrenoleukodystrophy/ and performing the exam, the clinician should
counsel the parents about signs and symptoms
of adrenal insufficiency. Signs and symptoms
Clinical Case Vignettes can often be subtle, and there are reports of male
Case 1 infants with ALD as young as 5 weeks with test
results consistent with adrenal insufficiency.
A 3-month-old male infant is identified on
The parents should be given an explanation of
New York State newborn screening as having
options for evaluation. The baby is unlikely to
elevations in VLCFA and a de novo variant of
have predictable diurnal secretion of ACTH
uncertain clinical significance in the ABCD1 gene.
and cortisol, as he is not sleeping through the
The variant is predicted to be pathogenic based on
night and is younger than 6 months. Therefore,
models. His parents receive genetic counseling at an
ACTH and cortisol can be drawn in the clinic in
outside institution, and the baby’s initial screening
the early afternoon (thus, Answer C is correct
cortisol and ACTH are reported to be “normal.”
and Answer D is incorrect). You may be able
His mother is of Western European descent and his
to draw an ACTH alone (Answer A), but if
father is of African descent. He now presents for a
the level is borderline elevated, you will need a
pediatric endocrine consultation visit at 1 PM. He
cortisol measurement to interpret when the level
is exclusively breastfed and typically gets up at night
is appropriate. You may miss evolving primary
to feed every 4 hours. Stool and urine patterns are
adrenal insufficiency by measuring cortisol
reported as typical for age, without concerns for
without ACTH (Answer B), as ACTH will rise
constipation.

ENDO 2021 • Pediatric Endocrinology  245

chased by Dr. Khalid H. Seedahmed, Jr., MRCP PGDip, CertEndocrinology SA - ID 283


before the cortisol is frankly low. If travel to descent and appears to have more pigmentation
the clinic is a significant burden for a family, a on his scrotum and in the axillary creases than you
cosyntropin-stimulation test (Answer E) can be recall from the previous visit.
discussed. It is not unusual to obtain ambiguous
test results at baseline and need to recall for a Question 1: At what age would you have
cosyntropin-stimulation test. Parents should be started stress doses of hydrocortisone?
counseled about the ambiguity of test results and A. 6 weeks
be included in decisions regarding additional B. 4 months
testing and treatment.1
C. 7 months
D. 13 months
Case 2 E. 16 months
A 16-month-old boy presents to the pediatric
endocrinology clinic for routine adrenal Answer: A) 6 weeks
insufficiency surveillance. He was identified on On the basis of the algorithm (Figure), stress doses
Connecticut newborn screening to have elevated of hydrocortisone should have been started at 6
VLCFA and was subsequently diagnosed with weeks of age (Answer A), when the peak cortisol
ALD based on confirmatory testing with a concentration was 17.1 µg/dL (471.8 nmol/L). It is
peroxisomal screen (confirmed elevated VLCFA) notable that the cutoff of 18 µg/dL (496.6 nmol/L)
and ABCD1 gene sequencing, which identified a is based on older polyclonal assays and it is possible
known pathogenic variant. He has had routine that newer monoclonal assays may have lower
adrenal insufficiency surveillance since diagnosis normal cutoffs.1
was confirmed at 6 weeks of age. He had an 8-AM
blood draw a few days ago in anticipation of Question 2: At what age would you have
today’s visit. started daily doses of hydrocortisone?
Laboratory test results are shown in the table.
A. 6 weeks
On physical examination, his weight is at
the 25th percentile (previously was at the 35th B. 4 months
percentile) and length is at the 25th percentile C. 7 months
(previously at the 23rd percentile). He seems D. 13 months
developmentally appropriate and is climbing on E. 16 months
the chairs in the exam room. He is of Hispanic
Answer: D) 13 months

Age ACTH Cortisol Stimulated Cortisol

6 weeks 92 pg/mL (SI: 20.2 pmol/L) 1.2 µg/dL (SI: 33.1 nmol/L) …

6 weeks … 4.3 µg/dL (SI: 118.6 nmol/L) 17.1 µg/dL (SI: 471.8 nmol/L)

4 months 176 pg/mL (SI: 38.7 pmol/L) 8.0 µg/dL (SI: 220.7 nmol/L) 16.9 µg/dL (SI: 466.2 nmol/L)

7 months 212 pg/mL (SI: 46.6 pmol/L) 9.0 µg/dL (SI: 248.3 nmol/L) …

10 months 120 pg/mL (SI: 26.4 pmol/L) 13.0 µg/dL (SI: 358.6 nmol/L) …

13 months 314 pg/mL (SI: 69.1 pmol/L) 12.1 µg/dL (SI: 333.8 nmol/L) …

16 months 429 pg/mL (SI: 94.4 pmol/L) 10.2 µg/dL (SI: 281.4 nmol/L) …

246  ENDO 2021 • Endocrine Case Management

chased by Dr. Khalid H. Seedahmed, Jr., MRCP PGDip, CertEndocrinology SA - ID 283


Based on the algorithm, daily hydrocortisone there is no explanation for decreased appetite
doses should have been started at age 13 months and poor weight gain. When asked, parents
(Answer D). The ACTH cutoff of 300 pg/mL note he has seemed to keep his tan from the
or higher (≥66 pmol/L) was chosen, as it is summer even though it is now midwinter.
reported that the adrenal cortex is maximally • Maternal grandmother has type 2 diabetes
stimulated with an ACTH concentration of mellitus and peripheral neuropathy in her 60s.
300 pg/mL (66 pmol/L). However, it is important
to emphasize that the algorithm is intended to be • Maternal grandfather has heart disease in
a guide and is not meant to replace good clinical his 60s.
judgment. In an otherwise asymptomatic child • Maternal uncle had unexpectedly passed away
who is being monitored closely and is receiving in his 20s during what was thought to be a
stress doses of hydrocortisone, the decision routine surgical procedure on his knee.
to continue stress doses of hydrocortisone as • Two maternal aunts have 4 children in total,
clinically indicated and to monitor for another all of whom are healthy.
short interval before starting daily doses may be
• Paternal grandmother passed away in her 40s
a viable option at 13 months (as it appears was
from breast cancer.
done in this patient’s case). It is not unusual for
ACTH and cortisol levels to fluctuate, and if the • Paternal grandfather is healthy in his 60s.
ACTH concentration were in the 200s pg/mL • Paternal aunt is healthy in her 30s.
(44 pmol/L) with a cortisol concentration greater
than 10 µg/dL (>275.9 nmol/L), the algorithm Confirmatory tests, including ABCD1 gene analysis
suggests holding off on daily doses. Conversely, and plasma peroxisomal screen, are ordered.
if at 7 months, the baby were not gaining weight The ABCD1 gene analysis has been held up for
well and had evidence of hyperpigmentation, insurance prior authorization, and the plasma
even with the ACTH concentration being less peroxisomal screen is interpreted to be consistent
than 300 pg/mL (<66 pmol/L), it may have with the patient being heterozygous for ALD.
been reasonable to consider starting daily
hydrocortisone therapy. The decisions regarding Question 1: In this scenario, which
hydrocortisone therapy should be based on a family member(s) should be evaluated
combination of clinical signs and symptoms, as by the metabolic center for testing?
well as laboratory testing.1 A. Patient’s mother
B. Patient’s father
Case 3  New Content  C. Patient’s brother
A 2-week-old girl is evaluated in the metabolic D. All of the above
clinic for a positive ALD newborn screen. The E. None of the above
parents are unaware of a family history of ALD.
Answer: D) All of the above
The baby was born full term, has been feeding and
gaining weight well, and is otherwise healthy. The This case illustrates the cascade of patients who
genetic counselor took a comprehensive family may be identified by one positive newborn
history and noted the following: screen. In a report of the early experience with
ALD newborn screening from Minnesota, of
• Patient’s mother and father are in their 30s
11 infants identified without a known family
and healthy.
history who subsequently had familial testing,
• Patient’s 5-year-old brother has had trouble 17 male hemizygotes and 24 female heterozygotes
gaining weight in the past year. He has been for ABCD1 variants were identified.11 The standard
evaluated by a dietitian and gastroenterologist;

ENDO 2021 • Pediatric Endocrinology  247

chased by Dr. Khalid H. Seedahmed, Jr., MRCP PGDip, CertEndocrinology SA - ID 283


of care for this female infant is to obtain ABCD1 Question 2: Which family member(s) should
gene sequencing. It is not unusual to encounter be evaluated by endocrinology before
challenges with insurance prior authorization for receiving results of confirmatory testing?
genetic testing, particularly for a female infant, who A. Patient’s mother
is unlikely to have any significant symptoms during B. Patient’s father
childhood. In states such as New York, ABCD1
C. Patient’s brother
sequencing is included as a third tier to newborn
screening for ALD, but for many states, subsequent D. All of the above
genetic testing is required and can delay diagnosis E. None of the above
and identification of other family members who
Answer: C) Patient’s brother
may need more urgent identification, screening, and
therapeutic interventions. The patient’s brother should be evaluated by a
In this patient’s scenario, because ALD is pediatric endocrinologist (Answer C) as soon
X-linked, she potentially could have inherited as possible for primary adrenal insufficiency, as
an ABCD1 variant from either her mother or her he is described to be symptomatic and adrenal
father. The history provided is suspicious for insufficiency is a potentially life-threatening
the mother being heterozygous, based on the condition. If his VLCFA levels are elevated, he
following potentially symptomatic maternal family should be referred to pediatric neurology to screen
members: for cerebral ALD. The patient’s mother and father
are not described as having symptoms of adrenal
• Maternal grandmother with peripheral
insufficiency; they should wait for their metabolic/
neuropathy, which could be due to ALD or
molecular testing before being referred to
type 2 diabetes mellitus.
endocrinology. As adrenal insufficiency is reported
• Maternal uncle with an uncommon death to be rare in female heterozygotes, routine testing,
during a surgical procedure at a young age, without symptoms of adrenal insufficiency, is not
which potentially may have been related to recommended.1
unrecognized adrenal insufficiency.
• Brother with poor weight gain and Case 4  New Content 
tanned skin, which could indicate adrenal
insufficiency. You have been following a 4-year-old boy
identified by newborn screening to have elevated
For female probands, the standard is to test both VLCFA and a variant of uncertain significance in
parents. Hemizygous males have 100% penetrance the ABCD1 gene. His parents have had molecular
of elevations in VLCFA, but 15% to 20% of testing and neither has an ABCD1 variant. He has
heterozygous females with ABCD1 pathogenic had routine monitoring for adrenal insufficiency
variants have normal VLCFA levels. It is ideal and cerebral ALD following the most up-to-date
to have ABCD1 gene sequencing completed in guidelines. He has not required any therapeutic
the patient and then to test both parents.2,3 In interventions. A little more than 3 months ago,
this particular scenario, because there are delays his 8-AM ACTH concentration was 34 pg/mL
in molecular testing and the patient’s brother (7.5 pmol/L) and his cortisol concentration
is described as having symptoms of adrenal was 10.2 μg/dL (281.4 nmol/L). His last MRI,
insufficiency, it would be most practical to have performed 3 months ago, had a Loes score of 0
the metabolic center order VLCFA measurement (normal). His mother calls today because she has
in the patient’s brother. Thus, the patient’s noted behavioral changes. He is a few months into
mother, father, and brother (Answer D) should all the prekindergarten school year, and last week the
be evaluated by the metabolic center. teacher reported that he was not paying attention

248  ENDO 2021 • Endocrine Case Management

chased by Dr. Khalid H. Seedahmed, Jr., MRCP PGDip, CertEndocrinology SA - ID 283


during story time. At home, he has been more Although this patient’s mother is not describing
irritable with his older sibling. The patient has also typical symptoms of adrenal insufficiency,
started to nap after school, but then stays up past behavioral changes and changes in sleep patterns
his usual bedtime. can occur in children with adrenal insufficiency. It
has been almost 4 months since his last screening
Which of the following is the best next step? and it would be reasonable to repeat testing
A. Reassure his mother that this is typical 4-year- (Answer B) to ensure that adrenal insufficiency
old behavior and screening tests were recently is not contributing to the behavioral changes. In
normal; recommend routine screening in ALD, the onset of adrenal insufficiency can be
2 months as early as infancy, but it commonly presents in
B. Make arrangements now for a repeated 8-AM preschool and school-aged children. The peak age
measurement of ACTH and cortisol and of onset of cerebral ALD is between 4 and 8 years.
reassess for clinical adrenal insufficiency MRI changes typically predate clinical neurologic
symptoms by 6 to 12 months.2 The recent MRI
C. Tell his mother to go immediately to the local
with normal findings is somewhat reassuring.
emergency department for brain MRI to assess
The mother should be advised to speak with
for cerebral ALD
the pediatric neurologist and request a sooner
D. Tell his mother to make an appointment with reevaluation, but the pediatric neurologist should
a clinical psychologist determine the urgency of a repeated MRI (thus,
Answer B) Make arrangements now for a repeated Answer C is incorrect). Reassurance (Answer A)
8-AM measurement of ACTH and cortisol and and an appointment with a clinical psychologist
reassess for clinical adrenal insufficiency (Answer D) may ultimately be appropriate, but
reassessment for primary adrenal insufficiency and
cerebral ALD should be considered first.

References
1. Regelmann MO, Kamboj MK, Miller BS, et al; Pediatric Endocrine Society 7. Bonkowsky JL, Wilkes J, Bardsley T, Urbik VM, Stoddard G. Association
Drug and Therapeutics/Rare Diseases Committee. Adrenoleukodystrophy: of diagnosis of leukodystrophy with race and ethnicity among pediatric and
guidance for adrenal surveillance in males identified by newborn screen. J Clin adolescent patients. JAMA Netw Open. 2018;1(7):e185031. PMID: 30646379.
Endocrinol Metab. 2018;103(11):4324-4331. PMID: 30289543 8. Dubey P, Raymond GV, Moser AB, Kharkar S, Bezman L, Moser
2. Raymond GV, Moser AB, Fatemi A. X-linked adrenoleukodystrophy. HW. Adrenal insufficiency in asymptomatic adrenoleukodystrophy
GeneReviews. February 15, 2018. Available at: from https://www.ncbi.nlm. patients identified by very long-chain fatty acid screening. J Pediatr.
nih.gov/books/NBK1315/. Accessed for verification December 2019. 2005(4);146(4):528-532. PMID: 15812458
3. Engelen M, Kemp S, de Visser M, et al. X-linked adrenoleukodystrophy 9. Eng L, Regelmann MO. Early onset primary adrenal insufficiency in males
(X-ALD): clinical presentation and guidelines for diagnosis, follow-up and with adrenoleukodystrophy: case series and literature review. J Pediatr.
management. Orphanet J Rare Dis. 2012;7:51. PMID: 22889154 2019;211:211-214. PMID: 31101408
4. Miller BS, Spencer SP, Geffner ME, et al. Emergency management of adrenal 10. Bornstein SR, Allolio B, Arlt W, et al. Diagnosis and treatment of primary
insufficiency in children: advocating for treatment options in outpatient and adrenal insufficiency: an Endocrine Society clinical practice guideline. J Clin
field settings. J Investig Med. 2019 [Epub ahead of print]. PMID: 30819831 Endocrinol Metab. 2016;101(2):364-389. PMID: 26760044
5. Vogel BH, Bradley SE, Adams DJ, et al. Newborn screening for X-linked 11. Wiens K, Berry SA, Choi H, et al. A report on state-wide implementation of
adrenoleukodystrophy in New York State: diagnostic protocol, surveillance newborn screening for X-linked adrenoleukodystrophy. Am J Med Genet A.
protocol and treatment guidelines. Mol Genet Metab. 2015;114(4):599-603. 2019;179(7):1205-1213. PMID: 31074578
PMID: 25724074
6. Huffnagel IC, Laheji FK, Aziz-Bose R, et al. The natural history of
adrenal insufficiency in X-linked adrenoleukodystrophy: an international
collaboration. J Clin Endocrinol Metab. 2019;104(1):118-126. PMID: 30252065

ENDO 2021 • Pediatric Endocrinology  249

chased by Dr. Khalid H. Seedahmed, Jr., MRCP PGDip, CertEndocrinology SA - ID 283


Youth-Onset Type 2
Diabetes Mellitus
Philip Zeitler, MD, PhD. Section of Endocrinology, Department of Pediatrics, University of
Colorado School of Medicine, Aurora, CO; E-mail: philip.zeitler@childrenscolorado.org

Petter Bjornstad, MD. Section of Endocrinology, Department of Pediatrics, University of


Colorado School of Medicine, Aurora, CO; E-mail: petter.bjornstad@childrenscolorado.org

Learning Objectives Significance of the


As a result of participating in this session, learners Clinical Problem
should be able to: Youth-onset T2DM is an emerging disorder in
• Describe the initial approach to evaluation and children, adolescents, and young adults, and it has
management of new-onset diabetes mellitus in unique clinical challenges. The incidence of T2DM
the obese adolescent. in youth has increased dramatically over the last
20 years, although it remains a rare disorder. In
• Select and modify pharmacologic agents the United States, the best estimates are that the
for treatment of youth with type 2 diabetes incidence is as high as 5000 new cases per year,
(T2DM). with a total prevalence of less than 50,000.1,2 The
• Develop an approach to the management of pathophysiology of T2DM in youth resembles that
complications and cardiovascular risk in youth in adults: insulin resistance and nonautoimmune
with T2DM. β-cell injury. However, studies indicate that youth-
onset T2DM has a number of unique aspects.
For example, there is an important association of
T2DM with pubertal development—the median
Main Conclusions age of onset of T2DM in youth is approximately
Youth-onset T2DM is a complex metabolic 14 years. This observation is most likely related
disorder characterized by progressive β-cell injury to the transient reduction in insulin sensitivity
and insulin resistance. With the rapidly rising that occurs in children as they enter puberty3 and
number of obese adolescents with T2DM, it is the need for compensation in insulin secretion,
critical to check pancreatic autoantibodies in the which may lead to hyperglycemia in youth with
workup of youth-onset T2DM. Compared with limited β-cell capacity. Furthermore, diabetes
T1DM, youth-onset T2DM also carries higher onset during puberty may be reversible in some
risk and earlier onset of severe complications, such youth due to the dynamic nature of the underlying
as heart disease, diabetic kidney disease, and fatty insulin resistance. And, while rates in adult men
liver disease. Options for management are limited and women are similar, adolescent girls have a
and have been largely understudied in youth-onset 60% higher prevalence rate than that of adolescent
T2DM. Currently FDA-approved medications boys.2 In addition, while decline in β-cell function
include insulin, metformin, and liraglutide. also occurs in adults with T2DM, β-cell failure
appears to be more rapid in youth than in adults,
leading to loss of glycemic control on oral therapy
that is more rapid.4-11 Finally, there is evidence

250  ENDO 2021 • Endocrine Case Management

chased by Dr. Khalid H. Seedahmed, Jr., MRCP PGDip, CertEndocrinology SA - ID 283


of microvascular complications and risk markers DCCT-aligned assay, not point-of-care testing,
for macrovascular complications at the time of and it has not been specifically validated in
diagnosis, along with rapid progression of these youth. Second, in the absence of symptoms,
complications.12-15 hyperglycemia detected incidentally or under
T2DM has a disproportionate impact on youth conditions of acute physiologic stress may be
from ethnic/racial minorities and disadvantaged transitory and should not be regarded as diagnostic
backgrounds and occurs in complex psychosocial of diabetes. Accordingly, a second test on a
and cultural environments that make durable different day is required.
lifestyle change elusive and adherence to medical After the diagnosis of diabetes is established,
recommendations a struggle. Furthermore, consideration should be given to determining
these complexities hinder successful recruitment diabetes type. There are features of presentation
into and completion of research programs, and phenotype that may be useful in developing
leaving large gaps in knowledge regarding a presumption of T2DM, although there is
pathophysiology and treatment optimization.2,16-18 substantial overlap between characteristics of
T2DM and T1DM in the obese adolescent, making
these features of limited value. The degree of
Barriers to Optimal Practice pubertal development may be the most useful,
although in a negative fashion; youth with T2DM
• Complex social milieu.
are almost always in puberty, with a mean age of
• Knowledge gaps in understanding underlying diagnosis of 13 to 14 years and Tanner stage 4 to 5
pathophysiology. and are rarely prepubertal.21,22 Most importantly,
• Rapidly progressive course with frequent oral diabetes autoantibody testing (glutamic acid
treatment failure. decarboxylase antibodies [GAD], microinsulin
• High risk of vascular complications. autoantibodies [mIAA], zinc transporter 8
antibodies [ZnT8], tyrosine phosphatase-
• Limited clinical experience. based islet antigen 2 antibody [IA2]) should be
• Restricted access to diabetes medications. done in all youth with the clinical diagnosis of
• No evidence-based treatment guidelines. T2DM because of the high frequency of islet-cell
autoimmunity in patients with otherwise ‘‘typical’’
T2DM.21 The presence of antibodies predicts rapid
development of insulin requirement, as well as risk
Strategies for Diagnosis,
for development of other autoimmune disorders.
Therapy, and/or Management Diabetes autoantibody testing also should also be
Diagnosis considered in overweight/obese pubertal children
with a clinical picture of T1DM (weight loss,
The diagnosis of T2DM in youth requires 2 steps:
ketosis/ketoacidosis), some of whom may have
confirmation of the presence of diabetes mellitus
T2DM and be able to be weaned off of insulin for
followed by determination of diabetes type. The
extended periods of time.23-25 Finally, single-gene
criteria and classification of diabetes mellitus are
forms of diabetes, such as maturity-onset diabetes
provided in the American Diabetes Association
of the young should be considered in individuals
annual guidelines and the International Society
who have a presentation and course that is not
for Pediatric and Adolescent Diabetes Clinical
characteristic of either T1DM or T2DM.
Practice Consensus Guidelines.19,20 However,
there are several important points. First, while
the American Diabetes Association has added
hemoglobin A1c as a diagnostic criterion, this
assumes measurement in a laboratory with a

ENDO 2021 • Pediatric Endocrinology  251

chased by Dr. Khalid H. Seedahmed, Jr., MRCP PGDip, CertEndocrinology SA - ID 283


Management poorly adherent population, particularly as
adolescents quickly recognize that daily glucose
Initial treatment of the obese adolescent with
readings are not used for medication adjustment.
diabetes must take into account that diabetes type is
Lifestyle change is critical to treatment of
often not certain in the first few weeks of treatment,
T2DM, and clinicians should initiate a lifestyle
that 10% to 15% of obese adolescents have T1DM,
modification program, including nutrition and
and that a substantial percentage of adolescents
physical activity, for children and adolescents at
with T2DM present with clinically significant
the time of diagnosis.27 The interventions include
ketoacidosis.21,26 Therefore, initial treatment
promoting a healthy lifestyle through behavior
should be based on the clinical presentation, while
change, including nutrition, exercise training,
maintaining an open mind regarding both the
weight management, and smoking cessation.
diabetes type and eventual therapy.19
However, the challenges in implementing lifestyle
Obese adolescents presenting with acidosis
modifications in adolescents are greater than in
require initiation of intravenous insulin. However,
adult patients, since many adolescent patients with
once acidosis is resolved, subsequent therapy
T2DM come from families where overeating and a
depends on the provisional clinical diagnosis.
sedentary lifestyle are considered the norm. Thus,
When the clinical impression is T2DM, basal
many adolescents with T2DM will not maintain
insulin at a dosage of 0.2 to 0.4 units/kg once
the recommended lifestyle changes and will
daily is started and titrated based on fingerstick
remain overweight with poor diabetes control.28
glucose measurements. Insulin is administered at
The target of therapy is to attain and maintain
whatever time of day is the most likely to promote
hemoglobin A1c below 6.5% (<48 mmol/mol).
good adherence. At the same time, metformin
In most cases, this target can be successfully
is initiated at 500 mg once daily and titrated
achieved and maintained for extended periods
weekly to a maximally tolerated dosage, with a
with metformin monotherapy, combined with
target of 2000 mg daily. Insulin can generally be
focused lifestyle counseling and support. The
discontinued within a few weeks once antibody
approach to the adolescent with T2DM who does
negativity has been confirmed.
not attain and/or maintain target hemoglobin
Asymptomatic patients with an initial
A1c has not been studied systematically. In most
hemoglobin A1c level greater than 9% or
cases, pediatric endocrinologists will add basal
10% (>75 or 86 mmol/mol) may also require
insulin as a second agent and titrate to achieve
initiation of once-daily basal insulin therapy to
target hemoglobin A1c.19 This combination of basal
allow sufficient recovery of β-cell function for
insulin and metformin is often effective and may
successful monotherapy with an oral medication.
provide reliable glycemic control for extended
However, results from the TODAY study suggest
periods. Failure to achieve target at an insulin
that initiation of metformin and basic dietary
dose of 1 unit/kg or evidence for poor control of
intervention will result in a hemoglobin A1c
postprandial glycemia may prompt consideration
level in the nondiabetes range, with or without
of the addition of rapid-acting insulin, but
insulin.25 Obese, asymptomatic adolescents
the known challenges with adherence in this
presenting with a lesser degree of decompensation
population should be kept in mind. Premixed
can be started on metformin alone, with a high
insulins may offer some benefits over rapid-acting
likelihood of initial success. Patients not on insulin
insulin regarding ease of administration, but they
are asked to check their fingerstick glucose twice
require a degree of attention to mealtimes and
daily a few days each week and whenever they feel
snacks that is not always achievable.
ill. This frequency of blood glucose monitoring
Although major therapeutic advances have
represents a sustainable balance between providing
been made in diabetes care for adults with T2DM,
adequate safety to identify gradual deterioration,
the only FDA-approved medications as of June
while avoiding excessive burden on this generally

252  ENDO 2021 • Endocrine Case Management

chased by Dr. Khalid H. Seedahmed, Jr., MRCP PGDip, CertEndocrinology SA - ID 283


2019 for youth with T2DM were metformin and Even though this patient has had diabetes for
insulin, although liraglutide, a GLP-1 receptor 3 years without severe metabolic decompensation
agonist, has recently been approved for youth off treatment, there is no clinical characteristic
based on data from the ELLIPSE trial.29 Further with 100% sensitivity in excluding T1DM. Since
compounding the issue, pharmacotherapy may have T1DM remains more common than T2DM
different effects in youth-onset T2DM vs adult- at all ages and in most ethnicities, the a priori
onset T2DM as illustrated in the recently completed risk for T1DM is higher than T2DM, despite
Restoring Insulin Secretion (RISE) trial.10,11 “typical” characteristics. Furthermore, even among
those race/ethnicity groups (African American,
American Indian) in which T2DM is more
Clinical Case Vignettes common in adolescents, T1DM still occurs. A
Case 1 substantial percentage of adolescents with T1DM
A 16-year-old Latina girl was diagnosed with in the United States are obese, and obesity does
diabetes mellitus 3 years ago by her primary not protect from autoimmunity.30
care physician. At that time, diabetes type was Therefore, even in this setting, the most
determined based on clinical presentation and important step in the evaluation of an adolescent
family history of T2DM. Metformin, 2000 mg daily, with diabetes is rigorous determination of
was prescribed, but she reports that she took this for diabetes type. The presence of positive antibodies
only a few months. Since that time, she has taken no (Answer E), no matter what the phenotype, is
medications and reports no symptoms of diabetes, associated with more rapid progression to insulin
including polyuria, polydipsia, or weight loss. requirement, and patients with positive antibodies
One month ago, she was admitted to the hospital should be treated with insulin regardless of how
following a suicide attempt. During that admission, they are doing on oral therapy.
she was noted to have a glucose value of 550 mg/dL Measurement of fasting C-peptide may be
(30.5 mmol/L) and glucosuria, with small ketonuria helpful in determining degree of insulin resistance
and normal venous pH. Her hemoglobin A1c level and stimulated C-peptide (Answer B) may be a
was 12.5% (113 mmol/mol). She was treated by reasonable measure of insulin secretory capacity,
the inpatient team with subcutaneous basal and potentially contributing to the distinction of
bolus insulin and restarted on metformin. She T2DM from T1DM. However, this is only true
was referred to the diabetes center for further in the setting of stable metabolic status. During
evaluation. At that visit, her BMI was 28 kg/m2 acute decompensation, insulin and C-peptide
and blood pressure was 125/72 mm Hg. secretion are transiently decreased. Measurement
of C-peptide may be more useful in asymptomatic
Given the length of time since original patients, patients who have recovered from
diagnosis, which of the following decompensation, or in patients with presumed
investigations is the most important T2DM who have a persistent insulin requirement.
next step in this patient’s evaluation? Fasting lipid panel (Answer A), home glucose
A. Fasting lipid panel measurements (Answer C), and liver enzyme
measurement (Answer D) would be important
B. Fasting and stimulated C-peptide
to obtain at diagnosis in an obese individual with
measurements
diabetes, regardless of diabetes type, but would
C. Home glucose measurements be unlikely to make an immediate difference
D. Liver enzyme measurements in therapeutic decisions. However, the specific
E. Pancreatic autoantibody measurements screening done would depend on whether the
individual has positive antibodies (TSH, celiac
Answer: E) Pancreatic autoantibody measurements

ENDO 2021 • Pediatric Endocrinology  253

chased by Dr. Khalid H. Seedahmed, Jr., MRCP PGDip, CertEndocrinology SA - ID 283


antibodies, lipids) or negative antibodies (lipids, indicate that a hemoglobin A1c level greater
AST/ALT, urine albumin, creatinine clearance). than 6.3% (>45 mmol/mol) after a few months
of metformin monotherapy is associated with a
4- to 10-fold increased risk for loss of glycemic
Case 2
control, depending on sex and race/ethnicity, with
A 15-year-old Latino boy with long history of a median time to loss of control of approximately
being overweight was recently found to have 11 months. While it may be premature to start
a fasting glucose concentration of 289 mg/dL basal insulin at this initial visit, failure to achieve
(16.0 mmol/L) during a yearly exam for school. a hemoglobin A1c level in the nondiabetes range
He has no complaints initially, but on questioning, on metformin monotherapy suggests that the
he recalls some polyuria, polydipsia, and fatigue provider taking care of this patient should initiate
for the last few months. His mother has noted discussion with the family (the response to
increased thirst, but thought it was due to the hot metformin has not been sufficient to date and add-
summer. The patient has been healthy and takes on therapy may be necessary soon). Initiation of
no medications. basal insulin (Answer D) is the standard approach
On physical examination, his BMI is 32 kg/m2 to add-on therapy in the absence of evidence for
and blood pressure is 120/58 mm Hg. Pubertal oral or injected agents other than metformin
development is Tanner stage 5, and the rest of the and insulin.19,28 Starting multiple daily injections
exam findings are unremarkable. (Answer E) would help with glycemia, but
His hemoglobin A1c level is 8.7% adherence among youth with T2DM is generally
(72 mmol/mol) and antibodies (GAD, IA2, ZnT8, poor and it is preferable to use the simplest
mIAA) are negative. regimen possible.
You prescribe metformin and titrate to Measurement of fasting C-peptide (Answer A)
2000 mg daily. He returns in 3 months and reports will provide information on insulin sensitivity,
that he has been taking his metformin every and measurement of stimulated C-peptide
day and his mother confirms that she has been (Answer B) will provide information on β-cell
watching him. His hemoglobin A1c level is now reserve, but these factors are already incorporated
7.1% (54 mmol/mol). into the hemoglobin A1c measurement and will
not affect management. Liraglutide (Answer C),
Which of the following is the best next a GLP-1 receptor agonist, was FDA approved
step in this patient’s management? in June 201929 and is a reasonable alternative
A. Measure fasting C-peptide for this patient. However, the starting dosage of
B. Measure stimulated C-peptide liraglutide is 0.6 mg daily for 1 week, increasing
C. Start liraglutide, 1.8 mg daily to 1.2 mg injections thereafter. Starting liraglutide
D. Start basal insulin once daily at the maximum dosage of 1.8 mg daily is not
recommended due to a higher likelihood of
E. Start multiple daily injections with glargine adverse reactions (nausea, vomiting, diarrhea,
and glulisine anorexia, dyspepsia, and constipation). Additional
Answer: D) Start basal insulin once daily agents, including SGLT-2 inhibitors, DPP-4
inhibitors, and dual gastric inhibitor polypeptide
The patient is nearly at the American Diabetes and GLP-1 receptor agonist are likely also useful,
Association and International Society for but not yet FDA approved in pediatrics and can
Pediatric and Adolescent Diabetes targets for therefore not be formally recommended.
glycemia,17,18 and a common response would be
that no changes are needed. However, recently
published results from the TODAY study9

254  ENDO 2021 • Endocrine Case Management

chased by Dr. Khalid H. Seedahmed, Jr., MRCP PGDip, CertEndocrinology SA - ID 283


Case 3 After 6 months, if blood pressure is still above
the 95th percentile, an ACE inhibitor could be
A 15-year-old Latina girl was recently
started to achieve blood pressure values that are
diagnosed with T2DM during a routine physical
less than the 90th percentile. If the ACE inhibitor
examination. Her primary care physician
is not tolerated because of adverse effects (mainly
prescribed metformin and referred her to the
cough), an angiotensin-receptor blocker may be
diabetes center. She has been healthy and takes no
used. Combination therapy may be required if
medications other than metformin. Her review of
hypertension does not normalize on single-agent
systems is negative. On physical examination, her
therapy. Workup of hypertension not responsive
BMI is 35 kg/m2. Her hemoglobin A1c level is 7.8%
to initial medication should also include renal
(62 mmol/mol).
ultrasonography and echocardiography.
Which of the following should be Urinary albumin excretion should be assessed
prescribed as the most important next at diagnosis and annually. Microalbuminuria
step in this patient’s management? is defined as an albumin-to-creatinine ratio
of 30 to 299 mg/g in a spot urine sample.
A. Atorvastatin, 10 mg daily, for an LDL-
Because an elevated value can be secondary to
cholesterol level of 160 mg/dL (4.14 mmol/L)
exercise, cigarette smoking, menstruation, and
B. Fenofibrate, 67 mg daily, for a triglyceride orthostasis, the diagnosis of persistent abnormal
level of 745 mg/dL (8.42 mmol/L) microalbuminuria requires documentation of
C. Lisinopril, 10 mg daily, for a blood pressure of 2 of 3 consecutive abnormal values obtained on
145/82 mm Hg different days, preferably on rising, as benign
D. Lisinopril, 20 mg daily, for a urinary albumin- orthostatic proteinuria is common in adolescents.
to-creatinine ratio of 48 mg/g Thus, starting lisinopril now (Answer D) is
E. Vitamin E, 600 IU daily, for an ALT level of incorrect. ACE inhibitors are the agents of choice
87 U/L (1.45 µkat/L) due to proven renal protection, even if blood
pressure is normal. Measurement of albumin
Answer: B) Fenofibrate, 67 mg daily, for a excretion should be repeated at 3- to 6-month
triglyceride level of 745 mg/dL (8.42 mmol/L) intervals, and therapy should be titrated to achieve
a normal albumin-to-creatinine ratio. Non-
T2DM generally occurs in the setting of other
diabetes–related causes of renal disease should be
insulin-resistant abnormalities, including lipid
excluded, and consultation should be obtained if
abnormalities, endothelial and cardiac dysfunction,
the albumin-to-creatinine ratio is greater than
increased procoagulant and inflammatory markers,
300 mg/g.
increased hepatic and muscle lipid deposition,
Testing for dyslipidemia should be performed
mitochondrial dysfunction, increased plasma
soon after diagnosis when blood glucose control
uric acid, ovarian hyperandrogenism, and sleep
has been achieved and annually thereafter.
disorders, all of which increase cardiovascular
Goals for lipids are an LDL-cholesterol
risk. Given the prevalence of comorbidities at
concentration less than 100 mg/dL
the time of diagnosis,31,32 evaluation should occur
(<2.59 mmol/L), triglyceride concentration
either at the time of initial diagnosis or upon
less than 150 mg/dL (<1.70 mmol/L), and
reestablishment of metabolic stability.19,27
HDL-cholesterol concentration greater than
Blood pressure should be measured at every
40 mg/dL (>1.04 mmol/L). If LDL cholesterol
clinic visit and normalized for sex, height, and age.
is above goal, blood glucose control should be
Initial treatment of blood pressure above the 95th
maximized and dietary counseling provided
percentile consists of weight loss, limitation of
(dietary cholesterol <200 mg daily, saturated
dietary salt, and increased physical activity. Thus,
fat <7% of total calories, and <30% calories from
prescribing lisinopril now (Answer C) is incorrect.

ENDO 2021 • Pediatric Endocrinology  255

chased by Dr. Khalid H. Seedahmed, Jr., MRCP PGDip, CertEndocrinology SA - ID 283


fat). If LDL cholesterol remains greater than Which of the following most accurately
130 mg/dL (>1.04 mmol/L) after 6 months, describes the onset and progression of
statin therapy should be started with a target diabetic kidney disease in youth with T2DM
of less than 100 mg/dL (<2.59 mmol/L) (thus, compared with those who have T1DM?
Answer A is incorrect). The use of statins in A. Diabetic kidney disease is more prevalent at
sexually active adolescent females must be onset in T2DM and progresses more slowly
carefully considered and the risks explicitly B. Diabetic kidney disease is more prevalent at
discussed. Elevated triglycerides are not onset in T2DM and progresses more rapidly
treated for cardiovascular disease prevention.
C. Diabetic kidney disease is more prevalent at
However, if fasting triglycerides are greater than
onset in T2DM, but the risk of chronic kidney
500 mg/dL (>5.65 mmol/L), a fibric acid is started
disease is similar to that in T1DM
(Answer B) due to significantly increased risk for
acute pancreatitis, with a treatment goal of less D. Diabetic kidney disease is rare at diagnosis in
than 150 mg/dL (<1.70 mmol/L). T1DM and T2DM and progression to chronic
Hepatic steatosis is present in 25% to 50% kidney disease is similar
of adolescents with T2DM, and more advanced E. None of the above
forms of fatty liver disease are increasingly
Answer: B) Diabetic kidney disease is more prevalent
common and associated with progression to
at onset in T2DM and progresses more rapidly
cirrhosis, portal hypertension, and liver failure.
Fatty liver is now the most frequent cause of Diabetic kidney disease (ie, elevated albumin
chronic liver disorders among obese youth and excretion (≥30 mg/g) is often present at diagnosis
is the most common reason for liver transplant in patients with youth-onset T2DM, whereas it
in adults in the United States. T2DM therapies typically does not manifest until after 5 years of
that improve insulin resistance appear to improve disease in young persons with T1DM. For young
fatty liver. Vitamin E therapy (Answer E) may persons with T1DM, screening should be initiated
improve fatty liver, but it has also been linked with either at puberty or at 10 years of age, whichever
elevated all-cause mortality. Vitamin E is currently is earlier, when a patient has had T1DM for 5
not recommended to treat hepatic steatosis in or more years (American Diabetes Association
adolescents. Due to the potential for progression criteria)20 or at age 11 years or older or when the
to steatohepatitis, fibrosis, and cirrhosis, ongoing patient has had T1DM for more than 2 to 5 years
monitoring of liver enzymes is recommended (International Society for Pediatric and Adolescent
in youth with T2DM, with referral for biopsy if Diabetes criteria).33 Screening should then be
enzymes remain markedly elevated. repeated annually. In contrast, young persons
In addition, the clinician should explore the with T2DM should be screened at diagnosis and
presence of polycystic ovary syndrome, depression, annually thereafter.20,33 The SEARCH for Diabetes
eating disorders, and sleep disturbance and address in Youth study found that youth with T2DM
these as appropriate. had 2.42-fold (95% CI, 1.68-3.49) greater odds
of diabetic kidney disease than their peers with
Case 4  New Content  T1DM (P <.0001).34 Youth-onset T2DM is also
associated with a 4-fold higher risk of progression
You are describing the risk of diabetic kidney
to chronic kidney disease compared with that in
disease in youth-onset T1DM and T2DM to a
T1DM (hazard ratio, 4.03; 95% CI, 1.64-9.95).35
group of pediatric residents.
Therefore, youth with T2DM represent a
population at particularly high risk for kidney
failure and premature morbidity and mortality.

256  ENDO 2021 • Endocrine Case Management

chased by Dr. Khalid H. Seedahmed, Jr., MRCP PGDip, CertEndocrinology SA - ID 283


Case 5  New Content  additional long-term nephroprotection. GLP-1
receptor agonists (Answer B) are FDA-approved
You are caring for an 18-year-old man with T2DM
for use in youth aged 10 years and older who have
and a history of albuminuria who was previously
youth-onset T2DM36 and have demonstrated
started on lisinopril. At his 2 most recent visits
antialbuminuric effects in cardiovascular outcome
6 months apart, his urine albumin-to-creatinine
trials in adults with T2DM. However, these
ratio was 312 mg/g and 418 mg/g, respectively.
medications have not yet been shown to prevent
Laboratory test results: the development of more definitive long-term
kidney outcomes, including a doubling of
Serum creatinine = 0.68 mg/dL (0.7-1.3 mg/dL)
(SI: 60.1 µmol/L [61.9-114.9 µmol/L])
creatinine or progression to end-stage kidney
Hemoglobin A1c = 7.6% (4.0%-5.6%) (60 mmol/mol disease.37,38 In contrast to GLP-1 receptor agonists,
[20-38 mmol/mol]) SGLT-2 inhibitors (Answer E) have demonstrated
beneficial effects in preventing the development
He is currently treated with metformin, 2000 mg of macroalbuminuria and progression to end-stage
daily, and lisinopril, 20 mg daily. His blood kidney disease.39,40 In the landmark Canagliflozin
pressure is 125/78 mm Hg, and BMI is 36 kg/m2. and Renal Events in Diabetes with Established
Nephropathy Clinical Evaluation (CREDENCE)
Which of the following treatment trial, the risk of progression to end-stage kidney
options is the best next step in
disease was 32% lower in those treated with
this patient’s management?
canagliflozin vs placebo.39 Although SGLT-2
A. Angiotensin-receptor blocker inhibitors are not approved for use in pediatrics,
B. GLP-1 receptor agonist this patient is 18 years old. Moreover, SGLT-2
C. Long-acting insulin inhibitors have been shown to modestly lower
D. Mineralocorticoid receptor antagonist both weight and hemoglobin A1c, which would be
added benefits for this patient. Mineralocorticoid
E. SGLT-2 inhibitor
receptor antagonists (Answer D) have also been
Answer: E) SGLT-2 inhibitor shown to mitigate progression of diabetic kidney
disease, but less than that reported with SGLT-2
Initiation of long-acting insulin (Answer C) is inhibitors. The Finerenone in Reducing Kidney
not recommended for a hemoglobin A1c level Failure and Disease Progression in Diabetic
of 7.6% (60 mmol/mol) and will not reduce the Kidney Disease study (FIDELIO-DKD) found
risk of kidney damage. This patient is already on that the mineralocorticoid receptor antagonist
an ACE inhibitor (lisinopril) and has a normal finerenone lowered the risk of chronic kidney
blood pressure; therefore, adding an angiotensin- disease progression in adult patients with T2DM.41
receptor blocker (Answer A) will not offer

References
1. Lawrence JM, Imperatore G, Dabelea D, et al; SEARCH for Diabetes in Youth 4. TODAY Study Group, Zeitler P, Hirst K, et al. A clinical trial to
Study Group. Trends in incidence of type 1 diabetes among non-Hispanic maintain glycemic control in youth with type 2 diabetes. N Engl J Med.
white youth in the U.S., 2002-2009. Diabetes. 2014;63(11):3938-3945. 2012;366(24):2247-2456. PMID: 22540912
PMID: 24898146 5. Rascati K, Richards K, Lopez D, Cheng LI, Wilson J. Progression to insulin
2. Dabelea D, Mayer-Davis EJ, Saydah S, et al; SEARCH for Diabetes in for patients with diabetes mellitus on dual oral antidiabetic therapy using the
Youth Study. Prevalence of type 1 and type 2 diabetes among children US Department of Defense Database. Diabetes Obes Metab. 2013;15(10):901-
and adolescents from 2001 to 2009. JAMA. 2014;311(17):1778-1786. 905. PMID: 23531154
PMID: 24794371 6. Kahn SE. Clinical review 135: the importance of beta-cell failure in the
3. Hannon TS, Janosky J, Arslanian SA. Longitudinal study of physiologic development and progression of type 2 diabetes. J Clin Endocrinol Metab.
insulin resistance and metabolic changes of puberty. Pediatr Res. 2001;86(9):4047-4058. PMID: 11549624
2006;60(6):759-763. PMID: 17065576

ENDO 2021 • Pediatric Endocrinology  257

chased by Dr. Khalid H. Seedahmed, Jr., MRCP PGDip, CertEndocrinology SA - ID 283


7. Kahn SE, Lachin JM, Zinman B, et al; ADOPT Study Group. Effects of 24. Laffel L, Chang N, Grey M, et al; TODAY Study Group. Metformin
rosiglitazone, glyburide, and metformin on β-cell function and insulin monotherapy in youth with recent onset type 2 diabetes: experience from
sensitivity in ADOPT. Diabetes. 2011;60(5):1552-1560. PMID: 21415383 the prerandomization run-in phase of the TODAY study. Pediatric Diabetes.
8. TODAY Study Group. Effects of metformin, metformin plus rosiglitazone, 2012;13(5):369-375. PMID: 22369102
and metformin plus lifestyle on insulin sensitivity and β-cell function in 25. Kelsey MM, Geffner ME, Guandalini C, et al; Treatment Options for
TODAY. Diabetes Care. 2013;36(6):1749-1757. PMID: 23704674 Type 2 Diabetes in Adolescents and Youth Study Group. Presentation and
9. Zeitler P, Hirst K, Copeland KC, et al; TODAY Study Group. HbA1c effectiveness of early treatment of type 2 diabetes in youth: lessons from the
after a short period of monotherapy with metformin identifies durable TODAY study. Pediatr Diabetes. 2016;17(3):212-221. PMID: 25690268
glycemic control among adolescents with type 2 diabetes. Diabetes Care. 26. Pinhas-Hamiel O, Dolan LM, Zeitler P. Diabetic ketoacidosis among
2015;38(12):2285-2292. PMID: 26537182 obese African-American adolescents with NIDDM. Diabetes Care.
10. RISE Consortium; RISE Consortium Investigators. Effects of treatment 1997;20(4):484-486. PMID: 9096965
of impaired glucose tolerance or recently diagnosed type 2 diabetes 27. Copeland KC, Silverstein J, Moore KR, et al; American Academy of
with metformin alone or in combination with insulin glargine on beta- Pediatrics. Management of newly diagnosed type 2 diabetes mellitus (T2DM)
cell function: comparison of responses in youth and adults. Diabetes. in children and adolescents [published correction appears in Pediatrics.
201968(8):1670-1680. PMID: 31178433 2013;131(5):1014]. Pediatrics. 2013;131(2):e648-e664. PMID: 23359574
11. RISE Consortium. Impact of insulin and metformin versus metformin 28. Nadeau KJ, Anderson BJ, Berg EG, et al. Youth-Onset Type 2 Diabetes
alone on beta-cell function in youth with impaired glucose tolerance or Consensus Report: Current Status, Challenges, and Priorities. Diabetes Care.
recently diagnosed type 2 diabetes. Diabetes Care. 2018;41(8):1717-1725. 2016;39(9):1635-1642. PMID: 27486237
PMID: 29941500 29. Tamborlane WV, Barrientos-Perez M, et al; Ellipse Trial Investigators.
12. Copeland KC, Zeitler P, Geffner M, et al; TODAY Study Group. Liraglutide in children and adolescents with type 2 diabetes. N Engl J Med.
Characteristics of adolescents and youth with recent-onset type 2 diabetes: 2019;381(7):637-646. PMID: 31034184
the TODAY cohort at baseline. J Clin Endocrinol Metab. 2011;96(1):159-167. 30. Foster NC, Beck RW, Miller KM, et al. State of type 1 diabetes management
PMID: 20962021 and outcomes from the T1D exchange in 2016-2018. Diabetes Technol Ther.
13. Sellers EA, Yung G, Dean HJ. Dyslipidemia and other cardiovascular risk 2019;21(2):66-72. PMID: 30657336
factors in a Canadian First Nation pediatric population with type 2 diabetes 31. West NA, Hamman RF, Mayer-Davis EJ, et al. Cardiovascular risk factors
mellitus. Pediatr Diabetes. 2007;8(6):384-390. PMID: 18036065 among youth with and without type 2 diabetes: differences and possible
14. Hannon TS, Arslanian SA. The changing face of diabetes in youth: lessons mechanisms. Diabetes Care. 2009;32(1):175-180. PMID: 18945923
learned from studies of type 2 diabetes. Ann N Y Acad Sci. 2015;1353:113-137. 32. Pinhas-Hamiel O, Zeitler P. Acute and chronic complications
PMID: 26448515 of type 2 diabetes mellitus in children and adolescents. Lancet.
15. Dart AB, Martens PJ, Rigatto C, Brownell MD, Dean JH, Sellers EA. 2007;369(9575):1823-1831. PMID: 17531891
Earlier onset of complications in youth with type 2 diabetes. Diabetes Care. 33. Donaghue KC, Marcovecchio ML, Wadwa RP, et al. ISPAD clinical practice
2014;37(2):436-443. PMID: 24130346 consensus guidelines 2018: microvascular and macrovascular complications
16. Zeitler P, Chou HS, Copeland KC, Geffner M. Clinical trials in youth-onset in children and adolescents. Pediatr Diabetes. 2018;19(Suppl 27):262-274.
type 2 diabetes: needs, barriers, and options. Curr Diab Rep. 2015;15(5):28. PMID: 30079595
PMID: 25777998 34. Maahs DM, Snively BM, Bell RA, et al. Higher prevalence of elevated
17. Fazeli Farsani S, van der Aa MP, van der Vorst mmJ, Knibbe CA, de Boer A. albumin excretion in youth with type 2 than type 1 diabetes: the SEARCH
Global trends in the incidence and prevalence of type 2 diabetes in children for Diabetes in Youth study. Diabetes Care. 2007;30(10):2593-2598.
and adolescents: a systematic review and evaluation of methodological PMID: 17630264
approaches. Diabetologia. 2013;56(7):1471-1488. PMID: 23677041 35. Dart AB, Sellers EA, Martens PJ, Rigatto C, Brownell MD, Dean HJ. High
18. Petitti DB, Klingensmith GJ, Bell RA, et al; SEARCH for Diabetes in Youth burden of kidney disease in youth-onset type 2 diabetes. Diabetes Care.
Study Group. Glycemic control in youth with diabetes: the SEARCH for 2012;35(6):1265-1271. PMID: 22432116
Diabetes in Youth Study. J Pediatr. 2009;155(5):668-672. PMID: 19643434 36. Tamborlane WV, Barrientos-Perez M, Fainberg U, et al; Ellipse Trial.
19. Zeitler P, Arslanian S, Fu J, et al. ISPAD clinical practice consensus guidelines Liraglutide in children and adolescents with type 2 diabetes. N Engl J Med.
2018: type 2 diabetes mellitus in youth. Pediatr Diabetes. 2018;19(Suppl 27):28- 2019;381(7):637-646. PMID: 31034184
46. PMID: 29999228 37. Mann JFE, Orsted DD, Buse JB. Liraglutide and renal outcomes in type 2
20. American Diabetes Association. 13. Children and adolescents: standards of diabetes. N Engl J Med. 2017;377(22):2197-2198. PMID: 29171823
medical care in diabetes-2019. Diabetes Care. 2019;42(Suppl 1):S148-S164. 38. Marso SP, Bain SC, Consoli A, et al. Semaglutide and cardiovascular
PMID: 30559239 outcomes in patients with type 2 diabetes. N Engl J Med. 2016;375(19):1834-
21. Fagot-Campagna A, Pettitt DJ, Engelgau mm, et al. Type 2 diabetes among 1844. PMID: 27633186
North American children and adolescents: an epidemiological review and 39. Perkovic V, Jardine MK, Neal B, et al; CREDENCE Trial Investigators.
public health perspective. J Pediatr. 2000;136(5):664-672. PMID: 10802501 Canagliflozin and renal outcomes in type 2 diabetes and nephropathy. N Engl J
22. Copeland KC, Zeitler P, Geffner M, et al; TODAY Study Group. Med. 2019;380(24):2295-2306. PMID: 30990260
Characteristics of adolescents and youth with recent-onset type 2 diabetes: 40. Wanner C, Inzucchi SE, Zinman, B. Empagliflozin and progression of
the TODAY cohort at baseline. J Clin Endocrinol Metab. 2011;96(1):159-167. kidney disease in type 2 diabetes. N Engl J Med. 2016;375(18):1801-1802.
PMID: 20962021 PMID: 27806236
23. Klingensmith GJ, Pyle L, Arslanian S, et al; TODAY Study Group. The 41. Bakris GL, Agarwal R, Anker SD, et al; FIDELIO-DKD Investigators. Effect
presence of GAD and IA-2 antibodies in youth with a type 2 diabetes of finerenone on chronic kidney disease outcomes in type 2 diabetes. N Engl J
phenotype: results from the TODAY study. Diabetes Care. 2010;33(9):1970- Med. 2020;383(23):2219-2229. PMID: 33264825
1975. PMID: 20519658

258  ENDO 2021 • Endocrine Case Management

chased by Dr. Khalid H. Seedahmed, Jr., MRCP PGDip, CertEndocrinology SA - ID 283


 New Content

What Pediatric
Endocrinologists Need to
Know About Dyslipidemia
Don P. Wilson, MD, FNLA. Department of Pediatric Endocrinology and Diabetes, Endowed
Chair, Pediatric Cardiovascular Health and Risk Prevention Program, Cook Children’s
Medical Center, Fort Worth, TX; E-mail: don.wilson@cookchildrens.org

Learning Objectives Clinical trial data have shown that


pharmacologic treatment to lower atherogenic
As a result of participating in this session, learners
cholesterol is safe and effective and improves
should be able to:
noninvasive measures of atherosclerosis. All
• Discuss the role of atherosclerosis in statins have been approved by the US FDA for use
cardiovascular disease (CVD), starting in youth. in children with familial hypercholesterolemia and
should be recommended to patients aged 10 years
• Describe current recommendations for CVD
and older when the LDL-cholesterol concentration
risk factor screening in clinical practice.
is greater than 190 mg/dL (>4.92 mmol/L) or
• List the appropriate interventions, including greater than 160 mg/dL (>4.14 mmol/L) in
therapeutic lifestyle changes and medical patients with 1 or more additional risk factor.
therapy. Statins should be withheld or discontinued in
• Explain why early identification and women who are pregnant or planning to become
treatment of at-risk children can help prevent pregnant and should not be resumed until
atherosclerosis and atherosclerosis-related breastfeeding is completed.
events during adulthood. While elevations in triglycerides are most
often lifestyle related, polygenic and small-effect
variants, particularly in those with secondary
causes, may contribute to alternations in lipid and
Main Conclusions lipoprotein metabolism. In patients with extreme
triglyceride elevation, the goal of treatment is
Universal screening and early treatment have
prevention of pancreatitis. For patients with less
the potential to significantly reduce future
severe elevation, the focus is CVD risk reduction.
CVD-related events in youth with familial
hypercholesterolemia. Pediatric health care
providers must be aware of current cholesterol Significance of the
screening recommendations and be encouraged to Clinical Problem
improve screening rates. Cascade screening should
CVD is the leading cause of morbidity and
be offered to all first-degree relatives of probands
mortality in the United States. Although
and, when appropriate, to second-degree relatives.
manifestations are rare during childhood, the

ENDO 2021 • Pediatric Endocrinology  259

chased by Dr. Khalid H. Seedahmed, Jr., MRCP PGDip, CertEndocrinology SA - ID 283


origin of CVD (ie, atherosclerosis) emerges early Universal testing has the advantage of
in life.1 While many children with increased risk identifying familial hypercholesterolemia
of premature CVD have a genetic cause, such as in asymptomatic youth and young adults,
familial hypercholesterolemia, unhealthy lifestyle creating opportunities for early intervention,
behaviors have a significant role as well. CVD risk avoidance of events, and reverse cholesterol
factors in youth, as in adults, are exacerbated by screening. Cascade screening, although
dyslipidemia, insulin resistance, prehypertension/ more cost effective than any other screening
hypertension, and unhealthy behaviors such as low strategy currently available, is dependent
physical activity and poor diet. on identification of an index case, including
Universal cholesterol screening, starting at age family members who often have had a fatal or
10 years and continued every 5 years thereafter, nonfatal myocardial infarction. Genetic testing
may help simplify the process for busy clinicians.2 is encouraged and may become more feasible
Early recognition of individuals with familial in the future.
hypercholesterolemia, coupled with therapy from • Controversy regarding the appropriate age
a young age, will impede, if not arrest, the onset for intervention. A reluctance to treat may
of atherosclerosis. Identification of a child with also hamper screening efforts, advocating
familial hypercholesterolemia and screening of promotion of a heart-healthy lifestyle for all
first- and second-degree relatives, combines the asymptomatic youth and young adults instead.
benefits of universal and cascade screening, and A heart-healthy lifestyle alone is unlikely to
has the potential, within 1 generation, to detect all prevent atherosclerosis in individuals with
cases of familial hypercholesterolemia. familial hypercholesterolemia.

Barriers to Optimal Practice


Despite the benefits of early identification of those Strategies for Diagnosis,
at moderate-to-high CVD risk, several barriers Therapy, and/or Management
impede the success of cholesterol screening, Universal cholesterol screening and increased
especially in children. public and professional awareness have helped to
identify youth with hypercholesterolemia. While
• Lack of understanding of the goals of
the main goal is to identify those with familial
cholesterol screening. Because of the very
hypercholesterolemia, screening also reveals
high mortality rates with coronary heart
a variety of other genetic and lifestyle-related
disease, identifying individuals with familial
abnormalities in plasma lipid and lipoproteins.
hypercholesterolemia should be the primary
Evidence from clinical trials has shown that
goal of cholesterol screening.
pharmacologic treatment starting in childhood is
• Lack of clarity regarding the best age to initiate safe and effective and improves surrogate measures
screening. Age 1 to 9 years may be the most of atherosclerosis. Twenty-year follow-up data
opportune time for screening. With a false- of children with familial hypercholesterolemia
positive rate of 0.1%, the detection rate in the who received daily pravastatin beginning in
1 to 9 year age group has been shown to be childhood demonstrate a remarkable absence of
88% for total cholesterol and 85% for LDL major adverse cardiovascular events (MACE) at
cholesterol. age 40 years and older, in contrast to the child’s
• The best method of screening. Reliance affected parent. Arterial findings are comparable
on family history alone as a basis for lipid to those of their unaffected siblings.3 Nonetheless,
screening fails to identify as many as 30% data from the United States and Europe suggest
to 60% of youth with elevated cholesterol. low rates of screening and inadequate treatment

260  ENDO 2021 • Endocrine Case Management

chased by Dr. Khalid H. Seedahmed, Jr., MRCP PGDip, CertEndocrinology SA - ID 283


of children with familial hypercholesterolemia, Question 1: Was it appropriate for the
resulting in missed opportunities to prevent pediatrician to perform cholesterol
premature cardiovascular disease. screening in a healthy 10-year-old girl?
What are the current cholesterol screening
guidelines in the United States?
Clinical Case Vignettes Current American Academy of Pediatrics and
Case 1 American Heart Association recommendations are
A 16-year-old girl returns for a follow-up as follows.
visit. She was initially seen at age 10 years for
Who to screen:
hypercholesterolemia that was detected during
Two categories of screening have been
a routine health maintenance screening by her
recommended.
pediatrician. Your clinic notes indicate that
when first seen, she was in good health, was 1. Selective screening of any child ≥2 years of age:
participating in school-sponsored sports, followed
an age-appropriate diet, had no history of chronic a. In whom 1 or both parents are known to
illness, and was taking no medications or dietary have hypercholesterolemia or are receiving
supplements. lipid-lowering medications.
On physical examination at her initial visit b. Who has a family history of premature
6 years ago, her height and weight were at the CVD (men <55 years of age; women <65
50th percentile for age and sex. BMI was 24 kg/m2. years of age).
Blood pressure was normal. There were no c. Whose family history is unknown (eg, child
xanthoma. The rest of the examination findings was adopted).
were normal. d. Children at moderate to high risk for
premature CVD.
Initial laboratory findings at age 10 years:
Total cholesterol = 224 mg/dL (<170 mg/dL) 2. Universal screening of all children 9 to
(SI: 5.80 mmol/L [<4.40 mmol/L]) 11 years of age regardless of general health
Triglycerides = 185 mg/dL (<150 mg/dL) or the presence/absence of CVD risk factors.
(SI: 2.09 mmol/L [<1.70 mmol/L])
HDL cholesterol = 45 mg/dL (>40 mg/dL)
If the initial cholesterol value is normal,
(SI: 1.17 mmol/L [>1.04 mmol/L]) screening should be repeated between 17 and
LDL cholesterol = 142 mg/dL (<130 mg/dL) 21 years of age.
(SI: 3.68 mmol/L [<3.67 mmol/L])
Non–HDL cholesterol = 179 mg/dL (<145 mg/dL)
(SI: 4.64 mmol/L [<3.76 mmol/L]) Question 2: What is the preferred
method of cholesterol screening?
At the time of her initial evaluation, a registered Screening can be performed using a fasting or
dietitian provided the family with instructions for nonfasting lipid profile. A nonfasting blood sample
a low-fat diet and tips for a heart-healthy lifestyle, is often more practical and efficient. Blood samples
including the benefits of 30 to 60 minutes of can be drawn by venipuncture or fingerstick. If
moderate-to-vigorous exercise each day. She was available, point-of-care lipid testing is reliable and
counseled to avoid cigarette smoking, including correlates well with standard laboratory results.
exposure to secondhand smoke. Dietary fiber (5 g Significant triglyceride elevation (eg,
+ age in years per day) and modest salt intake >400 mg/dL [>4.52 mmol/L]) precludes
were encouraged. A repeated lipid profile was calculation of LDL cholesterol using the
recommended in 4 to 6 weeks. However, the child Friedewald formula, in which case LDL cholesterol
was lost to follow-up. can be measured with a direct assay. Ingestion of

ENDO 2021 • Pediatric Endocrinology  261

chased by Dr. Khalid H. Seedahmed, Jr., MRCP PGDip, CertEndocrinology SA - ID 283


food or beverages has a minimal effect on directly How to interpret the results (see Figure):
measured LDL cholesterol and lipoprotein (a). Use Pediatric “norms” are generally based on
of the Martin-Hopkins calculation provides better population-derived percentiles rather than on
estimates of LDL cholesterol. outcome studies.

Figure. Overview of Atherogenic Lipoproteins

All Apo B containing lipoproteins are potentially atherogenic.

ApoB *Non HDL-C


Total-C

Advantages of non HDL-C


• No additional cost.
• No need for fasting.
• Better predictor of CVD risk.

(J Apo 8 100
. . Apo Al
\ ;: Apo(a)
t *Non HDL-C = Total Chol - HDL-C

Table 1. Acceptable, Borderline, and High Plasma Lipid and Lipoprotein Concentrations for Children and Adolescents

Category Acceptable Borderline High* Low

Total cholesterol <170 mg/dL 170-199 mg/dL ≥200 mg/dL —


(SI: <4.40 mmol/L) (SI: 4.40-5.15 mmol/L) (SI: ≥5.18 mmol/L)

LDL cholesterol <110 mg/dL 110-129 mg/dL ≥130 mg/dL —


(SI: <2.85 mmol/L) (SI: 2.85-3.34 mmol/L) (SI: ≥3.37 mmol/L)

Non-HDL cholesterol <120 mg/dL 120-144 mg/dL ≥145 mg/dL —


(SI: <3.11 mmol/L) (SI: 3.11-3.73 mmol/L) (SI: ≥3.76 mmol/L)

Triglycerides

  0-9 years <75 mg/dL 75-99 mg/dL ≥100 mg/dL —


(SI: <0.85 mmol/L) (SI: 0.85-1.12 mmol/L) (SI: ≥1.13 mmol/L)

  10-19 years <90 mg/dL 90-129 mg/dL ≥130 mg/dL —


(SI: <1.02 mmol/L) (SI: 1.02-1.46 mmol/L) (SI: ≥1.47 mmol/L)

HDL cholesterol >45 mg/dL 40-45 mg/dL — <40 mg/dL


(SI: >1.17 mmol/L) (SI: 1.04-1.17 mmol/L) (SI: <1.04 mmol/L)

*Low cutoff points for HDL cholesterol represent approximately the 10th percentile. The cutoff points for high and borderline-high values represent
approximately the 95th and 75th percentiles, respectively.1

262  ENDO 2021 • Endocrine Case Management

chased by Dr. Khalid H. Seedahmed, Jr., MRCP PGDip, CertEndocrinology SA - ID 283


Case 1 (Continued) Question 1: Would additional tests be helpful?
At age 16 years, the patient appears to be in good
Advanced Lipid Testing
health, has no history of significant medical
In children, routine measurement of lipoprotein
illnesses, and has not been on any medications
(a), apolipoprotein B, apolipoprotein A1, and
since her initial visit 6 years ago.
lipoprotein subclasses and their sizes by
On physical examination, she has no xanthoma.
advanced lipoprotein analysis is not currently
Acanthosis nigricans is present in both axilla and
recommended, although one or more may be
the neck. Her weight has increased 40 lb (18.2 kg),
helpful in selected cases.
resulting in a BMI greater than the 95th percentile.
There has been growing interest, for example,
Initial and current laboratory test results (see table in screening for lipoprotein (a), based on ample
below). support of its role as an independent, causative
CVD risk factor.4 Adult levels of lipoprotein (a)
Other current test results include the following: are achieved by 2 to 5 years of age, and, thereafter,
Fasting glucose = 88 mg/dL (70-99 mg/dL) may serve as a reliable biomarker in helping assess
(SI: 4.9 mmol/L [3.9-5.5 mmol/L]) CVD risk in youth. The cholesterol contained in
ALT, normal lipoprotein (a) is measured in the LDL-cholesterol
AST, normal fraction. Although acute inflammation may
TSH, normal
Hemoglobin A1c = 6.2% (4.0%-5.6%) (44 mmol/mol
temporarily alter the lipoprotein (a) level, in
[20-38 mmol/mol]) general the level is stable and remains constant
throughout the lifespan of an individual. Elevated
The patient is doing well academically. However, lipoprotein (a), defined as greater than 50 mg/dL
she is no longer participating in school-sponsored (>100 nmol/L), is the most common genetic
sports, rarely exercises, and admits to occasional dyslipidemia, affecting nearly 1 in 5 individuals in
cigarette smoking. She is sexually active but is the United States.
using no form of birth control.
The child’s mother, age 42 years, is taking a Question 2: Based on this patient’s findings,
statin for elevated cholesterol, and her maternal which of the following is the most
grandfather had a myocardial infarction at age likely cause of her lipid abnormality?
44 years and subsequently required a stent. The In the absence of secondary causes of
patient’s parents are divorced, and the paternal hypercholesterolemia, the most likely diagnosis
family history is unknown. is familial hypercholesterolemia (~1:200-1:500).5

Age

Analyte 10 Years 16 Years Expected Value

Total cholesterol 224 mg/dL (SI: 5.80 mmol/L) 260 mg/dL (SI: 6.73 mmol/L) <170 mg/dL (SI: <4.40 mmol/L)

Triglycerides 185 mg/dL (SI: 2.09 mmol/L) 165 mg/dL (SI: 1.86 mmol/L) <150 mg/dL (SI: <1.70 mmol/L)

HDL cholesterol 45 mg/dL (SI: 1.17 mmol/L) 55 mg/dL (SI: 1.42 mmol/L) >40 mg/dL (SI: >1.04 mmol/L)

LDL cholesterol 142 mg/dL (SI: 3.68 mmol/L) 172 mg/dL (SI: 4.46 mmol/L) <130 mg/dL (SI: <3.37 mmol/L)

Non–HDL cholesterol 179 mg/dL (SI: 4.64 mmol/L) 205 mg/dL (SI: 5.31 mmol/L) <145 mg/dL (SI: <3.76 mmol/L)

ENDO 2021 • Pediatric Endocrinology  263

chased by Dr. Khalid H. Seedahmed, Jr., MRCP PGDip, CertEndocrinology SA - ID 283


Familial hypercholesterolemia is usually Question 3: What treatment would you
due to a heterozygous pathogenic variant in recommend? What are your treatment goals?
the LDL-receptor gene causing decreased The treatment goal is an LDL-cholesterol
clearance of LDL particles from plasma. Clinical concentration of 100 mg/dL or less
manifestations include: (1) moderate to severe (≤2.59 mmol/L), or a minimum of 50% reduction
hypercholesterolemia; (2) cholesterol deposition from baseline. Choice of a statin is a matter
in tendons (eg, Achilles) and occasionally in the of preference, often influenced by the child’s
skin (eg, xanthelasma) and the cornea (arcus), but insurance and out-of-pocket costs. All statins have
usually not until middle age; and (3) premature been approved by the US FDA for use in children
atherosclerotic cardiovascular disease (2 to 5 times with familial hypercholesterolemia when the
in middle age for untreated heterozygous familial LDL-cholesterol level is greater than 190 mg/dL
hypercholesterolemia). (>4.92 mmol/L) or greater than 160 mg/dL
Youth with familial hypercholesterolemia have (>4.14 mmol/L) with 1 or more additional risk
severe alterations in blood cholesterol and incur factor.
the highest risk for CVD, yet they have few, if any, The risks of pharmacologic treatment in youth
signs or symptoms. With treatment, event rates appear to be low and quite similar to the short-
in heterozygous familial hypercholesterolemia are term risks in adults. In general, high-intensity
reduced close to that of the general population (in statin therapy has not been studied in children,
the absence of any other CVD risk factors). and the longest study is approximately 20 years.
Clinical experience suggests that statin-related
adverse effects in youth are rare.

Table 2. Initial Screening and Intervention Guidelines for LDL Cholesterol

Initial Screening and Intervention Guidelines - LDL Cholesterol


LDL-C 130-159 mg/di 160- 189 mg/di > 190mg/dl
.=::_ 2 High Level RF
o, 1 High Level RF
1 High Level RF o, NA
Risk + .=::. 2 Moderate Level
Counsel regarding
None 2: 2 Moderate Level None
Factors RF avoidance/ reduction
RF o,
o, Positive FHx
of acquired CVD
ainical CVD risk factors

Family Hx Negative +/- Negative Positive


Diet CHILD-2-LDL CHILD-2-LDL CHILD-2-LDL CHIL0-2-LOL CHI LD-2-LDL
TLC
Rx None* Start Sta.tin- None* Start Sta.tin- Start Stalin
Baseline Baseline Baseline
Year 1 Year 1 Year 1
Monitoring • 4-6weeks • 4-6 weeks • 4-6weeks
medication • q 3-6 mos. • q 3-6 mos. • q 3-6 mos.
NA NA
efficacy and Year2 Year2 Year2
safety. • q6mos. • q6mos. • q6mos.
Then Then Then
• q 12 mos. • q 12mos. • q12mos.
* Repeat fasting llpld profile and update FHx and RF q 6 months. **If TLC alone is unsuccessful

264  ENDO 2021 • Endocrine Case Management

chased by Dr. Khalid H. Seedahmed, Jr., MRCP PGDip, CertEndocrinology SA - ID 283


Case 1 (Continued) Findings included that women were less likely
than men to:
Following a discussion of the benefits and
potential adverse effects of statins, the patient • be taking lipid-lowering medication at baseline
agrees to start atorvastatin, 20 mg daily. Because (58.4% vs 61.1%; P <.01)
she is sexually active, an oral contraceptive pill is
• be on combination therapy (19.9% vs 22.7%;
also recommended. Adherence to her treatment
P <.01)
regimen is excellent, and follow-up laboratory
studies confirm a good response, with no adverse • achieve an LDL-cholesterol level less than
events (see lab results in table below). 100 mg/dL (12.7% vs 14.6%; P <.01)
There were 7982 children in the European
Question 1: Should treatment be delayed
Atherosclerosis FHSC Global Registry, including
because the patient is female?
4235 older than 9 years.
Although the 2011 Expert Panel made no gender
distinction with respect to statin treatment for Children
familial hypercholesterolemia, arguments for
treating female patients before adulthood are • ≤9 years—38.9% were receiving lipid-lowering
more complex given that only 30% of women medication (5.6% receiving statins)
with heterozygous familial hypercholesterolemia • ≥10 years—35.9% treated with lipid-lowering
develop symptomatic CVD by age 60 years medication (12.4% receiving statins)
compared with 50% of men who develop CVD by
age 50 years. Median LDL cholesterol
Recent data from the European Atherosclerosis
Society 2020 Virtual Congress suggest that female • ≤9 years—176.7 mg/dL (4.58 mmol/L) (95%
patients and children are generally undertreated. In CI, 143.1-218.1 mg/dL [3.71-5.65 mmol/L])
36,835 adults with probable or definite heterozygous • ≥10 years—174.0 mg/dL (4.51 mmol/L) (95%
familial hypercholesterolemia (45.8% men and 54.2% CI, 137.0-215.0 mg/dL [3.55-5.57 mmol/L])
women), the average age at diagnosis was 46.0 years
for women vs 43.5 years for men.

Age

Analyte 10 years 16 years Atorvastatin Expected

Total cholesterol 224 mg/dL 260 mg/dL 156 mg/dL <170 mg/dL
(SI: 5.80 mmol/L) (SI: 6.73 mmol/L) (SI: 4.04 mmol/L) (SI: <4.40 mmol/L)

Triglycerides 185 mg/dL 165 mg/dL 50 mg/dL <150 mg/dL


(SI: 2.09 mmol/L) (SI: 1.86 mmol/L) (SI: 0.57 mmol/L) (SI: <1.70 mmol/L)

HDL cholesterol 45 mg/dL 55 mg/dL 52 mg/dL >40 mg/dL


(SI: 1.17 mmol/L) (SI: 1.42 mmol/L) (SI: 1.35 mmol/L) (SI: >1.04 mmol/L)

LDL cholesterol 142 mg/dL 172 mg/dL 94 mg/dL <130 mg/dL


(SI: 3.68 mmol/L) (SI: 4.46 mmol/L) (SI: 2.43 mmol/L) (SI: <3.37 mmol/L)

Non–HDL cholesterol 179 mg/dL 205 mg/dL 104 mg/dL <145 mg/dL
(SI: 4.64 mmol/L) (SI: 5.31 mmol/L) (SI: 2.69 mmol/L) (SI: <3.76 mmol/L)

ENDO 2021 • Pediatric Endocrinology  265

chased by Dr. Khalid H. Seedahmed, Jr., MRCP PGDip, CertEndocrinology SA - ID 283


Question 2: Is this patient’s elevated Question 3: Is there evidence that
triglycerides of concern? lipid-lowering medications, such as
While most individuals with familial statins, are safe when prescribed at a
hypercholesterolemia have normal triglycerides, young age? If so, are they effective?
the presence of an elevated value does not exclude Clinical trials of children 10 years and older
familial hypercholesterolemia. It should be noted show that pharmacologic treatment is safe and
that this patient’s BMI is greater than the 95th effective and improves noninvasive measures
percentile, she has acanthosis nigricans, and her of atherosclerosis, such as carotid intima-media
hemoglobin A1c level is 6.2% (44 mmol/mol), thickness. Most of the available evidence focuses
suggesting that she has become insulin resistant on the relatively short-term use of statins; data on
or prediabetic. Such findings, which enhance the safety and efficacy of high-dosage statins and
CVD risk, are common in youth. Secondary cholesterol absorption inhibitors are limited.
causes of dyslipidemia (Table 3) must be excluded. A 20-year follow-up study of pravastatin
Treatment of hypertriglyceridemia is outlined treatment in children with familial
(Table 4). hypercholesterolemia (beginning in childhood)

Table 3. Secondary Causes of Dyslipidemia

Condition Screening Tests

Hypothyroidism Free T4, TSH

Liver diseases Complete metabolic panel

Kidney diseases Complete metabolic panel, urine analysis

Diabetes mellitus Complete metabolic panel, urine analysis, fasting or random glucose

Obesity, insulin resistance Glucose, hemoglobin A1c, complete metabolic panel, fasting glucose and insulin

Medications Including steroids, isotretinoin and, in females, oral contraceptive pills

Table 4. Treatment of Hypertriglyceridemia

Treatment of Hypertriglyceridemia
TG (mg/dl) <150 150-499 500-999 ~1000

Goal Health Maintenance CVD Risk Prevention Avoid Pancreatitis

Rx Target Non HDL-C < 130 Triglyeerides < 150


Diet
Fat* 25-30% 25-30% 15-30% <10-15%
CHO* 55-60% Avoid Excessive Consumption
TG Lowering Rx
. • st
Consider statm as 1 Consider Usually
None
line therapy Fibrate;O3FA;Niacin ineffective
• % of daily kcal/EER
-

266  ENDO 2021 • Endocrine Case Management

chased by Dr. Khalid H. Seedahmed, Jr., MRCP PGDip, CertEndocrinology SA - ID 283


demonstrated a remarkable absence of major Question 5: Now that you have
adverse cardiovascular events at age 40 years and initiated treatment, is there a
older in contrast to the affected parent. Arterial role for cascade screening?
findings were comparable to their unaffected Screening the family of a patient with familial
siblings. This study provided evidence that statin hypercholesterolemia by systematically identifying
therapy, started in childhood, is well tolerated, first-degree relatives (ie, cascade screening), is
slows the progression of carotid intima-media recommended to enhance detection of individuals
thickness, and significantly reduces the risk of at-risk for familial hypercholesterolemia.
atherosclerotic CVD–related events. Measurement of cholesterol is the most common
screening method, although false-negatives can
Question 4: Is there a role for genetic testing? occur. Screening for a known genetic pathogenic
Genetic testing is recommended for all variant is more definitive. Although treatment is
individuals clinically suspected to have familial recommended for both adults and children, adults
hypercholesterolemia.6 Genetic testing: (1) has in the family, many of whom may be unaware of
the potential to guide treatment decisions based their hypercholesterolemia, are at greater short-
on risk stratification; (2) may be important in term risk than affected children.
helping youth and parents understand the benefits
of adherence to lipid-lowering treatment; and (3)
Case 1 (Continued)
further increases our understanding of genotype-
phenotype correlations. Testing should be used This patient was diagnosed with heterogeneous
with caution, being sensitive to the cost of testing familial hypercholesterolemia. Genetic testing
and the potential of unintended consequences (eg, demonstrated a common heterozygous pathogenic
identifying nonpaternity). Pretest and posttest variant in the gene encoding the LDL receptor.
counseling should carefully review the child’s risk All first-degree relatives were offered cascade
and the potential benefits of lipid-lowering therapy screening.
with or without a known pathogenic variant.7 She is now 17 years old. At a follow-up clinic
When performed, genetic testing for suspected appointment, it is noted that she missed her last
familial hypercholesterolemia should include menstrual period. Subsequent laboratory studies
analysis of LDLR, APOB, and PCSK9, sequencing confirm that she is 8 weeks pregnant. Her only
of all LDLR and PCSK9 exons and exon/intron medications are atorvastatin, 20 mg daily, and an
boundaries, as well as LDLR deletion/duplication oral contraceptive pill daily. Her most recent lipid
analysis and the APOB exons encoding the LDLR panel and past results are shown (see lab results in
ligand-binding region. When a pathogenic variant table on the following page).
is found in an index case, family members should Other current test results include the following:
be offered genetic cascade testing, if feasible.
For individuals with negative genetic test Fasting glucose = 90 mg/dL (70-99 mg/dL)
results, other pathogenic variants (eg, APOE; (SI: 5.0 mmol/L [3.9-5.5 mmol/L])
ALT, normal
autosomal recessive dyslipidemias [eg, biallelic AST, normal
LDLRAP1]), other conditions associated with TSH, normal
hypercholesterolemia (eg, lysosomal acid lipase Hemoglobin A1c = 6.1% (4.0%-5.6%) (43 mmol/mol
deficiency), and as-yet unidentified genes or [20-38 mmol/mol])
polygenic inheritance should be considered. hCG, positive
Clinicians should be aware that some parents
might assume that a lipid-lowering medication is
unnecessary for children in whom no common
genetic pathogenic variant is found.

ENDO 2021 • Pediatric Endocrinology  267

chased by Dr. Khalid H. Seedahmed, Jr., MRCP PGDip, CertEndocrinology SA - ID 283


Age

Analyte 10 years 16 years On atorvastatin Current Expected

Total cholesterol 224 mg/dL 260 mg/dL 156 mg/dL 214 mg/dL <170 mg/dL
(SI: 5.80 mmol/L) (SI: 6.73 mmol/L) (SI: 4.04 mmol/L) (SI: 5.54 mmol/L) (SI: <4.40 mmol/L)

Triglycerides 185 mg/dL 165 mg/dL 50 mg/dL 80 mg/dL <150 mg/dL


(SI: 2.09 mmol/L) (SI: 1.86 mmol/L) (SI: 0.57 mmol/L) (SI: 0.90 mmol/L) (SI: <1.70 mmol/L)

HDL cholesterol 45 mg/dL 55 mg/dL 52 mg/dL 50 mg/dL >40 mg/dL


(SI: 1.17 mmol/L) (SI: 1.42 mmol/L) (SI: 1.35 mmol/L) (SI: 1.30 mmol/L) (SI: >1.04 mmol/L)

LDL cholesterol 142 mg/dL 172 mg/dL 94 mg/dL 148 mg/dL <130 mg/dL
(SI: 3.68 mmol/L) (SI: 4.46 mmol/L) (SI: 2.43 mmol/L) (SI: 3.83 mmol/L) (SI: <3.37 mmol/L)

Non–HDL 179 mg/dL 205 mg/dL 104 mg/dL 164 mg/dL <145 mg/dL
cholesterol (SI: 4.64 mmol/L) (SI: 5.31 mmol/L) (SI: 2.69 mmol/L) (SI: 4.25 mmol/L) (SI: <3.76 mmol/L)

Question. How does the patient’s pregnancy hypercholesterolemia in pregnant women with
affect your treatment recommendations? familial hypercholesterolemia.
Statins and other lipid-lowering medications All women with hypercholesterolemia who
are classified as pregnancy category X drugs. wish to become pregnant, especially those on
Evidence, however, for teratogenicity associated treatment, should receive prepregnancy counseling
with statin use is scant and contradictory. The and instructions to stop statins, ezetimibe, and
issue of pregnancy in a young woman with niacin at least 4 weeks before discontinuing
heterogeneous familial hypercholesterolemia raises contraception. These treatments SHOULD NOT
several concerning issues, both for the health of BE USED during pregnancy. In the case of an
the mother, as well as that of the fetus. Studies unintended pregnancy, a woman with familial
suggest that exposure to hypercholesterolemia hypercholesterolemia should discontinue statins,
in utero may predispose the fetus to increased ezetimibe, and niacin immediately. The use of
risk of premature atherosclerosis later in other lipid-lowering medications (eg, colesevelam)
life. Currently, no evidence-based guidelines may be considered. In women who are
exist on the optimal clinical approach to treat breastfeeding, lipid-lowering medications should
also be withheld.

References
1. Expert Panel on Integrated Guidelines for Cardiovascular Health and Risk 5. Blackett P, George M, Wilson DP. Integrating lipid screening with ideal
Reduction in Children and Adolescents, National Heart Lung and Blood cardiovascular health assessment in pediatric settings. J Clin Lipidol.
Institute. Expert panel on integrated guidelines for cardiovascular health 2018;12(6):1346-1357. PMID: 30253939
and risk reduction in children and adolescents: summary report. Pediatrics. 6. Sturm AC, Knowles JW, Gidding SS, et al; Convened by the Familial
2011;128(Suppl 5):S213-S256. PMID: 22084329 Hypercholesterolemia Foundation. Clinical genetic testing for familial
2. de Ferranti SD, Steinberger J, Ameduri R, et al. Cardiovascular risk reduction hypercholesterolemia: JACC Scientific Expert Panel. J Am Coll Cardiol.
in high-risk pediatric patients: a scientific statement from the American Heart 2018;72(6):662-680. PMID: 30071997
Association. Circulation. 2019;139(13):e603-e634. PMID: 30798614 7. Brown EE, Sturm AC, Cuchel M, et al. Genetic testing in dyslipidemia:
3. Luirink IK, Wiegman A, Kusters DM, et al. 20-year follow-up of a scientific statement from the National Lipid Association. J Clin Lipidol.
statins in children with familial hypercholesterolemia. N Engl J Med. 2020;14(4):398-413. PMID: 32507592
2019;381(16):1547-1556. PMID: 31618540
4. Wilson DP, Jacobson TA, Jones PH, et al. Use of lipoprotein (a) in
clinical practice: a biomarker whose time has come. A scientific statement
from the National Lipid Association. J Clin Lipidol. 2019;13(3):374-392.
PMID: 31147269

268  ENDO 2021 • Endocrine Case Management

chased by Dr. Khalid H. Seedahmed, Jr., MRCP PGDip, CertEndocrinology SA - ID 283


REPRODUCTIVE
ENDOCRINOLOGY

chased by Dr. Khalid H. Seedahmed, Jr., MRCP PGDip, CertEndocrinology SA - ID 283


Diagnosis and Management
of Functional Hypogonadism
in the Male Patient
Mathis Grossmann, MD, PhD, FRACP. University of Melbourne Austin Health, Heidelberg,
Victoria, Australia; E-mail: mathisg@unimelb.edu.au

Learning Objectives (eg, phosphodiesterase-5 inhibitors for


sexual dysfunction) or end-organ deficits (eg,
As a result of participating in this session, learners
antiresorptive therapy for osteoporosis).
should be able to:
Not uncommonly, measures to reverse
• Assess and evaluate men for clinically suppression of the functional HPT axis are
significant androgen deficiency in the setting difficult to implement successfully and, even if
of confounding comorbidities. successful, may not be sufficient. In this situation,
the question arises as to whether testosterone
• Manage men presenting with functional
treatment is appropriate. While recent
hypogonadism using an evidence-based
randomized controlled trials in carefully selected
approach for nontestosterone- and
middle-aged and older men have reported short-
testosterone-based treatment.
term effects of testosterone treatment, effects of
testosterone treatment on important long-term
health outcomes remain unknown. If testosterone
treatment is considered, it should be directed
Main Conclusions towards men with significant clinical features
Functional suppression of the male hypothalamic- and persistently low circulating testosterone after
pituitary-testicular (HPT) axis due to acute or exclusion of contraindications and appropriate
chronic health conditions is a common clinical counseling, with guideline-conforming
problem in older men, but it can also occur monitoring for efficacy and adverse effects.
in young men. Functional hypogonadism is
a diagnosis of exclusion and requires careful,
individualized assessment to exclude organic
hypogonadism due to anatomic pathology of the
Significance of the
HPT axis. Clinical Problem
While definitive randomized controlled trials Older men, especially if overweight or obese and
are lacking, the available evidence suggests that experiencing comorbidities, often present with
measures to optimize ill health (eg, achieving a nonspecific androgen deficiency-like symptoms,
healthy body weight, optimizing comorbidities, such as low energy and sexual dysfunction, and
stopping offending medications) can improve modestly lowered serum testosterone relative
androgen deficiency-like symptoms and lead to to reference ranges based on healthy young
a modest increase in circulating testosterone. men. Most such men do not have organic
Moreover, such men can respond to targeted hypogonadism due to classic pituitary or testicular
treatments addressing relevant symptoms disease, but instead have functional hypogonadism.

270  ENDO 2021 • Endocrine Case Management

chased by Dr. Khalid H. Seedahmed, Jr., MRCP PGDip, CertEndocrinology SA - ID 283


Hence, they have potentially reversible HPT Strategies for Diagnosis,
axis suppression due to acute and chronic health
Therapy, and/or Management
conditions external to the HPT axis, such as
medication use and/or lifestyle behaviors. Notably, Background: Organic vs
ill health can also cause functional hypogonadism Functional Hypogonadism
in young men in the context of energy deficit (eg, In the clinical approach to men with suspected
the reversible energy deficit of sport syndrome, androgen deficiency, distinguishing organic from
eating disorders, and body dysmorphic disorders) functional hypogonadism is important (Table 1),
or during recovery from anabolic steroid use. because organic hypogonadism can be due to
In the clinical approach to such men, making potentially serious pathology (eg, a pituitary
the distinction between organic and functional tumor). Moreover, this distinction influences the
hypogonadism is important. This is because treatment approach.
organic hypogonadism can be due to potentially
serious pathology, such as a pituitary lesion. Organic Hypogonadism
Moreover, the treatment approach differs. Organic hypogonadism due to medical disease
While organic hypogonadism is generally treated of the HPT axis, such as a pituitary tumor or
with testosterone replacement, in men with Klinefelter syndrome, is an important diagnosis
functional hypogonadism, lifestyle measures— not to be missed. Organic hypogonadism should
especially achieving a healthy body weight, be considered in a man of any age who presents
optimizing comorbidities, and stopping offending with clinical and biochemical features suggestive
medications—may improve symptoms and of androgen deficiency, including unexplained
increase serum testosterone. psychosexual complaints, osteoporosis, anemia, or
sarcopenia.2
Barriers to Optimal Practice While randomized controlled trials are lacking,
clinical experience and uncontrolled studies of
In some men, clinical and biochemical features
testosterone replacement in men with organic
consistent with androgen deficiency may
hypogonadism demonstrate marked benefits on
persist despite optimization of body weight and
sexual function, energy, hemoglobin, bone density,
comorbidities. In such men, reevaluation for
and body composition.3 Testosterone replacement
organic HPT axis pathology should be considered,
should be considered irrespective of age, noting
as this can be missed.
that testosterone does not improve fertility but
Measures to reverse functional hypogonadism
instead can suppress spermatogenesis.
may be unsuccessful in some patients (eg, weight
loss is not achieved or sustained) or are not
Functional Hypogonadism
feasible (eg, cessation of opioids for chronic pain).
Many older men, especially when overweight
Therefore, the clinician is commonly faced with the
or obese and living with chronic disease (such
question as to whether testosterone therapy should
as diabetes, depression, liver, or renal disease),
be considered, and there is vigorous debate about
present with nonspecific, androgen deficiency-
this issue. Although recent randomized controlled
like symptoms such as fatigue, low energy, and
trials have provided data on the short-term benefits
sexual dysfunction (low libido and/or erectile
and risks of testosterone treatment in older men,
dysfunction). They may also have clinical features
definitive studies assessing the long-term effects
overlapping with those of organic hypogonadism
of testosterone treatment on important health
such as central adiposity, sarcopenia, and reduced
outcomes are currently not available.
bone mineral density. Such men commonly have
modest reductions in their circulating testosterone
concentrations, fluctuating in the lower range

ENDO 2021 • Reproductive Endocrinology  271

chased by Dr. Khalid H. Seedahmed, Jr., MRCP PGDip, CertEndocrinology SA - ID 283


Table 1. Organic Hypogonadism vs Functional Hypogonadism in Middle-Aged and Older Men

Clinical Aspects Organic Hypogonadism Functional Hypogonadism

Condition Proven HPT axis pathology (structural, No recognizable structural intrinsic HPT axis pathology
destructive, or congenital disease)
No specific pathologic etiologies of functional
hypogonadism (diagnosis of exclusion)

Reversibility Established disease state, organic, and HPT axis suppression is functional and may be reversible
generally irreversible HPT axis pathology

Symptoms/signs Specific: eunuchoidism Less specific: erectile dysfunction, low energy and mood
More specific/objective: low libido, small
testes, loss of male hair, gynecomastia

Testosterone levels Unequivocally, consistently, and severely Borderline low, fluctuating around the lower limit of assay
low range, occasionally severely low

Gonadotropin levels Elevated (primary hypogonadism) or Usually in the normal range, occasionally low (secondary
low/inappropriately normal (secondary hypogonadism)
hypogonadism)

Association of low Causal Uncertain, symptoms may be predominantly or partially


testosterone with symptoms due to comorbid illness

Testosterone therapy Replacement Replacement?

Benefits of therapy Marked symptomatic and somatic Symptomatic and somatic response less well established
response (except fertility)

Risks of therapy Considered low relative to benefits Unknown

Abbreviation: HPT, hypothalamic-pituitary-testicular. Adapted in part from Endocr Pract. 2013;19(5):853-863.1

relative to reference ranges based on healthy (overtraining, eating and body dysmorphic
young men (typically ranging from 173-288 ng/dL disorders) or during recovery from anabolic
(6.0-10.0 nmol/L).4 steroid use is increasingly recognized.6,7
The estimated prevalence of functional
hypogonadism (in older men also referred Workup of Older Men Presenting
to as late-onset hypogonadism), defined as
the coexistence of androgen deficiency-like With Features Suggestive
features and lowered circulating testosterone in of Androgen Deficiency
the absence of organic HPT axis pathology, is The clinical workup is primarily geared towards
estimated to be 2% to 5% in community-dwelling excluding a diagnosis of organic hypogonadism.
older men.5 However, the extent to which lowered Given that hypogonadism is a clinical diagnosis
testosterone concentrations contribute to the supported by consistent biochemical findings, men
aging male phenotype is not known. It is also who present with features suggestive of androgen
possible that lowered testosterone is primarily a deficiency should have a thorough history and
consequence of functional hypothalamic-pituitary physical examination to determine the degree of
suppression due to ill health. Low testosterone, at clinically significant androgen deficiency.2 Clinical
the very least, is a sensitive marker of poor health. assessment should be focused on eliciting the
Of note, functional hypogonadism due to more specific features of androgen deficiency,
acute or chronic ill health can also occur in young such as gynecomastia, recent loss of body hair,
men. Both under- and overnutrition can suppress and decreasing testicular volume. Many features
the male HPT axis. Functional hypogonadism such as fatigue, low libido, and reduced muscle
in young men in the context of energy deficit

272  ENDO 2021 • Endocrine Case Management

chased by Dr. Khalid H. Seedahmed, Jr., MRCP PGDip, CertEndocrinology SA - ID 283


bulk are nonspecific and can be caused by almost Management of Functional
any chronic disease.4 Initial assessment should Hypogonadism
include the identification of comorbidities that
may confound the clinical picture or represent Lifestyle and Optimization of Comorbidities
potentially reversible causes such as obesity, Priority should lie in the management of reversible
depression, uncontrolled sleep apnea, or causes, most importantly the implementation
medications (eg, opioids or glucocorticoids). Clues of lifestyle measures to optimize body weight
to underlying organic etiology, such as signs of and comorbidities (eg, depression, sleep apnea,
pituitary dysfunction or mass effect, should not be or glycemic control in diabetes) and, if possible,
missed. Surreptitious anabolic steroid use should removal of offending medications (eg, opioids,
be considered. glucocorticoids) (Figure). This strategy can
The clinical impression of androgen deficiency improve nonspecific symptoms, lead to modest
should be confirmed by documenting at least 2 increases in testosterone, and have general
low serum total testosterone concentrations by health benefits. In some observational studies,
an accurate and reliable assay, with the sample implementation of continuous positive airway
drawn in the morning in the fasted state.2 pressure for sleep apnea, improvement of glycemic
Testosterone should not be measured during an control in men with diabetes, and opioid or
intercurrent illness, as this can lead to temporary glucocorticoid cessation, have been associated with
HPT axis suppression. Measurement of SHBG increases in testosterone concentrations of around
and determination of free testosterone (usually 58 to 144 ng/dL (2.0-5.0 nmol/L).4 Explaining this
calculated) may provide additional information to affected men can increase their motivation to
in men with borderline total testosterone and/or engage with these measures.
conditions in which abnormalities in SHBG are Obesity is the strongest risk factor for low
suspected. testosterone, even overriding the effects of age, in
Most men with functional hypogonadism part because obesity blunts the age-related rise in
have low-normal gonadotropin concentrations LH that can compensate for testicular dysfunction
due to nondestructive hypothalamic-pituitary occurring in some older men.8 Consistent with
inhibition from chronic disease, including being this, successful weight loss, whether by diet
obese or underweight.4 Especially in older men, or surgery, can lead to substantial increases
evaluation for underlying organic HPT axis in testosterone in obese men. The increase in
pathology is commonly of low yield and should testosterone is proportional to the amount of
be individualized. In men with nonelevated weight lost: 10% weight loss increases testosterone
gonadotropins, the probability of organic by 58 to 86 ng/dL (2.0-3.0 nmol/L), whereas
hypothalamic-pituitary pathology is inversely profound weight loss after bariatric surgery in
related to BMI, age, number of comorbidities, morbidly obese men can raise testosterone by
and testosterone concentrations. In the absence of more than 288 ng/dL (>10 nmol/L).4
clinical suspicion of pituitary disease, biochemical In addition to general measures, predominant
workup can be limited to measuring prolactin, clinical features can be addressed with targeted
and in men younger than 65 years, iron studies to therapy (Table 2).
exclude hemochromatosis. The testosterone cutoff
Testosterone Therapy
below which pituitary imaging is necessary is not
In some men, measures to reverse functional
well defined, but imaging should be considered in
hypogonadism may be unsuccessful, either because
men with a total testosterone concentration less
implementation is not feasible (eg, cessation of
than 150 ng/dL (<5.2 nmol/L) and low to normal
opioids) or they are not achieved or maintained
gonadotropins, even in the absence of clinical
(eg, weight loss). In others, symptoms may persist
suspicion of hypothalamic pituitary disease.2
even despite successful implementation of these

ENDO 2021 • Reproductive Endocrinology  273

chased by Dr. Khalid H. Seedahmed, Jr., MRCP PGDip, CertEndocrinology SA - ID 283


Figure. Approach to Possible Hypogonadism

Young Age Old

Normal Weight Obese

None Comorbidities Present

Specific Symptoms Non-specific

Consistently Testosterone Borderline


Low Fluctuating

Evaluate for Treat Reversible Causes:


HPT axis ....................................... Lifestyle measures
pathology Optimize comorbidities
Stop offending drugs*
Decision to treat with testosterone
Pathology confirmed Discuss uncertainties of benefits vs. risks
Set patient-specific goals
No No paternity Reevaluate after 3-6 months of treatment *
such as opioids,
Contraindications desired Follow a standardized monitoring plan1 glucocorticoids

Treat Persistent symptoms/


Low Testosterone

measures. In some men, the low testosterone healthy young men) or placebo for 12 months.
may contribute to fatigue or poor motivation that Eligible participants had a baseline testosterone
reduces their ability to initiate healthy lifestyle concentration less than 275 ng/dL (<9.54 nmol/L)
measures. This raises the question as to whether (averaged from 2 measures) and at least 1
testosterone therapy should be considered in symptom or sign consistent with hypogonadism
selected circumstances, and what the benefits and (eg, decreased libido, difficulty walking, or low
risks of testosterone treatment are. Of note, in such vitality). Men with organic hypogonadism were
men, reevaluation for organic HPT axis pathology excluded from the study.9 Testosterone treatment
should be considered, as this can be missed. resulted in modest benefits in sexual function
and slightly improved mood and depressive
Possible Benefits of Testosterone Therapy symptoms, but it did not improve cognitive
The Testosterone Trials (T-Trials), the function. Concomitant substudies demonstrated
largest randomized controlled clinical trials improvements in volumetric bone mineral
of testosterone treatment in older men, have density and anemia.10 Other smaller randomized
provided new data on organ-specific outcomes for controlled trials of testosterone treatment have
testosterone treatment in older men. The T-Trials reported a 4.4-lb (2-kg) gain in muscle mass
were a coordinated set of trials (a main trial (with associated increases in muscle strength) and
and several substudies) conducted in the United 4.4-lb (2-kg) loss of fat mass, as well as modest
States, which included 790 men aged 65 years improvements in insulin resistance. However,
or older who were randomly assigned to receive in men with generally well-controlled diabetes,
transdermal testosterone (dosed to maintain no improvements in glycemic control have been
serum testosterone within the normal range for observed (Table 2).11

274  ENDO 2021 • Endocrine Case Management

chased by Dr. Khalid H. Seedahmed, Jr., MRCP PGDip, CertEndocrinology SA - ID 283


Table 2. Effects of Testosterone Treatment in Randomized Controlled Trials of Middle-Aged and Older Men: Evidence to Datea

Concern Testosterone Treatment Effect Targeted Treatment

Sexual Modest improvement in most domains of sexual function (if testosterone Phosphodiesterase
<275-346 ng/dL [<9.5-12.0 nmol/L]) 5 inhibitor (for erectile
dysfunction)
In most randomized controlled trials, effect on libido more marked than on erectile
function

Muscle 3.5-5.9 lb (1.6-2.7 kg) increase in mass, and, in some studies, increased muscle Exercise
strength; modest improvements in some measures of muscle performance (eg, small
increase in walking distance)

Fat 3.5-4.4 lb (1.6-2.0 kg) decrease Weight loss

Glucose Modest improvement in insulin resistance in some randomized controlled trialsb; no Lifestyle, antidiabetes
metabolism effect on hemoglobin A1c medications

Bone 2%-7% increase in lumbar spine bone mineral density, with generally smaller Osteoporotic drug therapy
increases in femoral bone mineral density; randomized controlled trials have not
been large enough to assess effects on fractures

Mood Slight improvement in mood/depressive symptoms eg, Counseling,


antidepressants

Cognition No effect on cognition, memory eg, Cholinesterase inhibitors

a
In these randomized controlled trials, men with organic hypogonadism were excluded.
b
In these randomized controlled trials, men had relatively well-controlled diabetes at baseline.

Whether testosterone treatment reduces treatment increases the risk of developing prostate
important clinical endpoints such as fractures, cancer. Prostate monitoring during testosterone
disability, or mortality in older men is not known. therapy may, however, lead to overdiagnosis of
This is because existing randomized controlled preexisting, clinically insignificant prostate cancer.
trials of testosterone therapy in older men have Guidance to minimize this risk is available.2
been relatively small and of short duration. Some The current evidence regarding testosterone
randomized controlled trials have included men who treatment and cardiovascular outcomes is
either had no clinical evidence of androgen deficiency contradictory and inconclusive, with some studies
or did not have confirmed low testosterone.2 suggesting increased risks, whereas most studies
do not.
Possible Risks of Testosterone Therapy If treatment is considered, men with functional
Trials to date have been underpowered to provide hypogonadism should be informed about the
definitive outcome data regarding prostate and absence of high-level evidence regarding long-
cardiovascular events, and long-term risks are term benefits and risks. Clear patient-specific
unknown. It has been estimated that a randomized goals should be identified, and treatment should
controlled trial enrolling several thousand men over be stopped if these goals are not achieved.
3 to 5 years would be needed to define true long- Monitoring for adverse events should follow
term risks. The largest randomized controlled trial consensus recommendations.2
to date enrolled 790 men over 12 months.9
Regarding prostate events, the data have
Conclusions
been reassuring. Despite the fact that androgen
deprivation therapy is an effective treatment Men with organic hypogonadism should be
for established prostate cancer, clinical studies identified and considered for testosterone
to date have not demonstrated that testosterone replacement. More research is needed to clarify

ENDO 2021 • Reproductive Endocrinology  275

chased by Dr. Khalid H. Seedahmed, Jr., MRCP PGDip, CertEndocrinology SA - ID 283


the role for testosterone therapy in other settings. Which of the following is the best next
In most older men, low testosterone is a marker of step in this patient’s management?
poor health and should prompt a holistic approach A. Evaluate for sleep obstructive sleep apnea
with focus on lifestyle measures and optimization B. Refer to a dietician to achieve a healthy
of comorbidities. Successful weight loss combined body weight
with optimization of comorbidities can be
C. Evaluate for organic HPT axis pathology
sufficient to improve symptoms and increase
testosterone concentrations and will lead to other D. Refer to a psychologist for counseling
important health benefits in such men. Answer: C) Evaluate for organic HPT axis pathology
This middle-aged man presents with clinical
Clinical Case Vignettes
findings suggestive of organic pathology
Case 1 (gynecomastia, borderline testicular size). His serum
A 51-year-old man consults you for a second testosterone is frankly and repeatedly low. There
opinion. He previously saw a medical practitioner is no intercurrent illness suggesting temporary
for androgen deficiency-like symptoms and a low HPT axis suppression. He has mild unexplained
testosterone measurement, which the practitioner anemia, which is compatible with androgen
attributed to his comorbid metabolic syndrome deficiency. His clinical presentation should not be
and being overweight. He was advised to engage in attributed to functional hypogonadism, a diagnosis
healthy lifestyle measures. With dietary changes, of exclusion. Further evaluation of his HPT axis is
he lost 8.8 lb (4 kg) without any symptomatic indicated (Answer C). Upon further investigation,
improvement. He has been unable to exercise, his LH concentration was 2.4 mIU/L (2.4 IU/L),
which he attributes to ongoing lethargy, muscle inappropriately low in the face of his repeatedly
weakness, and low mood. He reports poor libido. and frankly low serum testosterone. His prolactin
Work has been stressful, and he is concerned concentration was mildly elevated at 31 ng/mL
about losing his job. (1.3 nmol/L), suggestive of pituitary stalk effect
On physical examination, his BMI is rather than a prolactin-secreting pituitary. Pituitary
29.0 kg/m2, waist circumference is 41 in (104 cm), imaging demonstrated a 2-cm macroadenoma
and blood pressure is 152/90 mm Hg. He has mild well away from the optic chiasm, consistent with a
gynecomastia and 15-mL testes bilaterally. Visual nonfunctioning adenoma on further workup.
fields are normal to confrontation. Obstructive sleep apnea (Answer A) is not
uncommon in older, obese men with functional
Laboratory test results (sample drawn at 8 AM hypogonadism and it may contribute to fatigue.
while fasting): However, this is not the foremost consideration
Total testosterone = 133 ng/dL (300-900 ng/dL) in this middle-aged, overweight man with
(SI: 4.6 nmol/L [10.4-31.2 nmol/L]) (repeated mild anemia (untreated obstructive sleep apnea
measurement = 144 ng/dL [SI: 5.0 nmol/L]) may increase hemoglobin due to hypoxemia).
Hemoglobin = 12.7 g/dL (13.8-17.2 g/dL) Likewise, a weight-loss program (Answer B) or
(SI: 127 g/L [138-172 g/L]) counseling (Answer D) might be appropriate
Glucose = 125 mg/dL (70-99 mg/dL)
(SI: 6.9 mmol/L [3.9-5.5 mmol/L]) flanking measures, but they are not the best
Hemoglobin A1c = 6.6% (4.0%-5.6%) (49 mmol/mol next step. This man, following review by a
[20-38 mmol/mol]) neurosurgeon, opted for expectant management of
Liver function, normal his pituitary adenoma. Testosterone replacement
Renal function, normal was subsequently initiated after noting that he
Iron studies, normal
had no desire for paternity. He reported marked
improvement of energy levels and sexual function.

276  ENDO 2021 • Endocrine Case Management

chased by Dr. Khalid H. Seedahmed, Jr., MRCP PGDip, CertEndocrinology SA - ID 283


His motivation to engage in healthy lifestyle Answer: B) Initiate lifestyle measures to achieve weight
measures increased, and, over the next 12 months, loss and prescribe a phosphodiesterase 5 inhibitor
he successfully lost 13.2 lb (6 kg).
In this man, the clinical suspicion of organic
hypogonadotropic hypogonadism is low, and
Case 2 pituitary-directed MRI (Answer A) is not
A 67-year-old man presents with low energy, necessary. His obesity is most likely the major
low libido, and erectile dysfunction. He has well- contributor to his lowered testosterone, and
controlled hypertension treated with an ACE successful weight loss may improve his energy
inhibitor. He has no history of cardiovascular levels and increase circulating testosterone. His
disease or diabetes mellitus. He is a former erectile function should improve with the use
cigarette smoker (40 pack-year history). He reports of a phosphodiesterase 5 inhibitor. Therefore,
poor dietary habits and does not exercise. He has among the options given, initiating lifestyle
no symptoms of sleep apnea. measures to achieve weight loss and prescribing
On physical examination, his BMI is a phosphodiesterase 5 inhibitor (Answer B) is the
34.6 kg/m2 and blood pressure is 132/84 mm Hg. best next step.
He has normal male-pattern body hair, and testes Selective estrogen-receptor modulators
are 20 mL bilaterally. There is no gynecomastia or (Answer D) or aromatase inhibitors (Answer C)
visual field defect. such as anastrozole are sometimes prescribed
off-label by practitioners for the treatment of
Laboratory test results (sample drawn at 8 AM functional hypogonadism, with the expectation
while fasting): that by decreasing the estradiol-mediated
Total testosterone = 202 ng/dL (300-900 ng/dL) central negative feedback on the hypothalamic
(SI: 7.0 nmol/L [10.4-31.2 nmol/L]) (repeated pituitary unit, LH increases and subsequently
measurement = 210 ng/dL [SI: 7.3 nmol/L]) serum testosterone increases. These agents are
SHBG = 2.7 µg/mL (1.1-6.7 µg/mL) (SI: 24 nmol/L not approved for male hypogonadism, and there
[10-60 nmol/L])
Calculated free testosterone = is no convincing evidence for clinical benefit.
5.0 ng/dL (9.0-30.0 ng/dL) (SI: 0.17 nmol/L Moreover, by decreasing estradiol production,
[0.31-1.04 nmol/L]) aromatase inhibitors have been reported to reduce
LH = 4.3 mIU/mL (1.0-9.0 mIU/mL) (SI: 4.3 IU/L bone density in men.
[1.0-9.0 IU/L])
FSH = 3.7 mIU/mL (1.0-13.0 mIU/mL) (SI: 3.7 IU/L
[1.0-13.0 IU/L]) Case 2 (Continued)
Prolactin, normal
Thyroid function, normal Over the next 9 months, with diligent
Iron studies, normal implementation of dietary measures and
increased physical activity, the patient manages
to lose 24 lb (11 kg). He reports a modest
Which of the following is the best next improvement in sexual function with the use of
step in this patient’s management? a phosphodiesterase 5 inhibitor, but still rates
A. Perform pituitary-directed MRI his libido and energy levels as “lower than they
B. Initiate lifestyle measures to achieve weight should be.” His morning fasting total testosterone
loss and prescribe a phosphodiesterase 5 is now 256 ng/dL (8.9 nmol/L) and 245 ng/dL
inhibitor (8.5 nmol/L) on repeated measurement. Recent
C. Prescribe an aromatase inhibitor bone mineral density measurements by DXA
demonstrated T-scores of +0.5 at the lumbar
D. Prescribe a selective estrogen-receptor
spine, –0.3 at the total hip, and –0.1 at the femoral
modulator

ENDO 2021 • Reproductive Endocrinology  277

chased by Dr. Khalid H. Seedahmed, Jr., MRCP PGDip, CertEndocrinology SA - ID 283


neck. He now seeks your advice regarding powered for fracture outcomes. While more
testosterone therapy. testosterone treated men experienced an increase
in PSA of 1.0 ng/mL or greater (≥1.0 µg/L) (23
On the basis of current evidence from men compared with 8 men receiving placebo),
randomized controlled trials, which of only 1 man (assigned to testosterone) was
the following is the least likely effect of diagnosed with prostate cancer during the
starting testosterone in this patient? 12-month randomized controlled trial.
A. Increased hemoglobin
B. Increased PSA Case 3  New Content 
C. Reduced coronary artery plaque volume A 24-year-old avid cyclist presents with fatigue
D. Increased bone mineral density and declining race performance. Over the last
6 months, he stepped up his training program in
Answer: C) Reduced coronary artery plaque volume
preparation for an endurance racing event, cycling
The clinical characteristics of the patient in 4 to 6 hours each day. He admits to dieting, to
this vignette are similar to those of eligible “be as light as possible,” and reports a weight loss
T-Trial participants. Most men enrolled in the of 15.5 lb (7.0 kg) over the last 3 months. He notes
T-Trials had cardiovascular risk factors such as reduced libido. He underwent normal puberty. He
obesity (63%) and hypertension (72%), and men states he does not use prescribed or illicit drugs.
participating in the bone trial had normal bone On physical examination, his BMI is 20.5 kg/m2.
mineral density as quantified by DXA, with There is no clinical visual field defect, and he has
T-scores ranging from –0.3 to +1.3 at baseline. full range of eye movements. Muscle bulk appears
In the main trial, testosterone treatment normal, and testicular volume is 25 mL bilaterally.
improved all aspects of sexual function (libido and
Laboratory test results:
sexual activity more so than erectile function) and
improved mood and depressive symptoms to a Serum total testosterone (sample drawn at 8 AM
small degree. while fasting) = 86 ng/dL (300-900 ng/dL)
(SI: 3.0 nmol/L [10.4-31.2 nmol/L]); repeated
The cardiovascular trial reported a
measurement 1 week later = 75 ng/dL
greater increase in coronary plaque volume in (SI: 2.6 nmol/L)
testosterone-treated men than in placebo-treated SHBG = 9.3 µg/mL (1.1-6.7 µg/mL)
men, raising a possible concern over adverse (SI: 82.7 nmol/L [10-60 nmol/L])
cardiovascular effects of testosterone. While LH = 0.2 mIU/mL (1.0-9.0 mIU/mL)
(SI: 0.2 IU/L [1.0-9.0 IU/L])
there was no difference in cardiovascular events
FSH = 0.3 mIU/mL (1.0-13.0 mIU/mL)
between groups, the trial was not large enough or (SI: 0.3 IU/L [1.0-13.0 IU/L])
long enough to assess cardiovascular outcomes. Hemoglobin = 12.6 g/dL (13.8-17.2 g/dL)
Overall, considering the literature as a whole, (SI: 126 g/L [138-172 g/L])
the evidence regarding cardiovascular effects of Prolactin = 22 ng/mL (4-23 ng/mL)
(SI: 0.96 nmol/L [0.17-1.00 nmol/L])
testosterone in older men remains inconclusive.
Ferritin = 220 ng/mL (15-200 ng/mL)
In the anemia trial, testosterone increased (SI: 494.3 pmol/L [33.7-449.4 pmol/L])
hemoglobin and corrected mild to moderate Transferrin saturation = 35% (14%-50%)
anemia. In the bone trial, testosterone treatment TSH = 0.28 mIU/L (0.5-5.0 mIU/L)
substantially improved volumetric bone mineral Free T4 = 0.90 ng/dL (0.8-1.8 ng/dL)
(SI: 11.6 pmol/L [10.30-23.17 pmol/L])
density in the spine, increasing 6.8% more in
Free T3 = 1.9 pg/mL (2.3-4.2 pg/mL)
testosterone-treated patients than in placebo- (SI: 2.9 pmol/L [3.53-6.45 pmol/L])
treated patients. Testosterone also improved
estimated bone strength. The study was not

278  ENDO 2021 • Endocrine Case Management

chased by Dr. Khalid H. Seedahmed, Jr., MRCP PGDip, CertEndocrinology SA - ID 283


Question 1: Which of the following is These iron studies are not consistent with
this patient’s most likely diagnosis? hemochromatosis (Answer C), which can present
A. Relative energy deficit in sport as hypogonadotropic hypogonadism in men.
B. Pituitary adenoma
Question 2: Which of the following is the
C. Hemochromatosis best next step in this patient’s management?
D. Covert anabolic steroid use A. Initiate testosterone replacement
Answer: A) Relative energy deficit in sport B. Reduce training intensity and restore energy
deficit
The International Olympic Committee has
C. Initiate clomiphene
introduced the gender-neutral term relative energy
deficiency in sport (RED-S) (Answer A), replacing D. Initiate an aromatase inhibitor
the term “female athlete triad” to recognize that Answer: B) Reduce training intensity
functional hypogonadotropic hypogonadism due and restore energy deficit
to overtraining leading to a relative energy deficit
may also occur in men. Like women, men can Treatment of RED-S consists of correcting the
present with the full triad of (1) disordered eating underlying energy deficit by increasing caloric
(or low energy availability relative to expenditure), intake and reducing training intensity (Answer B).
(2) hypogonadism, and (3) decreased bone mineral If successful, this should lead to gonadal axis
density. RED-S can affect multiple psychobiologic recovery, usually within weeks to months. If not,
domains and is a diagnosis of exclusion. alternative diagnoses should be considered, such
The history of excessive exercise combined as a missed diagnosis of organic hypogonadism or
with dietary restriction and weight loss is typical covert anabolic steroid abuse.
for RED-S. BMI may not be frankly low, but it
is typically lower than expected for an athlete. In Case 4  New Content 
the context of hypogonadotropic hypogonadism,
biochemical findings pointing to the diagnosis of A 54-year-old man is referred from the liver unit
RED-S in this man are the elevated serum SHBG for management of insulin-requiring, transplant-
concentration (a sensitive marker of energy associated type 2 diabetes. His current hemoglobin
deficit), the sick euthyroid picture, and the mild A1c level is 8.2% (66 mmol/mol). He received
anemia consistent with androgen deficiency. a liver transplant 18 months ago for alcohol-
Anabolic steroid abuse (Answer D) is an important related cirrhosis. Current medications include
differential, but his history, physical findings, and mycophenolate mofetil, tacrolimus, basal bolus
laboratory results do not suggest the diagnosis. insulin, and a vitamin D supplement.
Men abusing anabolic steroids typically present Despite a well-functioning transplant,
with a muscular physique, small testes, truncal on review of systems he reports low energy,
acne, and relative polycythemia, while SHBG tends weakness, and low libido. He describes a history
to be low. of mumps orchitis as a teenager. He recalls normal
This man’s normal prolactin level and the puberty and has fathered 2 children.
absence of clinical signs of a pituitary mass effect On physical examination, his BMI is 32 kg/m2.
make a pituitary adenoma (Answer B) less likely, He has gynecomastia, reduced body hair and
although pituitary imaging should be considered in muscle bulk, and soft testes (10 mL right, 5 mL
this case, as RED-S is a diagnosis of exclusion. left). Visual fields are normal. Two months before
His transferrin saturation is normal, and his transplant, the following laboratory test results
his ferritin concentration is only marginally were documented:
elevated, possibly reflecting physical stress.

ENDO 2021 • Reproductive Endocrinology  279

chased by Dr. Khalid H. Seedahmed, Jr., MRCP PGDip, CertEndocrinology SA - ID 283


Total testosterone = 72 ng/dL (300-900 ng/dL) Question 2: Which of the following is
(SI: 2.5 nmol/L [10.4-31.2 nmol/L]) the best explanation for the discrepant
LH = 1.0 mIU/mL (1.0-9.0 mIU/mL) androgenic profile before transplant
(SI: 1.0 IU/L [1.0-9.0 IU/L])
FSH = 1.2 mIU/mL (1.0-13.0 mIU/mL) compared with after transplant?
(SI: 1.2 IU/L [1.0-13.0 IU/L]) A. Covert use of anabolic steroids before liver
transplant
These findings were attributed to his ill health at
the time and were not assessed further. B. Primary hypogonadism masked by ill health
before liver transplant
Question 1: Which of the following is the C. Testicular suppression due to
best next step in this patient’s care? immunosuppressive therapy
A. Refer to a dietician for weight loss D. Laboratory error
B. Adjust insulin to optimize diabetes control Answer: B) Primary hypogonadism masked
C. Measure serum testosterone and LH by ill health before liver transplant
D. Perform pituitary MRI
Severe functional hypogonadotropic
Answer: C) Measure serum testosterone and LH hypogonadism is not uncommon in any severe
illness. In men with decompensated cirrhosis, low
This man has findings suggestive of organic testosterone is common (occurring in 60%-90%
hypogonadism, and he requires biochemical of men), and gonadotropins are typically low
evaluation (Answer C). His androgenic profile or low-normal, as with any other severe illness.
shows the following: Low testosterone is an excellent marker of poor
Fasting morning total testosterone health in men with cirrhosis and predicts mortality
= 106 ng/dL (300-900 ng/dL) even independently of established prognostic
(SI: 3.7 nmol/L [10.4-31.2 nmol/L]) parameters, such as the model for end-stage liver
LH = 35.0 mIU/mL (1.0-9.0 mIU/mL) disease (MELD) score.
(SI: 35.0 IU/L [1.0-9.0 IU/L])
FSH = 42.0 mIU/mL (1.0-13.0 mIU/mL) However, the biochemical severity of his
(SI: 42.0 IU/L [1.0-13.0 IU/L]) pretransplant hypogonadotropic hypogonadism
was unusual (pretransplant testosterone
Repeated androgenic profile 1 week later concentrations in cirrhotic men are typically
documents similar results. around 145-260 ng/dL [5.0-9.0 nmol/L]).
Performing a pituitary MRI (Answer D) Unfortunately, a thorough assessment of his
would only be indicated if a repeated androgen gonadal status was not performed at that time.
profile had shown persistent hypogonadotropic In most men with functional hypogonadotropic
hypogonadism. Weight loss (Answer A) and hypogonadism before liver transplant, the gonadal
optimizing diabetes control (Answer B) are axis recovers after a successful transplant, although
important general health measures and may lead this may take several months.
to a modest increase in serum testosterone in men Interestingly, at evaluation now, 18 months
with functional hypogonadism. However, such after his successful transplant, his testosterone
measures will not normalize the gonadal axis in is still repeatedly very low, but gonadotropins
men with organic hypogonadism. are now clearly elevated. Moreover, he has
clinical features suggesting genuine organic
hypogonadism. The best explanation for
these findings is poor health before transplant
(Answer B). This most likely led to hypothalamic-
pituitary suppression causing functional central

280  ENDO 2021 • Endocrine Case Management

chased by Dr. Khalid H. Seedahmed, Jr., MRCP PGDip, CertEndocrinology SA - ID 283


hypogonadism superimposed on, and hence are not associated with primary hypogonadism,
masking, his underlying genuine primary and a laboratory error (Answer D) is unlikely to
hypogonadism due to previous alcohol and explain the clinical findings.
orchitis. This has now been unmasked as his He had no desire for paternity. Testosterone
general health has improved. replacement was commenced, which led to
There is no history to suggest anabolic steroid improvements in libido and energy, thus enabling
abuse (Answer A). Immunosuppressants such as him to lose weight and improve his glycemic
mycophenolate mofetil and tacrolimus (Answer C) control.

References
1. Basaria S. Testosterone therapy in older men with late-onset hypogonadism: 7. Goldman AL, Pope HG, Bhasin S. The health threat posed by the hidden
a counter-rationale. Endocr Pract. 2013;19(5):853-863. PMID: 24014017 epidemic of anabolic steroid use and body image disorders among young men.
2. Bhasin S, Brito JP, Cunningham GR, et al. Testosterone therapy in men J Clin Endocrinol Metab. 2019;104(4):1069-1074. PMID: 30239802
with hypogonadism: an Endocrine Society clinical practice guideline. J Clin 8. Tajar A, Forti G, O’Neill TW, et al; EMAS Group. Characteristics of
Endocrinol Metab. 2018;103(5):1715-1744. PMID: 29562364 secondary, primary, and compensated hypogonadism in aging men:
3. Snyder PJ, Peachey H, Berlin JA, et al. Effects of testosterone replacement evidence from the European Male Ageing Study. J Clin Endocrinol Metab.
in hypogonadal men. J Clin Endocrinol Metab. 2000;85(8):2670-2677. 2010;95(4):1810-1818. PMID: 20173018
PMID: 10946864 9. Snyder PJ, Bhasin S, Cunningham GR, et al; Testosterone Trials
4. Grossmann M, Matsumoto AM. A perspective on middle-aged and older Investigators. Effects of testosterone treatment in older men. N Engl J Med.
men with functional hypogonadism: focus on holistic management. J Clin 2016;374(7):611-624. PMID: 26886521
Endocrinol Metab. 2017;102(3):1067-1075. PMID: 28359097 10. Snyder PJ, Bhasin S, Cunningham GR, et al. Lessons from the testosterone
5. Wu FC, Tajar A, Beynon JM, et al; EMAS Group. Identification of trials. Endocrine Rev. 2018;39(3):369-386. PMID: 29522088
late-onset hypogonadism in middle-aged and elderly men. N Engl J Med. 11. Kaufman JM, Lapauw B, Mahmoud A, T’Sjoen G, Huhtaniemi IT. Aging
2010;363(2):123-135. PMID: 20554979 and the male reproductive system. Endocrine Rev. 2019;40(4):906-972.
6. Wong HK, Hoermann R, Grossmann M. Reversible male hypogonadotropic PMID: 30888401
hypogonadism due to energy deficit. Clin Endocrinol (Oxf). 2019;91(1):3-9.
PMID: 30903626

ENDO 2021 • Reproductive Endocrinology  281

chased by Dr. Khalid H. Seedahmed, Jr., MRCP PGDip, CertEndocrinology SA - ID 283


Evaluation of Female Fertility:
Antimullerian Hormone and
Ovarian Reserve Testing
Marcelle I. Cedars, MD. University of California, San Francisco, San Francisco, CA; E-mail:
marcelle.cedars@ucsf.edu

Learning Objectives ѓ Decreased ovarian reserve in young women


is not implicated in infertility and/or
As a result of participating in this session, learners
lowered fecundity.
should be able to:
ѓ Older women, especially those with
• Evaluate the infertile woman. diminished ovarian reserve, may have a
shortened timeline for conception and
ѓ Describe an approach to assessing
should be managed more aggressively.
reproductive aging.
They often require assisted reproductive
ѓ Describe available tests and their technology to conceive after a short course
implications. of medical therapy and/or intrauterine
insemination.
• Select treatments that optimize patient
physiology and reduce risk. • Women with polycystic ovary syndrome
• Summarize the likelihood of livebirths after (PCOS) are best treated with weight reduction;
treatment of infertility. if this fails to induce ovulation and/or the
women is older, ovulation induction with
letrozole is indicated.
• Women with hypogonadotropic
Main Conclusions hypogonadism should have the underlying
The elements of the management of the infertile cause corrected (hypothyroidism,
couple are the following: hyperprolactinemia, weight gain). For women
with functional hypothalamic amenorrhea,
• Importance of evaluation of the couple. ovulation induction with pulsatile GnRH is the
• Primacy of the evaluation and management of most effective and physiological.
the woman.
ѓ Young women who are ovulating will
generally conceive with a subfertile male— Significance of the
restoration of ovulation in young women
who are infertile will often result in
Clinical Problem
conception for a couple. Approximately 1 in 8 couples have trouble
conceiving or sustaining a pregnancy according
to the Centers for Disease Control 2006-2010
National Survey of Family Growth. In the United

282  ENDO 2021 • Endocrine Case Management

chased by Dr. Khalid H. Seedahmed, Jr., MRCP PGDip, CertEndocrinology SA - ID 283


States, infertility is typically defined as the • Press/public focus on high-tech treatments
inability to conceive after 1 year, at which time an such as in vitro fertilization (suggesting these
evaluation should begin. For women older than expensive treatments are the only appropriate
35 years, evaluation is advised after 6 months. care and/or are so powerful they can overcome
While the percentage of women with infertility all factors, including age).
and no living children has increased largely due to
the increasing age at the time of first pregnancy,
the overall percentage of women with infertility Strategies for Diagnosis,
has been relatively stable over several decades. Therapy, and/or Management
Although infertility is frequently considered a
“women’s disease,” about 35% of couples have both Initial Evaluation of the
a male and female etiology of their infertility with Infertile Couple
another 10% to 20% having a sole male factor as Couples should begin an evaluation if they have
the identifiable cause. not conceived after 1 year of active attempts. This
Normal fertility requires a competent oocyte assumes that the woman is having regular cycles
(requiring normal ovulatory function in the and that there have been 12 “ovulatory, exposed”
female); a functional sperm (requiring normal cycles. A woman who has had irregular cycles with
endocrinological function and spermatogenic long episodes of amenorrhea should present earlier
capacity in the male); an anatomic capacity for for evaluation because she is likely anovulatory.
sperm/egg interaction (functional cervix, uterus Women older than 35 years should be evaluated
and fallopian tubes in female and functional vasa, for infertility after 6 months of unsuccessful active
epididymis, and penis in the man); and the capacity attempts.
for implantation (endocrinological and anatomic Both members of an infertile couple should
uterine environment). Infertility or reduced be evaluated, and the couple should be queried as
fecundity (ability to produce a live born) can result to whether they are having frequent (2-3 times
from abnormalities in any of these areas. We are weekly) vaginal intercourse (“exposure”). A careful
aware that today, there are many ways to build a history should be taken for each member of the
family and many types of families. This session couple for causes of infertility such as anovulation
will focus on the role of ovarian reserve in chances in the woman or hypogonadism in the man.
for conception.

Approach to the Subfertile/


Barriers to Optimal Practice Infertile Woman
• Delay in attempts at pregnancy (increasing age The evaluation always begins with the woman, as
of the female partner). women with normal fecundity usually conceive
even with men who have subnormal reproductive
• Incomplete understanding of physiology of
function. The first approach to an infertile
normal female and male reproductive function.
woman is to characterize the menstrual pattern.
• Cost and lack of access to fertility care, A history of regular, cyclic, predictable periods
particularly due to lack of insurance or is associated with ovulation in greater than 98%
underinsurance. of cases. However, it is also important to further
• Perception that infertility is caused by the characterize even “regular” cycles. Important
patient or couple (eg, lifestyle choices such as questions include changes in cycle interval (a
delay in decision to conceive or exposure to shortening of the intermenstrual interval may
sexually transmitted infections). suggest a decline in ovarian reserve). New-onset
spotting prior to menstrual flow may suggest a

ENDO 2021 • Reproductive Endocrinology  283

chased by Dr. Khalid H. Seedahmed, Jr., MRCP PGDip, CertEndocrinology SA - ID 283


reduction in luteal phase progesterone, also a across (or between) menstrual cycles and not
symptom of ovarian aging. For women whose to be affected by contraception or anovulation.
cycles are irregular, and specifically for those Early studies suggested a correlation with natural
women with oligomenorrhea and amenorrhea, fecundability. However, 2 large studies have
a careful history is important. This should begin not shown a correlation between AMH and
with the onset of menses and the transition (if fecundability.1,2 Thus, it is clear that AMH is not a
any) to irregularity. Additional questions should “fertility marker.”
ascertain the presence or absence of symptoms AMH is a predictor of response to stimulation
of hypothyroidism, hyperprolactinemia, (in vitro fertilization cycle), but it does not
hypoestrogenism, and hyperandrogenism. correlate with pregnancy.3 This suggests AMH
These questions can guide the examination and is a marker of “quantity” but not “quality” of the
laboratory investigation. Additionally, the history oocytes. It is a marker of the residual oocyte pool.
should focus on factors that would interfere Additionally, there is intracycle variability of
with normal tubal function such as a history of AMH. And perhaps most important for assessment
sexually transmitted infections, pelvic surgery, of “ovarian reserve” in a general population, AMH
dysmenorrhea, or dyspareunia. is lowered with the use of oral contraceptive pills,
Physical examination of the woman should by as much as 50%, and thus may not be reflective
include assessment of vital signs (may point to of true ovarian reserve.4,5
thyroid or adrenal dysfunction), presence or The 2 most common causes of anovulation
absence of hyperandrogenism (hair growth, acne, include PCOS, which is characterized by 2
androgenic alopecia), thyromegaly, galactorrhea, of the following 3 findings: irregular cycles,
and evidence of past and current estrogen (breast hyperandrogenism and polycystic-appearing
development, vaginal and cervical lubrication). A ovaries, and hypogonadotropic hypogonadism
pelvic examination to feel the size and mobility (which can be genetic, functional, or due to
of the uterus and any adnexal pain/tenderness hypothyroidism or hyperprolactinemia). For
should be performed. For those with unexplained women with PCOS, screening should exclude
infertility (no risk factors) or symptoms of dyslipidemia, fatty liver, and/or impaired glucose
endometriosis, a recto-vaginal examination can tolerance/diabetes that may require further
reveal uterosacral nodularity or tenderness that assessment, treatment, and/or weight loss before
might further suggest this diagnosis. Laboratory conception.6,7 For patients who are metabolically
testing for the anovulatory woman should normal, ovulation induction is the first treatment
include measurement of TSH, prolactin, FSH, approach. Agents that can be used include
and estradiol. Additional testing of DHEA-S, clomiphene citrate, metformin, and letrozole.
testosterone, and LH may be warranted in Randomized controlled trials have identified
certain instances. All women should have an letrozole as most effective first-line treatment.8,9
evaluation of ovarian reserve by measurement Given intact feedback mechanisms in women with
of serum antimullerian hormone (AMH) and/or PCOS, only a short course of the antiestrogen
ultrasound-guided antral follicle count. letrozole is enough to “jump start” the cycle
If AMH is low, decreased ovarian reserve is by increasing FSH and allowing physiological
suspected. AMH is a glycoprotein in the TGF-β response of the ovaries for development of the
family. It is produced by the granulosa cells of dominant follicle. Due to intact feedback from
preantral and small antral follicles. As such, the hypothalamic-pituitary axis, most cycles are
it correlates with the residual ovarian pool of followed by ovulation of a single follicle, but a
primordial follicles. AMH was heralded as a small risk (about 5%-8%) of multiple pregnancy
better marker of ovarian reserve, compared with is possible. About 85% of women with PCOS
FSH and estradiol, as it was thought to not vary conceive with these drugs.

284  ENDO 2021 • Endocrine Case Management

chased by Dr. Khalid H. Seedahmed, Jr., MRCP PGDip, CertEndocrinology SA - ID 283


Interestingly, while preantral development While this patient is still young, the fact she is
has been thought to be unaffected by FSH, women asking the question and that she has Hashimoto
with hypogonadotropic hypogonadism may have disease with risk for other autoimmune disorders
significantly suppressed antral follicle count and (including premature ovarian insufficiency)
AMH.10 For women with hypogonadotropic warrants testing (Answer B). Conception
hypogonadism, the etiology of the low should never be suggested (Answer A) until a
gonadotropins should be identified.11 Then, patient is ready for family building. Egg freezing
the primary source should be treated, including (Answer C) may or may not be advised based on
thyroid replacement or reduction in prolactin with the patient’s ovarian reserve. Lastly, while age is
dopamine agonists as appropriate. For women with a strong predictor of successful conception, age
Kallmann syndrome or functional hypothalamic alone is more relevant as a poor prognosticator
amenorrhea, the ideal treatment is pulsatile in a woman older than 40 years than as a good
administration of GnRH.12,13 This agent is not prognosticator for a young woman (Answer E).
available in the United States, but it is under FDA
review. Unless pulsatile GnRH becomes available, Case 1 (Continued)
the standard treatment is the use of exogenous
gonadotropins. Gonadotropin therapy is effective, Her AMH concentration is 0.5 ng/mL (3.6 pmol/L).
but it is much more likely than GnRH therapy to
be complicated by a high risk of multiple pregnancy Which of the following is the best next step?
and ovarian hyperstimulation due to its direct action A. Suggest she stop oral contraceptive pills and
on the ovaries. Women with hypogonadotropic repeat the testing
hypogonadism seem to have a very narrow window B. Refer to reproductive endocrinology for egg
between under-response and over-response, making freezing
dosing quite difficult. Additionally, they typically C. Counsel her regarding risk for early
have a high number of responsive follicles and are at menopause
high risk for multiple pregnancy.
D. Advise her of low chances for conception
Answer: A) Suggest she stop oral contraceptive
Clinical Case Vignettes
pills and repeat the testing
Case 1
Oral contraceptive pills, especially when given in a
A 28-year-old woman is seen for management of continuous fashion, can suppress antral follicle growth
Hashimoto disease. She is concerned about future and AMH levels. Most recovery is seen in the first 2
fertility and asks about egg freezing. She has taken months, but levels can continue to rise for 6 months.
continuous oral contraceptive pills for the last 5 years. The next step would be to stop the oral contraceptive
pills and repeat testing in 2 to 3 months.
Which of the following is the
Again, egg freezing (Answer B) may or may
best course of action?
not be indicated based on the patient’s ovarian
A. Suggest she stop oral contraceptive pill and reserve and her social situation. While there is an
attempt conception association between AMH and menopausal onset,
B. Measure serum AMH this cannot currently be prospectively predicted
C. Refer to reproductive endocrinology for egg and this AMH level may be falsely suppressed by
freezing the oral contraceptive pills. Thus, counseling her
D. Reassure her that given her young age, future regarding risk for early menopause (Answer C)
fertility will not be a problem would be inappropriate. AMH levels do not
correlate with chances for spontaneous conception
Answer: B) Measure serum AMH when she starts trying to conceive (Answer D).

ENDO 2021 • Reproductive Endocrinology  285

chased by Dr. Khalid H. Seedahmed, Jr., MRCP PGDip, CertEndocrinology SA - ID 283


Case 2 with measurement of serum testosterone and
gonadotropins and semen analysis.
A couple presents for evaluation. The 28-year-
old woman stopped oral contraceptive pills 6
months prior to consultation. Her cycle interval Case 2 (Continued)
seems shorter than she recalls before starting oral Her AMH concentration is 0.5 ng/mL
contraceptive pills 10 years ago. Her cycles are (3.6 pmol/L). Her antral follicle count is 5.
currently every 25 to 26 days. Additionally, she has
noticed 3 days of precycle spotting. Which of the following is the best
On physical examination of the woman, course of action for this couple?
her BMI is 24 kg/m2. She has no thyromegaly. A. Advise them to keep trying
Skin is normal. Pelvic examination documents
B. Recommend controlled ovarian stimulation
estrogenized mucus and no masses or tenderness.
Laboratory testing documents normal levels of C. Recommend in vitro fertilization
TSH and prolactin. D. Advise to freeze eggs

Which of the following is the best course Answer: B) Recommend controlled ovarian stimulation
of management for this couple? While any of these options may be reasonable
A. Advise them to keep trying to conceive and based on the couple’s desire for family
increase intercourse frequency to 4 times building (number and spacing of pregnancies),
weekly starting controlled ovarian stimulation
B. Measure serum AMH in the woman (Answer B) (typically with clomiphene citrate)
C. Measure serum AMH in the woman and and intrauterine insemination, to shorten time to
perform semen analysis in her husband pregnancy, may be warranted. Given her young
age and the apparent slower decline in oocyte
D. Start a trial of letrozole for the woman
loss with lower egg numbers, her management
Answer: B) Measure serum AMH in the woman may not need to change. She likely would be able
to conceive both now and with a subsequent
Although this couple does not meet the United pregnancy. She may want to freeze eggs (embryo)
States definition of infertility (no conception after (Answer D) for a subsequent pregnancy, but
1 year of trying), the woman may have diminished the greatest likelihood is that this would not be
ovarian reserve given the shortened intermenstrual necessary. As AMH does not predict fecundability,
interval and precycle spotting. Assessing ovarian it is not clear that in vitro fertilization
reserve may indicate a shortened reproductive (Answer C) as a treatment (vs for protection
window while not impacting the chance for of future pregnancy) would be warranted. An
spontaneous conception at the current time. Based advisement to continue trying (Answer A) may
on family planning goals, a shortened reproductive be reasonable, but most likely some low-dosage
window may be relevant to the couple. medication (eg, clomiphene citrate) is indicated
Some experts would evaluate this man with to stop the precycle spotting and likely lowered
a semen analysis, but healthy young men are progesterone production.
more than 90% likely to have a sperm in the
ejaculate. Thus, the focus should be evaluation
and treatment of menstrual changes and not Case 3
assessment of the man. All men with a risk A couple presents for evaluation. The 32-year-
factor for infertility, symptoms or signs of old woman has had oligomenorrhea (fewer than
hypogonadism, or small testes should be assessed 6 episodes of menstrual bleeding per year) since
puberty. She has mild hirsutism and no history

286  ENDO 2021 • Endocrine Case Management

chased by Dr. Khalid H. Seedahmed, Jr., MRCP PGDip, CertEndocrinology SA - ID 283


of conception. Her 34-year-old husband has no all, women with PCOS, AMH would be elevated
medical illnesses and no history of fathering a (most likely above 5 ng/mL [>35.7 pmol/L]). This
child. They have not been able to conceive after 9 value would also suggest which women would
months of trying to conceive (vaginal intercourse be at highest risk for multifollicular development
twice weekly without contraception). and risk for multiple pregnancy, suggesting the
On physical examination of the woman, need for ultrasonography monitoring of ovulation
her BMI is 28 kg/m2. She has no thyromegaly. induction.
She has mild hirsutism on the chin and chest Healthy young men are more than 90% likely
and acne on her face, chest, and back. Her waist to have a sperm in the ejaculate. Thus, the focus
circumference is 39 in (99 cm) and she has hair up should be treatment of her chronic anovulation
to the umbilicus. Pelvic examination documents and not assessment of the man. All men with a
estrogenized mucus and no masses or tenderness. risk factor for infertility, symptoms or signs of
Laboratory testing documents normal levels of hypogonadism, or small testes should be assessed
TSH and prolactin. with measurement of serum testosterone and
gonadotropins and semen analysis.
Which of the following is the best course
of management for this couple?
Case 4
A. Advise them to keep trying to conceive and
increase intercourse frequency to 4 times A couple presents for evaluation after 5 months
weekly of infertility. The 35-year-old woman has not
had a menstrual cycle since she stopped oral
B. Measure serum AMH in the woman
contraceptive pills 5 months ago. Her 31-year-
C. Measure serum AMH in the woman and old partner is healthy and has no significant
perform semen analysis in her husband medical history and no risk factors for infertility.
D. Start a trial of letrozole for the woman The woman has always been thin and was a
competitive athlete in her teens and twenties. She
Answer: D) Start a trial of letrozole for the woman
had regular menstrual cycles after puberty but
Although this couple does not meet the United became amenorrheic in high school. She has been
States definition of infertility (no conception on oral contraceptive pills since that time.
after 1 year of trying), the woman is very likely On physical examination of the woman, her
anovulatory. Her history and examination pulse rate is 54 beats/min and BMI is 18 kg/m2.
findings indicate that she has PCOS, and a She has a normal thyroid gland. She has no
history of fewer than 6 menstrual cycles per year hirsutism or acne. On pelvic examination, she
suggests that most, if not all, of her periods are has pale, atrophic vaginal mucosa and no cervical
anovulatory. Given the chronic oligoanovulation, mucus. Bimanual examination does not detect any
additional attempts at “trying” (Answer A) will pelvic masses or tenderness. The uterus is small.
have a very low likelihood of success. Measuring
Laboratory test results:
AMH (Answers B and C) will not add to the
likely diagnosis of PCOS with anovulation and Prolactin, normal
hyperandrogenism. Ovulation induction with the TSH, normal
FSH = 1.5 mIU/mL (SI: 1.5 IU/L)
oral agent letrozole (Answer D) is most effective Estradiol (ultrasensitive assay) = 10 pg/mL
and the correct answer. (SI: 36.7 pmol/L)
However, measurement of AMH has been AMH = 0.01 ng/mL (SI: 0.07 pmol/L)
suggested as helpful in assessing response to
ovulation induction, so some would suggest
measurement. In this patient, as in most, if not

ENDO 2021 • Reproductive Endocrinology  287

chased by Dr. Khalid H. Seedahmed, Jr., MRCP PGDip, CertEndocrinology SA - ID 283


Which of the following is the missing hormone: GnRH (Answer A). The patient
most likely diagnosis? would not be expected to respond to aromatase
A. Primary ovarian insufficiency inhibitors (Answer B) or clomiphene since her
B. PCOS serum estradiol is already low. FSH and hCG can
be given (Answer C), but the risk for ovarian
C. Hypogonadotropic hypogonadism
hyperstimulation and multiple pregnancy is high
D. Postpill amenorrhea in a young, likely otherwise fertile woman, so
Answer: C) Hypogonadotropic hypogonadism careful management is required by an experienced
provider. Ovarian biopsy would not be indicated,
While this patient’s AMH level is quite low, and in vitro fertilization (Answer D) is reserved for
perhaps suggesting primary ovarian insufficiency circumstances when single follicular development
(Answer A), the FSH level is also low in the is not achieved with exogenous gonadotropin and
face of a low estradiol. This combination of pulsatile GnRH therapy is not available.
findings suggests a central cause of amenorrhea
(Answer C). While oral contraceptive pills Case 5  New Content 
(Answer D) may suppress endogenous FSH
and hence ovarian estradiol, this suppression A same sex-couple presents for fertility. The
would not persist for 5 months after stopping 33-year-old partner is interested in conception
the pills and to this extent. Additionally, after 5 and has been using home inseminations for 6
months off the oral contraceptive pills, recovery months. However, this is becoming increasingly
of ovarian suppression (low AMH) would be difficult, as her cycles are irregular and it is hard to
expected. Patients with PCOS (Answer B) are not detect her ovulatory surge with urinary kits. She is
hypoestrogenic, and AMH would be expected to interested in proceeding to in vitro fertilization for
be high due to increased ovarian reserve. conception.
On physical examination of the woman, her
pulse rate is 65 beats/min and BMI is 20 kg/m2.
Case 4 (Continued) She has a normal thyroid gland. She has no
hirsutism or acne. On pelvic examination, she
For this woman, which of the has pale, atrophic vaginal mucosa and no cervical
following would be the best treatment mucus. Bimanual examination does not detect any
to improve her fertility? pelvic masses or tenderness. The uterus is small.
A. Pulsatile GnRH therapy
Laboratory test results:
B. Letrozole
Prolactin, normal
C. Recombinant FSH and hCG
TSH, normal
D. Referral to an assisted reproductive technology FSH = 15 mIU/mL (SI: 15 IU/L)
expert for assessment through an ovarian Estradiol (ultrasensitive assay) = 10 pg/mL
biopsy and in vitro fertilization (SI: 36.7 pmol/L)
AMH = 0.01 ng/mL (SI: 0.07 pmol/L)
Answer: A) Pulsatile GnRH therapy
Based on her history and laboratory results, this Which of the following is the
patient has hypogonadotropic hypogonadism. This most likely diagnosis?
is most likely functional in nature. She should be A. Primary ovarian insufficiency
queried about her diet and exercise patterns, and B. PCOS
she should be counseled that a slightly higher body
C. Hypogonadotropic hypogonadism
weight might be useful for ovulation. However,
the appropriate medical treatment is to replace the D. Diminished ovarian reserve

288  ENDO 2021 • Endocrine Case Management

chased by Dr. Khalid H. Seedahmed, Jr., MRCP PGDip, CertEndocrinology SA - ID 283


Answer: D) Diminished ovarian reserve Considering her young age and limited
exposure to sperm, a trial of clomiphene citrate
The line between diminished ovarian reserve (Answer A) may be reasonable to normalize her
(Answer D) and primary ovarian insufficiency cycles (ovulation) and potentially qualitatively
(Answer A) is not always clear, but typically, improve her cycle. Clomiphene citrate would
primary ovarian insufficiency implies amenorrhea. increase endogenous FSH, potentially increasing
This patient clearly has diminished ovarian the chances for recruitment up to her maximal
reserve with low AMH and elevated (although capacity (2-3 follicles), as well as increase luteal-
not menopausal) FSH. Given the elevated FSH phase progesterone production. At her young age,
value with low estradiol (hypothalamic/pituitary oocyte “quality” (genetic normality) is expected to
function intact), the patient does not have be high and thus conception is still possible with
hypogonadotropic hypogonadism (Answer C). clomiphene citrate and donor insemination.
The pattern of low AMH, low estradiol, and Referral to a reproductive endocrinologist for
elevated FSH does not suggest PCOS (Answer B). further evaluation is not unreasonable, but neither
ovarian biopsy nor in vitro fertilization is advised
Case 5 (Continued)  New Content  at this point (Answer D). Minimal stimulation
For this woman, which of the following and banking of oocyte/embryos may be considered
would be the most successful next while she is still cycling if she is interested in this
step to improve her fertility? option to preserve future fertility, but this would not
enhance success for a current pregnancy, as in vitro
A. Prescribe clomiphene citrate
fertilization would not “fix” her limited reserve.
B. Evaluate her partner’s ovarian reserve
C. Prescribe recombinant FSH and hCG
Case 6  New Content 
D. Refer to an assisted reproductive technology
expert for assessment with ovarian biopsy A couple presents for infertility evaluation after
followed by in vitro fertilization 1 year of unprotected intercourse. The 32-year-
old woman has regular, cyclic, predictable cycles
Answer: B) Evaluate her partner’s ovarian reserve of 26 to 28 days. She has no precycle spotting. Her
medical history and surgical history are negative,
This patient has severely diminished ovarian and her 33-year-old husband has no notable
reserve. She reports her mother went through medical history.
menopause at age 35 years. It may be reasonable to On physical examination of the woman,
assess her ovarian reserve further with an antral her BMI is 22 kg/m2. She has no thyromegaly.
follicle count, which in this case was 2. Given this, Skin is normal. Pelvic examination documents
there is no reason to use exogenous gonadotropins estrogenized mucus and no masses or tenderness.
(Answer C) because her capacity is limited and
this treatment would be quite expensive without Laboratory test results:
significant benefit.
TSH, normal
Given the same-sex relationship, her female Prolactin, normal
partner may have a higher ovarian reserve, and AMH = 0.9 ng/mL
a “shared conception” (one partner is the gamete FSH = 9.5 mIU/mL (9.5 IU/L)
provider and the partner interested in pregnancy Estradiol = 65 pg/mL (SI: 238.6 pmol/L)
is the gestational carrier) will most likely offer the
Her fallopian tubes are patent on
highest chance of success (Answer B). In fact, some
hysterosalpingogram. Her husband’s semen
couples prospectively choose this as the preferred
analysis is normal.
conception method.

ENDO 2021 • Reproductive Endocrinology  289

chased by Dr. Khalid H. Seedahmed, Jr., MRCP PGDip, CertEndocrinology SA - ID 283


Which of the following is the best course “unexplained infertility.” In the United States,
of management for this couple? treatment may begin after 12 months, while in
A. Advise them to keep trying to conceive and Europe, recommendations are typically to wait a
increase intercourse frequency to 4 times minimum of 2 years. Thus, in some circumstances,
weekly advising them to keeping trying (Answer A) might
B. Start clomiphene citrate and proceed with be recommended. However, given this patient’s
intrauterine insemination low ovarian reserve, pursuing an intervention at
this time is most appropriate. The initial treatment
C. Start exogenous gonadotropins and proceed
of unexplained infertility is ovarian stimulation
with intrauterine insemination
with clomiphene citrate and intrauterine
D. Proceed with in vitro fertilization insemination (Answer B).
Answer: B) Start clomiphene citrate and One would not treat this patient with
proceed with intrauterine insemination exogenous gonadotropins (Answer C), especially
because of her young age, due to the risk for
This couple meets the definition of infertility in multiple pregnancy. There is no indication to
the United States (no conception after 1 year of move directly to in vitro fertilization (Answer D).
trying). Her ovarian reserve is low for her age, Even though her ovarian reserve is likely low
but the rest of the infertility evaluation is normal. (and an antral follicle count might be helpful),
The low AMH level may indicate a shortened her chances for pregnancy at this time are not
reproductive window, but it does not affect her impacted by the low reserve. As she is 32 years
chance for conception at the current time. Given old, she would likely be able to have a subsequent
her young age, she should be treated as having pregnancy even after she delivers her first child.

References
1. Steiner AZ, Pritchard D, Stanczyk FZ, et al. Association between biomarkers 8. Legro RS, Barnhart HX, Schlaff WD, et al; Cooperative Multicenter
of ovarian reserve and infertility among older women of reproductive age. Reproductive Medicine Network. Clomiphene, metformin, or both for
JAMA. 2017;318(14):1367-1376. PMID: 29049585 infertility in the polycystic ovary syndrome. N Engl J Med. 2007;356(6):551-
2. Zarek SM, Mitchell EM, Sjaarda LA, et al. Is anti-müllerian hormone 566. PMID: 17287476
associated with fecundability? Findings from the EAGeR trial. J Clin 9. Legro RS, Brzyski RG, Diamond MP, et al; NICHD Reproductive Medicine
Endocrinol Metab. 2015;100(11):4215-4521. PMID: 26406293 Network. Letrozole versus clomiphene for infertility in the polycystic ovary
3. Depmann M, Broer SL, Eijkemans MJC, et al. Anti-Müllerian hormone does syndrome [published correction appears in N Engl J Med. 2014;317(15):1465].
not predict time to pregnancy: results of a prospective cohort study. Gynecol N Engl J Med. 2014;371(2):119-129. PMDI: 25006718
Endocrinol. 2017;33(8):644-648. PMID: 28393651 10. Tran ND, Cedars MI, Rosen MP. The role of anti-müllerian hormone (AMH)
4. Birch Petersen K, Hvidman HW, Forman JL, et al. Ovarian reserve in assessing ovarian reserve. J Clin Endocrinol Metab. 2011;96(12):3609-3614.
assessment in users of oral contraception seeking fertility advise on their PMID: 21937624
reproductive lifespan. Hum Reprod. 2015;30(10):2364-2375. PMID: 26311148 11. Gordon CM, Ackerman KE, Berga SL, et al. Functional Hypothalamic
5. Letourneau JM, Cakmak H, Quinn M, et al. Long-term hormonal Amenorrhea: An Endocrine Society Clinical Practice Guideline. J Clin
contraceptive use is associated with a reversible suppression of antral follicle Endocrinol Metab. 2017;102(5):1413-1439. PMID: 28368518
count and a break from hormonal contraception may improve oocyte yield. J 12. Dumont A, Dewailly D, Plouvier P, Catteau-Jonard S, Robin G. Comparison
Assist Reprod Genet. 2017;34(9):1137-1144. PMID: 28669055 between pulsatile GnRH therapy and gonadotropins for ovulation
6. Goodman NF, Cobin RH, Futterweit W, et al; American Association of induction in women with both functional hypothalamic amenorrhea and
Clinical Endocrinologists (AACE); American College of Endocrinology polycystic ovarian morphology. Gynecol Endocrinol. 2016;32(12):999-1004.
(ACE); Androgen Excess and PCOS Society (AES). American Association of PMID: 27258574
Clinical Endocrinologists, American College of Endocrinology, and Androgen 13. Tranoulis A, Laios A, Pampanos A, Yannoukakos D, Loutradis D, Michala
Excess and PCOS Society disease state clinical review: guide to best practices L. Efficacy and safety of pulsatile gonadotropin-releasing hormone therapy
in the evaluation and treatment of polycystic ovary syndrome--Part 1. Endocr among patients with idiopathic and functional hypothalamic amenorrhea:
Pract. 2015;21(11):1291-1300. PMID: 26509855 a systematic review of the literature and a meta-analysis. Fertil Steril.
7. Goodman NF, Cobin RH, Futterweit W, et al; American Association of 2018;109(4):708-719.e8. PMID: 29605411
Clinical Endocrinologists (AACE); American College of Endocrinology
(ACE); Androgen Excess and PCOS Society. American Association of Clinical
Endocrinologists, American College of Endocrinology, and Androgen Excess
and PCOS Society Disease State Clinical Review: guide to the best practices
in the evaluation and treatment of polycystic ovary syndrome - Part 2. Endocr
Pract. 2015;21(12):1415-1426. PMID: 26642102

290  ENDO 2021 • Endocrine Case Management

chased by Dr. Khalid H. Seedahmed, Jr., MRCP PGDip, CertEndocrinology SA - ID 283


Management of Hormone
Replacement Therapy in Post–
Reproductive-Aged Women
Genevieve Neal-Perry, MD, PhD. Reproductive Endocrinology, Department of Obstetrics
and Gynecology, University of Washington School of Medicine, Seattle, WA; E-mail:
nealperr@uw.edu

Learning Objectives Significance of the


As a result of participating in this session, learners Clinical Problem
should be able to: Reduced and variable circulating levels of
• Explain the basic physiology of menopause and estradiol during the menopausal transition and
modifiers of symptomology. early menopause are associated with vasomotor
symptoms, disrupted sleep patterns, and increased
• Manage common symptoms of the morbidity. The population at risk for menopausal
menopausal transition and describe the risks symptoms numbers in the hundreds of millions
associated with treatment. and varies by race, ethnicity, and socioeconomics.
HT is currently the most effective treatment for
these life-disrupting symptoms. However, study
outcomes from the Women’s Health Initiative
Main Conclusions have affected appropriate use of HT.
Female reproductive senescence reflects the gradual
loss of oocytes as the reproductive axis matures in
Barriers to Optimal Practice
a continuous, natural process from menarche to
menopause. It reflects the loss of the finite numbers • Fears of patients regarding the appropriate use
of oocytes produced during fetal development. of HT.
Menopause is defined as the permanent cessation of
menses; by convention, the diagnosis of menopause • Misinterpretation of clinical studies regarding
is not made until the individual has had 12 months HT.
of amenorrhea. Menopause is thus characterized by
the loss of menses. However, hormonal changes,
rather than the cessation of menstruation itself, Strategies for Diagnosis,
often give rise to bothersome vasomotor symptoms Therapy, and/or Management
that disrupt the quality of life of affected women. Menopause is characterized by the permanent
When used in the appropriate patient population, cessation of menses and follicular depletion
hormone replacement therapy (HT) is highly of the ovaries and defined retrospectively by
effective and safe. the absence of menses for a minimum of 12
months. Menopause typically occurs in the
fifth to sixth decades of life, with a median

ENDO 2021 • Reproductive Endocrinology  291

chased by Dr. Khalid H. Seedahmed, Jr., MRCP PGDip, CertEndocrinology SA - ID 283


age of 52.5 years. However, this definition period. There are racial and ethnic differences
relies upon a woman having menstrual cycles in menopausal symptoms and multiple other
of some degree of regularity. Menopause and factors that may influence the timing of the final
the final menstrual period are preceded by the menstrual period for any given woman (see Table).
menopausal transition, a window of time that is The menopausal transition is heralded by
characterized by several years of menstrual cycle ovarian follicular pool depletion (ovarian reserve),
instability and erratic hormone secretion. The menstrual cycle length changes, reduced luteal
average woman takes about 4 years to make the phase progesterone synthesis, variable estradiol
transition into menopause. However, there is a levels, and elevated FSH.2 The associated hormonal
wide variation about this median.1 Women who fluctuations may trigger debilitating symptoms in
enter the transition at a younger age tend to have the early menopausal transition, symptoms that
more symptoms and a longer duration, whereas may worsen and persist throughout the early
those who enter it later in life are more likely to postmenopausal period.
have a more rapid transit to the final menstrual

Table. Factors That Influence the Age at Menopause

Factor Direction Degree

Genetic/familial

 MSH6 Earlier menopause NA

 MCM8 Earlier menopause NA

  PvuII estrogen receptor (ER) polymorphism Earlier menopause, hysterectomy 6 months

  Turner syndrome mosaic Earlier menopause Variable

  Fragile X Earlier menopause Variable

Environmental

  Endocrine-disrupting chemicals Earlier menopause 1.8-3.8 years

  High altitude Earlier menopause 1-1.5 years

Lifestyle

  Cigarette smoking Earlier menopause 1-2 years

  Low socioeconomic status Earlier menopause 1-2 years

  Oral contraceptive use Later menopause 6 months

Race/ethnicity

  African American Earlier menopause* 2 years

 Hispanic Earlier menopause 2 years

 Asian Later menopause 1-2 years

Menstrual characteristics

  Shorter cycles (<26 days) Earlier menopause 1.4 years

*
Relative to Caucasian women.

292  ENDO 2021 • Endocrine Case Management

chased by Dr. Khalid H. Seedahmed, Jr., MRCP PGDip, CertEndocrinology SA - ID 283


Diagnosis of Menopause symptoms, but are highly prevalent and disruptive
of the quality of life. Affected women often
Laboratory tests are typically not indicated if
report vaginal dryness, chronic irritation, or
history and physical examination rule out other
burning of both the vagina and vulva; decreased
potential causes of amenorrhea and bothersome
lubrication; pain during sex; or bleeding after
symptoms.
sex. Urinary frequency, urethritis, and frequent
Measurement of FSH and estradiol or
urinary tract infections may also occur. Symptoms
antimullerian hormone may be indicated for
that women experience are believed to reflect
women who:
hypogonadism. Genitourinary syndrome of
• have had a hysterectomy without menopause rather than atrophic vaginitis is
oophorectomy a term used to describe vaginal dryness apart
• are younger than 40 years from intercourse, dyspareunia, and associated
urinary symptoms.5 Unlike vasomotor symptoms,
• are using estrogen-containing contraceptives symptoms of genitourinary syndrome of
(suppress FSH and elevate estradiol) menopause do not spontaneously resolve over
• continue to have menses after age 55 years time. Therefore, long-term treatment may be
(may be postmenopausal but with bleeding required to preserve quality of life. Ospemifene,
endometrial neoplasm or other uterine a selective estrogen receptor modulator with
pathology) estrogen receptor β-agonist properties, is
available; however, it is administered systemically.
To treat dyspareunia, the FDA recently approved
Menopausal Symptoms vaginal dehydroepiandrosterone. Additionally,
Menopausal symptoms are often composed several studies have raised the possibility that laser
of 4 cardinal symptoms that directly relate to treatment of the vagina may provide relief from
menopause. These include vasomotor symptoms/ genitourinary syndrome of menopause symptoms,
hot flashes, vaginal dryness, sleep, and mood/ although there is insufficient information to
cognition. recommend this approach at this time. Adverse
Vasomotor symptoms affect up to 80% of mood is associated with traversal of menopause in
women. Thermoregulatory dysfunction begins a complex manner.
in hypothalamus in response to hormonal Depressive symptoms are associated with
fluctuations. The threshold in core temperature night-time hot flashes and sleep disruption,
for cooling (flushes, sweats) is lowered, and the suggesting an intimate relationship between
threshold for warming (shivering) is raised. KNDy sleep and mood. Cohort studies have identified
neurons (kisspeptin/neurokinin B/dynorphin), the late transition as a period of vulnerability
located adjacent to the thermoregulatory center, for depression in midlife women. Women are
are hypothesized to be the primary driver of 2- to 4-fold more likely to experience new-
vasomotor symptoms.3 Vasomotor symptoms have onset major depression during the menopausal
a median duration of 7.4 years, with a mean post- transition than premenopausal women or after
final menstrual period persistence of 4.5 years.4 their final menstrual period.6 Neither major
However, further clinical history can help predict depression nor depressive symptoms are associated
the duration of vasomotor symptoms with more with absolute levels of hormones. However,
precision. Shared decision making about treatment concomitant anxiety and hot flashes predict the
type and duration can be facilitated by taking these risk for major depression during this time of life.
additional factors into account. Heavier flow (menorrhagia), irregular cycle length
Vaginal dryness, dyspareunia, and urogenital (metrorrhagia), and intermenstrual bleeding can
symptoms are less common than vasomotor reflect high estrogen relative to low progesterone

ENDO 2021 • Reproductive Endocrinology  293

chased by Dr. Khalid H. Seedahmed, Jr., MRCP PGDip, CertEndocrinology SA - ID 283


associated with anovulation. This hormonal lowest possible dosage, and for the shortest
imbalance can cause endometrial overgrowth, duration required to ease symptoms. Tissue-
precancer (endometrial hyperplasia), or cancer. targeted therapies should be used whenever
Postmenopausal bleeding or perimenopausal possible to decrease overall systemic dose
bleeding at intervals less than 21 days warrants (eg, vaginal estrogen for vaginal symptoms).
evaluation. Transdermal preparations may carry lower risk of
Sleep disturbances are common in women, but thromboembolic events while providing the same
also worsen with age. Thus, it may be challenging efficacy for treatment management. Progestogen is
to distinguish between age-related sleep changes indicated for women with a uterus, but the dosage
and poor sleep due to menopause. While some should be carefully considered. All women taking
epidemiologic studies suggest the menopausal HT should be evaluated annually.
transition contributes to sleep dysfunction in
advanced reproductive-aged women, others
Risks and Adverse Effects of HT
do not, except for a small, vulnerable subset of
women with relatively poor sleep at baseline. Risks and adverse effects of HT are modulated
Vasomotor symptoms can mediate sleep by the route of administration, hormonal
disruption in perimenopausal and recently formulation, patient age, time since the final
menopausal women, in part by prompting night- menstrual period, the presence or absence of
time awakenings. HT may help sleep dysfunction a uterus, and coexisting morbidities such as
exacerbated by vasomotor symptoms. cardiovascular disease, dementia, and diabetes.7

Uterine Neoplasia
Treatment of Bothersome The incidence of uterine neoplasia is increased
Menopausal Symptoms under conditions of chronic unopposed estrogen
Hormonal management includes estradiol, exposure in a woman with an intact uterus. Thus,
estrogens, and progestin when a uterus is present. the primary indication of progestin therapy
When selecting a menopausal HT regimen, during menopause is to prevent estradiol-induced
the following should be considered: (1) the endometrial hyperplasia and cancer. Commonly
presence of a uterus, (2) the type of menopausal used progestins include medroxyprogesterone
symptoms for which treatment is desired, and (3) acetate, norethindrone acetate, and micronized
comorbid medical conditions that might influence progestin (native progesterone). Progestin
management options. administered continuously or sequentially provides
Estrogen therapy given to women younger adequate endometrial protection. However,
than 60 years without a uterus carries minimal controversy exits as to whether micronized
risk. Estrogen and progestogen therapy should progestin protects the uterus as effectively as
be given to women with a uterus, but it carries synthetic progestin.
an increased breast cancer risk for women of all
ages. Estrogen and progestogen risks may be dose Breast Cancer
dependent and may vary by progestogen type. Breast cancer risk and menopausal HT are
Progestin intrauterine devices may also be used; reported in randomized controlled trials
however, their impact on breast cancer risk has and observational studies. Different types of
not been determined. Progesterone alone can menopausal HT formulations, the duration of
decrease hot flashes, but it is rarely used due to therapy, and the time at which menopausal HT is
breast cancer concerns. initiated may modify the risk for breast cancer.8
The FDA recommends that HT be used
in women with bothersome symptoms, at the

294  ENDO 2021 • Endocrine Case Management

chased by Dr. Khalid H. Seedahmed, Jr., MRCP PGDip, CertEndocrinology SA - ID 283


Venous Thromboembolism and heart disease. Observational studies and
Venous thromboembolism and stroke risk randomized controlled trials suggest the risk for
increases with advancing age and a diagnosis morbidity is increased with advanced age and
of thrombophilia. The risk is related to oral time since the onset of menopause. Additionally,
administration and estradiol dosage.9,10 Venous the risk and benefit of HT vary by organ system,
thromboembolism events are more likely to HT formulation, and the time since the final
occur within the first year of HT, and the menstrual period. Observational studies, as well
risk is magnified with coexisting risk factors as randomized controlled trials, also suggest that
such as obesity or thrombophilia. Venous HT, when administered within the first 5 years of
thromboembolism risk is not significantly the final menopause attenuates some but not all
affected by micronized progesterone, adverse effects of menopause that are related to
medroxyprogesterone acetate, or norethindrone hypogonadism.
but may be increased by norpregnane derivatives. Symptoms commonly associated with
menopause (hot flashes, sleep dysfunction, mood,
Cognitive Function and genitourinary syndrome of menopause) are
The effects of HT on cognitive function are readily improved with HT. Nonetheless, fear of
variable. Observational studies suggested cognitive hormones has led to widespread undertreatment
function may improve with HT. Results from of symptoms and a proliferation of nonrigorously
the Kronos Early Estrogen Prevention Study and tested but popular treatments such as custom
Early vs Late Intervention Trial with Estradiol compounded hormones, pellets, and a large variety
suggest that HT does not affect cognitive function of herbal preparations. Clinicians should assess
when it is taken by younger women and in early a patient’s symptoms and offer evidence-based
menopause. In contrast, the Women’s Health treatment, with periodic reevaluation of the
Initiative Memory Study found an increased necessity of treatment.
incidence of dementia in women older than 65
years initiating HT. These studies suggest that
cognitive health may be adversely affected by an Clinical Case Vignettes
interaction between age and HT. Case 1
A 48-year-old, G2P2, African American woman
Discontinuing Menopausal HT presents with complaints of hot flashes that are
Many women choose to discontinue hormones disruptive to her sleep and daily activity. She wakes
after the worst of their menopausal symptoms up 6 to 8 times each night and reports feeling
have subsided. For women planning to stop uncomfortable multiple times during the day.
HT, they should stop taking HT 1 week
Which of the following factors
before an annual visit to allow assessment
is likely to affect the duration of
of symptom severity. Most women prefer to
this patient’s hot flashes?
slowly wean, while others just stop. However,
a small proportion of women (3% to 15%) have A. Race
persistence of severe symptoms and will require B. Age
continued hormones or nonhormonal alternatives. C. Depression
D. Socioeconomic status
Summary
Answer: A) Race
The hypogonadism of menopause can adversely
affect multiple organ systems and increase the risk There are racial and ethnic differences in
for morbidities such as osteoporosis, dementia, the duration and persistence of hot flashes

ENDO 2021 • Reproductive Endocrinology  295

chased by Dr. Khalid H. Seedahmed, Jr., MRCP PGDip, CertEndocrinology SA - ID 283


(Answer A). For this patient, black race is a Which of the following would be the
modifier. When compared with other races and best treatment for her heavy bleeding?
ethnic groups, African American women are more A. Combined oral contraception
likely to have a longer duration and persistence of B. Copper intrauterine device
hot flashes. Other modifiers of the severity of hot
C. Progestin intrauterine device
flashes include a high BMI, lower socioeconomic
status, and depression. D. Vaginal contraceptive ring
See references 4 and 11. E. Barrier contraception
Answer: C) Progestin intrauterine device
Case 2
In women older than 45 years, combined
A 48-year-old, G0, sexually active woman presents
oral contraceptives and vaginal contraceptive
with extremely heavy bleeding after amenorrhea
rings impose an increased risk for venous
for 6 months. Prior to that, menses were every
thromboembolism,12 myocardial infarction,13
18 to 90 days. Her BMI is 62 kg/m2, and she has
and breast neoplasia.14 A copper intrauterine
a diagnosis of polycystic ovary syndrome, but she
device and a barrier would provide contraception,
is otherwise healthy. She has no allergies. She is
but they would not reduce the bleeding related
married and has an unremarkable family history.
to ovulatory dysfunction observed during
She would like help with her heavy periods and a
perimenopause. A progestin intrauterine device
contraceptive method.
would provide contraception and reduce the risk
for endometrial neoplasia.15

References
1. Gold EB, Crawford SL, Avis NE, et al. Factors related to age at natural 8. Breast cancer and hormone replacement therapy: collaborative reanalysis
menopause: longitudinal analyses from SWAN. Am J Epidemiol. of data from 51 epidemiological studies of 52,705 women with breast
2013;178(1):70-83. PMID: 23788671 cancer and 108,411 women without breast cancer. Collaborative Group on
2. Santoro N, Crawford SL, El Khoudary SR, et al. Menstrual cycle hormone Hormonal Factors in Breast Cancer [published correction appears in Lancet.
changes in women traversing the menopause: study of women’s health across 1997;350(9089):1484]. Lancet. 1997;350(9084):1047-1059. PMID: 10213546
the nation. J Clin Endocrinol Metab. 2017;102(7):2218-2229. PMID: 28368525 9. Chlebowski RT, Rohan TE, Manson JE, et al. Breast cancer after use of
3. Prague JK, Roberts RE, Comninos AN, et al. Neurokinin 3 receptor estrogen plus progestin and estrogen alone: analyses of data from 2 Women’s
antagonism as a novel treatment for menopausal hot flushes: a Health Initiative Randomized Clinical Trials. JAMA Oncol. 2015;1(3):296-305.
phase 2, randomised, double-blind, placebo-controlled trial. Lancet. PMID: 26181174
2017;389(10081):1809-1820. PMID: 28385352 10. Canonico M, Plu-Bureau G, Lowe GD, Scarabin PY. Hormone replacement
4. Avis NE, Crawford SL, Greendale G. Duration of menopausal vasomotor therapy and risk of venous thromboembolism in postmenopausal women:
symptoms over the menopause transition. JAMA Intern Med. 2015;175(4):531- systematic review and meta-analysis. BMJ. 2008;336(7655):1227-1231.
539. PMID: 25686030 PMID: 18495631
5. Portman DJ, Gass ML; Vulvovaginal Atrophy Terminology Consensus 11. Gold EB, Sternfeld B, Kelsey JL, et al. Relation of demographic and lifestyle
Conference Panel. Genitourinary syndrome of menopause: new terminology factors to symptoms in a multi-racial/ethnic population of women 40-55
for vulvovaginal atrophy from the International Society for the Study of years of age. Am J Epidemiol. 2000;152(5):463-473. PMID: 10981461
Women’s Sexual Health and the North American Menopause Society. 12. Burkman RT, Collins JA, Shulman LP, Williams JK. Current perspectives
Menopause. 2014;21(10):1063-1068. PMID: 25160739 on oral contraceptive use. Am J Obstet Gynecol. 2001;185(Suppl 2):S4-S12.
6. Bromberger JT, Kravitz HM, Chang YF, Cyranowski JM, Brown C, PMID: 11521117
Matthews KA. Major depression during and after the menopausal transition: 13. Carr BR, Ory H. Estrogen and progestin components of oral contraceptives:
study of Women’s Health Across the Nation (SWAN) [published correction relationship to vascular disease. Contraception. 1997;55(5):267-272.
appears in Psychol Med. 2011;41(9):1879-1888]. Psychol Med. 2011;41(9):1879- PMID: 9220222
1888. PMID: 21306662 14. Marchbanks PA, McDonald JA, Wilson HG, et al. Oral contraceptives and the
7. The NAMS 2017 Hormone Therapy Position Statement Advisory Panel. risk of breast cancer. N Engl J Med. 2002;346(26):2025-2032. PMID: 12087137
The 2017 hormone therapy position statement of The North American 15. Bitzer J. Overview of perimenopausal contraception. Climacteric.
Menopause Society. Menopause. 2017;24(7):728-753. PMID: 28650869 2019;22(1):44-50. PMID: 30562124

296  ENDO 2021 • Endocrine Case Management

chased by Dr. Khalid H. Seedahmed, Jr., MRCP PGDip, CertEndocrinology SA - ID 283


Detection and Management
of Abuse of Androgenic
Performance-Enhancing Drugs
Bradley D. Anawalt, MD. University of Washington School of Medicine, Seattle, WA;
E-mail: banawalt@medicine.washington.edu

Learning Objectives testosterone prescriptions to treat low serum


testosterone. Serum gonadotropins are suppressed,
As a result of participating in this session, learners
and seminal fluid analyses show azoospermia
should be able to:
or very low sperm concentrations. Recovery
• Recognize the clinical presentation of of gonadal axis function occurs spontaneously
androgenic performing-enhancing drug (PED) after discontinuation of androgenic PEDs.
abuse (including infertility). Treatment consists of encouraging cessation by
respectful and nonjudgmental education about
• Identify the laboratory studies that are useful
the health consequences of androgenic PEDs and
for identifying androgenic PED abuse.
management of underlying anxiety and depressive
• Recognize the pattern of serum hormone disorders. For some men who have abused high
results that are consistent with specific forms dosages of PEDs for years, there might be a role
of androgenic PED abuse. for adjunctive hormone therapy with a short
• Develop a rational approach to the use of course of testosterone or clomiphene to treat
psychotherapy and hormone therapy in anabolic androgenic steroid withdrawal syndrome
the management of patients who wish to or a short course of gonadotropin or clomiphene
discontinue androgenic PED abuse. therapy to hasten spermatogenic recovery and
improve fertility.

Main Conclusions Significance of the


Androgens and drugs that raise endogenous Clinical Problem
androgen production are the most commonly
Many drugs are used to enhance athletic
abused PEDs. In clinical practice, androgenic
performance and appearance, including androgens
PEDs are used almost exclusively by men. The
and drugs that increase serum androgen
proven health consequences include infertility,
concentrations, insulin, and growth hormone.
acne, and erythrocytosis in men. The other
Androgens administered at pharmacological
potential adverse effects include increased
dosages are the most commonly used PEDs, and
cardiovascular disease, hepatopathy (with oral
they are the only class of drugs with definitive
alkylated androgens), neuropsychopathology,
evidence that they enhance strength and athletic
and ruptured upper-extremity tendons. The
performance. Members of the general public are
common clinical presentations of androgenic
much less likely to use growth hormone, insulin,
PED abuse are male infertility and requests for
and other nonandrogens as potential PEDs.

ENDO 2021 • Reproductive Endocrinology  297

chased by Dr. Khalid H. Seedahmed, Jr., MRCP PGDip, CertEndocrinology SA - ID 283


For the purposes of this review and session, the and anxiety and to use tobacco and marijuana
focus will be androgenic PED abuse, and the products, alcohol, illicit drugs, and high dosages of
use of pharmacological dosages of androgens or dietary supplements and nutraceuticals that have
drugs that increase endogenous androgens will not been studied rigorously for safety. In addition,
be defined as androgenic PED use. Individuals androgenic PEDs are often purchased on the open
who are eugonadal and take androgenic PEDs market or through the Internet, and these drugs
are abusing them; prescription of androgen might contain or be contaminated with unsafe
and gonadotropin replacement to hypogonadal substances.
patients is not PED abuse.
It has been difficult to assess the prevalence of
androgenic PED abuse, but the lifetime prevalence
Barriers to Optimal Practice
of ever using these PEDs is probably 1% to 5%
• Social stigma and laws against androgenic PED
in men worldwide.1 There is a trend toward
abuse are barriers to disclosure of abuse.
decreased incidence in teenagers and young
adults, but there might be increased incidence in • Many clinicians do not recognize the clues of
middle-aged and older men. The chronic abuse of possible androgenic PED abuse.
androgenic PEDs is much lower. The prevalence • Except for disclosure of androgenic abuse by
of androgenic PED abuse is much higher in men the patient, there are no methods of making a
than in women (>50:1), and long-term abusers are definitive diagnosis of androgenic PED abuse
almost exclusively male. Most chronic androgenic in members of the general public.
PED abusers begin in their twenties and are • There are very limited data on the best
former elite or near-elite athletes or weightlifters therapeutic approach for individuals
with a fixation on being more muscular (often who abuse androgenic PEDs and want to
referred to as muscle dysmorphia or bigorexia). discontinue.
It has been difficult to assess the short-term
and long-term adverse effects of androgenic • The data about the short-term and long-term
PEDs. There are well-described adverse effects effects of androgenic PED abuse are based
of androgen therapy, including acne (more on epidemiological studies with numerous
common in younger patients), erythrocytosis confounders.
(more common in older patients), and infertility.1,2
Chronic androgenic PED use accelerates male-
pattern alopecia and causes hirsutism, alopecia, Strategies for Diagnosis,
and defeminization and virilization in women. Therapy, and/or Management
Hepatopathy occurs with use of oral alkylated
androgens only, and there appears to be increased
Clinical Presentation of
risk of tendinous ruptures, particularly in muscles Androgenic PED Abuse
of the arms and chest.3,4 There is increasing Men who chronically abuse androgenic PEDs
evidence that there is an androgenic PED typically present with infertility or with requests
withdrawal syndrome of depressed mood, anxiety, for evaluation for testosterone therapy. Men who
and low self-esteem after acute cessation.5,6 are chronic androgenic PED abusers may present
Other potential adverse effects of high with androgenic PED withdrawal syndrome,
dosages of androgenic PEDs include increased with symptoms and signs of hypogonadism, or
risk of cardiovascular events, hypertension, and infertility. The use of aromatizable, androgenic
psychiatric disorders,3,7 but the causal relationship PEDs may cause gynecomastia, and the use
is unclear because chronic androgenic PED abusers of nonaromatizable androgenic PEDs may be
are also more likely to have baseline depression associated with sexual dysfunction.

298  ENDO 2021 • Endocrine Case Management

chased by Dr. Khalid H. Seedahmed, Jr., MRCP PGDip, CertEndocrinology SA - ID 283


Women who are chronic androgenic abusers Table. Serum Hormone Concentrations
are almost exclusively elite athletes and are rarely During Androgenic PED Use
seen by most endocrinologists; they may present Serum Serum Serum
with amenorrhea, infertility, and variable degrees Androgenic PED Testosterone FSH LH
of defeminization and virilization. This session Testosterone ↑ ↓ ↓
will not address this rare endocrine disorder. ↑ ↓ ↓
Testosterone precursorsa

Nontestosterone anabolic ↓ ↓ ↓
Diagnosis of Androgenic PED Abuse androgenic steroid

The diagnosis of androgenic PED abuse should hCG ↑ ↓ ↓

be suspected in muscular men who present Aromatase inhibitor alone ↑ ↑ ↑


with infertility or an evaluation for testosterone ↑ ↑ ↑
Clomiphene
therapy. Most men who abuse androgenic
PEDs exercise frequently and many lift weights a
Very high dosages are required.
regularly. These men usually have bigorexia or
muscular dysmorphia, a distorted body image
that they are not muscular enough.1 They will hypogonadism, including sellar imaging and
often spontaneously report that their strength and measurement of serum prolactin, should be done
muscle bulk benefit has plateaued in the weight in patients with this biochemical profile who do
room. Another clue that may suggest chronic not report the use of androgenic PEDs. If
androgenic PED abuse is a history of use of androgenic PED use is suspected in a patient with
nutraceuticals and supplements to build muscles laboratory evidence of secondary hypogonadism, it
(eg, creatine powder). is worthwhile to repeat a respectful and
In addition to increased musculature, nonjudgmental enquiry about androgenic PED
the physical examination may demonstrate use. Sometimes, patients will report androgenic
gynecomastia and small testes, but these findings PED use if they understand that further evaluation
are not sensitive or specific. Chronic androgenic for other secondary causes of hypogonadism will
PED abuse is associated with variably decreased cost them time and money.
testicular volumes (typically ~25%-35%), and men Androgens raise serum hematocrit and
with baseline testicular volumes of 25 mL or more decrease serum HDL cholesterol, SHBG, and
may not have small testes after chronic androgenic thyroxine-binding globulin. In patients with
PED abuse. Testicular texture (eg, softness or suspected androgenic PED use, it is useful to
firmness) is not a specific or sensitive physical measure hematocrit and often helpful to measure
examination finding of androgenic PED abuse. HDL cholesterol, SHBG, and/or thyroxine-
The biochemical evaluation for suspected binding globulin. Seminal fluid analyses
androgenic PED abuse begins with measurement generally show azoospermia or very low sperm
of serum testosterone and gonadotropins on a concentrations.
blood sample obtained between 7 and 10 AM. There are methods for detecting androgenic
Serum testosterone and gonadotropin PEDs and their metabolites in urine samples of
concentrations differ depending on the type of elite athletes, but these methods are not available
androgenic PED use (Table). The use of in commercial laboratories.8 In addition, there are
nontestosterone androgenic anabolic steroids (eg, many ways to create spurious results, including
nandrolone, oxandrolone, danazol, stanozolol, or dilution, substitution of someone else’s sample,
tetrahydrogestrinone) is associated with low and submitting a sample at a time when the
testosterone and gonadotropin concentrations. patient is not using androgenic PEDs. For elite
The standard evaluation for secondary athletes, urine samples are obtained under very

ENDO 2021 • Reproductive Endocrinology  299

chased by Dr. Khalid H. Seedahmed, Jr., MRCP PGDip, CertEndocrinology SA - ID 283


strict circumstances, including random times and testosterone. Conversion to intramuscular
observed micturition into a sample container. testosterone formulations at up to 2 to 3 times
Thus, methods to accurately measure androgenic the usual replacement dosage with a tapering of
PEDs and their metabolites in urine are not the dosage over several months is likely to be
relevant diagnostic studies in clinical practice. safer than very high dosages of androgenic PEDs
of unknown safety and purity purchased from
unregulated sources. High dosages of testosterone
Treatment of Androgenic PED
are initially required because androgenic PED users
Abuse and Withdrawal Syndrome generally use high dosages of various androgens
There are no clinical trials or guidelines on the simultaneously (“stacking”), and they will experience
management of patients who abuse androgenic symptoms of withdrawal if immediately converted
PEDs or the management of cessation of directly to typical testosterone replacement
androgenic PEDs and androgenic PED withdrawal dosages used for treatment of male hypogonadism.
syndrome. Therefore, care of patients who abuse This approach permits the establishment of a
androgenic PEDs is based on the following: (1) relationship between the patient and clinician over
readiness to cease androgenic PEDs; (2) duration time; this might facilitate eventual discontinuation
and intensity of androgenic PED abuse; (3) near- of androgenic PED use.
team goals for fertility; and (4) a risk-benefit For men who are ready to quit androgenic
analysis of the treatment options compared PED use, there are the following options: (1)
with continuation or resumption of use of immediate discontinuation without medical
androgenic PEDs.1 therapy; (2) conversion to testosterone therapy
A key principle for the care of androgenic PED with a tapering dosage over many months (see
users is the establishment a relationship of trust above); (3) discontinuation with initiation of a
based on respectful and nonjudgmental behavior. limited course of subcutaneous hCG therapy, or
While this principle guides all patient-clinician (4) discontinuation with initiation of a limited
relationships, it is particularly important for this course of oral clomiphene therapy.
group of patients. They are often distrustful of Immediate discontinuation of androgenic
traditional health care providers and scientists. After PEDs without additional medical therapy is
all, scientists disputed the benefits of androgenic the best choice for men who have used these
PEDs decades after athletes empirically proved drugs for 1 year or less. Based on limited, low-
the performance-enhancing effects; why should quality data, it appears that after androgenic
androgenic PED abusers believe scientists and PED discontinuation, serum gonadotropin
clinicians about potential adverse effects? All concentrations spontaneously normalize within
androgenic PED users must be queried about anxiety 3 to 6 months, followed by recovery of serum
and depression and abuse of alcohol or illicit drugs testosterone to normal serum concentrations
and offered a referral for treatment if appropriate. within 6 months in most men who report a year
Some men are not prepared to stop using or less of androgenic PED use. Spermatogenesis
androgenic PEDs. The endocrinologist should typically returns to normal 3 to 6 months after
counsel patients about the known adverse effects serum testosterone normalizes.1,9
(including reduced fertility that might take 1 For men who report chronic androgenic
to 2 years to normalize after discontinuation PED use longer than 1 year, there is a higher risk
of androgenic PEDs, erythrocytosis, and of withdrawal syndrome, and the gonadal axis
dyslipidemia) and potential adverse effects with may take years to recover.1,10,11 Intramuscular
a focus on the cardiovascular system. Some men testosterone that is tapered over many months
who are not willing to discontinue androgenic might be a good option for many of these men, but
PEDs are willing to “convert” to prescription this approach is not an option for men who are

300  ENDO 2021 • Endocrine Case Management

chased by Dr. Khalid H. Seedahmed, Jr., MRCP PGDip, CertEndocrinology SA - ID 283


interested in conceiving. Exogenous testosterone What additional evaluation would you
therapy will continue to suppress spermatogenesis order now to help confirm your suspicion
and fertility. For men who are infertile because that he is using androgenic PEDs?
of androgenic PED use for more than 1 year A. Serum lipid panel and cortisol-binding
and who want to have a child within the next 2 globulin measurement
years, the best options may be hCG therapy or B. Serum lipid panel and SHBG measurement
clomiphene therapy. There is much more clinical
C. Serum thyroxine-binding globulin and
experience and safety data for hCG therapy than
cortisol-binding globulin measurement
for clomiphene, but there are scanty data on the
effectiveness for improving spermatogenesis and D. Screen of urinary metabolites of testosterone
fertility in this setting for either drug. and testosterone precursors and known
androgenic steroids by mass spectrometry

Clinical Case Vignettes Answer: B) Serum lipid panel and SHBG measurement
Case 1 This man has a discordance between his
A 32-year-old man requests evaluation for physical appearance (muscularity) and his
testosterone or clomiphene therapy. He reports serum testosterone concentration (very low),
decreased libido and decreased sexual pleasure. and he is a member of a social group at higher
He also reports that he is experiencing less benefit risk for use of androgenic PEDs (weightlifter
(muscle mass and strength) with his weightlifting at a gym). His hematocrit is higher than
workouts at the local gym. He reports no easy would be expected for his serum testosterone
bruisability, purple striae, hand arthralgias, or concentration, too. Androgens suppress serum
use of corticosteroids or androgens. He went HDL cholesterol, SHBG, and thyroxine-binding
through puberty at age 13 or 14 years. He reports globulin concentrations. Low concentrations of
no history of medical illness, and his only surgery HDL cholesterol, SHBG, and thyroxine-binding
was an appendectomy at age 9 years. He takes globulin concentrations would support the clinical
no medications. He states that he does not take suspicion of androgenic PED abuse (Answer B).
anabolic androgenic steroids (“steroids”). He Androgens do not affect serum cortisol-
reports taking no opioids or other illicit drugs. He binding globulin concentrations (thus, Answers
drinks 2 to 3 cocktails weekly. A and C are incorrect). Sophisticated methods for
On physical examination, his blood detection of androgen abuse are used by the World
pressure is 135/85 mm Hg, pulse rate is Anti-Doping Agency to screen for androgenic
80 beats/min, respiratory rate is 12 breaths/min, PED use by elite athletes. These methods are not
and BMI is 21 kg/m2. He is muscular. There is no widely available in commercial laboratories (thus,
gynecomastia, and testes are 15 mL bilaterally. No Answer D is incorrect). Furthermore, unlike
acne is observed. athletes who must provide urine samples under
observation at random times, a patient has many
Laboratory test results: methods of creating false results (eg, dilution,
Hematocrit = 46% (38%-48%)
providing a urine sample from another individual,
Serum total testosterone = 110 ng/dL (264-915 ng/dL) or providing a urine sample when not taking
(SI: 3.8 nmol/L [9.2-31.8 nmol/L]) androgenic PEDs).
Serum FSH = 0.2 IU/L (1.0-7.0 IU/L)
Serum LH = 0.1 IU/L (1.0-9.0 IU/L)

ENDO 2021 • Reproductive Endocrinology  301

chased by Dr. Khalid H. Seedahmed, Jr., MRCP PGDip, CertEndocrinology SA - ID 283


Case 1 (Continued) steroids for the past year. He reports that he takes
a combination of several at once (“stacking”) and
The patient’s serum HDL-cholesterol, SHBG,
rotates the combination (“cycling”). His wife has
and thyroxine-binding globulin concentrations
been urging him to stop taking them because she
are very low. His serum prolactin and ferritin
is concerned about his health. He is under the
concentrations are normal. Sellar MRI
care of a mental health professional for treatment
demonstrates a normal pituitary gland.
of anxiety and depression. He is very concerned
Which of the following is the about losing muscle mass, and he would like to
most likely diagnosis? know if there is any medical therapy that he can
take after stopping androgenic PEDs.
A. Androstenedione abuse
B. Clomiphene abuse In addition to advising immediate
C. Tetrahydrogestrinone (“Clear”) abuse cessation of androgenic PEDs, which of
D. Androgen-producing adrenal tumor the following would you recommend?
A. No additional therapy
Answer: C) Tetrahydrogestrinone (“Clear”) abuse
B. Testosterone gel taper over 6 months
This patient has low serum testosterone and C. Clomiphene taper over 12 months
gonadotropin concentrations. A nontestosterone D. hCG taper over 6 months
androgenic PED such as tetrahydrogestrinone
(Answer C) would produce these results by Answer: A) No additional therapy
suppressing gonadotropin secretion and reducing
In patients who have been taking androgenic
endogenous androgen production.
PEDs for 1 year or less, the best management of
Androstenedione (Answer A) is a precursor
cessation of androgens is supportive care (eg,
in the pathway of testosterone synthesis.
treatment of depression and anxiety) with no
Consumption of very large dosages of
additional medical therapy (Answer A). These
androstenedione can raise serum testosterone
patients generally recover normal function of the
concentration (by mass action) and act indirectly
gonadal axis within 6 to 12 months.
as a PED. In that setting, the serum testosterone
Additional medical therapy is unnecessary,
concentration would be high-normal or elevated.
and the cost or potential risks are not justified.
Clomiphene (Answer B) is a selective estrogen-
Exogenous testosterone and hCG (Answers
receptor modulator that increases gonadotropin
B and D) may delay gonadal axis recovery.
secretion and circulating gonadotropin and
Based on inferential data from women,
testosterone concentrations in men with intact
clomiphene (Answer C), a selective estrogen-
gonadotrope and Leydig-cell function. Adrenal
receptor modulator, may increase the risk of
tumors (Answer D) do not commonly produce
thromboembolic events, and that potential risk is
enough testosterone to affect serum testosterone
not justified in this setting.
concentrations in men. When these tumors
produce significant amounts of testosterone,
serum gonadotropins may be suppressed, but the Case 1 (Continued)
serum testosterone concentration is not low. He returns 2 months later with his wife. He
reports that he has felt tired and despondent since
Case 1 (Continued) stopping the androgenic PEDs. He has low libido
and low sexual satisfaction. He now reports that he
When the patient returns to your clinic in 2 weeks
has been taking androgenic PEDs for many years.
to review the results of his evaluation, he confides
His wife confirms these symptoms and reports that
that he has been taking anabolic androgenic
they have been trying to conceive during the past

302  ENDO 2021 • Endocrine Case Management

chased by Dr. Khalid H. Seedahmed, Jr., MRCP PGDip, CertEndocrinology SA - ID 283


6 months. She is 33 years old, healthy, takes no This patient is likely to have prolonged
medications, and has regular menses every 28 to absence or markedly decreased spermatogenesis;
32 days. reassurance (Answer A) is incorrect. An
intramuscular testosterone taper (Answer B)
Laboratory test results at this clinic visit: would lessen his withdrawal symptoms, but
Hematocrit = 41% (38%-48%) exogenous testosterone would continue to
Serum total testosterone = 80 ng/dL (264-915 ng/dL) suppress serum gonadotropin concentrations
(SI: 2.8 nmol/L [9.2-31.8 nmol/L]) and spermatogenesis. Aromatase inhibitors
Serum FSH = 0.3 IU/L (1.0-7.0 IU/L)
suppress serum estradiol concentrations, thereby
Serum LH = 0.2 IU/L (1.0-9.0 IU/L)
Seminal fluid analysis, no sperm increasing gonadotropin secretion; there is
anecdotal evidence for using this class of drugs
in selected men with hypospermatogenesis
Which of the following would and infertility (eg, men with infertility due to
you recommend? morbid obesity). Anastrozole (Answer D) might
A. Reassurance increase spermatogenesis in this man, but there
B. Intramuscular testosterone taper is much stronger evidence for effectiveness with
C. hCG therapy gonadotropin replacement therapy. Furthermore,
aromatase inhibitor therapy may cause loss of bone
D. Anastrozole therapy
mineral density, increase body fat, and decrease
Answer: C) hCG therapy sexual function in men.

This man continues to be significantly hypogonadal


after 2 to 3 months of cessation of androgenic PED Case 2  New Content 
abuse. His decline in hematocrit supports that he A 21-year-old man requests evaluation for
has discontinued. It is unclear how quickly his testosterone therapy. He reports that he does not
gonadal axis will recover, but his additional history get the same “cut” look (increased muscles) that
that he took androgenic PEDs for many years his friends at the gym do. He reports mild breast
suggests that it might be more than 1 to 2 years tenderness and no change in libido. He has had
before he recovers spermatogenesis. His wife will be no weight loss. He went through puberty at age
35 in 2 years, an age when female fertility begins to 13 years. He reports no history of medical illness
decline (on average). Gonadotropin therapy is very and no surgeries. He states that he does not take
effective in most men (~90%) with postpubertal any medications or anabolic androgenic steroids
hypogonadotropic hypogonadism. His testicular (“steroids”), opioids, or other illicit drugs. He does
volumes indicate that he completed normal puberty. not drink alcohol.
Treatment with hCG therapy (Answer C) (that has On physical examination, his blood pressure
LH) is likely to be effective in restoring his normal is 110/70 mm Hg, pulse rate is 72 beats/min,
endogenous testosterone concentrations and may respiratory rate is 12 breaths/min, and BMI is
restore spermatogenesis within 6 to 12 months. 21 kg/m2. He has a beard and normal musculature.
FSH therapy (usually as recombinant human FSH) There is 2 cm of slightly tender gynecomastia,
may be added if conception has not occurred and testes are 25 mL bilaterally. There are no
and spermatogenesis remains low after serum scrotal masses. He has normal findings on skin
testosterone concentrations have been normalized examination with no striae or ecchymoses. He has
with hCG therapy. Treatment with hCG to mild acne on his back.
normalize his serum testosterone concentration You have received old records from his
would also help alleviate his withdrawal symptoms. pediatrician. The notes corroborate the medical
history, including normal puberty and normal

ENDO 2021 • Reproductive Endocrinology  303

chased by Dr. Khalid H. Seedahmed, Jr., MRCP PGDip, CertEndocrinology SA - ID 283


growth curve. The last note indicates testicular Case 3  New Content 
volume of “15-20 cc” at age 16 years and no
A 55-year-old man reports 6 months of tender
testicular masses.
gynecomastia. He has had no change in libido. He
Laboratory test results: is one of the top runners (10K to half marathon)
in his age group in his region. He has a history of
Hematocrit = 48% (38%-48%)
hypertension and benign prostatic hyperplasia,
Serum total testosterone = 880 ng/dL (264-870 ng/dL)
(SI: 30.6 nmol/L [9.2-30.2 nmol/L]) and he takes lisinopril and finasteride. He reports
Serum FSH = 0.2 IU/L (1.0-7.0 IU/L) that he does not take opioids, anabolic steroids, or
Serum LH = 0.1 IU/L (1.0-9.0 IU/L) other illicit drugs. He does not drink alcohol.
Serum free T4, normal On physical examination, his blood pressure
TSH, normal
is 130/80 mm Hg, pulse rate is 60 beats/min,
Serum iron-binding capacity, normal
Ferritin, normal respiratory rate is 12 breaths/min, and BMI is
21 kg/m2. He has a beard and normal musculature.
There is 3 cm of tender tender gynecomastia, and
Which of the following is the testes are 20 mL bilaterally. There are no scrotal
most likely diagnosis? masses. He has no abnormal skin findings.
A. Nonfunctional pituitary macroadenoma
Laboratory test results:
B. Hemochromatosis
C. Abuse of intramuscular testosterone and hCG Hematocrit = 44% (38%-48%)
Serum total testosterone = 770 ng/dL (264-870 ng/dL)
D. Abuse of intramuscular nandrolone (SI: 26.7 nmol/L [9.2-30.2 nmol/L])
Serum FSH = 10.0 IU/L (1.0-7.0 IU/L)
Answer: C) Abuse of intramuscular Serum LH = 13.0 IU/L (1.0-9.0 IU/L)
testosterone and hCG Serum prolactin, normal

This man is in the demographic that is most


likely to abuse androgens (male, age ~16-25 Which of the following is the
years, weightlifter or bodybuilder). Testosterone most likely diagnosis?
and hCG abuse is consistent with the presentation A. LH-secreting pituitary macroadenoma
of a high serum testosterone concentration and B. Clomiphene abuse
low serum gonadotropin concentrations. Some
C. hCG-producing tumor
men take hCG in addition to an androgenic PED
because concomitant use of hCG may prevent D. Androstenedione abuse
testicular shrinkage due to suppressed circulating Answer: B) Clomiphene abuse
gonadotropins. The high serum testosterone
concentration excludes abuse with a nontestosterone It is increasingly common for older male athletes
androgenic PED such as nandrolone to abuse androgenic PEDs in order to perform
(Answer D). Nontestosterone androgens result better in regional and national competitions. This
in suppression of serum testosterone and athlete’s presentation of new-onset gynecomastia
gonadotropins. The high serum testosterone and high-normal serum testosterone and serum
and low LH concentrations are inconsistent gonadotropins suggests the use of a selective
with a nonfunctional pituitary macroadenoma estrogen receptor modulator such as clomiphene
(Answer A). This patient’s presentation with a (Answer B) or enclomiphene.12 Abuse of
high-normal serum testosterone concentration aromatase inhibitors such as anastrozole may
is inconsistent with hemochromatosis produce a similar biochemical profile. Pituitary
(Answer B) that commonly causes secondary tumors may produce immunoreactive LH and/
hypogonadotropic hypogonadism. or FSH, but they very rarely produce bioactive

304  ENDO 2021 • Endocrine Case Management

chased by Dr. Khalid H. Seedahmed, Jr., MRCP PGDip, CertEndocrinology SA - ID 283


gonadotropins. Thus, it would be extremely rare concentrations, but the increased serum
for a pituitary macroadenoma (Answer A) to make testosterone would suppress serum FSH and LH.
enough bioactive LH to result in normal or high A very high dosage of testosterone precursors such
serum testosterone concentrations. The rapidity as androstenedione (Answer D) may raise serum
of onset of gynecomastia and the normal serum testosterone and estradiol concentrations (causing
prolactin concentration also argue against this gynecomastia), but the increased serum sex steroid
very rare endocrinopathy. An hCG-producing hormone concentrations would also suppress
tumor (Answer C) may raise serum testosterone serum FSH and LH.

References
1. Anawalt BD. Diagnosis and management of anabolic androgenic steroid use. 8. Anawalt BD. Detection of anabolic androgenic steroid use by elite athletes
J Clin Endocrinol Metab. 2019;104(7):2490-2500. PMID: 30753550 and by members of the general public. Mol Cell Endocrinol. 2018;464:21-27.
2. Christou MA, Christou PA, Markozannes G, Tsatsoulis A, Mastorakos G, PMID: 28943276
Tigas S. Effects of anabolic androgenic steroids on the reproductive system of 9. Liu PY, Swerdloff RS, Christenson PD, Handelsman DJ, Wang C; Hormonal
athletes and recreational users: a systematic review and meta-analysis. Sports Male Contraception Summit Group. Rate, extent, and modifiers of
Med. 2017;47(9):1869-1883. PMID: 28258581 spermatogenic recovery after hormonal male contraception: an integrated
3. Pope HG Jr, Wood RI, Rogol A, Nyberg F, Bowers L, Bhasin S. Adverse analysis. Lancet. 2006;367(9520):1412-1420. PMID: 16650651
health consequences of performance-enhancing drugs: an Endocrine Society 10. Kanayama G, Hudson JI, DeLuca J, et al. Prolonged hypogonadism in
scientific statement. Endocr Rev. 2014;35(3):341-375. PMID: 24423981 males following withdrawal from anabolic-androgenic steroids: an under-
4. Kanayama G, DeLuca J, Meehan WP 3rd, et al. Ruptured tendons in recognized problem. Addiction. 2015;110(5):823-831. PMID: 25598171
anabolic-androgenic steroid users: a cross-sectional cohort study. Am J Sports 11. Rasmussen JJ, Selmer C, Østergren PB, et al. Former abusers of anabolic
Med. 2015;43(11):2638-2644. PMID: 26362436 androgenic steroids exhibit decreased testosterone levels and hypogonadal
5. Onakomaiya MM, Henderson LP. Mad men, women and steroid cocktails: a symptoms years after cessation: a case-control study. PLoS One.
review of the impact of sex and other factors on anabolic androgenic steroids 2016;11:e0161208. PMID: 275532478
effects on affective behaviors. Psychopharmacology (Berl). 2016;233(4):549-569. 12. Miller GD, Moore C, Nair V, et al. Hypothalamic-pituitary-testicular axis
PMID: 26758282 effects and urinary detection following clomiphene administration in males.
6. Piacentino D, Kotzalidis GD, Del Casale A, et al. Anabolic-androgenic J Clin Endocrinol Metab. 2019;104(3):906-914. PMID: 30295816
steroid use and psychopathology in athletes. A systematic review. Curr
Neuropharmacol. 2015;13(1):101-121. PMID: 26074746
7. Baggish AL, Weiner RB, Kanayama G, et al. Cardiovascular toxicity of
illicit anabolic-androgenic steroid use. Circulation. 2017;135(21):1991-2002.
PMID: 28533317

ENDO 2021 • Reproductive Endocrinology  305

chased by Dr. Khalid H. Seedahmed, Jr., MRCP PGDip, CertEndocrinology SA - ID 283


Evaluation of Male Infertility
Channa Jayasena, MD, PhD. Division of Diabetes and Endocrinology, Imperial College
London, London, United Kingdom; E-mail: c.jayasena@imperial.ac.uk

Learning Objectives use. Obstructive azoospermia (OA) is a common


and underrecognized form of male infertility,
As a result of participating in this session, learners
most often caused by genitourinary tract
should be able to:
infection. However, it may also be associated
• Investigate the diagnostic pathway for male with cystic fibrosis carrier status (pathogenic
infertility. variants in the CFTR gene). There is no role for
endocrine therapy in the management of OA, but
• Explain classic and contemporary management
endocrinologists should be aware of the diagnostic
options for treating men with infertility.
features of OA to ensure appropriate urological
referral. This session will provide endocrinologists
with a state-of-the art overview of the evaluation
and management of men with any form of male
Main Conclusions infertility in a multidisciplinary clinic setting.
Male infertility may be classified into secondary
hypogonadism, testicular failure, and obstruction.
Secondary hypogonadism (hypogonadotropic
hypogonadism) is congenital or acquired and is
Significance of the
most familiar to endocrinologists. Several genetic Clinical Problem
pathogenic variants have been identified that cause Infertility is the inability to achieve pregnancy after
congenital hypogonadotropic hypogonadism. 12 months of regular unprotected intercourse. Up to
Gonadotropin injections are highly effective at half of cases of infertility are attributed to a problem
inducing spermatogenesis in men with secondary in the male partner (“male-factor infertility”). Recent
hypogonadism. GnRH pulsatile therapy is reports suggest that sperm counts in North America
usually restricted to research settings when and Europe have halved since the 1970s. Combined
inducing spermatogenesis in men with secondary with increasing maternal age at first pregnancy
hypogonadism. Most cases of male infertility are worldwide, the management of male infertility is
due to testicular failure (ie, hypergonadotropic likely to become even more important in the future.
hypogonadism). The most severe form of
testicular failure is called nonobstructive
azoospermia (NOA). The last decade has seen Barriers to Optimal Practice
notable developments in the diagnostic evaluation
• Endocrine therapies are being increasing
and management of NOA. Microsurgical testicular
used empirically (by urologists) for men with
sperm extraction (mTESE) is used to successfully
testicular failure, but most endocrinologists are
retrieve sperm in 20% to 40% of men with NOA,
unaware of this practice.
including men with Klinefelter syndrome.
Hormonal stimulation with gonadotropins or • Successful treatment of male fertility
selective estrogen modulators are used routinely increasingly requires coordinated management
before mTESE in men with NOA, but there are with other specialties (eg, obstetrics/gynecology,
insufficient data to support their widespread reproductive endocrinology, and urology).

306  ENDO 2021 • Endocrine Case Management

chased by Dr. Khalid H. Seedahmed, Jr., MRCP PGDip, CertEndocrinology SA - ID 283


Strategies for Diagnosis, Etiology
Therapy, and/or Management See Table 1 for a summary of etiologies of male
infertility.1
Physiology of Male Fertility
Male fertility requires activity of the Secondary Hypogonadism
hypothalamic-pituitary-gonadal axis. GnRH is Secondary hypogonadism is caused by
released to the portal circulation in a pulsatile hypothalamic or pituitary disease resulting in
manner, stimulating the anterior pituitary to low testosterone with low LH and FSH secretion
secrete pulses of the gonadotropin hormones, LH (hypogonadotropic hypogonadism). Secondary
and FSH. LH stimulates the testicular Leydig cells hypogonadism may arise from congenital or
to synthesize testosterone, and FSH stimulates acquired GnRH deficiency. Congenital GnRH
Sertoli cells in the seminiferous tubules to support deficiency or congenital hypogonadotropic
spermatogenesis. LH and FSH secretion are hypogonadism is a rare disorder characterized by
negatively regulated by testosterone and inhibin, the deficient production, secretion, or action of
respectively. A cycle of spermatogenesis is 60 to 80 GnRH resulting in low FSH, LH, and testosterone
days in duration. with otherwise normal pituitary function.2

Table 1. Causes of Male Infertility

Etiology Examples Diagnostic Features

Hormonal • Pituitary tumor • Azoospermia or oligospermia


(hypogonadotropic
• Anabolic steroids • FSH: low
hypogonadism)
• Androgens (prescribed and illicit) • LH: low
• Strong opioids (eg, morphine) • Testosterone: low
• Congenital hypogonadotropic hypogonadism • Testicular volume: low
• Diabetes mellitus • Low libido, sparse body hair, erectile dysfunction

Testicular • Primary testicular failure • Azoospermia or oligospermia


• Previous cancer therapy • FSH: high
• Undescended testes (cryptorchidism) • LH: high or normal
• Varicocele • Testosterone: low or normal
• Lifestyle factors (obesity, tobacco smoking, cannabis) • Testicular volume: often low
• Infections (eg, mumps)
• Drugs (eg, sulfasalazine, methotrexate, calcium-channel
blockers, nitrofurantoin, erythromycin, tetracycline,
spironolactone)
• Genetic disorders (eg, Klinefelter syndrome,
Y-chromosome microdeletions, primary ciliary dyskinesia)

Obstruction • Genitourinary infection (chlamydia, ureaplasma, • Usually azoospermia


mycoplasma)
• FSH: normal
• Previous surgery (eg, hernia repair, orchidopexy)
• Testosterone: normal
• Vasectomy
• Testicular volume: normal
• Cystic fibrosis (associated with congenital bilateral
• Elevated semen leukocytes or round cells
absence of vas deferens)
indicate possible infection

Others • Retrograde ejaculation (eg, α-adrenergic blockers, • Low volume or no ejaculate


diabetes, pelvic surgery)
• Sexual dysfunction

ENDO 2021 • Reproductive Endocrinology  307

chased by Dr. Khalid H. Seedahmed, Jr., MRCP PGDip, CertEndocrinology SA - ID 283


Clinical features of congenital GnRH deficiency male sexual differentiation, but the absence of
are incomplete or absent puberty and infertility, other genes on the Y chromosome is incompatible
which may or may not occur in association with with spermatogenesis and is untreatable.
other developmental abnormalities (eg, cleft
lip or palate, hearing impairment). Congenital Y-Chromosome Microdeletions
GnRH deficiency in association with anosmia or The Y chromosome contains genes crucial for
hyposmia is termed Kallmann syndrome, which spermatogenesis.4 Microdeletions in the Yq11
results from incomplete migration in utero of region on the long arm of the Y chromosome (also
GnRH neurons from the olfactory placode to the called the azoospermic factor, AZF region) may
hypothalamus. Alternatively, GnRH neuronal involve several genes, and these microdeletions
function may be deficient, but smell can be intact are present in 5% to 10% of men with NOA (or
(ie, normosmic congenital GnRH deficiency). severe oligoasthenospermia [<5 million sperm/
Pathogenic variants in several genes have been mL]). Deletions of the AZFa and AZFb regions are
identified that cause either normosmic congenital largely incompatible with spermatogenesis, and
GnRH deficiency, Kallmann syndrome, or both.2 mTESE is not advised. However, deletions of the
Hypothalamic GnRH deficiency may also be AZFc region may be compatible with a degree of
acquired due to obesity, chronic disease, opioid spermatogenesis in some patients, so mTESE may
use, or severe weight loss or exercise. be considered with subsequent intracytoplasmic
sperm injection if surgical sperm retrieval is
Testicular Failure successful.
Testicular failure classically results in low
testosterone, high LH and FSH secretion, and Monogenic Causes
either impaired or absent spermatogenesis. Pathogenic variants in INSL3, TEX11, and the
Azoospermia may arise from complete gene encoding the androgen receptor (AR) have
testicular failure associated with genetic factors, been found in men with NOA, but screening is
cryptorchidism, mumps orchitis, testicular currently not performed routinely. Discovery of
cancer, radiotherapy, and cytotoxic agents such as more single genes associated with male infertility
cyclophosphamide. is anticipated in the future.

Klinefelter Syndrome Acquired Causes of Testicular Failure


Approximately 10% of men with NOA have Obesity, type 1 or type 2 diabetes mellitus, trauma,
Klinefelter syndrome caused by a 47,XXY cancer treatment, and several drugs may impair
karyotype. Klinefelter syndrome causes primary spermatogenesis.
testicular failure during early adulthood, with
increased gonadotropins and low testosterone. Obstructive Azoospermia
It had previously been assumed that these OA can be caused by genitourinary tract infections
patients had complete azoospermia, but it has such as chlamydia, ureaplasma, and mycoplasma,
now been reported that 25% may have evidence which lead to scarring and blockage of the
of spermatozoa on semen analysis, although epididymis. Epididymal obstruction is a recognized
spontaneous conception is rare. mTESE followed complication of any testicular surgery. OA may
by intracytoplasmic sperm injection can be used also be associated with CFTR gene carrier status.4
to successfully achieve conception in patients
with Klinefelter syndrome,3 but there is a lack of Assessment
data regarding the live birth rate following this
When evaluating a male patient with infertility,
methodology for such couples. Men with NOA
the clinician should ask questions about loss of
may rarely have a 46,XX karyotype; a fragment of
libido, early morning erections, anosmia, and
the Y chromosome containing the SRY gene causes

308  ENDO 2021 • Endocrine Case Management

chased by Dr. Khalid H. Seedahmed, Jr., MRCP PGDip, CertEndocrinology SA - ID 283


onset of puberty. Aspects of the history suggesting Management
a specific etiology include mumps, sexually
Induction of Spermatogenesis With
transmitted infections, previous malignancy, any Gonadotropin Injections
systemic or chronic disease, and undescended Gonadotropin therapy is the most common
testes. Cigarette smoking, alcohol, cannabis, form of sperm induction for men with secondary
opioids, and anabolic steroids can all contribute to hypogonadism desiring fertility with their partner.2
infertility. Pulsatile GnRH is an alternative treatment for
Clinical examination should look for facial, secondary hypogonadism due to hypothalamic
axillary, and pubic hair and gynecomastia. Both dysfunction, but it is unavailable in most countries.
testes should be palpated to exclude undescended A typical gonadotropin therapy protocol includes
testes. Testicular volume should be assessed using the initiation of hCG to mimic the biological
a Prader orchidometer or ultrasonography. Semen activity of LH and induce the testes to produce
analysis should be performed in all patients testosterone. This may be sufficient to induce
following abstinence of a minimum of 2 days and sperm in some patients, so semen analysis should
maximum of 7 days (Table 2).5 Hormonal analysis be performed after 6 months of hCG therapy.
should include early morning fasted serum The dose of hCG should be titrated to maintain a
testosterone, SHBG, estradiol, LH, FSH, and normal serum testosterone level. FSH-containing
prolactin. Free testosterone may be calculated or injections (such as menotropin) should be added
measured using equilibrium dialysis. Genetic to hCG therapy in patients who remaining
testing should be performed for all patients with azoospermic following 6 months of hCG
azoospermia; these tests should include karyotype treatment. A small randomized controlled trial
analysis and assessment for Y-chromosome has reported that patients with a testicular volume
microdeletions and CFTR pathogenic variants. OA less than 4 mL may have better outcomes from
should be suspected if men have azoospermia an alternative treatment regimen commencing
despite having normal testicular volume and no with FSH therapy for 6 months (to aid testicular
features of hypogonadism. maturation) before starting hCG therapy.6

Table 2. World Health Organization Minimum Selective Estrogen Receptor Modulators


Thresholds of Semen Parameters in Men and Aromatase Inhibitors
Fathering Children Without Fertility Problems2 Both selective estrogen receptor modulators
Lower Reference Limit and aromatase inhibitors indirectly stimulate
Parameter (95% CI) gonadotropin secretion by reducing estrogenic
Semen volume 1.5 mL pituitary feedback in men, so therefore elevate
Sperm concentration 15 × 106 per mL
levels of serum testosterone in men with
hypogonadism while preserving spermatogenesis.7
Total motility 40%
Clomiphene citrate, anastrozole, and letrozole
Progressive motility 32% have therefore been proposed as alternatives
Sperm morphology 4% to testosterone therapy in hypogonadal men of
(normal forms) reproductive age who wish to actively conceive.
However, little safety data exist for use of selective
estrogen receptor modulators and aromatase
inhibitors in male hypogonadism, and neither
drug class is licensed for the treatment of male
hypogonadism or male infertility.8 Selective
estrogen receptor modulators and aromatase

ENDO 2021 • Reproductive Endocrinology  309

chased by Dr. Khalid H. Seedahmed, Jr., MRCP PGDip, CertEndocrinology SA - ID 283


inhibitors work by reducing negative pituitary Clinical Case Vignettes
feedback, so they are ineffective in men
with panhypopituitarism and extremely low Case 1
testosterone levels. A 38-year-old man has been on testosterone
replacement since he had delayed puberty as a
Surgical Management of Male Infertility teenager. He is taking intramuscular testosterone
Surgical techniques have a vital role in the enanthate every 4 weeks. He now has a long-term
management of obstructive infertility and an partner and they would like to start a family. On
emerging role in the management of testicular physical examination, testicular volume is 3 mL
failure. Percutaneous epididymal sperm aspiration bilaterally.
or microsurgical epididymal sperm aspiration are
highly effective techniques for collecting sperm Laboratory test results after 3 months off
in men with OA, since spermatogenesis is often testosterone therapy:
intact in affected patients. Surgical sperm retrieval LH = 0.2 mIU/mL (1.0-9.0 mIU/mL) (SI: 0.2 IU/L
may also be used to obtain sperm from men with [1.0-9.0 IU/L])
NOA or severe oligoasthenospermia, which is FSH = 1.5 mIU/mL (1.0-13.0 mIU/mL) (SI: 1.5 IU/L
otherwise insufficient for assisted reproductive [1.0-13.0 IU/L])
Serum testosterone (8 AM in fasted state) =
technologies (such as intracytoplasmic sperm 58 ng/dL (300-900 ng/dL) (SI: 2.0 nmol/L
injection. The technique of TESE may have [10.4-31.2 nmol/L])
high associated rates of hematoma and testicular
devascularization. Therefore, mTESE was Semen analysis shows azoospermia.
introduced in 1999 and is now considered the gold
standard surgical procedure for surgical sperm Which of the following statements is correct?
retrieval. mTESE allows for the microdissection A. Prior testosterone therapy impairs chance
and identification of individual tubules appearing of successful sperm induction in men with
to be engorged with spermatogenesis in situ. hypogonadism
During mTESE, tubules are dissected and B. The goal should be a sperm count in the
examined microscopically for sperm during World Health Organization reference range
surgery. Testicular samples are homogenized with (15 million/mL) to achieve spontaneous
storage medium, and either cryopreserved for pregnancy
future use or used fresh for fertilization of eggs C. Pretreatment with 6 months of FSH will
collected during a synchronized cycle of assisted increase sperm count more when compared
reproductive technology. Hormonal stimulation with starting treatment with hCG or GnRH
with gonadotropins or selective estrogen receptor
D. He should be counseled about the risk of
modulators are often used before mTESE
gynecomastia when starting hCG therapy
to maintain serum testosterone secretion in
hypogonadal men with NOA without suppressing Answer: D) He should be counseled about the risk
spermatogenesis; however, there are insufficient of gynecomastia when starting hCG therapy
interventional data to support their safety or
efficacy in treating men with NOA. Recent meta-analyses suggest that testosterone
therapy does not significantly reduce
the effectiveness of sperm induction for
hypogonadotropic hypogonadism (thus,
Answer A is incorrect).9 Couples can become
pregnant with sperm counts much lower than
the World Health Organization reference range
(thus, Answer B is incorrect). A recent article

310  ENDO 2021 • Endocrine Case Management

chased by Dr. Khalid H. Seedahmed, Jr., MRCP PGDip, CertEndocrinology SA - ID 283


suggested that recombinant FSH pretreatment The patient is not symptomatic and may undergo
may aid seminiferous tubule development.5 mTESE soon. Testosterone therapy should not be
However, no significant increase in sperm count started, otherwise any residual spermatogenesis
or testicular volume was observed in this small is likely to be suppressed (thus, Answer A is
study (thus, Answer C is incorrect). hCG may incorrect). It remains controversial whether hCG
cause gynecomastia in a minority of cases, which or selective estrogen receptor modulators increase
can occasionally be irreversible and disfiguring chances of sperm retrieval before mTESE in
(Answer D). men with Klinefelter syndrome. Sperm retrieval
following mTESE is reported widely in men with
nonmosaic Klinefelter syndrome (thus, Answer
Case 2
B is incorrect). Men with Klinefelter syndrome
A 32-year-old man has a 3-year history of should be counseled about their increased risk
infertility with his 34-year-old partner. of thromboembolic disease (Answer C). New
Investigations reveal azoospermia on 3 occasions, hypogonadism has been reported in 20% of men
and his karyotype is documented to be 47,XXY with NOA following mTESE (thus, Answer D is
(nonmosaic). He feels otherwise well and has incorrect), but the incidence has not been studied
normal libido and erectile function. previously in men with Klinefelter syndrome.
On physical examination, his height is 70.1 in
(178 cm), he has slightly reduced male-pattern hair
growth, and testicular volume is 10 mL bilaterally. Case 3  New Content 
A 36-year-old man has a 2-year history of
Laboratory test results: infertility with his 37-year-old partner. Semen
Serum LH = 10.3 mIU/mL (1.0-9.0 mIU/mL) analysis has documented azoospermia on
(SI: 10.3 IU/L [1.0-9.0 IU/L]) 3 occasions. He feels otherwise well and has
Serum FSH = 20.1 mIU/mL (1.0-13.0 mIU/mL) normal libido and erectile function.
(SI: 20.1 IU/L [1.0-13.0 IU/L])
On physical examination, his BMI is 23 kg/m2
Serum testosterone (8 AM in fasted state) =
251 ng/dL (300-900 ng/dL) (SI: 8.7 nmol/L and he has normal male-pattern hair growth.
[10.4-31.2 nmol/L])
Laboratory test results:
He is interested in learning more about surgical Serum LH = 3.3 mIU/mL (1.0-9.0 mIU/mL)
sperm retrieval. (SI: 3.3 IU/L [1.0-9.0 IU/L])
Serum FSH = 5.1 mIU/mL (1.0-13.0 mIU/mL)
Which of the following statements is correct? (SI: 5.1 IU/L [1.0-13.0 IU/L])
Serum testosterone (8 AM in fasted state) =
A. Testosterone therapy should be started
354 ng/dL (300-900 ng/dL) (SI: 12.3 nmol/L
without delay to prevent osteoporosis [10.4-31.2 nmol/L])
B. It is highly unlikely that sperm can be
retrieved with mTESE since he has nonmosaic Ultrasonography of the testes shows normal
Klinefelter syndrome testicular appearance with testicular volumes
of 22.3 mL (left) and 21.7 mL (right). There is
C. Men with Klinefelter syndrome have increased
a grade 1 varicocele on the left side. No other
surgical risk due to thrombophilia
masses are seen.
D. Men have a 5% chance of developing new
symptomatic hypogonadism following mTESE
Answer: C) Men with Klinefelter syndrome have
increased surgical risk due to thrombophilia

ENDO 2021 • Reproductive Endocrinology  311

chased by Dr. Khalid H. Seedahmed, Jr., MRCP PGDip, CertEndocrinology SA - ID 283


Which of the following is the most On physical examination, his height is 71 in
likely underlying diagnosis? (180 cm) and weight is 225 lb (102 kg) (BMI =
A. Genitourinary infection 31 kg/m2). He has normal male-pattern body hair.
B. Y-chromosome microdeletion of AZFb Testicular volume is 15 mL (right) and 8 mL (left).
subtype Pituitary MRI documents a 3-mm adenoma.
C. Mumps orchitis Laboratory test results:
D. Klinefelter syndrome LH = 1.2 mIU/mL (1.0-9.0 mIU/mL) (SI: 1.2 IU/L
[1.0-9.0 IU/L])
Answer: A) Genitourinary infection FSH = 14.5 mIU/mL (1.0-13.0 mIU/mL)
(SI: 14.5 IU/L [1.0-13.0 IU/L])
This patient has features consistent with OA; he Serum testosterone (8 AM in fasted state) =
has normal testicular volume and no evidence 482 ng/dL (300-900 ng/dL) (SI: 16.7 nmol/L
of hypogonadism. Note that the serum FSH is [10.4-31.2 nmol/L])
normal, which suggests that spermatogenesis Serum prolactin = 15.0 µg/L (4-23 ng/mL)
is present. Klinefelter syndrome (Answer D), (SI: 0.65 nmol/L [0.17-1.00 nmol/L])
Serum cortisol, normal
mumps orchitis (Answer C), and any form ACTH, normal
of Y-chromosome microdeletion (Answer B) IGF-1, normal
are therefore highly unlikely. Genitourinary Thyroid function, normal
infections (Answer A), some of which are Hemoglobin Alc = 5.1% (4.0%-5.6%) (32 mmol/mol
sexually transmitted (eg, chlamydia) may cause [20-38 mmol/mol])
Results of semen analysis (lower reference limit
inflammation, scarring, and occlusion of the [95% CI] shown in parentheses):
epididymis. Genitourinary infections are the most Semen volume = 2.3 mL (≥1.5 mL)
probable causes of OA in populations worldwide. Sperm concentration = 13.2 × 106 per mL
However, less commonly, heterozygous loss-of- (≥15 × 106 per mL)
function pathogenic variants in the CFTR gene Total motility = 17% (≥40%)
cause congenital bilateral absence of the vas Progressive motility = 11% (≥32%)
deferens. The incidence of congenital bilateral Round cells = 3.2 mill/mL (<1.0 mill/mL)
Sperm morphology (normal forms) = 0% (>4%)
absence of the vas deferens is highest in men of
white ethnicity. No imaging modality can reliably
Which of the following cause(s) could be
detect congenital bilateral absence of the vas
contributing to subfertility in this case?
deferens. Testicular examination may reveal either
absence of the vasa or epididymal swelling, but A. Pituitary microadenoma
this is highly challenging to detect. OA can only B. Testicular failure secondary to cryptorchidism
be diagnosed intraoperatively by isolating sperm C. Genitourinary infection
during epididymal sperm aspiration. D. Obesity
E. Female-factor infertility
Case 4  New Content 
Answer: B, C, D, and E) Testicular failure
A 58-year-old man presents to clinic with a 2-year secondary to cryptorchism, genitourinary
history of subfertility with his 39-year-old female infection, obesity, and female-factor infertility
partner. He has a 19-year-old daughter from a
previous relationship. He underwent puberty at This patient has a normal endocrine profile, so
a normal age. Libido and erectile function are the pituitary adenoma (Answer A) is unlikely to
normal. At age 2 years, he needed an operation be related to his presentation. This case illustrates
to keep his left testis in the scrotum. He takes no a typical case of oligoasthenoteratospermia,
regular medications. sometimes referred to as OAT syndrome by

312  ENDO 2021 • Endocrine Case Management

chased by Dr. Khalid H. Seedahmed, Jr., MRCP PGDip, CertEndocrinology SA - ID 283


urologists. The principle learning point is that smaller than normal with reduced (or absent)
multiple factors may contribute to the etiology. spermatogenesis. Obesity (Answer D) may cause
Most round cells in the seminal plasma are secondary hypogonadism. However, it may also
leukocytes, so an elevated count may indicate directly impair testicular function by inducing
genitourinary infection (Answer C). Many cases heat stress, thereby increasing risk of sperm
are asymptomatic in men, but they may respond to DNA fragmentation. Concurrent female-factor
antibiotics. Persistent cryptorchidism (Answer B) infertility (Answer E) may affect this couple’s
is typically treated with orchidopexy (to fix the chance of conception, so joint investigation is
testis to the scrotum) during the first 2 years of critical to effective fertility counseling.
life. However, the affected testis is commonly

References
1. Wall J, Jayasena CN. Diagnosing male infertility. BMJ. 2018;363:k3202. 6. Dwyer AA, Sykiotis GP, Hayes FJ, et al. Trial of recombinant follicle-
PMID: 30287677 stimulating hormone pretreatment for GnRH-induced fertility in patients
2. Boehm U, Bouloux PM, Dattani MT, et al. Expert consensus document: with congenital hypogonadotropic hypogonadism. J Clin Endocrinol Metab.
European consensus statement on congenital hypogonadotropic 2013;98(11):E1790-E1795. PMID: 24037890
hypogonadism--pathogenesis, diagnosis and treatment. Nat Rev Endocrinol. 7. Scovell JM, Khera M. Testosterone replacement therapy versus clomiphene
2015;11(9):547-564. PMID: 26194704 citrate in the young hypogonadal male. Eur Urol Focus. 2018;4(3):321-323.
3. Schlegel PN. Testicular sperm extraction: microdissection improves sperm PMID: 30131284
yield with minimal tissue excision. Hum Reprod. 1999;14(1):131-135. 8. Bhasin S, Brito JP, Cunningham GR, et al. Testosterone therapy in men
PMID: 10374109 with hypogonadism: an Endocrine Society clinical practice guideline. J Clin
4. Wosnitzer MS. Genetic evaluation of male infertility. Transl Androl Urol. Endocrinol Metab. 2018;103(5):1715-1744. PMID: 29562364
2014;3(1):17-26. PMID: 26813518 9. Rastrelli G, Corona G, Mannucci E, Maggi M. Factors affecting
5. World Health Organization. WHO laboratory manual for the examination spermatogenesis upon gonadotropin-replacement therapy: a meta-analytic
and processing of human semen. World Health Organization. study. Andrology. 2014;2(6):794-808. PMID: 25271205
Available at: http://www.who.int/reproductivehealth/publications/
infertility/9789241547789/en/. Accessed for verification January 2020.

ENDO 2021 • Reproductive Endocrinology  313

chased by Dr. Khalid H. Seedahmed, Jr., MRCP PGDip, CertEndocrinology SA - ID 283


chased by Dr. Khalid H. Seedahmed, Jr., MRCP PGDip, CertEndocrinology SA - ID 283
THYROID

chased by Dr. Khalid H. Seedahmed, Jr., MRCP PGDip, CertEndocrinology SA - ID 283


Management of Patients With
Medullary Thyroid Cancer
Rossella Elisei, MD. Department of Clinical and Experimental Medicine, Unit of
Endocrinology, University of Pisa, Pisa, Italy; E-mail: rossella.elisei@med.unipi.it

Learning Objectives the aim of performing an early thyroidectomy.


Despite a rather bad prognosis for patients with
As a result of participating in this session, learners
advanced stage cancer (M1) at diagnosis, not all
should be able to:
patients with MTC have a poor prognosis and
• Increase their ability to recognize and diagnose several factors can be helpful in providing good
medullary thyroid carcinoma (MTC). information about the prognosis. Among others, a
serum calcitonin level before surgery (<500 pg/mL
• Explain the relevance of genetic screening in
[<146 pmol/L]) and after surgery (<20 pg/mL
patients with MTC.
[<5.8 pmol/L]) portends a good prognosis, as
• Improve their knowledge of prognostic factors does the absence of laterocervical lymph node
for survival in patients with MTC. metastases at diagnosis. This information is
• Increase their ability to treat and follow-up important to better manage patients with MTC,
patients with MTC. and, in particular, to identify those whose disease
will likely progress and require systemic therapies.
• Describe the clinical impact of the approved
Two multitarget tyrosine kinase inhibitors,
receptor tyrosine kinase–targeted therapies in
vandetanib and cabozantinib, are available to
advanced MTC.
treat advanced and progressive MTC or patients
• Discuss the clinical trials of new RET target with symptomatic MTC. Phase 3 clinical trials
therapies. with these 2 drugs, the ZETA and EXAM
trials, demonstrated significant prolongation
of progression-free survival in patients treated
with the drug compared with survival in those
Main Conclusions treated with placebo. Despite these good results,
MTC is a rare cancer whose cytologic diagnosis patients with MTC can experience progression
represents a challenge for pathologists. However, while being treated with these drugs and they need
serum calcitonin, when elevated, can be helpful other therapeutic options. New monotarget RET
in making a correct diagnosis. Early diagnosis, inhibitors, LOXO 292 and BLUE 667, are being
when MTC is still intrathyroidal, is fundamental evaluated in very promising clinical trials.
to definitively cure the patient. Because 6% to
10% of MTC cases that present as sporadic are
indeed hereditary, all patients with MTC should Significance of the
undergo RET genetic screening. All first-degree
relatives of patients with an identified pathogenic
Clinical Problem
variant should undergo genetic testing. Family MTC is a rare cancer deriving from
members who test positive will either immediately neuroendocrine C cells. It can be sporadic or
be treated or be kept under surveillance with hereditary, and in both cases early diagnosis

316  ENDO 2021 • Endocrine Case Management

chased by Dr. Khalid H. Seedahmed, Jr., MRCP PGDip, CertEndocrinology SA - ID 283


followed by early treatment is the only way to detectable if immunohistochemistry with a
guarantee definitive cure. Knowledge about calcitonin antibodies is performed. Because of its
this malignancy among endocrinologists must origin, MTC is considered a separate entity from
be increased to improve early identification of other differentiated thyroid carcinomas.
affected patients and refer them to specialized The incidence of MTC is unknown.
centers. It accounts for 5% to 10% of all thyroid
Patients who are diagnosed later in the disease malignancies,2 is present in 0.4% to 1.4% of all
course will never be cured and must be followed thyroid nodules, and is present in less than 1%
with periodic examinations and measurement of of thyroid glands of individuals at autopsy. The
serum markers (ie, calcitonin and CEA), and treating National Institutes of Health recognizes MTC
clinicians must interpret the clinical meaning of as a rare disease. The median age at diagnosis is
their values and, in particular, of observed changes. 45 to 55 years, but there is a wide range in terms
Targeted therapies have a significant impact on of age of onset. No difference in sex distribution
progression-free survival in patients with MTC, has been reported. No specific risk factors for the
but the administration of these medications requires development of MTC have been discovered.
that the prescribing endocrinologist have specific One peculiarity of MTC is that it can be
knowledge and competencies in the management of sporadic or arise in the context of familial
their adverse events. syndromes (75% and 25% of cases, respectively).
When sporadic, it affects 1 single person in the
kindred, while in familial cases, a dominant
Barriers to Optimal Practice mendelian autosomal transmission of the disease
is observed, often associated with other endocrine
• Difficulty in making a cytologic diagnosis due
neoplasias such as pheochromocytoma and/
to the nonspecific cytologic features.
or hyperparathyroidism due to parathyroid
• Incurable disease when the diagnosis is hyperplasia or multiple adenomatosis. These
delayed, particularly when the tumor is already syndromes are known as multiple endocrine
extrathyroidal. neoplasia and are classified into 3 phenotypes
• Rarity of the disease requires treatment in according to the combination of endocrine
referral centers. neoplasia and other nonendocrine findings. Only
• Few therapeutic options for advanced, the hereditary form of MTC affects children.
progressive, or symptomatic disease. Generally, multiple endocrine neoplasia type
2B is the most aggressive form with the earliest
presentation.3
The pathogenesis of MTC has been extensively
Strategies for Diagnosis, studied in the last 25 years after the recognition
Therapy, and/or Management that activation of the RET proto-oncogene was
Epidemiology and Pathogenesis strongly correlated with the development of
the hereditary form of MTC.4 More recently, a
MTC is a neuroendocrine tumor originating
very important role of RET pathogenic variants
from the parafollicular C cells, which are localized
has been demonstrated in the sporadic form of
in the thyroid and secrete calcitonin and other
the disease as well, since about 50% of cases are
peptides such as chromogranin, serotonin,
positive for a somatic RET pathogenic variant
somatostatin, carcinoembryonic antigen (CEA), or
that is related to greater aggressiveness and poor
calcitonin-gene related peptide.1 The parafollicular
outcome.5 No data are available regarding the
C cells represent only 0.1% of all thyroid cells
possible causes of induction of RET pathogenic
and, normally, they are not visible on standard
variants.
hematoxylin-eosin tissue staining, but become

ENDO 2021 • Thyroid  317

chased by Dr. Khalid H. Seedahmed, Jr., MRCP PGDip, CertEndocrinology SA - ID 283


Prognostic Factors 10% of patients with multiple endocrine neoplasia
type 2A), is almost exclusively found in individuals
The clinical behavior of MTC is much less
with this syndrome.
favorable than that of other well-differentiated
As above mentioned, sporadic MTC appears
thyroid carcinomas, although it is not as
as a thyroid nodule, single or in the context of
unfavorable as that of anaplastic carcinoma. A
a multinodular goiter. Physical examination of
10-year survival for about 50% of patients with
the neck does not offer any significant diagnostic
MTC is reported in several series. Disease stage
elements, especially now that most nodules are
at diagnosis is the most important prognostic
not palpable and are incidentally discovered by
factor for both cure and survival, as demonstrated
neck ultrasonography often performed for other
by the evidence that if an early diagnosis is
purposes. Once the nodule has been discovered,
made, possibly when the neoplastic disease is
a classic workup for thyroid nodular disease is
still intrathyroidal, 90% of patients survive up
then performed. Neck ultrasonography usually
to 35 years. Also, serum calcitonin can have a
shows a nodule with an intermediate- or high-
prognostic role. If the calcitonin value is less than
suspicion pattern of malignancy but without any
500 pg/mL (<146 pmol/L) before surgery, there is
specific or peculiar feature to prompt specific
a good probability of survival. Clinical remission
suspicion of MTC. FNA cytology with standard
is almost 100% when calcitonin decreases to
staining does not always show the typical MTC
less than 20 pg/mL (<5.8 pmol/L) after surgery.
“salt and pepper” and “plasmacytoid” appearance
Also, the presence of lymph node metastases in
of cells. Cell shape varies from oval to round,
the laterocervical neck region, but not those in
and they can be large polygonal or spindled
the central compartment, is considered a bad
and are usually isolated. Although the cytologic
prognostic factor for both persistence of disease
pattern of MTC is generally typical, several series
and survival.6
show a high percentage of failure in making a
presurgical diagnosis by FNA cytology. Calcitonin
Diagnosis is the most specific and sensitive MTC marker,
The most common clinical presentation of both before and after thyroidectomy. It is a
sporadic MTC is a thyroid nodule, either single small polypeptide hormone of 32 amino acids
or belonging to a multinodular goiter. With the normally produced almost exclusively by C cells.
exception of the simultaneous presence of diarrhea Several studies have demonstrated that routine
and/or flushing syndrome, which is rare and measurement of serum calcitonin is the most
usually related to advanced metastatic disease, accurate diagnostic tool for the detection of MTC
there are no symptoms or signs that prompt in patients with thyroid nodules. In all series,
suspicion for MTC. the sensitivity of serum calcitonin was more
When a family history of MTC is present or accurate than that of cytology.7,8 Nevertheless,
the patient has already had a pheochromocytoma or there are still controversial opinions on the role
hyperparathyroidism, the suspicion for hereditary of routine measurement of basal serum calcitonin
MTC is more likely. Mucosal neuromas on the in the workup of thyroid nodules. If the routine
tongue and/or in the conjunctiva, marfanoid measurement of serum calcitonin is not adopted
habitus, and/or skeletal alterations suggest the in the clinical workup of thyroid nodules, it could
diagnosis of multiple endocrine neoplasia type 2B be helpful at least when surgical treatment is
and should prompt evaluation for MTC. Similarly, planned for a nodule with indeterminate cytology.
the presence of an interscapular cutaneous itchy An elevated level is also helpful with malignant
lesion (ie, cutaneous lichen amyloidosis) is also cytology to better guide surgical decision making
highly suggestive of multiple endocrine neoplasia of whether to perform total thyroidectomy and
type 2A because this lesion, although rare (affects dissection of the central neck node compartment.

318  ENDO 2021 • Endocrine Case Management

chased by Dr. Khalid H. Seedahmed, Jr., MRCP PGDip, CertEndocrinology SA - ID 283


Diagnostic Tools When Serum to perform this procedure: (1) the discovery of
Calcitonin Is In the “Grey Zone” a somatic pathogenic variant (usually in 45% of
cases) confirms the sporadic nature of the tumor;
Elevated basal serum calcitonin, especially (2) the prognostic value of the presence/absence
a medium-low level (ie, <100 pg/mL of the somatic pathogenic variant; (3) the future
[<29.2 pmol/L]), should not be immediately possibility for patients with RET pathogenic
considered to be diagnostic of MTC. These variants to be treated with drugs specifically aimed
medium-low levels can be further investigated by at inhibiting the altered RET gene.
performing a stimulation test with the intravenous Once an index case has been confirmed to
administration of calcium to confirm that the be hereditary, all first-degree relatives should be
calcitonin is really secreted by MTC: a significant tested for the presence of the same pathogenic
increase in serum calcitonin (usually 3 to 4 times variant. Relatives who do not carry the pathogenic
the basal level) is observed in patients with variant can avoid being monitored over the years,
MTC, but not in those with elevated basal serum as they are not at increased risk to develop MTC.
calcitonin derived from other sources or artifacts. However, relatives who carry the pathogenic
If the cytologic diagnosis is not definitive for variant should undergo clinical, biochemical
MTC and the serum calcitonin value is in the grey (ie, basal calcitonin measurement), and imaging
zone of the assay (ie, >20 pg/mL [>5.8 pmol/L] assessments (neck ultrasonography) for early
but <100 pg/mL [<29.2 pmol/L]), the calcitonin discovery of the disease. If both basal calcitonin
measurement in the washout of the needle used for and neck ultrasonography are negative, a
the FNA cytology of the suspected thyroid nodule calcium-stimulation test for calcitonin is usually
may be crucial. This approach is of particular recommended.12 The therapeutic strategies
diagnostic utility to ascertain the nature of neck and follow-up protocols for family members
lymph nodes, especially before thyroidectomy, to carrying pathogenic variants have been changed
plan the surgical approach or the most appropriate over the years. Immediately after the discovery
therapeutic strategies.9 that RET pathogenic variants were responsible
for the hereditary forms, the surgical practice
Genetic Screening of prophylactic thyroidectomy in all children
After 20 years since the discovery that germline with pathogenic RET variants was strongly
RET oncogene pathogenic variants are responsible recommended. It is clear that RET pathogenic
for hereditary cases of MTC, it is now recognized variants vary in terms of penetrance and the age of
that RET genetic screening must be performed MTC development. The M918T mutation and the
in all patients with MTC, even in the absence noncysteine mutations (ie, V804M, S891A, L790F,
of a family history of thyroid cancer.10 The etc) are the most and the least transforming and
rationale for this screening is that 5% to 10% of aggressive, respectively.13 For these reasons, the
patients with apparently sporadic MTC harbor a American Thyroid Association guidelines classify
germline RET pathogenic variant—either de novo pathogenic variants into different classes of risk,
or misdiagnosed familial cases.11 This finding is and the timing of thyroidectomy in carriers should
of great relevance for the early identification of be based on several factors, including the level of
carriers in the family who are unaware of their risk of the pathogenic variant and the basal and
condition and are sometimes already affected. stimulated value of serum calcitonin. Periodic
Although not yet introduced as a standard of evaluation of serum calcitonin, both basal and
care, RET gene analysis would also be useful if stimulated, may be very useful in identifying
performed in the tumoral tissue, either in fresh or the right time to perform an early and safe
paraffin-embedded tumoral tissue, of ascertained thyroidectomy.2
sporadic cases. There are at least 3 main reasons

ENDO 2021 • Thyroid  319

chased by Dr. Khalid H. Seedahmed, Jr., MRCP PGDip, CertEndocrinology SA - ID 283


Follow-Up and Therapies been approved by both the FDA and European
for Advanced Disease Medicines Agency for treatment of these
patients after the phase 3 clinical trials, ZETA
Independent from the sporadic or hereditary and EXAM.15,16 Although they have different
nature of MTC in a given patient, 3 months after patterns, both vandetanib and cabozantinib
surgery, every patient should be evaluated with are small molecules able to block multiple
physical examination, neck ultrasonography, tyrosine kinases with different activities. These
and measurement of serum free T3, free T4, medications should be started if the progression
TSH, calcitonin, and CEA. Serum calcitonin is of the disease, as assessed according to Response
the specific marker to be used for the follow- Evaluation Criteria in Solid Tumors (RECIST),
up of patients with MTC. CEA measurement is has been documented in the last 12 months or
relevant because it provides a better idea about the according to clinical judgment in very advanced
tumoral burden and the degree of differentiation cases. The choice of one or the other drug is
of the tumor since, when dedifferentiated, serum very much dependent on the availability of the
calcitonin can be relatively low and CEA rather drug, since not all countries have both drugs
elevated. approved and reimbursed. However, in those
When patients are not cured by the primary countries where either can be prescribed, the
surgical treatment, which should include total choice must take into consideration the patient’s
thyroidectomy and central neck lymph node clinical features and each drug’s characteristics.
dissection, other therapeutic procedures are According to the results of the phase 3 studies,
indicated depending on the localization and the the effect of cabozantinib seems to be more rapid
number of lesions. In planning a therapeutic but with a safety profile more demanding in its
strategy, clinicians should take into account that management. Cabozantinib has also been tested
most distant metastases found during follow-up in patients who had been previously treated with
are small at the time of their recognition and that another tyrosine kinase inhibitor, and the results
their growth is usually rather slow. These lesions showed that it works in terms of prolongation
are compatible with a long period of good quality of progression-free survival, thus suggesting
of life. In these cases, an aggressive therapeutic that it can be used as second-line therapy. This
approach may not be indicated, unless an evident information is unavailable for vandetanib, which
rapidly progressive disease is demonstrated. is apparently more manageable but slower in
New drugs, tyrosine kinase inhibitors, have been determining disease control. Vandetanib, but
approved for the treatment of advanced and not cabozantinib, has also been tested in children
progressive MTC. Nevertheless, before starting with a dedicated scheme of administration, and
these systemic therapies, some considerations the results show that it is safe and effective in
must given to the number, size, and growth childhood MTC.17 Moreover, there are several
rate of the lesions. Whenever possible, local reported cases whereby the ectopic ACTH
treatment is preferred and systemic therapy should secretion and the consequent paraneoplastic
be postponed depending on the evidence for Cushing syndrome, which is frequently present
multimetastatic progressive disease.14 when the disease is multimetastatic and advanced,
Current systemic therapeutic options for is reverted and cured by the administration of
advanced and progressive MTC are represented vandetanib. Vandetanib cannot be used in patients
by targeted tyrosine kinase inhibitor therapies who have a prolonged QTc interval (>450 ms in
specifically directed against signal transduction men and >470 ms in women), while cabozantinib
pathways and or genetic alteration of MTC. does not have this limitation. Adverse effects are
Although several tyrosine kinase inhibitors have similar, but the prevalence of each one of them
been tested in advanced and progressive MTC, is different according to the drug that is used.
only 2 drugs, vandetanib and cabozantinib, have

320  ENDO 2021 • Endocrine Case Management

chased by Dr. Khalid H. Seedahmed, Jr., MRCP PGDip, CertEndocrinology SA - ID 283


Despite the fact that a significant advantage in Clinical Case Vignettes
prolonging the progression-free survival time has
been demonstrated by both drugs, so far neither Case 1
has shown an increase in overall survival. A 43-year-old woman notes a right neck lump.
Both drugs, as are all tyrosine kinase Neck ultrasonography reveals a suspicious
inhibitors, are cytostatic and not cytotoxic; thus, right 2.4-cm thyroid nodule. She has no local
they can stop cell growth, but they do not kill cells. symptoms. Neck ultrasonography does not show
For this reason, they must be continued until there any abnormal lymph nodes. The left lobe had
is evidence of clinical benefit. However, from the a subcentimeter nonsuspicious nodule. FNA
results of the 2 studies, it is clearly evident that cytology is THY III (atypia of undetermined
sooner or later a sort of resistance develops and significance or follicular lesion of undetermined
the clinician must decide to continue or stop the significance [AUS/FLUS]). Thyroid function is
drug. Further studies to analyze the possibility normal without evidence of thyroid antibodies,
of using the 2 drugs in an alternating modality, and the serum calcitonin concentration is
combining them, or using them with other 45 pg/mL (13.1 pmol/L).
drugs, either working via the same mechanism
or by modulating the immune system, will Which of the following is the best next step?
be the next challenge of the future. There are A. Wait and see; perform clinical and biochemical
already 2 new RET-specific inhibitors, LOXO 29218 examination every 6 months
and BLUE 667, that are characterized by good B. Total thyroidectomy with central neck
efficacy similar to that of the tyrosine kinase dissection
inhibitors, with a much smaller impact on quality
C. Lobectomy for diagnostic purpose
of life.
D. Repeated FNA for immunocytochemistry for
calcitonin and measurement of calcitonin in
Conclusions the needle washout
Early diagnosis of MTC, when the tumor is still
intrathyroidal, is the only way to guarantee that Answer: D) Repeated FNA for immunocytochemistry
surgery can cure the patient. Measurement of for calcitonin and measurement of
serum calcitonin can be helpful in making an early calcitonin in the needle washout
diagnosis. However, not all cases of MTC are When the serum calcitonin concentration is
aggressive and lethal. There are several markers of greater than 20 pg/mL but less than 100 pg/mL
a bad prognosis: (1) advanced stage at diagnosis; (5.8-29.2 pmol/L), it is not immediately diagnostic
(2) preoperative serum calcitonin values greater of MTC, but it should serve as an alert that
than 500 pg/mL and/or greater than 20 pg/mL MTC might be present and should prompt other
after surgery; (3) the presence of laterocervical diagnostic procedures. Other than these 2 above-
metastatic lymph nodes; and (4) RET somatic mentioned tools, a calcium-stimulation test can
pathogenic variants. be performed since “true” calcitonin secreted by
Considering the rarity of the disease, the MTC shows a very high increase after stimulation,
difficulty of making an early diagnosis, the while calcitonin from other origins (ie, other
possibility that MTC can be familial, and the fact neuroendocrine tumors, ectopic production,
that management of persistent disease can be interference from heterophilic antibodies, etc)
challenging, it is recommended that patients with does not respond to the stimulus.
MTC be followed by a multidisciplinary team in a
specialized center.

ENDO 2021 • Thyroid  321

chased by Dr. Khalid H. Seedahmed, Jr., MRCP PGDip, CertEndocrinology SA - ID 283


Case 2 pathogenic variant since there are patients who
are positive despite their apparently sporadic
A 56-year-old man undergoes workup for a
nature. Once the case has been ascertained to
multinodular goiter with 2 major thyroid nodules
be hereditary and a RET pathogenic variant is
of 1.8 and 2.9 cm. The cytology of the 2.9-cm
identified, his relatives can undergo testing for the
nodule shows the presence of MTC, and the serum
same pathogenic variant to identify those at risk to
calcitonin value is 420 pg/mL (122.6 pmol/L).
develop MTC (Answer A).
Neck ultrasonography does not show any lymph
node metastases. The patient undergoes total
thyroidectomy and central neck dissection. Case 3
Histology confirms MTC in the 2.9-cm nodule A 63-year-old man with a long history of MTC
associated with widespread C-cell hyperplasia with has undergone surveillance for many years. He
no lymph node metastases. The patient asks about has several mediastinal metastatic lymph nodes,
his prognosis and, being aware (from Dr Google) the largest of which is 23 mm in diameter, and
that MTC can be hereditary, he wonders whether several nodules in the lungs are very suspicious for
his relatives should have testing. metastases. For several years, CT showed stable
disease. His serum calcitonin concentration varied
Which of the following statements is correct? from 300 to 400 pg/mL (87.6-116.8 pmol/L),
A. His prognosis is good with a high probability and his CEA concentration varied from
for long survival: RET genetic screening should 40 to 60 ng/mL. The last 2 serum calcitonin
be first performed in the patient himself measurements were 630 pg/mL and 780 pg/mL
B. The prognosis of MTC is always severe and (184.0 and 228.0 pmol/L), and the CEA values
his relatives should be immediately screened were 68 ng/mL and 85 ng/mL. Current CT reveals
for serum calcitonin increased size of all already known metastatic
C. His prognosis is good and since there is no lesions (both lymph nodes and lung lesions) and a
history of other familial cases, there is no need 50% increase of liver lesions that were previously
to screen his relatives described as subcentimetric angiomas.
The patient has been in good general health
D. His prognosis depends on the sporadic or
except for the recent development of diarrhea.
hereditary nature of the disease, and his
relatives should be screened only if the
Which of the following is the best next
hereditary nature is confirmed
step in this patient’s management?
Answer: A) His prognosis is good with a high A. Start intravenous chemotherapy
probability for long survival: RET genetic screening B. Start a tyrosine kinase inhibitor
should be first performed in the patient himself C. Wait and see
The prognosis of MTC can widely vary according D. Start external radiotherapy targeting the
to the extension of the disease at the time of the mediastinal lymph nodes
diagnosis. Usually cases that are still intrathyroidal
have a good prognosis and a preoperative Answer: B) Start a tyrosine kinase inhibitor
serum calcitonin value less than 500 pg/mL Although advanced MTC is often rather indolent,
(<146 pmol/L) is consistent with a good prognosis, having a very long progression time, a short
at least in terms of long survival. The prognosis is doubling time of serum calcitonin and CEA
not influenced by the sporadic or hereditary nature suggests that the disease is rapidly growing.
of MTC, but all patients with MTC, regardless Moreover, the evidence, according to RECIST
of the familial or personal medical history, must (Response Evaluation Criteria in Solid Tumors),
be screened for the presence of a germline RET of progressive disease involving all the lesions

322  ENDO 2021 • Endocrine Case Management

chased by Dr. Khalid H. Seedahmed, Jr., MRCP PGDip, CertEndocrinology SA - ID 283


is sufficient to indicate that it is now time to prolong the time of progression-free survival.
start a systemic therapy. Currently, this would Since the patient is also symptomatic, vandetanib
be a new target therapy, such as vandetanib and should be the first choice because the ZETA
cabozantinib, able to simultaneously inhibit trial demonstrated its ability to work in
several tyrosine kinase receptors (Answer B). Both symptomatic patients.
drugs have been demonstrated to significantly

References
1. Williams ED: Histogenesis of medullary carcinoma of the thyroid. J Clin 10. Romei C, Tacito A, Molinaro E, et al. Twenty years of lesson learning: how
Pathol. 1966;19(2):114-118. PMID: 5948665 does the RET genetic screening test impact the clinical management of
2. Wells SA Jr, Asa SL, Dralle H, et al; American Thyroid Association medullary thyroid cancer? Clin Endocrinol (Oxf). 2015;82(6):892-899. PMID:
Guidelines Task Force on Medullary Thyroid Carcinoma. Revised American 25440022
Thyroid Association guidelines for the management of medullary thyroid 11. Wiench, M, Wygoda Z, Gubala E, et al. Estimation of risk of inherited
carcinoma. Thyroid. 2015;25(6):567‐610. PMID: 25810047 medullary thyroid carcinoma in apparent sporadicpatients. J Clin Oncol.
3. Raue F, Frank-Raue K. Epidemiology and clinical presentation of medullary 2001;19(5):1374-1380. PMID: 11230481
thyroid carcinoma. Recent Results Cancer Res. 2015;204:61-90. PMID: 12. Elisei R, Alevizaki M, Conte-Devolx B et al. 2012 European Thyroid
26494384 Association guidelines for genetic testing and its clinical consequences in
4. Eng C, Mulligan LM. Mutations of the RET proto-oncogene in the multiple medullary thyroid cancer. Eur Thyroid J. 2013;1(4):216-231. PMID: 24783025
endocrine neoplasia type 2 syndromes, related sporadic tumours, and 13. Romei C, Ciampi R, Elisei R. A comprehensive overview of the role
hirschsprung disease. Hum Mutat. 1997;9(2):97-109. PMID: 9067749 of the RET proto-oncogene in thyroid carcinoma. Nat Rev Endocrinol.
5. Elisei R, Cosci B, Romei C, et al. Prognostic significance of somatic RET 2016;12(4):192-202. PMID: 26868437
oncogene mutations in sporadic medullary thyroid cancer: a 10-year follow- 14. Schlumberger M, Brose M, Elisei R et al. Definition and management of
up study. J Clin Endocrinol Metab. 2008;93(3):682-687. PMID: 18073307 radioactive iodine-refractory differentiated thyroid cancer. Lancet Diabetes
6. Raue F, Kotzerke J, Reinwein D, et al. Clin Investig. Prognostic factors in Endocrinol. 2014;2(5):356-358. PMID: 24795243
medullary thyroid carcinoma: evaluation of 741 patients from the German 15. Elisei R, Schlumberger MJ, Müller SP, et al. Cabozantinib in progressive
Medullary Thyroid Carcinoma Register. 1993;71(1):7-12. PMID: 8095831 medullary thyroid cancer [published correction appears in J Clin Oncol.
7. Hahm JR, Lee MS, Min YK, et al. Routine measurement of serum calcitonin 2014;32(17):1864]. J Clin Oncol. 2013;31(29):3639-3646. PMID: 24002501
is useful for early detection of medullary thyroid carcinoma in patients with 16. Wells SA Jr, Robinson BG, Gagel RF, et al. Vandetanib in patients with
nodular thyroid diseases. Thyroid. 2001;11(1):73-80. PMID: 11272100 locally advanced or metastatic medullary thyroid cancer: a randomized,
8. Essig GF Jr, Porter K, Schneider D, et al. Fine needle aspiration and double-blind phase III trial. J Clin Oncol. 2012;30(2):134-141. PMID: 22025146
medullary thyroid carcinoma: the risk of inadequate preoperative evaluation 17. Fox E, Widemann BC, Chuk MK, et al. Vandetanib in children and
and initial surgery when relying upon FNAB cytology alone. Endocr Pract. adolescents with multiple endocrine neoplasia type 2B associated medullary
2013;19(6):920-927. PMID: 23757627 thyroid carcinoma. Clin Cancer Res. 2013;19(15):4239-4248. PMID: 23766359
9. Elisei R. Routine serum calcitonin measurement in the evaluation of thyroid 18. Subbiah V, Velcheti V, Tuch BB, et al. Selective RET kinase inhibition for
nodules. Best Pract Res Clin Endocrinol Metab. 2008;22(6):941-953. PMID: patients with RET-altered cancers. Ann Oncol. 2018;29(8):1869-1876. PMID:
19041824 29912274

ENDO 2021 • Thyroid  323

chased by Dr. Khalid H. Seedahmed, Jr., MRCP PGDip, CertEndocrinology SA - ID 283


When Thyroid Function Test
Results Don’t Make Sense
Mark Gurnell, MD, PhD. Wellcome Trust-MRC Institute of Metabolic Science,
University of Cambridge and Addenbrooke’s Hospital, Cambridge, United Kingdom;
E-mail: mg299@medschl.cam.ac.uk

Learning Objectives • HPT axis physiology and the mechanisms


governing thyroid hormone action at a cellular
As a result of participating in this session, learners
level.
should be able to:
• Key principles and methodologies
• Recognize the potential causes of discordant or underpinning commonly used thyroid
anomalous thyroid function test (TFT) results. hormone and TSH assays.
• Explain the basis of commonly used laboratory • Patterns of abnormal TFT results.
assays for measuring thyroid hormones and
• Roles of dynamic tests, radiologic
TSH and how assay interference can arise.
investigation, and genetic testing in the
• Adopt a systematic approach to the patient diagnosis of rare genetic and acquired
with discordant or anomalous TFT results to disorders of the HPT axis.
allow accurate diagnosis in a timely manner
and avoid inappropriate investigation and
treatment.
Significance of the
Clinical Problem
Main Conclusions TFTs are among the most commonly requested
laboratory investigations (~10 million TFTs are
Discordant or anomalous TFT results may arise in ordered each year in the United Kingdom to detect
the context of: thyroid dysfunction in ~4% of adults). While many
• Confounding intercurrent illness and/or of these tests yield results that are straightforward
medication use. to interpret, an important subgroup returns
apparently discordant or anomalous results that at
• Erratic or poor adherence to levothyroxine
first glance “don’t make sense.” When this occurs,
therapy.
the endocrinologist is required to disentangle
• Interference in commonly used laboratory abnormal results that are a consequence of
assays for thyroid hormone or TSH confounding intercurrent illness or medications
measurement. from those due to laboratory artefacts, while
• Rare genetic or acquired disorders of the bearing in mind the possibility of rarer but
hypothalamic-pituitary-thyroid (HPT) axis. important genetic or acquired disorders of the
HPT axis.
Successful resolution of discordant or Raised thyroid hormones (T4, T3) with
anomalous TFT results is dependent on a sound nonsuppressed TSH levels are an important, and
understanding of: relatively commonly encountered, pattern of

324  ENDO 2021 • Endocrine Case Management

chased by Dr. Khalid H. Seedahmed, Jr., MRCP PGDip, CertEndocrinology SA - ID 283


discordant TFTs; many cases can be explained by Consider Potential
confounding intercurrent illness, concomitant Confounding Factors1,2
medication use, or analytic interference in thyroid
hormone or TSH assays, but differentiating It is important to pay attention to the clinical
between these and other rarer causes (eg, context, as several factors may have particular
resistance to thyroid hormone, TSH-secreting relevance for the interpretation of TFTs,
pituitary tumor) can be difficult. including:
Misdirected investigation of these entities • Age: Many laboratories do not report age-
results in wastage of resources and/or incorrect specific reference ranges, with potential
therapeutic intervention. implications for interpreting TFT results at
the extremes of age.
Barriers to Optimal Practice • Pregnancy: Thyroid hormone and TSH levels
vary during normal pregnancy, and trimester-
• Limited awareness and knowledge of the specific reference ranges should be used
diverse array of conditions that can give rise to wherever possible.
discordant or anomalous TFT results.
• Levothyroxine therapy: Levothyroxine
• Failure to recognize the inherent susceptibility replacement in a physiological dosage to
of commonly used laboratory assays (for optimize TSH may be associated with mildly
T4, T3, and TSH measurement) to analytical elevated free T4 (but normal free T3) levels
interference. in some patients. Owing to their differing
• Relative rarity of genetic and acquired half-lives, intermittent hormone ingestion
disorders of the HPT axis such as resistance to may result in normal or even elevated thyroid
thyroid hormone and TSH-secreting pituitary hormone levels but fails to normalize TSH.
tumors—distinguishing between these • Confounding medications/supplements:
conditions is often challenging, and typically Medications and supplements have the
requires a combination of investigations, potential to alter thyroid physiology (eg,
several of which are only rarely performed in amiodarone) or cause artifactual laboratory
routine clinical endocrine practice. results (eg, heparin or biotin—see below).
• Nonthyroidal illness: Several different TFT
patterns may be seen during nonthyroidal
Strategies for Diagnosis, illness depending on the stage of the illness and
Therapy, and/or Management laboratory assays used (Figure 1).
Review the Causes of
Different TFT Patterns1,2 Exclude Laboratory
Recognition of the myriad causes of different Assay Interference3-5
patterns of TFT results is an essential first step in
the investigation and management of TFT results TSH Measurement
that “don’t make sense.” The most important Most TSH assays use a “sandwich” format with
conditions to consider are shown in Figure 1, along 2 antibodies—capture and (labeled) detection—
with those that account for most cases of classic directed against different epitopes on TSH,
primary hypothyroidism and thyrotoxicosis. with the TSH moiety acting as a bridge between
the them. The presence of human anti-animal
antibodies in a patient’s serum can interfere with
TSH measurement if directed against the same

ENDO 2021 • Thyroid  325

chased by Dr. Khalid H. Seedahmed, Jr., MRCP PGDip, CertEndocrinology SA - ID 283


Figure 1. Different Patterns of Thyroid Function Tests and Their Causes

• Graves disease
• Toxic multinodular goiter
• Toxic adenoma
• Thyroiditis (post-viral, post-partum)
• Drugs (amiodarone); excess iodine intake
• Excess thyroxine ingestion
• Pregnancy-related (hyperemesis gravidarum; hydatidiform mole)
• Congenital hyperthyroidism

• Subclinical hyperthyroidism
• Recent treatment for • NTI
hyperthyroidism • Central hypothyroidism
• Drugs (eg, steroids, dopamine) • Isolated TSH deficiency
• Assay interference • Assay interference
• NTI

• Subclinical hypothyroidism • Assay interference; FDH


• Poor adherence to levothyroxine • Levothyroxine replacement
• Malabsorption of levothyroxine therapy (including poor
• Drugs (eg, amiodarone) adherence)
• Assay interference • Drugs (eg, amiodarone,
• NTI recovery phase heparin)
• TSH resistance • NTI (including acute
psychiatric disorders)
• Neonatal period
• Autoimmune thyroiditis (Hashimoto, • TSH-secreting pituitary
atrophic) adenoma
• Post-radioiodine therapy/thyroidectomy • Resistance to thyroid hormone
• Hypothyoid phase of thyroiditis • Disorders of thyroid hormone
• Drugs (amiodarone, lithium, TKIs, ATDs) transport or metabolism
• Iodine deficiency or excess
• Neck irradiation
• Riedel thyroiditis
• Thyroid infiltration (tumor, amyloid)
• Congenital hypothyroidism

Abbreviations: ATDs, antithyroid drugs; FDH, familial dysalbuminemic hyperthyroxinemia; FT3, free triiodothyronine; FT4, free thyroxine;
NTI, nonthyroidal illness; TKI, tyrosine kinase inhibitors.

species as the assay antibodies. Thus, a human • Nonlinear TSH measurement following
anti-animal antibody that blocks TSH binding to sample dilution.
either capture or detection antibodies will result in • Altered TSH result following
“negative interference,” causing a falsely low TSH immunosubtraction (using PEG or protein
value. Conversely, a human anti-animal antibody G/A).
that is capable of cross-linking the capture
and detection antibodies may cause “positive Free T4 and Free T3 Measurement
interference,” leading to a falsely high TSH value. The relatively small size of T4 (and T3) precludes
Heterophile (weak, polyspecific) antibodies use of a “sandwich” assay format, so competition
may cause similar interference. Laboratory assays are commonly used. Here, labeled T4 (the
strategies for confirming interference include the tracer) competes with serum T4 for a fixed number
demonstration of: of T4 antibody-binding sites. Free hormone assays
are designed such that the equilibrium between
• Discordant TSH results in different assays that T4 and its binding proteins is conserved during
use different antibody pairs/methodology.

326  ENDO 2021 • Endocrine Case Management

chased by Dr. Khalid H. Seedahmed, Jr., MRCP PGDip, CertEndocrinology SA - ID 283


measurement, so the amount of tracer displaced Consider Specific Conditions
reflects the “free” rather than “total” hormone
Once potential confounding factors and artifactual
concentration. The presence of factors in serum
results have been excluded, then consideration can
that affect this equilibrium will confound hormone
be given to the investigations required to diagnose
measurement. Examples include:
rarer genetic and acquired disorders, such as
• Fractionated and unfractionated heparin: both resistance to thyroid hormone and TSH-secreting
can cause an artifactual elevation in measured pituitary tumors, which often present a specific
concentrations of free T4 and free T3 by challenge.
displacement of T4 and T3 from their carrier
proteins; the mechanism is poorly understood, Resistance to Thyroid Hormone vs
but it is likely to involve generation of free TSH-Secreting Pituitary Tumor6-10
fatty acids via heparin-mediated activation Resistance to thyroid hormone (estimated
of endothelial lipoprotein lipase, with free incidence 1 in 40,000 to 50,000 live births) and
fatty acids displacing thyroid hormones from TSH-secreting pituitary tumors (estimated
albumin. prevalence 1 to 3 per million) occur in patients
of a similar age range and either gender. A subset
• Iodothyronine antibodies that bind the tracer. of patients with predominant central/pituitary
• Human anti-animal antibodies or heterophilic resistance also exhibit thyrotoxic symptoms
antibodies that block the assay antibody. and signs, such that these features are not
• Variant thyroid hormone–binding proteins: discriminatory. An algorithm for distinguishing
dominantly-inherited genetic variants resistance to thyroid hormone and TSH-secreting
of albumin (familial dysalbuminemic pituitary tumors is shown in Figure 2.
hyperthyroxinemia [FDH] or transthyretin However, several potential pitfalls should be
[transthyretin-associated hyperthyroxinemia kept in mind, including:
(TTR-AH)]), may alter affinity for • Serum α-subunit: Elevated serum α-subunit
iodothyronines, and can cause free T4 (and levels are also found in nonfunctioning and
less frequently free T3) to be overestimated, GH-secreting pituitary tumors, while normal
particularly in “1-step” analogue hormone levels are a recognized finding in TSH-
assays. secreting microadenomas.
The use of a 2-step (“back titration”) assay method • SHBG: falsely low levels can occur in mixed
(which is less susceptible to such interference), GH/TSH-secreting tumors due to inhibition of
equilibrium dialysis, or mass spectrometry can be SHBG synthesis by GH; conversely, synthetic
useful in confirming or excluding this possibility. estrogen therapy in resistance to thyroid
hormone can falsely elevate SHBG.
Biotin (Vitamin B7)
• Pituitary imaging: TSH-secreting
Biotin used either as a medication or dietary
microadenomas may be difficult to visualize,
supplement presents a specific challenge for
while patients with resistance to thyroid
laboratory platforms that use biotin as part of
hormone do harbor “incidental” abnormalities
the core assay configuration (eg, streptavidin-
on imaging. In addition, persistently elevated
biotin complex). This issue can affect a multitude
TSH levels following thyroid ablation
of assays, not simply thyroid hormone or TSH
in resistance to thyroid hormone results
measurements.
in thyrotroph hyperplasia and pituitary
enlargement.

ENDO 2021 • Thyroid  327

chased by Dr. Khalid H. Seedahmed, Jr., MRCP PGDip, CertEndocrinology SA - ID 283


Figure 2. Algorithm to Aid in Distinction Between Resistance to Thyroid Hormone and TSH-Secreting Pituitary Tumors

Elevated free T4 and/or free T3 with non suppressed TSH

• Confirm elevated free T4 and/or free T3 by equilibrium dialysis or 2-step method


• Confirm linearity of TSH assayed in dilution
• Exclude confounding drug therapy or intercurrent illness

Check:
No • Affected relatives? Yes
Elevated • α-subunit, molar ratio Normal
Elevated • SHBG Normal
Adenoma • Pituitary MRI/CT/PET No adenoma
No/blunted • TRH response Normal/exaggerated
No • T3 suppression Yes
Yes • Depot SSA response No
Normal • THRB gene analysis Pathogenic variant in >90%

TSH-secreting pituitary adenoma Resistance to thyroid hormone

Abbreviation: TRH, thyrotropin-releasing hormone.

• Thyrotropin-releasing hormone test: 10% to Laboratory test results:


20% of patients with TSH-secreting pituitary
Free T4 = 3.5 ng/dL (0.8-1.6 ng/dL) (SI: 45 pmol/L
tumors show an apparent preserved TSH [10-20 pmol/L])
response. Free T3 =11.7 pg/mL (2.3-4.2 pg/mL) (SI: 18 pmol/L
• THRB gene analysis: Approximately 15% of [3.5-6.5 pmol/L])
TSH = 4.8 mIU/L (0.4-4.0 mIU/L)
cases of resistance to thyroid hormone are not
associated with THRB pathogenic variants.
Which of the following is the
most appropriate next step in
Clinical Case Vignettes this patient’s management?
A. Perform contrast-enhanced MRI of the
Case 1
pituitary
A 43-year-old woman has regular palpitations
B. Perform thyrotropin-releasing hormone test
and reports a 20-lb (9-kg) weight loss over the
past 3 months. On questioning, she also has poor C. Measure serum α-subunit
quality sleep and heat intolerance with occasional D. Measure serum SHBG
headaches. Her resting pulse rate is 110 beats/min E. Seek advice on further laboratory analyses
and regular, and blood pressure is 150/85 mm Hg.
She has a fine resting tremor, warm peripheries, Answer: E) Seek advice on further laboratory analyses
and a small symmetric goiter. Visual fields are Before embarking on further investigations to
normal to confrontation testing, and there are no distinguish a TSH-secreting pituitary adenoma
signs of dysthyroid eye disease.

328  ENDO 2021 • Endocrine Case Management

chased by Dr. Khalid H. Seedahmed, Jr., MRCP PGDip, CertEndocrinology SA - ID 283


(thyrotropinoma) from resistance to thyroid D. Resistance to thyroid hormone due to THRB
hormone, it is first necessary to exclude laboratory pathogenic variant
assay interference as a cause for the unusual TFT E. TSH-secreting pituitary adenoma
pattern. This patient manifests several features (thyrotropinoma)
that suggest she has genuine thyrotoxicosis,
and therefore suspicion lies more with the TSH Answer: C) Medication-induced
result—is this a potentially erroneous/artifactual artifactual elevation of free T4
result and in fact she has primary thyrotoxicosis
Although the patient has an apparently raised free
(with a fully suppressed TSH)? Most endocrine
T4 level, the TSH is not suppressed and subsequent
laboratories can readily screen for TSH assay
investigations on the same serum sample show
interference. Thus, seeking advice on further
an entirely normal total T4 level. Taken together,
laboratory analyses (Answer E) is correct.
the normal TSH and total T4 levels argue against
genuine thyrotoxicosis and also exclude familial
Case 2 dysalbuminemic hyperthyroxinemia (due to a
A 73-year-old woman has fractured her left pathogenic variant in the albumin gene). Most
femoral neck following a fall. She is otherwise fit patients undergoing elective or emergency
and healthy, aside from mild hypertension that is surgery receive thromboprophylaxis with low-
well-controlled with amlodipine. She undergoes molecular-weight heparin, and this likely explains
emergency hip replacement. Twenty-four hours the apparently raised free T4 result in this patient.
after surgery, she is noted to be in fast atrial Heparin (both conventional and low-molecular-
fibrillation (120 beats/min) without cardiovascular weight in prophylactic and treatment dosages)
compromise (her blood pressure is 135/80 mm Hg is able to activate endothelial lipoprotein lipase.
and the remaining cardiorespiratory examination In some patients, the accompanying rise in
is unremarkable). serum free fatty acid levels is sufficient to cause
The examining physician notes that she has a displacement of T4 from its binding sites (eg, on
small, smooth goiter and queries the possibility of thyroxine-binding globulin), thereby raising free,
underlying thyroid disease. but not total, thyroid hormone levels. This is an
in vitro phenomenon, which is exacerbated by
Initial laboratory test results: delayed processing of the blood sample.

Free T4 = 2.2 ng/dL (0.8-1.6 ng/dL) (SI: 28 pmol/L


[10-20 pmol/L])
Case 3
TSH = 1.8 mIU/L (0.4-4.0 mIU/L) A 33-year-old woman is referred to the endocrine
clinic because her primary care physician is
Additional investigations (on the same serum
concerned about her levothyroxine replacement
sample):
therapy. Five years ago, she underwent total
Total T4 = 9.2 µg/dL (4.3-10.9 µg/dL) (SI: thyroidectomy for relapsed Graves disease.
118 nmol/L [55-140 nmol/L]) Postoperatively, she was clinically and
biochemically euthyroid while taking 150 mcg of
Which of the following is the most likely levothyroxine daily.
explanation for the TFT findings? However, over the last 6 months, despite
treatment with up to 300 mcg of levothyroxine
A. Familial dysalbuminemic hyperthyroxinemia
daily, she has had a persistently elevated TSH level.
B. Iodinated contrast-induced thyrotoxicosis Her pulse rate is 60 beats/min and regular,
C. Medication-induced artifactual elevation of and blood pressure is 135/80 mm Hg. She is
free T4 obese (BMI = 32.5 kg/m2) and has dry skin but

ENDO 2021 • Thyroid  329

chased by Dr. Khalid H. Seedahmed, Jr., MRCP PGDip, CertEndocrinology SA - ID 283


is otherwise clinically euthyroid. There is no resistance to thyroid hormone (Answer D).
palpable thyroid remnant. However, in the absence of such information, and
without historical TFT results outside the context
Laboratory test results: of untreated acute Graves disease, the possibility
of coexistent autoimmune thyroid disease and
Free T4 = 1.3 ng/dL (0.8-1.6 ng/dL) (SI: 17.0 pmol/L resistance to thyroid hormone would otherwise
[10.0-20.0 pmol/L])
TSH = 18.5 mIU/L (0.4-4.0 mIU/L)
need to be considered. Similarly, and assuming
the original TFT results at the time of her
There is no evidence of laboratory assay presentation with Graves disease were correctly
interference in either the free T4 or TSH assay. interpreted (ie, the TSH was appropriately fully
suppressed) then, although the patient could
Which of the following is the most likely have subsequently developed a thyrotropinoma
explanation for her TFT results? (Answer E), this would be less likely than the
A. Erratic adherence to her levothyroxine other options listed. The development of a
regimen malabsorption syndrome (eg, celiac disease)
B. Malabsorption of levothyroxine (Answer B) could lead to the development of
hypothyroidism in a previously well-controlled
C. Nonthyroidal illness (sick euthyroid syndrome)
patient, but the finding of a free T4 level in the
D. Resistance to thyroid hormone due to THRB upper half of the reference range makes this
pathogenic variant option less likely. There is nothing specific in the
E. TSH-secreting pituitary adenoma clinical history to suggest nonthyroidal illness
(thyrotropinoma) (Answer C). Erratic adherence to levothyroxine
therapy (Answer A) could produce this pattern
Answer: A) Erratic adherence to
of results, especially if the patient takes a larger
her levothyroxine regimen
dosage of levothyroxine on the day(s) immediately
This patient has previously achieved clinical before the blood test, which can be sufficient to
and biochemical euthyroidism while taking produce serum T4 levels within the laboratory
levothyroxine, 150 mcg daily (ie, not requiring reference range, but is inadequate to normalize a
supraphysiologic dosages), which argues strongly chronically raised TSH.
against an underlying diagnosis of THRB-related

References
1. Gurnell M, Halsall DJ, Chatterjee VK. What should be done when thyroid 6. Gurnell M, Visser T, Beck-Peccoz P, Chatterjee K. Resistance to thyroid
function tests do not make sense. Clin Endocrinol (Oxf). 2011;74(6):673-678. hormone. In: Jameson JL, De Groot LJ, eds. Endocrinology: Adult and Pediatric.
PMID: 21521292 7th ed. Philadelphia, PA: Saunders Elsevier, Philadelphia; 2015:1648-1665.
2. Koulouri O, Moran C, Halsall D, Chatterjee K, Gurnell M. Pitfalls in the 7. Gurnell M, Rajanayagam O, Barbar I, Jones MK, Chatterjee VK. Reversible
measurement and interpretation of thyroid function tests. Best Pract Res Clin pituitary enlargement in the syndrome of resistance to thyroid hormone.
Endocrinol Metab. 2013;27(6):745-762. PMID: 24275187 Thyroid. 1998;8(8):679-682. PMID: 9737363
3. Després N, Grant AM. Antibody interference in thyroid assays: a potential for 8. Beck-Peccoz P, Persani L, Mannavola D, Campi I. Pituitary tumours: TSH-
clinical misinformation. Clin Chem. 1998;44(3):440-454. PMID: 9510847 secreting adenomas. Best Pract Res Clin Endocrinol Metab. 2009;23(5):597-606.
4. Stockigt JR, Lim CF. Medications that distort in vitro tests of thyroid PMID: 19945025
function, with particular reference to estimates of serum free thyroxine. Best 9. Mannavola D, Persani L, Vannuchhi G, et al. Different response to chronic
Pract Res Clin Endocrinol Metab. 2009;23(6):753-767. PMID: 19942151 somatostatin analogues in patients with central hyperthyroidism. Clin
5. Favresse J, Burlacu MC, Maiter D, Gruson D. Interferences with thyroid Endocrinol (Oxf). 2005;62(2):176-181. PMID: 15670193
function immunoassays: clinical implications and detection algorithm. Endocr 10. Koulouri O, Gurnell M. TSH-secreting pituitary adenomas. In: Huhtaniemi I,
Rev. 2018;39(5):830-850. PMID: 29982406 ed. Encyclopedia of Endocrine Diseases. 2nd ed, Vol 2. New York, NY: Elsevier;
2018:261-266.

330  ENDO 2021 • Endocrine Case Management

chased by Dr. Khalid H. Seedahmed, Jr., MRCP PGDip, CertEndocrinology SA - ID 283


Surveillance of the Patient
Who Has Had Therapeutic
Hemithyroidectomy
for Thyroid Cancer
Whitney Goldner, MD. Department of Internal Medicine, Division of Diabetes,
Endocrinology, and Metabolism, University of Nebraska Medical Center, Omaha, NE; E-mail:
wgoldner@unmc.edu

Learning Objectives • Imaging surveillance after hemithyroidectomy


is primarily with neck ultrasonography.
As a result of participating in this session, learners
Optimal frequency of follow-up is unknown.
should be able to:
• Dynamic risk stratification after
• List the indications for hemithyroidectomy for hemithyroidectomy has different thyroglobulin
well-differentiated thyroid cancer. thresholds that define response to therapy as
• Describe imaging and biochemical surveillance compared with thresholds of persons who
of well-differentiated thyroid cancer following have undergone total thyroidectomy.
hemithyroidectomy. • Many patients need thyroid hormone
• Explain the modified dynamic risk following hemithyroidectomy to achieve goal
stratification criteria following TSH.
hemithyroidectomy for well-differentiated • It is important to discuss all of these issues
thyroid carcinoma. preoperatively to determine the best treatment
• Describe the role of thyroid hormone therapy for the patient.
in the treatment and surveillance of well-
differentiated thyroid cancer.

Significance of the
Clinical Problem
Main Conclusions Hemithyroidectomy (thyroid lobectomy ±
isthmusectomy) is acceptable and often preferred
• Hemithyroidectomy as definitive therapy
for thyroid nodules with Bethesda 3 and 4
for well-differentiated thyroid cancer is
indeterminate cytopathology (follicular lesion of
recommended for low-risk thyroid cancers
undetermined significance, follicular neoplasm,
that do not need radioactive iodine therapy.
Hurthle-cell neoplasm) on preoperative FNA.1
If a patient meets criteria for radioactive
This approach is also acceptable for many low-
iodine, then completion thyroidectomy ±
risk well-differentiated thyroid carcinomas.1,2
central neck dissection is recommended.
There are defined histologic criteria that influence
whether a tumor is categorized as having a low,

ENDO 2021 • Thyroid  331

chased by Dr. Khalid H. Seedahmed, Jr., MRCP PGDip, CertEndocrinology SA - ID 283


intermediate, or high risk of recurrence, which evaluation of thyroid hormone levels to optimize
in turn influences the decision regarding whether TSH levels. Since thyroglobulin is produced by
a person needs completion thyroidectomy and both normal and malignant thyroid tissue, it can
adjuvant therapy with radioactive iodine.1,2 be detectable or elevated in patients who have
The decision to pursue hemithyroidectomy vs undergone total thyroidectomy without
total thyroidectomy ± lymph node dissection radioactive iodine or hemithyroidectomy.
is an individual decision based on preoperative Detectable thyroglobulin does not always indicate
imaging, FNA results, risk factors, and patient recurrence of thyroid cancer, and it may indicate
preference. Ideally, the discussion of pros and cons the presence of normal thyroid tissue. A dynamic
of hemithyroidectomy vs total thyroidectomy and risk stratification system has been developed to
indications for completion thyroidectomy occurs predict risk of recurrence over time for those who
preoperatively. However, many of the details that have undergone total thyroidectomy + radioactive
will inform decision-making are not known until iodine (Table).4 This assessment is designed to be
final histopathology is obtained. Historically, it was performed at each follow-up visit and is
common for all patients to undergo completion categorized into excellent, indeterminate,
thyroidectomy if histopathology revealed biochemically incomplete, and structurally
malignancy, given that most patients underwent incomplete response to therapy.4 The same criteria
adjuvant radioactive iodine therapy.3 However, to assess recurrence risk cannot be applied to those
current guidelines do not recommend radioactive who have undergone hemithyroidectomy, hence a
iodine in all patients. American Thyroid modified dynamic risk stratification has been
Association and National Comprehensive Cancer developed,5,6 which has a thyroglobulin threshold
Network guidelines recommend radioactive of less than 30 ng/mL (<30 µg/L) to define an
iodine in select patients with intermediate risk of excellent response to therapy (Table).
recurrence and in all patients at high risk.1,2 Hence, Unfortunately, the absolute thyroglobulin cutoffs
completion thyroidectomy is not indicated in most for indicating recurrence are still unknown and
low-risk thyroid malignancies. the thyroglobulin concentration at a single time
Follow-up and surveillance of well- point is less predictive than the overall trend of the
differentiated thyroid cancer consists of imaging, thyroglobulin level. American Thyroid Association
serum thyroglobulin concentration as a tumor guidelines recommend considering checking
marker, thyroglobulin antibodies to ensure serum thyroglobulin after lobectomy and then
accuracy of the thyroglobulin measurement, and checking it every 12 to 24 months.1 A rising

Table. Thyroglobulin Thresholds for Response to Therapy and TSH Targets for Each Category1,4-6

Response to Therapy Post Total Thyroidectomy + Radioactive Iodine Post Hemithyroidectomy TSH Goals

Excellent Nonstimulated thyroglobulin <0.2 ng/mL Nonstimulated thyroglobulin <30 ng/mL 0.5-2.0 mIU/L
(SI: <0.2 µg/L) (SI: <30 µg/L)
Stimulated thyroglobulin <1 ng/mL (SI: <1 µg/L)

Indeterminate Nonstimulated thyroglobulin 0.2-1.0 ng/mL Stable or declining thyroglobulin 0.1-0.5 mIU/L
(SI: 0.2-1.0 µg/L) antibody levels
Stimulated thyroglobulin 1-10 ng/mL (SI: 1-10 µg/L)

Biochemically Nonstimulated thyroglobulin >1.0 ng/mL Nonstimulated thyroglobulin >30 ng/mL 0.1-0.5 mIU/L
incomplete (SI: >0.1 µg/L) (SI: >30 µg/L)
Stimulated thyroglobulin >10 ng/mL (SI: >10 µg/L)

Structurally incomplete Thyroglobulin and thyroglobulin antibodies do not Thyroglobulin and thyroglobulin <0.1 mIU/L
define; based on imaging antibodies do not define; based on
imaging

332  ENDO 2021 • Endocrine Case Management

chased by Dr. Khalid H. Seedahmed, Jr., MRCP PGDip, CertEndocrinology SA - ID 283


thyroglobulin level over time is more predictive of some amount of thyroid hormone therapy.
recurrence than one that is stable or dropping. Predictors of not requiring postoperative thyroid
Recently, however, Park et al showed that hormone included lower preoperative TSH, male
thyroglobulin levels increase gradually by about gender, and normal thyroid parenchyma (no
10% per year following lobectomy without autoimmunity).11-13 However, there are no absolute
evidence of recurrence.7 The thyroglobulin-to- preoperative predictors of the need for thyroid
TSH ratio also did not differ between those with hormone therapy.
and without proven disease recurrence. Overall,
the risk of recurrence in low- and intermediate-
risk tumors is low and the misinterpretation of
Barriers to Optimal Practice
detectable thyroglobulin levels may lead to
• Serum thyroglobulin
unnecessary surveillance or intervention.
Thorough preoperative imaging of both the ѓ Thyroglobulin can be elevated in benign
thyroid and central and lateral neck compartments and malignant disease. Thyroglobulin
is essential when evaluating persons with suspicious thresholds to predict recurrence are not
thyroid nodules, especially with indeterminate or absolute. Thyroglobulin values can vary
malignant cytology, and when surgery is planned.8 between assays, so they must be evaluated
Preoperative imaging guides the extent of initial using the same assay every time.
surgery and serves as a baseline for postoperative ѓ Thyroglobulin thresholds following
follow-up imaging. If a patient has undergone hemithyroidectomy are individualized.
hemithyroidectomy and there is no preoperative Each patient serves as their own baseline.
assessment of the contralateral lobe and central/ However, thyroglobulin levels may rise
lateral neck compartments, this should be done over time without recurrence, which can
postoperatively to determine next steps in therapy. lead to increased surveillance and patient
The presence of contralateral nodules and/or anxiety.
abnormal lymph nodes prompts further workup
and possible therapy. If preoperative imaging is ѓ The presence of thyroglobulin antibodies
otherwise negative and hemithyroidectomy is interferes with the thyroglobulin
performed for definitive therapy, then surveillance immunoassay and interferes with the
consists of serial ultrasonography (with or without interpretation of serum thyroglobulin as a
thyroglobulin as discussed above). However, tumor marker.
studies have shown that false-positive findings on • Imaging
surveillance ultrasonography can occur in 57% to
67% of patients at low and intermediate risk.9,10 ѓ Preoperative imaging is essential. Often,
Hence, the potential for over-monitoring and endocrinologists first meet patients
unnecessary procedures exists, and the optimal postoperatively.
frequency of long-term follow-up imaging is still ѓ Optimal frequency of imaging surveillance
unknown. is unknown.
Guidelines recommend a TSH goal based
on the dynamic response to therapy categories • Thyroid hormone therapy
(Table).1 Many patients choose to have
ѓ Many patients require thyroid hormone
hemithyroidectomy to avoid long-term thyroid
postoperatively to achieve optimal TSH.
hormone therapy. However, studies have shown
There are no absolute predictors of patients
that in order for patients to attain goal TSH
who will require postoperative thyroid
(<2.0 mIU/L for most patients based on response
hormone therapy.
to therapy), 50% to 71% of patients require

ENDO 2021 • Thyroid  333

chased by Dr. Khalid H. Seedahmed, Jr., MRCP PGDip, CertEndocrinology SA - ID 283


ѓ If a patient does not take thyroid hormone, operative risk profile to determine whether
this may cause a rise in thyroglobulin a potential second operation for completion
without recurrence. thyroidectomy would be risk prohibitive
compared with total thyroidectomy as the index
operation. Ideally, this decision should be made in
Strategies for Diagnosis, a multidisciplinary forum.
Therapy, and/or Management If the clinician first meets the patient after they
Preoperatively, clinicians should fully evaluate the have already undergone hemithyroidectomy, then
thyroid and both central and lateral neck lymph the overall risk of recurrence should be evaluated.
nodes with ultrasonography. If there is bilateral If adequate preoperative imaging was not done,
thyroid nodularity, abnormal lymph nodes, or this should be completed postoperatively. If there
suspicion of extracapsular extension, this must is concern for contralateral disease or abnormal
be considered when discussing possibilities of lymph nodes, further evaluation should proceed
total thyroidectomy vs hemithyroidectomy. If with FNA of the suspicious areas. If staging and
cytopathology is Bethesda 3 or 4 and no other recurrence risk indicate the need for radioactive
worrisome features are present, or cytopathology iodine therapy, then many clinicians recommend
is Bethesda 5 or 6 and the patient is determined completion thyroidectomy ± appropriate lymph
to be at low or intermediate risk by imaging, node dissection and, when appropriate, adjuvant
then clinicians should discuss the pros and therapy with radioactive iodine.
cons of hemithyroidectomy and indications for If the recurrence risk is low or intermediate
completion thyroidectomy. Size should also be and hemithyroidectomy is appropriate, one should
factored into the discussion when determining the proceed with surveillance imaging, consideration
appropriate surgery. Both the American Thyroid of thyroglobulin monitoring, and evaluation
Association and National Comprehensive Cancer of the need for thyroid hormone therapy. On
Network guidelines recommend consideration of ultrasonography, if there are no abnormalities
hemithyroidectomy for nodules smaller than 4 cm in the remaining lobe and no pathologic-
without other worrisome features.1,2 Additionally, appearing lymphadenopathy, one can check yearly
one should discuss expectations for postoperative ultrasonography and reduce the frequency after 2 to
surveillance, including imaging and thyroglobulin 5 years. If there is a nodule in the remaining lobe,
monitoring (especially if the patient has bilateral the clinician should determine whether the patient
nodules). There should also be discussion needs to be monitored closely or if FNA should be
regarding preference for thyroid hormone therapy. performed if the nodule meets criteria based on
Many patients assume they will not need thyroid American Thyroid Association Thyroid Nodule
hormone if they have a hemithyroidectomy. Guidelines.1 If thyroid ultrasonography shows a
It is important to discuss the possibility of new nodule or increasing size of nodule or abnormal
postoperative thyroid hormone therapy (taking lymph node, then FNA should be performed
into account baseline autoimmunity, baseline (+ thyroglobulin washings if it is a lymph node).
TSH, and remnant thyroid volume), TSH goals, The clinician must determine whether to
and threshold for starting therapy. Ultimately, all follow thyroglobulin levels postoperatively. Not
of these issues should be addressed preoperatively all clinicians routinely check thyroglobulin in
to gauge patient comfort with both treatment patients who have undergone hemithyroidectomy.
and the long-term surveillance plan. It is equally If a baseline thyroglobulin concentration was
important to communicate with the surgeon documented postoperatively, then thyroglobulin
to ensure consistency of expectations from all can be included with yearly labs along with
members of the medical team and with the patient. periodic ultrasonography. If thyroglobulin
The surgeon must also consider the patient’s is rising, the neck should be imaged with

334  ENDO 2021 • Endocrine Case Management

chased by Dr. Khalid H. Seedahmed, Jr., MRCP PGDip, CertEndocrinology SA - ID 283


ultrasonography. However, it is important Which of the following is the best next step?
to note that recurrences have been reported A. Chest CT to evaluate for metastatic disease
with increasing, decreasing, and stable serum B. FNA of the right thyroid nodule
thyroglobulin measurements. One should also
consider levothyroxine therapy if the patient is not C. Initiation of levothyroxine, 50 mcg daily
already taking it or consider adjusting the dosage D. Continued observation and repeated
to achieve a goal TSH level of 0.1 to 0.5 mIU/L ultrasonography; measurement of
(indeterminate response to therapy). If the thyroglobulin, thyroglobulin antibodies, and
thyroglobulin is higher than expected or increases thyroid function in 6 months
dramatically, one should evaluate for other areas of E. Completion thyroidectomy followed by
recurrence (chest) if neck imaging is negative. radioactive iodine
Answer: C or D) Initiation of levothyroxine,
Clinical Case Vignettes 50 mcg daily or continued observation and
Case 1 repeated ultrasonography; measurement
of thyroglobulin, thyroglobulin antibodies,
A 35-year-old woman seeks evaluation of newly and thyroid function in 6 months
diagnosed papillary thyroid cancer. She has a
1.2-cm nodule in the left thyroid lobe. There is This patient’s neck imaging is negative, her TSH
also a small 5-mm spongiform nodule in the right concentration is above the goal of 0.5 to 2.0
thyroid lobe. No abnormal lymph nodes are seen mIU/L, and she is not currently on levothyroxine
on ultrasonography in the central or lateral neck. replacement. Therefore, starting low-dosage
She undergoes left lobectomy and isthmusectomy. levothyroxine (Answer C) is recommended,
Pathology documents a 1.2-cm classic papillary with follow-up measurement of TSH,
thyroid carcinoma (pT1bNxMx). She understands thyroglobulin, and thyroglobulin antibodies. If
that her right thyroid lobe nodule needs thyroglobulin remains high and continues rising
surveillance. She underwent hemithyroidectomy on levothyroxine, then additional imaging of the
because she did not want to take thyroid hormone. chest (Answer A) would be warranted.
Three months postoperatively, her thyroglobulin Ultrasonography does not show any
level is 35 ng/mL (35 µg/L) with negative abnormalities in the neck and the nodule in the
thyroglobulin antibodies. right lobe is stable. Thyroglobulin was measured
She is now 1 year out from surgery and would using a different assay, making it difficult to
like your recommendations. directly compare, but it is much higher than
Neck ultrasonography shows a small 4-mm previous measurements. This patient chose
spongiform nodule in the right lobe, which is hemithyroidectomy because she wanted to avoid
very low risk sonographically. There is no cervical long-term levothyroxine therapy. If she is opposed
lymphadenopathy. to taking thyroid hormone, continued surveillance
at a short interval (6 months) (Answer D) with the
Laboratory test results: same thyroglobulin and thyroglobulin antibody
TSH = 2.7 mIU/L assays is reasonable. If the thyroglobulin rises, then
Thyroglobulin (different assay) = 75 ng/mL additional imaging should be pursued.
(SI: 75 µg/L) This is a low-risk nodule sonographically and
Thyroglobulin antibodies = <20 IU/mL
it is subcentimeter in size. Hence, FNA (Answer B)
(SI: <20 kIU/L)
would not be recommended.
It is reasonable to consider completion
thyroidectomy (Answer E) as one of the options;
however, empirically treating with radioactive

ENDO 2021 • Thyroid  335

chased by Dr. Khalid H. Seedahmed, Jr., MRCP PGDip, CertEndocrinology SA - ID 283


iodine before evaluating the pathology results American Thyroid Association risk of recurrence
and postoperative thyroglobulin would not be classification.1 These tumors are also at higher
recommended. risk for distant metastatic disease. High-risk
angioinvasive follicular thyroid carcinoma is an
Case 2 indication for adjuvant radioactive iodine therapy
at the time of diagnosis.1,2 Continued surveillance
A 42-year-old man presents for evaluation and alone (Answer A) is not adequate. In this situation,
continued management of his thyroid cancer. many clinicians would choose completion
Follicular thyroid carcinoma was diagnosed 1 year thyroidectomy with adjuvant radioactive
ago at an outside hospital. His 1.5-cm tumor was iodine (Answer B), which would have been the
in the right thyroid lobe, and no lymph nodes were appropriate therapy at the time of diagnosis.
removed. Postoperative ultrasonography showed Initiating levothyroxine (Answer C) or performing
no nodules in the remaining left thyroid lobe. chest CT (Answer E) would both be reasonable
There was no cervical lymphadenopathy. if the patient does not agree to surgery and
You obtain his outside pathology reports, which radioactive iodine. However, the TSH treatment
reveal extensive vascular invasion (more than 4 foci goal should be less than 0.1 mIU/L initially, rather
of vascular invasion) in his primary tumor. than 0.5 to 2.0 mIU/L. With a remaining thyroid
Laboratory test results: lobe in place,131 I whole-body scan (Answer D)
would not be recommended before completion
Thyroglobulin = 20 ng/mL (SI: 20 µg/L) thyroidectomy.
Thyroglobulin antibodies = <20 IU/mL
(SI: <20 kIU/L)
TSH = 2.1 mIU/L Case 3
Current ultrasonography shows a normal left A 23-year-old woman presents for evaluation
thyroid lobe and no cervical lymphadenopathy. of a sonographically intermediate-risk solitary
thyroid nodule that measures 1.8 cm. FNA of the
Which of the following is the nodule determines that it is a follicular neoplasm.
best recommendation? Results of molecular testing are suspicious. The
A. Continued surveillance with ultrasonography contralateral lobe does not have any abnormalities,
and measurement of thyroglobulin, and there are no pathologic-appearing lymph
thyroglobulin antibodies, and TSH yearly nodes on ultrasonography. She does not have a
history of preexisting autoimmune thyroid disease
B. Completion thyroidectomy and consideration
or radiation exposure. Her TSH concentration is
of radioactive iodine
0.9 mIU/L.
C. Initiation of levothyroxine, 50 mcg daily, for She opts for hemithyroidectomy for definitive
goal TSH of 0.5 to 2.0 mIU/L diagnosis. Histopathology shows a follicular
D. 131I whole-body scan variant of papillary thyroid carcinoma, partially
E. Chest CT to evaluate for metastatic disease encapsulated, 1.8 cm. The margins are negative
and there is no vascular invasion.
Answer: B) Completion thyroidectomy and
consideration of radioactive iodine
This patient has angioinvasive (>4 vessels)
follicular thyroid carcinoma (American Joint
Committee on Cancer [AJCC] 8th ed pT1bNxMx,
stage 1). More than 4 vessels of angioinvasion
predicts a high recurrence risk according to the

336  ENDO 2021 • Endocrine Case Management

chased by Dr. Khalid H. Seedahmed, Jr., MRCP PGDip, CertEndocrinology SA - ID 283


It is now is 6 months after surgery. Case 3 (Continued)
Which of the following do you
recommend? (select all that apply)
Would your recommendations
A. TSH measurement and thyroid hormone change if the pathology showed:
therapy if TSH is >2.0 mIU/L A. Noninvasive follicular thyroid neoplasm with
B. Neck ultrasonography papillary-like nuclear features (NIFT-P)?
C. Measurement of thyroglobulin and
thyroglobulin antibodies NIFT-P has been reclassified as its own
entity with a very low malignant potential.14
D. Chest CT to evaluate for metastatic disease
Given that the contralateral lobe was
E. Completion thyroidectomy and consideration normal and there were no abnormal
of radioactive iodine lymph nodes, completion thyroidectomy
Answer: A, B, and possibly C) TSH measurement is not recommended. As with other cases
and thyroid hormone therapy if TSH is >2.0 mIU/L, discussed previously, thyroglobulin and
neck ultrasonography, and possibly measurement thyroglobulin antibodies are controversial
of thyroglobulin and thyroglobulin antibodies after hemithyroidectomy, and many clinicians
would not recommend thyroglobulin or
This patient has a follicular variant of papillary thyroglobulin antibody measurement. The risk
thyroid carcinoma (AJCC 8th ed pT1bNxMx, of distant metastatic disease is exceedingly low,
stage 1). According to the American Thyroid so chest CT is not indicated. TSH should be
Association, the tumor has a low risk of structural measured, and a normal TSH level should be
recurrence.1 Her contralateral lobe is normal maintained, but treating with levothyroxine
and there are no abnormal-appearing lymph for a goal TSH less than 2.0 IU/L is not
nodes. Therefore, completion thyroidectomy or necessary. Routine follow-up postoperative
radioactive iodine therapy (Answer E) would not ultrasonography is not required, but imaging
be recommended. There is no reason to suspect should be done if there are other clinical
a high risk for metastatic disease, so CT chest indications.
(Answer D) is not indicated. Postoperative neck
ultrasonography (Answer B) at 6 months could be B. Vascular invasion?
performed as a baseline to evaluate for suspicious If the tumor had vascular invasion, then
lesions in the contralateral lobe or lymph nodes. it would be classified at intermediate risk
TSH should also be measured and levothyroxine of structural recurrence according to the
therapy should be started if her TSH is not at goal American Thyroid Association classification
(0.5-2.0 mIU/L) (Answer A). There is controversy (approximately 15%-30%).1 Completion
about routine measurement of thyroglobulin and thyroidectomy should be considered +
thyroglobulin antibodies (Answer C) in patients radioactive iodine.
with low recurrence risk who have undergone
hemithyroidectomy. Thyroglobulin can rise slowly C. Microscopic positive margins on the tumor?
over time even without evidence of recurrence.7 In
There is conflicting evidence regarding
select patients, some clinicians check thyroglobulin
positive margins as a predictor of local
and thyroglobulin antibodies postoperatively
recurrence and the need for adjuvant
and use the response to therapy thyroglobulin
radioactive iodine therapy. Some studies
thresholds in the table above.
report no association between microscopically
positive surgical margins and local recurrence,
especially anterior positive margins.15,16
However, posterior microscopically positive

ENDO 2021 • Thyroid  337

chased by Dr. Khalid H. Seedahmed, Jr., MRCP PGDip, CertEndocrinology SA - ID 283


margins have been associated with higher Hobnail variants are in the American Thyroid
rates of local recurrence.15 In this case, Association intermediate risk of recurrence
completion thyroidectomy is reasonable to category1 and are considered to be aggressive
allow the clinician to further evaluate with histologic types. Hobnails variants have been
postoperative thyroglobulin and thyroglobulin associated with higher rates of extrathyroidal
antibodies and determine the need for extension and locoregional lymph node
adjuvant radioactive iodine. If there was a metastases.17 Hence, completion thyroidectomy
positive posterior margin, many clinicians and possible radioactive iodine are recommended
would recommend adjuvant radioactive (Answer E). Observation (Answer D) is not
iodine therapy. If there was a positive anterior preferred because the potential higher risk of
margin, then thyroglobulin and thyroglobulin recurrence. Even though hobnail variants have
antibodies should be evaluated 6 weeks been associated with the BRAF V600E variant
postoperatively, and if the thyroglobulin is and RET/PTC rearrangements, routine molecular
elevated, adjuvant radioactive iodine would testing to guide additional therapy (Answer A) is
be recommended. If the thyroglobulin is not currently recommended. Research is ongoing
undetectable, the clinician should have to determine whether molecular pathologic
a discussion with the patient about pros variants can guide surgical therapy in the future.
and cons of watching vs proceeding with In this case, preoperative imaging did not reveal
radioactive iodine therapy. any abnormal lymph nodes in the central or
lateral neck, so neck ultrasonography now
Case 4  New Content  (Answer B) is not absolutely indicated. However,
if there were concern that lymph nodes were not
A 34-year-old woman presents with a 2.5-cm left- adequately imaged preoperatively, then repeated
sided thyroid nodule. According to imaging, it is ultrasonography would be indicated. Measuring
of indeterminate risk (TIRADS 4). Preoperatively, thyroglobulin and thyroglobulin antibodies
there are no abnormal-appearing lymph nodes (Answer C) would not change the above
in her central or lateral neck. She undergoes left recommendations.
thyroid lobectomy and isthmusectomy. Pathologic
examination documents a 2.5-cm papillary thyroid
carcinoma, hobnail variant. Case 5  New Content 
A 56-year-old man presents with a bilateral
Which of the following should thyroid nodules and hyperthyroidism. Radioactive
be recommended now? iodine uptake and scan show a hot nodule (2.5 cm)
A. Molecular testing to determine next steps in in the right lobe and a cold nodule (3.4 cm) in the
treatment left lobe. Biopsy of the left cold nodule is benign.
B. Neck ultrasonography After discussion of all options, he opts to undergo
right thyroid lobectomy for definitive treatment
C. Measurement of thyroglobulin and
of his toxic right-sided thyroid nodule and
thyroglobulin antibodies
hyperthyroidism. He specifically states he does not
D. Observation and follow-up neck want to undergo total thyroidectomy to preserve
ultrasonography in 6 months the chance that he might not need to take any
E. Completion thyroidectomy and consideration thyroid hormone postoperatively.
of radioactive iodine He undergoes right thyroid lobectomy
and isthmusectomy. Pathologic examination
Answer: E) Completion thyroidectomy and
documents a 2.5-cm papillary thyroid carcinoma.
consideration of radioactive iodine
Two lymph nodes are removed from his right

338  ENDO 2021 • Endocrine Case Management

chased by Dr. Khalid H. Seedahmed, Jr., MRCP PGDip, CertEndocrinology SA - ID 283


central neck, but a formal right central neck recurrence risk when it is present in more than
dissection is not performed. In 1 of the lymph 3 lymph nodes.1 A meta-analysis of 23 studies
nodes, he has metastatic papillary thyroid evaluating the impact of extranodal extension in
carcinoma, 9 mm in greatest dimension, with thyroid cancer reported extranodal extension was
extranodal extension. associated with all-cause mortality and disease
recurrence.18 Similarly, microscopic extranodal
Which of the following is the best next step? extension was an independent risk factor for
A. Observation with thyroid ultrasonography locoregional recurrence of thyroid cancer, and the
and measurement of serum thyroglobulin and number and size of positive lymph nodes were
thyroglobulin antibodies in 6 to 12 weeks not independent factors.19 Given the increased
B. Completion thyroidectomy risk of recurrence, completion thyroidectomy
with central neck dissection (Answer C) and
C. Completion thyroidectomy with central neck
consideration of radioactive iodine is preferred.
dissection
Observation (Answer A) is not preferred because
D. TSH suppression with levothyroxine of the potential higher risk of recurrence.
E. Radioactive iodine uptake and scan Completion thyroidectomy alone (Answer B)
may not be sufficient given that he has central
Answer: C) Completion thyroidectomy
neck lymph node involvement with extranodal
with central neck dissection
extension and has not had a formal central
This patient has a T2N1aMx papillary thyroid neck dissection. He will need thyroid hormone
carcinoma. He had 1 lymph node removed with therapy, but not as the sole therapy (Answer D).
a 9-mm focus of metastatic tissue, but it also A radioactive iodine uptake and scan (Answer E)
had extranodal extension. Extranodal extension before completion thyroidectomy would not add
is considered a risk factor for structural disease additional clinical or diagnostic information.
recurrence and is estimated to have a 40%

References
1. Haugen BR, Alexander EK, Bible KC, et al. 2015 American Thyroid 7. Park S, Jeon MJ, Oh HS, et al. Changes in serum thyroglobulin levels after
Association Management guidelines for adult patients with thyroid nodules lobectomy in patients with low-risk papillary thyroid carcinoma. Thyroid.
and differentiated thyroid cancer: the American Thyroid Association 2018;28(8):997-1003. PMID: 29845894
Guidelines Task Force on Thyroid Nodules and Differentiated Thyroid 8. Yeh MW, Bauer AJ, Bernet VA, et al; American Thyroid Association Surgical
Cancer. Thyroid. 2016;26(1):1-133. PMID: 26462967 Affairs Committee Writing Task Force. American Thyroid Association
2. National Comprehensive Cancer Network. NCCN Thyroid Carcinoma statement on preoperative imaging for thyroid cancer surgery. Thyroid.
Guidelines Version 2.2019. Available at: www.NCCN.org. Accessed for 2015;25(1):3-14. PMID: 25188202
verification November 2019. 9. Peiling Yang S, Back AM, Tuttle RM, Fish SA. Frequent screening with serial
3. American Thyroid Association (ATA) Guidelines Taskforce on Thyroid neck ultrasound is more likely to identify false-positive abnormalities than
Nodules and Differentiated Thyroid Cancer, Cooper DS, Doherty GM, clinically significant disease in the surveillance of intermediate risk papillary
et al. Revised American Thyroid Association management guidelines for thyroid cancer patients without suspicious findings on follow-up ultrasound
patients with thyroid nodules and differentiated thyroid cancer. Thyroid. evaluation. J Clin Endocrinol Metab. 2015;100(4):1561-1567. PMID: 25632970
2009;19(11):1167-1214. PMID: 19860577 10. Yang SP, Bach AM, Tuttle RM, Fish SA. Serial neck ultrasound is more likely
4. Tuttle RM, Tala H, Shah J, et al. Estimating risk of recurrence in to identify false-positive abnormalities than clinically significant disease in
differentiated thyroid cancer after total thyroidectomy and radioactive iodine low-risk papillary thyroid cancer patients. Endocr Pract. 2015;21(12):1372-
remnant ablation: using response to therapy variables to modify the initial 1379. PMID: 26372300
risk estimates predicted by the new American Thyroid Association staging 11. Lee MC, Kim MU, Choi HS, et al. Postoperative thyroid-stimulating
system. Thyroid. 2010;20(12):1341-1349. PMID: 21034228 hormone levels did not affect recurrence after thyroid lobectomy in patients
5. Momesso DP, Tuttle RM. Update on differentiated thyroid cancer staging. with papillary thyroid cancer. Endocrinol Metab (Seoul). 2019;34(2):150-157.
Endocrinol Metab Clin N Am. 2014;43(2):401-421. PMID: 24891169 PMID: 31099202
6. Momesso DP, Vaisman F, Yang SP, et al. Dynamic risk stratification in 12. Ha TK, Kim DW, Park HK, Lee YJ, Jung SJ, Baek HJ. Factors influencing
patients with differentiated thyroid cancer treated without radioactive iodine. the successful maintenance of euthyroidism after lobectomy in patients
J Clin Endocrinol Metab. 2016;101(7):2692-2700. PMID: 27023446 with papillary thyroid microcarcinoma: a single-center study. Endocr Pract.
2019;25(10):1035-1040. PMID: 31241363

ENDO 2021 • Thyroid  339

chased by Dr. Khalid H. Seedahmed, Jr., MRCP PGDip, CertEndocrinology SA - ID 283


13. Lee YM, Jeon MJ, Kim WW, et al. Optimal thyrotropin suppression 17. Lubitz C, Economopoulos KP, Pawlak A, et al. Hobnail variant of papillary
therapy in low-risk thyroid cancer patients after lobectomy. J Clin Med. thyroid carcinoma: an institutional case series and molecular profile. Thyroid.
2019;8(9):E1279. PMID: 31443521 2014;24(6):958-965. PMID: 24417340
14. Nikiforov YE, Seethala RR, Tallini G, et al. Nomenclature revision 18. Veronese N, Luchini C, Nottegar A, et al. Prognostic impact of extra-nodal
for encapsulated follicular variant of papillary thyroid carcinoma: a extension in thyroid cancer: a meta-analysis. J Surg Oncol. 2015;112(8):828-
paradigm shift to reduce overtreatment of indolent tumors. JAMA Oncol. 833. PMID: 26493240
2016;2(8):1023-1029. PMID: 27078145 19. Lang BHH, Shek TWH, Wan KY. Impact of microscopic extra-nodal
15. Lang BH, Shek TW, Wan KY. Does microscopically involved margin extension (ENE) on locoregional recurrence following curative surgery
increase disease recurrence after curative surgery in papillary thyroid for papillary thyroid carcinoma. J Surg Oncol. 2016;113(5):526-531.
carcinoma? J Surg Oncol. 2016;113(6):635-639. PMID: 26843438 PMID: 26792294
16. Sanabria A, Rojas A, Arevalo J, Kowalski L, Nixon I. Microscopically positive
surgical margins and local recurrence in thyroid cancer. A meta-analysis. Eur J
Surg Oncol. 2019;45(8):1310-1316. PMID: 30795955

340  ENDO 2021 • Endocrine Case Management

chased by Dr. Khalid H. Seedahmed, Jr., MRCP PGDip, CertEndocrinology SA - ID 283


Management of Thyroid
Dysfunction Before and
During Pregnancy
Kristien Boelaert, MD, PhD, FRCP. Institute of Applied Health Research, School of Medical
and Dental Sciences, University of Birmingham, Birmingham, United Kingdom; E-mail:
k.boelaert@bham.ac.uk

Learning Objectives not necessarily result in improved obstetric or


neurocognitive outcomes and may in fact be
As a result of participating in this session, learners
associated with a degree of harm. While there
should be able to:
are clear links between thyroid autoimmunity
• Discuss the evidence of benefits and risks of and increased risks of miscarriage and preterm
treatment of mild thyroid hypofunction before delivery, a number of well-conducted clinical
and during pregnancy in relation to obstetric trials have failed to demonstrate beneficial
and child neurodevelopmental outcomes. effects on obstetric outcomes with levothyroxine
replacement in euthyroid women who have
• Review what is known about the relationship
raised concentrations of autoimmune antibodies.
between thyroid autoimmunity and
Uncontrolled hyperthyroidism during pregnancy
pregnancy outcomes, as well as the effects of
is associated with significant risks of adverse
levothyroxine therapy in this setting.
outcomes and should be distinguished from
• Articulate the optimal management of transient rises in circulating thyroid hormones.
hyperthyroidism before and during pregnancy. Specific consideration is required as to the choice
of antithyroid drugs to be used, especially before
and during early pregnancy, aiming to use the
lowest possible dosages with regular monitoring of
Main Conclusions thyroid function.
Significant controversy continues to surround the The field currently lacks clinical trial data,
management of thyroid dysfunction before and which are necessary to prove beneficial effects but
during pregnancy and indications for treatment also to evaluate the potential risk associated with
are not adopted in a universally uniform manner. treatment and overtreatment with levothyroxine
The physiological changes to thyroid function and antithyroid drugs.
associated with pregnancy make the interpretation
of thyroid function tests difficult, and ideally
population- and pregnancy-specific reference Significance of the
ranges should be used. The previous upper limit
for TSH concentrations of 2.5 mIU/L has been
Clinical Problem
revised in view of accumulating evidence that Thyroid hormones are essential for normal
levothyroxine replacement for women with pregnancy and optimal fetal growth and
serum TSH concentrations below 4 mIU/L does development.1 Pregnancy directly affects
thyroid function, and the fetal thyroid gland is

ENDO 2021 • Thyroid  341

chased by Dr. Khalid H. Seedahmed, Jr., MRCP PGDip, CertEndocrinology SA - ID 283


not functionally mature until 18 to 20 weeks’ Barriers to Optimal Practice
gestation. Early pregnancy is therefore a critical
period during which maternal thyroid dysfunction • Uncertainty regarding optimal reference
may have deleterious effects on the health of ranges for determination of thyroid
the mother and fetus.2 Careful and pragmatic dysfunction, taking into account factors
preconception planning and management of affecting thyroid physiology before and during
thyroid function is key to ensuring optimal pregnancy.
outcomes, aiming to mirror normal physiological
• A lack of large randomized controlled trials
adaptive changes during pregnancy and minimize
evaluating outcomes in mothers with mild
potential unwanted effects of the treatment
thyroid hypofunction and their offspring
administered.3
following therapy started before or during
Overt hypothyroidism affects 0.2% to 0.6% of
early gestation.
pregnancies, whereas subclinical hypothyroidism
occurs in 3.5% to 18%, depending on the definition • The demonstration that levothyroxine therapy
used.4 Thyroid autoimmunity, characterized by the is ineffective in optimizing live births in
presence of elevated TPO antibodies, is the main women with previous miscarriage who have
risk factor for maternal hypothyroidism and occurs raised TPO antibody concentrations.
in up to 30% of women with recurrent miscarriage • Emerging data demonstrating teratogenic
and subfertility.5 Isolated hypothyroxinemia effects of antithyroid drugs and deleterious
may reflect a state of mild iodine deficiency effects of high levels of circulating thyroid
but is likely multifactorial and may, in part, be hormones on pregnancy outcomes.
explained by assay interference due to changes in
binding proteins as a result of pregnancy. There is
continued debate regarding what constitutes overt Strategies for Diagnosis,
and subclinical hypothyroidism in pregnancy and
Therapy, and/or Management
which cutoffs should be used to identify high-risk
pregnancies requiring levothyroxine replacement Definition of Thyroid Dysfunction
therapy, especially in view of emerging evidence of Before and During Pregnancy
potential risks associated with overtreatment.1 Pregnancy affects thyroid physiology
Hyperthyroidism in pregnancy may be due dramatically. Fetal consumption of maternal
to Graves disease or may be caused by transient thyroid hormones, increased concentrations of
elevations of thyroid hormone concentrations thyroxine-binding globulin, increased urinary
due to high levels of hCG in early pregnancy. iodine clearance, and increasing degradation of
The former requires active management with thyroid hormone by placental type 3 deiodinase
antithyroid drugs, whereas the latter is usually require enhanced maternal thyroid hormone
treated conservatively. The goal of therapy production to ensure adequate availability for the
is to control hyperthyroidism while avoiding mother and fetus. High concentrations of hCG
unwanted adverse effects of medication, as well as stimulate the pituitary TSH receptor resulting in
overzealous induction of hypothyroidism.1,3 increased thyroid hormone production, which
The current state of knowledge leaves is usually associated with reduced serum TSH
physicians managing women before and during concentrations reaching a trough at 10 to 12
pregnancy uncertain regarding the indications weeks of pregnancy.1,3 A number of additional
for treatment of thyroid dysfunction and factors affect thyroid function during pregnancy.
autoimmunity, as well as the choice of treatment These include the population iodine status,
to be administered to optimize fetal and maternal ethnicity, BMI, age, parity, and cigarette smoking.
outcomes. Importantly, the usual physiological response to

342  ENDO 2021 • Endocrine Case Management

chased by Dr. Khalid H. Seedahmed, Jr., MRCP PGDip, CertEndocrinology SA - ID 283


hCG stimulation during pregnancy is impaired and 2.5 mIU/L. An empirical increase in the
in women with high concentrations of thyroid dosage of levothyroxine upon finding of a positive
autoantibodies.6 pregnancy test is appropriate, and often women
As a result of these changes, thyroid are advised to double the dose on 2 days of the
dysfunction is best defined according to week. Regular tests of thyroid function (every
pregnancy-specific reference ranges that are 4 weeks) are recommended, especially during the
calculated in a population of euthyroid, iodine- first 20 weeks of gestation.8
sufficient pregnant women free of major factors The definition of subclinical hypothyroidism
that interfere with thyroid function. If such ranges and the cutoffs to be used for starting
are not available, then the adoption of pregnancy- levothyroxine replacement are much less clear.
specific ranges determined in a population with Progression rates to overt hypothyroidism are
similar characteristics is recommended. If no 2% to 5% per year and are higher in women
suitable generalizable ranges can be found, then with raised TPO antibodies and with TSH
current guidance suggests the use of 4 mIU/L concentrations greater than 10 mIU/L. Pregnancy
as the cutoff for TSH concentrations during risks associated with subclinical hypothyroidism
pregnancy. The previous pragmatic upper limit of have been demonstrated in large retrospective
normal of 2.5 mIU/L is now deemed too low.7 studies, although TSH thresholds have not
Most laboratories use free T4 immunoassays, been consistently defined and the largest effect
but due to changes in thyroid hormone– sizes are seen when population-based reference
binding proteins, the interpretation of free T4 ranges are used. A recent large individual patient
concentrations is difficult and ideally this should data meta-analysis (IPDM), including 47,045
be determined using liquid chromatography/mass pregnant women from 35 cohorts with a mean
spectrometry or equilibrium dialysis. The use of age of 29 years who had gestational blood
total T4 concentrations has also been proposed as sampling at a median of 12.9 weeks of pregnancy,
an alternative, but since there is large variation identified 1234 women (3.1%) with subclinical
in thyroxine-binding globulin concentrations, hypothyroidism (increased TSH concentration
especially during early pregnancy, these provide with normal free T4 concentration). The risk
only relatively crude measures.1 of preterm birth was higher for women with
subclinical hypothyroidism than for euthyroid
women (6.1% vs 5.0%, respectively; absolute risk
Management of Hypothyroidism
difference, 1.4% [95% confidence interval (CI),
Before and During Pregnancy 0%-3.2%]; odds ratio, 1.29 [95% CI, 1.01-1.64]).9
Overt hypothyroidism—raised serum TSH and Thyroid autoimmunity is a major risk factor
below-normal free T4 concentration—is usually for subclinical hypothyroidism, and approximately
caused by autoimmune Hashimoto thyroiditis one-third of women with subclinical
or following previous thyroidectomy or hypothyroidism are TPO positive. The risk of
radioiodine ablation in iodine-replete areas. When adverse pregnancy outcomes is higher in women
uncorrected during pregnancy, it is associated with combined thyroid autoimmunity and serum
with adverse outcomes, including miscarriage, TSH concentrations greater than 2.5 mIU/L. In
preeclampsia, low birth weight, and preterm recent years, a number of studies have evaluated
birth. In addition, maternal hypothyroidism has the effects of levothyroxine treatment on obstetric
been associated with detrimental effects of child outcomes in pregnant women with subclinical
neurodevelopment, illustrating the importance hypothyroidism. A large observational study
of timely levothyroxine administration. The found that a median levothyroxine dosage of
preconception and first trimester of pregnancy 50 mcg daily in pregnant women with serum
TSH target is between the lower reference limit TSH values of 2.5 to 10.0 mIU/L was associated

ENDO 2021 • Thyroid  343

chased by Dr. Khalid H. Seedahmed, Jr., MRCP PGDip, CertEndocrinology SA - ID 283


with a lower risk of pregnancy loss but increased 677 women with subclinical hypothyroidism. They
risks of premature delivery, gestational diabetes, were randomly assigned to receive levothyroxine
and preeclampsia compared with nontreatment. treatment or placebo starting at a mean of 17 to
Importantly. the beneficial effects of levothyroxine 18 weeks of gestation. No differences in child
replacement were predominantly present in the IQ were observed in offspring tested from 3 to
subgroup of women with TSH values greater than 5 years of age.15 Several explanations have been
4.0 mIU/L. This study highlights that overzealous proposed, including late treatment initiation
levothyroxine replacement in women with (median gestational age 13-18 weeks), possible
subclinical hypothyroidism may not be without overtreatment in the CATS trial, and high loss
risks and careful consideration should be given to follow-up (CATS study). Secondary analyses
when considering the start of thyroid hormone of obstetric outcomes did not identify beneficial
therapy in this situation.10 effects of levothyroxine in either of these trials.
In a small randomized controlled trial, Taken together, there is insufficient evidence
preterm delivery rates but not premature labor to universally treat pregnant women with serum
were reduced following levothyroxine treatment TSH values between 2.5 and 4 mIU/L regardless
of 65 women who were TPO positive and had of TPO antibody status. Levothyroxine should be
mild thyroid dysfunction (TSH = 2.5-10 mIU/L) considered in women who are TPO positive and
compared with those who did not receive thyroid have serum TSH values greater than 4.0 mIU/L,
hormone replacement. When 366 women, with TPO antibody–negative women with TSH values
TSH values between 2.5 and 4 mIU/L and negative greater than 10 mIU/L, those with other risk
for TPO antibodies, were randomly assigned factors for thyroid dysfunction, and those with
to receive levothyroxine or not, there were no infertility or recurrent pregnancy loss.3,7
differences in preterm delivery rates. However,
in those with TSH values greater than 4 mIU/L, Isolated Hypothyroxinemia
beneficial effects of levothyroxine on preterm
delivery were observed. In contrast, 2 clinical trials
During Pregnancy
have shown beneficial outcomes with reduced Isolated hypothyroxinemia is the biochemical
adverse pregnancy events in women treated with finding of a low serum free T4 concentration in
levothyroxine for serum TSH concentrations conjunction with a normal TSH concentration.
between 2.5 and 10 mIU/L during pregnancy.11,12 Contributory factors include iodine deficiency;
Two large clinical trials have evaluated the thyroid autoimmunity; assay techniques; and
benefits of maternal screening and levothyroxine maternal characteristics including age, BMI, and
therapy on child cognition. The controlled coexistent diabetes. Its prevalence ranges from
antenatal thyroid screening study (CATS) 1% to 10% in iodine-sufficient countries and 20%
randomly assigned 21,846 pregnant women to to 30% in areas of iodine deficiency. Maternal
a screening or control group and treated the hypothyroxinemia has been predominantly linked
screened group with levothyroxine if they had to adverse neurobehavioral outcomes in offspring,
subclinical hypothyroidism or hypothyroxinemia including reduced IQ, increased risk of autism
at a median gestational age of 13 weeks. At 3 spectrum disorder, attention deficit–hyperactivity
years, IQ scores were not different in offspring disorder, schizophrenia, and lower grey matter
from treated vs nontreated women.13 A follow- and cortical volumes.1,3 The randomized
up study of the CATS child cohort at 9 years controlled trials investigating potential benefits of
also failed to demonstrated beneficial effects of levothyroxine in women with hypothyroxinemia
levothyroxine treatment.14 The other multicenter have not shown beneficial effects on child
randomized controlled trial in the United States neurocognition.13,15
included 526 women with hypothyroxinemia and

344  ENDO 2021 • Endocrine Case Management

chased by Dr. Khalid H. Seedahmed, Jr., MRCP PGDip, CertEndocrinology SA - ID 283


Many studies have focused on the effects women; P = .001) and treatment with
of maternal thyroid hypofunction on child levothyroxine increased live birth rates
neurodevelopment. However, analysis of outcomes significantly (adjusted odds ratio, 3.7 [95% CI,
in offspring of children in the generation R study 1.4-9.8]; P = .007).20
indicate that high maternal free T4 concentrations In contrast, a randomized clinical trial of
are also associated with lower IQ scores, reduced 600 women undergoing in vitro fertilization and
grey matter, and lower cortical volume.16 embryo transfer found similar rates of miscarriage
Importantly, offspring from mothers “overtreated” (10.3% vs 10.6%, P = NS) in women treated with
in the CATS trial when high levothyroxine levothyroxine compared with those who were not
dosages were used had higher rates of attention treated. Furthermore, there were no differences
deficit–hyperactivity disorder compared with in live birth rates or intrauterine pregnancy rates
offspring from mothers whose thyroid function when comparing the intervention group with
was treated suboptimally.14 Since there is no the control group.21 The TABLET trial randomly
clear evidence of benefits of treating gestational assigned 952 women with raised TPO antibodies
hypothyroxinemia with levothyroxine and in the and biochemical euthyroidism to receive 50 mcg
presence of studies indicating potential harm, of levothyroxine or placebo across 49 United
this approach is generally not recommended,7 Kingdom centers. The live birth rate at 34 weeks’
although some guidelines suggest consideration of gestation was 37.4% in the levothyroxine group
treatment in early pregnancy.17 and 37.9% in the placebo group (relative risk,
0.97 [95% CI, 0.83-1.14]; P = .74). There were no
significant between-group differences in other
Levothyroxine Replacement in
pregnancy outcomes, including pregnancy loss or
Euthyroid TPO-Positive Women preterm birth, or in neonatal outcomes. Serious
Before and During Pregnancy adverse events occurred in 5.9% of women in
Women who have thyroid autoimmunity, the levothyroxine group and 3.8% in the placebo
evidenced by raised concentrations of TPO group (P = .14).22
antibodies, are at increased risk of miscarriage Overall, current evidence does not
and preterm delivery even when they are demonstrate that the use of levothyroxine results
biochemically euthyroid.5 Approximately one- in beneficial pregnancy outcomes in women
fifth of untreated TPO antibody–positive women with thyroid autoimmunity and normal thyroid
develop elevated serum TSH during the course function, indicating that the effects of TPO
of pregnancy. Two major hypotheses have been antibodies may not be related through thyroid
proposed: (1) TPO antibody positivity reflects a dysfunction but rather reflect a generalized adverse
general diffuse autoimmune process potentially autoimmune state.
resulting in rejection of the fetal allograft or
(2) TPO antibody positivity drives thyroid Management of Hyperthyroidism
dysfunction and subsequently results in poor
obstetric outcomes.
Before and During Pregnancy
Previous studies have shown reduced The main subtypes of thyrotoxicosis during
miscarriage rates following levothyroxine pregnancy are Graves disease and transient
treatment in TPO antibody–positive euthyroid gestational thyrotoxicosis caused by high hCG
women.18,19 A recent prospective study of concentrations stimulating the TSH receptor.
825 women with recurrent pregnancy loss Preexisting Graves disease is present in about
(≥3 pregnancy losses) found that TPO antibody 0.5% to 1.3% of pregnancies and new-onset
positivity was associated reduced live birth rate autoimmune hyperthyroidism occurs in 0.05%.
(51.3% vs 65.2% in TPO antibody–negative When thyrotoxicosis develops newly during

ENDO 2021 • Thyroid  345

chased by Dr. Khalid H. Seedahmed, Jr., MRCP PGDip, CertEndocrinology SA - ID 283


early pregnancy, it is important to distinguish or during early pregnancy and if antithyroid drugs
the small minority of women with pathological are still required beyond 16 weeks’ gestation, then
hyperthyroidism from those who have gestational often propylthiouracil is switched to methimazole,
thyrotoxicosis. Measurement of TSH-receptor although not all studies indicate that this approach
antibodies (TRAb) and free T3 concentrations is associated with improved outcomes.23,24
are informative, as they will be raised in the Women of childbearing age with active
former but not in the latter. Transient rises in hyperthyroidism should be counseled on the risk
circulating thyroid hormones are usually treated in of fetal harm from uncontrolled hyperthyroidism
a supportive manner, as these patients often have and advised to delay pregnancy with effective
hyperemesis gravidarum.1,3 contraception until a stable euthyroid state
Uncontrolled overt hyperthyroidism before has been achieved. Definitive treatment with
and during pregnancy is associated with increased radioiodine or surgery is an option since this will
risks of miscarriage, stillbirth, intrauterine growth avoid exposure to the risks of antithyroid drugs
retardation, maternal hypertension, and heart during pregnancy, as well as the risk of relapse
failure, whereas subclinical hyperthyroidism of Graves disease following delivery. Pregnancy
has not been associated with adverse pregnancy should be avoided during the 6 months following
outcomes. If Graves disease is newly diagnosed radioiodine administration. Circulating maternal
during pregnancy, antithyroid drug treatment is TRAbs cross the placenta and may induce fetal
usually required. Outside pregnancy, methimazole hyperthyroidism and goiter.1,3 An important
is the most commonly used antithyroid drug, consideration is that pregnant women with
but since this is associated with a higher risk Graves disease who were rendered hypothyroid
of teratogenicity than propylthiouracil,23,24 the by radioiodine therapy or surgery may continue
latter is usually preferred before and during to harbor high levels of TRAbs, thereby placing
the first 16 weeks of pregnancy. The highest the fetus at risk of thyroid dysfunction. Persistence
risk window for birth defects is from gestation of TRAbs well beyond 18 months following
weeks 6 to 10,25 and some practitioners prefer treatment has been documented in 75% of those
to avoid antithyroid drugs altogether during who received radioiodine vs only 30% of patients
this period. Propylthiouracil has been associated treated with surgery and 3% of those treated with
with increased risks of liver dysfunction and antithyroid drugs.27 Measurement of TRAbs
liver failure both within and outside pregnancy.26 in early pregnancy is indicated in those with
Block-and-replace regimens are not advisable preexisting Graves disease and is often repeated
during pregnancy since antithyroid drugs around 20 weeks’ gestation. Regular monitoring
preferentially cross the placenta, resulting in with assessment of fetal heart rate, fetal growth,
increased risk of fetal hypothyroidism. and fetal thyroid is appropriate in those with
The aim of treatment of gestational significantly elevated TRAb concentrations.
hyperthyroidism is to control hyperthyroidism and
to limit fetal-maternal drug exposure, using the
lowest possible dosage to maintain normal thyroid
Clinical Case Vignettes
status. Thyroid function can change relatively Case 1
quickly during early pregnancy, and regular A 32-year-old woman presents for her first
testing every 2 to 4 weeks is advisable. Preexisting antenatal appointment at 8 weeks’ gestation. She
Graves disease often goes into remission during has had 3 first-trimester miscarriages. She reports
pregnancy as a result of decline in thyroid tiredness and some abdominal bloating. Physical
autoimmunity (TRAb concentrations) and often examination suggests she is clinically euthyroid.
antithyroid drugs can be discontinued. Usually Her BMI is 25 kg/m2.
methimazole is switched to propylthiouracil before

346  ENDO 2021 • Endocrine Case Management

chased by Dr. Khalid H. Seedahmed, Jr., MRCP PGDip, CertEndocrinology SA - ID 283


Laboratory test results: Laboratory test results:
TSH = 4.2 mIU/L (0.4-4.0 mIU/L) TSH = 1.4 mIU/L (0.4-4.0 mIU/L)
Free T4 = 0.9 ng/dL (0.8-1.8 ng/dL) (SI: 11.6 pmol/L Free T4 = 0.6 ng/dL (0.8-1.8 ng/dL) (SI: 7.7 pmol/L
[10.30-23.17 pmol/L]) [10.30-23.17 pmol/L])
TPO antibodies = 350 IU/mL (<50 IU/mL) TPO antibodies = 40 IU/mL (<50 IU/mL)

Which of the following should be advised? Which of the following should be advised?
A. Reassure patient and discharge to routine A. Measure free T3 concentration and start
antenatal care levothyroxine if this is low
B. Repeat thyroid function tests after 4 weeks B. Urgently measure 9-AM cortisol concentration
C. Repeat thyroid function tests at 20 weeks’ C. Repeat thyroid function tests after 4 weeks
gestation D. Start levothyroxine, 1.6 mcg/kg daily
D. Start levothyroxine, 1.6 mcg/kg daily E. Start levothyroxine, 25 to 50 mcg daily
E. Start levothyroxine, 25 to 50 mcg daily
Answer: C or E) Repeat thyroid function tests after
Answer: B or E) Repeat thyroid function tests after 4 weeks or start levothyroxine, 25 to 50 mcg daily
4 weeks or start levothyroxine, 25 to 50 mcg daily
This patient has isolated hypothyroxinemia. Since
This patient has a history of recurrent miscarriage she conceived spontaneously, it seems unlikely
and has evidence of thyroid autoimmunity and that she has significant pituitary dysfunction, and
very mild subclinical hypothyroidism. While measurement of a 9-AM cortisol concentration
the evidence supporting levothyroxine use in (Answer B) is not required. Clinical trials of
patients with thyroid autoimmunity who are levothyroxine replacement in this setting have
euthyroid is not convincing, the finding of a not indicated significantly improved obstetric
serum TSH concentration greater than 4 mIU/L in or neurocognitive outcomes. Current American
association with raised TPO antibodies represents Thyroid Association guidelines7 do not
a reasonable indication to start low-dosage recommend treatment of this condition, whereas
levothyroxine (Answer E). Repeating thyroid European guidance suggests that treatment
function testing after 4 weeks (Answer B) is may be considered in the first trimester of
appropriate regardless of whether levothyroxine pregnancy.17 Repeating thyroid function tests after
is started. 4 weeks (Answer C) seems a reasonable option,
although some practitioners may favor starting
levothyroxine, 25 to 50 mcg daily (Answer E).
Case 2
Measurement of free T3 (Answer A) is usually not
You are asked for advice regarding a 29-year-old indicated in the management of hypothyroidism
woman who is 13 weeks pregnant in her third either during or outside of pregnancy.
pregnancy. She conceived spontaneously. Her first
child was born prematurely at 33 weeks’ gestation,
and she underwent cesarean delivery at 38 weeks’ Case 3
gestation with her second child. Her second A 28-year-old woman with Graves disease
pregnancy was complicated by gestational diabetes. attends a follow-up appointment and states that
She is clinically euthyroid and has no palpable she wishes to become pregnant. Graves disease
goiter. Her BMI is 29 kg/m2. was diagnosed 18 months ago and TRAbs were
raised at presentation (5.2 IU/L [<1.0 IU/L]). She
is taking methimazole, 30 mg daily. She has not
been pregnant before. She is clinically euthyroid

ENDO 2021 • Thyroid  347

chased by Dr. Khalid H. Seedahmed, Jr., MRCP PGDip, CertEndocrinology SA - ID 283


and has a palpable small diffuse goiter. Her BMI is Case 4  New Content 
23 kg/m2.
You are asked for advice on a 34-year-old woman
Laboratory test results: who is 14 weeks pregnant. This is her first
pregnancy. She has a history of relapsing-remitting
TSH = <0.01 mIU/L (0.4-4.0 mIU/L)
multiple sclerosis and received a second dose of
Free T4 = 1.6 ng/dL (0.8-1.8 ng/dL) (SI: 20.6 pmol/L
[10.30-23.17 pmol/L]) alemtuzumab 10 months earlier. Graves disease
Total T3 = 160 ng/dL (80-180 ng/dL) (SI: 2.5 nmol/L was diagnosed 8 months ago and has been treated
[1.2-2.8 nmol/L]) with increasing dosages of antithyroid drugs. She
has a history of primary infertility but conceived
this pregnancy spontaneously. After a positive
Which of the following should be advised?
pregnancy test, her regimen was switched from
A. Continue current dosage of methimazole methimazole, 80 mg daily, to propylthiouracil,
B. Change to propylthiouracil, 300 mg twice daily 400 mg twice daily.
C. Advise radioiodine therapy On physical examination, her pulse rate is
D. Advise total thyroidectomy 110 beats/min in a regular rhythm, and she has a
fine tremor of the hands. A moderately enlarged
E. Discontinue antithyroid drug treatment
diffuse goiter is palpable in her neck. She reports
Answer: B, C, or D) Change to propylthiouracil, increasing grittiness of her eyes and has a clinical
300 mg twice daily or advise radioiodine activity score of 3. Her BMI is 24 kg/m2.
therapy or advise total thyroidectomy
Laboratory test results (see table below).
This patient has treated Graves disease that is
controlled, although serum TSH remains fully Which of the following should be advised?
suppressed. Discontinuation of antithyroid drugs A. Increase the dosage of propylthiouracil to
(Answer E) would be inappropriate. In view 400 mg 3 times daily
of risks of teratogenicity, many practitioners B. Change propylthiouracil to methimazole,
and current guidelines7 recommend changing 80 mg daily
methimazole to propylthiouracil (Answer B), and C. Recommend urgent thyroidectomy at 18 to
continuation of methimazole (Answer A) is a 20 weeks’ gestation
less preferred option. Consideration of definitive
treatment with radioiodine (Answer C) or surgery D. Advise termination of pregnancy
(Answer D) would also be appropriate. E. Start glucocorticoid treatment

Timepoint

Measurement 8 Months Ago 2 Months Ago Today

Serum TSH <0.01 mIU/L <0.01 mIU/L <0.01 mIU/L

Serum free T4 4.9 ng/dL (SI: 63.1 pmol/L) 4.5 ng/dL (SI: 57.9 pmol/L) 4.8 ng/dL (SI: 61.8 pmol/L)

Serum free T3 14.8 pg/mL (SI: 22.7 pmol/L) 13.8 pg/mL (SI: 21.2 pmol/L) 14.1 pg/mL (SI: 21.7 pmol/L)

TRAb 265 IU/L … 378 IU/L

Reference ranges: serum TSH, 0.4-4.0 mIU/L; serum free T4, 0.8-1.8 ng/dL (SI: 10.3-23.2 pmol/L); serum free T3, 2.3-4.2 pg/mL
(SI: 3.5-6.5 pmol/L); TRAb, <1.0 IU/L.

348  ENDO 2021 • Endocrine Case Management

chased by Dr. Khalid H. Seedahmed, Jr., MRCP PGDip, CertEndocrinology SA - ID 283


Answer: B or C) Change propylthiouracil to Which of the following should be advised?
methimazole, 80 mg daily, OR recommend urgent A. Start levothyroxine, 1.6 mcg/kg daily
thyroidectomy at 18 to 20 weeks’ gestation B. Start levothyroxine, 25 to 50 mcg daily
This patient has alemtuzumab-induced Graves C. Reassure the patient and recommend routine
disease,28 which has not responded to high antenatal care
dosages of antithyroid drugs as evidenced by high D. Repeat thyroid function tests in the first
concentrations of circulating thyroid hormones trimester of pregnancy
and TRAb. She is now in the second trimester E. Recommend glucocorticoid treatment in the
of pregnancy, and susceptibility of the fetus to first trimester of pregnancy
the teratogenic effects of methimazole is much
reduced.7 She is already taking high dosages Answer: D) Repeat thyroid function tests
of propylthiouracil, and it seems unlikely that in the first trimester of pregnancy
further increasing the dosage (Answer A) will
result in control of hyperthyroidism. Changing This patient has evidence of thyroid autoimmunity
antithyroid drug treatment to high-dosage (raised TPO antibodies) and is clinically and
methimazole (Answer B) is likely to be a better biochemically euthyroid. She also has a history
option, although use of this treatment prior to of recurrent miscarriages. While some smaller
pregnancy did not result in control of Graves trials18,19 have indicated potential beneficial
disease. Urgent thyroidectomy, ideally undertaken effects of levothyroxine treatment in reducing
around 18 to 20 weeks’ gestation7 (Answer C), miscarriage in euthyroid TPO antibody–
is likely to result in more rapid restoration of positive women, the TABLET trial22 did not
euthyroidism. There is no indication to terminate show improved live birth rates at 34 or more
pregnancy (Answer D). There is no evidence that weeks’ gestation following administration of
glucocorticoid treatment (Answer E) will help levothyroxine, 50 mcg daily. Similarly, in women
restore euthyroidism, although glucocorticoids undergoing in vitro fertilization egg transfer, early
may be beneficial if urgent thyroidectomy is and late miscarriage rates were not improved
undertaken. in TPO antibody–positive women following
levothyroxine therapy.21 Answers A and B are
therefore less preferred options. Women with
Case 5  New Content  raised TPO antibodies are at increased risk
A 38-year-old woman with a history of 2 first- of developing overt hypothyroidism, and it is
trimester miscarriages presents for prepregnancy reasonable to repeat thyroid function testing
counseling. She has no notable medical history. early in pregnancy (Answer D) rather than simply
She reports a family history of hypothyroidism providing routine antenatal care (Answer C).
in her mother. She is not taking any regular The mechanisms underlying the increased risk
medication. She is clinically euthyroid and has no of miscarriage in TPO antibody–positive women
palpable goiter. Her BMI is 36 kg/m2. remain poorly understood and may be related to a
more generalized autoimmune process. However,
Laboratory test results: there is no current evidence that the use of
TSH = 2.4 mIU/L (0.4-4.0 mIU/L) glucocorticoids (Answer E) is beneficial in terms
Free T4 = 1.1 ng/dL (0.8-1.8 ng/dL) (SI: 14.2 pmol/L of improved live birth rates.
[10.3-23.2 pmol/L])
TPO antibodies = 320 IU/mL (<50 IU/mL)

ENDO 2021 • Thyroid  349

chased by Dr. Khalid H. Seedahmed, Jr., MRCP PGDip, CertEndocrinology SA - ID 283


References
1. Korevaar TIM, Medici M, Visser TJ, Peeters RP. Thyroid disease in 15. Casey BM, Thom EA, Peaceman AM, et al. Treatment of subclinical
pregnancy: new insights in diagnosis and clinical management. Nat Rev hypothyroidism or hypothyroxinemia in pregnancy. N Engl J Med.
Endocrinol. 2017;13(10):610-622. PMID: 28776582 2017;376(9):815-825. PMID: 28249134
2. Moog NK, Entringer S, Heim C, Wadhwa PD, Kathmann N, Buss C. 16. Korevaar TI, Muetzel R, Medici M, et al. Association of maternal thyroid
Influence of maternal thyroid hormones during gestation on fetal brain function during early pregnancy with offspring IQ and brain morphology
development. Neuroscience. 2017;342:68-100. PMID: 26434624 in childhood: a population-based prospective cohort study. Lancet Diabetes
3. Okosieme OE, Khan I, Taylor PN. Preconception management of thyroid Endocrinol. 2016;4(1):35-43. PMID: 26497402
dysfunction. Clin Endocrinol (Oxf). 2018;89(3):269-279. PMID: 29706030 17. Lazarus J, Brown RS, Daumerie C, Hubalewska-Dydejczyk A, Negro R,
4. Dong AC, Stagnaro-Green A. Differences in diagnostic criteria mask the true Vaidya B. 2014 European Thyroid Association guidelines for the management
prevalence of thyroid disease in pregnancy: a systematic review and meta- of subclinical hypothyroidism in pregnancy and in children. Eur Thyroid J.
analysis. Thyroid. 2019;29(2):278-289. PMID: 30444186 2014;3(2):76-94. PMID: 25114871
5. Thangaratinam S, Tan A, Knox E, Kilby MD, Franklyn J, Coomarasamy A. 18. Negro R, Formoso G, Mangieri T, Pezzarossa A, Dazzi D, Hassan H.
Association between thyroid autoantibodies and miscarriage and preterm Levothyroxine treatment in euthyroid pregnant women with autoimmune
birth: meta-analysis of evidence. BMJ. 2011;342:d2616. PMID: 21558126 thyroid disease: effects on obstetrical complications. J Clin Endocrinol Metab.
6. Korevaar TI, Steegers EA, Pop VJ, Broeren MA, Chaker L, de Rijke YB, et al. 2006;91(7):2587-2591. PMID: 16621910
Thyroid autoimmunity impairs the thyroidal response to human chorionic 19. Nazarpour S, Ramezani Tehrani F, Simbar M, Tohidi M, Alavi Majd H, Azizi
gonadotropin: two population-based prospective cohort studies. J Clin F. Effects of levothyroxine treatment on pregnancy outcomes in pregnant
Endocrinol Metab. 2017;102(1):69-77. PMID: 27754809 women with autoimmune thyroid disease. Eur J Endocrinol. 2017;176(2):253-
7. Alexander EK, Pearce EN, Brent GA, Brown RS, Chen H, Dosiou C, et al. 265. PMID: 27879326
2017 Guidelines of the American Thyroid Association for the diagnosis 20. Bliddal S, Feldt-Rasmussen U, Rasmussen AK, et al. Thyroid peroxidase
and management of thyroid disease during pregnancy and the postpartum. antibodies and prospective live birth rate: a cohort study of women with
Thyroid. 2017;27(3):315-389. PMID: 28056690 recurrent pregnancy loss. Thyroid. 2019;29(10):1465-1474. PMID: 31407629
8. Chan S, Boelaert K. Optimal management of hypothyroidism, 21. Wang H, Gao H, Chi H, et al. Effect of levothyroxine on miscarriage among
hypothyroxinaemia and euthyroid TPO antibody positivity preconception women with normal thyroid function and thyroid autoimmunity undergoing
and in pregnancy. Clin Endocrinol (Oxf). 2015;82(3):313-326. PMID: 25200555 in vitro fertilization and embryo transfer: a randomized clinical trial. JAMA.
9. Consortium on Thyroid Pregnancy--Study Group on Preterm Birth, 2017;318(22):2190-2198. PMID: 29234808
Korevaar TIM, Derakhshan A, et al. Association of thyroid function test 22. Dhillon-Smith RK, Middleton LJ, Sunner KK, Cheed V, Baker K, Farrell-
abnormalities and thyroid autoimmunity with preterm birth: a systematic Carver S, et al. Levothyroxine in women with thyroid peroxidase antibodies
review and meta-analysis. JAMA. 2019;322(7):632-641. PMID: 31429897 before conception. N Engl J Med. 2019;380(14):1316-1325. PMID: 30907987
10. Maraka S, Mwangi R, McCoy RG, et al. Thyroid hormone treatment among 23. Andersen SL, Olsen J, Wu CS, Laurberg P. Birth defects after early pregnancy
pregnant women with subclinical hypothyroidism: US national assessment. use of antithyroid drugs: a Danish nationwide study. J Clin Endocrinol Metab.
BMJ. 2017;356:i6865. PMID: 28122781 2013;98(11):4373-4381. PMID: 24151287
11. Negro R, Schwartz A, Gismondi R, Tinelli A, Mangieri T, Stagnaro-Green 24. Seo GH, Kim TH, Chung JH. Antithyroid drugs and congenital
A. Universal screening versus case finding for detection and treatment of malformations: a nationwide Korean cohort study. Ann Intern Med.
thyroid hormonal dysfunction during pregnancy. J Clin Endocrinol Metab. 2018;168(6):405-413. PMID: 29357398
2010;95(4):1699-1707. PMID: 20130074 25. Laurberg P, Andersen SL. Therapy of endocrine disease: antithyroid drug use
12. Ma L, Qi H, Chai X, et al. The effects of screening and intervention in early pregnancy and birth defects: time windows of relative safety and high
of subclinical hypothyroidism on pregnancy outcomes: a prospective risk? Eur J Endocrinol. 2014;171(1):R13-R20. PMID: 24662319
multicenter single-blind, randomized, controlled study of thyroid 26. Taylor PN, Vaidya B. Side effects of anti-thyroid drugs and their impact
function screening test during pregnancy. J Matern Fetal Neonatal Med. on the choice of treatment for thyrotoxicosis in pregnancy. Eur Thyroid J.
2016;29(9):1391-1394. PMID: 26181769 2012;1(3):176-185. PMID: 24783017
13. Lazarus JH, Bestwick JP, Channon S, et al. Antenatal thyroid screening and 27. Laurberg P, Wallin G, Tallstedt L, Abraham-Nordling M, Lundell G, Torring
childhood cognitive function [published correction appears in N Engl J Med. O. TSH-receptor autoimmunity in Graves’ disease after therapy with anti-
2012;366(17):1650]. N Engl J Med. 2012;366(6):493-501. PMID: 22316443 thyroid drugs, surgery, or radioiodine: a 5-year prospective randomized
14. Hales C, Taylor PN, Channon S, et al. Controlled antenatal thyroid screening study. Eur J Endocrinol. 2008;158(1):69-75. PMID: 18166819
II: effect of treating maternal sub-optimal thyroid function on child 28. Muller I, Moran C, Lecumberri B, et al. 2019 European Thyroid Association
behaviour. J Clin Endocrinol Metab. [Epub ahead of print] guidelines on the management of thyroid dysfunction following immune
reconstitution therapy. Eur Thyroid J. 2019;8(4):173-185. PMID: 31602359

350  ENDO 2021 • Endocrine Case Management

chased by Dr. Khalid H. Seedahmed, Jr., MRCP PGDip, CertEndocrinology SA - ID 283


MISCELLANEOUS

chased by Dr. Khalid H. Seedahmed, Jr., MRCP PGDip, CertEndocrinology SA - ID 283


A Little Help From My
Friends: The Endocrinology–
Oncology Interface—How
Do We Help Each Other?
Claire Higham, DPhil, FRCP. Christie Hospital, Manchester, United Kingdom; E-mail:
cehigham@doctors.org.uk

Sacha Howell, PhD, FRCP. Manchester University, Manchester, United Kingdom; E-mail:
sacha.howell@manchester.ac.uk

Peter Trainer, MD, FRCPE. The Christie NHS Foundation Trust, Manchester, United
Kingdom; E-mail: peter.trainer@manchester.ac.uk

Learning Objectives Significance of the


Clinical Problem
• Explain why the initial clinical and biochemical
assessment of patients with hyponatremia is Improved survival and novel modalities of therapy
crucial to optimal management. mean that evermore patients with cancer will
experience endocrine complications. In particular,
• Recommend the correct management immunotherapy with checkpoint inhibitors is
of endocrine complications of immune improving outcomes in an increasing number of
checkpoint inhibitor therapy to avoid cancers, but at the expense of immune-mediated
discontinuation of this therapy. endocrinopathies such as hypophysitis, adrenalitis,
• Anticipate the implications of systemic thyroiditis, and diabetes mellitus, which can be
anticancer therapy for bone health and serious, or even fatal. There has never been a
mitigate these adverse effects with lifestyle greater need for endocrine input into the care of
advice, bone density monitoring, and timely patients being treated for cancer.
pharmacological intervention.
Barriers to Optimal Practice
Main Conclusions • Lack of patient education/awareness.
Endocrine complications of cancer and its • Failure to anticipate endocrine complications
treatment are common, varied in nature, and of systemic anticancer therapy.
frequently complex, and some are all too frequently • Inadequate collaboration between oncologists
overlooked (eg, dexamethasone-induced diabetes). and endocrinologists.
Endocrinologists are vital partners to oncologists in
ensuring that the endocrinopathies do not delay or
prohibit anticancer therapy.

352  ENDO 2021 • Endocrine Case Management

chased by Dr. Khalid H. Seedahmed, Jr., MRCP PGDip, CertEndocrinology SA - ID 283


Strategies for Diagnosis, He is euvolemic. Additional laboratory test results
include the following:
Therapy, and/or Management
Serum osmolality = 226 mOsm/kg
Hyponatremia (275-295 mOsm/kg)
Syndrome of inappropriate antidiuretic hormone Urinary osmolality = 611 mOsm/kg
secretion (SIADH) is common in patients with (150-1150 mOsm/kg)
Urinary sodium = 42 mEq/L (SI: 45 mmol/L)
cancer, but incorrect diagnosis and subsequent Urinary potassium = 70 mEq/L (SI: 70 mmol/L)
incorrect management of hyponatremia increases
the risk of mortality. The clinical presentation of
hyponatremia depends on the severity, speed of Question 1: What is this patient’s diagnosis?
onset, and underlying diagnosis. Answer: SIADH

Acute Life-Threatening Hyponatremia A diagnosis of SIADH requires 7 conditions to be


Severe symptoms of acute life-threatening met (in the untreated state):
hyponatremia include vomiting, seizures, and • Clinical euvolemia
reduced consciousness/coma.
• Serum osmolality <275 mOsm/kg
Non–Life-Threatening Hyponatremia • Urine osmolality ≥100 mOsm/kg
Patients with non–life-threatening hyponatremia • Urinary sodium ≥20 mEq/L (≥20 mmol/L)
may be asymptomatic or experience moderately
• Normal thyroid function
severe symptoms such as nausea without vomiting,
confusion, and headache. Mild symptoms can • Normal adrenal function
include lethargy and irritability. • Exclusion of drug-induced hyponatremia
Hyponatremia can also be classified based on
the serum sodium concentration: Evaluation and differential diagnosis include
exploring symptoms as a measure of severity and
Mild: 130-135 mEq/L (SI: 130-135 mmol/L) rapidity of onset. The medical history should be
Moderate: 126-129 mEq/L (SI: 126-129 mmol/L)
Profound: ≤125 mEq/L (SI: ≤125 mmol/L) documented, including a timeline of change of
serum sodium, comorbidities (eg, type and stage
of malignancy, alcohol dependency, liver disease,
Clinical Case Vignette 1 heart failure), and drug history (especially new
medications). Volume status should be determined
A 55-year-old man with small cell lung carcinoma (hypervolemia, hypovolemia, or euvolemia).
presents with breathlessness, anorexia, sweats, and Investigations for the differential diagnosis of
confusion. hyponatremia are shown in Figure 1.
Laboratory test results: • Measurement of serum osmolality, cortisol
Sodium = 110 mEq/L (136-142 mEq/L) (document exogenous glucocorticoids),
(SI: 110 mmol/L [136-142 mmol/L]) thyroid function, and glucose
Serum urea nitrogen = 12.6 mg/dL (8-23 mg/dL)
(SI: 4.5 mmol/L [2.9-8.2 mmol/L]) • Measurement of urine osmolality and urine
Potassium = 4.6 mEq/L (3.5-5.0 mEq/L) electrolytes (sodium and potassium)
(SI: 4.6 mmol/L [3.5-5.0 mmol/L])
Creatinine = 0.58 mg/dL (0.7-1.3 mg/dL)
(SI: 51 µmol/L [61.9-114.9 µmol/L]) Question 2: What is the best treatment?
Answer: Depends on the severity, speed
of onset, and underlying diagnosis

ENDO 2021 • Miscellaneous  353

chased by Dr. Khalid H. Seedahmed, Jr., MRCP PGDip, CertEndocrinology SA - ID 283


Figure 1. Algorithm for the Differential Diagnosis of Hyponatremia

[_ _ _] Hyponatremia
Serum sodium <130 mEq/L (<130 mmol/L)

Measure the following: *1


.........[_ _ _] If acute, life-threatening symptoms see
algorithm 1 for emergency treatment

>275 mOsm/kg Measure capillary blood glucose


Serum osmolality
(serum sample – brown tube)
r.::::::=====...............
Urine osmolality
Isotonic/hypertonic
hyponatremia I and plasma glucose: exclude
hyperglycemia *2
I
~~ [ ]
and urinary electrolytes (Na & K) <100 mOsm/kg
(send to biochemistry in 60 mL screw-top container) Appropriate renal dilution
If urinary sodium <20 mEq/L =
likely polydipsia/inappropriate IV fluid
Serum cortisol/thyroid function tests *3
(serum sample – brown tube)

<275 mOsm/kg >100 mOsm/kg


Hypotonic hyponatremia Inappropriate renal dilution

Extracellular Fluid Volume Status

• Hypovolemia
• Euvolemia
• Hypervolemia

[__][__][==]
• Renal sodium loss
(urine sodium >20 mEq/L)
• Extra-renal sodium loss
(urine sodium <20 mEq/L)
• SIADH
• Glucocorticoid deficiency
• Hypothyroidism
(urine sodium >20 mEq/L)
• Renal failure
(urine sodium >20 mEq/L)
• Heart failure/cirrhosis
(urine sodium <20 mEq/L)

Treatment of SIADH urinary sodium concentration +


The cause of SIADH should be treated (eg, urinary potassium concentration
chemotherapy for small cell lung cancer). Fluid serum sodium concentration
restriction is unlikely to be effective and has
limited application in the oncology population. A Demeclocycline is too slow to act and is of limited
high urine osmolality makes fluid restriction less benefit. It induces nephrogenic diabetes insipidus
likely to work. The urine to plasma electrolyte and can take 2 to 14 days to be effective. The
ratio is a useful guide for appropriate fluid starting dosage is 150 mg twice daily, increased
restriction (Table 1). to a maximum dosage of 300 mg 4 times daily.
Advantages are that it may be easier to tolerate
Table 1. Fluid Restriction Guide Based on
than fluid restriction and can be initiated
the Urine to Plasma Electrolyte Ratio as an outpatient or inpatient. Cautions and
disadvantages are that it may not be effective
Ratio Fluid Restriction
and it has slow onset of action. Adverse effects
>1 unlikely to be effective can include nausea, skin photosensitivity, and
0.5-1.0 <0.5 L nephrotoxicity. Demeclocycline should not to be
<0.5 <1 L
used in patients with renal impairment.
Tolvaptan, when used appropriately, corrects
sodium and permit chemotherapy to be started.
The Furst formula =

354  ENDO 2021 • Endocrine Case Management

chased by Dr. Khalid H. Seedahmed, Jr., MRCP PGDip, CertEndocrinology SA - ID 283


Figure 2. Safe Use of Tolvaptan
Check baseline serum sodium on day of administration Risks for over
rapid correction

) (
• Low Urea
• Older age
• Ensure thirst perception is intact and fluid restriction is stopped
• Ensure patient was not prescribed demeclocycline in previous 24 hours

) (
Single dose of tolvaptan, 7.5 mg, prescribed and administered before 11 AM
) (
• Ensure free access to water
• Advise patient to drink to thirst: monitor urine output

) (
Measure serum sodium 6-8 hours post tolvaptan (maximum diuresis usually 4-6 hours)
) (
• •
Sodium rise 12 hours: if sodium rise Sodium rise
≥6 mEq/L (≥6 mmol/L) ≥8 mEq/L (≥8 mmol/L) <6 mEq/L (<6 mmol/L)
Risk for central pontine demyelination *
J J L L
Recheck serum sodium
• Oral water: 1 L over 30 minutes and check
• 12 hours
serum sodium after 2 hours

+
• 24 hours
• d/w endocrinologist
...,_
, ' ,
24 hours: if sodium rise ≥12 mEq/L (≥12 mmol/L)
The safe use of tolvaptan is depicted in Figure 2. serum sodium within first 6 hours or an 8 mEq/L
Tolvaptan is a selective V2-receptor antagonist or greater (≥8 mmol/L) rise in 12 hours, hypotonic
in the renal collecting ducts and it increases fluid may be required (oral water). Cautions
water excretion. In the presence of hypovolemic and disadvantages include the risk of too rapid
hyponatremia and/or lack of appropriate correction of sodium, which could potentially lead
monitoring of serum sodium and fluid balance, to osmotic demyelination. It should be used with
there can be a potentially life-threatening rapid caution in patients with chronic liver disease/
rise in sodium. If switching from demeclocycline, cirrhosis.
it is ideal to wait 48 hours before administering
the first dose of tolvaptan. Importantly, the patient Management of Acute, Life-
must not be confused or fluid restricted and must Threatening Hyponatremia
have thirst perception. Initially, it is recommended The aim is to achieve a 5 mEq/L (5 mmol/L)
to start with a single dose of tolvaptan of 7.5 mg rise in serum sodium within the first hour. If the
(half the recommended dose). Subsequent doses clinical status of the patient does not improve
should be prescribed and administered in after a 5 mEq/L (5 mmol/L) rise in serum sodium,
accordance with serum sodium and symptoms. then additional hypertonic saline may be required
Following tolvaptan administration, serum sodium and further exploration of symptoms undertaken.
should be checked after 6 hours and 12 hours. If The management of acute, life-threatening
there is a 6 mEq/L or greater (≥6 mmol/L) rise in hyponatremia is depicted in Figure 3.
ENDO 2021 • Miscellaneous  355
hased by Dr. Khalid H. Seedahmed, Jr., MRCP PGDip, CertEndocrinology SA - ID 28
Figure 3. Management of Acute, Life-Threatening multiorgan immune adverse effects (eg, colitis and
Hyponatremia hypophysitis).
Early recognition and appropriate
Severe symptomatic hyponatremia — for hypertonic saline management of endocrinopathies associated
• Discuss with critical care/outreach —
with immune checkpoint inhibitor therapy are
*requires high monitoring environment*
• Baseline serum blood and urine samples as per algorithm 2
essential. Patients should be educated about risk

] [
and symptoms of cortisol deficiency. Thyroid
function should be checked with every cycle of

Within first hour immune checkpoint inhibitor therapy. Endocrine
• IV infusion 150 mL 3% hypertonic saline or *equivalent complications most commonly develop within
over 20 minutes
(* 2.7% 500 mL polyfuser available in Christie pharmacy/critical care) 4 cycles of commencing treatment. However,

] [
endocrine complications are not a reason to
discontinue immune checkpoint inhibitor therapy.

Check serum sodium
• IV infusion 150 mL 3% hypertonic saline or *equivalent Thyroiditis
over 20 minutes while awaiting result
(* 2.7% 500 mL polyfuser available in Christie pharmacy/critical care)
Thyroiditis in this setting tends to be

] [
transient, self-limiting hyperthyroidism that
is often asymptomatic. Thionamide treatment

• Repeat twice or until 5 mEq/L (5 mmol/L) increase in (carbimazole/propylthiouracil) is of no value.
serum sodium Thyroiditis usually progresses to irreversible
• Limit rise in serum sodium to 1 mEq/L (1 mmol/L) over 2 hours
and <8 mEq/L (<8 mmol/L) over 24 hours hypothyroidism. Cortisol deficiency must be

] [
excluded before levothyroxine is initiated. There
have been reports of Graves hyperthyroidism and

Follow-up management after 5 mEq/L (5 mmol/L) increase
ophthalmopathy, thyroid storm, and myxedema.
in serum sodium and clinical improvement:
• Stop infusion of hypertonic saline Hypophysitis
• Keep IV line open (minimum volume of 0.9% saline)
Hypophysitis presents with mass effect and/or
• Start diagnosis-specific management (see algorithm 2)
• Limit rise in serum sodium to 1 mEq/L (1 mmol/L) over 2 hours hormone deficiency, particularly hypoadrenalism.
and <8 mEq/L (<8 mmol/L) over 24 hours It inevitably results in permanent hypopituitarism.
Hypophysitis frequently presents as isolated ACTH
Ball S, Barth J, Levy M; Society for Endocrinology Clinical Committee. deficiency, and the differential diagnosis includes
Society for Endocrinology Endocrine Emergency Guidance: emergency
management of severe symptomatic hyponatraemia in adult patients. acute adrenalitis. A diagnostic pitfall is the use of
Endocr Connect. 2016;5(5):G4-G6.
exogenous glucocorticoid therapy (eg, inhaler), but
in particular dexamethasone used as an antiemetic.
Immune Checkpoint Inhibitors Evaluation includes tests of basal pituitary function
and pituitary MRI, with dynamic testing as indicated.
Immune checkpoint inhibitors are being used Management of hormone deficiency should be
to treat an increasing number of cancers. addressed as per standard protocols (except growth
Their use can be limited by immune-mediated hormone therapy, which is likely contraindicated).
complications, particularly with combination Methylprednisolone may be beneficial for
treatment. Immune-mediated endocrinopathies pressure effects such as optic chiasm compromise,
include hypophysitis, adrenalitis, thyroiditis, and visual field defects, cranial nerve palsies, and
diabetes mellitus. Deaths have been reported intractable headache. Methylprednisolone is
associated with checkpoint inhibitor–induced not an appropriate treatment for acute cortisol
hypophysitis and adrenalitis. Diagnosis and deficiency secondary to ACTH deficiency alone. It
management can be complicated by simultaneous
356  ENDO 2021 • Endocrine Case Management
hased by Dr. Khalid H. Seedahmed, Jr., MRCP PGDip, CertEndocrinology SA - ID 28
is associated with worse long-term prognosis than What is this patient’s diagnosis?
physiological glucocorticoid replacement therapy. Answer: Nivolumab-induced primary adrenal
failure, as evidenced by raised ACTH and renin
Adrenalitis
Adrenalitis is rare and should be suspected if The patient’s CT-PET imaging shows bilateral
hyponatremia is present. If adrenalitis is suspected, increased adrenal activity (see image).
ACTH, renin, and aldosterone should be
measured. Fludrocortisone can be used in addition
to glucocorticoid replacement therapy.

Clinical Case Vignette 2


A 43-year-old man with metastatic melanoma
who has undergone 3 cycles of nivolumab therapy
presents with a 4-week history of lethargy. His
serum sodium concentration is 127 mEq/L
(127 mmol/L). He is hypovolemic.
Other laboratory test results:
Urinary sodium = 102 mEq/L (SI: 102 mmol/L)
Random serum cortisol = 7.0 µg/dL (7.2-25.4 µg/dL) Fludrocortisone is started.
(SI: 192 nmol/L [200-700 nmol/L)
Laboratory test results:
A short cosyntropin-stimulation test gives the Serum sodium = 136 mEq/L (SI: 136 mmol/L)
following results: Renin activity = 1.6 nmol/L per h

0 minutes 6.7 µg/dL (SI: 186 nmol/L)


30 minutes 5.7 µg/dL (SI: 157 nmol/L)
60 minutes 6.2 µg/dL (SI: 172 nmol/L) Cancer Therapy–Related Bone Loss
Patients with nonmetastatic cancer are at risk for
He has no history of exogenous glucocorticoid use. bone loss as a result of the disease process, local
The working diagnosis is nivolumab-induced and systemic inflammation, and the treatment of
isolated ACTH deficiency. The patient continues the cancer (Figure 4). In particular, such therapy
nivolumab and starts hydrocortisone, 30 mg includes glucocorticoid and hormonal treatments
daily. Two weeks later, his condition is somewhat (eg, GnRH agonists and chemotherapy-induced
improved. ovarian failure in premenopausal women,
aromatase inhibitors in postmenopausal women,
Laboratory test results: and antiandrogens in men with nonmetastatic
Serum sodium = 128 mEq/L (136-142 mEq/L) prostate cancer).
(SI: 128 mmol/L [136-142 mmol/L]) Cancer therapy–related bone loss is more
ACTH = 200 pg/mL (10-60 pg/mL) rapid than postmenopausal bone loss in women
(44.0 pmol/L [2.2-13.2 pmol/L]) or normal age-related osteoporosis in men, with
Renin activity = 19.8 nmol/L per h rates being up to 7-fold higher than that seen with
(0.3-2.2 nmol/L per h)
Aldosterone = <3.6 ng/dL (4-21 ng/dL) normal aging.
(SI: <100 pmol/L [111.0-582.5 pmol/L])

ENDO 2021 • Miscellaneous  357

chased by Dr. Khalid H. Seedahmed, Jr., MRCP PGDip, CertEndocrinology SA - ID 283


Figure 4. Cancer Therapy–Related Bone Loss

• Clinicians should be aware that patients with nonmetastatic cancer may have baseline risks for osteoporosis, as well as
the added risks of treatment-related bone loss due to hypogonadism from endocrine therapy (ie, oophorectomy, GnRH
agonists, chemotherapy-induced ovarian failure, aromatase inhibitors, anti-androgens), chemotherapy, or other cancer
At Risk

therapy-associated medications (ie, glucocorticoids).


• All patients should be counseled on intake of calcium and vitamin D, weight-bearing exercises, minimizing the risk of
falls, and bone-healthy lifestyle and behaviors such as tobacco cessation and limiting alcohol consumption.
• Osteoporosis fracture risk assessment may include use of FRAX (www.sheffield.ac.uk/FRAX) or other tools.

When 1 or more risk factor for osteoporotic fracture is present, and there is consideration for use of a bone-modifying
agent, then evaluate bone mineral density to further quantify fracture risk. The preferred assessment uses DXA of total
spine, hip, and femoral neck.
Screening

All patients should be counseled on intake of calcium and vitamin D, weight-bearing exercises, minimizing the risk of falls,
and bone-healthy lifestyle and behaviors such as tobacco cessation and limiting alcohol consumption.

I I

Deferral of Bone-Modifying Agent Initiation of Bone-Modifying Agent


If the bone density result does not Thresholds to initiate a bone-modifying agent include:
demonstrate osteoporosis (or if there is not • FRAX (10-year risk of hip fracture of 3% or greater, or 10-year
significant osteopenia with additional risk risk of non-hip fracture of 20%)
factors) and if FRAX calculation does not
exceed 10-year risk of hip fracture of 3% or • The BMD (DXA) demonstrates osteoporosis or significant
Management

greater, or 10-year risk of non-hip fracture of osteopenia with additional risk factors
20%, and/or bone mineral density is not • The clinical scenario indicates significant risk for osteoporotic
sufficiently low to trigger use of a fracture (such as history of prior osteoporotic fracture that has
bone-modifying agent, then repeat DXA in not been treated), then initiate a bone-modifying agent.
2 years or in 1 year if medically indicated. Bisphosphonates (oral or IV) or denosumab are the preferred
agents dosed for osteopenia or osteoporosis as clinically indicated.
. .
Repeat DXA every 2 years or as clinically indicated.
I I Repeat DXA every 2 years or as clinically indicated.

Clinical Case Vignette 3 Monthly goserelin is initiated at a dosage of 3.6 mg


subcutaneously. Two months later, laboratory
A woman has a history of breast cancer diagnosed
testing documents the following:
at age 32 years (invasive ductal cancer with
lymph node involvement; ER/PR+ve HER2 Serum estradiol (immunoassay) = <19 pg/mL
negative) treated with neoadjuvant chemotherapy (SI: <70 pmol/L)
(epirubicin, cyclophosphamide, docetaxel), Highly-sensitive estradiol by liquid chromatography/
tandem mass spectrometry = 11 pg/mL
mastectomy, axillary node clearance, and (SI: 42 pmol/L)
adjuvant radiotherapy. At age 33 years, she was
amenorrheic after chemotherapy and started
Her regimen is switched from tamoxifen to
tamoxifen, 20 mg once daily. Her menstrual
exemestane, 25 mg once daily. Initial DXA scan
periods resumed.
is performed, and a second DXA is performed 2
Laboratory test results: years later (see images and tables).
The results of lumbar spine bone density
Serum estradiol = 285 pg/mL (SI: 1047 pmol/L)
LH = 12.0 mIU/mL (SI: 12.0 IU/L) from both DXA scans are shown (see graph),
FSH = 10.0 mIU/mL (SI: 10.0 IU/L) demonstrating that the patient’s bone mineral
Serum estradiol measurement by liquid density is worse now than at age 33 years.
chromatography/tandem mass spectrometry = FRAX is not applicable because she is younger
297 pg/mL (SI: 1089 pmol/L) than 40 years. She has no other risk factors for
fracture.

358  ENDO 2021 • Endocrine Case Management

chased by Dr. Khalid H. Seedahmed, Jr., MRCP PGDip, CertEndocrinology SA - ID 283


Results of Initial DXA Results of Repeated DXA 2 Years Later

Region BMD, g/cm2 Z-Score Region BMD, g/cm2 Z-Score

Anteroposterior spine (L1-L4) 0.971 –0.6 Anteroposterior spine (L1-L4) 0.854 –1.7

Femoral neck (left) 0.733 –0.9 Femoral neck (left) 0.685 –1.3

Total hip (left) 0.901 –0.3 Total hip (left) 0.813 –1.0

Comparison of Both DXA Studies

Region Exam Date Age BMD, g/cm2 T-Score BMD Change vs Baseline BMD Change vs Previous

Anteroposterior spine 02.01.2019 35 years 0.854 –1.8 –12.1%* –12.1%*


(L1-L4)
08.11.2016 33 years 0.971 –0.7

Total hip (left) 02.01.2019 35 years 0.813 –1.1 –9.8%* –9.8%*

08.11.2016 33 years 0.901 –0.3

*Denotes significance at 95% confidence level; least significant change for anterioposterior spine = 0.022 g/cm2; least significant change for total hip = 0.027 g/cm2.

Ll-L4
1.4
1.2
1.0 e 1.0 l!!
0
~
co
0.8
0.6
tl:I
- . 2.5 8
V)

i,.!.
0.4
0.2
0
3031323334353637383940
Age
Fracture Risk
Initial DXA images. □ Not increased □ Increased □ High

What is the best management for this patient? Cancer–Related  New Content 
Answer: Calcium, 700-1000 mg daily, Hormone Replacement Therapy
and vitamin D, 800 IU daily
Management of premature ovarian failure,
Lifestyle advice would include recommendations contraception, and fertility can be complex following
for exercise, smoking cessation (if applicable), a diagnosis of cancer and its treatment. The risks
and alcohol use. Screening for secondary causes is and benefits of contraception, hormone therapy,
recommended. The risks and benefits of treatment and fertility treatment must take a number of issues
with antiresorptive therapy (bisphosphonate into consideration such as background risk, family
vs denosumab) should be discussed with the history, gonadal hormone exposure, and oncologic
patient, with particular emphasis on lack of data treatments received.1 For example, women treated
in young patients in this setting. Other issues with chest radiotherapy for a childhood malignancy
to be discussed include the effects of tamoxifen have a very high risk of breast cancer, comparable
on bone loss in premenopausal women and the to that of women who carry BRCA pathogenic
measurement of serum estradiol levels in relation variants.2 This risk is highest in survivors of
to tamoxifen, aromatase inhibitor, and fulvestrant Hodgkin lymphoma, who have a 35% chance of
by immunoassay and mass spectrometry. developing breast cancer by age 50 years.3

ENDO 2021 • Miscellaneous  359

chased by Dr. Khalid H. Seedahmed, Jr., MRCP PGDip, CertEndocrinology SA - ID 283


Clinical Case  New Content  In the general population, progesterone-only
Vignette 4 contraception increases breast cancer risk while on
therapy with a hazard ratio of approximately 1.2.
A 39-year-old woman has a history of Hodgkin This risk may not decrease after stopping the
lymphoma diagnosed at age 19 years that was progesterone-only contraception. This patient’s
treated with chemotherapy and mediastinal risk of breast cancer therefore will most likely
radiotherapy (2500 cGy). She has regular increase to approximately 18% at 10 years
menstrual periods with menorrhagia (with normal (Answer D) and 36% at 20 years if she remains on
serum estradiol levels and confirmed ovulation). the progesterone-only pill. SNP-derived polygenic
She has a new partner, and she is sexually risk scores designed to identify those at increased
active but does not desire pregnancy. Since her risk of breast cancer in the general population
mediastinal radiotherapy, she has had regular are applicable to the postradiotherapy Hodgkin
breast screening with annual MRI. lymphoma population and will contribute to
Her primary care provider prescribed a individualizing advice.4 Following discussion of
progestagen-only pill for contraception and these risks, the patient decided to remain on the
management of menorrhagia. progesterone-only pill and continue with annual
MRI breast screening.
Which of the following is this patient’s
Chest irradiation also increases the risk for
approximate 10-year risk of breast cancer?
cardiomyopathy. Echocardiography surveillance,
A. 0.1% modification of risk factors, and exercise advice are
B. 1% recommended for some groups.5
C. 5% Key issues and questions include how to
D. 18% calculate the risk of breast cancer in childhood
cancer survivors receiving chest irradiation; how
E. 36%
to articulate the risk to patients and advise use
Answer: D) 18% of contraception/hormone replacement therapy;
and ensuring that patients at risk undergo
The patient in this vignette is at increased risk correct surveillance for breast cancer6,7 and
for breast cancer because of previous mediastinal cardiomyopathy/ischemic heart disease.5
radiotherapy. This risk is dependent on age at
treatment and the radiotherapy fields involved.
Untreated primary ovarian failure secondary to Cancer–Related  New Content 
chemotherapy is protective against breast cancer Bone Toxicity
(approximately 50% risk reduction). Patients Pelvic radiotherapy is effective management
with primary ovarian failure who are treated for many pelvic malignancies (eg, endometrial,
with hormone replacement therapy (estrogen cervical, prostate, colorectal), but it is associated
and progesterone) are at intermediate risk of with a number of long-term effects that can be
breast cancer, and those who maintain normal difficult to manage. Patients often present with
menstruation following treatment are at highest pelvic pain in the years following radiotherapy and
risk for breast cancer.2 There is also evidence that this is associated with significant anxiety related
women with underlying high genetic risk of breast to fear of cancer recurrence. However, the cause is
cancer have an even higher absolute risk of breast often related to radiotherapy damage to the bowel,
cancer following chest irradiation.4 Based on bladder, or bone.
this patient’s previous mediastinal radiotherapy,
her 10- and 20-year risks of breast cancer are
approximately 15% and 30%, respectively.3

360  ENDO 2021 • Endocrine Case Management

chased by Dr. Khalid H. Seedahmed, Jr., MRCP PGDip, CertEndocrinology SA - ID 283


Clinical Case  New Content 
Vignette 5
A 57-year-old woman has a history of FIGO stage
2 vaginal squamous cell carcinoma treated with
surgery and radical pelvic radiotherapy and vault
brachytherapy. She presents 12 months following
radiotherapy with sudden-onset deep pelvic pain
and difficulty mobilizing.
DXA is performed (see images and tables).
MRI of the pelvis is performed to exclude
DXA demonstrates normal bone mineral
recurrence and investigate the cause of the pain, and
density. She has no other risk factors for fracture
the T1-weighted coronal view demonstrates bilateral
and has no history of glucocorticoid exposure.
sacral insufficiency fractures (see image, arrows).

Neck

Age-Matched
Area, cm2

Z-score
T-score
Region

BMD
Neck 4.87 0.743 –1.0 0.1 102

Total 36.50 0.887 –0.5 0.2 103


••+--.--.---,-~~-.--.-~~~--.--. .....
20 25 SD 195 Cl '5 &O H ID 15 JD l'5 ID l!I

AfF
l'n clunt Ruic
C Nol - □ -n a.._

,~----
__. .._,._ ----- '.r«oft: \II.. 9i'Vltf--. &oouao118l.EDCS~~ WblJt ,..._
Pf:a.lU
z,....., "'-f..S.... s.aac.UIWDCS.~ \\.*F«Mlt
xia.ua.a
r- t•I.U", OO•Sl.l

IW►. 10.or,.i'

Age-Matched
Area, cm2

Z-score
T-score
Region

BMD

L1 13.47 0.937 –0.5 0.5 106

L2 13.92 0.984 –0.4 0.7 108

L3 18.26 0.952 –1.2 –0.1 99

L4 16.44 0.942 –1.1 0.1 101


Aft
P.-uno llia
□ Nol- □- O tltll1 Total 62.09 0.953 –0.5 0.2 103

T"""°"n_ 'WM•J'.millt. Soiau:lOiTDJDCS.Hoqk


Z...0..tt. \\''11,itt~ SOOC.;:JDllBMDC$1"iOLQpe

ENDO 2021 • Miscellaneous  361

chased by Dr. Khalid H. Seedahmed, Jr., MRCP PGDip, CertEndocrinology SA - ID 283


About the nsk facto,s

Questionnaire: 10, Secondary osteooQrOSIS


1. Age (between 40 ,nd 90 ye.m) or Date of 8irttl 11. Alcohol J or more oo•ts/dav
Aoe; Date ol 8!rtll:
12, fi!moral neck BMO (o/cm2)
57 Y: M: O:

2. Sex .., 0743 T-score:-1.0


O ~tole ® Female
3. We,ohl (kg) 60

➔ Het9ht (cm) 162

5. PrevlousF<aau-e @ No Oves
6. Parent Fr.K!Ured Hip @ rio Oves
1. Ci.rrent Smolono @ No Oves
s. GlucocottKotds @ rio Oves
9. Rheumatoid arthnll5 @ No 0Yes

Her FRAX score is calculated as shown (see Answer: A, B, and C) Analgesia;


image). physiotherapy; and calcium, 1000 mg once
A secondary screen for fracture is daily, and vitamin D, 800 IU once daily
unremarkable with the following laboratory test
results: This patient has radiotherapy-related insufficiency
fractures. Radiotherapy-related insufficiency
25-Hydroxyvitamin D = 38.6 ng/mL fractures occur in approximately 10% to 20%
(20.0-100.2 ng/mL) (SI: 96.3 nmol/L of patients who undergo pelvic radiotherapy.
[50-250 nmol/L])
PTH = 24.5 pg/mL (14.1-71.7 pg/mL) The underlying mechanism is not well
(SI: 24.5 ng/L [14.1-71.7 ng/L]) defined. Risk factors for radiotherapy-related
Calcium = 9.5 mg/dL (8.2-10.2 mg/dL) insufficiency fractures are thought to include
(SI: 2.38 mmol/L [2.1-2.6 mmol/L]) age, postmenopausal status, and presence of
Phosphate = 3.3 mg/dL (2.2-4.6 mg/dL) osteoporosis,8,9 although as in this case presented,
(SI: 1.08 mmol/L [0.7-1.5 mmol/L])
Thyroid function, normal they can occur in younger women with normal
Serum electrophoresis, normal total hip and spine bone mineral density by
Celiac screen, normal DXA. While these fractures are well described
Bone turnover markers in the oncology literature, there are no formal
CTX = 0.20 µg/L (0.1-0.5 µg/L) studies related to the impact of these fractures
P1NP = 40 µg/L (30-78 µg/L)
on quality of life and there are no interventional
studies.10 A recent meta-analysis demonstrated
Which of the following is/are the best that approximately 60% of these fracture are
treatment option(s) for this patient? symptomatic and 83% are generally managed
A. Analgesia conservatively with analgesia (Answer A),
B. Physiotherapy although up to 10% required hospitalization.9 It
is local practice to replace calcium (700-1000 mg
C. Calcium, 1000 mg once daily, and vitamin D, once daily) and vitamin D (800 IU once daily)
800 IU once daily if the patient is deplete (Answer C) and to
D. Alendronate, 70 mg once weekly recommend physiotherapy (Answer B) and gentle
E. Teriparatide exercise to the limits of pain. Approximately
6% of patients receive bisphosphonate therapy
(Answer D), but there is no current evidence

362  ENDO 2021 • Endocrine Case Management

chased by Dr. Khalid H. Seedahmed, Jr., MRCP PGDip, CertEndocrinology SA - ID 283


that this is effective, particularly as there is irradiated rats,11 but they are contraindicated in
the likelihood that this is a low bone-turnover humans who have received previous radiation
state. Patients should be warned of the risk of therapy to the skeleton. Modern radiotherapy
radiotherapy-related insufficiency fractures at the techniques should allow for the development
time of treatment, and osteoporosis risk factors of bone-sparing treatment schedules,12 but
should be reviewed and minimized. Anabolic further studies are required to investigate the
agents (Answer E) may theoretically be of benefit underlying mechanism and optimal treatment of
and have been shown to alleviate bone loss in radiotherapy-related insufficiency fractures.

References
1. Krul IM, Opstal-van Winden AW, Aleman BMP, et al. Breast cancer risk 9. Razavian N, Laucis A, Sun Y, et al. Radiation-induced insufficiency fractures
after radiation therapy for Hodgkin lymphoma: influence of gonadal hormone after pelvic irradiation for gynecologic malignancies: a systematic review. Int
exposure. Int J Radiation Oncol Biol Phys. 2017;99(4):843-853. PMID: 28888722 J Radiat Oncol Biol Phys. 2020;108(3):620-634. PMID: 32442476
2. Moskowitz CS, Chou JF, Sklar CA, et al. Radiation-associated breast cancer 10. van den Blink QU, Garcez K, Henson CC, Davidson SE, Higham CE.
and gonadal hormone exposure: a report from the Childhood Cancer Pharmacological interventions for the prevention of insufficiency fractures
Survivor Study. Br J Cancer. 2017;117(2):290-299. PMID: 28632729 and avascular necrosis associated with pelvic radiotherapy in adults. Cochrane
3. Swerdlow AJ, Cooke R, Bates A, et al. Breast cancer risk after Database Syst Rev. 2018;4(4):CD010604. PMID: 29683475
supradiaphragmatic radiotherapy for Hodgkin’s lymphoma in England 11. Chandra A, Lin T, Tribble MB, et al. PTH1-34 alleviates radiotherapy-
and Wales: a National Cohort Study. J Clin Oncol. 2012;30(22):2745-2752. induced local bone loss by improving osteoblast and osteocyte survival. Bone.
PMID: 22734026 2014;67:33-40. PMID: 24998454
4. Opstal-van Winden AW, de Haan HG, Hauptmann M, et al. Genetic 12. Mir R, Dragan AD, Mistry HB, Tsang YM, Radhani AR, Hoskin P. Sacral
susceptibility to radiation-induced breast cancer after Hodgkin lymphoma. insufficiency fracture following pelvic radiotherapy in gynaecological
Blood. 2019;133(10):1130-1139. PMID: 30573632 malignancies: development of a predictive model. Clin Oncol (R Coll Radiol).
5. Armenian SH, Hudson MM, Mulder RL, International Late Effects of 2020;S0936-6555(20)30394-0. PMID: 33127236
Childhood Cancer Guideline Harmonization Group. Recommendations for 13. Ball S, Barth J, Levy M; Society for Endocrinology Clinical Committee.
cardiomyopathy surveillance for survivors of childhood cancer: a report from Society for Endocrinology Endocrine Emergency Guidance: emergency
the International Late Effects of Childhood Cancer Guideline Harmonization management of severe symptomatic hyponatraemia in adult patients. Endocr
Group. Lancet Oncol. 2015;16(3):123-136. PMID: 25752563 Connect. 2016;5(5):G4-G6. PMID: 27935814
6. Howell SJ, Searle C, Goode V, et al. The UK national breast cancer screening 14. Higham CE, Olsson-Brown A, Carroll P, et al; Society for Endocrinology
programme for survivors of Hodgkin lymphoma detects breast cancer at an Clinical Committee. Society for Endocrinology Endocrine Emergency
early stage. Br J Cancer. 2009;101(4):582-588. PMID: 19672261 Guidance: acute management of the endocrine complications of checkpoint
7. Mulder RL, Hudson MM, Bhatia S, et al. Updated breast cancer surveillance inhibitor therapy. Endocr Connect. 2018;7(7):G1-G7. PMID: 29930025
recommendations for female survivors of childhood, adolescent, and young 15. Shapiro CL, Van Poznak C, Lacchetti C, et al. Management of osteoporosis in
adult cancer from the International Guideline Harmonization Group. J Clin survivors of adult cancers with nonmetastatic disease: ASCO clinical practice
Oncol. 2020;38(35):4194-4207. PMID: 33078972 guideline. J Oncol Pract. 2019;37(3):2916-2946. PMID: 31532726
8. Salcedo MP, Sood AK, Jhingran A, et al. Pelvic fractures and changes in bone 16. Owen LJ, Monaghan PJ, Armstrong A, et al. Oestradiol measurement during
mineral density after radiotherapy for cervical, endometrial, and vaginal fulvestrant treatment for breast cancer. Br J Cancer. 2019;120(4):404-406.
cancer: a prospective study of 239 women. Cancer. 2020;126(11):2607-2613. PMID: 30679781
PMID: 32125711

ENDO 2021 • Miscellaneous  363

chased by Dr. Khalid H. Seedahmed, Jr., MRCP PGDip, CertEndocrinology SA - ID 283


Endocrine Effects of Immune
Checkpoint Inhibitors:
How to Screen and Treat
Simon H. S. Pearce, MD, FRCP. Translational and Clinical Research Institute, Newcastle
University, United Kingdom; E-mail: simon.pearce@ncl.ac.uk

Learning Objectives of immune checkpoint inhibitor therapy, but


a temporary pause may be appropriate if the
As a result of participating in this session, learners
patient is significantly unwell (eg, hypophysitis).
should be able to:
Endocrinologists must liaise closely with
• Recognize and investigate the endocrine oncologists to provide optimal care for this patient
effects of immune checkpoint inhibitors. group.
• Manage the endocrine complications of
immune checkpoint inhibitors.
Significance of the
Clinical Problem
Ipilimumab, the first immune checkpoint
Main Conclusions inhibitor, was introduced to the market in 2011
Immune checkpoint inhibitors, including and has revolutionized the prognosis of patients
ipilimumab (anti-CTLA4), nivolumab, and with disseminated malignant melanoma. Since
pembrolizumab (anti-PD-1), are increasingly then, 6 additional checkpoint inhibitors have
used to treat several advanced cancers: melanoma, been licensed. In addition to treating melanoma,
non–small cell lung cancer, squamous cell head this class of drugs is now indicated for advanced
and neck cancer, renal and urothelial tumors, renal and urothelial cancer, advanced Hodgkin
and Hodgkin lymphoma. Destructive thyroiditis, lymphoma, non–small cell lung cancer, squamous
manifest as either transient hyperthyroidism cell head and neck cancer, and mismatch repair/
or permanent hypothyroidism, is the most microsatellite instability tumors (ie, Lynch
common endocrine adverse effect, typically seen syndrome). These drugs inhibit a series of
after use of PD1/PD-L1 blockers. Hypophysitis, molecules that act as brakes on the normal
manifest as secondary hypoadrenalism and central immune activation process, including CTLA4
hypothyroidism, is also relatively common with (cytotoxic T-lymphocyte antigen 4 [ipilimumab
CTLA4 blockade. Fulminant type 1 diabetes, and tremelimumab]), PD-1 (programmed death-1
often with ketoacidosis, and primary adrenal [nivolumab, pembrolizumab]), and PD-L1
insufficiency can also be observed but are more (programmed death ligand-1 [atezolizumab,
rare. Treatment is supportive with replacement durvalumab, and avelumab]).1,2 They are all
of relevant hormones as clinically indicated. human or humanized monoclonal antibodies and
Recovery from endocrine dysfunction is not are generally administered in cycles, at 3-week
expected in most instances. Endocrine adverse intervals.
effects should generally not lead to cessation

364  ENDO 2021 • Endocrine Case Management

chased by Dr. Khalid H. Seedahmed, Jr., MRCP PGDip, CertEndocrinology SA - ID 283


Cancer cells use positive molecular interactions cause acute and serious decompensation, requiring
with these lymphocyte cell-surface receptors to urgent intervention (Table). These toxicities may
evade detection by the healthy immune system. also be complicated by serious nonendocrine
These checkpoint inhibitor monoclonal antibodies autoimmune disorders, including pneumonitis,
block these “evasive mechanisms,” augmenting colitis, hepatitis, encephalitis, myasthenia, and
the natural immune responses against cancerous many others.
cells, thus harnessing the patient’s endogenous
immune response to attack the tumor and
metastases. The transformative nature of these
Barriers to Optimal Practice
compounds as cancer therapy means that they are
• Close liaison is required between the
being increasingly used, and there are currently
oncologist and the endocrine team to first
more than 200 open clinical trials with these
recognize that a deterioration is due to an
agents. Therefore, the number of cancers for
immune-related endocrine complication
which they are indicated is certain to increase.
and second to ensure optimal ongoing
Although initially used as sole agents, many trials
management.
now show that combination immune checkpoint
inhibition (eg, nivolumab and ipilimumab) can • Affected patients often carry a high tumor
lead to more complete and durable antitumor burden, and nonspecific deterioration,
immune responses. As an adverse effect of this including weight loss, poor appetite, lethargy,
immune activation, immune-mediated endocrine and hypotension may be misattributed to
dysfunction is observed in approximately 15% of advancing cancer rather than to a readily
patients treated with a single agent and in 20% treatable endocrinopathy, particularly primary
to 30% of those treated with a combination of 2 or secondary adrenal insufficiency.
checkpoint inhibitors. • The transformative nature of these treatments
While thyroid dysfunction is the most means that development of a treatable
common problem that occurs during checkpoint endocrine adverse effect should not routinely
inhibitor therapy, pituitary involvement, primary lead to discontinuation of the checkpoint
adrenal insufficiency, and type 1 diabetes may all inhibitor therapy.

Table. Prevalence of Endocrinopathies During Immune Checkpoint Inhibitor Therapy

Pituitary Primary Adrenal


Target Drug Hypothyroidism Thyrotoxicosis Dysfunction Insufficiency Type 1 Diabetes

CTLA4 Ipilimumab 3.8% 1.4% 5.6% 1.4% -

Tremelimumab - - 1.8% 1.3% -

PD-l Nivolumab 8% 2.8% 0.5% 2.0% 2.0%

Pembrolizumab 8.5% 3.7% 1.1% 0.8% 0.4%

PD-L1 Atezolizumab 6% - - - 1.4%

Durvalumab 4.7% - - - -

Avelumab 5.5% 2.3% - 1.1% 1.1%

CTLA4 + PD-1 Combination 10%-15% 10% 8%-10% 5%-7% 2.0%

Data adapted from de Filette J, Andreescu CE, Cools F, Bravenboer B, Velkeniers B. A systematic review and meta-analysis of endocrine-related adverse
events associated with immune checkpoint inhibitors. Horm Metab Res. 2019;51(3):145-156.1

ENDO 2021 • Miscellaneous  365

chased by Dr. Khalid H. Seedahmed, Jr., MRCP PGDip, CertEndocrinology SA - ID 283


Strategies for Diagnosis, initial management is identical, but a full recovery
of thyroid function would be expected with the
Therapy, and/or Management
former, whereas long-term hypothyroidism
Thyroid Dysfunction is expected in the latter. Nuclide imaging is
Thyroid dysfunction is the most common only warranted if there is a preexisting thyroid
endocrine adverse effect of immune checkpoint condition or clinically palpable thyroid nodules.
inhibitors. All patients should have serum TSH
and free T4 measured before starting checkpoint Hypothyroidism
inhibitor treatment and with every cycle of Hypothyroidism is a later complication than
therapy. thyrotoxicosis, typically occurring after 4 to
6 months of treatment. Primary, destructive
Thyrotoxicosis thyroiditis is most common, but a significant
Thyrotoxicosis occurs most frequently with proportion of affected patients develop central
the combination of either of the PD-1 blockers, hypothyroidism as a result of hypophysitis. As in
nivolumab or pembrolizumab, with ipilimumab other situations, it is not necessary to treat patients
in approximately 10% of cases, compared with with asymptomatic mild/subclinical primary
3% to 4% with a PD-1 blocker alone. It is one of hypothyroidism with serum TSH concentrations
the earliest immune complications, with onset in the range of 4 to 10 mIU/L and normal serum
as soon as 3 weeks after treatment initiation, free T4 levels, but treatment is indicated once the
and it typically occurs during the third or fourth TSH concentration rises to 10 mIU/L or higher.
cycle of treatment (8-12 weeks). A rapid-onset Primary hypothyroidism should be assumed to be
painless thyroiditis occurs, characterized by 3 to permanent. Patients receiving CTLA4 blockade
8 weeks of thyrotoxicosis followed by permanent are at particular risk of central hypothyroidism
hypothyroidism in approximately 50%, implying due to hypophysitis, which presents with declining
that this is frequently an irreversible destructive serum free T4 concentrations in the context of
process.2-5 The thyrotoxic phase is usually mild normal or low TSH. It is important to recognize
and best managed supportively with β-adrenergic this, as it should immediately trigger testing
blockers, as antithyroid drugs are not expected to for adrenocortical insufficiency, which may
work. A small number of patients presenting with coexist with other pituitary insufficiencies. It is
thyroid storm have been reported and this should mandatory to screen for and treat hypoadrenalism
be managed with high-dosage glucocorticoids. before starting levothyroxine replacement.
Rarely, thyrotoxic patients are positive for There can occasionally be diagnostic difficulty
TSH-receptor–stimulating antibodies and TSH- in distinguishing central hypothyroidism from
receptor antibody concentrations should be sick euthyroid syndrome (nonthyroidal illness) in
routinely measured to exclude Graves disease: patients with high tumor burden. However, serum
these patients will have a prolonged course of free T4 tends to be relatively preserved in sick
hyperthyroidism unless they receive antithyroid euthyroid syndrome and is likely to be lower in
drugs. Thyrotoxicosis is much less frequent central hypothyroidism.
with CTLA4 blockers than with PD-1/PD-L1
blockers, and Graves disease is as likely as transient Pituitary Dysfunction
thyroiditis in this situation.5 In addition, many
Pituitary dysfunction is most frequently seen
patients have received high loads of iodinated
with ipilimumab (~5%) and other CTLA4-
radiographic contrast media during staging CT
blocking agents. Headache and fatigue are the
imaging, and a transient iodine-induced thyroiditis
earliest symptoms and are typically found after
may practically be indistinguishable from
3 to 5 cycles (9-16 weeks). It is more common
destructive thyroiditis in the acute phase. The
in older men (male:female, 3:1) and with higher

366  ENDO 2021 • Endocrine Case Management

chased by Dr. Khalid H. Seedahmed, Jr., MRCP PGDip, CertEndocrinology SA - ID 283


dosages of ipilimumab (10 mg/kg). Hormonal multiplies the risks to encompass 5% to 7% of
deficiencies owing to infiltrative hypophysitis patients. It typically presents after 4 or more
are expected (85%-90%), but vision disturbance cycles of therapy (12 weeks onward) with fatigue,
due to mass effect has very rarely been reported. hypotension, loss of appetite, weight loss, nausea/
ACTH deficiency leading to secondary vomiting, hyponatremia, and hyperkalemia.
adrenal insufficiency is the most common An early morning serum cortisol value can be
manifestation and requires urgent treatment. diagnostic, with a formal cosyntropin-stimulation
Central hypothyroidism and hypogonadotropic test only if the result is not clear. It is also
hypogonadism also occur in 70% to 80% of cases. important to measure ACTH to distinguish
The presentation of hormonal deficiency is primary adrenal failure from central ACTH
typically nonspecific, with lethargy, weight loss, deficiency: first, because a primary adrenal
nausea, vomiting, poor appetite, hypotension, and insufficiency diagnosis heralds mineralocorticoid
hyponatremia.2,4,7 Confusion or hallucinations may replacement, as well as glucocorticoid
be prominent. Affected patients need pituitary replacement, and second, because in the presence
MRI whether they present with mass effects or of a normal or low ACTH, investigations
hormonal dysfunction, as other causes of pituitary for hypophysitis (including testing of other
failure (particularly metastasis) must be excluded. pituitary axes and pituitary MRI) are needed.
A moderately enlarged and bright pituitary is The differential diagnosis of primary adrenal
found and although stalk thickening is seen in insufficiency includes bilateral adrenal metastases,
50% of cases, diabetes insipidus is very rare. In as well as hemorrhage and drug-induced adrenal
20% of patients, the MRI shows no pituitary insufficiency, so cross-sectional imaging (usually
abnormality. Rarely, patients present with optic CT) should be undertaken. The adrenals may
nerve compression causing visual disturbance, appear hyperplastic on scanning. Treatment with
and they should be treated with high-dosage hydrocortisone, 15 to 20 mg daily, in divided
glucocorticoids such as methylprednisolone or doses and fludrocortisone, 50 to 300 mcg daily, is
dexamethasone until vision is restored. There expected to be lifelong with no reports of recovery
is little evidence of benefit from high-dosage thus far.
steroids in other circumstances. If the patient is
unwell, a random cortisol concentration should be Type 1 Diabetes Mellitus
measured, and parenteral hydrocortisone, 100 mg,
should immediately be administered and continued Type 1 diabetes is rare and seems to be exclusively
at a dosage of at least 50 mg every 6 hours. seen during PD-1/PD-L1 blockade. A “fulminant”
Replacement of levothyroxine and other anterior pattern of type 1 diabetes is the rule with a short
pituitary hormones should be undertaken as the interval between the onset of symptoms and
patient recovers. Secondary adrenal insufficiency presentation with ketoacidosis, reflecting rapid
should be assumed to be permanent, although destruction of insulin-secreting β cells, meaning
rare cases of recovery have been documented. that up to 70% of affected patients present with
Hypothyroidism and hypogonadism may recover ketoacidosis. Onset can be very early, within 1 or
in around 50% of patients and retesting after 6 2 weeks of therapy, although the average is after 4
months treatment is worthwhile. cycles (12 weeks). Plasma or urine C-peptide levels
are low and islet-cell antibodies (glutamic acid
decarboxylase, IA-2) are positive in approximately
Primary Adrenal Insufficiency 50% of patients. Hyperglycemia is more severe
Primary adrenal insufficiency seems to be rare than predicted by hemoglobin A1c levels, reflecting
with single-agent checkpoint inhibition, but the rapid onset of disease. It is good practice to
combination CTLA4/PD-1 blockade therapy warn patients about the possibility of developing

ENDO 2021 • Miscellaneous  367

chased by Dr. Khalid H. Seedahmed, Jr., MRCP PGDip, CertEndocrinology SA - ID 283


polyuria and thirst, and that they must seek Clinical Case Vignettes
evaluation immediately if this occurs. Insulin must
be started immediately to preempt ketoacidosis. Case 1
Lifelong insulin dependence is the rule and the A 53-year-old man presents to the emergency
chances of recovery are negligible. department with an 18-hour history of nausea, 1
episode of vomiting, headache, and “nearly passing
out” when moving from bed to the bathroom. His
Parathyroid Dysfunction
partner thinks he is now confused. Ten weeks ago,
There are a small number of case reports of he started receiving combination ipilimumab and
hypoparathyroidism in patients receiving immune nivolumab therapy for advanced melanoma (this
checkpoint inhibition. It is too early to be sure is now his third cycle). For the last 2 weeks, he has
there is a real causative relation, but it remains a been feeling fatigued and “under the weather.”
distinct possibility. On physical examination, he is pale and
drowsy. Glasgow Coma Scale score is 12.
Nonendocrine Considerations He has cool peripheries, blood pressure of
88/58 mm Hg, pulse rate of 102 beats/min and
Several studies now show that the development
regular, temperature of 98.4ºF (36.9ºC), oxygen
of immune-related endocrine adverse events
saturation of 95% on air, and respiratory rate of
during checkpoint inhibitor therapy is associated
22 breaths/min.
with a positive survival advantage from the
Neutropenic sepsis is suspected, and blood is
original tumor.8 This stands to reason, in that
drawn and cultures are sent. One liter of saline is
a more aggressive autoimmune response is
administered stat, as well as intravenous tazocin,
expected to correlate with a deeper antitumor
but 1 hour later he is still hypotensive with a
immune response. With neat circularity,
Glasgow Come Scale score of 10.
the development of vitiligo in patients with
melanoma undergoing checkpoint blockade is Laboratory test results:
associated with a better prognosis.9 Many of these
White blood cell count =
endocrine adverse effects are manageable with
12,300/µL (4500-11,000/µL)
observation alone (thyrotoxicosis) or one or more (SI: 12.3 × 109/L [4.5-11.0 × 109/L]) with mild
hormone replacement therapies. Furthermore, neutrophilia
the endocrine dysfunction is already irreversible C-reactive protein = 23 mg/L (0.8-3.1 mg/L)
at presentation and will not be worsened by (SI: 219.05 nmol/L [7.62-29.52 nmol/L])
Sodium = 126 mEq/L (136-142 mEq/L)
ongoing checkpoint blockade therapy. This leads
(SI: 126 mmol/L [136-142 mmol/L])
to the logic that in most patients experiencing Potassium = 4.2 mEq/L (3.5-5.0 mEq/L)
endocrine adverse effects from checkpoint (SI: 4.2 mmol/L [3.5-5.0 mmol/L])
inhibitor therapy, the checkpoint blockade should
be continued, either uninterrupted in the case of The endocrine fellow is called to advise about the
hypothyroidism, or following a short period to hyponatremia and hypotension.
allow physical recuperation following a diagnosis
of adrenal insufficiency, hypopituitarism, or type 1 Which of the following is the immediate
diabetes. Close liaison between the endocrinologist priority for management?
and oncologist can often lead to early resumption A. Measure serum cortisol, aldosterone, and
of checkpoint blockade and ultimately improve the ACTH
prognosis of this patient group. B. Measure serum cortisol and give intravenous
hydrocortisone, 100 mg stat
C. Measure urine sodium and urine osmolality

368  ENDO 2021 • Endocrine Case Management

chased by Dr. Khalid H. Seedahmed, Jr., MRCP PGDip, CertEndocrinology SA - ID 283


D. Order pituitary MRI and measure pituitary no weight loss, palpitation, or heat intolerance.
antibodies On physical examination, her pulse rate is 84
E. Perform a short 250-mcg cosyntropin- beats/min and regular and there is no goiter or
stimulation test eye signs.

Answer: B) Measure serum cortisol and give What further action should be taken?
intravenous hydrocortisone, 100 mg stat A. Cancel this cycle of pembrolizumab until
This patient may have either primary or secondary further test results are available
adrenal insufficiency associated with checkpoint B. Measure TSH-receptor antibodies and
inhibitor therapy. Three to 6 cycles would be the perform 99Tc thyroid uptake scan
typical time for this to present, and the associated C. Start methimazole, 15 mg daily
headache and confusion (plus normal potassium) D. Start propranolol, 80 mg daily
lead to a high index of suspicion that he has
E. Order repeated thyroid function tests in 2
hypophysitis with secondary adrenal insufficiency.
weeks with TSH-receptor antibodies
Measuring serum cortisol and giving intravenous
hydrocortisone, 100 mg stat (Answer B) is Answer: E) Order repeated thyroid function
correct because he is sick and should be treated tests in 2 weeks with TSH-receptor antibodies
immediately with parenteral hydrocortisone.
Measure serum cortisol, aldosterone, and This patient had no symptoms and was only
ACTH (Answer A), ordering MRI pituitary and mildly thyrotoxic, so it is safe to monitor the
measuring pituitary antibodies (Answer D), situation (Answer E). Two weeks later, repeated
and performing a short 250-mcg cosyntropin- tests showed TSH-receptor antibodies less than
stimulation test (Answer E) might be correct if he 1.0 U/L, TSH of 4.6 mIU/L, and free T4 of
was not ill. When he is better, a sample should be 1.1 ng/dL (14.1 pmol/L) showing spontaneous
taken for matched cortisol and ACTH to elucidate improvement. Before her sixth cycle, her TSH
whether the cause is primary or secondary adrenal was 39.0 mIU/L and free T4 was 0.6 ng/dL
insufficiency. Cosyntropin-stimulation testing (7.9 pmol/L). Levothyroxine, 75 mcg daily, was
would also be reasonable if the morning cortisol started. Thyrotoxicosis is frequently mild, and
is equivocal. Additional testing for TSH, free T4, its natural history is spontaneous; permanent
and testosterone will be needed if ACTH is normal hypothyroidism occurs in approximately 50% of
or low. Confusion is not unusual and might also cases. Cancelling this cycle of pembrolizumab
lead to an early cranial MRI, which may disclose (Answer A) is wrong because she is not ill
radiologic evidence of hypophysitis. Measuring enough to interrupt the chemotherapy that is
urine sodium and urine osmolality (Answer C) is a life-prolonging treatment for her. Measuring
wrong because he is obviously hypotensive and TSH-receptor antibodies and performing a 99Tc
hypovolemic, so you already know the answer thyroid uptake scan (Answer B) is not wholly
(low urine sodium, high osmolality). wrong, but it will be a waste of resources to order
the scan because it would not distinguish between
immune-mediated thyroiditis and iodine contrast
Case 2 load. It would be appropriate to measure TSH-
A 64-year-old woman is found to have abnormal receptor antibodies to rule out Graves disease.
thyroid function (TSH = <0.05 mIU/L, free T4 Starting methimazole (Answer C) is incorrect, as
= 2.0 ng/dL [26 pmol/L]; free T3 = 4.6 pg/mL it will not improve thyrotoxicosis in thyroiditis; it
[7.0 pmol/L]) on routine bloodwork before her will just make any rebound hypothyroidism more
fourth cycle of pembrolizumab for non–small cell prolonged or severe. Starting propranolol (Answer
lung cancer. She has been feeling well and reports D) is wrong because she is asymptomatic.

ENDO 2021 • Miscellaneous  369

chased by Dr. Khalid H. Seedahmed, Jr., MRCP PGDip, CertEndocrinology SA - ID 283


References
1. de Filette J, Andreescu CE, Cools F, Bravenboer B, Velkeniers B. A systematic 6. Gan EH, Mitchell AL, Plummer R, Pearce S, Perros P. Tremelimumab-
review and meta-analysis of endocrine-related adverse events associated with induced Graves hyperthyroidism. Eur Thyroid J. 2017;6(3):167-170.
immune checkpoint inhibitors. Horm Metab Res. 2019;51(3):145-156. PMID: PMID: 28785544
30861560 7. Joshi MN, Whitelaw BC, Palomar MT, Wu Y, Carroll PV. Immune
2. Chang LS, Barroso-Sousa R, Tolaney SM, Hodi FS, Kaiser UB, Min L. checkpoint inhibitor-related hypophysitis and endocrine dysfunction: clinical
Endocrine toxicity of cancer immunotherapy targeting immune checkpoints. review. Clin Endocrinol (Oxf). 2016;85(3):331-339. PMID: 26998595
Endocr Rev. 2019;40(1):17-65. PMID: 30184160 8. Osorio JC, Ni A, Chaft JE, et al. Antibody-mediated thyroid dysfunction
3. Morganstein DL, Lai Z, Spain L, et al. Thyroid abnormalities following the during T-cell checkpoint blockade in patients with non-small-cell lung
use of cytotoxic T-lymphocyte antigen-4 and programmed death receptor cancer. Ann Oncol. 2017;28(3):583-589. PMID: 27998967
protein-1 inhibitors in the treatment of melanoma. Clin Endocrinol (Oxf). 9. Nakamura Y, Tanaka R, Asami Y, et al. Correlation between vitiligo
2017;86(4):614-620. PMID: 28028828 occurrence and clinical benefit in advanced melanoma patients treated
4. Ryder M, Callahan M, Postow MA, Wolchok J, Fagin JA. Endocrine-related with nivolumab: a multi-institutional retrospective study. J Dermatol.
adverse events following ipilimumab in patients with advanced melanoma: 2017;44(2):117-122. PMID: 27510892
a comprehensive retrospective review from a single institution. Endocr Relat
Cancer. 2014;21(2):371-381. PMID: 24610577
5. Higham CE, Olsson-Brown A, Carroll P, et al; Society for Endocrinology
Clinical Committee. Society for Endocrinology Endocrine Emergency
Guidance: acute management of the endocrine complications of checkpoint
inhibitor therapy. Endocr Connect. 2018;7(7):G1-G7. PMID: 29930025

370  ENDO 2021 • Endocrine Case Management

chased by Dr. Khalid H. Seedahmed, Jr., MRCP PGDip, CertEndocrinology SA - ID 283


A Little Help From My
Friends: The Endocrinology–
Psychiatry Interface—How
Do We Help Each Other?
Elizabeth J. Murphy, MD, DPhil. Department of Medicine, Division of Endocrinology UCSF
and San Francisco General Hospital, San Francisco, CA; E-mail: lisa.murphy@ucsf.edu

Kewchang Lee, MD. Mental Health Service, San Francisco VA Health Care System, and
Department of Psychiatry and Behavioral Sciences, UCSF, San Francisco, CA; E-mail:
kewchang.lee@ucsf.edu

Learning Objectives needs of the patient, clinical manifestation of


the hyperprolactinemia (eg, galactorrhea or
As a result of participating in this session, learners
hypogonadism), and whether there is underlying
should be able to:
pituitary pathology.
• Develop an approach to addressing Weight gain, glucose abnormalities, and
hyperprolactinemia in psychiatric patients. dyslipidemia are common adverse effects of
antipsychotic medication use. Providers need to
• Develop an approach to addressing the
be on the lookout and monitor for these effects in
metabolic effects of antipsychotic medications.
patients started on a new medication. Treatment
• Describe the potential risks and benefits of the focuses on managing the various risk factors and
use of liothyronine (T3) for refractory severe trying alternative antipsychotic treatments when
depression. feasible.
• Engage in meaningful communication with Depression can be a life-threatening condition.
psychiatric colleagues around common areas of Despite limited data on its clinical efficacy,
clinical overlap. liothyronine (T3) remains a consideration for
add-on treatment in combination with traditional
antidepressants for refractory depression.
Monitoring liothyronine use with appropriate
Main Conclusions thyroid function testing and bone mineral
density assessment by DXA is a crucial safety
Hyperprolactinemia is a common finding in
consideration.
patients taking antipsychotic medications. In most
cases, the patient has no underlying pituitary
disorder. Treatment with a dopamine agonist with
a sole focus of reducing the hyperprolactinemia
may result inadvertent worsening of the patient’s
psychiatric condition. Careful consideration should
be given in these situations to the psychiatric

ENDO 2021 • Miscellaneous  371

chased by Dr. Khalid H. Seedahmed, Jr., MRCP PGDip, CertEndocrinology SA - ID 283


• Widespread use and, at times, overuse
Significance of the of atypical antipsychotic medications for
non–FDA-approved indications leads to
Clinical Problem unnecessary adverse events.
The complex interplay between endocrine
hormones and psychiatric disease has been
recognized for more than a century, dating back Strategies for Diagnosis,
to the original descriptions of disorders such
Therapy, and/or Management
as hypothyroidism and Cushing syndrome.
Prior to surgical options, suicide was one of Antipsychotic Therapy Overview
the leading causes of death for patients with In 2013, it was estimated that 1 in 6 adults in the
Cushing syndrome. The wide range of psychiatric United States was taking a psychiatric medication
manifestations of both hypothyroidism and or sedative, speaking both to the prevalence of
hyperthyroidism is well known, and approximately psychiatric disorders, but also to the possibility
1 in 20 adults in the United States has some of unintended adverse events related to their
abnormality in thyroid function. treatment. One medication class with the most
The complexity of these relationships has significant adverse effect profile is antipsychotics.
been further exacerbated by medication adverse These medications, available since the 1950s, are
effects. It is estimated that 1 in 6 adults in the the mainstay treatment for schizophrenia, but
United States is taking a psychiatric medication are also used to treat a host of other disorders.
or sedative. These medications can lead to a host Atypical or second-generation antipsychotic
of adverse endocrine effects, including weight agents became available starting in the 1970s
gain, insulin resistance, pancreatic failure, diabetes with clozapine, which was found to cause
mellitus, and hyperprolactinemia. Moreover, there agranulocytosis. During the 1990s and 2000s,
is a strong association between depression and olanzapine, risperidone, quetiapine, ziprasidone,
both obesity and diabetes complicating treatment. aripiprazole, and paliperidone were introduced
It is critical for optimal management in with the promise of reduced extrapyramidal
these overlap situations to understand the risks adverse effects and improved clinical efficacy.
and benefits for both psychiatric and endocrine Most antipsychotic agents work in part as
outcomes. During this session, several common dopamine D2 receptor antagonists. Thus, it is not
medical scenarios will be discussed considering surprising that hyperprolactinemia is a common
both the endocrine and psychiatric perspective. adverse effect. The second-generation drugs were
Suggestions will be provided regarding optimal designed to also target 5-HT2, histamine, and
provider communication to allow for thoughtful acetylcholine receptors. It is these actions that are
clinical decision-making. thought to contribute to the significant metabolic
adverse effects seen with these agents. Effects
Barriers to Optimal Practice include weight gain, metabolic syndrome, type 2
diabetes, diabetic ketoacidosis, pancreatic failure,
• There is a shortage of both mental health and dyslipidemia.
providers and endocrinologists.
• Opportunities are often rare for easy Antipsychotic Agents and
communication between endocrinologists Hyperprolactinemia
and psychiatrists, which makes collaborative The potency of antipsychotic medications
decision-making around treatment difficult. blocking the D2 receptor is highly variable,
resulting in significant variation in inducing

372  ENDO 2021 • Endocrine Case Management

chased by Dr. Khalid H. Seedahmed, Jr., MRCP PGDip, CertEndocrinology SA - ID 283


hyperprolactinemia. First-generation The Endocrine Society guidelines4 recommend
antipsychotics and some second-generation agents that patients with suspected drug-induced
such as risperidone and paliperidone result in hyperprolactinemia stop the offending medication
prolactin elevation in 90% or more of patients, and recheck prolactin after 3 days. If prolactin
while other second-generation agents such as has not normalized, MRI is recommended.
aripiprazole and quetiapine have a minimal effect Unfortunately, stopping an antipsychotic
on prolactin concentrations (Table).1-3 These medication can be difficult and should never be
effects on prolactin secretion are seen in patients done without consultation with the patient’s
without underlying prolactinomas. mental health care provider. If stopping an
antipsychotic agent is not an option, it might
Table. Effect of Common Antipsychotic be possible to switch to aripiprazole, which has
Medications on Increasing Prolactin both dopamine agonist and antagonist properties,
making it the atypical antipsychotic of choice in
Medication Prolactin effect
situations of problematic hyperprolactinemia.
First generation Haloperidol ++ But again, for a patient well-controlled from
Chlorpromazine ++ a psychiatric perspective, medication changes
come with what may be unacceptable risks. If a
Perphenazine ++
medication change is not possible, the Endocrine
Second generation Amisulpride (not +++
approved in the
Society guidelines recommend MRI. However,
United States) in the case of some antipsychotic agents that
Clozapine –
can result in prolactin elevation in close to
100% of patients, this would result in significant
Risperidone +++
unnecessary testing leading to incidental findings
Olanzapine ++ and increased morbidity. In these cases, the
Quetiapine +/– degree of prolactin elevation can help guide
Ziprasidone +
next steps. It is often thought that medication-
induced hyperprolactinemia is limited to prolactin
Aripiprazole –
elevations less than 100 ng/mL (<4.3 nmol/L).
Paliperidone +++ That said, risperidone and phenothiazines
can result in prolactin elevations well over
200 ng/mL (>8.7 nmol/L). In patients on an
In addition to galactorrhea in women, the antipsychotic agent known to increase prolactin
primary adverse effect of hyperprolactinemia who have a prolactin concentration less than
is due to subsequent hypogonadism. This 100 ng/mL (<4.3 nmol/L), many clinicians
includes amenorrhea in premenopausal women would not routinely recommend MRI. To
and low testosterone in men, which both rule out hypothyroidism as a contributor and
importantly contribute to bone loss. While to assess another aspect of pituitary function,
there have been other postulated effects of measuring TSH and free T4 would be reasonable.
hyperprolactinemia, none have been firmly For patients with higher prolactin elevations,
established in human studies. Therefore, while it especially those on risperidone or paliperidone,
is important to monitor patients for symptoms it may also be appropriate to defer MRI,
of hyperprolactinemia such as galactorrhea, especially if a temporal link can be established
amenorrhea, and decreased libido, we do not with the hyperprolactinemia and starting the
recommend routinely checking prolactin levels in medication. But for the remainder of patients,
patients on these medications. MRI is reasonable to rule out underlying pituitary
pathology.

ENDO 2021 • Miscellaneous  373

chased by Dr. Khalid H. Seedahmed, Jr., MRCP PGDip, CertEndocrinology SA - ID 283


Treatment Antipsychotic Agents and
Treatment of antipsychotic-induced Metabolic Alterations
hyperprolactinemia is focused on symptoms
rather than on correcting an abnormal lab At baseline, patients with schizophrenia have
value. In an asymptomatic patient with regular an increased risk of obesity, diabetes mellitus,
menses and gonadal function, no treatment and cardiovascular disease.5 Second-generation
is indicated. If the patient is experiencing antipsychotic medications have a whole host
hypogonadism, replacement therapy with estrogen of metabolic effects that can exacerbate that
or testosterone is appropriate for symptoms underlying risk.1-3,5-7 In 2003, the FDA required
and for bone protection. If possible, from a a warning label for these adverse events, and
psychiatric perspective, switching to a different in 2004, the American Diabetes Association,
antipsychotic agent or lowering the dosage of the American Psychiatric Association, and North
current medication may also be effective options. American Association for the Study of Obesity
While treatment with dopamine agonists such teamed up to create a guidance document
as cabergoline or bromocriptine can normalize recommending routine testing of glucose and
prolactin, use of these agents should be avoided lipids in patients on these medications.8
because activating the dopamine receptors that As with hyperprolactinemia, there is
antipsychotic medications are designed to block significant variation in the degree of metabolic
can lead to severe exacerbations of the underlying effects seen with the different antipsychotic
psychiatric condition.4 agents. While it was initially thought these
While most cases of prolactin elevation with effects were limited to only a few medications,
antipsychotic medications are medication related it is now recognized that to a certain extent
without pituitary pathology, it is also possible to they all can have unwanted metabolic effects,
have an underlying prolactinoma. There is no including weight gain, glucose dysregulation
clear evidence to suggest antipsychotic medication (including diabetes and diabetic ketoacidosis),
use causes prolactinomas; however, it may be that hypertriglyceridemia, and metabolic syndrome.
an underlying prolactinoma is exacerbated by Evidence suggests olanzapine, clozapine,
these medications. For microadenomas, treatment zotepine, and chlorpromazine result in some of
is the same as it is with purely medication-induced the largest weight gains,2,3,5,6 which can be rapid
hyperprolactinemia. In addition, an annual and significant (gain of >7% of baseline weight).
pituitary MRI should be obtained to assess for While there is clear evidence for a class effect on
tumor growth for at least the first several years. If the other metabolic effects besides weight gain,
a patient has a macroadenoma or a microadenoma clear and consistent comparative data between
is showing growth, an attempt should be made medications for these effects are lacking. As
to switch to aripiprazole, which has resulted with weight gain, the effects can be rapid, with
in excellent responses in terms of improved studies showing a doubling in triglycerides in
prolactin and lack of tumor growth.1,4 There are 2 weeks and acute decompensation with diabetic
also case reports of treatment with cabergoline ketoacidosis.5 An important observation is the
or bromocriptine in addition to continued use heterogeneity of effects across individuals. It
of an antipsychotic agent, but as mentioned appears that underlying genetic factors that are as
previously, this runs the risk of exacerbating the of yet not well understood significantly contribute
underlying psychiatric disorder and should only be to the degree of metabolic effects seen in a given
done in very close consultation with the patient’s individual.3,5,7 For example, weight gain of 30%
psychiatrist. If the medical therapies are not or more of body weight is not uncommon in
possible or not successful, surgical resection can be some patients, but for other patients on the same
considered for macroadenomas. medication, weight gain may be minimal.

374  ENDO 2021 • Endocrine Case Management

chased by Dr. Khalid H. Seedahmed, Jr., MRCP PGDip, CertEndocrinology SA - ID 283


The mechanism for these metabolic changes most appropriate with less frequent follow-up
is likely multifactorial.5,7 Effects on central thereafter.
neurotransmitters no doubt affect appetite, leading For patients who do experience medication-
to weight gain which in turn leads to increased related metabolic derangements, the initial
insulin resistance, dyslipidemia, and potentially treatment should be a change in medication, if
diabetes. However, glucose dysregulation also possible, to an agent with a potentially better
occurs independent of weight gain with an adverse effect profile. Again, this should be done
increased incidence of diabetic ketoacidosis in in conjunction with the patient’s mental health
patients without weight gain. There is evidence care provider. Ziprasidone and aripiprazole are
for direct effects on insulin signaling and thought to be the most weight-neutral options
glucose transport, including effects on skeletal with less diabetes-related adverse events than
muscle glucose transporters impairing glucose with other antipsychotic agents.5,6 In addition
effectiveness and direct effects on the pancreatic to lifestyle modification, other therapeutic
β cells impairing insulin secretion. The latter is options that have had success include traditional
thought to contribute to the increased incidence treatments such as metformin and GLP-1 receptor
of diabetic ketoacidosis.7 Second-generation agonists for diabetes and weight gain, as well
antipsychotic agents are also thought to stimulate as statin therapy for dyslipidemia. Recognizing
AMP-kinase in the hypothalamus leading to these metabolic adverse events early is probably
downstream changes such as effects on hepatic the most important intervention, which allows
gluconeogenesis. It also has been proposed that for subsequent treatment and/or medication
changes in the gut microbiome and epigenetics adjustment.
may contribute to these changes.5,7
Liothyronine as Augmentation
Management of Metabolic Changes
In addition to the guideline from the 2004 Therapy for Depression
consensus conference, almost 20 other Major depressive disorder can be fatal, with
guidelines for monitoring patients on suicide currently the 10th leading cause of death in
antipsychotic medications exist with inconsistent adults in North America. Strategies for treatment-
recommendations9 on how best to catch the refractory depression include augmentation
unintended metabolic effects early. Compounding therapy with nonantidepressant medications such
the problem, data suggest monitoring guidelines as liothyronine (T3).10 Numerous studies have
are not consistently followed, perhaps in part examined liothyronine as augmentation therapy
due to inconsistencies in recommendations and given as an add-on after a trial of a traditional
poor patient follow-up in this population. There antidepressant such as a tricyclic antidepressant
remains much that can be done with respect or selective serotonin reuptake inhibitor. Older
to better screening and evaluation. Providers uncontrolled, nonblinded studies suggest there
treating patients newly started on antipsychotic may be improved response rates with liothyronine
medications or with a recent antipsychotic in combination with tricyclic antidepressants.
medication change should be on the lookout for Data for liothyronine in combination with
any acute metabolic changes. Close monitoring selective serotonin reuptake inhibitors show
for changes in BMI and new-onset diabetes is mixed results. Perhaps the greatest enthusiasm
important, although recommendations for the for liothyronine use came after results of the
timing of these evaluations vary. Given the Sequenced Treatment Alternatives to Relieve
significant patient variability and what can be Depression (STAR*D) trial published in 2006.11
very rapid-onset changes, earlier initial evaluation This 35 million-dollar, multiarmed study assessed
(eg, 4-6 weeks after start of treatment) may be the effectiveness of medications or cognitive

ENDO 2021 • Miscellaneous  375

chased by Dr. Khalid H. Seedahmed, Jr., MRCP PGDip, CertEndocrinology SA - ID 283


therapy in treatment of refractory depression. One Communication Strategies
study arm compared augmentation therapy with
As discussed, there are often barriers that make it
lithium vs liothyronine after 2 failed trials of other
difficult for endocrinologists and psychiatrists to
medications. The authors concluded that, “T3 has
communicate with each other regarding shared
slight advantages over lithium in effectiveness
patients. The use of technology can certainly help.
and tolerability. T3 also offers the advantages
With the advent of the electronic health record,
of ease of use and lack of need for blood level
“tagging” another provider when writing a note
monitoring.” Limitations of the study were that it
can be helpful, although there is the risk that the
was nonblinded, was without a placebo control,
receiving clinician will overlook the notification
and had no assessment of thyroid function
or take only a very cursory look at the note.
before or during treatment. An assessment of
E-mail can certainly be effective, although there
thyroid function exploring a correlation between
are different levels of responsiveness. Instant
TSH suppression and efficacy could have been
messaging through the electronic health record
very informative in guiding subsequent studies
has become commonplace in many settings, and
and therapy. Despite limitations on the data
this can be particularly useful when there is a sense
supporting liothyronine use, psychiatric textbooks
of urgency to the communication, although this
and the American Psychiatric Association major
method may also be intrusive to one’s workflow.
depressive disorder guidelines in 2010 recommend
The telephone remains a helpful tool, though
the use of liothyronine for refractory depression
again, there is no guarantee that one will receive
without recommendations for monitoring
a call back in response. Ultimately, the choice of
thyroid function.12 The dosing recommendation
communication method relies on the preference
is typically 25 mcg daily increased to 50 mcg
of the clinicians involved and the culture of the
daily after a week, but it is also suggested in the
health care system where they work.
psychiatric literature that dosages above 50 mcg
In recent years, there has been increased
daily may be appropriate, with some publications
interest in collaborative care involving mental
citing dosages as high as 150 mcg daily.
health and endocrinology. In one such model,
More recently, it has been recognized that
called TEAMCare, the goal was to improve
many psychiatrists and endocrinologists feel
outcomes in patients with type 2 diabetes and
uncomfortable treating euthyroid patients with
psychosis receiving care at a community mental
liothyronine without adequate monitoring.11-12
health center.13 The team consisted of a nurse
Subsequent guidelines now recommend safety
care manager, the primary care provider, and a
monitoring. The psychiatric recommendation
psychiatrist, all of whom were on-site, as well
for efficacy is a TSH value at the lower limit
as a consulting endocrinologist. Interventions
of the normal range or lower in the absence of
included regular visits to support chronic disease
symptoms. It is also recommended to assess bone
self-management (eg, medication adherence and
mineral density by DXA every 2 years while on
nutrition), behavioral interventions to target
therapy. These recommendations will hopefully
barriers to care, and diabetes education materials
limit the number of cases of unintended severe
modified to address issues raised by psychosis. The
adverse events due to hyperthyroidism while
team also met weekly to review cases. In a study
helping to treat a life-threatening condition—
that enrolled patients with a hemoglobin A1c level
depression. Future studies should focus on a clear
greater than 8.0% or blood pressure greater than
safety assessment of liothyronine treatment and
140/90 mm Hg, those in the TEAMCare arm had
explore the hypothesis that there is a correlation
a significant mean decrease in hemoglobin A1c
between the degree of TSH suppression and
when compared with that observed in the usual
efficacy of treatment for depression.
care arm, suggesting that collaborative care can

376  ENDO 2021 • Endocrine Case Management

chased by Dr. Khalid H. Seedahmed, Jr., MRCP PGDip, CertEndocrinology SA - ID 283


improve the health outcomes of patients with it the potential for exacerbating her psychiatric
diabetes and psychosis. condition. Communicating with the psychiatrist
(Answer D) is always a good option. However,
administration of a depot formulation of an
Clinical Case Vignettes antipsychotic agent suggests more severe disease,
Case 1 and if her psychiatric condition is currently well
A 38-year-old woman with controlled but severe controlled, switching medications will no doubt
schizophrenia is treated with paliperidone carry significant risk. With the MRI findings,
injection every 3 months. She has no other it is unclear whether the patient even has a
notable medical history. Her primary care prolactinoma. Other than lack of menses, there is
provider measured prolactin, which was elevated no clear risk of the elevated prolactin. Therefore,
at 125 ng/mL (2-18 ng/mL) (SI: 5.4 nmol/L providing estrogen for bone health (Answer C)
[0.1-0.8 nmol/L]). She is referred to you for would be the best option for this patient.
evaluation. Concentrations of creatinine, TSH,
and free T4 are normal. She has no galactorrhea Case 2
but has not had menses for 2 years. She is not
A 50-year-old man with hypertension, HIV,
bothered by this and has no concerns. Her primary
hypogonadism, major depressive disorder, and
care provider ordered an MRI, which reveals a
obesity is admitted to the hospital with chest pain
2-mm pituitary lesion.
and a non–ST-elevation myocardial infarction.
Telemetry reveals paroxysmal atrial fibrillation. He
Which of the following is the best next step?
is discharged on medical therapy and referred to
A. Treat with a dopamine agonist (cabergoline or an endocrinologist because of inpatient laboratory
bromocriptine) testing that documented the following:
B. Ignore the prolactin elevation
TSH = 0.02 mIU/L (0.37-4.42 mIU/L)
C. Start an oral contraceptive pill or other Free T4 = 0.52 ng/dL (0.65-1.80 ng/dL)
estrogen replacement (SI: 6.7 pmol/L [8.4-23.2 pmol/L])
D. Ask the psychiatrist to switch her to an
antipsychotic agent with fewer dopaminergic Which of the following could be going on?
effects
A. Pituitary tumor with central hypothyroidism
Answer: C) Start an oral contraceptive B. Euthyroid sick
pill or other estrogen replacement C. Laboratory error
While it is always important to not just treat D. T3 thyrotoxicosis
a number (eg, the high prolactin) for the sake E. Any of the above
of making ourselves feel better, in this case,
the patient is not menstruating and has not for Answer: E) Any of the above
several years. While she may not be symptomatic, This could be any of the proposed options
there will likely be adverse effects on her bones, (Answer E). Central hypothyroidism should
making ignoring the prolactin (Answer B) an always be a consideration with an inappropriately
inappropriate option. While treating with a low TSH in the setting of a low free T4. In a sick
dopamine agonist (Answer A) may be potentially hospitalized patient, those results could also be
appealing, starting one of these medications while consistent with euthyroid sick syndrome. Issues
being treated with an agent that is acting as a affecting laboratory testing should always be
dopamine antagonist is a little like stepping on the considered with unusual thyroid function test
gas and the brake at the same time and carries with results. Antibody interference, biotin interference,

ENDO 2021 • Miscellaneous  377

chased by Dr. Khalid H. Seedahmed, Jr., MRCP PGDip, CertEndocrinology SA - ID 283


and changes in binding proteins affect assay with him, but he reports polyuria and polydipsia.
results. And finally, as was the case for this patient, Point-of-care blood glucose and hemoglobin A1c
further testing in the outpatient setting showed he measurements are 420 mg/dL (23.3 mmol/L) and
had T3 thyrotoxicosis. This results in suppression 9.1% (76 mmol/mol), respectively. He has gained
of TSH and a subsequently low T4 level. 24.2 lb (11 kg) (10% of his body weight) since
hospital discharge.
Further laboratory test results:
TSH = <0.01 mIU/L (0.37-4.42 mIU/L) Question 1: Which of the following
Free T4 = 0.57 ng/dL (0.65-1.80 ng/dL) is the most likely cause of his
(SI: 7.3 pmol/L [8.4-23.2 pmol/L]) worsening glycemic control?
Free T3 = 7.62 pg/mL (2.3-4.2 pg/mL)
(SI: 11.7 pmol/L [3.5-6.5 pmol/L]) A. Weight gain due to poor diet adherence
because of worsening schizophrenia
A review of the patient’s medical history revealed B. Poor adherence to diabetes medications due to
that he had been started on liothyronine 1.5 years worsening schizophrenia
earlier for refractory depression. The dosage had C. Olanzapine
slowly been increased to 75 mcg once daily. The
patient endorsed weight loss (which he was happy D. New-onset latent autoimmune diabetes of
about) and improved mood on the liothyronine. adulthood (LADA) on top of his underlying
He was very reluctant to decrease the dosage or type 2 diabetes and insulin resistance
stop the medication. In such instances, dialogue Answer: C) Olanzapine
between the patient’s endocrinologist and
psychiatrist is crucial to review the relative risks Weight gain is clearly a driving factor in the
of cardiac disease (non–ST-segment myocardial patient’s worsening diabetes control. The rapidity
infarction, atrial fibrillation) and the impact on the with which he has gained such a significant
patient’s depression. amount of weight is very suggestive of an
olanzapine adverse effect (Answer C) rather
than what is seen with typical poor dietary
Case 3 adherence (Answer A). While poor medication
A 42-year-old man (BMI = 31 kg/m2) with type adherence (Answer B) can definitely be seen in
2 diabetes and schizophrenia returns to you after this setting, that might, if anything, be associated
being hospitalized 3 months ago for worsening with weight loss given the worsening glycemic
auditory hallucinations and delusions that led control with glucosuria and caloric loss in the
him to make a suicide attempt. He was started on urine. Although anything is possible, new-
olanzapine, which improved his symptoms to the onset latent autoimmune diabetes of adulthood
point that he was discharged. (Answer D) is highly unlikely. Independent of the
You have been following him for several weight gain seen, olanzapine can also have effects
years for his diabetes, which has been well treated on insulin resistance and pancreatic function
with metformin, glipizide, and a GLP-1 receptor contributing to this worsening glycemic control.
agonist. Given his severe psychiatric condition and It is important to identify patients who have these
risk of hypoglycemia, you have established a goal rapid and profound changes early to consider
hemoglobin A1c of less than 8.0% (<64 mmol/mol). psychiatric medication changes to prevent more
At the start of his hospitalization, his hemoglobin severe outcomes such as hospitalization for
A1c value was 7.5% (58 mmol/mol). hyperglycemia and diabetic ketoacidosis in ketosis-
He sees you 5 weeks after discharge. He is prone patients.
more disheveled and significantly less interactive
than at his last visit. He has no glucose meter

378  ENDO 2021 • Endocrine Case Management

chased by Dr. Khalid H. Seedahmed, Jr., MRCP PGDip, CertEndocrinology SA - ID 283


 New Content  diabetic ketoacidosis in patients on these second-
generation antipsychotic agents and the increased
Question 2: Which of the following
is the most appropriate next risk of euglycemic diabetic ketoacidosis with
intervention at this visit? SGLT-2 inhibitors, other treatment options would
be preferred in a patient without renal disease or
A. Change olanzapine to aripiprazole
heart failure.
B. Start an SGLT-2 inhibitor While insulin will help acutely, adherence is
C. Refer to a nutritionist for dietary always difficult and may be made more difficult
recommendations by the underlying psychiatric disorder. In the long
D. Start insulin term, switching to a different antipsychotic agent
with fewer metabolic effects, such as aripiprazole
Answer: D) Start insulin or ziprasidone (Answer A), would be ideal. If a
medication change can be made, many of these
Given this patient’s symptoms of polyuria and
acute metabolic effects are reversible. However,
polydipsia, very high glucose concentration
given the severity of the psychiatric disease
in clinic, and the rapidity of weight gain and
in this patient and his recent suicide attempt,
change in hemoglobin A1c, he needs insulin
mental health providers would be very hesitant
(Answer D) acutely. While referral to a
to change a medication that is working. Thus,
nutritionist (Answer C) is always important in
starting insulin to control his diabetes is the most
diabetes care, this alone would not address the
appropriate course of action. Any discussion
acute hyperglycemia. Similarly, SGLT-2 inhibitors
of medication change should be done in close
(Answer B), while excellent for renal disease and
consultation with the patient’s mental health
heart failure, have relatively modest effects on
provider, reviewing the risks and benefits of such
hemoglobin A1c and would not address the acute
a change.
hyperglycemia. Also, given the increased risk of

References
1. Henderson DC, Doraiswamy PM. Prolactin-related and metabolic adverse 8. American Diabetes Association; American Psychiatric Association; American
effects of atypical antipsychotic agents. J Clin Psychiatry. 2008;69(Suppl 1):32-44. Association of Clinical Endocrinologists; North American Association for the
PMID: 18484806 Study of Obesity. Consensus development conference on antipsychotic drugs
2. Young SL, Taylor M, Lawrie SM. “First do no harm.” A systematic review and obesity and diabetes. Diabetes Care. 2004;27(2):596-601. PMID: 14747245
of the prevalence and management of antipsychotic adverse effects. 9. De Hert M, Vancampfort D, Correll CU, et al. Guidelines for screening and
J Psychopharmacol. 2015;29(4):353-362. PMID: 25516373 monitoring of cardiometabolic risk in schizophrenia: systematic evaluation.
3. Smith RC, Leucht S, Davis JM. Maximizing response to first-line Br J Psychiatry. 2011;199(2):99-105. PMID: 21804146
antipsychotics in schizophrenia: a review focused on findings from meta- 10. Touma KTB, Zoucha AM, Scarff JR. Liothyronine for depression: a review
analysis. Psychopharmacology. 2019;236(2):545-559. PMID: 30506237 and guidance for safety monitoring. Innov Clin Neurosci. 2017;14(3-4):24-29.
4. Melmed S, Casanueva FF, Hoffman AR, et al; Endocrine Society. Diagnosis PMID: 28584694
and treatment of hyperprolactinemia: an Endocrine Society clinical practice 11. Nierenberg AA, Fava M, Trivedi MH, et al. A comparison of lithium and T(3)
guideline. J Clin Endocrinol Metab. 2011;96(2):273-288. PMID: 21296991 augmentation following two failed medication treatments for depression: a
5. Abosi O, Lopes S, Schmitaz S, Fiedorowicz JG. Cardiometabolic effects STAR*D report. Am J Psychiatry. 2006;163(9):1519-1530. PMID: 16946176
of psychotropic medications. Horm Mol Biol Clin Investig. 2018;36(1). 12. Rosenthal LJ, Goldner WS, O’Reardon JP. T3 augmentation in major
PMID: 29320364 depressive disorder: safety considerations. Am J Psychiatry. 2011;168(10):1035-
6. Huhn M, Nikolakopoulou A, Schneider-Thoma J, et al. Comparative efficacy 1040. PMID: 21969047
and tolerability of 32 oral antipsychotics for the acute treatment of adults with 13. Chwastiak LA, Luongo M, Russo J, et al. Use of a mental health center
multi-episode schizophrenia: a systematic review and network meta-analysis. collaborative care team to improve diabetes care and outcomes for patients
Lancet. 2019;394(10202):939-951. PMID: 31303314 with psychosis. Psychiatr Serv. 2018;69(3):349-352. PMID: 29191136
7. Kowalchuk C, Castellani LN, Chintoh A, Remington G, Giacca A, Hahn MK.
Antipsychotics and glucose metabolism: how brain and body collide. Am J
Physiol Endocrinol Metab. 2019;316(1):E1-E15. PMID: 29969315

ENDO 2021 • Miscellaneous  379

chased by Dr. Khalid H. Seedahmed, Jr., MRCP PGDip, CertEndocrinology SA - ID 283


A Little Help From My
Friends: The Endocrinology–
Dermatology Interface—How
Do We Help Each Other?
Joy Y. Wu, MD, PhD. Department of Medicine, Division of Endocrinology, Stanford
University School of Medicine, Stanford, CA; E-mail: jywu1@stanford.edu

Matthew A. Lewis, MD, MPH. Department of Dermatology, Stanford University School of


Medicine, Redwood City, CA; E-mail: mlewis5@stanford.edu

Learning Objectives • Patients treated with glucocorticoids should be


counseled on adequate calcium and vitamin D
As a result of participating in this session, learners
intake and exercise.
should be able to:
• For those at high risk of fracture,
• Learn the mechanisms involved in the pharmacologic options include
development of glucocorticoid-induced bisphosphonates, denosumab, and teriparatide.
osteoporosis.
• Identify patients with inflammatory skin
conditions at high risk for bone loss as a result
of glucocorticoid therapy. Significance of the
• Explain the therapeutic options for prevention Clinical Problem
of glucocorticoid-induced osteoporosis. In treating inflammatory skin diseases, particularly
pemphigus, bullous pemphigoid, dermatomyositis,
and vasculitis, high-dosage prednisone (1 mg/kg)
is often used in the first 3 to 6 months of therapy
Main Conclusions with a slowly tapering dosage while steroid-sparing
agents are taking effect. Due to time constraints
• Glucocorticoid treatment is an integral and knowledge gaps, dermatologists may not begin
component of the management of appropriate therapy to prevent bone loss at the
inflammatory skin conditions, but it can be initiation of glucocorticoid therapy.
associated with rapid bone loss and increased Glucocorticoid treatment results in bone
fracture risk. loss due to suppression of bone formation by
• Patients treated with high-dosage or long- osteoblasts and enhanced bone resorption by
term glucocorticoids should be screened for osteoclasts.1 This bone loss occurs rapidly, often
osteopenia/osteoporosis by DXA scan. within the first 6 months of treatment, and the
risk of fractures correlates with the dosage and
duration of glucocorticoid therapy.

380  ENDO 2021 • Endocrine Case Management

chased by Dr. Khalid H. Seedahmed, Jr., MRCP PGDip, CertEndocrinology SA - ID 283


Barriers to Optimal Practice T-score ≤ –2.5 in men aged ≥50 years
or postmenopausal women, or FRAX
• Although awareness of glucocorticoid-induced (glucocorticoid-adjusted) 10-year risk of
adverse effects is high, dermatologists already major osteoporotic fracture ≥20% or hip
managing the complexities of inflammatory fracture ≥3%
skin diseases lack the time and familiarity ѓ Adults <40 years of age with prior
with DXA interpretation and medications osteoporotic fracture(s)
for glucocorticoid-induced osteoporosis to
additionally manage bone loss. • Moderate fracture risk
• There is a growing treatment gap in ѓ Adults ≥40 years of age with FRAX
osteoporosis, in part driven by concerns (glucocorticoid-adjusted) 10-year risk of
regarding rare adverse effects such as major osteoporotic fracture 10% to 19% or
osteonecrosis of the jaw and atypical femur hip fracture >1% and <3%
fractures, with the result that treatment rates ѓ Adults <40 years of age with hip or spine
are declining and fracture rates are rising.2,3
BMD Z-score < –3 or ≥10% bone loss
• A lack of professional relationships between over 1 year and continuing glucocorticoid
dermatologists/rheumatologists and treatment ≥7.5 mg daily for ≥6 months
endocrinologists may result in referral delays.
• Low fracture risk
ѓ Adults ≥40 years of age with FRAX
Strategies for Diagnosis, (glucocorticoid-adjusted) 10-year risk of
Therapy, and/or Management major osteoporotic fracture <10% or hip
The 2017 American College of Rheumatology fracture ≤1%
Guideline for the Prevention and Treatment ѓ Adults <40 years of age on glucocorticoid
of Glucocorticoid-Induced Osteoporosis treatment only
recommends that clinical fracture risk assessment
should be performed within 6 months of starting All adults taking prednisone at a dosage of
glucocorticoid treatment.4 For adults 40 years and 2.5 mg daily or higher for 3 or more months
older, and for adults younger than 40 years with should optimize calcium and vitamin D intake
history of osteoporotic fracture or significant risk and lifestyle modifications (balanced diet, weight
factors for osteoporosis, fracture risk should be maintenance in the recommended range, smoking
estimated with FRAX (https://www.shef.ac.uk/ cessation, weight-bearing or resistance training
FRAX/tool.jsp) adjusted for glucocorticoid dosage exercise, limited alcohol intake). For those with
(multiply the risk calculated with FRAX by 1.15 moderate fracture risk, there is a conditional
for major osteoporotic fracture and 1.2 for hip recommendation for pharmacologic treatment,
fracture if glucocorticoid treatment exceeds 7.5 mg with oral bisphosphonates preferred; alternatives
daily) with bone mineral density (BMD) testing include intravenous bisphosphonates, denosumab,
if available. Fracture risk assessment should be or teriparatide (Table). For those with high
repeated yearly. fracture risk, there is a strong recommendation
for pharmacologic treatment, with oral
Fracture risk is categorized as follows: bisphosphonates preferred; alternatives include
intravenous bisphosphonates, denosumab, or
• High fracture risk teriparatide.
ѓ Adults ≥40 years of age with prior
osteoporotic fracture(s), hip or spine BMD

ENDO 2021 • Miscellaneous  381

chased by Dr. Khalid H. Seedahmed, Jr., MRCP PGDip, CertEndocrinology SA - ID 283


Table. Medications Approved for the Treatment of Glucocorticoid-Induced Osteoporosis

Medication Pros Cons

Oral bisphosphonates: • Safe and effective • Gastrointestinal reflux disease


• Alendronate • Patient concerns over adverse effects
• Risedronate • Contraindicated with estimated glomerular
filtration rate <35 mL/min per 1.73 m2

Intravenous bisphosphonate: • Convenient (once yearly) • Patient concerns over adverse effects
• Zoledronic acid • Contraindicated with estimated glomerular
filtration rate <35 mL/min per 1.73 m2

Anti-RANKL monoclonal antibody • Marked increase in BMD • Cannot be stopped abruptly


• Denosumab • Can be used in renal insufficiency • Caution with immune-compromised patients

Anabolic • Builds new bone • Daily self-injection


• Teriparatide • No hip fracture risk reduction
• Limited to 2 years due to osteosarcoma in rodents

Clinical Case Vignettes C. Start calcitonin


D. Start teriparatide
Case 1
A 38-year-old woman with severe pemphigus Answer: A) Continue calcium, 1200 mg
vulgaris affecting her oral mucosa and 20% daily, and vitamin D3, 800 IU daily
of her body surface area has been referred for
Since this patient is younger than 40 years, has
consultation regarding management of bone loss.
not had any fractures, and does not have a Z-score
Baseline BMD testing revealed a T-score of –1.6
less than –3, she is at low risk for fracture and
at the femur and –1.4 at the lumbar spine. She
therefore should be counseled on adequate calcium
has been on prednisone for 2 months, starting at
and vitamin D intake (Answer A) and appropriate
a dosage of 70 mg daily (1 mg/kg) and gradually
lifestyle modifications. Should a repeat DXA scan
tapering 10 mg every 2 weeks. Her current dosage
in 1 year demonstrate greater than 10% bone loss
is 40 mg daily. Two months ago, she also was
while she is taking prednisone exceeding 7.5 mg
started on mycophenolate mofetil, 1000 mg twice
daily, it would be appropriate to consider adding
daily, and she will receive rituximab infusions
pharmacologic therapy at that time.
next week. She is taking calcium, 1200 mg daily,
and vitamin D3, 800 IU daily. The dermatologist
hopes to have her off prednisone after 6 months Case 2
if the rituximab will put the pemphigus into An 80-year-old man with bullous pemphigoid
remission. She is menstruating and has no history is referred for management of bone loss. Recent
of fractures. BMD testing reveals a T-score of –2.0 at the
femur and a T-score of –1.8 at the lumbar spine.
Which of the following is the most He has been on prednisone for 6 months, starting
appropriate treatment for bone at 30 mg daily for 1 month, 20 mg for 1 month,
loss to recommend now? 10 mg for 1 month, then 5 mg daily for the past 3
A. Continue calcium, 1200 mg daily, and vitamin months. He has been unable to reduce the dosage
D3, 800 IU daily below 5 mg daily without flaring. He is not taking
B. Start alendronate calcium or vitamin D. The dermatologist’s plan is

382  ENDO 2021 • Endocrine Case Management

chased by Dr. Khalid H. Seedahmed, Jr., MRCP PGDip, CertEndocrinology SA - ID 283


to continue prednisone, 5 mg daily, as a long-term Of the listed medications, raloxifene (Answer C) is
treatment. He has no history of fractures. the only one that is not approved for the treatment
of glucocorticoid-induced osteoporosis.
Which of the following is the most
appropriate treatment for bone
loss to recommend now?
Case 4  New Content 

A. Perform BMD testing again in 2 years A 65-year-old woman with type 2 diabetes
mellitus and stage 4 chronic kidney disease
B. Start calcium, 1200 mg daily, and vitamin D3,
(estimated glomerular filtration rate =
800 IU daily, and estimate fracture risk by
25 mL/min per 1.73 m2) presents with drug-
FRAX adjusted for glucocorticoid dosage
induced bullous pemphigoid due to initiation of
C. Start teriparatide sitagliptin. The bullous pemphigoid improves
D. Start denosumab after discontinuation of sitagliptin and tapering
of prednisone. BMD testing reveals she has
Answer: B) Start calcium, 1200 mg daily, and
osteoporosis; PTH and bone-specific alkaline
vitamin D3, 800 IU daily, and estimate fracture
phosphatase concentrations are elevated.
risk by FRAX adjusted for glucocorticoid dosage
Although this patient’s DXA score is within the Which of the following medications
osteopenia range, given his advanced age, his risk would be most appropriate to start?
of fracture should be estimated by FRAX (Answer A. Zoledronic acid
B). There is no need to increase the FRAX B. Alendronate
estimated fracture risk since his glucocorticoid C. Risedronate
dosage is less than 7.5 mg daily. Based on his
D. Calcium and vitamin D
estimated fracture risk, pharmacologic therapy
may be indicated as discussed above. E. Denosumab
Answer: E) Denosumab
Case 3  New Content 
This patient has osteoporosis and should
A 53-year-old woman with ocular cicatricial receive pharmacologic treatment. In contrast
pemphigoid has been taking prednisone, 10 mg to bisphosphonates (Answers A, B, and C),
daily, for the past 3 years. She has a history of a denosumab (Answer E) is not cleared by
wrist fracture sustained when she slipped on an icy the kidneys and can be used in patients with
sidewalk. osteoporosis and chronic kidney disease with
elevated bone turnover. This patient should
Which of the following is NOT have adequate calcium and vitamin D intake
approved for the treatment of (Answer D), although this treatment alone would
glucocorticoid-induced osteoporosis? be insufficient. She should be monitored for
A. Teriparatide hypocalcemia.
B. Alendronate
C. Raloxifene
D. Denosumab
E. Zoledronic acid
Answer: C) Raloxifene

ENDO 2021 • Miscellaneous  383

chased by Dr. Khalid H. Seedahmed, Jr., MRCP PGDip, CertEndocrinology SA - ID 283


References
1. Hardy RS, Zhou H, Seibel MJ, Cooper MS. Glucocorticoids and bone: 3. Khosla S, Cauley JA, Compston J, et al. Addressing the crisis in the treatment
consequences of endogenous and exogenous excess and replacement therapy. of osteoporosis: a path forward. J Bone Miner Res. 2017;32(3):424-430.
Endocr Rev. 2018;39(5):519-548. PMID: 29905835 PMID: 28099754
2. Compston J. Reducing the treatment gap in osteoporosis. Lancet Diabetes 4. Buckley L, Guyatt G, Fink HA, et al. 2017 American College of Rheumatology
Endocrinol. 2020;8(1):7-9. PMID: 31757770 guideline for the prevention and treatment of glucocorticoid-induced
osteoporosis. Arthritis Rheumatol. 2017;69(8):1521-1537. PMID: 28585373

384  ENDO 2021 • Endocrine Case Management

chased by Dr. Khalid H. Seedahmed, Jr., MRCP PGDip, CertEndocrinology SA - ID 283


2021 Endocrine Case Management: Meet the Professor is your
source for the latest updates in the diagnosis and management
of a wide range of endocrine disorders. This valuable resource
allows you to evaluate your endocrine knowledge and gain
insight into the strategies used by clinical experts.

Features of 2021 Endocrine Case Management:


Meet the Professor include:

• Significance of the Clinical Problem


• Key Learning Objectives
• Strategies for Diagnosis and Management
• Clinical Pearls and Pertinent References
• Cases and Questions

ENDOCRINE._.,
SOCIETY_,1
Hormone Science to Health

Endocrine Society
2055 L Street NW, Suite 600
Washington, DC 20036
endocrine.org

chased by Dr. Khalid H. Seedahmed, Jr., MRCP PGDip, CertEndocrinology SA - ID 283

You might also like